You are on page 1of 574

Telegram @unacademyplusdiscounts

Join Us on Telegram for More Such Books

https://telegram.me/unacademyplusdiscounts

Join us from the above link or search ''unacademyplusdiscounts'' in Telegram


EBD_7801
• Corporate Office : 45, 2nd Floor, Maharishi Dayanand Marg, Corner Market,
Malviya Nagar, New Delhi-110017
Tel. : 011-49842349 / 49842350

Typeset by Disha DTP Team

DISHA PUBLICATION
ALL RIGHTS RESERVED

© Copyright Publisher

No part of this publication may be reproduced in any form without prior permission of the publisher. The author and the
publisher do not take any legal responsibility for any errors or misrepresentations that might have crept in. We have
tried and made our best efforts to provide accurate up-to-date information in this book.

For further information about the books from DISHA,

Log on to www.dishapublication.com or email to info@dishapublication.com


Contents

TREND ANALYSIS

SOLVED PAPERS.
JEE Advanced – 2019 (Paper I & II) Solved Paper 2019- 1 - 48

JEE Advanced – 2018 (Paper I & II) Solved Paper 2018- 1 - 40

JEE Advanced – 2017 (Paper I & II) Solved Paper 2017- 1 - 36

JEE Advanced – 2016 (Paper I & II) Solved Paper 2016- 1 - 36

JEE Advanced – 2015 (Paper I & II) Solved Paper 2015- 1 - 40

JEE Advanced – 2014 (Paper I & II) Solved Paper 2014- 1 - 40

JEE Advanced – 2013 (Paper I & II) Solved Paper 2013- 1 - 36

MOCK PAPERS
Mock Test - 1 (Paper I & II) MT-1- 1 - 22

Mock Test - 2 (Paper I & II) MT-2 -1 -22

Mock Test - 3 (Paper I & II) MT-3-1 - 22

Mock Test - 4 (Paper I & II) MT-4-1 - 32

Mock Test - 5 (Paper I & II) MT-5-1 - 32

Mock Tests Solutions 1 - 100

130 + Passage Cum Matching Based Questions A-1 - 40


EBD_7801
TREND ANALYSIS JEE ADVANCED
Chapters as per Physics Chemistry Mathematics
NCERT No. of Questions
2019

2018

2017

2016

2015

2014

2013

2019

2018

2017

2016

2015

2014

2013

2019

2018

2017

2016

2015

2014

2013
Ch. No.

1 – – – – – – – – – – – – 2 – – – – – – – –
2 2 4 1 3 4 2 3 3 – 3 1 1 1 1 – 1 – – – – 1
3 – – – – – 1 – – – – 1 – – – 1 2 1 3 1 2 1
4 1 1 1 – – 1 1 1 – 1 1 1 3 1 – – – – – – –
5 1 1 – – – 1 – 1 – – 2 1 2 – 2 1 3 1 2 1 1
6 2 3 1 – 1 3 7 1 4 3 1 1 2 4 – – – – – – –
7 1 4 6 5 3 3 1 1 2 – 2 3 – 3 1 2 1 1 1 3 1
8 1 1 1 – 2 1 1 – – – – – 1 1 1 1 – 1 1 2 1
9 1 1 – – 1 – 1 – – – – – – – 2 1 1 1 1 2 1
10 1 2 1 1 2 4 1 – – – – – – – – – – – – – 1
11 2 1 2 4 1 1 2 – – 1 1 1 1 – 4 4 6 6 5 4 4
12 1 3 3 1 – 1 1 1 – 3 – 1 1 2 – – – – – – –
13 3 – – – 2 2 1 1 – – – 3 1 – 1 – 1 1 1 1 –
14 – – – 1 1 – – – – – – – – – – – – – – – –
15 3 2 2 1 1 2 2 – 1 1 1 1 – 1 – – – – – – –
16 2 3 1 – 3 3 2 2 2 2 2 1 1 – – 2 1 – – – –
17 3 1 – – 1 1 1 – 2 2 1 2 1 1 – 1 1 – 1 1 –
18 – 0 – 3 2 4 – 3 2 1 2 1 1 2 2 2 – – 1 1 2
19 1 3 5 1 3 4 2 1 – 1 – 1 – 1 – – 1 2 1 – 1
20 – – – – – – – 2 1 – 1 2 1 2 5 2 2 3 1 2 1
21 1 – 2 2 – – 2 4 3 6 3 2 3 5 2 4 1 3 2 4 –
22 1 1 4 2 – – 2 4 3 2 3 4 5 3 3 1 6 2 2 1 8
23 – – – – – – – 1 4 1 2 3 2 3 4 4 4 4 8 8 2
24 4 2 2 4 5 3 2 1 – 1 2 – 2 2 1 2 1 1 1 1 3
25 1 0 1 1 1 1 1 2 3 – 2 2 4 2 1 2 1 1 2 1 1
26 2 1 1 2 3 2 2 4 5 6 4 4 3 4 1 2 3 1 4 3 3
27 – 1 1 2 – – 1 1 3 2 1 3 2 – 3 2 1 2 2 1 4
28 2 1 1 3 4 – 4 1 1 – 2 1 1 1 – – – – – – –
29 – – – – – – – 1 – – 1 1 – 1 2 – 1 3 3 2 4
30 – – – – – – – – – – – – – –
Total 36 36 36 36 40 40 40 36 36 36 36 40 40 40 36 36 36 36 40 40 40
Telegram @unacademyplusdiscounts

Join Us on Telegram for More Such Books

https://telegram.me/unacademyplusdiscounts

Join us from the above link or search ''unacademyplusdiscounts'' in Telegram


JEE ADVANCED 2019
PAPER - 1
Section - I (Maximum Marks: 12)
Directions for (Qs. 1 - 4, 19 - 22 and 37 - 40).
• This section contains FOUR (04) questions.
• Each question has FOUR options. ONLY ONE of these four options is the correct answer.
• For each question, choose the option corresponding to the correct answer.
• Answer to each question will be evaluated according to the following marking scheme :
Full Marks : +3 If ONLY the correct option is chosen;
Zero Marks : 0 If none of the options is chosen (i.e. the question is unanswered);
Negative Marks : –1 In all other cases
Section - II (Maximum Marks: 32)
Directions for (Qs. 5 - 12, 23 - 30 and 41 - 48).
• This section contains EIGHT (08) questions.
• Each question has FOUR options. ONE OR MORE THAN ONE of these four option(s) is(are)correct answer(s).
• For each question, choose the option(s) corresponding to (all) the correct answer(s).
• Answer to each question will be evaluated according to the following marking scheme:
Full Marks : + 4 If only (all) the correct options(s) is(are) chosen;
Partial Marks : + 3 If all the four options are correct but Only three options are chosen;
Partial Marks : + 2 If three or more options are correct but ONLY two options are chosen and both of which are correct;
Partial Marks : + 1 If two or more options are correct but ONLY one option are chosen and it is a correct option;
Zero Marks : 0 If none of the options is chosen (i.e. the question is unanswered);
Negative Marks : – 1in all other cases.
For example, in a question if (A), (B) and (D) are the ONLY three options corresponding to correct answers, then choosing ONLY
(A), (B) and (D) will get +4 marks;
choosing ONLY (A) and (B) will get +2 marks;
choosing ONLY (A) and (D) will get +2 marks;
choosing ONLY (B) and (D) will get +2 marks;
choosing ONLY (A) will get +1 marks;
choosing ONLY (B) will get +1 marks;
choosing ONLY (D) will get +1 marks;
choosing no option (i.e. the question is unanswered) will get 0 marks; and choosing any other combination of options will get –1 mark.
Section - III (Maximum Marks: 18)
Directions for (Qs. 13 - 18, 31 - 36 and 49 - 54).
• This section contains SIX (06) questions. The answer to each question is NUMERICAL VALUE.
• For each question, enter the correct numerical value of the answer using the mouse and the on-screen virtual numeric keypad in the
place designated to enter the answer. if the numerical value has more than two decimal places, truncate/round-off the value to Two
decimal places.
• Answer to each question will be evaluated according to the following marking scheme.
Full Marks : + 3 If ONLY the correct numerical value is entered;
Zero Marks : 0 In all other cases.

PHYSICS (b) The ratio of the potential at the centre of the shell to that
1
Section - I of the point at R from the centre towards the hole
2
1. A thin spherical insulating shell of radius R carries a
uniformly distributed charge such that the potential at its 1− α
will be
surface is V0. A hole with a small area α4πR2 (a << 1) is 1− 2α
made on the shell without affecting the rest of the shell.
(c) The magnitude of electric field at the centre of the shell
Which one of the following statements is correct ?
(a) The magnitude of electric field at a point, located on a αV0
is reduced by
line passing through the hole and shell’s center, on a 2R
distance 2R from the centre of the spherical shell will
(d) The potential at the centre of the shell is reduced by
αV0
be reduced by 2αV0
2R
EBD_7801
2019 - 2 JEE Advanced 2019 Solved Paper

2. Consider a spherical gaseous cloud of mass density r(r) in (c) For n = 1.5, Dn = 10–3 and f = 20 cm, the value of |Df|
free space where r is the radical distance from its center. will be 0.02 cm (round off to 2nd decimal place).
The gaseous cloud is made of particles of equal mass m
moving in circular orbits about the common centre with the Df Dn
(d) <
same kinetic energy K. The force acting on the particles is f n
their mutual gravitational force. If r(r) is constant in time,
6. A charged shell of radius R carries a total charge Q. Given
the particle number density n(r) = r(r)/m is
[G is universal gravitational constant] φ as the flux of electric field through a closed cylindrical
3K K surface of height h, radius r and with its center same as
(a) 2 2 (b) 2 2 that of the shell. Here, center of the cylinder is a point on
πr m G 2πr m G
the axis of the cylinder which is equidistant from its top
K K and bottom surfaces. Which of the following option(s) is/
(c) 2 2 (d)
πr m G 6πr 2 m 2G are correct ?
3. A current carrying wire heats a metal rod. The wire provides [ε0 is the permittivity of free space]
a constant power P to the rod. The metal rod is enclosed in (a) If h > 2R and r = 3R/ 5 then φ = Q / 5 ε 0
an insulated container. It is observed that the temperature (T) (b) If h > 2R and r >R then φ = Q/ε0
in the metal rod changes with time (t) as T(t) = T0 (1+βt1/4) (c) If h < 8R / 5 and r = 3R/5 then φ = 0
Where β is a constant with appropriate dimensions while
(d) If h > 2R and r = 4R / 5 then φ = Q/ 5 ε 0
T0 is a constant with dimensions of temperature. The heat
capacity of metal is : 7. In the circuit shown, initially there is no charge on capacitors
4 P(T (t ) − T ) 4 P(T (t ) − T0 ) 2 and keys S1 and S2 are open. The values of the capacitors
(a) 4 2 0 (b) are C1 = 10µF, C2 = 30µF and C3 = C4 = 80µF.
β T0 β4T03
4 P(T (t ) − T0 ) 4 4 P(T (t ) − T0 )3
(c) (d)
β4T05 β 4T04
40
4. In a radioactive sample, 19 K nuclei either decay into stable
40
20 Ca nuclei with decay constant 4.5 × 10–10 per year or into
40
stable 18 Ar nuclei with decay constant 0.5 × 10–10 per year.
40 40
Given that in this sample all the stable 20 Ca and 18 Ar
40
nuclei are produced by the 19 K nuclei only. In time t×109 Which of the statement(s) is/are correct?
years, if the ratio of the sum of stable
40
and 40 (a) If key S1 is kept closed for long time such that capacitors
20 Ca 18 Ar
are fully charged, the voltage difference between points
40
nuclei to the radioactive 19 K nuclei is 99, the value of t P and Q will be 10V.
will be [Given: ln 10 = 2.3] (b) The key S1 is kept closed for long time such that
(a) 9.2 (b)
4.6 capacitors are fully charged. Now key S2 is closed, at
(c) 1.15 (d)
2.3 this time, the instantaneous current across 30 Ω resistor
Section - II (between points P and Q) will be 0.2A (round off to
1stdecimal place)
5. A thin convex lens is made of two (c) At time t = 0, the key S1 is closed, the instantaneous
materials with refractive indices n1 and current in the closed circuit will be 25mA.
n2, as shown in figure. The radius of (d) If key S1 is kept closed for long time such that capacitors
curvature of the left and right spherical are fully charged, the voltage across the capacitors C1
surface are equal. f is the focal length of
will be 4V.
the lens when n1 = n2 = n. The focal length
is f + Df when n1 = n and n2 = n + Dn. 8. Let us consider a system of units in which mass and angular
Assuming Dn << (n – 1) and 1 < n < 2, the momentum are dimensionless. If length has dimension of L,
correct statement(s) is/are. which of the following statement(s) is/are correct?
Dn Df (a) The dimension of force is L–3
(a) If < 0 then > 0
n f (b) The dimension of linear momentum is L–1
Df Dn (c) The dimension of energy is L–2
(b) The relation between and (d) The dimension of power is L–5
f n
remains unchanged if both the convex surfaces are 9. Two identical moving coil galvanometers have 10 Ω
replaced by concave surface of the same radius of resistance and full scale deflection at 2µA current. One of
curvature. them is converted into a voltmeter of 100mV full scale
JEE Advanced 2019 Solved Paper 2019 -3

reading and the other into an Ammeter of 1mA full scale (a) The correction in the height of water column raised
current using appropriate resistors. These are then used to in the tube, due to weight of water contained in the
measure the voltage and current in the Ohm’s law experiment meniscus, will be different for both cases.
with R = 1000W resistor by using an ideal cell. Which of the (b) For case II, if the capillary joint is 5cm above the water
following statement(s) is/are correct? surface, the height of water column raised in the tube
(a) The resistance of the voltmeter will be 100k W will be 3.75cm. (Neglect the weight of the water in the
(b) The measured value of R will be 978 W <R< 982 W meniscus).
(c) If the ideal cell is replaced by a cell having internal (c) For case I, if the joint is kept at 8cm above the water
resistance of 5 W then the measured value of R will be surface, the height of water column in the tube will be
more than 1000 W 7.5cm. [Neglect the weight of the water in the meniscus]
(d) The resistance of the Ammeter will be 0.02 W (round (d) For case I, if the capillary joint is 5 cm above the water
off to 2nd decimal place) surface, the height of water column raised in the tube
10. One mole of a monatomic ideal gas goes through a will be more than 8.75 cm. [Neglect the weight of the
thermodynamic cycle, as shown in the volume versus water in the meniscus]
temperature (V–T) diagram. The correct statement(s) is/are: 12. A conducting wire of parabolic shape, initially y = x2 is

[R is the gas constant] moving with velocity V = V i in a non – uniform magnetic
0

   y β
field
= B B0 1 +    k , as shown in figure. If V0, B0, L and
 L 
 
β are positive constants and Dφ is the potential difference
developed between the ends of the wire, then the correct
statement(s) is/are:
(a) Dφ is proportional to

(a) Work done in this thermodynamic cycle the length of the wire
1 projected on the y-axis.
(1 → 2 → 3 → 4 → 1) is W = RT0
2 (b) Dφ remains the same
(b) The above thermodynamic cycle exhibits only isochoric if the parabolic wire is
and adiabatic processes. replaced by a straight
(c) The ratio of heat transfer during processes 1 → 2 and wire, y = x initially, of
Q1→ 2 5 length 2L
2 → 3 is =
Q2→ 3 3 1
(c) Dφ = B0V0 L for β = 0
(d) The ratio of heat transfer during processes 1 → 2 and 2
Q1→ 2 1 4
3 → 4 is = (d) Dφ = B0V0 L for β = 2
Q3→ 4 2 3
Section - III
11. A cylindrical capillary tube of 0.2 mm radius is made by
joining two capillaries T1 and T2 of different materials 13. A planar structure of length L and width W is made of two
having water contact angles of 0o and 60o, respectively. The different optical media of refractive indices n1 = 1.5 and
capillary tube is dipped vertically in water in two different n2 = 1.44 as shown in figure. If L>>W, a ray entering from
configurations, case I and II as shown in figure. Which of end AB will emerge from end CD only if the total internal
the following option(s) is (are) correct? reflection condition is met inside the structure. For L = 9.6 M,
[surface tension of water = 0.075 N/m, density of water if the incident angle θ is varied, the maximum time taken by
= 1000 kg/m3, take g = 10 m/s2] a ray to exit the plane CD is t × 10–9s, where t is _____
EBD_7801
2019 - 4 JEE Advanced 2019 Solved Paper

14. A block of weight 100N is suspended by copper and steel towards S2 as shown in figure. Both the trains are blowing
wires of same cross sectional area 0.5 cm2 and, length 3 whistles of same frequency 120 Hz. When O is 600m away
from S2 and distance between S1 and S2 is 800m, the
m and 1 m, respectively. Their other ends are fixed on a
number of beats heard by O is _____
ceiling as shown in figure. The angles subtended by copper
[Speed of the sound = 330m/s]
and steel wires with ceiling are 30o and 60o, respectively. If
elongation in copper wire is (Dlc ) and elongation in steel
Dlc
wire is (Dls ) , then the ratio is_____
Dls
[Young’s modulus for copper and steel are 1 × 1011 N/M2
and 2 × 1011 N/M2 , respectively.]

CHEMISTRY

15. A liquid at 30oC is poured very slowly into a Calorimeter that is Section - I
at temperature of 110oC. The boiling temperature of the liquid
19. Molar conductivity (∧ m ) of aqueous solution of sodium
is 80oC. It is found that the first 5 gm of the liquid completely
evaporates. After pouring another 80 gm of the liquid the stearate, which behaves as a strong electrolyte is recorded at
equilibrium temperature is found to be 50 oC. The ratio of latent varying concentration (C) of sodium stearate. Which one of
heat of the liquid to its specific heat will be ___ oC. the following plots provides the correct representation of
[Neglect the heat exchange with surrounding] micelle formation in the solution?
16. A parallel plate capacitor of capacitance C has spacing d (Critical micelle concentration (CMC) is marked with an
between two plates having area A. The region between the arrow in the figures)
plates is filled with N dielectric layers, parallel to its plates,
d
each with thickness δ = . The dielectric constant of the
N (a) (b)
 m
mth layer is =
K m K  1+  . For a very large N(>103),
 N
 K ∈0 A 
the capacitance C is α  . The value of α will be ___
 d ln 2 
[ ∈0 is the permittivity of free space]
17. A particle is moved along a path AB-BC-CD-DE-EF-FA, as (c) (d)

shown in figure, in presence of a force F = (αyi + 2αx j ) N ,
where x and y are in meter and α = –1Nm–1. The work done

on the particle by this force F will be ___ Joule.
20. Calamine, malachite, magnetite and cryolite, respectively, are
(a) ZnCO3, CuCO3.Cu(OH)2, Fe3O4, Na3A1F6
(b) ZnSO4, Cu(OH)2, Fe3O4, Na3AlF6
(c) ZnSO4, CuCO3, Fe2O3, AlF3
(d) ZnCO3, CuCO3, Fe2O3, Na3AlF6
21. The green colour produced in the borax bead test of a
chromium (III) salt is due to
(a) Cr2(B4O7)3 (b) Cr2O3
18. A train S1, moving with a uniform velocity of 108 km/h (c) Cr(BO2)3 (d) CrB
approaches another train S2 standing on a platform. An 22. The correct order of acid strength of the following carboxylic
observer O moves with a uniform velocity of 36km/h acid is
JEE Advanced 2019 Solved Paper 2019 -5

(c)

(d)

(a) I > II > III > IV (b) II > I > IV > III 26. Choose the reaction(s) from the following options, for
(c) I > III > II > IV (d) III > II > I > IV which the standard enthalpy of reaction is equal to the
standard enthalpy of formation
Section - II
1
23. Which of the following statement(s) is (are) true? (a) S8 (s) + O 2 (g) → SO 2 (g)
8
(a) Oxidation of glucose with bromine water gives glutamic (b) 2H 2 (g) + O 2 (g) → 2H 2 O(l)
acid.
(b) Hydrolysis of sucrose gives dextrorotatory glucose and 3
(c) O 2 (g) → O3 (g)
laevorotatory fructose. 2
(c) The two six-membered cyclic hemiacetal forms of (d) 2C(g) + 3H 2 (g) → C2 H 6 (g)
D-(+)- glucose are called anomers. 27. A tin chloride Q undergoes the following reactions (not
(d) Monosaccharides cannot be hydrolysed to give poly- balanced)
hydroxy aldehydes and ketones Q + Cl − → X
24. Fusion of MnO2 with KOH in presence of O2 produces a salt Q + Me3 N → Y
W. Alkaline solution of W upon electrolytic oxidation yields Q + CuCl2 → Z + CuCl
another salt X. The manganese containing ions present in X is a mono anion having pyramidal geometry. Both Y and
W and X, respectively are Y and Z. Correct statement(s) is Z are neutral compounds. Choose the correct option(s).
(are) (a) The oxidation state of the central atom in Z is +2
(a) In both Y and Z, p-bonding occurs between p-orbitals (b) The central atom in Z has one lone pair of electrons
of oxygen and d-orbitals of manganese (c) The central atom in X is sp3 hybridized
(b) In aqueous acidic solution, Y undergoes dispro- (d) There is a coordinate bond in Y
portionation reaction to give Z and MnO2 28. Each of the following options contains a set of four
(c) Both Y and Z are coloured and have tetrahedral shape’ molecules. Identify the option(s) were all four molecules
(d) Y is diamagnetic in nature while Z is paramagnetic posses permanent dipole moment at room temperature.
25. Choose the correct option(s) for the following set of (a) BF3, O3, SF6, XeF6
reactions (b) NO2, NH3, POCl3, CH3Cl
i)MeMgBr
C6 H10 O → Q →
conc.HC l
S (c) SO2, C6H5Cl, H2Se, BrF5
ii)H 2O (major) (d) BeCl2, CO2, BCl3, CHCl3
29. Which of the following statement(s) is(are) correct
20% H3PO4 , 360K regarding the root mean square speed (Urms) of a molecule
in a gas at equilibrium?
i)H .Ni HBr,benzoylperoxide (a) eav at a given temperature does not depend on its
T ←
2
 R → U molecular mass
(major) ii)Br2 ,hν (major) ∆ (major)
(b) Urms is inversely proportional to the square root of its
molecular mass
(c) Urms is doubled when its temperature is increased four
(a) times
(d) eav is doubled when its temperature is increased four
times
30. In the decay sequence,
238 − x1 234 −x 234 −x 234 −x 230
92 U  → 90 
Th 3→ 
2→ Z 
4→
91 Pa 90 Th
(b)
x1, x2, x3 and x4 are particles/radiation emitted by the
respective isotopes. The correct option(s) is (are)
EBD_7801
2019 - 6 JEE Advanced 2019 Solved Paper

(a) Z is an isotope of uranium MATHEMATICS


(b) x1 will deflect towards negatively charged plate
(c) x3 is γ-ray Section - I
(d) x2 is β-ray
Section - III {
37. The area of the region ( x, y : xy ≤ 8,1 ≤ y ≤ x
2
} is
31. For the following reaction, the equilibrium constant Kc at 14 14
298 K is 1.6 × 1017 (a) 8log e 2 − (b) 16log e 2 −
3 3
Fe 2+ (aq) + S2− (aq)  FeS(s) 7
(c) 8log e 2 − (d) 16 log e 2 − 6
When equal volumes of 0.06 M Fe2+ (aq) and 0.2M S2– (aq) 3
solutions are mixed, the equilibrium concentration of Fe2+ 38. A line y = mx + 1 intersectrs the circle (x – 3)2 + (y + 2)2 = 25
(aq) is found to be Y × 10–17 M. The value of Y is at the points P and Q. If the midpoint of the line segment PQ
32. At 143 K, the reaction of XeF4 with O2F2, produces a xenon 3
compound Y. The total number of lone pair(s) of electron has x-coordinate − , then which one of the following
5
present on the whole molecule of Y is ______
33. Consider the kinetic data given in the following table for the options is correct ?
reaction A + B + C → product. (a) 2 < m < 4 (b) –3 < m < –1
(c) 4 < m < 6 (d) 6 < m < 8
Experiment [A] [B] [C] Rate of reaction 39. Let S be the set of all complex numbers z satisfying
No. (mol dm–3) (mol dm–3) (mol dm–3) (mol dm–3)
z − 2 + i ≥ 5 . If the complex number z0 is such that
1 0.2 0.1 0.1 6.0 × 10–5
2 0.2 0.2 0.1 6.0 × 10–5 1  1 
is the maximum of the set  : z ∈ S  , then
3 0.2 0.1 0.2 1.2 × 10–4 z0 − 1  z − 1 
4 0.3 0.1 0.1 9.0 × 10–5
4 − z0 − z0
The rate of the reaction for [A] = 0.15 mol–3, [B] = 0.25 mol the principal argument of is
z0 − z0 + 2i
dm–3 and [C] = 0.15 mol dm–3 is found to be Y × 10–5 mol
dm–3 s–1. The value of Y is _____ π 3π π π
(a) (b) (c) (d) −
34. On dissolving 0.5 g of a non-volatile non-ionic solute to 4 4 2 2
39 g of benzene, its vapour pressure decreases from 650  sin 4 θ −1 − sin 2 θ  −1
mm Hg to 640 mm Hg. The depression of freezing point of 40. Let M =  2  = αI + β M
1 + cos θ cos 4 θ 
Benzene (in K) upon addition of the solute is _____
(Given data : Molar mass and the molal freezing point Where α = α(θ) and β = β(θ) are real numbers, and I is
depression constant of benzene are 78 g mol-1 and 5.12 K the 2 × 2 identity matrix. If α* is the minimum of the set
kg mol-1, respectively) {α(θ) : θ ∈ [0, 2 π)} and β* is the minimum of the set
35. Among B2H6, B3N3H6, N2O, N2O4, H2S2O3, H2S2O8, {β( θ) : θ ∈ [0, 2π)}. Then the value of α* + β* is
the total number of molecules containing covalent bond 31 17 37 29
(a) − (b) − (c) − (d) −
between two atoms of the same kind is _______ 16 16 16 16
36. Schemes 1 and 2 describes the conversion of P and Q and Section - II
R to S, respectively, scheme 3 describes the synthesis of T
from Q and S. The total number of Br atoms in a molecule 41. In a non-right angled triangle DPQR , let p, q, r denote the
of T is ____ lengths of the sides opposite to the angles at P, Q, R
Scheme 1 : respectively. The median from R meets the side PQ at S, the
perpendicular from P meets the side QR at E, RS and PE
intersect at O. If= p = 3, q 1 and the radius of the
circumcircle of the DPQR equals 1, then which of the
following options is/are correct ?
3
(a) Radius of incircle of DPQR = (2 – 3 )
Scheme 2 : 2
3
(b) Area of DSOE =
12
1
(c) Length of OE =
Scheme 3 : 6
7
(d) Length of RS =
2
JEE Advanced 2019 Solved Paper 2019 -7

42. Let G denote a curve y = y(x) which is in the first quadrant x2 y 2


and let the point (1, 0) lie on it. Let the tangent to G at a point E1 : + = 1;
9 4
P intersect the y-axis at Yp. If PYp has length 1 for each point
P on G, then which of the following options is/are correct? R1 : rectangle of largest area, with sides parallel to the axes,
inscribed in E1 ;
 1 + 1 − x2 
(a) y = − log e   + 1 − x2 x2 y2
 x  En : ellipse = +1 of largest area inscribed in Rn – 1, n > 1;
  an2 bn2
2 Rn : rectangle of largest area, with sides parallel to the axes,
(b) xy '− 1− x =0
inscribed in En, n> 1.
 1 + 1 − x2  Then which of the following options is/are correct?
(c) y log e 
=  − 1 − x2 (a) The eccentricities of E18 and E19 are NOT equal
 x  1
 
(b) The length of latus rectum of E9 is
2
6
(d) xy '+ 1− x =0 N
43. Let α and β be the roots of x2 – x – 1 = 0, with a > b. For all (c) ∑ (area of Rn) < 24, for each positive integer N
positive integers n, define n= 1

α n − βn 5
=
an ,n ≥1 (d) The distance of a focus from the centre in E9 is
α −β 32
b1 = 1 and bn =an−1 + an+ 1 , n ≥ 2 47. There are three bags B1, B2, and B3. The bag B1 contains 5
red and 5 green balls, B2 contains 3 red and 5 green balls,
Then which of the following options is/are correct ? and B3 contains 5 red and 3 green balls. Bags B1, B2 and B3

α n 10 3 3 4
(a) ∑ n = have probabilities , and respectively of being
n =1 10 89 10 10 10
(b) bn = αn + βn for all n > 1 chosen. A bag is selected at random and a ball is chosen at
(c) a1 + a2 + a3 + ….. an = an+2 – 1 for all n> 1 random from the bag. Then which of the following options
∞ is/are correct ?
b 8
(d) ∑ nn = (a) Probability that the selected bag is B3 and the chosen
n =1 10 89 3
ball is green equals
 0 1 a  −1 1 −1 10
44. Let M =  1 2 3  and (adj M ) =  8 −6 2  where (b) Probability that the chosen ball is green, given that the
 3 b 1  −5 3 −1 3
   
selected bag is B3, equals
a and b are real numbers. Which of the following options 8
is/are correct ? (c) Probability that the selected bag is B3, given that the
(a) a + b = 3 5
chosen ball is green, equals
(b) det (adj M2) = 81 13
(c) (adjM)–1 + adjM–1 = –M 39
 α   1 (d) Probability that the chosen ball is green equals
80
(d) If M  β  =  2  , then α – β + γ = 3
 γ   3 48. Let f : R → R given by
   
45. Let L1 and L2 denote the lines  x5 + 5 x 4 + 10 x3 + 10 x 2 + 3 x + 1, x < 0;

r = i + λ(−i + 2 j + 2k ), λ ∈ R and r = µ(2i − j + 2k ), µ ∈ R
→ → 2
− + 1,
 x x 0 ≤ x < 1;
 2 8
respectively. If L3 is a line which is perpendicular to both L1 f ( x) =  3 2
x − 4x + 7x − , 1 ≤ x < 3;
 3 3
and L2 and cuts both of them, then which of the following  10 x≥3
options describe(s) L3 ?  ( x − 2) log e ( x − 2) − x + 3 ,
2   
(4i + j + k ) + t (2i + 2 j − k ), t ∈ R

(a) =
r Then which of the following options is/are correct ?
9 (a) f ' has a local maximum at x = 1
2   (b) f is increasing on (–∞, 0)
(2i − j + 2k ) + t(2i + 2 j − k ), t ∈ R

(b) =
r
(c) f ′ is NOT differentiable at x = 1
9
(d) f is onto
(c) r = t (2i + 2 j − k ), t ∈ R


Section - III
1   49. Let S be the sample space of all 3 × 3 matrices with entries
(2i + k ) + t (2i + 2 j − k ), t ∈ R

(d) r=
3 from the set {0,1}. Let the events E1 and E2 be given by
46. Define the collections {E1, E2, E3, ……} of ellipses and E1 = { A ∈ S : det A = 0}and
{R1, R2, R3, …..} of rectangles as follows : E2 = {A ∈ S: sum of entries of A is 7}.
EBD_7801
2019 - 8 JEE Advanced 2019 Solved Paper

If a matrix is chosen at random from S, then the conditional AP (1;3) ∩ AP (2;5) ∩ AP (3;7) = AP (a;d) then a + d equals
probability P(E1/E2) equals _____. ____.
50. Let ω ≠ 1 be a cube root of unity. Then the minimum of the π /4

{ }
2 dx
set a + bω + cω 2 2
: a, b, c distinct non-zero integers 53. If I = ∫
π −π/4 (1 + e )(2 − cos 2 x)
sin x , then 27 I2 equals ____.

equals _____. 54. Three lines are given by


51. Let the point B be the reflection of the point A(2,3) with 
r = λ i, λ ∈ R
respect to the line 8x – 6y – 23 = 0. Let GA and GB be circles 
of radii 2 and 1 with centers A and B respectively. Let T be
a common tangent to the circles GA and GB such that both
( )
r = µ i + j , µ ∈ R and

the circles are on the same side of T. If C is the point of
intersection of T and the line passing through A and B, then
( )
r= v i + j + k , v ∈ R .
the length of the line segment AC is _____. Let the lines cut the plane x + y + z = 1 at the points A, B
52. Let AP(a;d) denote the set of all the terms of an infinite and C respectively. If the area of the triangle ABC is ∆ then
arithmetic progression with first term a and common
the value of ( 6∆ ) equals ____.
2
difference d > 0. If

PAPER - 2
Section - I (Maximum Marks: 32)
Directions for (Qs. 1 - 8, 19 - 26 and 37 - 44).
• This section contains EIGHT (08) questions.
• Each question has FOUR options. ONE OR MORE THAN ONE of these four option(s) is(are)correct answer(s).
• For each question, choose the option(s) corresponding to (all) the correct answer(s).
• Answer to each question will be evaluated according to the following marking scheme:
Full Marks : + 4 If only (all) the correct options(s) is(are) chosen;
Partial Marks : + 3 If all the four options are correct but Only three options are chosen;
Partial Marks : + 2 If three or more options are correct but ONLY two options are chosen and both of which are correct;
Partial Marks : + 1 If two or more options are correct but ONLY one option are chosen and it is a correct option;
Zero Marks : 0 If none of the options is chosen (i.e. the question is unanswered);
Negative Marks : – 1in all other cases.
For example, in a question if (A), (B) and (D) are the ONLY three options corresponding to correct answers, then choosing ONLY
(A), (B) and (D) will get +4 marks;
choosing ONLY (A) and (B) will get +2 marks;
choosing ONLY (A) and (D) will get +2 marks;
choosing ONLY (B) and (D) will get +2 marks;
choosing ONLY (A) will get +1 marks;
choosing ONLY (B) will get +1 marks;
choosing ONLY (D) will get +1 marks;
choosing no option (i.e. the question is unanswered) will get 0 marks; and
choosing any other combination of options will get –1 mark.
Section - II (Maximum Marks: 18)
Directions for (Qs. 9 - 14, 27 - 32 and 45 - 50).
• This section contains SIX (06) questions. The answer to each question is NUMERICAL VALUE.
• For each question, enter the correct numerical value of the answer using the mouse and the on-screen virtual numeric keypad in the place
designated to enter the answer. if the numerical value has more than two decimal places, truncate/round-off the value to Two decimal places.
• Answer to each question will be evaluated according to the following marking scheme.
Full Marks : + 3 If ONLY the correct numerical value is entered;
Zero Marks : 0 In all other cases.
Section - III (Maximum Marks: 12)
Directions for (Qs. 15 - 18, 33 - 36 and 51 - 54).
• This section contains TWO (02) List Match sets.
• Each List-Match set has TWO (02) Multiple Choice Questions.
• For List-Match set has two list: List-I and List-II.
• List-I has four entires (i), (ii), (iii) and (iv) and List-II has Six entires (P), (Q), (R), (S), (T) and (U).
• FOUR options are given in each Multiple Choice Question based on List-I and List-II and ONLY ONE of these four options satisfies
the condition asked in the Multiple choice Question.
• Answer to each question will be evaluated according to the following marking scheme :
Full Marks : +3 if ONLY the correct option is chosen;
Zero Marks : 0 If none of the options is chosen (i.e. the question is unanswered);
Negative Marks : –1 In all other cases
JEE Advanced 2019 Solved Paper 2019 -9

PHYSICS which of the option(s) is/are correct?


[Use hc = 1242 eV nm; 1 nm = 10–9 m, h and c are Planck’s
Section - I constant and speed of light, respectively]
1
1. A thin and uniform rod of mass M and length L is held (a) Dpa / Dpe =
vertical on a floor with large friction. The rod is released 2
from rest so that it falls by rotating about its contact-point (b) The ratio of kinetic energy of the electron in the state
with the floor without slipping. Which of the following 1
n = m to the state n = 1 is
statement(s) is/are correct, when the rod makes an angle 60° 4
with vertical ? (c) m = 2
[g is the acceleration due to gravity] (d) le = 418 nm
3g 4. A block of mass 2M is attached to a massless spring with
(a) The angular speed of the rod will be
2L spring-constant k. This block is connected to two other blocks
(b) The radical acceleration of the rod’s center of mass will of masses M and 2M using two massless pulleys and strings.
3g The acceleration of the blocks are a1­, a2 and a3 as shown in
be the figure. The system is released from rest with the spring in
4
its unstretched state. The maximum extension of the spring is
(c) The normal reaction force from the floor on the rod will x0. Which of the following option(s) is/are correct? [g is the
Mg acceleration due to gravity. Neglect friction]
be
16
2g
(d) The angular acceleration of the rod will be
L

2. An electric dipole with dipole moment


2
(
p0  
i+ j ) is held

fixed at the origin O in the presence of an uniform electric


field of magnitude E0. If the potential is constant on a circle
of radius R centered at the origin as shown in figure, then
the correct statement(s) is/are : x0
(ε0 is permittivity of free space. R >>dipole size) (a) At an extension of of the spring, the magnitude of
4
3g
acceleration of the block connected to the spring is
10
4Mg
(b) x0 =
k
x0
(c) When spring achieves an extension of for the first time,
2
M
the speed of the block connected to the spring is 3g
(a) The magnitude of total electric field on any two points 5k
of the circle will be same (d) a2 – a1 = a1 – a3

(b) Total electric field at point B is E B = 0 5. In a Young’s double slit experiment, the slit separation d is

(c) Total electric field at point A=is E A (
2 E0 i + j) 0.3 mm and the screen distance D is 1 m. A parallel beam of
light of wavelength 600 nm is incident on the slits at angle α
1/3
 p0  as shown in figure. On the screen, the point O is equidistant
(d) R =   from the slits and distance PO is 11.0 mm. Which of the
 4π ∈0 E0 
following statement(s) is/are correct ?
3. A free hydrogen atom after absorbing a photon of
wavelength λa gets excited from the state n= 1 to the state
n = 4. Immediately after that the electron jumps to n = m
state by emitting a photon of wavelength λe. Let the change
in momentum of atom due to the absorption and the
1
emission are pa and D pe, respectively. If λa / λe = ,
5
EBD_7801
2019 - 10 JEE Advanced 2019 Solved Paper

(a) For α = 0, there will be constructive interference at point P.


(b) Fringe spacing depends on α
0.36
(c) For α= degree, there will be destructive (a) The particle’s kinetic energy increases by a factor of 4
π
interference at point P. when the piston is moved inward from L0 to 1 L
0
2
0.36
(d) For α = degree, there will be destructive (b) If the piston moves inward by dL, the particle speed
π dL
interference at point O. increases by 2v
L
6. A mixture of ideal gas containing 5 moles of monatomic gas and (c) The rate at which the particle strikes the piston is v/L
1 mole of rigid diatomic gas is initially at pressure P0, volume (d) After each collision with the piston, the particle speed
increases by 2 V.
V0, and temperature T0. If the gas mixture is adiabatically
compressed to a volume V0/4, then the correct statement(s) is/ Section - II
are, (Given 21.2 = 2.3; 23.2 = 9.2; R is gas constant) 226
(a) The work W done during the process is 13RT0 9. Suppose a Ra nucleus at rest and in ground state
88
(b) The final pressure of the gas mixture after compression 222
undergoes α-decay to a Rn nucleus in its excited state.
is in between 9P0 and 10P0 86
(c) The average kinetic energy of the gas mixture after The kinetic energy of the emitted α particle is found to be
compression is in between 18RT0 and 19RT0 222
4.44 MeV. Rn nucleus then goes to its ground state by
(d) Adiabatic constant of the gas mixture is 1.6 86
7. Three glass cylinders of equal height H = 30 cm and same γ -decay. The energy of the emitted γ photon is ____ keV.
refractive index n = 1.5 are placed on a horizontal surface 226
[Given : atomic mass of Ra = 226.005 u, atomic mass
as shown in figure. Cylinder I has a flat top, cylinder II has 88
a convex top and cylinder III has a concave top. The radii 222
of curvature of the two curved tops are same (R = 3 m), of Rn = 222.000 u, atomic mass of α particle = 4.000 u,
86
If H1, H2, and H3 are the apparent depths of a point X on
1 u = 931 MeV/c2, c is speed of the light]
the bottom of the three cylinders, respectively, the correct
10. A 10 cm long perfectly conducting wire PQ is moving with
statement(s) is/are :
a velocity 1 cm/s on a pair of horizontal rails of zero
resistance. One side of the rails is connected to an inductor
L = 1 mH and a resistance R = 1 Ω as shown in figure. The
horizontal rails, L and R lie in the same plane with a uniform
magnetic field B = 1 T perpendicular to the plane. If the key
S is closed at certain instant, the current in the circuit after
1 millisecond is x × 10–3A, where the value of x is _____
[Assume the velocity of wire PQ remains constant (1 cm/s)
after key S is closed. Given : e-1 = 0.37, where e is base of
(a) 0.8 cm < (H2 – H1) < 0.9 cm the natural logarithm]
(b) H2> H1
(c) H3> H1
(d) H2> H3
8. A small particle of mass m moving inside a heavy, hollow
and straight tube along the tube axis undergoes elastic
collision at two ends. The tube has no friction and it is
closed at one end by a flat surface while the other end is
fitted with a heavy movable flat piston as shown in figure. 11. A monochromatic light is incident from air on a
When the distance of the piston from closed end is L= L0 the refracting surface of a prism of angle 750 and refractive
particle speed is v = v0 . The piston is moved inward at a index n0 = 3 . The other refracting surface of the prism is
dL
very low speed V such that V << v0 , where dL is the coated by a thin film of material of refractive index n as
L shown in figure. The light suffers total internal reflection at
infinitesimal displacement of the piston. Which of the the coated prism surface for an incidence angle of θ < 600.
following statement(s) is/are correct ? The value of n2 is
JEE Advanced 2019 Solved Paper 2019 - 11

strings by varying the free length in between the range L0


and 2L0. It is found that in string-1 (µ) at free length L0 and
tension T0 the fundamental mode frequency is f0.
List-I gives the above four strings while list-II lists the
magnitude of some quantity.
List-I List-II
12. A ball is thrown from ground at angle θ with horizontal and (I) String-1 (µ) (P) 1
with an initial speed u0. For the resulting projectile motion, (II) String-2 (2 µ) (Q) 1/2
the magnitude of average velocity of the ball up to the point
when it hits the ground for the first time is V1. After hitting (III) String-3 (3 µ) (R) 1/ 2
the ground, the ball rebounds at the same angle θ but with a
(IV) String-4 (4 µ) (S) 1/ 3
reduced speed of u0/a. Its motion continues for a long time
as shown in figure. If the magnitude of average velocity of (T) 3/16
the ball for entire duration of motion is 0.8 V1, the value of
(U) 1/16
α is ____
The length of the strings 1, 2, 3 and 4 are kept fixed at L0,
3L0 5 L0 7 L0
, , and , respectively. Strings 1, 2, 3, and 4 are
2 4 4
vibrated at their 1st, 3rd, 5th, and 14th harmonics, respectively
13. A perfectly reflecting mirror of mass M mounted on a
such that all the strings have same frequency. The correct
spring constitutes a spring-mass system of angular
match for the tension in the four strings in the units of T0
4πM W
frequency W such that = 1024m-2 with h as Planck’s will be
h
(a) I→T, II→Q, III→R, IV→U
constant. N photons of wavelength λ = 8π × 10–6 m strike
(b) I→P, II→Q, III→T, IV→U
the mirror simultaneously at normal incidence such that the
(c) I→P, II→Q, III→R, IV→T
mirror gets displaced by 1µm. If the value of N is x × 1012,
then the value of x is ______ (d) I→P, II→R, III→T, IV→U

[Consider the spring as massless] 16. Answer the following by appropriately matching the lists
based on the information given in the paragraph.
A musical instrument is made using four different metal
strings 1, 2, 3 and 4 with mass per unit length µ, 2µ,3µ and
4µ respectively. The instrument is played by vibrating the
strings by varying the free length in between the range L0
and 2L0. It is found that in string-1 (µ) at free length L0 and
tension T0 the fundamental mode frequency is f0.
List-I gives the above four strings while list-II lists the
14. An optical bench has 1.5 m long scale having four equal
magnitude of some quantity.
divisions in each cm. While measuring the focal length of a
convex lens, the lens is kept at 75 cm mark of the scale and List-I List-II
the object pin is kept at 45 cm mark. The image of the object (I) String-1 (µ) (P) 1
pin on the other side of the lens overlaps with image pin that (II) String-2 (2 µ) (Q) 1/2
is kept at 135 cm mark. In this experiment, the percentage
(III) String-3 (3 µ) (R) 1/ 2
error in the measurement of the focal length of the lens is ___

Section - III (IV) String-4 (4 µ) (S) 1/ 3


(T) 3/16
15. Answer the following by appropriately matching the lists
based on the information given in the paragraph. (U) 1/16
A musical instrument is made using four different metal If the tension in each string is T0, the correct match for the
strings 1, 2, 3 and 4 with mass per unit length µ, 2µ ,3µ and highest fundamental frequency in f0 units will be,
4µ respectively. The instrument is played by vibrating the
EBD_7801
2019 - 12 JEE Advanced 2019 Solved Paper

(a) I → Q,II → P, III → R, IV → T infinitesimal change in a thermodynamic quantity X of the


system. For a mole of monatomic ideal gas
(b) I → Q, II → S, III → R, IV → P
3  T   V 
(c) I → P, II → R, III → S, IV → Q = X R 1n  + R 1n  . Here R is gas constant,
2  TA   VA 
(d) I → P, II → Q, III → T, IV → S V is volume of gas. TA and VA­ are constants.
17. In a thermodynamic process on an ideal monatomic gas, the The List-I below gives some quantities involved in a process
infinitesimal heat absorbed by the gas is given by T∆X, and List-II gives some possible values of these quantities.
where T is temperature of the system and ∆X is the List-I List-II
infinitesimal change in a thermodynamic quantity X of the
(I) Work done by the system (P) 1
system. For a mole of monatomic ideal gas in process1→2→3 RT0ln 2
3
3  T   V 
=x R In   + R In  . Here R is gas constant, V (II) Change in internal energy (Q)
2  TA   VA  in process 1→2→3
1
RT0
3
is volume of gas. TA and VA are constants. (III) Heat absorbed by the (R) RT0
The List-I below gives some quantities involved in a process system in process
and List-II gives some possible values of these quantities. 1→2→3
List-I List-II (IV) Heat absorbed by the (S) 4
system in process RT
(I) Work done by the system (P) 3 0
1 1→2
in process1→2→3 RT0ln 2
3 (T) 1
(II) Change in internal energy (Q) RT (3 + ln2)
1 3 0
in process 1→2→3 RT0
3 (U) 5
(III) Heat absorbed by the (R) RT0 RT0
system in process 6
1→2→3 If the process carried out on one mole of monatomic ideal
(IV) Heat absorbed by the (S) gas is as shown in the figure in the PV-diagram with
4
system in process RT 1
3 0 P0V0 = RT0, the correct match is,
1→2 3
(T) P
1 3P0
RT (3 + ln2) 3
3 0 2
(U) 5 1
RT0 P0 2
6
V
If the process on one mole of monatomic ideal gas is as V0 2V0
1 (a) I→Q, II→R, III→S, IV→U
shown in figure in the TV-diagram with P0V0 = RT0,the
3 (b) I→S, II→R, III→Q, IV→T
correct match is, (c) I→Q, II→R, III→P, IV→U
T (d) I→Q, II→S, III→R, IV→U
T0 3 CHEMISTRY

T0 1 Section - I
3 2
19. With reference to aqua regia, choose the correct option(s)
V
V0 2V0 (a) Reaction of gold with aqua regia produces NO2 in the
(a) I→P, II→T, III→Q, IV→T absence of air
(b) I→S, II→T, III→Q, IV→U (b) Reaction of gold with aqua regia produces an anion
(c) I→P, II→R, III→T, IV→P having Au in +3 oxidation state
(d) I→P, II→R, III→T, IV→S (c) Aqua regia is prepared by mixing conc. HCl and conc.
18. In a thermodynamic process on an ideal monatomic gas, the HNO3 in 3 : 1 (v/v) ratio
infinitesimal heat absorbed by the gas is given by T∆X, (d) The yellow colour of aqua regia is due to the presence
where T is temperature of the system and ∆X is the of NOCl and Cl2
JEE Advanced 2019 Solved Paper 2019 - 13

20. The cyanide process of gold extraction involves leaching 25. Choose the correct option(s) for the following reaction
out gold from its ore with CN– in the presence of Q in water sequence
to form R. Subsequently, R is treated with T to obtain Au
and Z. Choose the correct option(s)
(a) Q is O2 (b) T is Zn
(c) Z is [Zn(CN)4]2– (d) R is [Au(CN)4]–
21. Which of the following reactions produce(s) propane as a
major product ?
(a) Consider Q, R and S as major products.

(b) (a)

(c)

(d)

22. Choose the correct option(s) from the following. (b)


(a) Nylon-6 has amide linkages
(b) Cellulose has only α-D-glucose units that are joined by
glycosidic linkages
(c) Teflon is prepared by heating tetrafluoroethene in
presence of a persulphate catalyst at high pressure
(d) Natural rubber is polyisoprene containing trans alkene
units
23. Choose the correct option(s) that give(s) an aromatic
compound as the major product.
(c)
(a)

(b)
(d)

(c)

(d)

24. The ground state energy of hydrogen atom is –13.6 eV.


Consider an electronic state ψ of He+ whose energy, 26. Consider the following reactions (unbalanced) Zn + hot
azimuthal quantum number and magnetic quantum number conc. H2SO4→ G + R + X
are –3.4 eV, 2 and 0, respectively. Which of the following Zn + conc. NaOH→ T + Q
statement(s) is (are) true from the state ψ? G + H2S + NH4OH → Z (a precipitate) + X + Y
(a) It is a 4d state
Choose the correct option(s)
(b) It has 3 radial nodes
(a) The oxidation state of Zn in T is + 1
(c) It has 2 angular nodes
(d) The nuclear charge experienced by the electron in this (b) Bond order of Q is 1 in its ground state
state is less than 2e, where e is the magnitude of the (c) Z is dirty white in colour
electronic charge (d) R is a V-shaped molecule
EBD_7801
2019 - 14 JEE Advanced 2019 Solved Paper

Section - II and List–II contains options showing how they depend on n

27. Total number of cis N – Mn – Cl bond angles (that is Mn – N List–I List–II


and Mn – Cl bonds in cis positions) present in a molecule of (I) Radius of the nth orbit (P) ∝ n−2
cis –[Mn(en)2Cl2] complex is ___ (en = NH2CH2CH2NH2)
(II) Angular momentum of the (Q) ∝ n−1
28. The amount of water produced (in g) in the oxidation of electron in the nth orbit
1 mole of rhombic sulphur by conc.HNO3 to a compound
(III) Kinetic energy of the electron in (R) ∝ n0
with the highest oxidation state of sulphur is ____ (Given the nth orbit
data : Molar mass of water = 18g mol–1)
(IV) Potential energy of the electron (S) ∝ n1
29. The decomposition reaction in the nth orbit
D
2N 2 O 2 (g) → 2N 2 O 4 (g) + O 2 (g) (T) ∝ n2
is started in a closed cylinder under isothermal isochoric (U) ∝ n1/2
condition at an initial pressure of 1 atm. After Y×103s the
Which of the following options has the correct combination
pressure inside the cylinder is found to be 1.45 atm. If the
considering List–I and List–II?
rate constant of the reaction is 5 × 10–4 s–1, assuming ideal
(a) (II), (R) (b) (II), (Q)
gas behaviour, the value of Y is __
(c) (I), (P) (d) (I), (T)
30. Total number of hydroxyl groups present in a molecule of
34. Consider the Bohr’s model of a one-electron atom where
major product P is ___
the electron moves around the nucleus. In the following
List–I contains some quantities for the nth orbit of the atom
and List–II contains options showing how they depend on n.

List–I List–II
(I) Radius of the nth orbit (P) ∝ n–2
(II) Angular momentum of (Q) ∝ n–1
31. Total number of isomers, considering both structural and the electron in the
nth orbit
stereoisomers of cyclic ethers with the molecular formula
C4H8O is____ (III) Kinetic energy of the (R) ∝ n0
electron in the nth orbit
32. The mole fraction of urea in an aqueous urea solution
containing 900 g of water is 0.05. If the density of the (IV) Potential energy of the (S) ∝ n1
electron in the nth orbit
solution is 1.2 g cm–3, the molarity of urea solution is ___
(Given data: Molar masses of urea and water are 60 g mol–1 (T) ∝ n2
and 18 g mol–1, respectively) (U) ∝ n1/2

Section - III Which of the following options has the correct combination
considering List–I and List–II?
33. Consider the Bohr’s model of a one – electron atom where
(a) (III), (S) (b) (IV), (Q)
the electron moves around the nucleus. In the following
(c) (III), (P) (d) (IV), (U)
List–I contains some quantities for the nth orbit of the atom

35. Answer the following by appropriately matching the lists based on the information given in the paragraph. List–I includes
starting materials and reagents of selected chemical reactions. List–II gives structures of compounds that may be formed as
intermediate products and/ or final products from the reactions of List–I.
List–I List–II
(I) (P)
JEE Advanced 2019 Solved Paper 2019 - 15

(II) (Q)

(III) (R)

(IV) (S)

(T)

(U)

Which of the following options has correct combination considering List–I and List–II?
(a) (I), (S), (Q), (R) (b) (II), (P), (S), (U) (c) (I), (Q), (T), (U) (d) (II), (P), (S), (T)
36. List – I includes starting materials and reagents of selected chemical reactions. List – II gives structures of compounds that may
be formed as intermediate products and/ or final products from the reactions of List – I
List–I List–II
(I) (P)

(II) (Q)

(III) (R)

(IV) (S)

(T)

(U)

Which of the following options has correct combination considering List – I and List –II?
(a) (IV), (Q), (R) (b) (IV), (Q), (U) (c) (III), (S), (R) (d) (III), (T), (U)
EBD_7801
2019 - 16 JEE Advanced 2019 Solved Paper

MATHEMATICS (a) xn+1 – xn> 2 for every n


 1
Section -I (b) xn ∈  2n, 2n +  for every n
 2

37. Let f : R → R be given by f (x) = (x – 1) (x – 2) (x – 5). (c) xn − yn > 1 for every n


x (d) x1 < y1
Define
= F ( x) ∫ f (t )dt , x > 0. 41. Three lines

0 L1 : r = λ i, λ ∈ R
Then which of the following options is/are correct? 
(a) F has a local maximum at x = 2 L2 : r= k + µ j , µ ∈ R and

(b) F has a local minimum at x = 1 L : r = i + j + vk , v ∈ R
3
(c) F has two local maxima and one local minimum in (0, ∞) are given. For which point(s) Q on L2 can we find a point P
(d) F (x) ≠ 0 for all x ∈ (0,5) on L1 and a point R on L3 so that P, Q and R are collinear?
38. Let 1 1
(a) k − j (b) k (c) k + j (d) k + j
1 0 0  1 0 0  0 1 0  2 2
P1= I= 0 1 0  , P2= 0 0 1  , P3= 1 0 0  ,
42. For, a ∈ R, a > 1 , let
0 0 1  0 1 1  0 0 1 
     
 
0 1 0  0 0 1  0 0 1   

= P4 =0 0 1  , P5 =1 0 0  , P6 0 1 0   1 + 3 2 + ..... + 3 n 
lim  = 54
1 0 0 
 
0 1 0 
 
1 0 0 
  x →∞  1 1 1 
 n7/3  + + .... +  
 ( an + 1) 2
( an + 2) 2
( an + n) 2
6  2 1 3   
and X = ∑ k  1 0 2  PkT
P Then the possible value(s) of a is/are
k =1  3 2 1 (a) – 9 (b) 7 (c) – 6 (d) 8
T
Where Pk denotes the transpose of the matrix Pk. Then 43. Let x ∈ R and let
 1 1 1  2 x x
which of the following options is/are correct? = P =0 2 2  , Q  0 4 0  and R = PQP-1
(a) X is a symmetric matrix  0 0 3  x x 6
(b) The sum of diagonal entries of X is 18    
(c) X – 30I is an invertible matrix Then which of the following options is/are correct?
 
1  
1  2 x x
(d) If X  1 = α  1 , then α = 30 (a) det R det  0 4 0  + 8 , for all x ∈ R
=
 1  1  x x 5
     
39. For non-negative integers n, let (b) For x = 1, there exists a unit vector α i + β j + γ k for
n
 k +1  k +2   α   0
∑ sin  n + 2 π  sin  n + 2 π  which R  β  = 0
k =0
f ( n) =  γ   0
n
 k +1     
∑ sin 2  n + 2 π  (c) There exists a real number x such that PQ = QP
k =0
Assuming cos–1x
takes values in [0, p], which of the  1  1
following options is/are correct? if R =
(d) For x = 0, = a  6  a  , then a + b = 5
 b  b
1    
(a) lim f (n) =
n→∞ 2 44. Let f : R → R be a function. We say that f has
3 f (h) − f (0)
(b) f (4) = PROPERTY 1 if lim exists and is finite, and
2 h→ 0 h

−1
( )
(c) If α = tan cos f (6) , then α2 + 2α – 1 = 0 f (h) − f (0)
PROPERTY 2 if lim exists and is finite

−1
(d) sin(7 cos f (5)) = 0
h →0 h2
Then which of the following options is/are correct?
sin πx
=
40. Let f ( x) ,x > 0
2/3
(a) f ( x) = x has PROPERTY 1
x2
Let x1 < x2 < x3 < …….. < xn < ……. be all the points of local (b) f ( x) = sin x has PROPERTY 2
maximum of f and y1 < y2 < y3< ……. < yn < ….. be all the (c) f ( x) = x has PROPERTY 1
points of local minimum of f. Then which of the following
options is/are correct? (d) f ( x) = x x has PROPERTY 2
JEE Advanced 2019 Solved Paper 2019 - 17

Section - II List-I List-II


45. Suppose  π 3π 
(I) X (P) ⊇  , , 4π,7 π 
 n  n 2 2 
 ∑k ∑ n Ck k 2  (II) Y (Q) an arithmetic progression
=
det  k 0=k 0  =0
 n n n  (III) Z (R) not an arithmetic progression
 ∑ Ck k ∑ Ck 3 
n k

=  k 0=k 0  (IV) W  π 7π 13π 


(S) ⊇  , , 
n n
C  6 6 6 
holds for some positive integer n. The ∑ k + k1 equals _____  π 2π 
k= 0 (T) ⊇  , ,π 
   3 3 
i 2 j + k be two vectors.
46. Let a = 2i + j − k and b =+
  π 3π 
Consider a vector c = α a + βb , α, β ∈ R . If the projection (U) ⊇  , 
 6 4 
  
( )
of c on the vector a + b is 3 2 , then the minimum Which of the following is the only CORRECT combination?
    (a) (IV), (P), (R), (S) (b) (III), (P), (Q), (U)
( ( ))
value of c − a × b .c equals_____ (c) (III), (R), (U) (d) (IV), (Q), (T)
52. Let f(x) = sin (π cos x) and g(x) = cos (2π sin x) be two
47. Let X denote the number of elements in a set X. Let functions defined for x > 0. Define the following sets whose
S = {1, 2, 3, 4, 5, 6} be a sample space, where each element elements are written in the increasing order :
is equally likely to occur. If A and B are independent events =
X {x : f= =
( x) 0}, Y {x : f =
' ( x ) 0}

associated with S, then the number of ordered pairs (A, B) =Z {x := =


g ( x) 0}, W {x :=
g ' ( x) 0}
such that 1 ≤ B < A , equals ____.
List – I contains the sets X, Y, Z and W. List – II contains
48. Five persons A, B, C, D and E are seated in a circular some information regarding these sets.
arrangement. If each of them is given a hat of one of the List-I List-II
three colours red, blue and green, then the number of ways (I) X  π 3π 
of distributing the hats such that the persons seated in (P) ⊇  , , 4π,7 π 
2 2 
adjacent seats get different coloured hats is ___.
(II) Y (Q) an arithmetic progression
49. The value of
(III) Z (R) NOT an arithmetic progression
 1 10  7π kπ   7π (k + 1)π   (IV) W
sec  ∑ sec 
−1
+  sec  +   π 7π 13π 
 k= 0
4  12 2 12 2   (S) ⊇  , , 
 6 6 6 
 π 3π 
in the interval  − ,  equals ____.  π 2π 
 4 4 (T) ⊇  , ,π 
 3 3 
50. The value of the integral
 π 3π 
π /2 (U) ⊇  , 
3 cos θ  6 4 
∫ dθ equals _____.
( )
5
0 cos θ + sin θ Which of the following is the only CORRECT combination?
(a) (I), (Q), (U) (b) (I), (P), (R)
Section - III (c) (II), (R), (S) (d) (II), (Q), (T)
51. Let f (x) = sin (π cos x) and g (x) = cos (2π sin x) be two 53. Let the circles C1 : x2 + y2 = 9 and C2 : (x – 3)2 + (y – 4)2 = 16,
functions defined for x> 0. Define the following sets whose intersect at the points X and Y. Suppose that another circle
C3 : (x – h)2 + (y – k)2 = r2 satisfies the following conditions:
elements are written in the increasing order.
(i) Centre of C3 is collinear with the centres of C1 and C2
=
X {x : f= =
( x) 0}, Y {x : f =
' ( x ) 0}
(ii) C1 and C2 both lie inside C3, and
(iii) C3 touches C1 at M and C2 at N
=Z {x := =
g ( x) 0}, W {x :=
g ' ( x) 0}
Let the line through X and Y intersect C3 at Z and W, and let
List – I contains the sets X, Y, Z and W. List – II contains a common tangent of C1 and C3 be a tangent to the parabola
some information regarding these sets. x2 = 8αy.
EBD_7801
2019 - 18 JEE Advanced 2019 Solved Paper

There are some expressions given in the List – I whose (iii) C3 touches C1 at M and C2 at N
values are given in List – II below Let the line through X and Y intersect C3 at Z and W, and let
List-I List-II a common tangent of C1 and C3 be a tangent to the parabola
x2 = 8αy.
(I) 2h + k (P) 6
There are some expressions given in the List – I whose
Length of ZW (Q) 6 values are given in List – II below
(II)
Length of XY List-I List-II
(I) 2h + k (P) 6
Area of triangle MZN 5
(III) (R)
Area of triangle ZMW 4 Length of ZW (Q) 6
(II)
Length of XY
(IV) α 21
(S)
5 Area of triangle MZN 5
(III) (R)
Area of triangle ZMW 4
(T) 2 6
10 (IV) α 21
(U) (S)
3 5
Which of the following is the only CORRECT combination? (T) 2 6
(a) (I), (U) (b) (I), (S)
10
(c) (II), (T) (d) (II), (Q) (U)
3
54. Let the circles C1 : x2 + y2 = 9 and C2 : (x – 3)2 + (y – 4)2 = 16,
intersect at the points X and Y. Suppose that another circle Which of the following is the only INCORRECT
C3 : (x – h)2 + (y – k)2 = r2 satisfies the following conditions: combination?
(i) Centre of C3 is collinear with the centres of C1 and C2 (a) (IV), (S) (b) (I), (P)
(ii) C1 and C2 both lie inside C3, and (c) (III), (R) (d) (IV), (U)
Telegram @unacademyplusdiscounts

Join Us on Telegram for More Such Books

https://telegram.me/unacademyplusdiscounts

Join us from the above link or search ''unacademyplusdiscounts'' in Telegram


JEE Advanced 2019 Solved Paper 2019 - 19

SOLUTIONS
Paper - 1
Physics K
1. (b) Let Q be the total charge on the sphere. Then surface
Q
charge density is . A hole is now cut of area m
4pR 2 dr
a(4pR2). The charge on this hole is r dM
q Q KQ
= \q = aQ Also V0 = . M
a (4pR ) 4pR 2
2
R
Now we visualise this situation as a complete
spherical distribution of positive charge on the
surface with a negative charge of the same surface
charge density on the hole. This negative charge Differentiating the above equation w.r.t 'r' we get
can be treated as a point charge. dM 2 K 2K
= or dM = dr
dr Gm Gm
2K K
\ 4pr 2 dr r = dr \ r=
R/2 Gm 2 pr 2 mG
M r K
P A B \ =
m 2 pr 2 m 2 G
R R dQ d dT d
3. (d) P= = ( mc )T = ( mc ) = ( mc ) éT0 (1 + b t1 4 ) ù
dt dt dt dt ë û
bt -3 4
Potential P = (mc)T0 = where (mc) is the heat capacity
4
KQ K aQ KQ 4 Pt 3 4
VP = - = (1 - a ) \ (mc ) = ..... (i)
R R R T0b
= V0 (1 - a ) = V0 - aV0 T (t ) - T0
Therefore option (d) is incorrect. But t1 4 =
bT0
KQ K aQ KQ V 1- a
VA =
R
-
R2
=
R
(1 - 2a ) \ P =
V A 1 - 2a \ t3 4 =
[T (t ) - T0 ]3 ..... (ii)
Therefore option (b) is correct. b3T03
Electric field From (i) & (ii),
4 P [T (t ) - T0 ] 4 P [T (t ) - T0 ]
3 3
KQ0 (mc ) = =
( EB )initial = T0b b T0 3 3
b 4T04
(2 R) 2
dN
KQ K (aQ) KQ Here - = l1 N + l 2 N
( EB )final = 2
- 2
= - aV0 4. (a)
dt
(2R ) R (2 R )2
2.303 N 2.303 100
Option (a) is incorrect. \ t= log10 0 = log10
l1 + l 2 N 5 ´ 10-10 1
( EP )initial = 0 \ 9
t = 9.2 × 10 year
aV
( EP )final = K (a2Q) = R0 5. (a, b, c)
1 2
= (n - 1) Þ f =
R
R
Option (c) is incorrect. f R 2(n - 1)
2. (b) The required centripetal force of particle of mass 'm'
to revolve in a circular path is provided by 1 (n - 1) (n + Dn - 1) 2( n - 1) + Dn
= + =
gravitational pull of the mass 'M ' present in the f +Df R R R
sphere of radius 'r'. Therefore R
\ f +Df =
mv2 GMm 1 GMm GMm 2(n - 1) + Dn
= 2 Þ mv 2 = ÞK=
r r 2 2r 2r
\
f +Df
=
R
´
[ 2(n - 1) ]
2Kr f 2(n - 1) + Dn R
\ M=
Gm
EBD_7801
2019 - 20 JEE Advanced 2019 Solved Paper

At t = 0, capacitors do not have any charge. Therefore


=
[ 2(n - 1)]
5
[ 2(n - 1) + Dn] I=
70 + 100 + 30
= 0.025 A = 25 mA.
D f 2n - 2 - 2n + 2 - Dn Option (c) is correct
\ = when S1 is closed for a long time the all the capacitors
f [ 2(n - 1) + Dn] are fully charged. As the capacitors are in series these
-Dn -Dn
= = ..... (i) carry equal charge q. Current in the circuit is now
2n - 2 + Dn 2(n - 1) zero. Applying Krichhoff 's law
[Q Dn << (n – 1)] C4 5 V
I= 0 S1 30W
Dn Df
From equation (i) if < 0, then > 0. Therefore –q +q –q
n f +q
option (a) is correct. C1 C3
–q +q
From equation (i) if convex surface are replaced by 70W
concave surface of the same radius of curvature then
Df Dn 100W
the relation between an d remains
f n q q q
5- - - =0 \ q = 40 µc
unchanged. Therefore option (b) is also correct. 80 10 80
For n = 1.5, Dn = 10–3 and f = 20 cm then from (i) Potential difference across C1 is
q 40 ´ 10 -6
Df 10 -3 = = 4V
=- Þ Df = -0.02 cm C1 10 ´ 10-6
20 2(1.5 - 1) (d) is the correct option.
or |D f | = 0.02 cm \ Option (c) is correct. VP – 4 – 70 × 25 × 10–3 = VQ
qin Q \ VP – VQ = 4 + 1.75 = 5.75 V
6. (a, b, c) (b) +Q f= =
Î0 Î0 (a) is an incorrect option.
h 2R Now when key S2 is closed
R
In loop MPQS +10 – 30 i1 – 4 – 70 i = 0
70 i + 30 i1 = 6 ..... (i)
r 5V 30W
M P – + O
R qin + 8 0 µF
(c) 4R f= =0 4V 10 µF
3R 5 Î0 – 30 µ F
5 –
8 0 µF
30 W +
4R 4R 5 4 70W
(d) 5 sin q = = = 0.8
q R 5 10V
R i1
Þ q = 53°
S i Q i. – i1 100W R
é 3ù 2Q
qin = Q[1- cos q] = Q ê1- ú = In loop QROPQ,
ë 5û 5 40 40
+10 – 30 i1 + – 5 + (i – i1) × 130 + =0
2Q 80 80
\ f = 5Î 130 i – 160 i1 = –6 ..... (ii)
0
(a) On solving (i) & (ii), we get i = 0.05 A \ i1 = 0.077 A
(b) is an incorrect option.
3R 5 3
q sin q =
= = 37° 8. (a, b, c) Given [mvr] = M 0L0T 0 and [m] = M 0L0T 0
R R 5
3R ML2T –1 = M 0L0T 0 Þ T = L2
é 4ù Q
5 qin = Q [1 - cos37°] = Q ê1 - ú = Given, momentum
ë 5û 5 mvr M 0 L0T 0
Q p = mv = = L-1
r L
\ f = 5Î
1 1 (mv) 2
0
Energy E = mv 2 = = L-2
7. (c, d) S1 closed and S2 open (t = 0) 2 2 m
E L-2 L-2
S1 5V 30W Power P = = = 2 = L- 4
t T L
E L-2
Force F = = = L-3
70W x L
9. (b, d) G = 10 W ; Ig = 2 × 10–6 A, V = 100 m V = 0.1V, I = 10–3A
110W Here V = Ig (G + R) where R is the resistance
JEE Advanced 2019 Solved Paper 2019 - 21

connected in series with galvanometer Work done during cyclic process = area enclosed in
0.1 = 2 × 10–6 Rv P0
the loop = V0
\ Rv = 5 × 104 W = Resistance of voltmeter 2
PV nRT0
Also Ig G = (I – Ig)s For point, 1, P0V0 = n R T0 \ 0 0 =
2 2
nRT0 RT0
2 × 10–6 ×10 = (10–3 – 2× 10–6)s \ S = 2 × 10–2 W \ loop done =
1
=
2
[ as n = 1]
GS 10 ´ 0.02 (a) is the correct option.
RA = = » 0.02W
G + S 10 + 0.02 11. (a, b, c) For case 1
option (d) is correct 2T cos q1 2 ´ 0.75 ´ cos 0°
h1 = = = 7.5cm
E E rr g 2 ´ 10-4 ´ 1000 ´ 10
I= =
50,000 ´ 1000
+ 0.02 980.41 option (c) is correct
51000 For case 2
E 50,000 ´ 1000 E 2T cos q2 2 ´ 0.75 ´ cos60°
Vab = ´ = ´ 980.39 h2 = = = 3.75cm
980.41 51000 980.41 r rg 2 ´ 10-4 ´ 1000 ´ 10
Vab E 980.39 option (b) is correct
\ R measured = = ´ = 980.39W The correction in the height of water column raised
I 980.41 E / 980.41
Option (b) is Correct V in the tube, due to weight a in the meniscus will be
If the ideal cell is different in both option (a) is correct
replaced by a cell A
In case I, if the capillary joint is 5 cm above the water
having internal a 1000W b I surface then the height of water raised in tube 1 will
resistance of 5W be 5 cm and the shape of liquid meniscus will change
E to adjust extra liquid.
then I = E 12. (a, b, d) Let us consider the projection of wire on f-axis
985.41 We now consider a infinite small length of wire dy at
E 50, 000 ´ 1000 E a distance y from the origin on this projection. The
and Vab = ´ = ´ 980.39
985.41 51000 985.41 induced emf developed across 'dy' is
Vab E ´ 980.39
\ R measured = = ´ 985.41 = 980.39
I 985.41 E
10. (a, c) Let P0 be the pressure at 1
P
dy
(T0)
1 2 y
P0
(2T0)

P0
2 4 (T0)
3 é æ yöBù
df = B ( dy ) V0
(T0)
2 d f = B0 ê1 + ç ÷ ú V0 dy
êë è L ø úû
Therefore the induced emf across the complete
V0 2V0 V projection
Lé b
1 ® 2 Process is isobaric pressure at '2' is P0 æ yö ù é 1 ù
Df = B0 V0 ò ê1 + ç ÷ ú V0dy = B0V0L ê1 + ú
P0 P P è ø
L úû ë β + 1û
2 ® 3 Process is isochoric = 3 \ P3 = 0 P0 0êë
2T0 T0 2 Option (a) is correct
3 ® 4 Process is isobaric
4 ® 1 Process is isochoric For β = 0, Df = 2B0V0L. Option (c) is correct
Option (b) is incorrect For a straight wire of length 2L placed along y = x
The P-V diagram is shown
then the value of Df will remain the same as its
| DQ1® 2 | = | n Cp DT | = | n Cp ( 2T0 - T0 ) | = | n Cp T0 |
projection of y-axis is same as that of previous case.
| DQ 2®3 | = | D U | = | n Cv DT | = | n C v T0 | Therefore option (a) is also correct
DQ1® 2 Cp 5 13. (50) Let 'C' be the critical angle. Then
\ = = (monoatomic gas) P
DQ 2®3 C v 3
option (c) is correct C
1.44 n2
nCp T0 2 sin C = M
T0 DQ1®2 1.50 q Q medium 2
DQ3® 4 = nC p \ = =
2 DQ3®4 T 1
nCp 0
2
9.6 m
\ option (d) is correct.
EBD_7801
2019 - 22 JEE Advanced 2019 Solved Paper

QM d
But sin C = d dx
k Î0 A ò d + x
QP =
QM 1.44 1.50 0
\ = \ QP = QM
QP 1.50 1.44 1 d k Î0 A
If we replace QM by 9.6 then the total path length = ln 2 Þ C = \ a = 1.00
C k Î0 A d ln 2
1.50
travelled by light is ´ 9.6 = 10m 17. (0.75) We know that
1.44 r r
Now time taken to do so is
d
( )(
dW = F × dr = ay $i + 2ax $j × dx $i + dy $j )
t= where v2 is velocity of light in medium 2 \ dW = aydx + 2axdy
v2 Work done from A ® B dy = 0, as y = 1
10
\t = 8
= 5 ´ 108 s = 50 ´ 10-9 s 1 1
´
3 10 / 1.5 \ W1 = ò ay dx = a ò dx = a
é Cù
êQ v2 = ú 0 0
ë n2 û Work done from B ® C dx = 0, as x = 1
t = 50.00 0.5 0.5
14. (2) Due to equilibrium
Ts cos60° = Tc u cos30° Þ Ts = 3 Tcu ..... (1)
\ W2 = ò 2an dy = ò 2ady = 2a ( -0.5) = -a
1 1
Work done from C ® D dy = 0, as y = 0.5
60° 30°
TCu sin 30° 0.5
a
TS TCu \ W3 = ò a ´ 0.5dx = - 4
TS sin 60° 1
60°
Work done from D to E dx = 0, as x = 0.5
T S cos 60° TCu cos 30°
0
a
98 N
\ W4 = ò 2a ´ 0.5dy = - 2
0.5
Dlcu T l T l y Work done from E to F dy = 0, as y = 0
Now = cu cu = cu cu ´ s
Dls A ´ l cu ycu Ts ls
\ W5 = ò a ´ 0 ´ dx = 0
Ts ls
Work done from F to A dx = 0 as x = 0
A ´ ys
Tcu ´ 3 2 ´ 10112 ò
\ W6 = 2a ´ 0dx = 0
= 11
´ = = 2.00 Total work W = W + W + W + W + W + W
10 3Tcu ´ 1 1 1 2 3 4 5 6
15. (270) Let mc is mass of calorimeter and its specific heat. a a 3a -3 ( -1)
=a -a - - = - = = +0.75J
Capacity Further let C be the specific heat capacity 4 2 4 4
of liquid and L to latent heat of voportion 18. (8.13) Frequency heard by observer due to source S is
5 × C × 50 + 5 × L = mcsc (110 – 80) ... (1) 1
é v + v0 ù é 330 + 10 ù 34
For the second case 80 × C × 20 = mcsc (80 – 50) ... (2) v1 = v ê ú = 120 ê 330 ú = 120 ´ 33 = 123.636
ë v û ë û
250C + 5L 30 Frequency heard by observer due to S is
on dividing we get =
1600c 30 2
L é v + v0 ù é 330 + 10 cos53° ù
\ = 270 ° C = 270.00 °C v2 = v ê ú = 120 ê ú
C ë v - v sû ë 330 - 30cos 37° û
dx
æ m ö x é 330 + 10 ´ 0.6 ù é 336 ù
k ç1 + Î0 A÷ \ v 2 = 120 ê = 120 ê
è N ø ú ú = 131.764
16. (1.00) dc = ë 330 - 30 ´ 0.8 û ë 306 û
dx \ Beat frequency = v – v = 8.125 Hz » 8.13 Hz
2 1
1 dx
\ = 30 ms–1
dc æ mö
k ç 1 + ÷ Î0 A 37° S2
è Nø
1 dx 30 cos 37°
= ò dc = ò
C æ mö
k ç1 + ÷ Î0 A
è Nø
dx x D 10m/s
=ò Q = 53°
æ xö m N 10 cos 53°
k ç1 + ÷ Î0 A
è dø
JEE Advanced 2019 Solved Paper 2019 - 23

Chemistry are anomers because they differ in configuration


only around C1 position.
19. (d) Sodium stearate at low concentration (i.e., below (d) Monosaccharides do not undergo further hydrolysis
CMC) behaves as normal strong electrolyte, but due to absence of glycosidic linkages.
at higher concetration (i.e. above CMC) exhibits 24. (a, b, c)
colloidal behaviour due to the formation of micelles.
Thus, plot (d) correctly represents relation between ∆
2MnO 2 + 4KOH + O 2 
→ 2K 2 MnO 4 + 2H 2 O
∧m and C for sodium strearate.
20. (a) Calamine → ZnCO3 (W)
Malachite → CuCO3.Cu (OH)2 2K2MnO4 + 2H2O ® 2KMnO4 + 2KOH (g) + H2
Magnetite → Fe3O4 (W) (X)
Cryolite → Na3AlF6 2−
K2MnO4 ® 2K+ + MnO 4 (mangnate ion)
21. (c)
(Y)
∆ ∆
Na 2 B4 O7 .10H 2 O 
→ Na 2 B4 O7 
→ 2NaBO 2 +B2 O3

Cr2O3 + 3B2O3 æææÆ 2Cr (BO2)3 KMnO4 ® K+ + MnO 4 (permangnate ion)
Chromium meta borate (Z)
(green colour)
22. (a) Acidic strength depends upon the stability
of anion formed. If stability of anion formed
is high, its acidic strength will also be
high. Order of stability of formed anion :
HC º COO– > H2C = CHCOO– > C6H4 (OMe)
COO– >
• sp3, Tetrahedral • sp3, Tetrahedral

H3CCH2COO– • Green colour • Purple colour

• Mn 6+ : [Ar] 4S 0 3d 1 • Mn 7 + : [Ar] 4S 0 3d 0
Therefore,
• Paramagnatic • Diamagnetic

Disproportionation of MnO 24 − undergoes in acidic


medium but not in base, concerned reaction is as

23. (b, c, d) under :
Br2
(a) Glucose  → Gluconic acid. 2− +
→ MnO 2 + 2MnO 4− + 2H 2 O
3MnO 4 + 4H 
H O 2

Bromine water oxidises only aldehyde group to a, b, c are correct


carboxylic group. It neither oxidises –OH group
nor C = O group. 25. (a, b)

H O
(b) Sucrose 
2 → Glucose + Fructose
(+) (–)
Sucrose is dextrorotatory (+ 66.6°) in nature but
on hydrolysis it gives dextrorotatory glucose and
laevorotatory fructose (–90.4°). Overall solution
is laevorotatory since laevorotation is more than
dextrorotation.

(c) and
EBD_7801
2019 - 24 JEE Advanced 2019 Solved Paper

26. (a, c) Enthalpy of formation is the enthalpy change for Since,


formation of 1 mole of substance from its element Kc >> 103 ; 0.03 – x ≈ 0
present in the most stable natural form. \ x = 0.03 and 0.1 – x = 0.07
27. (c, d) 1
Κ c= 2+
= 1.6 × 1017

SnCl2 + Cl → SnCl3− (0.07) × [Fe ]
(X)
1 250
[Fe2+ ] = × 10 –17 = × 10 –17
SnCl2 + Me3 N → SnCl2 [NMe3 ] 0.07 × 1.6 28

(Y)
= 8.93 × 10–17 (i)
There is a coordinate bond between NMe3 and
comparing (i) with given value in question we get
SnCl2 due to sharing of lone pairs of NMe3 with
SnCl2. Y = 8.93.
Structure of X, Y, Z are respectively :
32. (19.00)

XeF4 + O 2 F2 → XeF6 + O 2


SnCl2 + 2CuCl2 → SnCl4 + 2CuCl
(Z)
28. (b, c) Dipole moment (m) value of BF3, SF6, BeCl2, CO2, Shape of XeF6 is distorted octahedral. It contains
BCl3 is Zero. one lone pair of e– s on central atom 3 lone pair of
29. (a, b, c) According to kinetic theory of gases, all gases e– s on each F atom surrounded by Xe.
at a given temperature have same average kinetic Total no. of lone pairs: 1 + 18 = 19.00
energy.
33. (6.75)
3
Eav = RT ; Eav ∝ T (absolute temp) Rate of the reaction = K[A]x [B]y [C]–z
2
Root mean square velocity is directly proportional Comparing experiment 1 with 2 we get that, y = 0
to square root of absolute temperature and inversely
proportional to square root of molecular weight of Comparing experiment 1 with 3 we get that z = 1
the gas. Comparing experiment 1 with 4 we get that, putting
1 values of x, y, z in rate equation for experiment 1.
Vrms ∝ T (absolute temp) : Vrms ∝
M x=1
30. (a, b, d)
6 × 10–5 = K × 0.2 × 0.1
238 234 4
92 U 
→ 90 Th + 2 He; x1= α
K = 3 × 10–3
234 234 0
90 Th 
→ 91 Pa + −1e; x 2= β Now, for the given concentration of A, B and C rate
234 234 0
of reaction will be,
91 Pa 
→ 92 Z + −1e; x 3= β
Rate = 3 × 10–3× 0.15 × 1 × 0.15 = 6.75 × 10–5
234 234
Th + 24 He; Therefore value of y = 6.75.
92 Z 
→ 90 x 4= α

31. (8.93) 34. (1.03) We know that,


  Fe 2+
+ S 2–  FeS(K c =

17
1.6 × 10 )
P0 – P w × m 10 0.5 × 78

= 2
= 1
;
t = 0 0.03M 0.1 M 0
P m 2 × w1 640 m 2 × 39
At equilibrium (0.03 – x) (0.1 – x)
m2 = 64 g
JEE Advanced 2019 Solved Paper 2019 - 25

K × w 2 × 1000
∆Tf =f
m2 × w l

5.12 × 0.5 × 1000


∆T=
f = 1.0256 ≈ 1.03
64 × 39
35. (4.00)

Scheme – II


Total no. of molecules containing covalent bond
between two atoms of the same kind are 4. Scheme – III
36. (4.00)
Scheme – I
EBD_7801
2019 - 26 JEE Advanced 2019 Solved Paper

Mathematics (-3m + 15) m


Þ = -1 Þ m 2 - 5m + 6 = 0
-18
37. (b) xy £ 8, 1 £ y £ x2
Intersection points of xy = 8 and y = 1 is (8, 1); xy = 8 Þ m = 2, 3
and y = x2 is (2, 4) and y = x2 and y = 1 is (1, 1)
Y Alternately :- The x-coordinates of intersection
y = x2 points P and Q of circle and line can be given by
solving their equations for x.
(2, 4) Putting y = mx + 1 in (x –3)2 + (y + 2)2 = 25, we
xy get (x – 3) 2 + (mx + 3) 2 = 25
= 8 (8, 1) Þ (m2 + 1) x 2 + 6(m - 1) x - 16 = 0
(1, 1) y=1
-6( m - 1)
X' O X Þ x1 + x2 = where x1 and x2 are x-
m2 + 1
coordinates of P and Q respectively. As per
-3
question, x-coordinate of mid point of PQ is
Y' 5
2 8 8 x1 + x2 3 -3( m - 1) -3
2 8 \ =- Þ =
Required area = ò x dx + ò dx - ò1dx 2 5 m2 + 1 5
x
1 2 1
2 Þ m2 – 5m + 6 = 0 Þ m = 2, 3
æ x3 ö 39. (d) S : |z – 2 + i| ³ 5 represents boundary and outer
= ç ÷ + (8lnx)82 - (x)18
è 3 ø1 region of circle with centre (2, –1) and radius 5 .
8 1
= - + 8 ln 8 - 8ln 2 - (8 - 1) 1
3 3 z0 Î S, such that is the maximum.
7 14 | z0 - 1 |
= + 24 ln 2 - 8 ln 2 - 7 = 16 ln 2 -
3 3
\ |z0 –1| is minimum
\ correct option is (b)
z0 Î S with |z0 – 1| as minimum will be a point on
38. (a) Circle (x – 3)2 + (y + 2)2 = 25, with centre C(3, –2) and
boundary of circle of region S which lies on radius of
radius 5 is intersected by a line y = mx + 1 at p & Q
this circle, and passes through (1, 0).
such that mid point R of PQ has its
\ z0, 1, 2 – i are collinear, or (x0, y0), (1, 0),
3
x-coordinate as - . (2, –1) are collinear.
5
æ 3 -3m ö \ Using slopes of paralled lines,
Let R ç - , + 1÷
è 5 5 ø y0 -1
= Þ y0 = 1 – x0
x0 - 1 2 - 1
Y

y = mx + 1 P
R Q z0
O (1, 0)
X' X
O (2, –1)

C (3, –2)

4 - z0 - z0 4 - ( z0 + z0 )
Now, =
z0 - z 0 + 2i ( z0 - z0 ) + 2i
Y' 4 - 2 x0 4 - 2 x0 2(2 - x0 ) 1
-3m = = = = -i
+1 + 2 2iy0 + 2i 2i - 2 x0i + 2i
=
5 2(2 - x0 )i i
Then CR ^ PQ Þ ´ m = -1
3
- -3 æ 4 - z0 - z0 ö -p
5 \ Arg ç
z - z - 2i ÷ = Arg (–i ) = 2
è 0 0 ø
JEE Advanced 2019 Solved Paper 2019 - 27

Using sine law in DPQR


é sin 4 q –1 – sin 2 q ù
40. (d) M =ê ú p q r
êë1 + cos2 q = =
cos 4 q úû sin P sin Q sin R
= 2 × radius of circumcircle,

M = sin 4 q cos4 q + 1 + sin 2 q + cos 2 q 3 1 r


We get, = = = 2 ´1 = 2
+ sin 2 q cos 2 q sin P sin Q sin R

= 2 + sin 2 q cos 2 q + sin 4 q cos 4 q 3 1


Þ sin P = and sin Q =
2 2
1 é cos q 1 + sin 2 q ù
4
M –1 = ê ú Þ ÐP = 60° or 120° and ÐQ = 30° or 150°
M ê –1– cos 2 q
ë sin 4 q úû Q D cannot have two obtuse angles
\ ÐP = 120°, ÐQ = 150° is rejected.
Given that M = aI + b M–1 Q sum of three angles can not be > 180°,
Þ é sin 4 q –1 – sin 2 q ù éa 0 ù \ ÐP = 60°, ÐQ =150° is rejected.
ê ú=ê ú Q DPQR is non right triangle, \ ÐP = 60° and
êë1 + cos 2 q cos4 q úû ë 0 a û
ÐQ = 30° is rejected.
b é cos q 1 + sin 2 q ù
4
\ ÐP = 120° and ÐQ = 30° is the only option.
+ ê ú
M ê –1– cos 2 q sin 4 q úû Þ ÐQPE = ÐRPE = 60° and ÐPRQ = 30°
ë
r
b b Þ =2Þr=1
Þ = –1 and a + cos 4 q = sin 4 q sin 30°
M M
Also PE is bisector of ÐQPR
Þ a = sin4q + cos4q
Þ b = –[2 + sin2q cos2q + sin4qcos4q] 3
\ QE : ER = PQ : PR Þ QE = ER = .
Now, a = (sin2q + cos2q)2 – 2sin2q cos2q 2
Area of DPQR
1 2
= 1 – 2 sin2qcos2q = 1 – sin 2q
2 pqr é abc ù
=
4(radius of circumcircle) êëQ D = 4 R úû
For a to be minimum sin22q is maximum i.e.1.
1 1
\ a* = 1– = 3
2 2 =
4
é 1 2 1 4 ù \ In DPQR, radius of incircle
Also, b = – ê 2 + sin 2q + sin 2qú
ë 4 16 û
Ar(DPQR) æ Dö
For b to be minimum, sin22q is maximum i.e. = çQ r = ÷
Semi perimeter è sø
é 1 1ù 32 + 4 + 1 –37
\ b* = – ê 2 + + ú = – = 3
ë 4 16 û 16 16
4 3 3
1 37 –29 = = = (2 - 3)
\ a * + b* = – = 3 + 1 + 1 2(2 + 3) 2
2 16 16
41. (a, c, d) RS is median and PE ^ QR. 2
\ option (a) is correct.
p = 3, q = 1
Q RS and PE are medians of DPQR intersecting at O,
radius of circumcircle (R) = 1
O is centriod of DPQR.
1 3
P \ ar(DOQR) = ar(DPQR) =
3 12
1 3 1
60° 60° q
Þ × 3 × OE = Þ OE =
r 2 12 6
S F \ option (c) is correct.
30° O 30° 1
Now Ar(DSOE) = OE × SF
Q E R 2
p 1 1 r
= × × sin 60°
2 6 2
EBD_7801
2019 - 28 JEE Advanced 2019 Solved Paper

1 1 3 3 43. (a, b, c) Given a, b are roots of x2 – x – 1 = 0 with a > b


= ´ ´ =
12 2 2 48
\ option (b) is incorrect. Also in DRQS, by cosine law 1+ 5 1- 5
\ a= ,b=
2 2
QS2 + QR 2 - RS2
cos 30° = Also, a + b = 1, ab = –1, a–b= 5
2QS ´ QR
2
æ1ö a2 – a – 1 = 0 and b2 – b – 1 = 0
+ ( 3)2 - RS2
3 çè 2 ÷ø
Þ = a n - bn
2 1
2´ ´ 3 an = , n ³ 1; b1 = 1 and bn = an – 1 + an + 1, n ³ 2
2 a -b
3 13 Let us now check the given options, one by one
Þ = – RS2
2 4
¥ ¥
a a n - bn
13 3 7 7
(a) å 10nn = å 5(10n )
- = n =1 n =1
ÞRS2 = or RS =
4 2 4 2
1 é¥ æaö ¥ n nù
æbö
\ Option (d) is correct. = 5 ê å ç ÷ - å ç ÷
êë n =1 è 10 ø n =1 è 10 ø
ú
úû
42. (c, d) Tangent to the curve y = y (x) at point P (x, y) is given
by Y – y = y ' (x) (X –x) é a b ù
It intersects y-axis at Yp, putting x = 0 1 ê 10 10 ú 1 é a b ù
= ê - ú = ê10 - a - 10 - b ú
5 ê1 - a b 5ë û
Y – y = –xy' (x) 1- ú
ë 10 10 û
Þ Y = y – xy' (x) \ Yp (0, y – xy ' (x))
Given PYp = 1 1 é10a - ab - 10b + ab ù
= ê ú
5 ë (10 - a )(10 - b) û
2
Þ (x - 0)2 + ( y – y + xy '(x) ) = 1
1é 10( a - b) ù 1 é 10 5 ù 10
2 2
Þ x + x (y '(x)) = 12 = ê ú= ê ú=
5 ë100 - 10(a + b) + ab û 5 ë100 - 10 - 1 û 89

1 – x2 \ option (a) is correct.


Þ y' (x) = ±
x (b) bn = an + 1 + an – 1
Now y = y (x) less in first quadrant and its tangent
passes through (1, 0), therefore it has to be a de- a n +1 - b n +1 a n -1 - b n -1
= +
creasing function, so derivative should be negative a -b a -b

– 1 – x2 é or xy' (x) + 1– x 2 = 0 ù (a n+1 + a n-1 ) - (b n+1 + b n-1 )


\ y'(x) = êë úû =
x a-b
a n -1 (a 2 + 1) - bn -1 (b2 + 1)
1 – x2 =
Þ y(x) = – ò dx 5
x
put x = sin q Þ dx = cos q dq 1
= [an – 1 (a + 2) – bn – 1 (b + 2)]
cos q 5
y = –ò cos q dq = – ò (cosec q – sin q) dq
sin q (using a2 = a + 1, b2 = b + 1)
y = + log |cosec q + cot q| – cos q + c
1 é n-1 æ 1 + 5 ö æ1- 5 öù
= êa çç + 2 ÷ - bn -1 ç + 2 ÷ú
1 + 1 - x2 ÷ ç ÷
y = log - 1 - x2 + c 5 êë è 2 ø è 2 ø úû
x
for x = 1 and y = 0, we get c = 0 1 é n-1 æ 5 + 5 ö n-1 æ 5 - 5 ö ù
= êa çç ÷÷ - b çç ÷÷ ú
æ1+ 1 – x2 ö
5 êë è 2 ø è 2 ø úû
y = ln ç ÷ – 1 – x2
ç
è
x ÷
ø 5 é n-1 æ 5 + 1 ö n-1 æ 1 - 5 ö ù
= ê a çç ÷÷ + b çç ÷÷ ú
Hence options (c) and (d) are correct. 5 ëê è 2 ø è 2 ø úû
JEE Advanced 2019 Solved Paper 2019 - 29

= an – 1 a + bn – 1 b = an + bn 44. (a, c, d)

\ option(b) is correct é0 1 a ù é -1 1 -1ù


ê1 2 3 ú ê 8 -6 2 ú
(c) a1 + a2 + a3 + ..... + an M= ê ú ; Adj M = ê ú
ëê 3 b 1 úû ëê -5 3 -1úû
a1 - b1 a 2 - b2 a3 - b3 a n - bn
= + + + .... +
a -b a-b a-b a -b Cofactor of a11 in M = 2 – 3b = –1 = a11 in Adj M
Þb=1
1
= [(a + a2 + a3 + ... + an) – (b + b2 + b3 + ... + bn)] Cofactor of a31 in M = 3 – 2a = –1 = a13 in Adj M
5
Þa=2
1 é a(1 - a n ) b(1 - b n ) ù \ a + b = 2 + 1 = 3 Þ option (a) is correct.
= ê - ú
5 ëê 1 - a 1 - b ûú
0 1 2
|M| = 1 2 3 = –(1 – 9) + 2(1 – 6) = 8 – 10 = –2
1 é a(1 - b)(1 - a n ) - b(1 - a )(1 - bn ) ù
= ê ú 3 1 1
5 ëê (1 - a)(1 - b) ûú
|adj M2| = |M2|2 = |M|4 = (–2)4 = 16
1 é (a - ab)(1 - a n ) - (b - ab)(1 - b n ) ù \ option (b) is incorrect.
= ê ú
5 ëê (1 - a )(1 - b) Also (adj M)–1 = adj M–1
ûú
\ (adj M)–1 + adj M–1 = 2adj (M–1)
1 é (a + 1)(1 - a ) - (b + 1)(1 - b ) ù
n n
2 | M -1 |
= ê ú = adj(M -1 ) = 2|M–1| × (M–1)–1
5 êë 1 - (a + b) + ab úû |M -1
|
1
=2× ×M=–M
1 é a 2 (1 - a n ) - b2 (1 - b n ) ù -2
= ê ú
5 ëê 1-1-1 ûú \ option (c) is correct.
(using a2 – a – 1 = 0, b2 – b – 1 = 0) éa ù é1 ù éa ù é1 ù
ê ú ê ú ê
Now, M ê b ú = ê2 ú Þ b = M ú -1 ê ú
ê ú ê 2ú
1 é (a 2 - b2 ) - (a n+ 2 - bn + 2 ) ù ëê g ûú ëê3 ûú êë g úû êë 3úû
= ê ú
5 ëê -1 ûú é -1 1 -1ù é1 ù éa ù é -2 ù é 1 ù
1 ê ú ê2 ú ê b ú = 1 ê 2 ú = ê -1ú
= 8 -6 2 Þ
é -(a - b)(a + b) (a n+ 2 - b n+ 2 ) ù -2 ê úê ú ê ú -2 ê ú ê ú
= ê + ú = –1 + an + 2 êë-5 3 -1úû êë3 úû êë g úû êë -2úû êë 1 úû
êë a -b a -b úû
\ a – b + g = 1 – (–1) + 1 = 3 \option (d) is correct.
\ option (c) is correct. ®
45. (a, b, d) L1 : r = i$ + l(-i + 2 j + 2k$ )
®
¥ bn ¥
a n + bn L2 : r = m(2i - j + 2k$ )
(d) å n

10n L3 being perpendicular to both L1 and L2, is the
n =1 10 n =1
shortest distance line between L1 & L2.
(using bn = an + bn as proved in option (b))
L is in the direction of (-$i + 2 $j + 2k$ ) × (2$i - $j + 2k$ )
3
a b
¥
æaö
n
æbö
n $i $j k$
= å çè 10 ÷ø +ç ÷ =
10 + 10
n =1 è 10 ø
1-
a
1-
b = -1 2 2 = 6$i + 6 $j - 3k$
10 10 2 -1 2
a b a (10 - b) + b(10 - a ) L1
= + = A
10 - a 10 - b (10 - a)(10 - b)

10(a + b) - 2ab 10 + 2 12 L3
= = =
100 - 10(a + b) + ab 100 - 10 - 1 89
B
\ option (d) is incorrect. L2
If AB is the line of shortest distance then we can
EBD_7801
2019 - 30 JEE Advanced 2019 Solved Paper

consider A(1 – l, 2l, 2l), B(2m, – m, 2m). 3 2


Similarly for E3 : a = ,b= and so on.
\ dr's of AB = 2m + l – 1, –m – 2l, 2m – 2l ( 2) 2
( 2) 2
b
Q AB and L3 are representing the same line Now eccentricity depends on which is same for
a
2m + l - 1 -m - 2l 2m - 2l
\ = = all En, therefore eccentricity for all the En's will remain
6 6 -3
Þ 3l + 3m = 1 ...(1) 4 5
6l – 3m = 0 ...(2) 1- =
9 3
1 2
Solving eqn. (1) and (2): l = , m = \ option (a) is incorrect
9 9
æ8 2 2ö æ 4 -2 4 ö 3 2
\ A ç , , ÷ and B ç , , ÷ for E9 : a = ,b=
è9 9 9ø è9 9 9ø ( 2) 8
( 2)8
\ L3 can be described by
2 $ $ $ 4
® $ 2´
r = (4i + j + k ) + t (2i + 2 j - k ) 2b 2 256 = 1
9 \ length of latus rectum = =
2 $ $ a 3 6
® $ $
or r = (2i - j + 2 k ) + t (2i + 2 j - k ) 16
9
\ option (b) is correct.
æ2 1ö
Also mid-point of AB is ç , 0, ÷ 3 2 24
è 3 3ø Area of R1 = 4 × × =
2 2 2
\ L3 can also be given by
1 ˆ ˆ 3 2 24
(2i + k ) + t (2iˆ + 2 $j - k$ ) , where t Î R
® Area of R2 = 4 × × = 2
r = ( 2) 2
( 2) 2 2
3
Clearly (0, 0, 0) does not lie on 3 2 2
r 2 Area of R3 = 4 × × = and so on
r = (4iˆ + ˆj + kˆ ) + t (2$i + 2 $j - k$ )
3
( 2) 3 ( 2) (2)3
9
\ rr = t (2iˆ + 2 ˆj – kˆ) can not describe the line L3.
We observe
¥ 24 24 24 12
Hence options (a), (b) and (d) are correct, but (c) is å area of R n = + 2 + 3 +.... =
2 2 2 1
= 24
incorrect. n= 1 1-
2
x2 y2 N
46. (b, c) E1 :
9
+
4
=1 \ å (area of R n ) < 24 for each positive integer N.
n=1
R1 : rectangle ABCD with largest area. \ option (c) is correct.
E1 3 2 5
For E9 : a = 8
,b= 8 ,e= 3
D A(x, y) ( 2) ( 2)
2 R1
æ 5 ö
\ focus = (ae, 0) = çç 16 , 0 ÷÷
è ø
3
5
\ distance of focus from centre =
B 16
C
\ option (d) is incorrect.
Area of R1 = A = 2x × 2y 5R 3R 5R
47. (b, d) B1 B2 B3
2 8 5G 5G 3G
Þ A = 4x × 9 - x2 = x 9 - x 2
3 3
3 3 4
é 2 ù P(B1) = , P(B2) = , P(B3) =
dA 8 ê 9 - x 2 - x ú
10 10 10
= 3
dx êë 9 - x 2 úû 5 5 3
P(G / B1) = , P(G / B2) = , P(G / B3) =
10 8 8
8 é 9 - 2x2 ù 3 2 9 2
Þ 3ê ú = 0Þ x= ,y= 9- = 4 3 3
´ =
êë 9 - x úû
2 2 3 2 2 P(B3 Ç G) = P(B3) P(G / B3) =
10 8 20
3 2 3
\ for E2 : a = ,b= \ option (a ) is incorrect P(G / B3) =
2 2 8
JEE Advanced 2019 Solved Paper 2019 - 31

\ option (b) is correct. Þ f ' is not differentiable at x = 1


P(G / B3 )P(B3 ) \ option (c) is correct. For x < 0 , f'(x) = 5(x + 1)4 – 1
P(B3 / G) =
P(G / B1 ) P(B1 ) + P(G / B2 )P(B2 ) + P(G / B3 )P(B3 ) 1 1/4
æ1ö
f'(x) = 0 Þ (x + 1)4 = Þ x = -1 ± ç ÷
3 4 12 5 è5ø
´
8 10 80 1/4
=
5 3 5 3 3 4
=
15 15 12 æ1ö
´ + ´ + ´ + + Þ f is changing its nature at x = –1 – ç ÷
è5ø
10 10 8 10 8 10 100 80 80
\ f is not increasing on (–¥, 0)
12 400 60 4
= ´ = = \ option (b) is incorrect.
80 60 + 75 + 60 195 13
ìï2 x - 1 , 0 £ x <1
\ Option (c) is incorrect. f 'x = í 2
ïî2( x - 2) - 1 , 1 £ x < 3
P(G) = P(G / B1)P(B1) + P(G / B2)P(B2) + P(G / B3)P(B3) From its graph f'(x) has local maxima at x =1.

5 3 5 3 3 4 60 + 75 + 60 195 39 (1, 1)
= ´ + ´ + ´ = = =
10 10 8 10 8 10 400 400 80
(3, 1)
\ option (d) is correct.
48. (a, c, d) X
ì( x5 + 5 x 4 + 10 x3 + 10 x 2 + 5 x + 1) - 2 x , x<0
ï (0, –1)
ï x2 - 2 ´ 1 ´ x + 1 + 3 , 0 £ x <1 (2, –1)
ï 2 4 4
ï
f ( x) = í 2 3 8
2 \ option (a) is correct.
ï x - 4x + 7 x - , 1£ x < 3
ï 3 3 49. (0.50) n(S) = 29 = 512
ï 10 E2 contains those matrices in which sum of entries is 7.
ïî( x - 2) log e ( x - 2) - x + 3 , x ³3
\ there will be 7 one's and 2 zeroe's.
ì( x + 1)5 - 2 x , x< 0 \ n(E2) = 9C2 = 36
ï
ïæ 1ö
2
3 E1 Ç E2 contains those matrices in which 7 ones, 2
ïç x - ÷ + , 0 £ x <1 zeroes are there and det is zero.
ïè 2 ø 4
=í Det(A) = 0 in cases when two rows/columns are
2 8
ï x3 - 4 x 2 + 7 x - , 1£ x < 3 identical for example
ï 3 3
ï 10 0 0 1 1 0 0 0 1 0
ï( x - 2) loge ( x - 2) - x + , x³3
î 3 1 1 1 or 1 1 1 or 1 1 1
For x = 0, f(x) = 1. For x < 0, f(x) = (x + 1)5 – 2x 1 1 1 1 1 1 1 1 1
It decreases to –¥. \ f(x) Î (–¥, 1] for x £ 0 \ n(E1 Ç E2) = 3C1 × 3C1 × 2 = 18

P( E1 Ç E2 ) 18 / 512 1
1 \ P ( E1 / E2 ) = = = = 0.50
For x = 3, f(x) = P ( E2 ) 36 / 512 2
3
For x ³ 3, f(x) increases to ¥ 50. (3) a, b, c are distinct non zero integers
Min. value of |a + bw + cw2|2 is to be found
é1 ö |a + bw + cw2|2
\ f(x) Î ê , ¥ ÷ for x ³ 3
ë3 ø æ -1 + i 3 ö æ -1 - i 3 ö
2

Combining the two f(x) Î R Þ f is onto. = a + bç ÷ + cç ÷


è 2 ø è 2 ø
\ option (d) is correct. 2
1 i 3
ì5( x + 1)4 - 2 = ( 2a - b - c ) + (b - c )
, x< 0 2 2
ï
ï2 x - 1 , 0 £ x <1 1 2 3 2
f '( x ) = í = (2a - b - c ) + (b - c)
2 4 4
ï 2( x - 2) - 1 ,1£ x < 3
ïlog ( x - 2) 1 2 2 2
î e , x³3 = (4a + b + c – 4ab + 2bc – 4ac + 3b2 + 3c2 – 6bc)
4
Lf " (1) = 2, Rf " (1) = –4, = a2 + b2 + c2 – ab – bc – ca
EBD_7801
2019 - 32 JEE Advanced 2019 Solved Paper

2 p /4 esin x
= 1 éë( a - b )2 + (b - c)2 + (c - a)2 ùû
p ò-p /4 esin x + 1 ( 2 – cos 2 x )
I=
2
For min. value, let us consider a = 3, b = 2, c = 1
( ) ........(2)

1 6 Adding (1) and (2):


\ min value = [1 + 1 + 4] = = 3
2 2
2 p /4 1 + esin x
p ò-p /4 esin x + 1 ( 2 – cos 2 x )
16 - 18 - 23 25 5 2I = dx
51. (10) AN =
64 + 36
= = = BN
10 2 ( )
2 p /4 1
=
p ò – p /4 2 – cos 2 x
dx

P 2 A(2,3)

2 p /4 1 2 p /4 dx
p ò0 2 – cos 2 x
I= dx = ò
8x – 6y – 23 = 0 p 0 1 + 2sin 2 x
N
2 p /4 sec2 x 2 p /4 sec2 x
p ò0 sec2 x + 2tan 2 x p ò0 1 + 3tan 2 x
Q 1 BC = dx = dx

Put tan x = t Þ sec2 x dx = dt


p
At x = 0, t = 0 ; At x = , t = 1
4
1
\I= ò
2 1 1
p 0 1 + 3t 2
dt =
2é 1
p êë 3
tan –1
3 t
ù
ú
û0
( )
2 é 1 pù 2
= ê ´ = \ 27 I 2 = 4
C p ë 3 3 úû 3 3
From DCPA ~ DCQB 54. (0.75) Given lines are rr = liˆ ......(1)
CA PA CA 2 r
= = r = m(iˆ + ˆj ) ......(2)
CB QB
Þ r
CA - 5 1 ˆ
r = v(iˆ + ˆj + k ) .......(3)
Þ CA = 2CA – 10 Þ CA = 10 These lines cut the plane x + y + z = 1 at points
52. (157) AP (1, 3) : 1, 4, 7, 10, 13 ......... A (l, 0, 0), B ( m, m,0 ) and C ( n, n, n) respectively
AP (2, 5) : 2, 7, 12,17, 22 ......... Q A lies on plane
AP (3, 7) : 3, 10, 17, 24, 31 ......... Þ l = 1 Þ A (1, 0, 0) Q B lies on plane
For AP (1, 3) Ç AP (2, 5) Ç AP (3, 7)
1 æ1 1 ö
first term will be the minimum common value of a term. Þ m+m=1 Þm = ÞB ç 2 , 2 ,0 ÷
2 è ø
\ we need to find that minimum number which
when divided by 7 leaves remainder 3 (7m + 3) Q C lies on plane
and when divided by 5 leaves remainder 2 (5p + 2) 1 æ1 1 1ö
and when divided by 3 leaves remainder 1 (3q + 1) Þ n +n + n =1 Þn = ÞCç , , ÷
3 è 3 3 3ø
By hit and trial 52 is such number (7 × 7 + 3)
iˆ ˆj kˆ
\ first term 'a' of intersection AP = 52
Also common difference 'd' of intersection 1 uuur uuur 1 –1 1
Area (D ABC) = AB ´ AC = 0
AP = LCM (7, 5, 3) = 105 \ a + d = 52 + 105 = 157 2 2 2 2
–2 1 1
2 p /4 dx
p ò– p /4 1 + esin x ( 2 – cos 2 x )
53. (4) I= ......(1) 3 3 3
( ) 1 1ˆ 1 ˆ 1 ˆ 1 1 3
= i + j+ k = ´ 3=
2 p /4 dx 2 6 6 6 2 6 12
I= ò
p – p /4
1+ e(- sin x
)
( 2 – cos 2 x )
2 3 3
\ ( 6D ) = 36 ´ = = 0.75
é using b f ( x)dx = b f (a + b – x )dx ù 144 4
êë òa òa úû
JEE Advanced 2019 Solved Paper 2019 - 33

Paper - 2
Physics point A lies on the axial line of electric dipole. The
1. (a, b, c) external electric field E 0 should also be in the
Gain in kinetic energy = Loss in potential energy direction of O to A. Now considering point B which
is a point on the equitorial line of the electric dipole.
Kp
The electric field here due to dipole is in a
R3
direction opposite to the dipole. The external electric
field should cancel out this field. Therefore (b) is
the correct option.
® r
- Kp
L/2 Further 0E = ...(i)
L cos 60° R3
60° a 60° 30°
2 r at The electric field at A is
® r ® ® ®
ar cos 60° + at cos 30° 2Kp ®
EA = + E 0 = -2 E0 + E 0 = - E 0
3
R
1 2 l
Iw = mg (1 – cos 60°) Option (c) is wrong.
2 2
ml 2 2 l 3g E0 E0
\ w = mg \ w =
3 2 2l B
Option (a) is correct Eax
Now, t = Ia 45° 45° A
Eeq
l 1 P
\ mg × sin 60° = ml2a
2 3 O
3 3g
\ a=
4 l
\ (d) is an incorrect option
l 3 3g 1 p0
Further at = a= Further from (i) E0 = ´ 2
2 8 4 p Î0 2( R 3 )
l 3g l 3g 1/3
Also ar = w2 = ´ = p0 é p0 ù
3
2 2 l 2 4 \ R = \ R= ê ú
4p Î0 E 0 ë 4pÎ0 E 0 û
So (b) is a correct option.
For vertical motion of C.M Option (d) is correct.
mg – N = m(arcos 60° + at cos 30°) hc
é 3g 1 3 3g 3. (b, c) Here, E4 – E1 = = Dpa × C ...(i)
3ù la
\ mg – N = m ê 4 ´ 2 + 8 ´ 2 ú
ë û hC
E4 – Em = = Dpe × C ...(ii)
4/Mg le
\ N= le
16 Dpa
\ = =5
Option (c) is correct Dpe la
2. (b, d) It is given at all points on the circle are at the same \ (a) is an incorrect option.
potential. We further know that electric field is Dpa -0.85 - (-13.6)
perpendicular to such a line that is the direction of \ = =5
Dpe -0.85 - (-E m )
electric field is either radial or the magnitude of
Þ 12.75 = 5(Em – 0.85) Þ Em = 3.4eV
electric field should be zero at points on the circle.
Þ m=2
Now considering point A, the electric field due to
Þ (c) is a correct option
2Kp 22
dipole should be (directed from O to A) as K × E 2 V22 æ n1 ö æ1ö 1
3 = =ç ÷ = ç ÷ =
R K × E1 V12 è n 2 ø è2ø 4
EBD_7801
2019 - 34 JEE Advanced 2019 Solved Paper

Þ (b) is the correct option. 5. (c) Y

hc P
From (ii), E4 – E2 =
le
1242 aq
O
\ le = -0.85 - (-3.4) = 487 nm d q
d sin
a sin
Þ Option (d) is incorrect. d
D
4. (d) According to constraint relation
a 2 + a3 yd
a1 = Þ a1 – a3 = a2 – a1 Path difference = d sin a + d sin q = da +
2 D
Þ Option (d) is correct [when a and q are small]
a1
yd
Kx 2T For a = 0, path difference =
D
–3 –4
= 11 × 0.3 × 10 = 33 × 10 mm

2T path difference 33 ´ 10 -4 11
Now = =
l 600 ´ 10-6 2

T 1
a2
T = odd multiple of
2
a3 This implies destructive interference at P.
mg
\ Option (a) is wrong.
2M
Let ‘x’ be the extension of the spring at a certain lD
Fringe width b = is independent of path
instant. Then d
2Mg – T = 2Ma3 difference
Mg – T = Ma2 \ Option (b) is wrong.
On solving we get,
0.36
4g 2kx For a = degree (at point P)
a1 = - ...(i) p
7 14M
é yù
3k 3k Path difference = d êa + ú
\ w2 = \ w= ë Dû
14M 14M
é 0.36 11´10-3 ù
4Mg 3kx
and T = + ...(ii) = 0.3 × 10 180 +
–3 ê
1
ú m = 3900 nm
7 7 ëê ûú
For a1 = 0 (Maximum extension of spring) we have
from (i) 3900 13
Now path difference = = is an odd multiple
4g 3kx 600 2
- =0
7 14M 1
3kx 8Mg of
\ 4g = \ x= 2
2M 3k This implies destructive intereference at P.
16Mg Option (c) is correct.
\ x0 = 2x =
3k
0.36
x0 1 æ 16Mg ö 4Mg For a = degree (at point O)
= = p
4 4 çè 3k ÷ø 3k
For x=
0.36
4g 3k 4Mg 2g Path difference = da = 0.3 × 10–3 ×
From (i) a1 = - ´ = 180
7 14M 3x 7
= 600 × 10–9m = 600 nm
x
At x = 0 the particle is at the mean position and its path difference
2 Now =1
l
x0 3k 8Mg 3k
velocity = Aw = = This implies constructive interference at O. Option
2 14M 3k 14M (d) is incorrect.
JEE Advanced 2019 Solved Paper 2019 - 35

8. (a,d) When the small particle moving with velocity v0


5R 7R undergoes an elastic collision with the heavy
n1Cp1 + n 2 Cp2 5´ + 1´
6. (a, b, d) g m = = 2 2 = 1.6 movable that piston moving with velocity V, it
n1C v1 + n 2 Cv2 3R 5R
5´ + 1´ acquires a new velocity v0 + 2v. Therefore the
2 2
Option (d) is correct. increase in velocity after every collision is 2V. Option
For an adiabatic process (d) is correct.
1.6
æV ö
= P0 ( 4 )
1.6
P = P0 ç 0 ÷
è V ø
= P0(22)1.6 = P0 23.2 = 9.2 P0 v
Option (b) is correct.
x
P2V2 - P1V 1 9.2P0 ´ (V0 / 4) - P0 V0 –13P0V0
W= = =
1- g 1 - 1.6 6 Time period of collision when the piston is at a dis-
But P0V0 = 6RT0 (as n = 5 + 1 = 6) tance ‘x’ from the closed end is
–13 ( 6RT0 ) distance 2x
\ W= = -13RT0 \ |W| = 13 RT0 T= =
6 speed v
Option (a) is correct.
Where v is the speed of the particle at that time.
K.E = n(C v )T
v
\ rate at which the particle strikes the piston =
PVT0 9.2P0 ( V0 / 4 ) T0 2x
where T = = = 2.3T0
P0 V0 P0 V0
v
when x = L, the frequency =
2L
é 3R 5R ù
ê 5 ´ 2 + 1´ 2 ú Option (c) is incorrect.
= 6´ ê ú ´ 2.3T0 = 23T0
5 +1 The rate of change of speed of the particle
ê ú
ë û dv
= = (frequency) × 2v
Option (c) is incorrect. dt
7. (b, d) Case - I
v dv Vdt -dx
\ dv = 2vdt \ = =
H 30 2x v x x
H1 = = = 20cm
1.5 1.5 Where dx is the distance travelled by the piston in
Case - II time dt. The minus sign indicates decrease in ‘x’ with
time.
- n1 n 2 n 2 - n1
+ = Therefore option (b) is incorrect.
u v R V x
dv dx
–1.5 1 1 - 1.5
\ ò v
=-ò
x
\ + = V0 L0
-30 V -300
v x v 0 L0
\ ln = - ln or |v|=
1 1 1 v0 L0 x
\ = -
V 600 20 L0 V0 L0
when x = we have | V | = = 2V0
\ V = –20.68 cm 2 L 0 /2
\ H2 = 20.68 cm 1
\ K.E L0/ 2 = m(2V0 ) 2
Case - III 2
when x = L0 we have |V| = v0
–1.5 1 1 - 1.5
\ + =
-30 V 300 K.E L0/ 2
1 \ =4
\ K.E L0 = mV02
2 K.E L0
1 1 1
\ =- - Þ V = -19.35cm
V 600 20 \ Option (a) is correct.
\ H3 = 19.35 cm
EBD_7801
2019 - 36 JEE Advanced 2019 Solved Paper

9. (135.00)
226
Ra ®
222 4
Rn + 2He é 1 ù
88 86 2u 20 ê ú
= sin q cos q ê1 - 1 ú
By momentum conservation g ë a2 û
2m a K.Ea = 2m Rn K.E Rn 2u 0 sin q é 1 1 ù 2u sin q
ST = ê1+ + + ....ú = 0
g ë a a 2
û g
4
\ K.E Rn = ´ 4.44 = 0.08 Mev
222 é 1 ù
ê 1ú SR
ê1 - ú \ 0.8V1 =
Energy of g-photon = [Dm × 931] – (K.Ea + K.ERn) ST
ë aû
= [226.005 – (222 + 4)] × 931 – (4.44 + 0.08) MeV
é a2 ù
= 135 KeV ê 2 ú
ê a - 1 úû
Rt Rt \ 0.8(u 0 cos q) = (u 0 cos q) ë
E - Blv - é a ù
10. (0.63) Here i = (1 - e L ) = [1 - e L ] êë a - 1úû
R R

é -1´10-3 ù a -1 a2
a
1 ´ 0.1 ´ 10 -2
ê -3 ú
\ 0.8 = = ´
\ 0.8a + 0.8 = a
= ë1 - e 10 û a -1 a a
2 +1
1
\ a = 4.00
= 10–3 [1 – e–1] = 10–3 [1 – 0.37] = 0.63 × 10–3
11. (1.50) In DAPQ 13. (1) Change in momentum of photon = change in mo-
90 × r + 90 – C + 75 = 180 mentum of mirror
\ r + C = 75 `....(i) é æ h öù
Applying snell’s law at Q we have 2 ê N ç ÷ ú = MVmax
ë è l øû
Nh
A \ 2 = M(AW) [ Q Vmax = Aw]
l
75° 24
10-6 ´ 8p ´ 10 -6
æ MW ö Al 10
N=ç ÷ = ´
–r è h ø 2 4p 2
P 90 90–c Q
q = 1 × 1012
r c 90°
14. (1.39%)
u ± Du = (75 – 45) ± (0.25 + 0.25) = (30 ± 0.5) cm
v ± Dv = (135 – 75) ± (0.25 + 0.25) = (60 ± 0.5) cm
3 sin C = n sin 90° We know that

3 sin C = n ....(ii) l 1 1
- =
Applying snell’s law at P we have v u f

1 × sinq = dv du df
3 sin r = 3 sin (75 – C) From (i) \+ + = ....(i)
2 2
For q = 60° v u f2

3 1 1 1 1 1 1
= 3 sin (75 – C) Now - =Þ - =
\ C = 45° v u f 60 -30 f
2
3 1 1
From (ii) n = 3 sin 45° = \ = \f=20cm
2 f 20
\ n2 = 1.50 Substituting the values in (1)
R u 02 sin 2q´ g 0.5 0.5 df
12. (4.00) V1 = = = u 0 cos q (i) + =
T g ´ 2u 0 sin q \ df = 0.277
( 60 )2
( 30 ) 2
( 20 )2
Now
u 20 é 1 1 ù df 0.277
SR = (sin 2q) ê1 + + + ....ú \ ´ 100 = ´ 100 = 1.388% = 1.39%
g ë a 2
a 4
û f 20
JEE Advanced 2019 Solved Paper 2019 - 37

[There is no change in temperature from 1 ® 2]


p T n 2 l2 m
15. (b) We know that n = 2l m Þ T =
p2 3 æ T ö
= 1´ R ç T0 - 0 ÷ = RT0
2 è 3 ø
f 02 4L20m
Case 1 T0 = [For monoatomic gas f = 3]
l2
\ II – R
2
æ3ö III. Q1®2®3 = Q1®2 + Q2®3 = W1®3 + DU2®3
f02 4 ç ÷ L20 ( 2m )
è2ø T RT0 RT0
Case 2 T2 = = 0
( 3)2 2 = ln 2 + RT0 = [ln 2 + 3]
3 3
\ III – T
2 IV. Q1®2 = W1®2 [Q DU1®2 = 0]
æ5ö
f 02 4 ç ÷ L20 ( 3m )
Case 3 T = è2ø 3 RT0
3 = T0 = ln 2
2 16
5 3
IV – P
2
æ7ö
f02 4 ç ÷ L20 ( 4m ) 18. (a) I. W1®2®3 = W1®2 + W2®3
è4ø T
Case 4 T4 = = 0 [W2®3 = 0 as there is no change in volume]
(14 )2 16
= P0 × V0 + O
\ Correct option is I-P, II-Q, III-T, IV-U
RT0 1
= P0 V0 = [Q P0 V0 = RT0 given]
1 T 3 3
16. (c) We know that n = for first mode of vibra- \I®Q
2l m
II. DU1®2®3 = DU1®2 + DU2®3
tion
= nCvDT1®2 + nCvDT2®3
For ‘n’ to be maximum, ‘l’ should be minimum.
3R 3R
1 T0 = 1´ (Tf - Ti )1®2 + 1´ [Tf - Ti ]2®3
Case (i) f0 = 2 2
2L0 m
3 3 é 3P0 ù
1 T0 f = [2P0 V0 - P0 V0 ] + ê ´ 2V0 - P0 ´ 2V0 ú
2 2ë 2 û
Case (ii) f 2 = = 0
2L0 2m 2 = 3P0V0 = RT0
1 T0 f II ® R
Case (iii) f3 = = 0
2L0 4m 3 III. Q1®2®3 = Q1®2 + Q2®3

1 T0 f0 = nCpDT1®2 + nCvDT2®3
Case (iv) f 4 = =
2L0 4m 2 5 3 é 3P ù
= P0 V0 + 1 ´ ê 0 ´ 2V0 - P2 (2V0 ) ú
The correct option is I - P, II - R, III - S, IV - Q 2 2ë 2 û
8 8 RT0 4
17. (c) I. W1®2®3 = W1®2 + W2-3 = P0 V0 = ´ = RT0
2 2 3 3
T0 2V0 III – S
1R ln + zero
3 V0 IV. Q1®2 = nCpDT1®2 = nCp(Tf – Ti)
[Q n = 1, and W2 ® 3 there is no change in volume]
5 é P (2V0 ) P0 V0 ù
= 1´ R ê 0 -
\ I–P 2 ë R R úû
II. DU1®2®3 = DU1®2 + DU2®3 [Q PV = nRT]

f 5 5 æ RT ö 5RT0
= O + nCv DT = n RDT = P0 V0 = ç 0 ÷ =
2 2 2è 3 ø 6
\ IV – U
EBD_7801
2019 - 38 JEE Advanced 2019 Solved Paper

Chemistry This is a dehalogenation reaction of vicinal


dihalide. In this reaction, an alkene is produced on
treatment of dihalide with Zn metal by loosing a
19. (b, c, d) Aqua regia is a mixture of conc. HCl and conc. molecule of ZnX2.
HNO3 in 3:1 ratio. 22. (a, c)
When gold dissolves in aqua regia, the species Nylon-6 is obtained by heating caprolactum with
− water at high temperature. It has amide linkages.
formed is AuCl4 in which gold is in +3 oxidation

state.
In the absence of air the reaction between gold and
aquaregia

Au + HNO3 + 4HCl → H+ + [ AuCl4 ] + 2H 2 O + NO




NO2 will not be produced. Yellow colour is due to Cellulose has only β-D-glucose units that are
its decomposition into NOCl and Cl2. joined by glycosidic linkages between C–1 of one
20. (a, b, c) Gold extraction: glucose unit and C–4 of the next glucose unit
4Au + 8NaCN + 2H 2 O + O 2 → 4Na [ Au(CN) 2 ] + 4NaOH
(Q) (R ) Teflon is prepared by heating tetrafluoromethane in
presence of a persulphate catalyst at high pressure.
4Na [ Au(CN) 2 ] + 2Zn → 2Na 2 [ Zn(CN) 4 ] + 4Au
(R ) (T) (Z) Catalyst
CF2 = CF2 = → [ CF2 – CF2 ] n
High pressure
21. (a, c)
Tetrafluoroe thene Teflon
Natural rubber is a linear polymer of isoprene
(2-methyl-1, 3-butadine) containing cis alkene
units. It is also called cis - 1, 4 - polyisoprene.
23. (b, c)

This process of elimination of carbon dioxide from


a carboxylic acid is known as decarboxylation.

This is Kolbe's electrolytic method.

Alkyl halides (except fluorides) on reduction with



zinc and dilute hydrochloric acid give alkanes.


JEE Advanced 2019 Solved Paper 2019 - 39

26. (b, c, d)
Zn + H 2SO 4 → G + R + X

24. (a, c) Given, azimuthal quantum no. (l) = 2 (d-subshell) Hot & Conc.

Zn + H 2SO 4 → ZnSO 4 + SO 2 + 2H 2 O
Magnetic quantum no.(m) = 0(dz2) (G) (R ) (X)
2 2
z 2 Zn + Conc. NaOH → Na 2 ZnO 2 + H 2
E=
−13.6 =
−13.6 × =
−3.4
n 2
n2 (T) (Q)

ZnSO 4 + H 2S + NH 4 OH → ZnS + (NH 4 ) 2 SO 4 + H 2 O
22 (G) (Z) (Y) (X)
13.6 × =
3.4
n2 (a) Oxidation state of Zn in Na2ZnO2 is +2
n2 = 42  ⇒ n=4 (b) Bond order of Q is one for H2.
(c) ZnS is white in colour
Radial node = n – l – 1 = 4 – 2 – 1 = 1
(d) SO2 is angular in shape
Angular node = l = 2 27. (6)

Wave function corresponds to ψ4,2,0. It represents


4d z 2 -orbital which has only one radial nodes and

two angular nodes. It experiences nuclear charge of
2e units.
25. (a, d)

cis Bond angles are:


N(1) Cl(a) – Mn – N(1)
Cl(a) N(2) Cl(a) – Mn – N(2)
Cl(a) – Mn – N(4)
Mn
Cl(b) N(3) Cl(b) – Mn – N(1)
N(4) Cl(b) – Mn – N(3)
Cl(b) – Mn – N(4)

28. (288) Concentrated nitric acid oxidises sulphur to


sulphuric acid (+6 oxidation state).
S8 + 48HNO3 → 8H2SO4 + 48NO2 + 16H2O
Rhombic sulphur

1 mole S8 produces 16 × 18 g of H2O i.e. 288 g of H2O.


EBD_7801
2019 - 40 JEE Advanced 2019 Solved Paper


29. (2.30) 2N 2O5 (g) 
→ 2N 2O 4 (g) + O 2 (g)

Given rate constant of the reaction = 5 × 10–4sec–1

1 dPN 2O5
= K overall PN 2O5 32. (2.98)
2 dt
dPN 2O5
⇒ =
2K overall PN 2O5 Xurea = 0.05
dt
2N 2 O5 (g) → 2N 2 O 4 (g) + O 2 (g)
Mass of water = 900 g
x
1+ =1.45
2 900
=
Number of moles of water = n H 2O
= 50
x = 0.90 atm 18
For a first order reaction,
Let moles of urea = nU
2.303 [ P ]o
t= log
K [P]
nU
= 0.05
2.303 [ P ]o n U + 50
t= log
2K [P]
50
2.303 3 1 n=
U = 2.63
y × 10 = −4
log 19
2 × 5 × 10 0.1
23.03 Mass of urea = 2.63 × 60 =
157.8
=y = 2.30
10
30. (6) Given density (d) of solution = 1.2 g/mL.

mass of solution 900 + 157.8


=
Vsolution = = 881.5 mL
density (g/cc) 1.2

nV 2.63
M= × 1000 = × 1000 = 2.98
VmL 881.5

33. (d) n2 n2
r∝ or r =
0.529 ×
Z Z

nh
L ∝ n or mvr =

Z2 Z2
KE ∝ 2
or KE =
+13.6 2
n n

31. (10)
−Z 2 Z2
PE ∝ or PE =−2 × 13.6 ×
n2 n2

n2
34. (c) rα ; Lα n
Z

Z2 −Z2
KE α ; PE α
n2 n2
JEE Advanced 2019 Solved Paper 2019 - 41

35. (b)

36. (a)
EBD_7801
2019 - 42 JEE Advanced 2019 Solved Paper

1. Mathematics 6 6
= å ( P 'k ) 'A 'Px' = å Pk APk' =X
37. (a, b, d) F(x) = (x – 1) (x – 2) (x – 5) k =1 k =1
\ X is a symmetric matrix.
x
Option (a) is correct.
F(x) = ò f (t) dt, x > 0
Sum of diagonals entries of X = Trace X
0
6 6
F'(x) = f (x) = (x - 1)(x - 2)(x - 5) = Trace å (Pk APk' ) = å Trace (Pk APk' )
k =1 k =1
F'(x) = 0 Þ x = 1, 2, 5
6
F"(x) = (x - 2)(x - 5) + (x - 1)(x - 5) + (x - 1)(x - 2)
= å Trace(APk Pk' ) ( using Trace AB = Trace BA )
F"(1) = + ve, F"(2) = –ve, F"(5) = +ve k =1
\ F(x) has local minima at x = 1 and x = 5 and local
6 6
maxima at x = 2.
= å Trace (AI) = å TraceA = 6 ´ TraceA
Also
k =1 k =1
x x = 6 × (2 + 0 + 1) = 18
F(x) = ò (t - 1)(t - 2)(t - 5)dt =ò (t 3 - 8t 2 + 17t - 10)dt \ option (b) is correct
0 0
é1ù 6 æ é1ù ö é1ù 6 é6ù
x 4 8x 3 17x 2 ç ' ê ú÷ 6
= - + - 10x X ê1ú = å Pk A ç Pk ê1ú ÷ = å Pk A ê1ú = å Pk êê3úú
ê ú ê ú
4 3 2 k =1 ç ê1ú ÷ k =1 k =1
ëê1ûú è ë ûø ëê1ûú ëê6ûú
x
= (3x 3 - 32x 2 + 102x - 120)
12 é 2 2 2ù é6 ù é30ù é1ù é1ù
= ê 2 2 2ú ê3 ú = ê30ú = 30 ê1ú = a êê1úú
ê ú ê ú ê ú ê ú
F(1), F(2), F(5) < 0 êë 2 2 2úû êë6 úû êë30úû êë1úû êë1úû
\ approximate graph of F(x) for x Î (0, 5) is
\ a = 30
Y Hence option (d) is correct.
é1ù
é1ù é1ù êú
êú êú ê1ú
Also X ê1ú < 30 êê1úú
ê ú Þ (X – 30 I) ê ú =0
ê1ú
X ê1ú ê1ú ëû
O ëû ëû
Þ X – 30 I = 0 Þ |X – 30 I| = 0
Þ X – 30 I is not invertible
\ option (c) is incorrcet.
æ k ∗1 ö æk ∗2 ö
å 2sin ççèç n ∗ 2 p÷÷÷øsin èççç n ∗ 2 pø÷÷÷
n
_
\ F(x) < 0, for all x Î (0,5)
k <0
Hence options (a), (b) and (d) are correct but (c) is 39. (b, c, d) f (n) =
æ ö
å 2sin çç k ∗ 1 ÷÷ p
n
2
incorrect. çè n ∗ 2 ÷ø
k <0
38. (a,b, d) We observe that P1¢ = P1 , P2¢ = P2 , P3¢ = P3 ,
where n is non negative integer
P4¢ = P5 , P5¢ = P4 , P6¢ = P6 ,
é æ p ö ∋2k ∗ 3(p ùú
å êêcos èççç n ∗ 2 ÷ø÷÷ – cos
n
Also Pk Pk¢ = I for k = 1 to 6.
n∗2 ú
Now <
k< 0 ë û
ê1 – cos ∋
n é
2 k ∗ 1(p ù
6 é2 1 3ù å ê
k <0 ë n ∗ 2 úû
ú
X= å Pk APk' where A = êê1 0 2 úú and A ' = A
k =1 êë3 2 1 úû æ p ÷ö é 3p 5p ∋2n ∗ 3( p ùú
∋ n ∗ 1(cos ççç ÷÷ – êê cos ∗ cos ∗ ......... ∗ cos
èn ∗ 2ø ë n ∗2 n∗2 n ∗ 2 úû
<
æ 6 ö 6 é 2p 4p 2 ∋ n ∗ 1(p ù
n ∗ 1 – ê cos ú
'
X ' = ç å Pk APk¢ ÷ = å ( Pk APk¢ ) ∗ cos ∗ ........ ∗ cos
ç ÷ ê n ∗2 n∗2 n ∗ 2 úû
ë
è k =1 ø k =1
JEE Advanced 2019 Solved Paper 2019 - 43

cos px ( px - 2 tan px )
sin ∋n ∗ 1( p Þ =0
æ p ö÷
∋n ∗ 1( cos ççç n ∗ 2 .cos ∋2n ∗ 6( p x3
÷÷ –
è n ∗ 2ø æ p ÷ö 2 ∋ n ∗ 2( 1 3 5 7
sin çç
çè n ∗ 2 ø÷÷ Þ cos px = 0 Þ x = , , , ,...
< 2 2 2 2
sin ∋n ∗ 1(p and px– 2tanpx = 0 which can be solved by drawing the
n ∗1 – n ∗ 2 .cos ∋2n ∗ 4( p graphs of y = px and y = 2 tanpx, as follows
æ p ÷ö 2 ∋ n ∗ 2(
sin çç
çè n ∗ 2 ÷÷ø
Y
y = px
p p æ p ö÷
∋n ∗1( cos ∗ c os ∋n ∗ 2(cos ççç ÷
n ∗2 n∗2 < è n ∗ 2 ø÷
<
n ∗1 ∗ 1 n∗2

æ p ö
\ f ( n ) = cos ç ÷ X
èn+2ø 1 1 3 2 5 3 4 9 5 11 6
2 2 2 2 2
æ p ö
lim f ( n ) = lim cos ç ÷=1
n®¥ n®¥ èn+2ø
\ option (a) is incorrect.

æ p ö p 3
f ( 4) = cos ç = cos =
è 4 + 2 ÷ø 6 2 Plotting the stationary points on number line and
finding the sign of f '(x) in different intervals we ob-
\ option (b) is correct serve
(
It a = tan cos f ( 6)
-1
) 0 1 2 3 4

æ æ pö ö p – – + + –5 – –
= tan ç cos -1 ç cos ÷ ÷ = tan 0 1 3
+
7
+
9
è è 8øø 8 y1 x1 2 y2 2 x2 2
2 2
p i.e. xn+1 – xn > 2 for every n
2 tan
p 8 =1
tan = 1 Þ æ 1ö
xn Î ç 2n, 2n + ÷ for every n
4 1 - tan 2 p / 8 è 2ø
2a | xn - yn | > 1 for every n
Þ = 1 or a2 + 2a – 1 = 0
1 - a2 x1 > y1
\ option (c) is correct Hence options a, b, c are correct, but option d is
incorrect.
( ) æ æ pö ö æ pö
sin 7 cos –1 f ( 5) = sin ç 7cos -1 ç cos ÷ ÷ = sin ç 7 ´ ÷
è è 7øø è 7ø
41. (a, d) We can choose
P ( l,0,0) on L1
= sin p = 0
\ option (d) is correct. Q ( 0, m,1) on L2
sin px R (1,1, v ) on L3
40. (a, b, c) f ( x ) = ,x > 0
x 2 uuur uuur
If P, Q, R are collinear, PQ || PR
For points of local max/min, f ' (x) = 0
l +m +1 l l -1
px2 cos px - 2 x sin px Þ = = Þm= ,v =
Þ =0 l - 1 +1 + v l -1 l
x4
Clearly l ¹ 0,1
x p cos px - 2sin px
Þ =0 æ l ö
x3 q ç 0, ,1
è l - 1 ÷ø
EBD_7801
2019 - 44 JEE Advanced 2019 Solved Paper

ˆ 1 ˆj
For Q = k– é 1 1 1ù é 2 x xù
(a) ê ú ê ú
2 43. (a, d) P = ê 0 2 2ú ;Q < ê 0 4 0ú
ê ú ê ú
l 1 ê 0 0 3ú ê x x 6ú
= - Þ 3l = +1 , which is possible. ë û ë û
l -1 2 R = PQP–1
(b) For Q = k$ Þ det R = det (PQP–1) = |P| |Q| |P–1| = |Q| = 4 (12 – x2)
é 2 x xù
l ê ú
= 0 Þ l = 0 , not possible Also, det ê 0 4 0ú + 8 = 4 (10 – x2) + 8 = 4(12 – x2)
l -1 ê ú
ê x x 5ú
(c) For Q = k$ + $j ë û

l é 2 x xù
= 1 Þ l = l - 1, not possible ê ú
l -1 ê 0 4 0ú ∗ 8 ! x Î R
\ det R = det ê ú
ê x x 5ú
1$ ë û
(d) For Q = k$ + j
2 \ option (a) is correct
For x = 1, det R = 4 (12 – 1 ) ¹ 0
l 1
= Þ 2l = l - 1 Þ l = - 1 , é ù é0ù
l -1 2 ê ú ê ú
which is possible [ R ê α ú < ê0ú
ê ú ê ú will have only trivial solution
Hence options (a) and (d) are correct and options (b) ê φ ú ê0ú
ë û ë û
and (c) are incorrect.
42. (a, d) i.e., a = 0 b = 0, g = 0
\ option (b) is incorrect
3
1 + 3 2 + ... + 3 n For PQ = QP
lim = 54
n®¥ æ 1 1 1 ö a11 in PQ = a11 in QP
7/3
n ç + + ... + 2÷
è ( an + 1) ( an + 2) ( an + n) ø
2 2 Þ2+x=2Þ x=0
a12 in PQ = 2x + 4 and a12 in QP = 2 + 2x
1/3 Þ 2x + 4 = 2 + 2x Þ 4 = 2 not possible
1 n æ rö
lim å ç ÷ \ PQ = QP is not possible for any real x.
n®¥ n è nø
Þ r =1
= 54 \ option (c) is incorrect.
1 n 1 For x = 0,
lim å 2
n®¥ n r =1 æ rö
a + é 1 1 1ù é 2 0 0ù é1 –1/ 2 0 ù
çè ÷
nø ê úê úê ú
ê 0 2 2ú ê 0 4 0ú ê 0 1 / 2 –1 / 3ú
R= ê úê úê ú
ê 0 0 3ú ê 0 0 6ú ê 0 0 1/ 3 úû
1 1/3
ë ûë ûë

Þ
ò0 x dx
= 54 é 2 4 6 ù é 1 –1/ 2 0 ù é 2 1 2 / 3ù
1 1 ê úê ú ê ú
ò0 ( a + x )2 dx < 0 8 12 0 1/ 2 –1/ 3ú < ê0 4 4 / 3ú
ê ú ê
ê úê ú ê ú
ê 0 0 18ú ê 0 0 1/ 3 úû êë 0 0 6 úû
ë ûë

æ 3x 4/3 ö
1
é1 ù é1 ù é 2 1 2 / 3ù é 1ù é 6 ù
3 ê ú ê ú ê úê ú ê ú
ç 4 ÷ Now R a < 6 a Þ ê 0 4 4 / 3ú ê a ú < ê 6a ú
ê ú ê ú
è ø0 4 ê ú ê ú ê úê ú ê ú
Þ = 54 Þ = 54 ê bú ê bú ê 0 0 6 ú ê bú ê6bú
1 1 1 ë û ë û ë ûë û ë û
æ -1 ö -
çè ÷ a a +1
a + xø 0 2
Þ2+a+ b = 6 Þ 3a + 2b = 12
3
3 a ( a + 1)
Þ ´ = 54 Þ a2 + a – 72 = 0 4
4 1 4a + b = 6a Þ 3a – 2b = 0
3
Þ (a + 9) (a – 8) = 0 Þ a = 8 or – 9
Solving a = 2, b = 3 Þ a + b = 5
\ options (a) and (d) are correct.
\ option (d) is correct
JEE Advanced 2019 Solved Paper 2019 - 45

44. (a,c) Let's check each option for the given properties.
é n n ù
f (h) – f (0) ê åk å n Ck k 2 ú
(a) f(x) = x2/3 for PROPERTY 1 lim exists and ê k =0 k =0 ú
h↑ 0
|h| \ det ê ú=0
n n
ê Ck k å n Ck 3k ú
êå
n
is finite. ú
ë k =0 k =0 û
h 2/3 – 0 | h |2/3
Then lim < lim < lim | h |1/6 < 0
h↑ 0 |h| h ↑ 0 | h |1/2 h ↑0 n(n + 1)
n(n – 1)2 n –2 + n ´ 2n –1
2 =0
\ option (a) is correct. Þ
(b) f(x) = sin x for PROPERTY 2 n ´ 2 n –1 4n

sin h – sin 0 sin h 1


lim < lim ≥ Þ n(n + 1) ´ 2
2n–1
– n 2 é( n - 1) 22n -3 + 22n–2 ù = 0
h↑0 h 2 h ↑ 0 h h ë û
when does not exist.
Þ 22n–3 ´ n éë 4 ( n + 1) – n [ n –1 + 2]ùû = 0
\ (b) is incorrect option.
(c) f(x) = |x| for PROPERTY 1 Þ2
2n –3
´ n é 4n + 4 – n 2 – n ù = 0
ë û
|h| –0 Þ n2 – 3n – 4 = 0
lim < lim | h | < 0 Þ n2 – 4n + n – 4 = 0 Þ n = 4
h↑ 0 h ↑0
|h|

\ option (c) is correct


n n
Ck 4 4
Ck 4 4
C 4
C 2 4 C3 4 C4
\å =å = C0 + 1 + + +
(d) f(x) = x |x| for PROPERTY 2 k =0 k + 1 k =0 k + 1 2 3 4 5

h h –0 h 1
lim = lim =1+2+2+1+ = 6.20
5
h ®0 h2 h®0 h
r
r
LHL = –1 and RHL = 1 46. (18) a = 2iˆ + ˆj – k,
ˆ b = ˆi + 2ˆj + kˆ
r r r
h c = aa + bb = ( 2a + b ) ˆi + ( a + 2b ) ˆj + ( – a + b ) kˆ
\ lim does not exist r r
h ®0 h a + b = 3iˆ + 3jˆ
r r
\ option (d) is incorrect. Projection of rc on a + b = 3 2
n
n(n + 1) r r r
45. (6.20) Here åk= (
c. a + b )
k =0 2 Þ r r =3 2
a+b
n n n
n
å n
Ck k 2 = å .n–1 Ck–1 .k 2 = å n.n–1 Ck –1.k 3 ( 2a + b ) + 3 ( a + 2b)
Þ =3 2
k =0 k =1 k k =1 3 2
n Þ a+b = 2 ...........(1)
= nå Ck–1 ( k –1 + 1)
n –1
r
k =1 Using eqn (1) c = ( a + 2 ) iˆ + ( 4 – a ) ˆj + ( 2 – 2a ) kˆ
r r r r r
é n n –1 n–2 n ù (
Now rc is in the plane of a and b Q c = aa + b b )
= nêå Ck–2 (k –1) + å n–1 Ck –1 ú
êë k =2 k –1 r r r
k =1 úû
(
\ a ´ b .c = 0 )
n–2 n–1
r r r r rr
= n(n –1)2 + n´2
( ( ))
Hence c – a ´ b .c = c.c
n n
n
å Ck ´ k = å
( )
n n–1
Ck–1 ´ k = n ´ 2n –1 2 2 2
= ( a + 2 ) + ( 4 – a ) + ( 2 – 2a ) = 6 a – 2a + 4
2
k =0 k =1 k

å
n
n
Ck 3k = 4n (
= 6 ( a – 1) + 3
2
)
k =0 which has minimum value of 18 when a = 1
EBD_7801
2019 - 46 JEE Advanced 2019 Solved Paper

47. (422) Let n(A) = a, n(B) = b, n(A Ç B) = c Say B and E have blue hat.
Then as per question 1 £ b < a Then C and D can have either red and green or green
Also given that A and B are independent events and red i.e. 2 ways.
\ P(A Ç B) = P(A) P(B) Similarly if B & E have green hat, there will be 2 ways
n(A Ç B) n(A) n(B) for C & D.
Þ = × n(S) Hence there are 2 + 2 = 4 ways.
n(S) n(S)
Case II : B and E have different coloured hats blue
c a b and green or green or blue.
Þ = ×
6 6 6
AR
Þ ab = 6c BB
If a = 6 then b = c = 5, 4, 3, 2, 1 (Q b < a)
There is only one way to select all 6 elements of set G
E
A. Number of ways of selecting 5, 4, 3, 2 or 1 elements
in B and A Ç B are C G
6
C5 + 6C4 + 6C3 + 6C2 + 6C1 = 26 – 2 = 62 R
D
6c R or B
If a = 5 then b = , which is not possible because B
5
if c = 5 then b = 6, while b < a. Let B has blue and E has green.
If C has green then D can have red or blue.
6c 3c
If a = 4 then b = = If C has red then D can have only blue.
4 2
\ three ways.
Þ c = 2 and b = 3
Similarly 3 ways will be there when B has green and
2 elements in A Ç B can be selected in 6C2 ways
E has blue.
2 additional elements in A can be selected in 4C2
ways Hence there are 3 + 3 = 6 ways.
1 additional element in B can be selected in 2C1 way Combining the two cases, there will be 4 + 6 = 10
\ No. of ways for a = 4, b = 3, c = 2 are ways.
6
C1 × C1 × 2C1 = 15 × 6 × 2 = 180
4 When similar discussion is repeated with A as blue
If a = 3 then b = 2c Þ c = 1, b = 2 and green hat, we get 10 ways for each.
which can be done in 6C1 × 5C1 + 4C2 = 6 × 5 × 6 = 180 Hence in all, there will be 10 + 10 + 10 = 30 ways.
ways.
é 1 10 æ 7 p kp ö æ 7p (k + 1)p ö ùú
If a = 2 then b = 3c which is not possible 49. (0) sec -1 ê å sec ç + ÷ sec ç + ÷
\ Total number of required ways êë 4 k = 0 è 12 2 ø è 12 2 øú
û
= 62 + 180 + 180 = 422. é ù
48. (30) 5 persons A, B, C, D and E are seated in circular ê 1 10 1 ú
= sec ê å
-1 ú
arrangement. ê 2 k = 0 2cos æ 7 p + kp ö cos æ 7 p + kp + p ö ú
êë ç ÷ ç ÷
è 12 2 ø è 12 2 2 ø úû
A
é ù
B ê 1 10 -1 ú
= sec
-1
ê å ú
ê 2 k = 0 sin æ 7p + kp ö ú
êë ç ÷ú
E è 6 øû

æ 7p ö 7 p -1
C For k = even, sin ç + kp ÷ = sin =
è 6 ø 6 2
D æ 7p ö 7p 1
For k = odd, sin ç + kp ÷ = –sin =
Let A be given red hat, then there will be two cases. è 6 ø 6 2
Case I : B and E have same coloured hat blue/green. So in summation all pairs from 0 to 9 will be cancelled
and term for k = 10 will be left.
JEE Advanced 2019 Solved Paper 2019 - 47

\ we get f '(x) = 0
Þ cos ( p cos x )( – p sin x ) = 0
é æ ù
1 -1 ö÷ ú
sec -1 ê ç = sec–1(1) = 0 Þ cos ( p cos x ) = 0, sin x = 0
ê 2 ç -1 ÷ ú
ë è 2 øû
p
Þ p cos x = ( 2n –1) , x = np
p /2 2
3 cos q
50. (0.50) I = ò dq ...(1) 1
( )
5
0 cos q + sin q Þ cos x = ( 2n –1) , x = p, 2p,3p,.......
2
–1 1
p /2
3 sin q Þ cos x = , .
2 2
ò dq
( )
I= 5 ...(2)
0 sin q + cos q p 2p 4p 5p 7p 8p 10p 11p 13p
Þx= , , , , , , , , ,.....
3 3 3 3 3 3 3 3 3
Adding two values of I, we get:
ì p 2p 4p 5p ü
p /2
\ Y = í , , p, , , 2p,............ý.
1 î 3 3 3 3 þ
I= ò
2 dq
( )
4 \ (II) – Q, T.
3 0 cos q + sin q
g(x) = 0 Þ cos ( 2p sin x ) = 0

2 p /2 sec2 q p 2n –1
I= ò dq Þ 2p sin x = ( 2n –1) Þ sin x =
2 4
3
(1 + )
4
0 tan q
1 –1 3 –3
2 2 Þ sin x = , , ,
(put tan q = t Þ sec q dq = 2t dt) 4 4 4 4
¥
2 + dt ì 3 1 1 3ü
Þ
2I
=ò \ Z = í – sin –1 , – sin –1 ,sin –1 ,sin –1 ý.
3 4 î 4 4 4 4þ
0 (1 + t )
(III) – R.
¥ ¥æ ö g' (x) = 0 Þ – sin ( 2p sin x ) .2p cos x = 0
t + 1 -1 1 1
Þ I = 3ò dt = 3 ò ç - ÷ dt
4 ç 3
(t + 1) 4 ÷ Þ sin ( 2p sin x ) = 0, cos x = 0
0 (t + 1) 0 è (t + 1) ø

¥ Þ 2p sin x = np, x = ( 2n –1) p .


é 1 1 ù é1 1ù 1 2
= 3 ê- 2
+
3
ú = 3ê - ú =
ëê 2(t + 1) 3(t + 1) ûú 0 ë2 3û 6 n
Þ sin x = ,
2
1
Þ I= = 0.50 p 3p 5p 7p
2 x= , , , ,........
2 2 2 2
For Q 51 and 52.
f(x) = 0 Þ sin (p cosx) = 0 –1 1
Þ sin x = –1, , 0, ,1.
Þ p cosx == np 2 2
Þ cosx = n p p 5p 3p 11p 13p
Þ cosx = –1, 0, 1 Þ x = , , , p, , ,2 p, ,.........
6 2 6 2 6 6
p 3p 5p 7p
Þ x = , p, ,2p, ,3p, ,4 p........ ì p p 5p 3p 11p 13p ü
2 2 2 2 \ W = í , , , p, , , 2p, ,......ý.
î6 2 6 2 6 6 þ
ì p 3p 5p 7p ü (IV) – P, R, S.
\ X = í , p, , 2p, ,3p, ,4 p,......ý
î2 2 2 2 þ 51. option (a) is correct.
\ (I) – P, Q 52. option (d) is correct.
EBD_7801
2019 - 48 JEE Advanced 2019 Solved Paper

For Q 53and 54
24
\ XY = 2PX =
5

N ZW is chord of C3.
Z
æ 9ö 24 6
C2(3, 4) C3P = MC3 – MP = 6 – ç 3 + ÷ = 6 – =
è 5 ø 5 5
x
P C3(h, k) 2
æ6ö 6 24 12 6
\ ZP = 62 - ç ÷ = =
è5ø 5 5
C1(0, 0) y
24 6
M \ ZW =
5
W
t Hence, Length of ZW = 24 6 / 5 = 6
Length of XY 24 / 5
C1 : x2 + y2 = 9
\ (II) – (Q)
C2 : (x – 3)2 + (y – 4)2 = 16
1 1 12 6
C3 : (x – h)2 + (y – k)2 = r2 Area of DMZN = MN × ZP = × 12 ×
2 2 5
Centres of C1(0, 0), C2 (3, 4), C3(h, k)
72 6
radii of C1 : 3, C2 : 4, C3 : r =
5
4
Eqn of C1C2 : y = x 1
3 Area of DZMW = × ZW × MP
2
æ 4 ö
C1, C2, C3 are collinear Þ C3 ç h, h ÷ 1 24 6 24 288 6
è 3 ø = × × =
2 5 5 25
MN = MC1 + C1C2 + C2N = 3 + 5 + 4 = 12 Þ r = 6
Area of DMZN 72 6 25 5
\ C1C3 = 6 – 3 = 3 \ = × =
Area of DZMW 5 288 6 4
16 2 81 \ (III) – (R)
Þ h2 + h = 9 Þ h2 =
9 25
Now common tangent of C1 and C3 is S1 – S3 = 0
9 Þ 2hx + 2ky – h2 – k2 = 9 – r2
Þ h = taking h +ve, as lies between C1 & C2
5
18 24 81 144
æ 9 12 ö or x+ y– – = 9 – 36
\ C3 ç , ÷ 5 5 25 25
è5 5 ø Þ 3x + 4y + 15 = 0
18 12 30 It is tangent to x2 = 8ay
\ 2h + k = + = =6 Putting value of y from common tangent in parabola,
5 5 5
we get
\ (I) – (P)
XY is common chord of C1 and C2 æ 3x + 15 ö
x2 = – 8a ç ÷
è 4 ø
\ Eqn of XY : S1 – S2 = 0 Þ 6x + 8y – 18 = 0
Þ x2 + 6ax + 30a = 0
or 3x + 4y – 9 = 0 It should have equal roots
9 10
Length of ^ from C1 to XY = = C1P \ 36a2 – 4 × 30a = 0 Þ a =
5 3
\ (IV) – (U)
81 225 - 81 12 Thus (II) – (Q) is the only correct combination
Also C1X = 3 \ PX = 9- = =
25 25 5 and (IV) – (S) is the only incorrect combination.
53. option (d) is correct
54. option (a) is incorrect
Telegram @unacademyplusdiscounts

Join Us on Telegram for More Such Books

https://telegram.me/unacademyplusdiscounts

Join us from the above link or search ''unacademyplusdiscounts'' in Telegram


JEE ADVANCED 2018
PAPER - 1

PHYSICS 2. Consider a body of mass 1.0 kg at rest at the origin at time


r
t = 0. A force F = (at iˆ + bˆj ) is applied on the body, where
SECTION - I (MAXIMUM MARKS: 24) a = 1.0 Ns–1 and b = 1.0 N. The torque acting on the body
r
• This section contains SIX (06) questions. about the origin at time t = 1.0 s is t . Which of the following
• Each question has FOUR options for correct answer(s). ONE statements is (are) true?
OR MORE THAN ONE of these four option(s) is (are) correct r 1
option(s). (A) | t |= Nm
3
• For each question, choose the correct option(s) to answer r
(B) The torque t is in the direction of the unit vector + k̂
the question.
• Answer to each question will be evaluated according to the r 1
(C) The velocity of the body at t = 1 s is v = (iˆ + 2 ˆj )ms –1
following marking scheme: 2
Full Marks : +4 If only (all) the correct option(s) is (are) 1
chosen. (D) The magnitude of displacement of the body at t = 1s is m
6
Partial Marks : +3 If all the four options are correct but 3. A uniform capillary tube of inner radius r is dipped vertically
ONLY three options are chosen. into a beaker filled with water. The water rises to a height h
Partial Marks : +2 If three or more options are correct but in the capillary tube above the water surface in the beaker.
ONLY two options are chosen, both of which are correct The surface tension of water is s. The angle of contact
options. between water and the wall of the capillary tube is q. Ignore
Partial Marks : +1 If two or more options are correct but the mass of water in the meniscus. Which of the following
ONLY one option is chosen and it is a correct option. statements is (are) true?
Zero Marks : 0 If none of the options is chosen (i.e. the (A) For a given material of the capillary tube, h decreases
question is unanswered). with increase in r
Negative Marks : –2 In all other cases. (B) For a given material of the capillary tube, h is
• For Example: If first, third and fourth are the ONLY three independent of s
correct options for a question with second option being an (C) If this experiment is performed in a lift going up with a
incorrect option; selecting only all the three correct options constant acceleration, then h decreases
will result in +4 marks. Selecting only two of the three correct (D) h is proportional to contact angle q
options (e.g. the first and fourth options), without selecting 4. In the figure below, the switches S1 and S2 are closed
any incorrect option (second option in this case), will result simultaneously at t = 0 and a current starts to flow in the
in +2 marks. Selecting only one of the three correct options circuit. Both the batteries have the same magnitude of the
(either first or third or fourth option), without selecting any electromotive force (emf) and the polarities are as indicated
incorrect option (second option in this case), will result in +1 in the figure. Ignore mutual inductance between the
marks. Selecting any incorrect option(s) (second option in inductors. The current I in the middle wire reaches its
this case), with or without selection of any correct option(s) maximum magnitude Imax at time t = t. Which of the following
will result in –2 marks. statements is (are) true?

1. The potential energy of a particle of mass m at a distance r R L R 2L


from a fixed point O is given by V(r) = kr2/2, where k is a
positive constant of appropriate dimensions. This particle
is moving in a circular orbit of radius R about the point O. If
V I V
v is the speed of the particle and L is the magnitude of its
angular momentum about O, which of the following
statements is (are) true? S1 S2

k k
(A) v = R (B) v = R V V
2m m (A) Imax = (B) Imax =
2R 4R
mk 2 L 2L
(C) L = mk R 2 (D) L = R (C) t = ln 2 (D) t = ln 2
2 R R
EBD_7801
2018 - 2 JEE Advanced 2018 Solved Paper
5. Two infinitely long straight wires lie in the xy-plane along 8. Two men are walking along a horizontal straight line in the
the lines x = ±R. The wire located at x = +R carries a constant same direction. The man in front walks at a speed 1.0 ms–1
current I1 and the wire located at x = –R carries a constant and the man behind walks at a speed 2.0 ms–1. A third man
current I2. A circular loop of radius R is suspended with its is standing at a height 12 m above the same horizontal line
centre at (0,0, 3R ) and in a plane parallel to the xy-plane. such that all three men are in a vertical plane. The two
This loop carries a constant current I in the clockwise walking men are blowing identical whistles which emit a
direction as seen from above the loop. The current in the sound of frequency 1430 Hz. The speed of sound in air is
wire is taken to be positive if it is in the + ĵ direction. 330 ms–1. At the instant, when the moving men are 10 m
Whichr of the following statements regarding the magnetic apart, the stationary man is equidistant from them. The
field B is (are) true?r frequency of beats in Hz, heard by the stationary man at
(A) If I1 = I2, then B cannot be equal to zero at the origin this instant, is __________.
(0, 0, 0) 9. A ring and a disc are initially at rest, side by side, at the top
r
(B) If I1 > 0 and I2 < 0, then B can be equal to zero at the of an inclined plane which makes an angle 60° with the
origin (0, 0, 0) horizontal. They start to roll without slipping at the same
r
(C) If I1 < 0 and I2 > 0, then B can be equal to zero at the instant of time along the shortest path. If the time difference
origin (0, 0, 0) between their reaching the ground is (2 – 3) / 10 s, then
(D) If I1 = I2, then the z-component of the magnetic field at
the height of the top of the inclined plane, in metres, is
æ m0 I ö __________. Take g = 10 ms–2.
the centre of the loop is ç – ÷
è 2R ø 10. A spring-block system is resting on a frictionless floor as
6. One mole of a monatomic ideal gas undergoes a cyclic shown in the figure. The spring constant is 2.0 Nm–1 and
process as shown in the figure (where V is the volume and the mass of the block is 2.0 kg. Ignore the mass of the
T is the temperature). Which of the statements below is spring. Initially the spring is in an unstretched condition.
(are) true?
Another block of mass 1.0 kg moving with a speed of 2.0
T
ms–1 collides elastically with the first block. The collision is
II such that the 2.0 kg block does not hit the wall. The distance,
in metres, between the two blocks when the spring returns
to its unstretched position for the first time after the collision
I III is _________.

IV

V
(A) Process I is an isochoric process –1
2 ms
(B) In process II, gas absorbs heat
1 kg 2 kg
(C) In process IV, gas releases heat
(D) Processes I and III are not isobaric
11. Three identical capacitors C1, C2 and C3 have a capacitance
SECTION - II (MAXIMUM MARKS: 24) of 1.0 mF each and they are uncharged initially. They are
• This section contains EIGHT (08) questions. The answer connected in a circuit as shown in the figure and C1 is then
to each question is a NUMERICAL VALUE. filled completely with a dielectric material of relative
• For each question, enter the correct numerical value (in permittivity er. The cell electromotive force (emf) V0 = 8V.
decimal notation, truncated/rounded-off to the second First the switch S1 is closed while the switch S2 is kept
decimal place; e.g. 6.25, 7.00, –0.33, –.30, 30.27, –127.30) open. When the capacitor C3 is fully charged, S1 is opened
using the mouse and the on-screen virtual numeric keypad and S2 is closed simultaneously. When all the capacitors
in the place designated to enter the answer. reach equilibrium, the charge on C3 is found to be 5 mC.
• Answer to each question will be evaluated according to The value of er = ____________.
the following marking scheme:
Full Marks : +3 If ONLY the correct numerical value is
entered as answer.
V0
Zero Marks : 0 In all other cases. S2

r r r r
7. Two vectors A and B are defined as A = aiˆ and B = a C1 C3

S1
(cos wt iˆ + sin wt ˆj ), where a is a constant and w = p/6 C2
r r r r
rad s–1. If | A + B |= 3 | A – B | at time t = t for the first time,
the value of t, in seconds, is __________.
JEE Advanced 2018 Solved Paper 2018 - 3
12. In the xy-plane, the region y > 0 has a uniform magnetic PARAGRAPH “X”
field B1kˆ and the region y < 0 has another uniform magnetic In electromagnetic theory, the electric and magnetic phenomena
field B kˆ . A positively charged particle is projected from
2
are related to each other. Therefore, the dimensions of electric
the origin along the positive y-axis with speed v0 = p ms–1 at and magnetic quantities must also be related to each other. In the
t = 0, as shown in the figure. Neglect gravity in this problem. questions below, [E] and [B] stand for dimensions of electric and
Let t = T be the time when the particle crosses the x-axis magnetic fields respectively, while [e0] and [m0 ] stand for
from below for the first time. If B2 = 4B1, the average speed dimensions of the permittivity and permeability of free space
of the particle, in ms–1, along the x-axis in the time interval respectively. [L] and [T] are dimensions of length and time
T is __________. respectively. All the quantities are given in SI units.
y
15. The relation between [E] and [B] is
(A) [E] = [B][L][T] (B) [E] = [B] [L]–1 [T]
(C) [E] = [B] [L] [T]–1 (D) [E]=[B] [L]–1 [T]–1
B1
16. The relation between [e0] and [m0] is
–1
v0 = p ms (A) [m0] = [e0] [L]2 [T]–2 (B) [m0] = [e0] [L]–2 [T]2
x
(C) [m0] = [e0]–1 [L]2 [T]–2 (D) [m0] = [e0]–1 [L]–2 [T]2
B2 PARAGRAPH “A”
If the measurement errors in all the independent quantities are
known, then it is possible to determine the error in any dependent
13. Sunlight of intensity 1.3 kW m–2 is incident normally on a quantity. This is done by the use of series expansion and
thin convex lens of focal length 20 cm. Ignore the energy truncating the expansion at the first power of the error. For example,
loss of light due to the lens and assume that the lens aperture consider the relation z = x/y. If the errors in x, y and z are Dx, Dy
size is much smaller than its focal length. The average
and Dz, respectively, then
intensity of light, in kW m–2, at a distance 22 cm from the
lens on the other side is __________. –1
x ± Dx x æ Dx ö æ Dy ö
14. Two conducting cylinders of equal length but different radii z ± Dz = = ç1 ± ÷ ç1 ± ÷ .
are connected in series between two heat baths kept at y ± Dy yè x øè y ø
temperatures T1 = 300 K and T2 = 100 K, as shown in the –1
figure. The radius of the bigger cylinder is twice that of the æ Dy ö
The series expansion for ç 1 ± ÷ , to first power in Dy/y, is
smaller one and the thermal conductivities of the materials è y ø
of the smaller and the larger cylinders are K1 and K2 1 m (Dy/y). The relative errors in independent variables are always
respectively. If the temperature at the junction of the two added. So the error in z will be
cylinders in the steady state is 200 K, then K1 /K2 =
__________. æ Dx Dy ö
Dz = z ç + ÷.
è x y ø
Insulating material
The above derivation makes the assumption that Dx/x = 1,
T1 K1 K2 T2 Dy/y = 1. Therefore, the higher powers of these quantities are
neglected.
L
(1 – a )
L 17. Consider the ratio r = to be determined by
(1 + a )
SECTION - III (MAXIMUM MARKS: 12) measuring a dimensionless quantity a. If the error in the
• This section contains TWO (02) paragraphs. Based on measurement of a is Da (Da/a = 1, then what is the error
each paragraph, there are TWO (02) questions. Dr in determining r?
• Each question has FOUR options. ONLY ONE of these
four options corresponds to the correct answer. Da 2D a 2D a 2a D a
(A) 2 (B) 2 (C) 2 (D)
• For each question, choose the option corresponding to (1 + a ) (1 + a ) (1 – a ) (1 – a 2 )
the correct answer.
• Answer to each question will be evaluated according to 18. In an experiment the initial number of radioactive nuclei is
the following marking scheme: 3000. It is found that 1000 ± 40 nuclei decayed in the first
Full Marks : +3 If ONLY the correct option is chosen. 1.0 s. For |x| = 1, ln(1 + x) = x up to first power in x. The
Zero Marks : 0 If none of the options is chosen (i.e. the error Dl, in the determination of the decay constant l, in
question is unanswered). s–1, is
Negative Marks : –1 In all other cases. (A) 0.04 (B) 0.03 (C) 0.02 (D) 0.01
EBD_7801
2018 - 4 JEE Advanced 2018 Solved Paper

CHEMISTRY 22. In the following reaction sequence, the correct


structure(s) of X is (are)
SECTION - I (MAXIMUM MARKS: 24) Me N3
1) PBr3, Et 2O
• This section contains SIX (06) questions. X ¾¾¾¾¾®
2) NaI, Me 2CO
• Each question has FOUR options for correct answer(s). ONE
3) NaN3, HCONMe2 enantiomerically pure
OR MORE THAN ONE of these four option(s) is (are) correct
option(s).
(A) Me OH (B) Me OH
• For each question, choose the correct option(s) to answer
the question.
• Answer to each question will be evaluated according to the
following marking scheme:
Full Marks : +4 If only (all) the correct option(s) is (are)
chosen. (C) (D)
Partial Marks : +3 If all the four options are correct but Me Me OH
OH
ONLY three options are chosen.
Partial Marks : +2 If three or more options are correct but
ONLY two options are chosen, both of which are correct 23. The reaction (s) leading to the formation of
options. 1,3,5-trimethylbenzene is (are)
Partial Marks : +1 If two or more options are correct but
ONLY one option is chosen and it is a correct option. O Conc. H2SO4
(A) ¾¾¾¾¾®
Zero Marks : 0 If none of the options is chosen (i.e. the D
question is unanswered).
heated iron tube
Negative Marks : –2 In all other cases. (B) Me H ¾¾¾¾¾¾®
873 K
• For Example: If first, third and fourth are the ONLY three
correct options for a question with second option being an O
incorrect option; selecting only all the three correct options
1) Br2 , NaOH
will result in +4 marks. Selecting only two of the three correct +
2) H 3O
options (e.g. the first and fourth options), without selecting ¾¾¾¾¾®
(C) 3) sodalime, D
any incorrect option (second option in this case), will result
in +2 marks. Selecting only one of the three correct options O O
(either first or third or fourth option), without selecting any
incorrect option (second option in this case), will result in +1 CHO
marks. Selecting any incorrect option(s) (second option in
this case), with or without selection of any correct option(s) Zn/Hg, HCl
(D) ¾¾¾¾¾®
will result in –2 marks. OHC CHO
19. The compound(s) which generate(s) N2 gas upon thermal 24. A reversible cyclic process for an ideal gas is shown below.
decomposition below 300 °C is (are) Here, P, V, and T are pressure, volume and temperature,
(A) NH4NO3 (B) (NH4)2Cr2O7 respectively. The thermodynamic parameters q, w, H and U
(C) Ba(N3)2 (D) Mg3N2 are heat, work, enthalpy and internal energy, respectively.
20. The correct statement(s) regarding the binary transition metal
carbonyl compounds is (are) (Atomic numbers: Fe = 26, Ni = 28) A (P1, V1, T1) C (P2, V1, T2)
(A) Total number of valence shell electrons at metal centre
Volume (V)

in Fe(CO)5 or Ni(CO)4 is 16
(B) These are predominantly low spin in nature
(C) Metal–carbon bond strengthens when the oxidation
state of the metal is lowered
(D) The carbonyl C–O bond weakens when the oxidation B (P2, V2, T1)
state of the metal is increased
21. Based on the compounds of group 15 elements, the correct Temperature (T)
statement(s) is (are) The correct option(s) is (are)
(A) Bi2O5 is more basic than N2O5 (A) qAc = DUBC and wAB = P2(V2 – V1)
(B) NF3 is more covalent than BiF3
(B) wBC = P2(V2 – V1) and qBC = DHAC
(C) PH3 boils at lower temperature than NH3
(D) The N–N single bond is stronger than the P–P single (C) DHCA < DUCA and qAC = DUBC
bond (D) qBC = DHAC and DHCA > DUCA
JEE Advanced 2018 Solved Paper 2018 - 5
SECTION - II (MAXIMUM MARKS: 24) separating the two compartments is fixed and is a perfect
• This section contains EIGHT (08) questions. The answer heat insulator (Figure 1). If the old partition is replaced by
to each question is a NUMERICAL VALUE. a new partition which can slide and conduct heat but does
• For each question, enter the correct numerical value (in NOT allow the gas to leak across (Figure 2), the volume
decimal notation, truncated/rounded-off to the second (in m3) of the compartment A after the system attains
decimal place; e.g. 6.25, 7.00, –0.33, –.30, 30.27, –127.30) equilibrium is_______.
using the mouse and the on-screen virtual numeric keypad
in the place designated to enter the answer.
• Answer to each question will be evaluated according to 1 m3, 5 bar,
400 K 3 m 3, 1 bar, 300 K
the following marking scheme: B
A
Full Marks : +3 If ONLY the correct numerical value is
entered as answer.
Figure 1
Zero Marks : 0 In all other cases.

25. Among the species given below, the total number of


diamagnetic species is_______. A B

H atom, NO2 monomer, O2– (superoxide), dimeric sulphur in


vapour phase, Mn 3O 4, (NH4) 2[FeCl 4],(NH4) 2[NiCl 4],
Figure 2
K2MnO4, K2CrO4
30. Liquids A and B form ideal solution over the entire range of
26. The ammonia prepared by treating ammonium sulphate with composition. At temperature T, equimolar binary solution
calcium hydroxide is completely used by NiCl2.6H2O to form of liquids A and B has vapour pressure 45 Torr. At the same
a stable coordination compound. Assume that both the temperature, a new solution of A and B having mole fractions
reactions are 100% complete. If 1584 g of ammonium xA and xB, respectively, has vapour pressure of 22.5 Torr.
sulphate and 952 g of NiCl 2.6H2O are used in the The value of xA/xB in the new solution is________.
preparation, the combined weight (in grams) of gypsum
and the nickel-ammonia coordination compound thus (given that the vapour pressure of pure liquid A is 20 Torr at
produced is________. temperature T)
31. The solubility of a salt of weak acid (AB) at pH 3 is Y×10–3
(Atomic weights in g mol–1: H = 1, N = 14, O = 16, S = 32,
mol L–1. The value of Y is________.
Cl = 35.5, Ca = 40, Ni = 59)
(Given that the value of solubility product of AB
27. Consider an ionic solid MX with NaCl structure. Construct
a new structure (Z) whose unit cell is constructed from the (Ksp) = 2×10–10 and the value of ionization constant of HB
unit cell of MX following the sequential instructions given (Ka) = 1×10–8)
below. Neglect the charge balance. 32. The plot given below shows P — T curves (where P is the
(i) Remove all the anions (X) except the central one pressure and T is the temperature) for two solvents X and
Y and isomolal solutions of NaCl in these solvents. NaCl
(ii) Replace all the face centered cations (M) by anions
completely dissociates in both the solvents.
(X)
(iii) Remove all the corner cations (M) 1 2 34
(iv) Replace the central anion (X) with cation (M)
760
æ number of anions ö÷
Pressure (mmHg)

The value of çç ÷
çè number of cations ø÷÷ in Z is____. 1. solvent X
2. solution of NaCl in solvent X
28. For the electrochemical cell, 3. solvent Y
Mg(s) | Mg2+ (aq, 1 M) || Cu2+ (aq, 1 M) | Cu(s) 4. solution of NaCl in solvent Y

the standard emf of the cell is 2.70 V at 300 K. When the


360
362

367
368

concentration of Mg2+ is changed to x M, the cell potential


changes to 2.67 V at 300 K. The value of x is_______. Temperature (K)

F On addition of equal number of moles of a non-volatile


(given, < 11500K V,1 , where F is the Faraday constant solute S in equal amount (in kg) of these solvents, the
R
and R is the gas constant, ln (10=2.30) elevation of boiling point of solvent X is three times that of
solvent Y. Solute S is known to undergo dimerization in
29. A closed tank has two compartments A and B, both filled these solvents. If the degree of dimerization is 0.7 in solvent
with oxygen (assumed to be ideal gas). The partition Y, the degree of dimerization in solvent X is______.
EBD_7801
2018 - 6 JEE Advanced 2018 Solved Paper
SECTION - III (MAXIMUM MARKS: 12) 1) H2/Pd-C
• This section contains TWO (02) paragraphs. Based on 2) NH3/D 1) H2/Pd-C 1) HCl
each paragraph, there are TWO (02) questions. 3. Br2/NaOH 2) SOCl 2
2) Mg/Et2O
S P ¾¾¾¾¾¾¾¾ ® Q ¾¾¾¾¾¾¾¾ ® R
• Each question has FOUR options. ONLY ONE of these 4) CHCl 3, KOH, D 3) MeMgBr, CdCl2 3) CO2 (dry ice)
four options corresponds to the correct answer. 5) H2/ Pd-C 4) NaBH4 4) H3O
+

• For each question, choose the option corresponding to


the correct answer. 35. The compound R is
• Answer to each question will be evaluated according to
the following marking scheme:
Full Marks : +3 If ONLY the correct option is chosen. (A) CO2H
Zero Marks : 0 If none of the options is chosen (i.e. the
question is unanswered).
Negative Marks : –1 In all other cases. HO2C

PARAGRAPH “X”
(B)
Treatment of benzene with CO/HCl in the presence of anhydrous
AlCl 3/CuCl followed by reaction with Ac2O/NaOAc gives
compound X as the major product. Compound X upon reaction
with Br2/Na2CO3, followed by heating at 473 K with moist KOH
CO2H (D) CO2H
furnishes Y as the major product. Reaction of X with H2/Pd-C, (C)
followed by H3PO4 treatment gives Z as the major product.

33. The compound Y is


36. The compound S is
OH
COBr Br
(A) (B)
HO (A) (B)
O
NH2 HN
Br
COBr
(C) (D) H
Br N
(C) NH2 (D)
34. The compound Z is

MATHEMATICS
(A) (B)
SECTION - I (MAXIMUM MARKS: 24)
O O
• This section contains SIX (06) questions.
• Each question has FOUR options for correct answer(s). ONE
OH OR MORE THAN ONE of these four option(s) is (are) correct
option(s).
(C) (D) • For each question, choose the correct option(s) to answer
the question.
O • Answer to each question will be evaluated according to the
following marking scheme:
PARAGRAPH “A” Full Marks : +4 If only (all) the correct option(s) is (are)
chosen.
An organic acid P (C11H12O2) can easily be oxidized to a dibasic
Partial Marks : +3 If all the four options are correct but
acid which reacts with ethylene glycol to produce a polymer ONLY three options are chosen.
dacron. Upon ozonolysis, P gives an aliphatic ketone as one of Partial Marks : +2 If three or more options are correct but
the products. P undergoes the following reaction sequences to ONLY two options are chosen, both of which are correct
furnish R via Q. The compound P also undergoes another set of options.
reactions to produce S.
JEE Advanced 2018 Solved Paper 7 2018 -
Partial Marks : +1 If two or more options are correct but (D) If P3 is the plane passing through the point (4, 2, – 2)
ONLY one option is chosen and it is a correct option. and perpendicular to the line of intersection of P1 and
Zero Marks : 0 If none of the options is chosen (i.e. the P2, then the distance of the point (2, 1, 1) from the
question is unanswered). 2
Negative Marks : –2 In all other cases. plane P3 is
3
• For Example: If first, third and fourth are the ONLY three
correct options for a question with second option being an 40. For every twice differentiable function f : ¡ ® [ -2, 2] with
incorrect option; selecting only all the three correct options
( f (0))2 + ( f ¢(0)) 2 = 85 , which of the following statement(s)
will result in +4 marks. Selecting only two of the three correct
options (e.g. the first and fourth options), without selecting is (are) TRUE?
any incorrect option (second option in this case), will result (A) There exist r , s Î ¡ , where r < s, such that f is one-one
in +2 marks. Selecting only one of the three correct options on the open interval (r, s)
(either first or third or fourth option), without selecting any (B) There exists x0 Î (-4,0) such that | f ¢( x0 ) | £ 1
incorrect option (second option in this case), will result in +1
marks. Selecting any incorrect option(s) (second option in (C) lim f ( x) = 1
x ®¥
this case), with or without selection of any correct option(s)
will result in –2 marks. (D) There exists a Î (-4, 4) such that f (a ) + f ¢¢( a) = 0
and f ¢(a ) ¹ 0
37. For a non-zero complex number z, let arg(z) denote the
principal argument with -p < arg( z ) £ p . Then, which of 41. Let f : ¡ ® ¡ and g : ¡ ® ¡ be two non-constant
the following statement (s) is (are) FALSE? differentiable functions. If

p f ¢( x ) = (e ( f ( x)- g ( x)) ) g ¢( x ) for all x Î ¡ ,


(A) arg(-1 - i) = , where i = -1
4 and f (1) = g (2) = 1 , then which of the following
(B) The function f : ¡ ® (-p, p] , defined by statement (s) is (are) TRUE ?
f (t ) = arg(-1 + it ) for all t Î ¡ , is continuous at all (A) f (2) < 1 - loge 2 (B) f (2) > 1 - log e 2
points of ¡ , where i = -1 (C) g (1) > 1 - loge 2 (D) g (1) < 1 - log e 2
(C) For any two non-zero complex numbers z1 and z2, 42. Let f :[0, ¥) ® ¡ be a continuous function such that
æz ö x
ò0 e
x -t
arg ç 1 ÷ - arg( z1 ) + arg( z2 ) f ( x) = 1 - 2 x + f (t ) dt
è z2 ø
is an integer multiple of 2p for all x Î[0, ¥) . Then, which of the following statement (s)
(D) For any three given distinct complex numbers z1, z2 is (are) TRUE?
and z3, the locus of the point z satisfying the condition (A) The curve y = f ( x ) passes through the point (1, 2)
æ ( z - z1 )( z2 - z3 ) ö (B) The curve y = f ( x ) passes through the point (2, –1)
arg ç ÷ = p , lies on a straight line
è (z - z3 )(z 2 - z1 ) ø (C) The area of the region
38. In a triangle PQR, let ÐPQR = 30° and the sides PQ and QR
have lengths 10 3 and 10, respectively. Then, which of
{(x, y) Î[0,1]´ ¡ : f ( x) £ y £ }
1 - x 2 is
p-2
4
the following statement(s) is (are) TRUE? (D) The area of the region
(A) ÐQPR = 45°
(B) The area of the triangle PQR is 25 3 and ÐQRP = 120°
{(x, y)}Î[0,1] ´ ¡ : f ( x) £ y £ }
1 - x 2 is
p -1
4
(C) The radius of the incircle of the triangle PQR is
SECTION - II (MAXIMUM MARKS: 24)
10 3 - 15 • This section contains EIGHT (08) questions. The answer
(D) The area of the circumcircle of the triangle PQR is to each question is a NUMERICAL VALUE.
100 p. • For each question, enter the correct numerical value (in
39. Let P1 : 2 x + y - z = 3 and P2 : x + 2 y + z = 2 be two planes. decimal notation, truncated/rounded-off to the second
Then, which of the following statement(s) is (are) TRUE? decimal place; e.g. 6.25, 7.00, –0.33, –.30, 30.27, –127.30)
(A) The line of intersection of P1 and P2 has direction using the mouse and the on-screen virtual numeric keypad
ratios 1, 2, – 1 in the place designated to enter the answer.
• Answer to each question will be evaluated according to
3x - 4 1 - 3 y z the following marking scheme:
(B) The line = =
9 9 3 Full Marks : +3 If ONLY the correct numerical value is
is perpendicular to the line of intersection of P1 and entered as answer.
P2 Zero Marks : 0 In all other cases.
(C) The acute angle between P1 and P2 is 60°.
EBD_7801
2018 - 8 JEE Advanced 2018 Solved Paper
1 1 • For each question, choose the option corresponding to
2 log 2 (log 2 9) log 4 7 the correct answer.
43. The value of ((log 2 9) ) ´ ( 7) • Answer to each question will be evaluated according to
is __________. the following marking scheme:
44. The number of 5 digit numbers which are divisible by 4, with Full Marks : +3 If ONLY the correct option is chosen.
digits from the set {1, 2, 3, 4, 5} and the repetition of digits is Zero Marks : 0 If none of the options is chosen (i.e. the
allowed, is _________. question is unanswered).
45. Let X be the set consisting of the first 2018 terms of the Negative Marks : –1 In all other cases.
arithmetic progression 1, 6, 11, ..., and Y be the set consisting
of the first 2018 terms of the arithmetic progression 9, 16, 23, PARAGRAPH - X
.... . Then, the number of elements in the set X È Y is ____.
Let S be the circle in the xy-plane defined by the equation
46. The number of real solutions of the equation
x2 + y2 = 4.
æ ¥ ¥
æxö ö p
i æ ¥ æ x öi ¥ ö
sin -1 ç
ç å å
xi +1 - x ç ÷ ÷ = - cos -1 ç ç - ÷ -
2 ÷ 2 ç è 2ø å ( i
-x) ÷
÷ å 51. Let E1E2 and F1F2 be the chords of S passing through the
è i =1 i =1 è ø ø è i =1 i =1 ø point P0(1, 1) and parallel to the x-axis and the y-axis,
respectively. Let G1G2 be the chord of S passing through
æ 1 1ö P0 and having slope –1. Let the tangents to S at E1 and E2
lying in the interval ç - , ÷ is ______.
è 2 2ø meet at E3, the tangents to S at F1 and F2 meet at F3, and
(Here, the inverse trigonometric functions sin –1 x and the tangents to S at G1 and G2 meet at G3. Then, the points
p p E3, F3, and G3 lie on the curve
cos–1x assume values in é - , ù and [0, p], respectively.) (A) x + y = 4
êë 2 2 úû
(B) (x – 4)2 + (y – 4)2 = 16
47. For each positive integer n, let
(C) (x – 4)(y – 4) = 4
1 (D) xy = 4
1
yn = (n + 1)(n + 2)...(n + n) n
n 52. Let P be a point on the circle S with both coordinates being
For x Î ¡ , let [x] be the greatest integer less than or equal positive. Let the tangent to S at P intersect the coordinate
to x. If lim yn = L , then the value of [L] is ______. axes at the points M and N. Then, the mid-point of the line
n ®¥ segment MN must lie on the curve
r r r r (A) ( x + y )2 = 3xy
2/3
(B) x + y = 2
2/3 4/3
48. Let a and b be two unit vectors such that a × b = 0 . For
r r r r r r
some x, y Î ¡ , let c = xa + yb + (a ´ b ) . If | c |= 2 and the (C) x 2 + y 2 = 2 xy
2 2 2 2
(D) x + y = x y
r r r
vector c is inclined at the same angle a to both a and b , PARAGRAPH - A
then the value of 8 cos2 a is ________.
49. Let a, b, c be three non-zero real numbers such that the There are five students S1, S2, S3, S4 and S5 in a music class and
for them there are five seats R1, R2, R3, R4 and R5 arranged in a
equation : é p pù
3a cos x + 2b sin x = c, x Î ê - , ú , has two row, where initially the seat Ri is allotted to the student Si, i = 1, 2,
ë 2 2û 3, 4, 5. But, on the examination day, the five students are randomly
p allotted the five seats.
distinct real roots a and b with a + b = . Then, the value
3 53. The probability that, on examination day, the student S1
b gets the previously allotted seat R1, and NONE of the
of is _______.
a remaining students gets the seat previously allotted to him/
50. A farmer F1 has a land in the shape of a triangle with vertices her is
at P(0, 0), Q(1, 1) and R(2, 0). From this land, a neighbouring
farmer F2 takes away the region which lies between the side 3 1 7 1
(A) (B) (C) (D)
40 8 40 5
PQ and a curve of the form y = x n (n > 1) . If the area of the
region taken away by the farmer F2 is exactly 30% of the 54. For i = 1, 2, 3, 4, let Ti denote the event that the students Si
area of DPQR, then the value of n is ______. and Si + 1 do NOT sit adjacent to each other on the day of
the examination. Then, the probability of the event
SECTION - III (MAXIMUM MARKS: 12)
T1 Ç T2 Ç T3 Ç T4 is
• This section contains TWO (02) paragraphs. Based on
each paragraph, there are TWO (02) questions. 1 1 7 1
• Each question has FOUR options. ONLY ONE of these (A) (B) (C) (D)
15 10 60 5
four options corresponds to the correct answer.
JEE Advanced 2018 Solved Paper 2018 - 9

PAPER - 2
PHYSICS (C) The tangential (shear) stress on the floor of the tank
increases with m0
(D) The tangential (shear) stress on the plate varies linearly
SECTION - I (MAXIMUM MARKS: 24) with the viscosity h of the liquid
• This section contains SIX (06) questions. 3. An infinitely long thin non-conducting wire is parallel to
• Each question has FOUR options for correct answer(s). ONE the z-axis and carries a uniform line charge density l. It
OR MORE THAN ONE of these four option(s) is (are) correct pierces a thin non-conducting spherical shell of radius R
option(s). in such a way that the arc PQ subtends an angle 120° at the
• For each question, choose the correct option(s) to answer centre O of the spherical shell, as shown in the figure. The
the question. permittivity of free space is Î0. Which of the following
• Answer to each question will be evaluated according to the statements is (are) true?
following marking scheme: l Z
Full Marks : +4 If only (all) the correct option(s) is (are)
chosen. P
Partial Marks : +3 If all the four options are correct but R
ONLY three options are chosen. o O
Partial Marks : +2 If three or more options are correct but 120
ONLY two options are chosen, both of which are correct
Q
options.
Partial Marks : +1 If two or more options are correct but
ONLY one option is chosen and it is a correct option.
Zero Marks : 0 If none of the options is chosen (i.e. the
(A) The electric flux through the shell is 3Rl /Î0
question is unanswered). (B) The z-component of the electric field is zero at all the
Negative Marks : –2 In all other cases. points on the surface of the shell
• For Example: If first, third and fourth are the ONLY three (C) The electric flux through the shell is 2Rl /Î0
correct options for a question with second option being an
incorrect option; selecting only all the three correct options (D) The electric field is normal to the surface of the shell at
will result in +4 marks. Selecting only two of the three correct all points
options (e.g. the first and fourth options), without selecting 4. A wire is bent in the shape of a right angled triangle and is
any incorrect option (second option in this case), will result placed in front of a concave mirror of focal length f, as
in +2 marks. Selecting only one of the three correct options shown in the figure. Which of the figures shown in the four
(either first or third or fourth option), without selecting any options qualitatively represent(s) the shape of the image
incorrect option (second option in this case), will result in +1 of the bent wire? (These figures are not to scale.)
marks. Selecting any incorrect option(s) (second option in
this case), with or without selection of any correct option(s)
will result in –2 marks. o
45
1. A particle of mass m is initially at rest at the origin. It is f f
subjected to a force and starts moving along the x-axis. Its 2
kinetic energy K changes with time as dK/dt = gt, where g is
a positive constant of appropriate dimensions. Which of
the following statements is (are) true? ¥
(A) The force applied on the particle is constant
(B) The speed of the particle is proportional to time (A) a > 45o (B)
a
(C) The distance of the particle from the origin increases
linearly with time
(D) The force is conservative ¥
2. Consider a thin square plate floating on a viscous liquid in 0< a <45 (D)
o
(C) a
a large tank. The height h of the liquid in the tank is much
less than the width of the tank. The floating plate is pulled 232
5. In a radioactive decay chain, 90Th nucleus decays to
horizontally with a constant velocity m0. Which of the
following statements is (are) true? 212
82 Pb nucleus. Let Na and Nb be the number of a and
(A) The resistive force of liquid on the plate is inversely -
b particles, respectively, emitted in this decay process.
proportional to h Which of the following statements is (are) true?
(B) The resistive force of liquid on the plate is independent (A) Na = 5 (B) Na = 6 (C) Nb = 2 (D) Nb = 4
of the area of the plate
EBD_7801
2018 - 10 JEE Advanced 2018 Solved Paper
6. In an experiment to measure the speed of sound by a 11. A steel wire of diameter 0.5 mm and Young’s modulus
resonating air column, a tuning fork of frequency 500 Hz is 2 × 1011 N m–2 carries a load of mass M. The length of the
used. The length of the air column is varied by changing wire with the load is 1.0 m. A vernier scale with 10 divisions
the level of water in the resonance tube. Two successive is attached to the end of this wire. Next to the steel wire is
resonances are heard at air columns of length 50.7 cm and a reference wire to which a main scale, of least count 1.0
83.9 cm. Which of the following statements is (are) true? mm, is attached. The 10 divisions of the vernier scale
(A) The speed of sound determined from this experiment correspond to 9 divisions of the main scale. Initially, the
is 332 ms–1 zero of vernier scale coincides with the zero of main scale.
If the load on the steel wire is increased by 1.2 kg, the
(B) The end correction in this experiment is 0.9 cm
vernier scale division which coincides with a main scale
(C) The wavelength of the sound wave is 66.4 cm
division is __________. Take g =10 m s–2 and p = 3.2.
(D) The resonance at 50.7 cm corresponds to the 12. One mole of a monatomic ideal gas undergoes an adiabatic
fundamental harmonic expansion in which its volume becomes eight times its initial
value. If the initial temperature of the gas is 100 K and the
SECTION - II (MAXIMUM MARKS: 24)
universal gas constant R = 8.0 J mol –1 K –1, the decrease in
• This section contains EIGHT (08) questions. The answer its internal energy, in Joule, is__________.
to each question is a NUMERICAL VALUE. 13. In a photoelectric experiment a parallel beam of
• For each question, enter the correct numerical value (in monochromatic light with power of 200 W is incident on a
decimal notation, truncated/rounded-off to the second perfectly absorbing cathode of work function 6.25 eV. The
decimal place; e.g. 6.25, 7.00, –0.33, –.30, 30.27, –127.30) frequency of light is just above the threshold frequency so
using the mouse and the on-screen virtual numeric keypad that the photoelectrons are emitted with negligible kinetic
in the place designated to enter the answer. energy. Assume that the photoelectron emission efficiency
• Answer to each question will be evaluated according to is 100%. A potential difference of 500 V is applied between
the following marking scheme: the cathode and the anode. All the emitted electrons are
Full Marks : +3 If ONLY the correct numerical value is incident normally on the anode and are absorbed. The anode
entered as answer. experiences a force F = n × 10–4 N due to the impact of the
Zero Marks : 0 In all other cases. electrons. The value of n is __________. Mass of the
electron me = 9 ×10–31 kg and 1.0 eV = 1.6 ×10–19 J.
7. A solid horizontal surface is covered with a thin layer of oil. 14. Consider a hydrogen-like ionized atom with atomic number
A rectangular block of mass m = 0.4 kg is at rest on this Z with a single electron. In the emission spectrum of this
surface. An impulse of 1.0 Ns is applied to the block at time atom, the photon emitted in the n = 2 to n = 1 transition has
t = 0 so that it starts moving along the x -axis with a velocity energy 74.8 eV higher than the photon emitted in the n = 3
n(t) = n0e–t/t, where n0 is a constant and t = 4s. The to n = 2 transition. The ionization energy of the hydrogen
displacement of the block, in metres, at t = t is __________. atom is 13.6 eV. The value of Z is __________.
Take e –1 = 0.37.
8. A ball is projected from the ground at an angle of 45o with SECTION - III (MAXIMUM MARKS: 12)
the horizontal surface. It reaches a maximum height of 120 m • This section contains FOUR (04) questions.
and returns to the ground. Upon hitting the ground for the • Each question has TWO (02) matching lists: LIST-I and
first time, it loses half of its kinetic energy. Immediately LIST-II.
after the bounce, the velocity of the ball makes an angle of • FOUR options are given representing matching of elements
30o with the horizontal surface. The maximum height it from LIST-I and LIST-II. ONLY ONE of these four options
reaches after the bounce, in metres, is ___________. corresponds to a correct matching.
9. A particle, of mass 10–3 kg and charge 1.0 C, is initially at rest.
• For each question, choose the option corresponding to
At time t = 0,rthe particle comes under the influence of an
the correct matching.
electric field E (t) = E 0 sin w tiˆ, where E0 = 1.0 NC–1 and
• For each question, marks will be awarded according to the
w =103 rad s–1. Consider the effect of only the electrical force
following marking scheme:
on the particle. Then the maximum speed, in ms–1, attained by
Full Marks : +3 If ONLY the option corresponding to the
the particle at subsequent times is ____________.
correct matching chosen.
10. A moving coil galvanometer has 50 turns and each turn has
Zero Marks : 0 If none of the options is chosen (i.e. the
an area 2 ×10–4 m2. The magnetic field produced by the
question is unanswered).
magnet inside the galvanometer is 0.02 T. The torsional
constant of the suspension wire is 10–4 N m rad–1. When a Negative Marks : –1 In all other cases.
current flows through the galvanometer, a full scale 15. The electric field E is measured at a point P (0, 0, d) generated
deflection occurs if the coil rotates by 0.2 rad. The resistance due to various charge distributions and the dependence of E
of the coil of the galvanometer is 50 W. This galvanometer on d is found to be different for different charge distributions.
is to be converted into an ammeter capable of measuring List-I contains different relations between E and d. List-II
current in the range 0 –1.0 A. For this purpose, a shunt
describes different electric charge distributions, along with
resistance is to be added in parallel to the galvanometer.
their locations. Match the functions in List-I with the related
The value of this shunt resistance, in ohms, is __________.
charge distributions in List-II.
JEE Advanced 2018 Solved Paper 2018 - 11
LIST–I LIST–II LIST–I LIST–II
P. E is independent of d 1. A point charge Q at the origin P. In process I 1. Work done by the gas is zero
Q. E µ 1/d 2. A small dipole with point Q. In process II 2. Temperature of the gas
charges Q at (0, 0, l) and – Q remains unchanged
at (0, 0, –l). Take 2l << d R. In process III 3. No heat is exchanged between
R. E µ 1/d2 3. An infinite line charge the gas and its surroundings
coincident with the x-axis, S. In process IV 4. Work done by the gas is 6 P0V0
with uniform linear charge (A) P ® 4; Q ® 3; R ® 1; S ® 2
density l
(B) P ® 1; Q ® 3; R ® 2; S ® 4
S. E µ 1/d3 4. Two infinite wires carrying
uniform linear charge density (C) P ® 3; Q ® 4; R ® 1; S ® 2
parallel to the x - axis. The one (D) P ® 3; Q ® 4; R ® 2; S ® 1
along (y = 0, z = l) has a charge 18. In the List-I below, four different paths of a particle are
density +l and the one along given as functions of time. In these functions, a and b are
(y = 0, z = –l) has a charge positive constants of appropriate dimensions and a ¹ b In
density – lTake 2l << d each case, the force acting on the particle is either zero or
5. Infinite plane charge conservative. In List-II, five physical quantities of the
coincident with the xy-plane r r
with uniform surface charge particle are mentioned p is the linear momentum, L is the
density angular momentum about the origin, K is the kinetic energy,
(A) P ® 5; Q ® 3, 4; R ® 1; S ®2 U is the potential energy and E is the total energy. Match
(B) P ® 5; Q ® 3; R® 1, 4; S ® 2 each path in List-I with those quantities in List-II, which
(C) P® 5; Q ® 3; R ® 1, 2; S ® 4 are conserved for that path.
(D) P ® 4; Q ® 2, 3; R ® 1; S ®5 LIST–I LIST–II
r r
16. A planet of mass M, has two natural satellites with masses P. r(t) = a t iˆ+ b t ˆj 1. p
m1 and m2. The radii of their circular orbits are R1 and R2 r r
respectively, Ignore the gravitational force between the Q. r(t) = a cos wt iˆ+ b sin wt ˆj 2. L
satellites. Define v1, L1, K1 and T1to be, respectively, the r
orbital speed, angular momentum, kinetic energy and time R. r(t) = a(cos wt iˆ+ sin wt ˆj) 3. K
period of revolution of satellite 1; and v2, L2, K2, and T2 to be r b 2
the corresponding quantities of satellite 2. Given m1/m2 = 2 S. r(t) = at iˆ+ t ˆj 4. U
2
and R1/R2 = 1/4, match the ratios in List-I to the numbers in
5. E
List-II.
(A) P ® 1, 2, 3, 4, 5; Q ® 2, 5; R ® 2, 3, 4, 5; S ® 5
LIST–I LIST–II
P. v1 /v 2 1. 1/8 (B) P ® 1, 2, 3, 4, 5; Q ® 3, 5; R ® 2, 3, 4, 5; S ® 2, 5
Q. L1/L2 2. 1 (C) P ® 2, 3, 4; Q ® 5; R ® 1, 2, 4; S ® 2, 5
R. K1/K2 3. 2 (D) P ® 1, 2, 3, 5; Q ® 2, 5; R ® 2, 3, 4, 5; S ® 2, 5
S. T1/T 2 4. 8
(A) P ® 4; Q ® 2; R ® 1; S ® 3 CHEMISTRY
(B) P ® 3; Q ® 2; R ® 4; S ®1
(C) P ® 2; Q ® 3; R ® 1; S ® 4
SECTION - I (MAXIMUM MARKS: 24)
(D) P ® 2; Q ® 3; R ® 4; S ® 1
17. One mole of a monatomic ideal gas undergoes four • This section contains SIX (06) questions.
thermodynamic processes as shown schematically in the • Each question has FOUR options for correct answer(s). ONE
PV-diagram below. Among these four processes, one is OR MORE THAN ONE of these four option(s) is (are) correct
isobaric, one is isochoric, one is isothermal and one is option(s).
adiabatic. Match the processes mentioned in List-I with • For each question, choose the correct option(s) to answer
the corresponding statements in List-II. the question.
• Answer to each question will be evaluated according to the
P following marking scheme:
Full Marks : +4 If only (all) the correct option(s) is (are)
II chosen.
3P0
Partial Marks : +3 If all the four options are correct but
IV III ONLY three options are chosen.
Partial Marks : +2 If three or more options are correct but
I ONLY two options are chosen, both of which are correct
P0
options.
V Partial Marks : +1 If two or more options are correct but
V0 3V0 ONLY one option is chosen and it is a correct option.
EBD_7801
2018 - 12 JEE Advanced 2018 Solved Paper
Zero Marks : 0 If none of the options is chosen (i.e. the 22. The Fischer presentation of D-glucose is given below.
question is unanswered). CHO
Negative Marks : –2 In all other cases. H OH
• For Example: If first, third and fourth are the ONLY three HO H
H OH
correct options for a question with second option being an H OH
incorrect option; selecting only all the three correct options
will result in +4 marks. Selecting only two of the three correct CH2OH
options (e.g. the first and fourth options), without selecting D-glucose
any incorrect option (second option in this case), will result The correct structure(s) of b-L-glucopyranose is (are)
in +2 marks. Selecting only one of the three correct options
H H
(either first or third or fourth option), without selecting any O
incorrect option (second option in this case), will result in +1 HO CH OH OH O
2 HO CH OH OH
marks. Selecting any incorrect option(s) (second option in 2
(A) H H HO H (B)
this case), with or without selection of any correct option(s) H H H H
will result in –2 marks. OH H OH OH
19. The correct option(s) regarding th e complex CH2OH
H
[Co(en)(NH3)3(H2O)]3+ (en = H2NCH2CH2NH2) is (are) O O
H OH HO
(A) It has two geometrical isomers H CH2OH H
(B) It will have three geometrical isomers if bidentate ‘en’ (C) HO OH HO H (D)
H H HO OH
is replaced by two cyanide ligands H H
(C) It is paramagnetic OH H
23 . For a first order reaction A(g) ® 2B(g) + C(g) at constant
(D) It absorbs light at longer wavelength as compared to volume and 300 K, the total pressure at the beginning (t = 0)
[Co(en)(NH3)4]3+ and at time t are P0 and Pt, respectively. Initially, only A is
20. The correct option(s) to distinguish nitrate salts of Mn 2+ present with concentration [A]0, and t1/3 is the time required
and Cu2+ taken separately is (are) for the partial pressure of A to reach 1/3rd of its initial value.
The correct option(s) is (are) (Assume that all these gases
(A) Mn2+ shows the characteristic green colour in the flame behave as ideal gases)
test ­ ­
(B) Only Cu2+ shows the formation of precipitate by
passing H2S in acidic medium
ln(3P0 –Pt)

(C) Only Mn2+ shows the formation of precipitate by

t1/3
passing H2S in faintly basic medium (A) (B)
(D) Cu 2+ /Cu has higher reduction potential than
® ®
Mn2+/Mn (measured under similar conditions) Time [A]0
21. Aniline reacts with mixed acid (conc. HNO3 and conc. H2SO4) ­ ­
at 288 K to give P (51 %),Q (47%) and R (2%). The major
Rate constant

product(s) of the following reaction sequence is (are)


ln(P0 –Pt)

1)Ac2 O,pyridine 1)Sn / HCl


2)Br ,CH CO H 2)Br / H O(excess) (C) (D)
R ¾¾¾¾¾¾¾¾¾¾
2
+
3 2 ® S ¾¾¾¾¾¾¾¾¾¾
2 2 ®
3)H3O 3)NaNO 2 ,HCl / 273- 278 K
4)NaNO2 ,HCl / 273- 278 K 4)H3PO2 ® ®
Time [A]0
5)EtOH, D
major product(s) 24. For a reaction, A ƒ P, the plots of [A] and [P] with time
at temperatures T1 and T2 are given below.
Br Br
­ 10
­
[A]/(mol L–1)

Br
[A]/(mol L–1 )

Br 10
T1
(A) (B) 5 T2
5
Br Br T2
T1 ®
Br Br ®
Time Time

Br If T 2 > T 1 , the correct statement(s) is (are)


Br
(Assume DH° and DS° are independent of temperature and
(C) (D) T2
Br Br ratio of lnK at T 1 to lnK at T2 is greater than . Here H,
Br Br T1
Br S,G and K are enthalpy, entropy, Gibbs energy and
equilibrium constant, respectively.)
JEE Advanced 2018 Solved Paper 2018 - 13
(A) DH° < 0, DS° < 0 (B) DG° < 0, DH° > 0 O
(C) DG° < 0, DS° < 0 (D) DG° < 0, DS° > 0
. NaOBr NH ,D
¾¾¾¾ ® A ¾¾¾¾
3 ® B
SECTION - II (MAXIMUM MARKS: 24) +
H3O (60%) (50%)
• This section contains EIGHT (08) questions. The answer
to each question is a NUMERICAL VALUE. Br /KOH Br (3 equiv)
¾¾¾¾¾
2 ® C ¾¾¾¾¾¾
2 ® D
• For each question, enter the correct numerical value (in (50%) AcOH (100%)
decimal notation, truncated/rounded-off to the second 30. The surface of copper gets tarnished by the formation of
decimal place; e.g. 6.25, 7.00, –0.33, –.30, 30.27, –127.30) copper oxide. N2 gas was passed to prevent the oxide
using the mouse and the on-screen virtual numeric keypad formation during heating of copper at 1250 K. However, the
in the place designated to enter the answer. N2 gas contains 1 mole % of water vapour as impurity. The
• Answer to each question will be evaluated according to water vapour oxidises copper as per the reaction given
the following marking scheme: below: 2Cu(s) + H2O(g) ® Cu2O(s) + H2(g)
Full Marks : +3 If ONLY the correct numerical value is pH is the minimum partial pressure of H2 (in bar) needed to
2
entered as answer. prevent the oxidation at 1250 K. The value of ln(pH )
2
is_____.
Zero Marks : 0 In all other cases.
(Given: total pressure = 1 bar, R (universal gas constant) =
25. The total number of compounds having at least one 8 J K–1 mol–1, ln (10) = 2.3. Cu (s) and Cu2O (s) are mutually
bridging oxo group among the molecules given below immiscible.
is______. N2O3, N2O5, P4O6, P4O7, H4P2O5, H5P3O10, At 1250 K : 2 Cu(s) + ½O2(g) ® Cu2O(s) ;
H2S2O3, H2S2O5 DG° = –78,000 J mol–1
26. Galena (an ore) is partially oxidized by passing air through H2(g) + ½O2(g) ® H2O(g); DG° = –1,78,000J mol–1 ;
it at high temperature. After some time, the passage of air is (G is the Gibbs energy)
stopped, but the heating is continued in a closed furnace 31. Consider the following reversible reaction,
such that the contents undergo self-reduction. The weight A(g) + B(g)®AB(g).
(in kg) of Pb produced per kg of O2 consumed is______ . The activation energy of the backward reaction exceeds
(Atomic weights in g mol–1: O = 16, S = 32, Pb = 207) that of the forward reaction by 2RT (in J mol –1). If the
27. To measure the quantity of MnCl2 dissolved in an aqueous pre-exponential factor of the forward reaction is 4 times
solution, it was completely converted to KMnO4 using the that of the reverse reaction, the absolute value of DG°
reaction, (in J mol–1) for the reaction at 300 K is_____.
MnCl2 + K2S2O8 + H2O® KMnO4 + H2SO4 + HCl (Given; ln(2) = 0.7, RT = 2500 J mol–1 at 300 K and G is the
(equation not balanced). Gibbs energy)
Few drops of concentrated HCl were added to this solution 32. Consider an electrochemical cell:
and gently warmed. Further, oxalic acid (225 mg) was added A(s) | An+ (aq, 2 M) |B2n+ (aq, 1 M) | B(s).
in portions till the colour of the permanganate ion The value of DH° for the cell reaction is twice that of DG° at
disappeared. The quantity of MnCl2 (in mg) present in the 300 K. If the emf of the cell is zero, the DS° (in J K–1 mol–1)
initial solution is ______. of the cell reaction per mole of B formed at 300 K is______.
(Atomic weights in g mol–1: Mn = 55, Cl = 35.5) (Given: ln(2) = 0.7, R (universal gas constant)
28. For the given compound X, the total number of optically = 8.3 J K–1 mol–1. H, S and G are enthalpy, entropy and
active stereoisomers is ______. Gibbs energy, respectively.)

This type of bond indicates SECTION - III (MAXIMUM MARKS: 12)


that the configuration at the • This section contains FOUR (04) questions.
HO HO specific carbon and the • Each question has TWO (02) matching lists: LIST-I and
geometry of the double bond is fixed
LIST-II.
• FOUR options are given representing matching of elements
This type of bond indicates from LIST-I and LIST-II. ONLY ONE of these four options
that the configuration at the
HO HO corresponds to a correct matching.
specific carbon and the geometry
of the double bond is NOT fixed • For each question, choose the option corresponding to
X the correct matching.
• For each question, marks will be awarded according to the
29. In the following reaction sequence, the amount of D (in g) following marking scheme:
Full Marks : +3 If ONLY the option corresponding to the
formed from 10 moles of acetophenone is ______.
correct matching chosen.
(Atomic weights in g mol–1: H = 1, C = 12, N = 14, O = 16, Zero Marks : 0 If none of the options is chosen (i.e. the
Br = 80. The yield (%) corresponding to the product in each question is unanswered).
Negative Marks : –1 In all other cases.
step is given in the parenthesis)
EBD_7801
2018 - 14 JEE Advanced 2018 Solved Paper
33. Match each set of hybrid orbitals from LIST-I with Ph
Br H
complex(es) given in LIST-II
S. Ph OH + AgNO3 4. HCHO, NaOH
List - I List- II
2 Me
P. dsp 1. [FeF6]4– 5. NaOBr
Q. sp 3 2. [Ti(H2O)3Cl3] The correct option is
R. sp d 3 2 3. [Cr(NH3)6]3+
2 3 (A) P-1; Q-2, 3 ; R-1, 4; S-2, 4
S. d sp 4. [FeCl4]2–
5. Ni(CO)4 (B) P-1, 5 ; Q-3, 4 ; R-4, 5; S-3
6. [Ni(CN)4]2– (C) P-1, 5; Q-3, 4 ; R-5; S-2, 4
The correct option is (D) P-1, 5; Q-2, 3 ; R-1, 5; S-2, 3
(A) P-5; Q-4, 6 ; R-2, 3; S-1 35. LIST-I contains reactions and LIST-II contains major
(B) P-5, 6; Q-4; R- 3; S-1, 2 products.
(C) P-6; Q-4, 5 ; R-1; S-2, 3
(D) P-4, 6; Q-5, 6 ; R-1, 2; S-3 List - I List- II
34. The desired product X can be prepared by reacting the
major product of the reactions in LIST-I with one or more P. + ¾® 1.
appropriate reagents in LIST-II. (given, order of migratory ONa Br OH
aptitude: aryl > alkyl > hydrogen)
O Q. + HBr ¾® 2. Br
OMe
Ph
OH R. + NaOMe ¾® 3.
Me Ph Br OMe
X
List - I List- II S. + MeBr ¾® 4.
Ph ONa
HO Me O
5.
P. Ph OH + H2SO4 1. I2, NaOH
Me Match the reaction in LIST-I with one or more products in
Ph LIST-II and choose the correct option.
H2N H
Q. Ph OH + HNO2 2. [Ag(NH3)2]OH (A) P-1, 5; Q-2 ; R-3; S-4
Me
Ph (B) P-1, 4 ; Q-2 ; R-4; S-3
HO Ph
R. (C) P-1, 4; Q-1, 2 ; R-3, 4; S-4
Me OH + H2SO4 3. Fehling solution
Me (D) P-4, 5; Q-4 ; R-4 ; S-3, 4
36 . Dilution processes of different aqueous solutions, with water, are given in LIST-I. The effects of dilution of the solutions on [H +]
are given in LIST-II.
(Note: Degree of dissociation (a) of weak acid and weak base is << 1; degree of hydrolysis of salt <<1; [H+] represents the
concentration of H+ ions)
LIST-I LIST-II
P. (10 mL of 0.1 M NaOH + 20 mL of 0.1 M acetic acid) 1. the value of [H+] does not change on dilution
diluted to 60 mL
Q. (20 mL of 0.1 M NaOH + 20 mL of 0.1 M acetic acid) 2. the value of [H+] changes to half of its initial value on dilution
diluted to 80 mL
R. (20 mL of 0.1 M HCl + 20 mL of 0.1 M ammonia solution) 3. the value of [H+] changes to two times of its initial
diluted to 80 mL value on dilution
1
S. 10 mL saturated solution of Ni(OH)2 in equilibrium 4. the value of [H+] changes to times of its initial value
2
with excess solid Ni(OH)2 is diluted to 20 mL on dilution
(solid Ni(OH)2 is still present after dilution). 5. the value of [H+] changes to 2 times of its initial value on
dilution
JEE Advanced 2018 Solved Paper 2018 - 15
MATHEMATICS such that S1 and S2 touch each other at a point, say, M. Let
E1 be the set representing the locus of M as the pair (S1, S2)
varies in F1. Let the set of all straight line segments joining
SECTION - I (MAXIMUM MARKS: 24) a pair of distinct points of E1 and passing through the point
• This section contains SIX (06) questions. R(1, 1) be F2. Let E2 be the set of the mid-points of the line
• Each question has FOUR options for correct answer(s). ONE segments in the set F2. Then, which of the following
OR MORE THAN ONE of these four option(s) is (are) correct statement(s) is (are) TRUE?
option(s). (A) The point (–2, 7) lies in E1
• For each question, choose the correct option(s) to answer
the question. æ4 7ö
(B) The point ç , ÷ does NOT lie in E2
• Answer to each question will be evaluated according to the è5 5ø
following marking scheme:
æ1 ö
Full Marks : +4 If only (all) the correct option(s) is (are) (C) The point ç ,1÷ lies in E2
chosen. è2 ø
Partial Marks : +3 If all the four options are correct but
æ 3ö
ONLY three options are chosen. (D) The point ç 0, ÷ does NOT lie in E1
Partial Marks : +2 If three or more options are correct but è 2ø
ONLY two options are chosen, both of which are correct
options. éb1 ù
ê ú
Partial Marks : +1 If two or more options are correct but 39. Let S be the set of all column matrices êb2 ú such that b1,
ONLY one option is chosen and it is a correct option. êëb3 úû
Zero Marks : 0 If none of the options is chosen (i.e. the
question is unanswered). b2, b3¸ Î ¡ and the system of equations (in real variables)
Negative Marks : –2 In all other cases. –x + 2y + 5z = b1
• For Example: If first, third and fourth are the ONLY three 2x – 4y + 3z = b2
correct options for a question with second option being an x – 2y + 2z = b3
incorrect option; selecting only all the three correct options has at least one solution. Then, which of the following
will result in +4 marks. Selecting only two of the three correct system(s) (in real variables) has (have) at least one solution
options (e.g. the first and fourth options), without selecting
éb1 ù
any incorrect option (second option in this case), will result ê ú
in +2 marks. Selecting only one of the three correct options for each êb2 ú Î S?
(either first or third or fourth option), without selecting any êëb3 úû
incorrect option (second option in this case), will result in +1
marks. Selecting any incorrect option(s) (second option in (A) x + 2y + 3z = b1, 4y + 5z = b2 and x + 2y + 6z = b3
this case), with or without selection of any correct option(s) (B) x + y + 3z = b1, 5x + 2y + 6z = b2 and –2x – y – 3z = b3
will result in –2 marks. (C) –x + 2y – 5z = b1, 2x – 4y + 10z = b2 and x – 2y + 5z = b3
(D) x + 2y + 5z = b1, 2x + 3z = b2 and x + 4y – 5z = b3
37. For any positive integer n, define fn : (0, ¥) ® ¡ as
40. Consider two straight lines, each of which is tangent to
æ 1 ö
f n (x) = å nj = 1 tan –1 ç
è 1 + ( x + j ) ( x + j -1) ÷ø
for all both the circle x2 + y2 =
1
2
and the parabola y2 = 4x. Let

x Î (0, ¥). these lines intersect at the point Q. Consider the ellipse
whose center is at the origin O (0, 0) and whose semi-major
æ Here, theinverse trigonometricfunction tan -1 x assumes ö axis is OQ. If the length of the minor axis of this ellipse is
ç ÷
ç æ p pö ÷ 2 , then which of the following statement(s) is (are) TRUE?
ç valuesin ç - 2 , 2 ÷ . ÷
è è ø ø 1
(A) For the ellipse, the eccentricity is and the length
Then, which of the following statement(s) is (are) TRUE? 2
of the latus rectum is 1
(A) å 5
j =1 tan2 ( fj (0)) = 55
(B) For the ellipse, the eccentricity is
1
and the length of
2
(B) å 10j =1 (1 + fj¢ (0)) sec2 ( fj (0)) = 10 1
the latus rectum is
1 2
(C) For any fixed positive integer n, lim tan (fn (x)) = (C) The area of the region bounded by the ellipse between
x ®¥ n
1 1
(D) For any fixed positive integer n, lim sec2 (fn (x)) = 1 the lines x = and x = 1 is ( p - 2)
x ®¥ 2 4 2
38. Let T be the line passing through the points P(–2, 7) and (D) The area of the region bounded by the ellipse between
Q(2, –5). Let F1 be the set of all pairs of circles (S1, S2) such 1 1
that T is tangent to S1 at P and tangent to S2 at Q, and also the lines x = and x = 1 is (p – 2)
2 16
EBD_7801
2018 - 16 JEE Advanced 2018 Solved Paper
41. Let s, t, r be non-zero complex numbers and L be the set dy
= (2 + 5y) (5y – 2),
of solutions z = x + iy ( x, y, Î ¡, i = -1) of the equation dx
sz + t z + r = 0, where z = x – iy. Then, which of the then the value of lim f ( x) is _____.
x®-¥
following statement(s) is (are) TRUE? 47. Let f : ¡ ® ¡ be a differentiable function with f (0) = 1 and
(A) If L has exactly one element, then |s| ¹ |t| satisfying the equation
(B) If |s| = |t|, then L has infinitely many elements f (x + y) = f(x) f ¢ (y) + f ¢ (x) f (y) for all x, y Î ¡
(C) The number of elements in L Ç {z : |z – 1 + i| = 5} is at Then, the value of loge (f (4)) is _____.
most 2 48. Let P be a point in the first octant, whose image Q in the
(D) If L has more than one element, then L has infinitely plane x + y = 3 (that is, the line segment PQ is perpendicular
many elements to the plane x + y = 3 and the mid-point of PQ lies in the
42. Let f : (0, p) ® ¡ be a twice differentiable function such that plane
f ( x) sin t - f (t )sin x x + y = 3) lies on the z-axis. Let the distance of P from the
lim = sin 2 x for all x Î (0, p). x-axis be 5. If R is the image of P in the xy-plane, then the
t ®x t-x
length of PR is _____ .
p p
If f æç ö÷ = - , then which of the following statement(s) 49. Consider the cube in the first octant with sides OP, OQ and
6
è ø 12 OR of length 1, along the x-axis, y-axis and z-axis,
is (are) TRUE?
æ1 1 1ö
respectively, where O (0, 0, 0) is the origin. Let S ç , , ÷
p p
(A) f æç ö÷ = è2 2 2ø
è4ø 4 2 be the centre of the cube and T be the vertex of the cube
x4 opposite to the origin O such that S lies on the diagonal OT.
(B) f ( x) < - x 2 for all x Î (0, p) r uur r uuur r uur r uuur
6 If p = SP , q = SQ , r = SR and t = ST , then the value of
(C) There exists a Î (0, p) such that f ¢ (a) = 0 r r r r
| ( p ´ q ) ´ ( r ´ t ) | is _____ .
æpö æpö
(D) f ¢¢ ç ÷ + f ç ÷ = 0 50. Let
è2ø è2ø X = (10C1)2 + 2(10C2)2 + 3(10C3)2 + ... + 10(10C10)2,
SECTION - II (MAXIMUM MARKS: 24) where 10Cr, r Î {1, 2, ..., 10} denote binomial coefficients.
• This section contains EIGHT (08) questions. The answer 1
Then, the value of X is _____ .
to each question is a NUMERICAL VALUE. 1430
• For each question, enter the correct numerical value (in
decimal notation, truncated/rounded-off to the second SECTION - III (MAXIMUM MARKS: 12)
decimal place; e.g. 6.25, 7.00, –0.33, –.30, 30.27, –127.30) • This section contains FOUR (04) questions.
using the mouse and the on-screen virtual numeric keypad • Each question has TWO (02) matching lists: LIST-I and
in the place designated to enter the answer. LIST-II.
• Answer to each question will be evaluated according to • FOUR options are given representing matching of elements
the following marking scheme: from LIST-I and LIST-II. ONLY ONE of these four options
Full Marks : +3 If ONLY the correct numerical value is corresponds to a correct matching.
entered as answer. • For each question, choose the option corresponding to
Zero Marks : 0 In all other cases. the correct matching.
• For each question, marks will be awarded according to the
43. The value of the integral following marking scheme:
Full Marks : +3 If ONLY the option corresponding to the
1 correct matching chosen.
2
1+ 3 Zero Marks : 0 If none of the options is chosen (i.e. the
ò (( x +1)2 (1- x)6 ) 1
4
dx is _____ .
question is unanswered).
0 Negative Marks : –1 In all other cases.
44. Let P be a matrix of order 3 × 3 such that all the entries in P
are from the set {–1, 0, 1}. Then, the maximum possible ì x ü
value of the determinant of P is _____ . 51. Let E1 = í x Î ¡ : x ¹ 1 and > 0ý
î x - 1 þ
45. Let X be a set with exactly 5 elements and Y be a set with
exactly 7 elements. If a is the number of one-one functions ìï -1 æ æ x öö üï
from X to Y and b is the number of onto functions from Y to and E2 = í x Î E1 :sin ç log e ç ÷ ÷ is a real number ý .
îï è è x -1 øø þï
X, then the value of 1 (b – a) is _____ .
5!
(Here, the inverse trigonometric function sin -1 x
46. Let f : ¡ ® ¡ be a differentiable function with f (0) = 0. If
y = f(x) satisfies the differential equation é p p ùö
assumes valuesin ê - , ú ÷ .
ë 2 2 ûø
JEE Advanced 2018 Solved Paper 2018 - 17
Let f : E 1 ® ¡ be the function defined by
x2 y2
æ x ö 53. Let H : - = 1 , where a > b > 0, be a hyperbola in the
f (x) = loge ç ÷ and g : E2 ® ¡ be the function defined a 2
b2
è x -1 ø
xy-plane whose conjugate axis LM subtends an angle of 60°
æ æ x öö at one of its vertices N. Let the area of the triangle LMN be
by g(x) = sin–1 ç log e ç ÷÷ .
è è x -1 ø ø
4 3.
LIST - I LIST - II LIST - I LIST - II
P. The length of the conjugate axis of H is 1. 8
æ 1 ù é e ö
P. The range of f is 1. ç - ¥, úÈê , ¥÷ 4
è 1 - e û ë e - 1 ø Q. The eccentricity of H is 2.
3
Q. The range of g contains 2. (0, 1)
2
é 1 1ù R. The distance between the foci of H is 3.
R. The domain of f contains 3. ê- 2 , 2 ú 3
ë û S. The length of the latus rectum of H is 4. 4
S. The domain of g is 4. (– ¥, 0) È (0, ¥)
The correct option is:
5. æ e ù
(A) P ® 4; Q ® 2; R ® 1; S ® 3
ç -¥, e - 1 ú
è û (B) P ® 4; Q ® 3; R ® 1; S ® 2
æ1 e ù (C) P ® 4; Q ® 1; R ® 3; S ® 2
6. (– ¥, 0) È ç , ú
è 2 e -1û (D) P ® 3; Q ® 4; R ® 2; S ® 1
The correct option is: æ p pö æ p ö
54. Let f1 : ¡ ® ¡, f2 : ç - , ÷ ® ¡, f3 : çè - 1, e - 2÷ø ® ¡
2
(A) P ® 4; Q ® 2; R ® 1; S ® 1 è 2 2ø
(B) P ® 3; Q ® 3; R ® 6; S ® 5
and f4 : ¡ ® ¡ be functions defined by
(C) P ® 4; Q ® 2; R ® 1; S ® 6
(D) P ® 4; Q ® 3; R ® 6; S ® 5 æ 2 ö
(i) f1 (x) = sin ç 1 - e - x ÷ ,
52. In a high school, a committee has to be formed from a group è ø
of 6 boys M1, M2, M3, M4, M5, M6 and 5 girls G1, G2, G3,
G4, G5. ì | sin x |
ï -1 if x ¹ 0
(i) Let a1 be the total number of ways in which the (ii) f 2 (x) = í tan x , where the inverse
committee can be formed such that the committee has ï 1 if x = 0
î
5 members, having exactly 3 boys and 2 girls. trigonometric function tan –1 x assumes values in
(ii) Let a2 be the total number of ways in which the
æ p pö
committee can be formed such that the committee has ç- 2 , 2 ÷,
at least 2 members, and having an equal number of è ø
boys and girls. (iii) f3 (x) = [sin (loge (x + 2))], where, for t Î ¡, [t] denotes
(iii) Let a3 be the total number of ways in which the the greatest integer less than or equal to t,
committee can be formed such that the committee has
5 members, at least 2 of them being girls.
ìï x 2 sin
(iv) f4 (x) = í
( 1x ) if x ¹ 0
.
(iv) Let a4 be the total number of ways in which the ïî 0 if x = 0
committee can be formed such that the committee has LIST - I LIST - II
4 members, having at least 2 girls and such that both P. The function f1 is 1. NOT continuous at x = 0
M1 and G1 are NOT in the committee together. Q. The function f2 is 2. continuous at x = 0 and
LIST - I LIST - II NOT differentiable at x = 0
P. The value of a1 is 1. 136 R. The function f3 is 3. differentiable at x = 0 and
Q. The value of a2 is 2. 189 its derivative is NOT
continuous at x = 0
R. The value of a3 is 3. 192
S. The function f4 is 4. differentiable at x = 0 and
S. The value of a4 is 4. 200 its derivative is
5. 381 continuous at x = 0
6. 461 The correct option is:
The correct option is: (A) P ® 2; Q ® 3; R ® 1; S ®4
(A) P ® 4; Q ® 6; R ® 2; S ® 1
(B) P ® 4; Q ® 1; R ® 2; S ®3
(B) P ® 1; Q ® 4; R ® 2; S ® 3
(C) P ® 4; Q ® 6; R ® 5; S ® 2 (C) P ® 4; Q ® 2; R ® 1; S ®3
(D) P ® 4; Q ® 2; R ® 3; S ® 1 (D) P ® 2; Q ® 1; R ® 4; S ®3
Telegram @unacademyplusdiscounts

Join Us on Telegram for More Such Books

https://telegram.me/unacademyplusdiscounts

Join us from the above link or search ''unacademyplusdiscounts'' in Telegram


EBD_7801
2018 - 18 JEE Advanced 2018 Solved Paper

SOLUTIONS
Paper - 1
Physics
2s cos q
3. (A, C) We know that h =
dv d é kr 2 ù r rg eff
1. (B, C) We know that, F = = ê ú = kr
dr dr êë 2 úû As 'r' increases, h decreases
Q Potential energy, V(r) = Kr2/2 given [all other parameter remaining constant]
For r = R, F = kR Also h µ s
Further if lift is going up with an acceleration 'a'
mv 2 then geff = g + a. As geff increases, 'h' decreases.
Also F = (circular motion)
R Also h is not proportional to 'q' but h µ cos q
4. (B, D) Here I + I2 = I1 \ I = I1 – I2
mv 2 k
\ = kR \ v= ´R
R m é - Rt ù é - Rt ù
Vê V
\ I= 1 - e 2 L ú - ê1 - e L ú
æ k ö Rê ú Rê ú
Angular momentum L = mvR = m çç R ÷ R = kmR 2 ë û ë û
m ÷
è ø
r r
md v é - Rt - Rt ù
2. (A, C) Given F = tiˆ + ˆj \ = t iˆ + ˆj Vê L
- e 2L ú
dt Þ I= e
Rê ú
r ë û
\ dv = t dt iˆ + dt ˆj [Q m = 1]
v r t t R L R 2L
\ ò0 dv = ò0 t dt iˆ + ò0 dt ˆj
r t2
v = iˆ + tjˆ I
V
2
r 1 1
At t = 1s, v = iˆ + ˆj = (iˆ + 2 ˆj )ms -1
2 2 V I1 I2
r
dr t 2 ˆ ˆ dI
Further = i + tj For I to be maximum, =0
dt 2 dt
r t2 é - Rt - Rt ù
\ dr = dt iˆ + t dt ˆj V ê -R L æ - R ö 2L ú
2 \ e -ç ÷ e =0
R êë L è 2L ø úû
r r t t2 t
\ ò0 dr = ò0 2 dt iˆ + ò0 t dt ˆj - Rt
1 æ Rö
\ e 2L = Þ ç ÷ t = ln 2
3 2 2 è 2L ø
r t t
Þ r = iˆ + ˆj
6 2 2L
\ t= ln 2
r 1 1 r 1 1 10 R
At t = 1, r = iˆ + ˆj \ r = + =
6 2 36 4 36 This is the time when I is maximum
r r r æ 1 ˆ 1 ˆö ˆ ˆ é - R æ 2L ö -R æ 2L öù
t = r ´ F = ç i + j ÷ ´ (i + j ) (At t = 1s) V ê L çè R l n 2 ÷ø ç l n 2÷
è6 2 ø 2 L è R øú
Further Imax = êe -e
ú
R
êë úû
iˆ ˆj kˆ
1 1 æ1 1ö 1 V é1 1ù
= 0 = iˆ(0 - 0) - ˆj (0 - 0) + kˆ ç - ÷ = - kˆ Þ I max = ê - ú
6 2 è6 2ø 3 R ë4 2û
1 1 0
V
\ I max =
r 1 4R
\ t = Nm
3
JEE Advanced 2018 Solved Paper 2018 - 19
5. (A, B, D) If I1 = I2, then the magnetic fields due to I1 and I2 7. (2.00)
at origin 'O' will cancel out each other. But the r r r r
A+ B = 3 A- B
magnetic field at 'O' due to the circular loop will be
\ a iˆ + a cos wt iˆ + a sin wt ˆj = 3 a iˆ - a cos wt iˆ - a sin wt ˆj
present. Therefore 'A' is correct.
If I1 > 0 and I2 < 0, then the magnetic field due to both Þ (1 + cos wt )iˆ + sin w t ˆj = 3 (1 - cos w t )iˆ - sin w t ˆj
current will be in + Z direction and add-up. The
magnetic field due to current I will be in – Z direction 2 + 2cos wt = 3 2 - 2cos wt
and if its magnitude is equalr to the combined \ 1 + cos wt = 3(1 - cos wt )
magnitudes of I1 and I2, then B can be zero at the
1 p
origin. Therefore option 'B' is correct. Þ 4cos wt = 2 \ cos wt = or , wt =
2 3
X I1 p p
\ ´t= \ t = 2.00 seconds
6 3
R 8. (5.00) Observer/listener
I2 I
P é v ù 13 m
nA = n ê
ë v - 2cos q úû
12 m
O Z 2 cos q
–R é v ù q 2 ms–1 q 1 ms
–1
nB = n ê
ë v + 1cos q úû
A
Y 5m 5m B q
1 cos q
Beat frequency = n éê ù é ù
v v
If I1 < 0 and I2 > 0 then their net magnetic field at \ ú - nê úû
origin will be in –Z direction and the magnetic field ë v - 2cos q û ë v + cos q
due to I at origin will also be in –Z direction. Therefore é 1 1 ù
r = n vê -
B at origin cannot be zero. Therefore 'C' is incorrect. ë v - 2cos q v + cos q úû
If I1 = I2 then the resultant of the magnetic field BR at é 1 1 ù
P (the centre of the circular loop) is along +X direction. = 1430 ´ 330 ê -
5 5ú
Therefore the magnetic field at P is only due to the ê 330 - 2 ´ 330 + ú
ë 13 13 û
current I which is in –Z direction and is equal to
r m I é 1 1 ù
= 1430 ´ 330 ´ 13 ê -
B = 0 (-kˆ) . ë 330 ´13 - 10 330 ´ 13 + 5 úû
2R
é 1 1 ù
= 1430 ´ 330 ´ 13 ê - » 5 Hz
ë 4280 4295 úû
X
BR 9. (0.75)
The time taken to reach the ground is given by
B2 B1
I1 1 2h æ I ö
t= 1+ C
O P Z sin q g çè MR 2 ÷ø
I
1 2h æ MR 2 ö 4 h
Y I2 For ring t1 = ç1 + ÷ =
sin 60° g è MR 2 ø 3 g

Therefore option 'D' is correct. æ 1 ö


MR 2
6. (B, C, D) In process I, volume is changing. Therefore it is 1 2h ç ÷ 2 3h
For disc t 2 = 1+ 2 2 ÷ =
not isochoric. Therefore 'A' is incorrect. sin 60° g ç MR ÷ 3 g
çè ø
In process II, q = DU + W. DU = 0 as temperature is
constant. Therefore q = W. Here W = P(Vf – Vi) is
positive therefore q is positive i.e., gas absorbs heat. 2- 3
Given t1 - t2 =
Therefore 'B' is correct. 10
For process IV, q = DU + W. Here DU = 0 and W is
4 h 2 3h 2 - 3
negative (volume decreases). Therefore q is negative \ - =
i.e., gas releases heat. 'C' is correct. 3 g 3 g 10
For an isobaric process, V µ T i.e., we will get a
h 3h ( 2 - 3 ) æ 3 ö
straight inclined line in T-V graph. Therefore I and II 2 - = ç ÷
are NOT isobaric. 'D' is correct. 10 10 10 è 2 ø
EBD_7801
2018 - 20 JEE Advanced 2018 Solved Paper

3 But V0 = p
2 h - 3h = 3 - \ Average speed along X-axis = 2.00
2 13. (130.00)
0.232 DAFB and DCFD are similar
h (2 – 1.732) = 1.732 – 1.5 \ h=
0.268 d 2 1
= = A
\ h » 0.75 m D 20 10
C
10. (2.09)
d21 D d
(m - m2 )u1 2m2u2 (1 - 2)2 -2 \ Ratio of area = = F
v1 = 1 + = = ms -1 D 2 100 D
m1 + m2 m1 + m2 1+ 2 3 20 cm
As there is no energy loss B 2 cm
(m - m1 )u2 2m1u1 2 ´ 1 ´ 2 4 -1 \ Average intensity of light at a distance 22 cm
v2 = 2 + = = ms
m1 + m2 m1 + m2 1+ 2 3
1.3 ´ p D 2 4
The time period of mass 2 kg after attaining velocity is = = 1.3 × 100 = 130.00 kWm–2
pd2 4
m 2 14. (4.00)
T = 2p = 2p = 2p
k 2 The intermediate temperature is given by the formula

u1 = 2 ms
–1
u2 = 0 k1 A1T1 k2 A2T2
+
l1 l2
T=
1 kg 2 kg k1 A1 k 2 A2
+
l1 l2
Therefore the time taken to return the original position by Here, T = 200 k, T1 = 300 k, T2 = 100 k
2 kg mass is p sec. l1 = l2 and A1 = pr2, A2 = 4pr2
\ Distance between the two blocks
2 2 22 k1pr 2 ´ 300 + k2 p (4r 2 )100
= ´p = ´ = 2.09 m \ 200 =
3 3 7 k1pr 2 + k2 p (4r 2 )
11. (1.50)
Initially 300k1 + 400k2
\ 200 =
The charge on C3 is q3 = C3V = 1 × 8 µC = 8 µC k1 + 4k 2
\ 200 k1 + 800 k2 = 300 k1 + 400 k2

++ k
8V 1 µF = C3 Þ 400 k2 = 100 k1 \ 1 = 4.00
– – k2
E
Finally 15. (C) We know that, C = where C = speed of light
B
As the charge on C3 is found to be 5 µC therefore charges \ E = CB = LT–1B
on C1 and C2 are 3 µC each 16. (D) We know that
Applying Kirchhoff loop law 3 µC
+ + 1 1
5 3 3 C1 = er µF
– – + +
5 µC
C= \ C2 =
- - =0 m 0e0 m 0e0
1 er 1 – – C3 = 1 µF

\ m0 = e0-1L-2T 2
é 1ù
\ 5 = 3 ê1 + ú
ë er û
+ +
1- a dr (1 + a )(-1) - (1 - a) -2
C2 = 1 µF – –3 µC 17. (B) r= \ = =
1+ a da (1 + a) 2
(1 + a )2
1 5 2
\ = -1= \ e r = 1.5
er 3 3 2 Da
\ Dr =
12. (2.00) (1 + a ) 2

Average speed along X-axis = D1 + D2 = 2( R1 + R2 ) 18. (C) We know that, N = N 0e -lt


t1 + t 2 t1 + t2
Taking log on both sides loge N = loge N0 – lt
differentiating with respect to 'l' we get
é mv0 mv0 ù
ê qB + q (4 B ) ú 1 dN dN 40
= 2ê 1 1 ú = 0-t \ dl = = = 0.02
ê pm + pm ú N dl tN 1 ´ 2000
êë qB1 q (4B1 ) úû [Q N = 3000 – 1000 = 2000]
JEE Advanced 2018 Solved Paper 2018 - 21
Chemistry
19. (B,C) (B) Me H ¾¾¾¾¾®
(A) NH 4 NO 3 ¾¾¾¾¾®
below 300°C
N 2 O + 2H 2 O

N 2O ¾¾¾¾¾¾
® N 2 + O2 O COOH
above 600°C
(C)
D
(B) (NH 4 ) 2 Cr2 O 7 ¾¾ ® N 2 + Cr2 O3 + 4H 2 O ¾ ¾¾ ¾®
D HOOC COOH
(C) Ba(N 3 ) 2 ¾¾¾®
180°C
Ba + 3N 2 O O
(D) Mg3 N 2 ¾¾¾¾¾¾
® 3Mg + N 2 ¾ ¾¾¾ ®
above 700°C
Hence only (NH4)2Cr2O7 and Ba(N3)2 can provide N2
gas on heating below 300°C
CHO
20. (B, C) (D)
(A) [(Fe(CO5)] & [Ni(CO)4] complexes have 18-electrons in
¾¾¾¾¾®
their valence shell.
OHC CHO
(B) Due to strong ligand field, carbonyl complexes are
predominantly low spin complexes.
24. (B, C)
(C) As electron density increases on metals (with lowering
oxidation state on metals), the extent of synergic A– C Þ isochoric process
bonding increases. Hence M-C bond strength increases A– B Þ isothermal process
(D) While positive charge on metals increases and the ex-
B– C Þ isobaric process
tent of synergic bond decreases and hence C–O bond
becomes stronger. Þ qAC = DUAC = nCV,m (T2 – T1) = D UBC
21. (A, B, C)
(A) Basic character of oxides increase on moving down æV ö
the group therefore Bi2O5 is more basic than N2O5. Þ WAB = –nRT1 ln ç 2 ÷
(B) Covalent nature depends on electronegativity è V1 ø
difference between bonded atoms. In NF3 , N and F are Þ WBC = –P2(V1–V2) = P2(V2–V1)
non-metals but in BiF3, Bi is metal while F is non metal Þ qBC = DHBC = nCP,m (T2–T1) = DHAC
therefore NF3 is more covalent than BiF3.
(C) In PH3 hydrogen bonding is absent but in NH3 Þ DHCA = nCP,m(T1–T2)
hydrogen bonding is present, therefore PH3 boils at Þ DUCA = nCV,m(T1–T2)
lower temperature than NH3.
DHCA < DUCA since both are negative (T1 < T2)
(D) Due to small size in N–N single bond l.p. – l.p.
repulsion is more than P–P single bond therefore N–N
single bond is weaker than the P–P single bond. 25. (1)
22. (B) • H-atom =1s1 Paramagnetic
Me OH Me Br
PBr3
¾¾¾¾®
Et 2O(S N2)
• NO 2= || N odd electron species Paramagnetic
Me O O
I
NaI
¾¾¾® • O2– (superoxide) = One unpaired electron
acetone
(S N2) in p* M.O. Paramagnetic
NaN3(DMF) • S2 (in vapour phase) = same as O2, two
(S N2)
unpaired e–s are present in p* M.O. Paramagnetic
Me N3 +2 +4
• Mn3O4 = 2MnO . MnO 2 Paramagnetic

23. (A, B, D) • (NH4)2[FeCl4] = Fe2+ = 3d 6 4s0 Paramagnetic

O • (NH4)2 [NiCl4] = Ni = 3d 8 4s2


(A) ¾¾¾¾® Ni2+ = 3d 8 4s0 Paramagnetic
D
EBD_7801
2018 - 22 JEE Advanced 2018 Solved Paper
29. (2.22) P1 = 5 bar P2 = 1 bar
é O– ù
| V1 = 1 m3 V2 = 3 m3
ê ú
Mn T1 = 400 K T2 = 300 K
• K2MnO4 = 2K+ êO ú
|| O– , Mn6+ = [Ar] 3d1
êë úû

|
O 5 3
n1 = n2 =
400R 300R
Paramagnetic Let volume be (V + x) V = (3 – x)

é O ù PA PB
|| =
ê ú TA TB
Cr
• K2CrO4 = 2K ê
+
| – –ú , Cr = [Ar] 3d
6+ 0
êëO O O úû
|

n b1 ´ R n b2 ´R
Þ =
Vb1 Vb2
Diamagnetic
5 3
26. (2992) Þ 400 ∋ 4´ x ( = 300R ∋3 , x (

(NH 4 ) 2 SO 4 + Ca(OH) 2 ® CaSO 4 .2H 2 O + 2NH 3 Þ 5(3 – x) = 4 + 4x


1584 g gypsum(M =172) 24 mole
=12 mole 12 mole 11
Þx=
9
NiCl2 .6H 2O + 6NH 3 ® [Ni(NH3 ) 6 ]Cl 2 + 6H 2 O
952 g = 4 mole 24 mole (M = 232) 11 æç 20 ö÷
4 mole V= 1+x =1 + = çèç ø÷÷ = 2.22
9 9
Total mass = 12 × 172 + 4 × 232 = 2992 g
30. (19) PT = p°AXA + p°BXB
27. (3) As per given information, cations form fcc lattice and
anions occupy all the octahedral voids. 45 = 20(0.5) + poB (0.5)
So M+ X– Formula MX p oB = 70
4 ions 4 ions 22.5 = 20 XA + 70(1 – XA)
After step I 4 ions 1 ion 50XA = 47.5
After step II 1 ion 4 ions
4.75
After step III 0 ion 4 ions XA = = 0.95
5
After step IV 1 ion 3 ions
XB = 0.05
No.of anions 3
So ratio of = XA
No.of cations 1
X B = 19

28. (10) ® Mg 2 + (aq) + 2e -


Mg (s) ¾¾ æ é H∗ ù ö÷
çç ê úû ÷÷ æ10,3 ö÷
Cu 2+ (aq) + 2e - ¾¾
® Cu (s) Ksp çç ë ∗ 1÷÷ = ç
20´10,10 çç ∗ 1÷÷÷
31. (4.47) S = çç K a ÷÷ çè10,8
çè ÷ø ø÷
Mg (s) + Cu 2+ (aq) ¾¾
® Mg 2+ (aq) + Cu (s)
; 2´10,5 = 4.47 × 10 M
–3
RT
E cell = E°cell - lnx 32. (0.05) From graph
nF
For solvent ‘X’ DTb(x) = 362 – 360 = 2
RT x DTb(x) = mNaCl × Kb(x)
E = 2.67 = 2.7 - ln
nF 1 ...(1)
For solvent ‘Y’ DTb(y) = 368 – 367 = 1
300
0.03 = lnx DTb(y) = mNaCl × Kb(y)
2 ´ 11500
...(2)
2.3 = lnx
Dividing equation (1) by (2)
x = 10
JEE Advanced 2018 Solved Paper 2018 - 23

K b∋x ( æ a ö
DTb(y)(s) = ç1 - 2 ÷ K b(y)
Þ K =2 è 2 ø
b∋ y(

For solute S Given DTb(x)(s) = 3DTb(y)(s)


Given solute S dimerizes in solvent. Hence,
æ a1 ö æ a2 ö
2(S) ® S2 ç1 - ÷ K b(x) = 3 ´ ç 1 - ÷ ´ K (b)(y)
è 2 ø è 2 ø
1
(1-a ) a2
æ a ö æ a ö
2 ç1 - 1 ÷ = 3ç1 - 2 ÷
è 2 ø è 2 ø
i = (1 - a 2)
a2 = 0.7
æ a ö
DTb(x)(s) = ç1 - 1 ÷ K b(x) so a1 = 0.05
è 2 ø

CHO CH CH COOH

CO + HCl Ac2O
¾¾¾¾®
33. (C) AlCl /CuCl
3 AcONa
(X)

Br2/Na2CO3
Br Br

C CH CH CH
moist KOH
473 K COONa
(Y)

CHO CH CH COOH

34. (A) CO + HCl Ac2O H2/Pd


¾¾¾¾®
AlCl /CuCl
3 AcONa
HOCO
(X)
H3PO4

(Z)
O

35. (A) 36. (B)

Sol. 35 & 36
COOH
OH
[O] OH
C11H12O2 ¾¾® ¾ ¾¾
+
® dacron
(P) H

COOH
EBD_7801
2018 - 24 JEE Advanced 2018 Solved Paper

COOH

¾
¾
¾H2/Pd-C
¾
H2/Pd-C ¾
®
COOH COOH COC1
SOCl2
¾¾¾®

NH3/D
MeMgCl/CdCl2
CONH2
O
C Me
Br2 ,NaOH
NH2
NaBH4

D CHCl3/KOH OH
NC NH – CH3
H
¾¾®
2
(Q)
Pd-C

(S) HCl

COOH 1.CO2 MgCl Mg/Et2O Cl


+
2.H3O
(R)

Mathematics ( z - z1 )( z2 - z3 )
= k, kÎR
Þ
37. (A, B, D) ( z - z3 )( z2 - z1 )

-3p
(A) arg(-1 - i) = æ z - z1 ö æ z2 - z1 ö
4 Þ ç ÷ = kç ÷
è z - z3ø è z2 - z3 ø
\ (A) is false
Þ z, z1, z2, z3 are concyclic. i.e. z lies on a circle.
é p - tan -1 (t ), t ³ 0 \ (D) is false.
(B) f (t ) = arg(-1 + it ) = ê
êë -p + tan -1 (t ), t < 0 38. (B, C, D)
P
PQ 2 + QR 2 - PR 2 10 3
lim f (t ) = -p and lim f (t ) = p (A) cos 30° =
t ®0- t ®0+ 2PQ × QR 30°
LHL ¹ RHL Þ f is discontinuous at t = 0 Q 10 R
\ (B) is false. 3 (10 3) 2 + 10 2 - PR 2
Þ =
2 2 ´ 10 3 ´10
æz ö
(C) arg ç 1 ÷ - arg z1 + arg z2 Þ PR2 = 100 or PR = 10
è z2 ø \ Ð P = Ð Q = 30°
\ (A) is false.
= 2np + arg z1 - arg z2 - arg z1 + arg z2
= 2np, multiple of 2p 1
(B) Area of DPQR = PQ ´ QR ´ sin30°
\ (C) is true. 2

æ ( z - z1 )( z2 - z3 ) ö 1 1
= ´ 10 3 ´ 10 ´ = 25 3
(D) arg ç ÷=p 2 2
è ( z - z3 )( z2 - z1 ) ø
Also Ð R = 180° – 30° – 30° = 120°
\ (B) is true.
JEE Advanced 2018 Solved Paper 2018 - 25

D 25 3 25 3 5 3 f (0) - f ( -4)
(C) r= = = = f ¢ ( x0 ) =
s æ 10 3 + 10 + 10 ö 10 + 5 3 2 + 3 0 - (-4)
çç 2 ÷÷
è ø f (0) - f (-4)
Þ | f ¢( x0 ) | =
4
= 5 3(2 - 3) = 10 3 - 15
\ (C) is true. Q -2 £ f ( x ) £ 2

abc 10 3 ´ 10 ´10 \ -4 £ f (0) - f (-4) £ 4


(D) R= = = 10
4D 4 ´ 25 3 Þ | f ¢( x0 ) | £ 1
\ (B) is true.
\ Area of circumcircle = pR 2 = 100 p
\ (D) is true. (C) If we consider f ( x) = sin( 85 x) then f (x) satisfies
39. (C,D)
(A) Direction ratios of line of intersection of two planes the given condition [f (0)]2 + [f ¢(0)]2 = 1
r r
will be given by n1 ´ n2 . But lim (sin 85x) does not exist
x ®¥
$i $j k$ \ (C) is false.
r r
n1 ´ n2 = 2 1 -1 = 3$i - 3 $j + 3k$ (D) Let us consider g(x) = [f(x)]2 + [f ¢ (x)]2
1 2 1
By Lagrange’s Mean Value theorem
\ dr’s of line of intersection of P1 and P2 are 1, – 1, 1 | f ¢(x) | £ 1
\ (A) is false.
Also | f(x1 ) |£ 2 as f ( x) Î[ -2, 2]
(B) Given line can be written as
\ g( x1 ) £ 5, for same x1 Î (-4, 0)
4 1
x- y-
3= 3=z Similarly g(x 2 ) £ 5, for same x2 Î (0, 4)
3 -3 3 Also g(0) = 85
Clearly this line is parallel to line of intersection of P1 Hence g(x) has maxima in (x1, x2) say at a such that
and P2 g¢(a) = 0 and g(a) ³ 85
\ (B) is false.
(C) If q is the angle between P1 and P2 then g ¢( a ) = 0 Þ 2f (a )f ¢ (a ) + 2f ¢ (a )f ¢¢ (a ) = 0

2 ´ 1 + 1 ´ 2 + ( -1) ´ 1 3 1 Þ 2f ¢(a)[f(a ) + f ¢¢(a)] = 0


cos q = = =
6 6 6 2
If f ¢(a) = 0 Þ g(a) = [f (a)]2
\ q = 60°
2
Hence (C) is true. and [f (a )] £ 4
(D) Equation of plane P3 :
1(x – 4) – 1(y – 2) + 1(z + 2) = 0 \ g( a) ³ 85 (is not possible.)
or x – y + z = 0 Hence f (a ) + f ¢¢ ( a ) = 0 for a Î (x1, x 2 ) Î (-4, 4)
2 - 1+ 1 2 \ (D) is true.
Distance of (2, 1, 1) from P3 = =
1+ 1 + 1 3 41. (B, C)
\ (D) is true. Given f ¢(x) = e(f(x) -g(x)) × g¢(x)
40. (A, B, D)
-f (x)
(A) f(x) being twice differentiable, it is continuous but can’t Þ e f ¢(x) = e -g(x) g ¢(x)
be constant throughout the domain.
Integrating both sides, we get
\ We can find x Î (r , s ) such that f(x) is one one.
-e -f (x) = -e - g(x) + c
Hence (A) is true.
(B) By Lagrange’s Mean Value theorem for f(x) in [–4, 0], Þ - e - f (x) + e - g(x) = c
there exists
Þ -e -f (1) + e- g(1) = -e -f (2) + e -g(2)
x0 Î (-4,0) such that
But given that f (1) = g(2) = 1
EBD_7801
2018 - 26 JEE Advanced 2018 Solved Paper

\ -e -1 + e -g(1) = -e -f (2) + e -1 1 1
2 log 2 (log 2 9) log 4 7
43. (8) ((log 2 9) ) ´ ( 7)
2
Þ e -f (2) + e -g(1) =
e 1
´ log 7 4
= ( log 2 9 )2 ´ log (log 2 9) 2 ´ 7 2
2 2
Þ e -f (2) < and e -g(1) <
e e = ( log 9 )log(log 2 9) 4 ´ 7log7 2
2
Þ -f (2) < ln 2 - 1 and -g(1) < ln 2 - 1 = 4 ´ 2 = 8.
Þ f (2) > 1 - ln 2 and g(1) > 1 - ln 2 44. (625) The last 2 digits, in 5-digit number divisible by 4, can
be 12, 24, 32, 44 or 52.
\ (B) and (C) are True.
42. (B, C) Also each of the first three digits can be any of
{1, 2, 3, 4, 5}
x
f (x) = 1 - 2x + ò e x - t f (t) dt Hence 5 options for each of the first three digits and
0
total 5 options for last 2-digits
x \ Required number of 5 digit numbers are
Þ f (x) = 1 - 2x + e x ò e - t f (t)dt
0 = 5 × 5 × 5 × 5 = 625
45. (3748)
x
Þ f ¢(x) = -2 + e x ò e- t f (t) dt + e x [e - x f (x)] The given sequences upto 2018 terms are
0
1, 6, 11, 16, ........., 10086
Þ f ¢(x) = -2 + [f(x) - 1 + 2x] + f(x) and 9, 16, 23, ........., 14128
Þ f ¢(x) - 2f(x) = 2x - 3 The common terms are
Its a linear differential equation. 16, 15, 86, ........... upto n terms, where Tn £ 10086

IF = e ò
-2dx
= e -2x Þ 16 + (n – 1) 35 £ 10086
Þ 35n – 19 £ 10086
Solution: f (x) ´ e -2x = ò e-2x (2x - 3) dx
10105
Þ n£ = 288.7
35
e -2x e -2x
f (x) ´ e -2x = (2x - 3) - ò ´ 2 dx \ n = 288
-2 -2
\ n(XÈ Y) = n(X) + n(Y) - n(X Ç Y)
-2x -2x = 2018 + 2018 – 288 = 3748
e e
e -2x f (x) = (2x - 3) + +c
-2 -2
æ x ö æ x ö
-
3 1 -1 çx2 2 ÷ -1 ç 2 -x ÷
f (x) = - x + + + ce2x 46. (2) sin ç -x×

= sin ç - ÷
2 -2 ç1- x 1- ÷ ç 1+ x 1+ x ÷
è 2ø è 2 ø
f (x) = - x + 1 + ce 2x
From definition of function, f(0) = 1 x2 x2 -x x
Þ - = +
\1=1+c Þ c=0 1- x 2 - x 2 + x 1+ x
\ f(x) = 1 – x
Clearly curve y = 1 – x, does not pass through (1, 2) but it x2 x x x2
Þ - + - =0
passes through (2, – 1) 1- x 1+ x 2 + x 2 - x
\ (A) is false and (B) is true
Also the area of the region B x(x + x 2 - 1 + x) x(2 - x - 2 x - x 2 )
Þ 2
+ =0
1- x 4 - x2
1 - x £ y £ 1 - x 2 , is shown in
the figure, is given by x(x 2 + 2 x - 1) x(2 - 3x - x 2 )
O A Þ + =0
= Area of quadrant – Area DOAB 1- x2 4 - x2
1 1 p-2 2 2 2 2
= ´ p´12 - ´ 1´ 1 = Þ x[(x + 2 x - 1)(4 - x ) + (1 - x )(2 - 3x - x )] = 0
4 2 4
\ (C) is true and (D) is false.
JEE Advanced 2018 Solved Paper 2018 - 27
3 2
Þ x[x + 2x + 5x - 2] = 0 \ 3 a cos a + 2b sin a = c ... (1)
Þ x = 0 or x3 + 2x2 + 5x – 2 = 0 = p(x) (say) and 3 a cos b + 2b sin b = c ... (2)
æ1ö subtracting equation (2) from (1) we get
We observe p(0) < 0 and p ç ÷ > 0
è2ø 3 a (cos a – cos b) + 2b (sin a – sin b) = 0
a +b a -b a+b a -b
\ One root of p(x) = 0 lies in æç 0, ö÷ .
1 - 3 a2sin + 2b.2cos =0
Þ sin sin
è 2ø 2 2 2 2

p p
1 1 Þ -2 3 a sin + 4b cos = 0
Thus two solutions lie between , and . 6 6
2 2
1/n 1 3
æ n +1 n + 2 n + 3 n+nö Þ -2 3 a ´ + 4b =0
47. (1) yn = ç . . ..... ÷ 2 2
è n n n n ø
b 1
n Þ = = 0.5
1 æ rö a 2
Þ log yn =
n å log ç1 + ÷
è nø
r =0
50. (4) y
n
n y=x
1 æ rö
Þ
( )
lim y n = lim
n ®¥ n ®¥ n
å
r=0
log ç1 + ÷
è nø Q(1,1)

1 1 x
ò0 log(1 + x)dx = [ x log(1 + x) ]0 - ò0 1 + x dx
1
Þ log L =

P(0,0) R(2,0) x
= log 2 - [ x - log |1 + x |]0
1

= log 2 – 1 + log 2 = 2 log 2 – 1


30
Shaded area = ´ Ar( DPQR)
æ4ö 100
= log 4 – log e = log ç ÷
èeø
1 3 1
ò0 (x - x
n
Þ )dx = ´ ´ 2 ´1
4 é4ù 10 2
\ L= Þ [ L] = ê ú = 1
e ëeû 1
r r rr r æ x 2 x n +1 ö 3
48. (3) Given | a | = | b | = 1, a.b = 0, | c | = 2 Þ çç - ÷÷ =
r r r è 2 n + 1 ø0 10
c makes angle a with both a and b
r r r r r 1 1 3
Also, c = x a + y b + a ´ b Þ - =
2 n + 1 10
rr
c.a = 2 cos a Þ x = 2 cos a
1 1 3 1
rr Þ = - = Þ n=4
c.b = 2 cos a Þ y = 2 cos a n + 1 2 10 5
r2 r r r r r r2
c = c.c = (2cos a)a + (2cos a) b + a ´ b y
51. (A) (0,2) G G3
r r
2

Þ (2)2 =4 cos2 a + 4 cos2 a + | a ´ b |2 F1 E1


E2
r
(Q | ar ´ b | = 1 ´ 1 ´ sin 90° = 1)
P(
1,
1)
Þ 4 = 8 cos2 a + 1 x
1 x
O G1(2,0)
Þ 8 cos2 a =
3
49. (0.5) Given that the equation
F2
3 a cos x + 2b sin x = c
y1
p
has two roots a and b, such that a + b =
3
EBD_7801
2018 - 28 JEE Advanced 2018 Solved Paper
Equation of E1 E2 : y = 1 53. (A) No. of dearrangements for 4 students
Equation of F1 F2 : x = 1
æ 1 1 1 1ö
Equation of G1 G2 : x + y = 2 = 4!ç1 - + - + ÷
è 1! 2! 3! 4! ø
By symmetry, tangents at E1 and E2 will meet
= 12 – 4 + 1 = 9
on y-axis and tangents at F1 and F2 will meet
Total no. of arrangements of seating of 5 students
on x-axis
= 5! = 120
E1 º ( )
3, 1 & F1 º 1, 3 ( ) 9 3
\ required probability = =
120 40
Equation of tangent at E1 : 3x + y = 4
54. (C) Total cases = 5! = 120
Equation of tangent at F1 : x + 3y = 4 Favourable cases :
\ Points E3 (0, 4) and F3(4, 0) 1, 3, 5, 2, 4 ü
ý2
Tangents at G1 and G2 are x = 2 and y = 2 1, 4, 2, 5, 3þ
intersecting each other at G3(2, 2). 2, 4, 1, 3, 5ü
Clearly E3, F3 and G3 lie on the curve x + y = 4. ï
2, 5, 3, 1, 4 ý 3
52 (D) Let point P be (2 cos q, 2 sin q)
2, 4, 1, 5, 3ïþ
Tangent at P : x cos q + y sin q = 2
3, 1, 4, 2, 5ü
æ 2 ö ï
\ Mç ,0 ÷ . and N æ 0, 2 ö 3, 5, 2, 4, 1ý 3
è cos q ø ç ÷
è sin q ø 3, 1, 5, 2, 4 ïþ
æ 1 1 ö 4, 2, 5, 1, 3ü
Mid point of MN = ç , ÷ ï
è cos q sin qø 4, 2, 5, 3, 1ý 3
For locus of mid point (x, y) of MN, 4, 1, 3, 5, 2 ïþ
1 1 5, 2, 4, 1, 3 ü
x= , y= ý2
cos q sin q 5, 3, 1, 4, 2 þ
\ favourable cases = 14
1 1 14 7
Þ 2
+ 2
=1 \ required probability = =
x y 120 60
Þ x2 + y2 =x2y2
JEE Advanced 2018 Solved Paper 2018 - 29

Paper - 2
Physics
dk d æ1 ö A' ¥
1 mV 2 ÷ = gt
= gt and k = mV 2 \
dt çè 2
1. (A, B, D) A
dt 2 ø

m dV dV B' P Bf/2 f C
Þ ´ 2V = gt \ mV = gt
2 dt dt

V t
m ò VdV = g ò tdt mV 2 gt 2
\ Þ = f
0 0 2 2 The object distance decreases from to f. Therefore the
2
g final result is (D).
\V= ´ t. i.e., V µ t
m
232 – 212 20
As V is proportional to 't', distance cannot be proportional 5. (A, C) No. of a-particles = = =5
4 4
to 't'.
232 212 4 0
\ Th Pb + 5 He + 2 b
dV dé g ù g 90 82 2 –1
Now F = ma = m =m ê ´ tú = m = gm = constant
dt dt ë m û m
l
6. (A, B, C) Given (2n + 1) = 50.7 + e
4
u
2. (A, C, D) We know that |F| = hA 0
h l
and (2n + 3) = 83.9 + e
u 4
where 0 = velocity gradient
h 3l / 4 50.7 + e
If n = 1, = \ 3 × 83.9 + 3e = 5 × 50.7 + 5e
|F| u 5l / 4 83.9 + e
Also =h 0 \ 2e = 1.8 Þ e = 0.9 cm
A h
3. (A, B) According to Gauss's Law, 3l
\ = 50.7 + 0.9 = 51.6 \ l = 66.4 cm
4
1 1 3lR
Electric flux, f = qin = [l × 2 R sin 60°] = Also V = vl = 500 × 0.664 ms–1 = 332.0 ms–1
Î0 Î0 Î0
7. (6.30) Impulse = Change in linear momentum
Further electric field is perpendicular to the wire therefore
its z-component will be zero. J 1
\ J = mV0 or V0 = = = 2.5 ms–1
m 0.4
z
l ds
Also V = v0e–t/t \ = v0e–t/t Þ ds = v0e–t/t dt
A R dt
AB = R sin 60°
60° t
B –t / t
\ s = v0 ò e dt = v0t (1 – e–1) = 2.5 × 4 × 0.63 = 6.30 m
0

æ1ö
u2 ç ÷
u 2 sin 2 q è2ø
8. (30.00) H= Þ 120 =
2g 2g
4. (D) The image of AB will be A'B' as AB lies between pole
and focus. Further as the object is moved towards the focus \ u2 = 480 g
the image also moves away.
1
\ K.Einitial = mu 2 = 240 mg
2
EBD_7801
2018 - 30 JEE Advanced 2018 Solved Paper
13. (24.00) Number of electrons emitted per second
1
K.Efinal = (240 mg) = 120 mg
2
200 W
1 2 =
\ mv = 120 mg \ v2 = 240 g 6.25 ´ 1.6 ´ 10 -19 J
2
Force = Rate of change of linear momentum = N 2mk
æ1ö
240 g ´ ç ÷ 200
v 2 sin 2 q è4ø = × 2 ´ 9 ´10 –31 ´1.6 ´ 10 –19 ´ 500
\ H¢ = = = 30 m 6.25 ´1.6 ´ 10–19
2g 2g
[Q K = eV: e = 1.6 × 10–19 = V = 500]
9. (2.00) Given E = sin 103 t iˆ = 24.00
F = ma 14. (3.00) DE2–1 = 74.8 + DE3–2

dv qEdt q sin1000t iˆ é 1ù é1 1ù
\ qE = m \ dv = = dt 13.6z2 ê1 – ú = 74.8 + 13.6z2 ê4 – 9ú
dt m m ë 4û ë û
\ z= 3
v p/w 2p ù
q é T
\ ò dv = ò sin1000t dt ê max. speed is at = kQ kp
m ë 2 w´ 2 úû 15. (B) For a point charge E = 2 and for a dipole E =
0 0 d d3
p/ w 2k l
q é cos1000t ù 1 [cos1000t ]0p / w Further for an infinite long line charge E =
d
and for
\ V= – ê = – –3 ´
m ë 1000 úû 0 10 1000 s
infinite plane charge E =
2 Î0
é p ù
\ V = – êcos1000 ´ – cos 0 ú = –[–1–1] = 2 ms–1 2k l
ë 1000 û r
10. (5.56) We know that Cq = NBA Ig a

Cq 10 – 4 ´ 0.2
\ Ig = = = 0.1A 2k l
NBA 50 ´ 2 ´10 –4 ´ 0.02 r
r
Further for a galvanometer d
Ig × G = (I – Ig) S
I gG
+l l –l
0.1 ´ 50 50
\ S= I – I = = W Also for two infinite wires carrying uniform linear charge
g 1 – 0.1 9 density.
Fl 2k l 2k l l 2k l l
E= cos a = ´ =
11. (3.00) We know that Dl = r d 2 + l2
AY d 2 + l2 d 2 + l2
GM V1 R2 2
1.2 ´ 10 ´ 1 16. (B) Vo= , \ =
= ; 0.3 mm R V2 R1 = 1
2
æ 5 ´ 10 –4 ö
pç ÷÷ ´ 2 ´ 10
11 L1 m1v1 R1 2 ´ 2 ´1 1
ç 2 Further L = m v R =
=
è ø 2 1 2 2 1´1´ 4 1
The third marking of vernier scale will coincide with the GMm k1 m1 R1 2´ 4 8
main scale because least count is 0.1 mm. Also K.E. = . Therefore k = m ´ R = =
R 2 2 2 1´ 1 1
12. (900.00) For an adiabatic process TVg–1 = T2 (8V)g–1 3/ 2
T1 æ R1 ö 1
Further T2 µ R3 \ =ç ÷ =
5 T T2 è R2 ø 8
where g = \ T2 =
3 4 17. (C) Process 1 is adiabatic therefore DQ = 0
Process 2 is isobaric therefore W = P(V2 – V1)
æf ö nfR æ –3T ö = 3P0(3V0–V0) = 6P0V0
Further DV = nCvDT = n ç R ÷ DT = ç ÷
è2 ø 2 è 4 ø Process 3 is isochoric therefore W = P(V2 – V1) = 0
Process 4 is isothermal therefore temperature is
1´ 3 ´ 8 3 constant, Du = 0
\ DV = – ´ ´100 = –900J
2 4
JEE Advanced 2018 Solved Paper 2018 - 31
r As the force is central therefore total energy remains
r dr
18. (A) P ® v = = aiˆ + bˆj which is constant constant.
dt
r
\ r r dr
a =0 R® v = = a[– w sin wt iˆ + w cos wt ˆj ]
r dt
r
Further P = mv is constant \ v = aw i.e., speed is constant
1 r
and K = mv2 is constant r = dv = –aw2[cos wt ˆ + sin wt ĵ ]
2 a i
dt
r æ ¶U ˆ ¶U r r
F = –ç i+ ˆj ö÷ = 0 (Q ar is constant)
\ a = – w2 r
è ¶x r = rr ´ Fr = 0
¶y ø
t
Þ U = constant
Force is central in nature
Also E = K + U
U and K are also constant.
r
dL r r r r r
\ = t= r´F =0 \ L = constant dr
dt S ® vr = = at iˆ + bt ˆj \ V = f(t)
dt
r
r dr r
Q® v = = -aw(sin wt ) ˆi + bw (cos wt ) ˆj a = bĵ i.e., constant
dt r r
r F = ma constant
r dv r
\ a= = -w2 [a cos wt iˆ + b sin wt ˆj] = -w 2 r t
dt r r – mb2t 2
DU = – ò F × dr = – m ò bˆj × (aiˆ + btjˆ)dt =
r r r r r 2
Also t = r ´ F = 0 (\ r and F are parallel) 0

r uur r 1 2 1
mw2 r 2 K= mv = m(a 2 + b 2 t 2 )
DU = - ò F .dr = + ò mw2rdr = \ U µ r2 2 2
2
0
1
Also E = K + U = ma 2 which is constant.
Also r = a cos wt + b sin wt
2 2 2 2
\ r = f (t) 2

Chemistry
19. (A, B, D)
(A) [Co(en)(NH3)3(H2O)]3+ has 2 geometrical isomers.
3+ 3+
NH3 NH3
NH3 NH3
en Co en Co
OH2 NH3
NH3 H2O

(B) Compound [Co(CN)2(NH3)3(H2O)]+ will have three geometrical isomers.


+ + +
H2O H2O H2O
H3N NH3 H3N CN H3N CN
Co Co Co
H3N CN H3N CN NC NH3
CN NH3 NH3

(C) [Co(en)(NH3)3(H2O)]3+ is diamagnetic


(D) [Co(en)(NH3)4]3+ has larger gap between eg and t2g than [Co(en)(NH3)3(H2O)]3+ . So [Co(en)(NH3)3(H2O)]3+absorbs light
at longer wavelength as compared to [Co(en)(NH3)4]3+.
EBD_7801
2018 - 32 JEE Advanced 2018 Solved Paper
20. (B, D)
(A) Cu2+ shows characteristic green colour in the flame test whereas Mn 2+ shows the pale colour in flame test.
(B) Only Cu2+ can give black precipitate of CuS in acidic medium on passing H2S.
(C) Both Cu2+ and Mn2+ show the formation of precipitate by passing H2S in faintly basic medium.

(D) E°Cu 2+ /Cu (+ 0.34V) > E °Mn 2+ /Mn (– 1.18V) as per electrochemical series.

NH2 NH2 NH2 NH2


Conc. HNO
NO2
21. (D) ¾ ¾¾ ¾ ®
3
+ +
Conc. H2SO4
NO2
NO2
(P) 51% (Q) 47% (R) 2%

O O
NH2 NH C CH3 NH C CH3
NO2 NO2 NO2
Ac O, Pyridine Br
¾¾¾¾¾®
2
¾¾¾¾®
2
CH 3COOH
(R)
Br
¾¾®
+
H3O

N2+ Cl– NH2


NO2 NO2 NO2
EtOH NaNO , HCl
¬¾¾¾ ¬¾¾¾¾¾
2
D 273-278 K

Br Br Br
(S)
Br
NO2 NH2 NH2
Sn/HCl Br
¾¾¾® ¾¾¾¾®
2
H2O (excess)
Br Br
Br Br Br
(S)
¾¾®

NaNO2, HCl

Br Br
H3PO4
N2+ Cl–
¬¾¾¾
Br Br Br Br
Br Br
22. (D) Structure of b-L-glucopyranose is First order
A(g) ¾¾¾¾¾
® 2B(g) + C(g) V = constant,T = 300K
H t=0 P0
O
HO
CH2OH H æ 2P0 ö 4P0 2P0
t = t1/3 çè P0 - ÷
3 ø 3 3
H H HO OH P0
=
OH H 3
t=t P0 – x 2x x
23. (A, D) So, Pt = P0 – x + 2x + x = P0 + 2x
or 2x = Pt – P0
JEE Advanced 2018 Solved Paper 2018 - 33
1 P0 O O O
t= ln
k (P0 - x) H 5P 3O 10 = HO P O P O P OH

1 P0 1 2P0 OH OH OH
or t= ln = ln S
k (P - P0 ) k 2P0 - Pt + P0
P0 - t
2 H2S2O3=HO S OH
2P0 O
or kt = ln , kt = ln 2P0 - ln(3P0 - Pt )
3P0 - Pt O O
or ln(3P0–Pt) = – kt + ln 2P0 H2S2O5 = HO S S OH
Graph between ln(3P0 – Pt) vs ‘t’ is a straight line with O
negative slope. 26. (6.47)
Since rate constant is a constant quantity and independent PbS + O2 ¾® Pb + SO2
of initial concentration.
So graph (A) and (D) are correct. 103
No. of moles of O2 =
24. (A, C) 32
On increasing temperature, concentration of product 103
decreases Moles of Pb formed =
Hence reaction is exothermic Þ DH°<0 32
lnK T1 103
> 1 Þ lnK T1 > lnKT2 so, K T > K T \ Mass of Pb formed = ´ 207 = 6468.75 g
lnK T2 1 2 32
= 6.46875 kg
lnK T1 T = 6.47 kg
Also, > 2 27. (126)
lnK T2 T1 The balanced equations are
T1lnKT1 > T2 lnKT2 Þ -RT1lnK T > -RT2lnKT 2MnCl2 + 5K2S2O8 + 8H2O ¾®
or 1 2 2KMnO4 + 4K2SO4 + 6H2SO4 + 4HCl ...(1)
2KMnO4 + 5H2C2O4 + 3H2SO4 ¾®
or DGT°1 < DG°T2 (Q DG = –RT lnK)
K2SO4 + 2MnSO4 + 8H2O + 10CO2 ...(2)
or DH°–T 1DS <DH –
° ° T2DS° Mass of oxalic acid added = 225 mg
(Also DG°=DH°–TDS°: Gibbs Helmholtz equation) 225
Milimoles of oxalic acid added = = 2.5
As DGT°1 < DGT° 2 , this is possible only when DS° < 0 90
From equation (2)
O Milimoles of KMnO4 used to react with oxalic acid = 1
N N
25. (6) N2O3= O N N O (5 m mole H2C2O4 = 2 m mole of KMnO4)
O and milimoles of MnCl2 required initially = 1
O
O O \ Mass of MnCl2 required initially = 1 ´ 126 = 126 mg
O
N2O5= (Molar mass of MnCl2 = 126)
N N
28. (7)
O O 29. (495) Moles of D formed = 10 × 0.6 × 0.5 × 0.5 × 1 = 1.5
P Mass of D formed = 1.5 × 330 = 495 g
O O 30. (– 14.6)
O
P4O6= P 1
O P ® Cu 2O (s) : D G ° = -78 kJ/mol
(i) 2Cu (s) + O2 (g) ¾¾
2
O P O 1
® H 2O(g), D G ° = -178 kJ/mol
(ii) H 2 (g) + O2 (g) ¾¾
2
P O (i) – (ii) then
O
O
P4O7= P 2Cu (s) + H 2 O (g) ¾¾
® Cu 2 O (s) + H 2 (g)
O P=O
DG° = –78 + 178 = 100 kJ/mol = 105 J/mol
O P O Now for the above reaction

O O æ PH ö
DG = DG° + RTln ç 2
÷
P P è PH2 O ø
H4P2O5 = H O H To prevent the above reaction:
OH OH DG > 0
EBD_7801
2018 - 34 JEE Advanced 2018 Solved Paper

æ PH ö Anode ® A n + + ne - ) ´ 2
(A ¾¾
DG° + RTln ç 2
÷ ³0
è PH2 O ø Cathode B2n + + 2ne - ¾¾
®B
æ PH ö Overall reaction :
2
105 + 8 × 1250 ln ç ÷ ³0 2A + B2n + ¾¾
® 2A n + + B
è PH 2O ø
RT
104 (lnPH 2 - ln PH2O ) ³ -105 E = E° - lnQ
2nF
lnPH 2 ³ -10 + ln PH 2O RT [A n + ]2
Now, 0 = E° - ln
2nF [B2n + ]
PH 2O = X H 2O ´ Ptotal = 0.01 ´ 1 = 10-2
RT 22
> –10 – 2ln10 E° = ln
> –10 – 2×2.3 (Given ln10 = 2.3) 2nF 1
ln PH 2 ³ -10 - 4.6 RT
E° = ln 4
2nF
ln PH 2 ³ -14.6 Now,
\ Minimum ln PH 2 = -14.6 2nFRT
DG° = -2nFE° = - ln 4 = - RTln 4
31. (– 8500) 2nF
ˆˆ† AB(g) DG° = DH° – TDS° = 2DG° – TDS° (Given DH° = 2DG°)
A (g) + B(g) ‡ˆˆ
TDS° = DG°
Ea b = Ea f + 2RT & Af = 4A b DGº - RTln 4
DS° = = = - Rln 4
Now, T T
= –8.3 × 2 × 0.7 = –11.62 JK–1 mol–1
Rate constant of forward reaction k f = Af e - Ea f /RT
33. (c) P – 6; Q – 4, 5; R – 1; S – 2, 3
Rate constant of reverse reaction K b = A b e - E ab /RT 1. [FeF6]4–, Fe2+ = 3d6 & F– is weak field ligand
Equilibrium constant \ Hybridization is sp3d2 (high spin complex)
2. [Ti(H2O)3Cl3], Ti3+ = 3d 1 (No effect of ligand field
K f Af - (Eaf - E ab )/RT strength)
K eq = = e
Kb Ab \ Hybridization is d 2sp3
3. [Cr(NH3)6]3+, Cr 3+ = 3d3 (No effect of ligand field
K eq = 4e 2RT /RT = 4e 2 strength)
Now, DG° = –RT ln Keq = –2500 ln (4e2) \ Hybridization is d 2sp3
= –2500 (ln 4 + ln e2) 4. [FeCl4]2–, 3d6 & Cl– is weak field ligand
= –2500 (1.4 + 2)
\ Hybridization is sp3
= –2500 ´ 3.4
= –8500 J/mol. 5. Ni (CO)4, Ni = 3d10 & CO is strong field ligand
32. (– 11.62) \ Hybridization is sp3
A (s) | An+ (aq, 2M) || B2n+ (aq, 1M) | B (s) 6. [Ni(CN)4]2–, Ni2+ = 3d 8 & CN is strong field ligand
Reactions \ Hybridization is dsp2

34. (D)
Ph OH CH3 CH3
Ph O O
Ch3 Ph
(P) H2SO4 –H +
I2/NaOH
Ph ¾¾¾® C+ CH3 ¾¾¾®
(1) H3C ¾¾¾®(1)
Ph
OH Ph
H3C Ph CH3 OH
OH
.. Ph
..

(P)
O
Ph
NaOBr
¾¾¾¾¾® OH
(5)
Ph
CH3
JEE Advanced 2018 Solved Paper 2018 - 35
Ph NH2 H O CH3 O
Ph
(Q) H Ph
HNO + .. –H +
Fehling solution
Ph ¾¾¾®
2
C . ¾¾¾® H3C
.OH ¾¾¾¾¾® Ph
(3)
H3C OH Ph
Ph H H
Me Ph
O
Ph
[Ag(NH ) ]OH
¾¾¾¾¾¾¾®
3 2
OH
(2)
Ph
CH3

Ph OH Ph O CH3 O
Ph Ph
Ph
H SO .. –H +
I /NaOH
(R)
Me ¾ ¾¾
2 4
® C+ . ¾ ¾¾ ® H3C
.OH ¾¾¾®
2
(1)
Ph
Me OH Me Ph CH3 OH
Me Ph

O
Ph
NaOBr
¾ ¾¾ ¾
(5)
¾®
OH
Ph
CH3

Ph Br H O CH3 O
Ph
(S) H Ph
AgNO + .. –H + [Ag(NH ) ]OH
Ph ¾ ¾¾ ®
3
C . ¾ ¾¾ ® H3C
.OH ¾¾¾¾¾®
3 2
Ph
(2)
Me OH Ph
Ph H OH
CH3 Ph
O
Ph
Fehling solution
¾ ¾¾ ¾ ¾¾¾ ® OH
(3)
35. (B) P – 1, 4; Q – 2; R – 4; S – 3 Ph
CH3
H3C H3C H3C
CH3 CH3 CH3 CH2
P: + E2
¾¾®
+
H3C H3C H3C
ONa Br H3C CH3
OH
(1) (4)

CH3
H3C CH3
Q : H3C O + HBr ¾¾® + MeOH

H3C Br
CH3 Me
(2)

H3C
CH3 CH 2
R: MeO– Na +
¾¾¾¾®
E2
H3C H 3C CH 3
Br (4)
EBD_7801
2018 - 36 JEE Advanced 2018 Solved Paper
n
H3C
H3C
CH3 CH3 = å[tan -1(x + j) - tan -1(x + j-1)]
S 2 j=1
S: + MeBr ¾¾®
N
Þ fn(x) = tan–1(x + n) – tan–1 (x)
H3C H3C
ONa OMe æ n ö
= tan -1 ç ÷
(3) è 1 + x(n + x) ø
36. (D) P – 1; Q – 5; R – 4; S – 1 1 1
Þ f n¢ (x) = -
(P) NaOH + CH3COOH ¾® CH3COONa + H2O 1 + (x + n) 1 + x 2
2
m mole 10 × 0.1 20 × 0.1 and fn(0) = tan–1(n) \ tan2(tan–1n) = n2
= 1 m.mol = 2 m.mol Here x = 0 is not in the given domain, i.e., x Î (0, µ).
\ Solution contains 1 m. mol CH3COOH & 1 m.mol \ Options (A) & (B) are not correct options.
CH3COONa in 30 mL solution.
It is a Buffer solution. Hence pH does not change with æ n ö
(C) lim tan(f n (x)) = lim ç ÷=0
dilution. x ®¥ x ®¥ è 1 + x(n + x) ø
(Q) NaOH + CH3COOH ¾® CH3COONa + H2O
m mole 20 × 0.1 20 × 0.1 (D) lim sec 2 (fn (x)) = nlim 1 + tan 2 (f n (x))
= 2 m.mol = 2 m.mol x ®¥ ®¥
\ Solution contains 2 m. mol of CH3COONa in 40 mL
solution (salt of weak acid and strong base) = 1 + lim tan 2 (f n (x)) = 1
x ®¥
For salts of weak acid and strong base :
38. (B, D)
K w Ka P A Q
[H+]initial = T
C (1, 1)B
C
On dilution upto 80 mL, new conc. will be = M
2 R C(0, 1)
K w K a
\ [H+]new = = [H+]initial × 2 common tangent
C/ 2
(radical axis)
(R) HCl + NH3 ¾® NH4Cl
m mole 20 × 0.1 20 × 0.1 Here, AP = AQ = AM
= 2m.mol = 2 m.mol \ locus of M is a circle having PQ as its
\ Solution contains 2 m. mol of NH4Cl in 40 mL solution diameter and is given as :
(salt of strong acid and weak base) E1 : (x – 2) (x + 2) + (y – 7)(y + 5) = 0 and x ¹ ±2
For salts of strong acid and weak base and its centre is (0, 1)
Locus of B (midpoint) is a circle having RC as its diameter
Kw C
[H+]initial = and is given as :
Kb E2 : x(x – 1) + (y – 1)2 = 0
C (A) Option (A) is incorrect although it satisfies E1 otherwise
On dilution upto 80 mL, new conc. will be = . the line T would touch the second circle on two points.
2 (B) Also (4/5, 7/5) satisfies E2 but again in this case one
K w C [H∗ ]initial end of the chord would be (–2, 7) which is not included
\ [H+]new = Kb 2 = 2 æ4 7ö
in E1. Therefore ç , ÷ does not lie in E2.
(S) Ni(OH)2(s) ¾¾
® Ni2+ + 2OH– è5 5ø
(C) (1/2, 1) does not satisfy E2 therefore it does not lie in
Q it is sparingly soluble salt E2.
(D) (0, 3/2) does not satisfy E1, so it does not lie in E1.
\ On dilution [OH–] conc. in saturated solution of 39. (A, D) Here D= 0 so for at least one solution, we have
Ni(OH)2 remains constant
D1 = D 2 = D3 = 0
\ [H+]new = [H+]initial Þ b1 + 7b2 = 13b3 (i)

Mathematics (A) D ¹ 0
\ The equations have unique solution
n \ Option (A) is correct.
-1 æ 1 ö
37. (D) f n (x) = å tan ç ÷ (D) D ¹ 0
è 1 + (x + j)(x + j- 1) ø
j=1 \ The equations have unique solution
n
\ option (D) is correct
é (x + j) - (x + j - 1) ù
= å tan -1 ê 1 + (x + j)(x + j- 1) ú (C) D = 0
Þ equations are x – 2y + 5z = – b1
j=1 ë û
JEE Advanced 2018 Solved Paper 2018 - 37
(C)
b2
x - 2y + 5z = y
2
x - 2y + 5z = b3
The planes given in option (c) are parallel so they must be
coincident
b x
Þ -b1 = 2 = b3 ...(ii) (–1, 0) (1, 0)
2
\ Equation (ii) satisfies equation (i) for all b1, b2, b3, but
not every value of (b1, b2, b3) of equation (i) satisfies 1
x=
equation (ii) 2
\ Option (C) is not correct.
1 1
1 1 3 1 1 1 \ Required area = 2 × ò 1 × 1 - x 2 dx
2 2
(B) D = 5 2 6 = -3 5 2 2 = 0
1
-2 -1 -3 2 1 1 éx 1 ù
= 2 ê 1 - x 2 + sin -1 x ú
ë2 2 û1/ 2
-1 1 3
Also D1 = 2 2 6 b3 = 0 é p æ 1 p öù æp 1ö p-2
= 2 ê - ç + ÷ú = 2 ç - ÷ =
1 -1 -3 ë 4 è 4 8 øû è8 4ø 4 2
For infinite solutions, D2 and D3 must be 0 41. (A, C, D)
sz + tz + r = 0 ...(i)
1 b1 3 ...(ii)
s z+ t z+ r =0
Þ 5 b2 6 =0 Adding (i) and (ii), we get :
-2 b3 -3 (t + s )z + (s+ t) z + (r + r) = 0 (1)
Þ b1 + b2 + 3b3 = 0 which does not satisfy (i) for all b1, b2, b3 Subtracting (ii) from (i), we get:
so option (B) is incorrect. (t - s) z + (s- t ) z + (r - r) = 0 (2)
40. (A, C)
Equation (1) and (2) represent set of lines.
For equation (1) and (2) to have unique solution, we have:
1 t+ s s+ t
¹
2 t -s s-t
Q (0, 0) On solving the above equation we get
|t|¹|s|
\ Option (A) is correct
For equation (1) and (2) to have infinitely many solutions,
1 we have :
Let the equation of common tangent is : y = mx +
m t+ s t +s r + r
= = Þ |t|=|s|
1 t -s s- t r - r
0+0+
m = 1 and tr - t r + sr - sr = sr + s r - t r - t r
\
1 + m2 2 Þ 2 tr = 2s r Þ tr = s r
\ | t || r |=| s || r |
Þ m4 + m2 – 2 = 0 Þ m = ±1
\ Equation of common tangents are Þ | t || r |=| s || r | Þ | t |=| s |
\ If | t | = | s |, lines will be parallel for sure but may not be
y = x + 1 and y = - x - 1 coincident (i.e., does not have infinitely many solutions).
\ Q º (–1, 0) (C) : Locus of Z is a null set or singleton set or a line, in all
x 2 y2 threse cases it will intersect given circle at most two points.
\ Equation of ellipse is : + =1 (D) : in this case locus of Z is a line so L has infinte elements.
1 1/ 2 42. (B, C, D)
1 1 f (x)sin t - f (t)sin x
(A) e = 1 - = lim = sin 2 x
2 2 t ®x t-x
2 f (x) cos t - f ¢(t)sinx
æ 1 ö Þ lim = sin 2 x
2ç ÷ t ®x 1
2b 2
= è

and latus rectum = =1 (Using L’Hospital’s Rule)
a 1
EBD_7801
2018 - 38 JEE Advanced 2018 Solved Paper

Þ f (x) cos x - f ¢(x)sinx = sin 2 x 1/3


(1 + 3) dt - (1 + 3) -2
1/3

æ f ¢(x)sin x - f (x)cos x ö
\ I= ò -2t 6/4
=
2
´
t 1
Þ -ç ÷ =1 1
è sin 2 x ø
(
= 1+ 3 )( )
3 -1 = 2
æ f (x) ö f (x)
Þ -d ç ÷ =1 Þ = -x + c
è sin x ø sin x é a1 a 2 a3 ù
p æpö p 44. (4) Suppose P = êê b1 b2 b3 úú
Now, Put x = also it is given that fç ÷=-
6 è6ø 12 êë c1 c2 c3 úû
-p a1 a2 a3
p -p -p
\ 12 = - + c Þ = +c So, det (P) = b1 b2 b3
1 6 12 12
c1 c2 c3
2
Þ c = 0 Þ f(x) = – x sin x = a1 (b2c3 – b3c2) – a2 (b1c3 – b3c1) + a3 (b1c2 – b2c1)
æ p ö -p 1 Maximum value can be 6 when a1 = 1, a2 = –1, a3 = 1 and
(A) f ç ÷ =
è4ø 4 2 b2c3 = b1c3 = b1c2 = 1 & b3c2 = b3c1 = b2c1 = –1.
(B) f(x) = –x sin x So, (b2c3) (b3c1) (b1c2) = – 1
x3 and (b1c3) (b3c2) (b2c1) = 1.
as sin x > x - "x Î (0, p)
6 Therefore b1b2b3c1c2c3 has two values 1 and –1
which is not possible.
x4
\- x sin < - x 2 + Contradiction also occurs if a1 = 1, a2 = 1, a3 = 1 and
6 b2c2 = b3c1 = b1c2 = 1 & b3c2 = b1c3 = b1c2 = –1.
4 For maximum value to be 5 one of the terms should be
\ f (x) < - x 2 + x "x Î (0, p) zero but this will make 2 terms zero therefore answer
6
should not be 5.
(C) f ¢(x) = - sin x - x cos x
1 0 1
f ¢(x) = 0 Þ tan x = - x
As -1 1 1 = 4.
Þ there exist a Î (0, p) for which f ¢(a)=0
-1 -1 1
Hence maximum value of the determinant of P = 4.
45. (119) Here n (X) = 5 and n (Y) = 7
Number of one-one function = a = 7C5 × 5!
and Number of onto function Y to X is given as:

a1
(D) Here, f ¢¢(x) = -2cos x + x sin x
a2 b2
\ f ¢¢ æç p ö÷ = p and f æç p ö÷ = - p . .
è2ø 2 è2ø 2 . .
æpö æpö . .
Þ f ¢¢ ç ÷ + f ç ÷ = 0
è2ø è2ø a7 b5
1 1, 1, 1, 1, 3 1, 1, 1, 2, 2
2
(1 + 3) dx
43. (2) Let I = ò é(1 + x) (1 - x)
2 6ù
1/4 b=
7!
3! 4!
× 5! +
7!
(2!)3 3!
× 5! = (7C3 + 3 × 7C3) 5!
0 ë û
= 4 × 7C3 × 5!
1
2 b-a
(1 + 3) dx Þ = 4 × 7C3 – 7C5 = 4 × 35 – 21 = 119
= ò é (1 - x)6 ù
1/4 5!
0 2
(1 + x) ê ú dy æ 2ö æ 2ö
6 46. (0.4) = (5y + 2) (5y – 2) = 25 ç y + ÷ çy - ÷
ëê (1+ x) ûú dx è 5ø è 5ø
1- x -2dx 1 dy
Now put
1+ x
=t Þ
(1 + x) 2
= dt Þ
25 ò æ y + 2 ö æ y - 2 ö = ò dx
ç ÷ç ÷
è 5øè 5ø
JEE Advanced 2018 Solved Paper 2018 - 39
49. (0.5) y
1 5é 1 1 ù
Þ
25 ò -
4 y- 2 y+ 2 ú
ê
ê ú
dy = ò dx
Q (0, 1, 0)
ë 5 5û

2
y- T (1, 1, 1)
1 5 5 = x + c, where c is constant of
Þ ´ ln . S ( 12 , 12 , 12 )
25 4 2
y+
5 O (0, 0, 0) P (1, 0, 0) x
integration.
1 5y - 2 R
Þ ln =x +c (0, 0, 1)
20 5y + 2
Z
As, f(0) = 0 Þ at x = 0, y = 0 Þ 0 = 0 + c Þ c = 0 r uur ˆi - ˆj - kˆ
Here, p = SP =
5y - 2 2
Therefore, = e20x
5y + 2 r uuur - ˆi + ˆj - kˆ
q = SQ =
5 f (x) - 2 2
Þ lim = lim e 20x = e–¥ = 0 uuu
r
x ® -¥ 5 f (x) + 2 x ® -¥ r - i - ˆj + kˆ
ˆ
r = SR =
2
2
Þ 5 lim f(x) – 2 = 0 Þ lim f(x) = = 0.4 r uur ˆi + ˆj + kˆ
x ® -¥ x ® -¥ 5 t = ST =
2
47. (2) f (x + y) = f (x) f ¢ (y) + f ¢ (x) f (y) (1)
After putting x = y = 0, we get ˆi ˆj kˆ
r r 1 2iˆ + 2ˆj ˆi + ˆj
1 So, p ´ q = 1 -1 -1 = =
f (0) = 2f ¢ (0) f (0) Þ f ¢ (0) = [Q f (0) = 1] 4 4 2
2 -1 1 -1
Now putting y = 0 in equation (1), we get
ˆi ˆj kˆ
f (x) = f (x) f ¢ (0) + f ¢ (x) f (0)
r r 1 - 2iˆ + 2ˆj -ˆi + ˆj
f (x) f ¢ (x) 1 and, r ´ t = -1 -1 1 = =
Þ f ¢(x) =
2
Þ ò f (x)
dx =
2 ò dx 4
1 1 1
4 2

[Q f(0) = 1 and f ¢(0) = 1 ] ˆi ˆj kˆ


2 r r r r 1 kˆ
Hence, (p ´ q) ´ (r ´ t ) = 1 1 0 =
x 4 2
Þ loge f (x) = + loge c -1 1 0
2
Þ f (x) = cex/2 Þ f (x) = ex/2 (Q f (0) = 1) r r r r 1
Þ (p ´ q) ´ (r ´ t ) = = 0.5
x 2
Þ loge (f (x)) = Þ loge (f (4)) = 2 50. (646)
2
48. (8) Suppose coordinates of P are (a, b, c). n n
So, coordinates of Q are (0, 0, c) and coordinates of R å r(n Cr )2 = n å n Cr n -1Cr -1
are (a, b, –c). r =0 r =0
Here, PQ is perpendicular to the plane x + y = 3. n
So, PQ is parallel to the normal of given plane = n å n Cn - r n -1Cr -1 = n2n–1C
n–1
r =1
i.e. (aiˆ + bj)
ˆ is parallel to (iˆ + ˆj) So, X = (10C1)2 + 2(10C2)2 + 3(10C3)2 + ...+10(10C10)2
Þ a=b 10
As mid-point of PQ lies in the plane x + y = 3, so = å r(10 Cr ) 2 = 1019 C9
n =0
a b
+ = 3Þa +b= 6Þa= 3=b X 1 19
2 2 Hence, = C9 = 646
1430 143
Therefore, distance of P from the x-axis = b2 + c 2 = 5 x
51. (A) For E1, > 0 and x ¹ 1 Þ x Î ( -¥,0) È (1, ¥)
(given) x -1
Þ b2 + c2 = 25 Þ c2 = 25 – 9 = 16 æ x ö 1 x
Þ c=±4 For E 2 , - 1 £ log e ç ÷ £1 Þ £ £e
Hence, PR = |2c| = 8 è x -1 ø e x -1
1 1
Þ £ 1+ £e
e x -1
EBD_7801
2018 - 40 JEE Advanced 2018 Solved Paper
1 1 æ e ù é 1 ö 2
-1 £ £ e - 1 Þ (x - 1) Î ç -¥, ú Èê ,¥ ÷ Þ The eccentricity of H= e = and
e x -1 è 1 - e û ë e - 1 ø 3
The distance between the foci of H = 2ae
æ 1 ù é e ö
Þ x Î ç -¥, ú Èê ,¥ ÷ 2
è e - 1 û ë e - 1 ø = 2´ 2 3 ´ =8
3
x
For E1, Î (0, ¥) - {1} 2b2 2 ´ 4 4
x -1 and length of latus ractum of H = = =
æ x ö a 2 3 3
Þ log e ç ÷ Î ( -¥, ¥ ) - {0} 54. (D)
è x -1ø
é 2 ù
Þ f (x) Î (-¥,0) È (0, ¥ ) sin 1 - e - h - 0 ú
(i) f '1 (0) = lim ê
æ æ x öö é p p ù h ®0 ê h ú
g(x) = sin -1 ç loge ç ÷ ÷ Î - , - {0} ëê úû
è è x - 1 ø ø ëê 2 2 ûú
52. (C) Here, a committee has to be formed form a group of 6 é 2 2 ù
ê sin 1 - e - h sin 1 - e- h hú
Boys and 5 girls. = lim ´ ´
Total number of ways for selecting exactly 3 boys and h ®0 ê -h 2 h2 hú
2 girls = 6C3 × 5C2 = 20 × 10 = 200 = a1. ëê 1 - e úû
Total number of ways for selecting at least 2 members é hù h é sin x ù
with equal number of boys and girls = lim ê1´ 1´ ú = lim
h ®0 ë h û h ®0 h êëQ xlim
®0 x
= 1ú
û
= (6C1 × 5C1) + (6C2 × 5C2) + (6C3 × 5C3 ) + (6C4 × 5C4)
+ (6C5 × 5C5) = 11C5 – 1 = 461 = a2. which does not exit.
Total number of ways for selecting 5 members having So for (P), (2) is correct.
at least 2 girls = 11C5 – 6C5 – 6C4 × 5C1 = 11C5 – 81 = 381 é sin x ù
= a3 (ii) lim f 2 (x) = lim ê ú
x ®0 x ®0 ë tan –1 x û
Total number of ways for selecting 4 memebrs having
at least 2 girls M1 and G1 are not selected together é sin x x xù
= n(M1 selected & G1 not selected) + n(G1 selected & lim ê ´ ´ ú
x ®0 ë x –1
M1 not selected) + n(M1 and G1 both not selected) tan x x û
= (4C2 × 5C1 + 4C3) + (4C1 × 5C2 + 4C2 × 5C1+ 4C3) é xù x é x ù
+(4C4 + 4C3 × 5C1 + 4C2× 5C2) lim ê1´1´ ú = lim êQ lim = 1ú
x û x ®0 x - 1
x ®0 ë ë x ®¥ tan x û
= 34 + 4 + 81 = 189 = a 4 which does not exist, so for Q, (1) is correct.
53. (B) y
(iii) lim f3 (x) = lim [sin(log e (x + 2))]
x ®0 x ®0
L (0, b)
if x ® 0 Þ (x + 2) ® 2
Þ log e (x + 2) ® log e 2 < 1
90°
30° N Þ 0 < lim sin(log e (x + 2) < sin1
x x ®0
(–a, 0) O 30° (a, 0)
Þ lim [sin(log e (x + 2))] = 0
x®0

f3 (x) = 0 "x Î [ -1, e p /2 - 2)


M (0, –b)
Þ f '3 (x) = 0 "x Î ( -1, ep /2 - 2)
Area of DLMN = 4 3 (given)
Þ f ''3 (x) = 0 "x Î ( -1, ep /2 - 2)
1
Þ ´ LM ´ ON = 4 3 So for (R), (4) is correct.
2
æ 2 1ö 2æ 1ö
1
Þ (2b)( 3 b) = 4 3 (iv) lim f 4 (x) = lim ç x sin ÷ = lim x ç sin ÷ = 0
x ®0 x ®0 è x ø x ®0 è xø
2
æ1ö
Þ b2 = 4 Þ b = 2 h 2 sin ç ÷ - 0
So, length of the conjugate axis of H = 2b = 4 èhø æ1ö
f '4 (0) = lim = lim h sin ç ÷ = 0
OL b x ®0 h x ®0 è4ø
tan 30° = = Þ a = 3b Þ a = 2 3
ON a 1 1
f '4 (x) = - cos + 2x sin , x ¹ 0
\ b2 = a2 (e2 – 1) x x
Þ 4 = 12(e2–1)
é 1 1ù 1
1 4 lim f 4¢ (x) = lim ê - cos + 2x sin ú = - lim cos
Þ e2 = 1 + = x ®0 x ®0 ë x xû x ®0 x
3 3 which does not exist
So for (S), (3) is correct.
JEE ADVANCED 2017
1. The question paper consists of three parts (Physics, Chemistry and Mathematics). Each part consists of three
sections.
2. Section I contains 7 multiple choice questions. Each question has four choices (a), (b), (c) and (d) out of which
ONE OR MORE THAN ONE is/are correct.
3. Section II contains 5 questions. The answer to each of the questions is a single-digit integer ranging from 0 to 9
(both inclusive).
4. Section III contains 6 questions of Matching type, contains two tables each having 3 columns and 4 rows. Each
question has four choices (a), (b), (c) and (d). ONLY ONE of these four options is correct.

PAPER - 1

PHYSICS mR
(A) The position of the point mass m is: x = - 2
M+m
SECTION - I
2gR
This section contains 7 questions. Each question has 4 options m
(A), (B), (C) and (D). ONE or MORE THAN ONE of these four (B) The velocity of the point mass m is: v = 1+
M
options is (are) correct.
(C) The x component of displacement of the center of mass
1. A flat plate is moving normal to its plane through a gas
under the action of a constant force F. The gas is kept at a mR
of the block M is: -
very low pressure. The speed of the plate v is much less M+m
than the average speed u of the gas molecules.
Which of the following options is/are true? m
(D) The velocity of the block M is: V = - 2gR
(A) The pressure difference between the leading and M
trailing faces of the plate is proportional to uv 3. A block M hangs vertically at the bottom end of a uniform
(B) The resistive force experienced by the plate is rope of constant mass per unit length. The top end of the
proportional to v rope is attached to a fixed rigid support at O. A transverse
(C) The plate will continue to move with constant non- wave pulse (Pulse 1) of wavelength l0 is produced at point
zero acceleration, at all times
O on the rope. The pulse takes time TOA to reach point A. If
(D) At a later time the external force F balances the resistive
force the wave pulse of wavelength l0 is produced at point A
2. A block of mass M has a circular cut with a frictionless (Pulse 2) without disturbing the position of M it takes time
surface as shown. The block rests on the horizontal TAO to reach point O. Which of the following options is/are
frictionless surface of a fixed table. Initially the right edge of correct?
the block is at x = 0, in a co-ordinate system fixed to the table.
A point mass m is released from rest at the topmost point of
the path as shown and it slides down. O Pulse 1
When the mass loses contact with the block, its position is
x and the velocity is v. At that instant, which of the following
options is/are correct?

R
Pulse 2
A M
m
R
y (A) The time TAO = TOA
(B) The velocities of the two pulses (Pulse 1 and Pulse 2)
are the same at the midpoint of rope
x M (C) The wavelength of Pulse 1 becomes longer when it
reaches point A
x=0 (D) The velocity of any pulse along the rope is
independent of its frequency and wavelength
EBD_7801
2017-2 JEE Advanced 2017 Solved Paper
4. A human body has a surface area of approximately 1 m2. The 6. In the circuit shown, L = 1 mH, C = 1 mF and R = 1 kW. They
normal body temperature is 10 K above the surrounding are connected in series with an a.c. source V = V0 sin wt as
room temperature T0. Take the room temperature to be shown. Which of the following options is/are correct?
T0 = 300 K. For T0 = 300 K, the value of sT04 = 460 Wm-2 L = 1mH C = 1mF R = 1kW
(where s is the Stefan-Boltzmann constant). Which of the
following options is/are correct?
(A) The amount of energy radiated by the body in 1 second ~ V0 sin wt
is close to 60 joules
(B) If the surrounding temperature reduces by a small
amount DT0 << T0, then to maintain the same body (A) The current will be in phase with the voltage if w = 104
temperature the same (living) human being needs to rad.s-1
(B) The frequency at which the current will be in phase
radiate DW = 4sT03 DT0 more energy per unit time with the voltage is independent of R
(C) Reducing the exposed surface area of the body (e.g. (C) At w ~ 0 the current flowing through the circuit
by curling up) allows humans to maintain the same becomes nearly zero
body temperature while reducing the energy lost by (D) At w >> 106 rad. s-1, the circuit behaves like a capacitor
radiation 7. For an isosceles prism of angle A and refractive index m, it is
(D) If the body temperature rises significantly then the found that the angle of minimum deviation dm= A.
peak in the spectrum of electromagnetic radiation Which of the following options is/are correct?
emitted by the body would shift to longer wavelengths (A) For the angle of incidence i1 = A, the ray inside the
5. A circular insulated copper wire loop is twisted to form two prism is parallel to the base of the prism
loops of area A and 2A as shown in the figure. At the point (B) For this prism, the refractive index µ and the angle of
of crossing the wires remain electrically insulated from each 1 -1 æ m ö
other. The entire loop lies in the plane (of the paper). A uniform prism A are related as A = cos ç ÷
2 è2ø
®
magnetic field B points into the plane of the paper. At t = 0, (C) At minimum deviation, the incident angle i1 and the
the loop starts rotating about the common diameter as axis refracting angle r 1 at the first refracting surface are
with a constant angular velocity w in the magnetic field. related by r1 = (i1/2)
Which of the following options is/are correct? (D) For this prism, the emergent ray at the second surface
will be tangential to the surface when the angle of
incidence at the first surface is
× × × × × × × ×
B -1 é 2 A ù
× × × × × × × × i= - 1 - cos A ú
1 sin êsin A 4 cos
× × × × × × × × ë 2 û
× × × ×area
× A× × ×
× × × × × × × × SECTION - II
× × × × × × × × This section contains 5 questions. The answer to each question
× × × × × × × × is a SINGLE DIGIT INTEGER ranging from 0 to 9, both inclusive.
× × × × × × × ×
× ×
area 2A
× × × × × × 8. A drop of liquid of radius R = 10–2 m having surface tension
× × × × × × × × 0.1
× × × × × × × × S= Nm-1 divides itself into K identical drops. In this
4p
× × × × × × × × process the total change in the surface energy DU = 10–3 J.
w
If K = 10a then the value of a is
9. An electron in a hydrogen atom undergoes a transition from
an orbit with quantum number ni to another with quantum
(A) The emf induced in the loop is proportional to the sum number nf. Vi and Vf are respectively the initial and final
of the areas of the two loops
(B) The amplitude of the maximum net emf induced due to Vi
potential energies of the electron. If V = 6.25, then the
both the loops is equal to the amplitude of maximum f
emf induced in the smaller loop alone smallest possible nf is
(C) The net emf induced due to both the loops is 10. A monochromatic light is travelling in a medium of refractive
proportional to cos wt index n = 1.6. It enters a stack of glass layers from the bottom
(D) The rate of change of the flux is maximum when the side at an angle q = 30°. The interfaces of the glass layers are
plane of the loops is perpendicular to plane of the parallel to each other. The refractive indices of different glass
paper
JEE Advanced 2017 Solved Paper 2017-3

layers are monotonically decreasing as nm= n – mDn, where 11. A stationary source emits sound of frequency f0 = 492 Hz.
nm is the refractive index of the mth slab and Dn = 0.1 (see The sound is reflected by a large car approaching the source
the figure). The ray is refracted out parallel to the interface with a speed of 2 ms–1. The reflected signal is received by
between the (m – 1)th and mth slabs from the right side of the the source and superposed with the original.
stack. What is the value of m? What will be the beat frequency of the resulting signal in
Hz? (Given that the speed of sound in air is 330 ms–1 and the
m n – mDn car reflects the sound at the frequency it has received).
m–1 n – (m – 1)Dn 12. 131I is an isotope of Iodine that B decays to an isotope of

Xenon with a half-life of 8 days. A small amount of a serum


labelled with 131I is injected into the blood of a person. The
activity of the amount of 131I injected was 2.4 × 105 Becquerel
3 n – 3Dn (Bq). It is known that the injected serum will get distributed
2 n – 2Dn uniformly in the blood stream in less than half an hour. After
1 n – Dn 11.5 hours, 2.5 ml of blood is drawn from person's body, and
n gives an activity of 115 Bq. The total volume of blood in the
q
person¡¦s body, in liters is approximately
(you may use ex » 1 + x for |x| < < 1 and ln2 » 0.7).

SECTION - III
This section contains 6 questions of MATCHING TYPE, contains two tables each having 3 columns and 4 rows. Based on each table,
there are three questions. Each question has four options (A), (B), (C) and (D) ONLY ONE of these four options is correct.
Answer (Qs. 13-15) : By appropriately matching the information given in the three columns of the following table.
®
A charged particle (electron or proton) is introduced at the origin (x = 0, y = 0, z = 0) with a given initial velocity v . A uniform
® ® ® ® ®
electric field E and a uniform magnetic field B exist everywhere. The velocity v, electric field E and magnetic field B are given in
columns 1, 2 and 3, respectively. The quantities E0, B0 are positive in magnitude.
Column 1 Column 2 Column 3
® E0 ® ®
(I) Electron with v = 2 xˆ (i) E = E 0 zˆ (P) B = - B xˆ
B0 0

® E0 ® ®
(II) Electron with v = yˆ (ii) E = - E 0 yˆ (Q) B = B0 xˆ
B0

® ® ®
(III) Proton with v = 0 (iii) E = - E 0 xˆ (R) B = B0 yˆ

® E0 ® ®
(IV) Proton with v = 2 xˆ (iv) E = E 0 xˆ (S) B = B0 zˆ
B0

13. In which case will the particle move in a straight line with constant velocity?
(A) (III) (ii) (R) (B) (IV) (i) (S) (C) (III) (iii) (P) (D) (II) (iii) (S)
14. In which case will the particle describe a helical path with axis along the positive z direction?
(A) (IV) (i) (S) (B) (II) (ii) (R) (C) (III) (iii) (P) (D) (IV) (ii) (R)
15. In which case would the particle move in a straight line along the negative direction of y-axis (i.e., move along – ŷ )?
(A) (II) (iii) (Q) (B) (III) (ii) (R) (C) (IV) (ii) (S) (D) (III) (ii) (P)

Answer (Qs. 16-18) : By appropriately matching the information given in the three columns of the following table.
An ideal gas is undergoing a cyclic thermodynamic process in different ways as shown in the corresponding P–V diagrams in
column 3 of the table. Consider only the path from state 1 to state 2. W denotes the corresponding work done on the system. The
equations and plots in the table have standard notations as used in thermodynamic processes. Here Y is the ratio of heat capacities at
constant pressure and constant volume. The number of moles in the gas is n.
EBD_7801
2017-4 JEE Advanced 2017 Solved Paper
Column 1 Column 2 Column 3
1
(I) W1®2 = (P2 V2 - P1V1 ) (i) Isothermal (P) P 1
g -1 2

(II) W1®2 = -PV2 + PV1 (ii) Isochoric (Q) P


1

P 1 2

(III) W1®2 = 0 (iii) Isobaric (R)

P
1

æV ö
(IV) W1®2 = -nRT ln ç 2 ÷ (iv) Adiabatic (S)
è V1 ø 2

16. Which of the following options is the only correct representation of a process in which DU = DQ – PDV?
(A) (II) (iv) (R) (B) (III) (iii) (P) (C) (II) (iii) (S) (D) (II) (iii) (P)
17. Which one of the following options is the correct combination?
(A) (IV) (ii) (S) (B) (III) (ii) (S) (C) (II) (iv) (P) (D) (II) (iv) (R)
18. Which one of the following options correctly represents a thermodynamic process that is used as a correction in the determination
of the speed of sound in an ideal gas?
(A) (I) (ii) (Q) (B) (IV) (ii) (R) (C) (III) (iv) (R) (D) (I) (iv) (Q)
JEE Advanced 2017 Solved Paper 2017-5

CHEMISTRY 22. The colour of the X2 molecules of group 17 elements changes


gradually from yellow to violet down the group. This is due
to
SECTION - I (A) The physical state of X2 at room temperature changes
This section contains 7 questions. Each question has 4 options from gas to solid down the group
(A), (B), (C) and (D). ONE or MORE THAN ONE of these four (B) Decrease in ionization energy down the group
options is (are) correct. (C) Decrease in p* - s* gap down the group
(D) Decrease in HOMO-LUMO gap down the group
19. An ideal gas is expanded from (p1, V1, T1) to (p2, V2, T2) 23. Addition of excess aqueous ammonia to a pink coloured
under different conditions. The correct statement(s) among aqueous solution of MCl2.6H2O(X) and NH4Cl gives an
the following is(are) octahedral complex Y in the presence of air. In aqueous
(A) The work done on the gas is maximum when it is solution, complex Y behaves as 1:3 electrolyte. The reaction
compressed irreversibly from (p2, V2) to (p1, V1 ) of X with excess HCl at room temperature results in the
against constant pressure p1 formation of a blue coloured complex Z. The calculated spin
(B) If the expansion is carried out freely, it is simultaneously only magnetic moment of X and Z is 3.87 B.M., whereas it is
both isothermal as well as adiabatic zero for complex Y. Among the following options, which
(C) The work done by the gas is less when it is expanded statement(s) is(are) correct?
reversibly from V1 to V2 under adiabatic conditions as (A) Addition of silver nitrate to Y gives only two
compared to that when expanded reversibly from V1 equivalents of silver chloride
to V2 under isothermal conditions (B) The hybridization of the central metal ion in Y is d2sp3
(D) The change in internal energy of the gas is (i) zero, if it (C) Z is a tetrahedral complex
is expanded reversibly with T1 = T2, and (ii) positive, if (D) When X and Z are in equilibrium at 0°C, the colour of
it is expanded reversibly under adiabatic conditions the solution is pink
with T1 ¹ T2 24. The IUPAC name(s) of the following compound is(are)
20. For a solution formed by mixing liquids L and M, the vapour
pressure of L plotted against the mole fraction of M in
solution is shown in the following figure. Here xL and xM H3C Cl
represent mole fractions of L and M, respectively, in the
solution. The correct statement(s) applicable to this system (A) 1-chloro-4-methylbenzene
is (are) (B) 4-chlorotoluene
(C) 4-methylchlorobenzene
(D) 1-methyl-4-chlorobenzene
Z 25. The correct statement(s) for the following addition reactions
is(are)
pL
H3C H
Br 2/CHCl 3
M and N
(i)
H CH3
1 xM 0
H3C CH3
(A) The point Z represents vapour pressure of pure liquid Br 2/CHCl 3
(ii) O and P
M and Raoult's law is obeyed from xL= 0 to xL= 1
(B) The point Z represents vapour pressure of pure liquid H H
L and Raoult's law is obeyed when xL ® 1 (A) O and P are identical molecules
(C) The point Z represents vapour pressure of pure liquid (B) (M and O) and (N and P) are two pairs of diastereomers
M and Raoult's law is obeyed when xL ® 0 (C) (M and O) and (N and P) are two pairs of enantiomers
(D) Attractive intermolecular interactions between L-L in
(D) Bromination proceeds through trans-addition in both
pure liquid L and M-M in pure liquid M are stronger
the reactions
than those between L-M when mixed in solution.
21. The correct statement(s) about the oxoacids, HClO4 and SECTION - II
HClO, is(are)
(A) The central atom in both HClO4 and HClO is sp3 This section contains 5 questions. The answer to each question
hybridized is a SINGLE DIGIT INTEGER ranging from 0 to 9, both inclusive.
(B) HClO4 is more acidic than HClO because of the 26. A crystalline solid of a pure substance has a face-centred
resonance stabilization of its anion cubic structure with a cell edge of 400 pm. If the density of
(C) HClO4 is formed in the reaction between Cl2 and H2O the substance in the crystal is 8 g cm-3, then the number of
(D) The conjugate base of HClO4 is weaker base than H2O
atoms present in 256 g of the crystal is N × 1024. The value
of N is
EBD_7801
2017-6 JEE Advanced 2017 Solved Paper
27. The conductance of a 0.0015 M aqueous solution of a weak (Atomic numbers: H = 1, He = 2, Li = 3, Be = 4, B = 5, C = 6,
monobasic acid was determined by using a conductivity N = 7, O = 8, F = 9)
cell consisting of platinized Pt electrodes. The distance 30. Among the following, the number of aromatic compound(s)
between the electrodes is 120 cm with an area of cross section is
of 1 cm2. The conductance of this solution was found to be
5 × 10–7S. The pH of the solution is 4. The value of limiting
o
( )
molar conductivity L m of this weak monobasic acid in
aqueous solution is Z × 102 S cm-1 mol-1 . The value of Z is
28. The sum of the number of lone pairs of electrons on each
central atom in the following species is
[TeBr6]2-, [BrF2]+, SNF3 and [XeF3]–
(Atomic numbers: N = 7, F = 9, S = 16, Br = 35, Te = 52,
Xe = 54)
29. Among H2, He2+, Li2, Be2, B2, C2, N2,O2– and F2, the number
of diamagnetic species is

SECTION - III
This section contains 6 questions of MATCHING TYPE, contains two tables each having 3 columns and 4 rows. Based on each table,
there are three questions. Each question has four options (A), (B), (C) and (D) ONLY ONE of these four option is correct.
(Qs. 31-33) : By appropriately matching the information given in the three columns of the following table.
The wave function, yn,l,ml is a mathematical function whose value depends upon spherical polar coordinates (r, q, f) of the
electron and characterized by the quantum numbers n, l and ml. Here r is distance from nucleus, q is colatitude and f is azimuth. In the
mathematical functions given in the table, Z is atomic number and a0 is Bohr radius.
Column 1 Column 2 Column 3

3 æ Zr ö
æ Z ö 2 -ç a ÷
yn,l,ml,(r)

(i) 1s orbital (i) y n,l,ml µ ç ÷ e è 0 ø (P)


è a0 ø 0
r
a0

1
(ii) 2s orbital (ii) One radial node (Q) Probability density at nucleus µ
a 30

5 æ Zr ö
æ Z ö 2 -ç 2a ÷
(iii) 2pz orbital (iii) y n,l,ml µ ç ÷ re è 0 ø cos q (R) Probability density is maximum at nucleus
è a0 ø

(iv) 3d orbital (iv) xy-plane is a nodal plane (S) Energy needed to excite electron from n = 2
z2

27
state to n = 4 state is
times the energy
32
needed to excite electron from n = 2 state to
n = 6 state
31. For the given orbital in Column 1, the only CORRECT combination for any hydrogen-like species is
(A) (I) (ii) (S)
(B) (IV) (iv) (R)
(C) (II) (ii) (P)
(D) (III) (iii) (P)
JEE Advanced 2017 Solved Paper 2017-7

32. For hydrogen atom, the only CORRECT combination is 39. Let [x] be the greatest integer less than or equals to x. Then,
(A) (I) (i) (S) at which of the following point(s) the function
f(x) = x cos(p(x + [x])) is discontinuous?
(B) (II) (i) (Q)
(A) x = – 1 (B) x = 0
(C) (I) (i) (P) (C) x = 1 (D) x = 2
(D) (I) (iv) (R) 40. Let f : R ® (0,1) be a continuous function. Then, which of
33. For He+ ion, the only INCORRECT combination is the following function(s) has(have) the value zero at some
point in the interval (0, 1)?
(A) (I) (i) (R)
(A) x9 – f(x)
(B) (II) (ii) (Q)
p
(C) (I) (iii) (R) -x
(B) x-ò2 f (t) cos t dt
(D) (I) (i) (S) 0

(Qs. 34-36) : By appropriately matching the information given in x


(C) ex - ò f (t) sin t dt
the three columns of the following table. 0
Columns 1, 2 and 3 contain starting materials, reaction
p
conditions, and type of reactions, respectively.
(D) f (x) + ò 2 f (t) sin t dt
0
Column 1 Column 2 Column 3
(I) Toluene (i) NaOH/Br2 (P) Condensation 41. Which of the following is(are) not the square of a 3 × 3 matrix
(II) Acetophenone (ii) Br 2/hv (Q) Carboxylation with real entries?
(III) Benzaldehyde (iii) (CH3CO)2O/ (R) Substitution
é1 0 0ù é1 0 0 ù
CH3COOK ê0 1 0ú ê0 1 0 ú
(IV) Phenol (iv) NaOH/CO2 (S) Haloform (A) ê ú (B) ê ú
êë0 0 1 úû êë 0 0 -1úû
34. For the synthesis of benzoic acid, the only CORRECT
combination is
(A) (II) (i) (S) (B) (IV) (ii) (P) é1 0 0 ù é -1 0 0 ù
(C) (I) (iv) (Q) (D) (III) (iv) (R) ê 0 -1 0 ú ê 0 -1 0 ú
(C) ê ú (D) ê ú
35. The only CORRECT combination that gives two different êë 0 0 -1úû êë 0 0 -1úû
carboxylic acids is
(A) (II) (iv) (R) (B) (IV) (iii) (Q) 42. Let a, b, x and y be real numbers such that a – b = 1 and
(C) (III) (iii) (P) (D) (I) (i) (S) y ¹ 0. If the complex number z = x + iy satisfies
36. The only CORRECT combination in which the reaction
proceeds through radical mechanism is æ az + b ö
Im ç
è z + 1 ÷ø
= y, then which of the following is(are) possible
(A) (III) (ii) (P) (B) (IV) (i) (Q)
(C) (II) (iii) (R) (D) (I) (ii) (R) value(s) of x ?

MATHEMATICS (A) -1 + 1 - y 2

SECTION - I (B) -1 - 1 - y 2
This section contains 7 questions. Each question has 4 options (C) 1 + 1 + y2
(A), (B), (C) and (D). ONE or MORE THAN ONE of these four
options is (are) correct.
(D) 1 - 1 + y2

x2 y2 1 1
37. If 2x – y + 1 = 0 is a tangent to the hyperbola - = 1, 43. Let X and Y be two events such that P(X) = , P(X|Y) =
2 16
a 3 2
then which of the following cannot be sides of a right angled
triangle? 2
and P (Y|X) = . Then
(A) a, 4, 1 (B) a, 4, 2 5
(C) 2a, 8, 1 (D) 2a, 4, 1
4 1
38. If a chord, which is not a tangent, of the parabola y2 = 16x (A) P(Y) = (B) P(X¢ | Y) =
has the equation 2x + y = p, and midpoint (h, k), then which 15 2
of the following is(are) possible value(s) of p, h and k ? 1 2
(A) p = –2, h = 2, k = –4 (C) P(X Ç Y) = (D) P(X È Y) =
(B) p = –1, h = 1, k = –3 5 5
(C) p = 2, h = 3, k = –4
(D) p = 5, h = 4, k = –3
EBD_7801
2017-8 JEE Advanced 2017 Solved Paper
SECTION - II
é1 a a 2 ù é x ù é 1ù
This section contains 5 questions. The answer to each question ê ú
a 1 a ú êê y úú = êê - 1úú
is a SINGLE DIGIT INTEGER ranging from 0 to 9, both inclusive. 46. For a real number a, if the system ê
ê 2 ú
ëa a 1 û êë z úû êë 1úû
44. For how many values of p, the circle x2 + y2 + 2x + 4y – p = 0
and the coordinate axes have exactly three common points? of linear equations, has infinitely many solutions,
45. Let f : R ® R be a differentiable function such that f(0) = 0, then 1 + a + a2 =
47. Words of length 10 are formed using the letters A, B, C, D, E,
æpö F, G, H, I, J. Let x be the number of such words where no letter
f ç ÷ = 3 and f ¢(0) = 1.
è2ø is repeated; and let y be the number of such words where
exactly one letter is repeated twice and no other letter is
p
2
y
If g(x) = ò [f ¢(t)cosec t – cot t cosec t f(t)]dt for repeated. Then, =
x 9x
48. The sides of a right angled triangle are in arithmetic
æ pù progression. If the triangle has area 24, then what is the
x Î ç 0, ú , then lim g(x) =
è 2û x®0 length of its smallest side?

SECTION - III
This section contains 6 questions of matching type. This section contains two tables each having 3 columns and 4 rows. Based on
each table, there are three questions. Each question has four options (A), (B), (C) and (D) only ONE OF these four option is correct.
(Qs. 49-51) : By appropriately matching the information given in the three columns of the following table.
Column 1, 2, and 3 contain conics, equations of tangents to the conics and points of contact, respectively.
Column 1 Column 2 Column 3

æ a 2a ö
(I) x2 + y2 = a2 (i) my = m2x + a (P) ç 2, ÷
èm m ø

æ - ma a ö
(II) x2 + a2y2 = a2 (ii) y = mx + a (Q) ç , ÷
m2 + 1 ç 2 2 ÷
è m +1 m +1 ø

æ -a 2 m 1 ö
(III) y2 = 4ax (iii) y = mx + 2 2 (R) ç , ÷
a m -1 ç 2 2 2 2 ÷
è a m + 1 a m + 1 ø

æ -a 2 m -1 ö
(IV) x2 – a2y2 = a2 (iv) y = mx + 2 2 (S) ç , ÷
a m +1 ç 2 2 2 2 ÷
è a m - 1 a m - 1 ø

49. For a = 2 , if a tangent is drawn to a suitable conic (Column 1) at the point of contact (– 1, 1), then which of the following
options is the only correct combination for obtaining its equation?
(A) (I) (i) (P) (B) (I) (ii) (Q) (C) (II) (ii) (Q) (D) (III) (i) (P)
50. If a tangent to a suitable conic (column 1) is found to be y = x + 8 and its point of contact is (8, 16), then which of the following
options is the only correct combination?
(A) (I) (ii) (Q) (B) (II) (iv) (R) (C) (III) (i) (P) (D) (III) (ii) (Q)
JEE Advanced 2017 Solved Paper 2017-9

æ 1ö
51. The tangent to a suitable conic (Column 1) at ç 3, ÷ is found to be 3x + 2y = 4, then which of the following options is the
è 2ø
only correct combination?
(A) (IV) (iii) (S) (B) (IV) (iv) (S) (C) (II) (iii) (R) (D) (II) (iv) (R)

(Qs. 52-54) : By appropriately matching the information given in the three columns of the following table.
Let f(x) = x + loge x – x loge x, x Î (0, ¥)
Column 1 contains information about zeros of f(x), f ¢(x) and f ¢¢(x) .
Column 2 contains information about the limiting behaviour of f (x), f ¢(x) and f ¢¢(x) at infinity.
Column 3 contains information about increasing/decreasing nature of f (x) and f ¢(x) .
Column 1 Column 2 Column 3

(I) f (x) = 0 for some x Î (1, e2) (i) lim f (x) = 0 (P) f is increasing in (0, 1)
x®¥

(II) f ¢ (x) = 0 for some x Î (1, e) (ii) lim f (x) = -¥ (Q) f is increasing in (e, e2)
x®¥

(III) f ¢(x) = 0 for some x Î (0, 1) (iii) lim f ¢(x) = -¥ (R) f ¢ is increasing in (0, 1)
x ®¥

(IV) f ¢¢(x) = 0 for some x Î (1, e) (iv) lim f ¢¢(x) = 0 (S) f ¢ is decreasing in (e, e2)
x®¥

52. Which of the following options is the only correct combination?


(A) (I) (i) (P) (B) (II) (ii) (Q) (C) (III) (iii) (R) (D) (IV) (iv) (S)
53. Which of the following options is the only correct combination?
(A) (I) (ii) (R) (B) (II) (iii) (S) (C) (III) (iv) (P) (D) (IV) (i) (S)
54. Which of the following options is the only incorrect combination?
(A) (I) (iii) (P) (B) (II) (iv) (Q) (C) (III) (i) (R) (D) (II) (iii) (P)
EBD_7801
2017-10 JEE Advanced 2017 Solved Paper

PAPER - 2
1. The question paper consists of three parts (Physics, Chemistry and Mathematics). Each part consists of three
sections.
2. Section I contains 7 questions. Each question has four options (a), (b), (c) and (d). ONLY ONE of these four options
is correct.
3. Section II contains 7 multiple choice questions. Each question has four choice (a), (b), (c) and (d) out of which ONE
OR MORE THAN ONE is/are correct.
4. Section III contains 2 paragraphs each describing theory, experiment and data etc. 4 questions relate to two
paragraphs with two questions on each paragraph. Each question pertaining to a particular passage should have
ONLY ONE correct answer among the four given choices (a), (b), (c) and (d).

PHYSICS æ ö
ç 1 ÷
SECTION - I æpö D = hç - 1÷
(A) D = h sin 2 ç ÷ (B) ç æpö ÷
ènø ç cos ç n ÷ ÷
This section contains 7 questions. Each question has 4 options è è ø ø
(A), (B), (C) and (D). ONLY ONE of these four options is correct.
1. Consider an expanding sphere of instantaneous radius R æ 2 pö 2æ p ö
(C) D = hsin ç ÷ (D) D = h tan ç ÷
whose total mass remains constant. The expansion is such è nø è 2n ø
that the instantaneous density r remains uniform throughout 3. A photoelectric material having work–function f0 is
æ 1 drö æ hc ö
the volume. The rate of fractional change in density ç
è r dt ÷ø illuminated with light of wavelength l ç l < f ÷ . The fastest
è 0ø
is constant. The velocity v of any point on the surface of the photoelectron has a de–Broglie wavelength ld. A change in
expanding sphere is proportional to wavelength of the incident light by Dl result in a change Dld
(A) R (B) R3 in ld. Then the ratio Dld/Dl is proportional to
1 (A) ld/l (B) ld2 / l2
(C) (D) R2/3
R
2. Consider regular polygons with number of sides n = 3, 4, 5.... (C) l3d / l (D) l3d / l2
as shown in the figure. The center of mass of all the polygons 4. A symmetric star shaped conducting wire loop is carrying a
is at height h from the ground. They roll on a horizontal steady state current I as shown in the figure. The distance
surface about the leading vertex without slipping and sliding between the diametrically opposite vertices of the star is 4a.
as depicted. The maximum increase in height of the locus of The magnitude of the magnetic field at the center of the loop
the center of mass for each polygon is D. Then D depends is
on n and h as

4a
h h

m0 l m0 l
(A) 6 [ 3 - 1] (B) 6 [ 3 + 1]
4 pa 4 pa
h
m0 l m0l
(C) 3 [ 3 - 1] (D) 3 [2 - 3]
4 pa 4pa
JEE Advanced 2017 Solved Paper 2017-11
r r r
5. Three vectors P, Q and R are shown in the figure. Let S be
r
any point on the vector R . The distance between the points Region 1 y Region 2 Region 3
r r r × × ×
P and S is b | R |. The general relation among vectors P, Q
r × × B ×
and S is
× × ×
× × ×
Y P b |R|
O × × ×
× × × P2 x
P R = Q- P +Q P1
S × × ×
S (y = –R) ×
Q Q × ×
× × ×
O X 3 R/2
r r r
(A) S = (1 - b)P + bQ
r r r 2 p
(B) S = (b - 1)P + bQ (A) For B > , the particle will re–enter region 1
3 QR
r r r
(C) S = (1 - b2 )P + bQ
8 p
r r r (B) For B = , the particle will enter region 3 through
(D) S = (1 - b)P + b 2 Q 13 QR
6. A rocket is launched normal to the surface of the Earth, the point P2 on x–axis
away from the Sun, along the line joining the Sun and the (C) When the particle re–enters region 1 through the
Earth. The Sun is 3 × 105 times heavier than the Earth and is longest possible path in region 2, the magnitude of the
at a distance 2.5 × 104 times larger than the radius of the change in its linear momentum between point P1 and
Earth. The escape velocity from Earth's gravitational field is
ve = 11.2 km s–1. The minimum initial velocity (vs) required the farthest point from y–axis is p/ 2
for the rocket to be able to leave the Sun–Earth system is (D) For a fixed B, particles of same charge Q and same
closest to (Ignore the rotation and revolution of the Earth velocity v, the distance between the point P1 and the
and the presence of any other planet) point of re–entry into region 1 is inversely proportional
(A) vs= 22 km s–1 (B) vs= 42 km s–1 to the mass of the particle
(C) vs= 62 km s –1 (D) vs= 72 km s–1 9. The instantaneous voltages at three terminals marked X, Y
7. A person measures the depth of a well by measuring the and Z are given by
time interval between dropping a stone and receiving the Vx = V0 sin wt,
sound of impact with the bottom of the well. The error in his
measurement of time is dT = 0.01 seconds and he measures æ 2p ö
VY = V0 sin ç wt + ÷ and
the depth of the well to be L = 20 meters. Take the acceleration è 3 ø
due to gravity g = 10 ms–2 and the velocity of sound is 300
ms–1. Then the fractional error in the measurement, dL/L, is æ 4p ö
closest to VZ = V0 sin ç wt + ÷
è 3 ø
(A) 0.2% (B) 1%
An ideal voltmeter is configured to read rms value of the
(C) 3% (D) 5%
potential difference between its terminals. It is connected
SECTION - II between points X and Y and then between Y and Z. The
reading(s) of the voltmeter will be
This section contains 7 questions. Each question has 4 options
(A), (B), (C) and (D). ONE or MORE THAN ONE of these four rms 3
(A) VXY = V0
options is (are) correct. 2

8. A uniform magnetic field B exists in the region between x = 0 rms 1


(B) VYZ = V0
3R 2
and x = (region 2 in the figure) pointing normally into
2 rms
(C) VXY = V0
the plane of the paper. A particle with charge +Q and
momentum p directed along x–axis enters region 2 from region (D) Independent of the choice of the two terminals
1 at point P1 (y = –R). Which of the following option(s)
is/are correct?
EBD_7801
2017-12 JEE Advanced 2017 Solved Paper
10. A point charge +Q is placed just outside an imaginary
hemispherical surface of radius R as shown in the figure. S
Which of the following statements is/are correct?
R
Q + V
– L1 L2

R
(A) After a long time, the current through L1 will be
V L2
(A) The electric flux passing through the curved surface R L1 + L 2

Q æ 1 ö (B) After a long time, the current through L2 will be


of the hemisphere is - ç1 - ÷
2e0 è 2ø V L1
(B) Total flux through the curved and the flat surfaces is R L1 + L 2
Q (C) The ratio of the currents through L1 and L2 is fixed at
e0 all times (t > 0)
(C) The component of the electric field normal to the flat V
surface is constant over the surface (D) At t = 0, the current through the resistance R is
R
(D) The circumference of the flat surface is an equipotential
11. Two coherent monochromatic point sources S1 and S2 of 13. A rigid uniform bar AB of length L is slipping from its vertical
wavelength l = 600 nm are placed symmetrically on either position on a frictionless floor (as shown in the figure).
side of the centre of the circle as shown. The sources are At some instant of time, the angle made by the bar with the
separated by a distance d = 1.8 mm. This arrangement vertical is q. Which of the following statements about its
produces interference fringes visible as alternate bright and motion is/are correct?
dark spots on the circumference of the circle. The angular
separation between two consecutive bright spots is Dq.
Which of the following options is/are correct?
A
P1 q
L

Dq
B
P2
S1 S2 O
d
(A) The midpoint of the bar will fall vertically downward
(B) The trajectory of the point A is a parabola
(C) Instantaneous torque about the point in contact with
(A) A dark spot will be formed at the point P2 the floor is proportional to sinq
(B) At P2 the order of the fringe will be maximum (D) When the bar makes an angle q with the vertical, the
(C) The total number of fringes produced between P1 and displacement of its midpoint from the initial position is
P2 in the first quadrant is close to 3000 proportional to (1 – cosq)
(D) The angular separation between two consecutive 14. A wheel of radius R and mass M is placed at the bottom of a
bright spots decreases as we move from P1 to P2 along fixed step of height R as shown in the figure. A constant
the first quadrant
force is continuously applied on the surface of the wheel so
12. A source of constant voltage V is connected to a resistance
that it just climbs the step without slipping. Consider the
R and two ideal inductors L1 and L2 through a switch S as
shown. There is no mutual inductance between the two torque t about an axis normal to the plane of the paper passing
inductors. The switch S is initially open. At t = 0, the switch through the point Q. Which of the following options is/are
is closed and current begins to flow. Which of the following correct?
options is/are correct?
JEE Advanced 2017 Solved Paper 2017-13

S V
V0 Process 1

Q 2V0 /3
P Process 2
R V0 /3 T > > RC
X

T 2T t

Figure 2
(A) If the force is applied at point P tangentially then 15. In Process 1, the energy stored in the capacitor E C and heat
decreases continuously as the wheel climbs dissipated across resistance ED are related by :
(B) If the force is applied normal to the circumference at (A) EC = ED (B) EC = ED In 2
point X then t is constant 1
(C) If the force is applied normal to the circumference at (C) EC = ED (D) EC = 2ED
2
point P then t is zero
(D) If the force is applied tangentially at point S then t ¹ 0 16. In Process 2, total energy dissipated across the resistance
but the wheel never climbs the step ED is :
1 æ1 2ö
SECTION - III (A) ED = CV02 (B) ED = 3 ç CV0 ÷
2 è2 ø
This section contains 2 paragraphs, each describing theory,
experiments, data etc. four questions related to the two paragraphs 1æ 1 2ö
with two questions on each paragraph. Each question has only (C) ED = ç CV0 ÷ (D) ED = 3CV02
one correct answer among the four given options (A), (B), (C) and 3è 2 ø
(D). PARAGRAPH 2
PARAGRAPH 1 One twirls a circular ring (of mass M and radius R) near the tip of
Consider a simple RC circuit as shown in Figure 1. one's finger as shown in Figure 1. In the process the finger never
Process 1: In the circuit the switch S is closed at t = 0 and the loses contact with the inner rim of the ring. The finger traces out
capacitor is fully charged to voltage V0 (i.e., charging continues the surface of a cone, shown by the dotted line. The radius of the
for time T >> RC). In the process some dissipation (ED) occurs path traced out by the point where the ring and the finger is in
across the resistance R. The amount of energy finally stored in contact is r. The finger rotates with an angular velocity w0. The
the fully charged capacitor is EC. rotating ring rolls without slipping on the outside of a smaller
V0 circle described by the point where the ring and the finger is in
Process 2: In a different process the voltage is first set to and contact (Figure 2). The coefficient of friction between the ring and
3
the finger is m and the acceleration due to gravity is g.
maintained for a charging time T >> RC. Then the voltage is raised
2V0
to without discharging the capacitor and again maintained
3
for a time T >> RC. The process is repeated one more time by R
raising the voltage to V0 and the capacitor is charged to the same r
final voltage V0 as in Process 1. R
These two processes are depicted in Figure 2.
Figure 1
S
Figure 2
R
17. The total kinetic energy of the ring is
V + C
– 1
(A) Mw02 R 2 (B) Mw20 (R - r)2
2
3
Figure 1 (C) Mw20 (R - r) 2 (D) Mw20 (R - r)2
2
EBD_7801
2017-14 JEE Advanced 2017 Solved Paper
18. The minimum value of w0 below which the ring will drop
down is
1 Wat
g 2g er
(A) (B)
m(R - r) m(R - r) Ice

V.P./bar
(D)
3g g
(C) (D) Water + E
2m(R - r) 2m(R - r) than ol
271 273
T/K
CHEMISTRY
20. For the following cell,
SECTION - I Zn(s) | ZnSO4(aq) || CuSO4(aq) | Cu(s)
This section contains 7 questions. Each question has 4 options when the concentration of Zn2+ is 10 times the concentration
(A), (B), (C) and (D). ONLY ONE of these four options is correct. of Cu2+, the expression for DG (in J mol–1) is [F is Faraday
constant; R is gas constant; T is temperature; E° (cell) = 1.1 V]
19. Pure water freezes at 273 K and 1 bar. The addition of 34.5 g
(A) 1.1F (B) 2.303RT – 2.2F
of ethanol to 500 g of water changes the freezing point of the
solution. Use the freezing point depression constant of water (C) 2.303RT + 1.1F (D) – 2.2F
as 2 K kg mol–1. The figures shown below represent plots of 21. The standard state Gibbs free energies of formation of
vapour pressure (V.P.) versus temperature (T). [molecular C(graphite) an d C(diamond) at T = 298 K ar e
weight of ethanol is 46 g mol–1] Among the following, the DƒG° [C(graphite)] = 0 kJ mol–1
option representing change in the freezing point is DƒG° [C(diamond)] = 2.9 kJ mol–1
The standard state means that the pressure should be 1 bar,
1 and substance should be pure at a given temperature. The
Water conversion of graphite [C(graph ite)] to diamond
Ice [C(diamond)] reduces its volume by 2 × 10–6 m3 mol–1. If
V.P./bar

C(graphite) is converted to C(diamond) isothermally at


(A) T = 298 K, the pressure at which C(graphite) is in equilibrium
Water + Ethanol
with C(diamond), is
270 273 [Useful information : 1 J = 1 kg m2s–2; 1 Pa = 1 kg m–1 s–2; 1
T/K bar = 105 Pa]
(A) 14501 bar (B) 58001 bar
(C) 1450 bar (D) 29001 bar
ter 22. Which of the following combination will produce H2 gas?
Wa (A) Fe Metal and conc. HNO3
V.P/bar

1 ol (B) Cu metal and conc. HNO3


(B) Ethan
ice er + (C) Zn metal and NaOH(aq)
Wat (D) Au metal and NaCN(aq) in the presence of air
23. The order of the oxidation state of the phosphorus atom in
271 273
H3PO2, H3PO4, H3PO3 and H4P2O6 is
T/K
(A) H3PO3 > H3PO2 > H3PO4 > H4P2O6
(B) H3PO4 > H3PO2 > H3PO3 > H4P2O6
(C) H3PO4 > H4P2O6 > H3PO3 > H3PO2
ter (D) H3PO2 > H3PO3 > H4P2O6 > H3PO4
Wa
(C) 24. The major product of the following reaction is
V.P/bar

l
1 ano
ice + Eth OH
er
Wat
i) NaNO ,HCl,0°C
270 273 ¾¾¾¾¾¾¾
2 ¾
®
ii) aq.NaOH
T/K
NH2
JEE Advanced 2017 Solved Paper 2017-15

(B) With increase in temperature, the value of K for


OH
endothermic reaction increases because unfavourable
change in entropy of the surroundings decreases
(A) (C) With increase in temperature, the value of K for
endothermic reaction increases because the entropy
Cl change of the system is negative
(D) With increase in temperature, the value of K for
–+
ONa exothermic reaction decreases because favourable
change in entropy of the surroundings decreases
(B) 28. In a bimolecular reaction, the steric factor P was experimentally
determined to be 4.5. The correct option(s) among the
N2Cl following is(are)
(A) The activation energy of the reaction is unaffected by
the value of the steric factor
OH
(B) Experimentally determined value of frequency factor
is higher than that predicted by Arrhenius equation
(C) (C) Since P = 4.5, the reaction will not proceed unless an
effective catalyst is used
N=N (D) The value of frequency factor predicted by Arrhenius
equation is higher than that determined experimentally
N=N
OH 29. For the following compounds, the correct statement(s) with
respect to nucleophilic substitution reaction is(are)
(D)
Br Br

25. The order of basicity among the following compounds is


NH NH2 (I) (II)

N NH HN N CH3
H3C NH2 H2N NH CH3
I II III IV H3C—C — Br Br
(A) II > I > IV > III (B) IV > II > III > I
(C) IV > I > II > III (D) I > IV > III > II CH3

(III) (IV)
SECTION - II
(A) I and III follow SN1 mechanism
This section contains 7 questions. Each question has 4 options
(B) I and II follow SN2 mechanism
(A), (B), (C) and (D). ONE or MORE THAN ONE of these four
options is (are) correct. (C) Compound IV undergoes inversion of configuration
(D) The order of reactivity for I, III and IV is : IV > I > III
26. The correct statement(s) about surface properties is(are) 30. Among the following, the correct statement(s) is(are)
(A) Adsorption is accompanied by decrease in enthalpy (A) Al(CH3)3 has the three–centre two–electron bonds in
and decrease in entropy of the system its dimeric structure
(B) The critical temperatures of ethane and nitrogen are
(B) BH3 has the three–centre two–electron bonds in its
563 K and 126 K, respectively. The adsorption of
ethane will be more than that of nitrogen on same dimeric structure
amount of activated charcoal at a given temperature (C) AlCl3 has the three–centre two–electron bonds in its
(C) Cloud is an emulsion type of colloid in which liquid is dimeric structure
dispersed phase and gas is dispersion medium (D) The Lewis acidity of BCl3 is greater than that of AlCl3
(D) Brownian motion of colloidal particles does not depend 31. The option(s) with only amphoteric oxides is(are)
on the size of the particles but depends on viscosity (A) Cr2O3, BeO, SnO, SnO2
of the solution (B) Cr2O3, CrO, SnO, PbO
27. For a reaction taking place in a container in equilibrium with (C) NO, B2O3, PbO, SnO2
its surroundings, the effect of temperature on its equilibrium
(D) ZnO, Al2O3, PbO, PbO2
constant K in terms of change in entropy is described by
(A) With increase in temperature, the value of K for 32. Compounds P and R upon ozonolysis produce Q and S,
exothermic reaction decreases because the entropy respectively. The molecular formula of Q and S is C8H8O. Q
change of the system is positive undergoes Cannizzaro reaction but not haloform reaction,
whereas S undergoes haloform reaction but not Cannizzaro
reaction
EBD_7801
2017-16 JEE Advanced 2017 Solved Paper
i) O /CH Cl COCH3
(i) P ¾¾¾¾¾¾
3 2 2
ii) Zn/H O
® Q
2 (C8 H8O)
(H3C)3C CH3
i) O / CH Cl
(ii) R ¾¾¾¾¾¾
3 2 2® S (B)
ii) Zn / H 2O (C8H8O)
The option(s) with suitable combination of P and R,
respectively, is(are) H3COC

(H3C)3C H3C CH3


(A) H3C and
CH 3 (C)
CH3 H3C
CH3
and HO3S
(B)
H3C CH3 O CH3
(H3C)3C
H3C (D)
CH3 CH3
CH3
(C) and
CH3 CH3 COCH3
36. The reactions, Q to R and R to S, are
H3C (A) Dehydration and Friedel–Crafts acylation
(D) H3C and
(B) Aromatic sulfonation and Friedel–Crafts acylation
CH3 (C) Friedel–Crafts alkylation, dehydration and Friedel–
SECTION - III Crafts acylation
(D) Friedel–Crafts alkylation and Friedel–Crafts acylation
This section contains 2 paragraphs, each describing theory,
experiments, data etc. four questions related to the two paragraphs
with two questions on each paragraph. Each question has only MATHEMATICS
one correct answer among the four given options (A), (B), (C)
and (D). SECTION - I
This section contains 7 questions. Each question has 4 options
PARAGRAPH 1 (A), (B), (C) and (D). ONLY ONE of these four options is correct.
Upon heating KClO3 in the presence of catalytic amount of MnO2,
a gas W is formed. Excess amount of W reacts with white phosphorus 37. The equation of the plane passing through the point (1, 1, 1)
to give X. The reaction of X with pure HNO3 gives Y and Z. and perpendicular to the planes 2x + y – 2z = 5 and 3x – 6y –
33. W and X are, respectively 2z = 7, is
(A) O3 and P4O6 (B) O2 and P4O6 (A) 14x + 2y – 15z = 1
(C) O2 and P4O10 (D) O3 and P4O10 (B) 14x – 2y + 15z = 27
34. Y and Z are, respectively (C) 14x + 2y + 15z = 31
(A) N2O3 and H3PO4 (B) N2O5 and HPO3 (D) –14x + 2y + 15z = 3
(C) N2O4 and HPO3 (D) N2O4 and H3PO3 38. Let O be the origin and let PQR be an arbitrary triangle. The
point S is such that
uuur uuur uuur uuur uuur uuur uuur uuur uuur uuur uuur uuur
PARAGRAPH 2 OP.OQ + OR.OS = OR.OP + OQ.OS = OQ.OR + OP.OS
The reaction of compound P with CH3MgBr (excess) in (C2H5)2O Then the triangle PQR has S as its
followed by addition of H2O gives Q. The compound Q on treatment (A) Centroid (B) Circumcentre
with H2SO4 at 0ºC gives R. The reaction of R with CH3COCl in the (C) Incentre (D) Orthocenter
presence of anhydrous AlCl3 in CH2Cl2 followed by treatment with 39. If y = y(x) satisfies th e differential equation
H2O produces compound S. [Et in compound P is ethyl group]
( )
-1

(H3C)3C CO2Et Q R S 8 x ( 9+ )
x dy = 4+ 9+ x dx, x > 0 and
y (0) = 7 , then y (256) =
(A) 3 (B) 9
P (C) 16 (D) 80
35. The product S is 40. If f : R ® R is a twice differentiable function such that
H3C æ1ö 1
f ¢¢ (x) > 0 for all x Î R, and f f ç ÷ = , f(1) = 1, then
CH3 è2ø 2
(H3C)3C
1
(A) f ¢ (1) £ 0 (B) 0 < f ¢(1) £
(A) 2
1
(C) < f ¢(1) £ 1 (D) f ¢ (1) > 1
COCH3 2
JEE Advanced 2017 Solved Paper 2017-17

41. How many 3 × 3 matrices M with entries from {0, 1, 2} are


cos(2x) cos(2x) sin(2x)
there, for which the sum of the diagonal entries of MT M is 5?
(A) 126 (B) 198 48. If f(x) = - cos x cos x - sin x , then
(C) 162 (D) 135 sin x sin x cos x
42. Let S = {1, 2, 3, ..., 9}. For k = 1, 2, ..., 5, let Nk be the number
of subsets of S, each containing five elements out of which (A) f ¢(x) = 0 at exactly three points in (–p, p)
exactly k are odd. Then N1 + N2 + N3 + N4 + N5 = (B) f ¢(x) = 0 at more than three points in (–p, p)
(A) 210 (B) 252 (C) f(x) attains its maximum at x = 0
(C) 125 (D) 126 (D) f(x) attains its minimum at x = 0
43. Three randomly chosen non–negative integers x, y and z are 49. If the line sx = a divides the area of region
found to satisfy the equation x + y + z = 10. Then the
probability that z is even, is R = {(x, y) Î R 2 : x3 £ y £ x, 0 £ x £ 1} into two equal
36 6 parts, then
(A) (B) 1
55 11 0<a£
(A)
1 5 2
(C) (D) 1
2 11 (B) < a <1
2
SECTION - II
This section contains 7 questions. Each question has 4 options
(C) 2a 4 - 4 a 2 + 1 = 0
(A), (B), (C) and (D). ONE or MORE THAN ONE of these four (D) a 4 + 4a 2 - 1 = 0
options is (are) correct.
k +1 k +1
å k =1 òk
98
50. If I = dx , then
sin(2x) x(x + 1)
44. If g(x) = ò sin -1 (t)dt, then
sin x (A) l > loge 99 (B) l < loge 99
æpö æ pö 49 49
(A) g¢ ç ÷ = -2p (B) g¢ ç - ÷ = 2p (C) 1< (D) l >
è2ø è 2ø 50 50
æpö æ pö SECTION - III
(C) g¢ ç ÷ = 2p (D) g¢ ç - ÷ = -2p
2
è ø è 2ø This section contains 2 paragraphs, Based on each paragraph,
45. Let a and b be non–zero real numbers such that there are 2 questions. Each question has four options (A), (B), (C)
2(cosb – cosa) + cosa cosb = 1. Then which of the following and (D) ONLY ONE of these four options is correct.
is/are true?
PARAGRAPH 1
æaö æbö uuur uuur uuur
(A) tan ç ÷ + 3 tan ç ÷ = 0 Let O be the origin, and OX, OY, OZ be three unit vectors in the
è2ø è2ø uuur uuur uuur
directions of the sides QR, RP, PQ respectively, of a triangle
æaö æbö
(B) 3 tan ç ÷ + tan ç ÷ = 0 PQR.
è2ø è2ø uuur uuur
51. | OX ´ OY | =
æaö æbö
(C) tan ç ÷ - 3 tan ç ÷ = 0 (A) sin (P + Q) (B) sin 2R
è2ø è2ø (C) sin (P + R) (D) sin (Q + R)
æaö æbö 52. If the triangle PQR varies, then the minimum value of
(D) 3 tan ç ÷ - tan ç ÷ = 0 cos(P + Q) + cos (Q +R) + cos (R + P) is
è2ø è2ø
46. If f : R ® R is a differentiable function such that f ¢(x) > 2f(x) 5 3
(A) - (B) -
for all x Î R, and f(0) = 1, then 3 2
(A) f(x) is increasing in (0,¥) 3 5
(B) f(x) is decreasing in (0, ¥) (C) (D)
(C) f(x) > e2x in (0, ¥) 2 3
(D) f ¢(x) < e2x in (0, ¥) PARAGRAPH–2
Let p, q be integers and let a, b be the roots of the equation, x2 –
1 - x(1+ | 1 - x |) æ 1 ö x – 1 = 0, where a ¹ b. For n = 0, 1, 2, ...., let an = pan + q bn.
47. Let f(x) = cos ç ÷ for x ¹ 1. Then
| 1- x | è 1- x ø FACT : If a and b are rational numbers and a +b 5 = 0, then
(A) lim f (x) = 0 a = 0 = b.s
x ®1-
53. a12 =
(B) lim f (x) does not exist (A) a11 – a10 (B) a11 + a10
x ®1-
(C) 2a11 + a10 (D) a11 + 2a10
(C) lim f (x) = 0
x ®1+ 54. If a4= 28, then p + 2q =
lim f (x) does not exist (A) 21 (B) 14
(D) x ®1+ (C) 7 (D) 12
EBD_7801
2017-18 JEE Advanced 2017 Solved Paper

SOLUTIONS
Paper - 1
PHYSICS 1 2 1
1. (A, B, D) mgR = mv + MV 2
2 2

v Flat plate Flat plate 2 gR


+ – v On solving we get, v = m
1+
M

u u v1 v2 m 2 gR
\V=
M m
1+
M
Before collision Just after collision \ (B) is the correct option.
v 1 T
v -v v + v2 3. (A, D) We know that l = =
1= 1 1= f f m
v +u u–v
Where T = tension of string.
\ v1 = u + 2v \ v2 = u - 2v Here To > TA \ lo > lA so option (C) is wrong.
\ Dv1 = 2u + 2v and Dv2 = 2u - 2v Velocity being a vector quantity has direction. The
velocities of the two pulses cannot be same at
dp1 midpoint.
Now F1 = = rA(u + v )(2u + 2v )
dt TAo = T oA because speed (or velocity) of wave
depends on mediums (and not on the wavelength or
dp2 frequency of wave)
and F2 = = rA(u - v)(2 u - 2 v)
dt (A), (D) are the correct options
4. (C)
\ F1 = 2rA(u + v) 2 and F2 = 2rA(u - v )2
Energy radiated = sA(T 4 - T0 4 ) t
[For a black body e = 1]
F2 DF
P= = 8ruv = sA[(T0 + 10)4 - T0 4 ]t
F1 DF = F1 - F2 = 8rAuv \ A
é 4 ù
4 æ 10 ö
The net force Fnet = F - DF = ma = sAT ê
0 ç 1 + ÷ - 1ú t
êëè T0 ø úû
\ F - 8rAuv = ma
2. (B, C) Let the block be displaced by x . If initially the centre 4 é 40 ù 40
of mass of the system is at origin then = sAT0 ê T ú ´ t = 460 ´1´ 300 ´1 = 61.33J
ë 0û
M ´ x + m( x + R )
O= Energy radiated
M +m P= = sAT 4 - sAT0 4
time
- mR
O = Mx + mx + mR \ x= dp 3
m+M \ dT = sA(4T0 ) \ dp = sA(4T03 )dT0
0
\ 'C' is the correct option
If v is the velocity of mass 'm' as it leaves the block
\ DP = 4sAT03
and V is the velocity of block at that instant then
according to conservation of linear momentum A, B are not correct options as human body is not a
mv = MV black body.
By energy conservation Energy radiated µ A where A is the surface area of
the body
\ 'C' is the correct option
JEE Advanced 2017 Solved Paper 2017-19

5. (B, D) For smaller loop f = BA cos wt


sin 90° 1 -1 æ 1 ö
The rate of change of flux m= = Þ r2 = sin ç ÷
sin r2 sin r2 èmø
df
= – BAw sin wt
dt A
df
For to be maximum, sin wt should be maximum
dt i1 P Q
and this will happen when wt = 90° i.e., the plane of r2
loop is perpendicular to the plane of paper.
Option (D) is correct. B C
The emf produced will oppose each other. The net Emergent ray tangential
emf will also be proportional to sin wt. to the surface
enet = B(2A)wsin wt – BAw sin wt = BAw sin wt
\ option (B) is also correct. But r1 + r2 = A
6. (B, C) The angular frequency at which the current and \ r1 = A – r2
voltage will be at same phase is
-1 æ 1 ö
\ r1 = A – sin ç ÷
wr =
1
=
1
= 106 rad s –1 èmø
LC (10-6 ´10-6 )1/2 Applying Snell's law at 'P' we get
This value is independent of 'R' So (B) is correct sin i1 -1 é -1 1 ù
option. m= \ i1 = sin êm sin( A - sin )
sin r1 ë m úû
V (D) is correct option.
At w » 0 , the current i =
1 2 For minimum deviation PQ || BC
R 2 + (w L - )
wC \ (A) is also a correct option.
i = 0 (The circuit behaves as d.c circuit) 8. (6) DU = S[k ´ 4pr 2 - 4pR 2 ]
\ C is a correct option
é 4 3 4 3ù
If w >> w0, circuit behaves as an inductor. êë where 3 pR = k ´ 3 pr úû
7. (A, C, D) 1
For minimum deviation (when i1 = A) \ R=K 3r

i1 = e
é R2 ù
\ DU = 4ps ê k ´ 2/3 - R ú = 4pSR éëk - 1ùû
2 2 1/3
A
r1 = r2 = r (say) = ë k û
2

A \ DU = 4pSR 2 éë10a /3 - 1ùû


0.1
\ 10–3 = 4p ´ ´ (10-2 ) 2 éë10a /3 - 1ùû
Q 4p
i1 P e
r1 r2 \ 10 2 = 10 a /3 - 1
a
B C Neglecting 1 we get 102 = 10a/3 \ =2 \ a=6
3
dm = 2i1 - A ,
n 2 n
Here d m = A \ i1 = A \ e=A 9. (5) Here U i = f = 6.25 \ f = 2.5
Uf 2 ni
ni
i1 If ni = 2 then n f = 5
\ r1 =
2 10. (8) Here n × sin 30° = [n – m × 0.1] sin 90°
option (C) is correct \ 1.8 × sin 30° = 1.8 – m × 0.1 \ m = 8
sin i1 sin A é 332 ù
m= = 11. (6) Frequency perceived by reflector = f1 = 492 ê
sin r1 sin A / 2 ë 330 úû
Frequency perceived by the source f2
2sin A / 2 cos A / 2
= = 2 cos A / 2
sin A / 2 é 332 ù é 330 ù
= 492 ê ´ = 498Hz
Applying Snell's law at Q ë 330 úû êë 328 úû
\ Beat frequency = 498 – 492 = 6Hz
EBD_7801
2017-20 JEE Advanced 2017 Solved Paper
12. (5) A = A0 e–lt \ A0 = Aelt = 115 (1 + lt)

é ln 2 ù 15. (B) x
\ A0 = 115 ê1 + ´ 11.5ú
ë t1/ 2 û
E = - E0 ˆj
é 0.7 ù
= 115 ê1 + ´ 11.5ú vel = 0
ë 8 ´ 24 û

A0 » 120 Bq Z
120 Bq activity level is in 2.5 ml
\ 2.4 × 10 5 Bq activity level will be in B = B0 ˆj

Y
2.5 ´ 2.4 ´105
ml
120
The electric field will apply a force on –Y axis thereby
2.5 ´ 2.4 ´ 105 accelerating the charge along –Y axis.
l = 5l
120 ´1000 FB = qv B sin q
13. (D) For the particle to move in straight line , electric force Here q = 180° therefore , FB = 0
should be equal and opposite to the magnetic force. 16. (D) DU = DQ – PDV
(D) is the correct option. show that work done = PDV
r r which is the formula for isobaric process.
FE = -eE = -e(- E0 xˆ ) = eE0 xˆ
17. (B) Work done in isochoric process is zero for which we
r r get a vertical line in P-V graph.
éE ù
FB = q(v ´ B) = -e ê 0 yˆ ´ B0 zˆ ú 18. (D) Laplace's correction of the speed of sound in ideal
ë B0 û gas is related to adiabatic process.
r CHEMISTRY
FB = -eE0 xˆ
19. (A, B, C)
(A) During adiabatic expansion, the final temperature is
x less than the initial temperature. Therefore the final
14. (A)
volume in adiabatic expansion will also be less than
2E0 the final volume in isothermal expansion. This can be
xˆ FB
B0 graphically shown as

B P
Z
E
reversible isothermal
reversible adiabatic
Y

The force due to magnetic field FB will provide the


necessary centripetal force for circular motion which The magnitude of work done by the gas is equal to the
will be in X-Y plane. The force due to electric field area under the curve. As seen from the figure the area
will accelerate proton in Z-direction. Thus the path under curve in reversible isothermal is more. Hence,
will be helical with increasing pitch. the magnitude of work done is lesser in adiabatic
reversible expansion as compared to the corresponding
work in isothermal expansion.
JEE Advanced 2017 Solved Paper 2017-21

(B) In free expansion, Pext = 0 \ W = 0 (C) Cl2 + H2O ® HCl + HOCl


If carried out isothermally (DU = 0) Þ q = 0 (Adiabatic) ;
(D) HClO4 + H 2 O ® ClO4- + H3O +
From I law Acid 1 Base 1 Base 1 acid 1
If carried out adiabatically (q = 0) Þ DU = 0 Since H2O is accepting H+
from HClO4 so H2O is
(Isothermal) ; From I law stronger base compared to ClO-4 .
(C) P 22. (C, D)
(P1, V1) The colour of X2 molecules of halogens is due to
absorption of light in the visible region. The energy
acquired in this manner excites the valence electron
(P2, V2)
from the highest occupied molecular orbital (HOMO)
to the lowest unoccupied molecular orbital (LUMO),
V i.e., transition from p* to s* molecular orbital.
During irreversible compression, maximum work is done
X : s1s , s*1s , s2s , s*2s , s2pz , p2px
2 2 2 2 2 2
on the gas (corresponding to shaded area) 2
(D) When T1 = T2 Þ DU = nCVDT = 0
In reversible adiabatic expansion, T2 < T1. p * 2 p 2x = p * 2 p 2y
= p2py2, , s * 2 pz0
\ DT = –ve \ DU = –ve ( HOMO )
20. (B, D)
a ( LUMO )
PL
23. (B, C, D)
Z
Excess NH (aq.)
[Co(H O) ] Cl 2 6 2 ¾ ¾NH ¾ ¾® [Co(NH 3 )6 ] Cl3
¾4Cl¾+ 3Air
(X) (Y)
PL (ideal) Pink Solution 1 : 3 electrolyte
2 3
1 0 HCl (excess) (d sp hybridisation of central
XM
0 1 room temperature metal atom as NH3 is a strong
XL
The graph representing +ve deviation from Raoult’s field ligand)

law therefore M – L < M – M or L – L [CoCl4] 3 AgNO3 (aq)

PL ³ P°L XL (Z)
Blue colour (tetrahedral: 3 AgCl (3 eq of AgCl)
but when XL = 1, mixture has almost pure liquid L so PL
= P°L sp3 as Cl¯ is a weak field ligand)
21. (A, B, D) [CoCl4] + 6H2O
2– 2+
Co(H 2O)6 + 4Cl¯
(A) In both the acids central atom is sp3 hybridized. (Blue) (Pink)
O X and Z in equilibrium at 0°C Þ then equilibrium is
|| sp 3 H–O–Cl shifted towards X, making colour of solution pink.
Cl
HO || sp 3
||

(B) ClO-4 is resonance stabilized anion


HClO4 ¾® ClO-4 + H+
HClO ¾® ClO¯ + H+
Hence HClO4 is more acidic than HClO.

24. (A, B)
IUPAC name : 1-chloro-4-methylbenzence
Cl CH3
1 1
2 2
(A) (B) 4-chlorotoluene
3 3

4 4

CH3 Cl
EBD_7801
2017-22 JEE Advanced 2017 Solved Paper
25. (B, D)
(B) Bromination proceeds through trans-addition in both the reactions.
M and N are identical, hence, M and O and N and P are two set of diastereomers.

(D)
Br
CH 3
H
CH 3 H Br
H Br
CH 3 H H Br
a
Br—Br CH 3 H CH 3 CH 3
Br
b (N)
CH3 CH3
H
a

H Br
b
trans-2-Butene trans-Bromonium ion H CH 3 CH 3
Br Br H
Br
Br H
CH 3 H CH3
(M)
M and N are same
(meso-2, 3-Dibromobutane)

Br
CH 3
H CH 3 H Br
H Br H
CH 3 Br H
c
Br—Br CH3 H CH 3 CH 3
Br
(O)
d
H H
CH 3
c
CH3 Br
d H CH 3
cis-2-Butene cis -Bromonium ion CH3

Br Br H
Br H Br
H CH 3 CH 3
(P)
II and III are enantiomers
rac-2, 3-Dibromobutane
JEE Advanced 2017 Solved Paper 2017-23

Z´M N2 :
2
s1s2, s*1s2, s2s2, s*2s2, p2p x = p 2p2y , s 2p2z
26. (2) Density (d) = 3 (d = density)
a ´ NA (Diamagnetic)
For FCC, Z = 4 2 2
Given a = 4 × 10–8 cm O2- : s1s2, s*1s2, s2s2, s*2s2, p 2pz , p2p x
4´ M = p2p2y , p * 2p x2 = p *2p1y
8=
( 4 ´ 10 ) ´ 6 ´ 10
-10 3 23
(Paramagnetic)
2 2
s1s2, s*1s2, s2s2, s*2s2, s2pz , p2p x
8 ´ ( 4 ´ 10 ) ´ 6 ´ 10
-8 3 23
F2 :
M= = p2p2y , p * 2p x2 = p *2p2y
4
(Diamagnetic)
8 ´ 6 ´ 1023 ´ 64 ´ 10 -24
=
4
30. (5)
wt
No. of atoms = ´ NA Non-aromatic Anti-aromatic Non-aromatic Anti-aromatic
Molar mass
256 ´ 10 ´ 6 ´ 10 23
= 2 × 1024
8 ´ 6 ´ 16
\ Value of N = 2
a Aromatic Aromatic Aromatic
27. (6) The formula for conductance is G = k ´
l
1
5 ´ 10 -7 = k ´
120
k = 6 × 10–5 s cm–1
k ´ 1000 6 ´ 10 -5 ´ 1000
L cm = = = 40
M 0.0015 Aromatic Aromatic
Q pH = 4 31. (C) 1s wave function for He+ is given by
\ [H+] = 10–4 = ca = 0.0015 a
10-4 Z -
æ Zr ö
a= Y(1s) = Yn, l, ml = exp çè a 0 ÷ø
0.0015 pa 0 3/2

L cm 10-4
a= Þ = 40 3/ 2 é Zr ù
L om 0.0015 æZö -ê ú
or µ ç ÷ exp ë a0 û
è a0 ø
L om = 6 ´ 102 s cm 2 mole -1
i.e., z = 6 i.e., it is independent of cos q.
The probability of finding an electron at zero distance
28. (6) Species Number of lone pairs from the nucleus is zero. The probability increases
on central atom gradually as the distance increases, goes to maximum
(i) [TeBr6]2– 1 and then begins to decrease.
(ii) [BrF2]+ 2 32. (A) For a given orbital with principal quantum number, n
(iii) SNF3 0 and azimuthal quantum numbr, l.
(iv) [XeF3]– 3
Number of radial nodes = (n – l – 1)
Total number of lone pairs = 1 + 2 + 0 + 3 = 6
29. (6) (H2, Cl2, Be2, C2, N2, F2)
H2 : s1s2 (Diamagnetic)
He +2 : s1s2, s*1s1 (Paramagnetic)
Y n,l,m (r)

Li2 : s1s2, s*1s2, s2s2 (Diamagnetic)


Be2 : s1s2, s*1s2, s2s2, s*2s2 (Diamagnetic)
1
B2 : s1s2, s*1s2, s2s2, s*2s2, p2p x = p2p1y r

(Paramagnetic) a0
2
C2 : s1s2, s*1s2, s2s2, s*2s2, p2p x = p2p2y
(Diamagnetic)
EBD_7801
2017-24 JEE Advanced 2017 Solved Paper
33. (C) Refer ans 31.
Energy needed to excite from n = 2 to n = 4
3
DE2 – 4 = 13.6 Z2 × eV
16
Similarly,
8
DE2 – 6 = 13.6 Z2 × eV
36
on dividing
DE 2 - 4 3 36 27
= ´ =
DE 2 -6 16 8 32
34. (A) Alkylbenzenes when treated with Br2 at high temperature, in the presence of sunlight and absence of halogen carrier
undergo halogenation in the side chain. Thus
CH3 CH2 Br CHBr2 CBr3

Br2 / heat Br2 Br2


¾¾¾¾® ¾¾¾¾® ¾¾¾¾®
light heat, light heat, light
Toluene Benzyl bromide Benzyl trichloride

35. (C)

36. (D) Perkin condensation of benzaldehyde with (CH3CO)2O/CH3COOK yields cis and trans form of cinnamic acid.
(i) CH 3 COONa
C6H5CHO + (CH3CO)2O ¾¾¾¾¾®   C6H5CH = CHCOOH
Benzaldehyde Acetic anhydride (ii) H+ Cinnamic acid
cis and trans

MATHEMATICS Þ yk – 8(x + h) = k2 – 16h


37. (A, B, C) Þ 8x – ky = 8h – k2 …(1)
\ 2x – y + 1 = 0 i.e. y = 2x + 1 is a tangent to hyperbola But given, the equation of chord is
2x + y = p …(2)
x2 y2 \ (1) and (2) are identical lines
- =1
a 2 16 8 -k 8h - k 2
\ c2 = a2m2 – b2 Þ = =
12 = a2 × 22 – 16 2 1 p
Þ k = –4 and 8h – 16 = 4p
17 Þ k = –4 and p = 2h – 4
Þ a2 =
4 which are satisfied by option (C).
39. (A, C, D)
17 Let x = n be any integer not equal to zero.
Þ a=
2 Then
\ a, 4, 1; a, 4, 2; 2a, 8, 1 lim - x cos(p (x + [x]))
x®n
17 17 = n cos(p(n + n – 1))
i.e. , 4, 1; , 4, 2; 17 , 8, 1
2 2 = n cos (2n – 1)p = –n
cannot be the sides of a right triangle.
lim+ x cos(p (x + [x]))
38. (C) If (h, k) is the mid point of chord of parabola y2 = 16x, x®n
then equation of chord will be given by = n cos(p(n + [n]))
T = S1 = n cos(p(n + n))
JEE Advanced 2017 Solved Paper 2017-25
= n cos 2np = n 2
LHL ¹ RHL Þ limit does not exist at any non P(X Ç Y) 15 4
zero integer n. = =
P(Y) = P(X / Y) 1 15
\ f is discontinuous at x = –1, 1, 2
At x = 0, LHL = RHL = 0 = f(0) 2
\ f is continuous at x = 0. P(X ' Ç Y) P(Y) - P(X Ç Y)
40. (A, B) Let us check the given options one by one. P(X'/Y) = =
(A) Let g(x) = x9 – f(x) P(Y) P(Y)
Þ g(0) = – f(0) < 0 \ f(x) Î (0, 1) = 1 – P(X/Y)
Also g(1) = 1 – f(1) > 0 1
\ x9 – f(x) = 0 for some x Î (0, 1) =
2
p
-x 1 4 2 7
(B) Let h(x) = x ò 2 f (t) cos t dt P(X È Y) = + - =
0 3 15 15 15
p \ A and B are the correct options.
h (0) = - ò 2 f (t) cos t dt < 0 44. (2) Centre (–1, –2)
0
Geometrically, circle will have exactly 3 common
p -1 points with axes in the cases
and h(1) = 1 - ò 2 f (t) cos t dt > 0 (i) Passing through origin Þ p = 0
0
(ii) Touching x-axis and intersecting y-axis at two
p -x
\ h(x) = x - ò 2 -f (t) cos t dt = 0 points i.e. f2 > C and g2 = C.
0 i.e. 4 > – p and 1 = –p
at some x Î (0, 1) Þ p > –4 and p = –1
x Þ p = –1
(C) ex - ò - f (t) sin t dt (iii) Touching y-axis and intersecting x-axis at two
0
points i.e. f2 = c and g2 > C
Q x Î (0, 1) Þ ex Î (1, e) Þ 4 = –p and 1 > –p
and 0 < f(t) < 1 and 0 < sin t < " x Î (0, 1) Þ p = –4 and p > –1
x which is not possible.
\ 0< ò0 f (t) sin t dt < 1 \ Only two values of p are possible.
x
\ ex - ò f (t) sin t dt ¹ 0 for any x Î (0, 1) æ pö
45. (2) Given f(0) = 0, f ç ÷ = 3, f'(0) = 1
0 è 2ø
p
p
(D) f (x) + ò0
2 f (t) sin t dt > 0 " x Î (0, 1) g(x) = òx éë f ' ( t ) cosec t - cot t cosec t f ( t )ûù dt
2

41. (B, D) In options (A) and (C) |A2| = 1


p
and in option (B) and (D) |A2| = – 1 d
We know |A2| = |A|2 g(x) = lim òx2 ( f ( t ) cosec t ) dt
x ®0 dt
and |A|2 ¹ –1 Þ matrices given in options B & D
cannot be the squares of any 3 × 3 matrix with real = f æç p ö÷ cosec p – f(x) cosec x
entries. è 2ø 2
42. (A, B) a – b = 1, y ¹ 0
f (x)
æ az + b ö = 3 – f(x) cosec x = 3 –
Im çè ÷=y sin x
z +1 ø
f (x) f (x)
é a ( x + iy) + b ( x + 1) - iy ù lim g(x) = lim 3 – = 3 – lim
x® 0 sin x ® sin x
Þ Im ê x + 1 + iy ´ x + 1 - iy ú = y
x® 0 x 0

ë ( ) ( ) û f '( x )
= 3 – xlim = 3 – f' (0) = 3 – 1 = 2
- ( ax + b ) y + ay ( x + 1) ® 0 cos x
Þ =y 46. (1) For infinite many solutions
( x + 1) 2 + y2
1 a a2
-axy - by + axy + ay
=y a 1 a
Þ
( x + 1) 2 + y 2 2
= 0 Þ (1 – a2)2 = 0 Þ a = ± 1
Þ a – b = (x + 1)2 + y2 a a 1
Þ 1 = (x + 1)2 + y2 For a = 1, the system will have no solution and for
a = –1, all three equations reduce to x – y + z = 1
Þ x = –1 ± 1 - y 2 giving infinite many dependent solutions.
1 1 2 \ 1 + a + a2 = 1 – 1 + 1 = 1
43. (A, B) P(X) = , P(X/Y) = , P(Y/X) = 10!
3 2 5 47. (5) x = 10! and y = 10C1 × = 50 × 9!
2 1 2 2!
P(X Ç Y) = P (Y/X) P(X) = ´ = y 50 ´ 9!
5 3 15 \ = =5
9x 10!
EBD_7801
2017-26 JEE Advanced 2017 Solved Paper
48. (6) Let the sides be a – d, a, a + d where d is positive.
Using Pythagoras theorem, a2 2 1
Þ =1 and =
(a + d)2 = (a – d)2 + a2 3a + 42 2
3a + 4 2
Þ a = 4d
\ Sides are 3d, 4d, 5d Þ a4 – 3a2 – 4 = 0 and 3a2 = 12
Þ a2 = 4
1 Also for a2 = 4 equation of ellipse
Area = 24 Þ ´ 3d ´ 4d = 24
2 æ 1ö
Þ d=2 x2 + a2y2 = a2 is satisfied for the point ç 3, ÷
è 2ø
\ Sides are 6, 8, 10.
\ Smallest side = 6. \ II, (iv), R is the correct combination.
49. (B) For a = 2 and point of contact (–1, 1). (For questions 52-54) : We observe the following, in the given
Equation of circle is satisfied table.
x2 + y2 = 2 f(x) = x + logex – xlogex, x Î (0, ¥)
then eqn. of tangent is
–x + y = 2 Þ m = 1 1 1+ x
Þ f ' (x) = - log e x and f ''(x) = – 2
and point of contact x x
æ - ma ö æ- 2 f(1) = 1 > 0 and f (e2) = e2 + 2 – 2e2 = 2 – e2 < 0
a 2ö \ f(x) = 0 for some x Î (1, e2)
ç , ÷=ç , ÷ = (–1, 1)
è m +1 m +1ø è 2
2 2 2ø \ (I) is true.
\ (I) (ii), (Q) is the correct combination. 1
50. (C) Tangent y = x + 8 Þ m = 1 f ' (1) = 1 > 0 and f ' (e) = –1 <0
e
point (8, 16) \ f ' (x) = 0 for some x Î (1, e)
\ both the coordinates as well as m, are positive, \ (II) is true.
æ a 2a ö 1
the only possibility of point is çè 2 , ÷ø = (8, 16) If x Î (0, 1), > 0 and loge x < 0
m m x
Þ a= 8 1
Also it satisfies the equation of curve. \ f ' (x) = – logex > 0 Þ f is increasing on (0, 1)
x
y2 = 4ax for the point (8, 16)
\ f ' (x) ¹ 0 for some x Î (0, 1)
And equation of tangent my = m2x + a is satisfied by
\ (III) is false.
m = 1 and a = 8
If x Î (1, e), f '' (x) < 0 Þ f ' is decreasing on (i, e)
\ (III), (i), (P) is the correct combination.
\ f '' (x) ¹ 0 for some x Î (1, e)
æ 1ö \ (IV) is false.
51. (D) Point of contact çè 3, ÷ø and tangent 3x + 2y =
2 Also lim f(x) = lim x + (1 – x) logex = – ¥
x ®¥ x ®¥
4.
\ (i) is false and (ii) is true.
3 1
\ m= - lim f ' (x) = lim
2 – logex = –¥
x ®¥ x®¥ x
\ Both the coordinates are positive and m is \ (iii) is true
negative the possibilities for points. are
1 1
æ ma a ö lim f '' (x) = lim - 2 - = 0
Qç - , ÷ OR
x ®¥ x ®¥ x x
è m2 + 1 m2 + 1 ø \ (iv) is true.
f is increasing on (0, 1) already discussed
æ a2m 1 ö \ (P) is true.
Rç- , ÷ If x Î (e, e2) then
è a 2 m2 + 1 a 2 m2 + 1 ø
1
f ' (x) = – logex < 0
æ 3a 2a ö æ 1ö x
For point Q ç , ÷ = ç 3, ÷ø Þ f is decreasing in (e, e2)
è 7 7ø è 2
\ (Q) is true.
7 For x Î (0, 1), f "(x) < 0
We get a = 7 and a = Þ f' is decreasing in (0, 1)
4 \ R is false.
which is not possible. For x Î (e, e2) f " (x) < 0
æ a2 3 2 ö æ Þ f' decreasing in (e, e2)
1ö \ (S) is true.
For point R ç , ÷ = ç 3, ÷
2 2
è 3a + 4 3a + 4 ø è 2ø 52. (B) The only correct combination is (II), (ii), (Q)
53. (B) The only correct combination is (II), (iii), (S)
54. (C) The only incorrect combination is (III), (i), (R).
JEE Advanced 2017 Solved Paper 2017-27

Paper - 2
PHYSICS OC
In D OAC cos 60° =
1 dr OA
1. (A) = constant
r dt 1
\ OC = 2a ´ =a
4pR d é m ù 3 2
\ 3m dt ê 4 ú = constant The magnetic field at 'O' due to
ê pR3 ú
ë3 û m0 I
AB = [ sin 60° - sin 30°]
4p a
3 d -3
\ R ( R ) = constant
dt m0 I é 3 1 ù m0 I 1
= ê - ú= ´ ( 3 - 1)
3 -4 dR 4 p a ë 2 2 û 4pa 2
\ R ( -3R ) = constant The total magnetic field due to all the straight segments
dt
of the star is
dR
\ µR é m0 I 1 ù m I
dt =ê ´ ( 3 - 1) ú ´12 = 0 ´ 6( 3 - 1)
ë 4p a 2 û 4p a
O O r r
r r r r S -P
h 5. (A) Here P + bR = S \ R =
p/n p/n b
2. (B) h A r r r
Also R = Q - P
r r
A B B S-P r r r r r r
\ = Q - P \ S - P = bQ - bP
b
p OA h r r r
In DOAB cos = \ OB = p \ S = bQ + (1 - b) P
n OB cos
n
1 GM e m GM e m ´ 3 ´105
h é 1 ù 6. (B) mVe2 - - =0
D= -h = hê - 1ú 2 Re 2.5 ´104 Re
p p
cos ê cos ú
n ë n û Ve2 GM e é 3 ´105 ù
= ê1 + ú
hC p2 h2 2 Re ë 2.5 ´104 û
3. (D) - f0 = K .E = =
l 2me 2me l d 2
æ 2GM e ö
Differentiating on both sides Ve = 13 ç ÷ = 13 ´11.2 » 42
è Re ø
-hC h 2 æ -2 ö
dl = ´ d ld ÷
l 2 2me çè l d 3 ø
2L L
+
7. (B) T = g v
d ld l 3
\ µ d2 with error limits
dl l
2( L + dL) L + dL
T + dT = +
A A g v
4. (A)
2 L æ dL ö L æ dL ö
B I B
30° \ T + dT = ç1 + ÷ + ç1 + ÷
2a g è L ø vè L ø
60° 30°
30°
C 60° 2L æ dL ö L æ dL ö
4a \ T + dT = ´ ç1 + ÷ + ç1 + ÷
O g è 2L ø v è L ø
O

2L 2 L dL L L dL
\ T + dT = + + +
g g 2L v v L
EBD_7801
2017-28 JEE Advanced 2017 Solved Paper

2 L dL L dL Also CP2 = CO 2 + OP2 2


T + dT = T + +
g 2L v L
2 2
æ 5 R ö æ 3R ö
dL é 1 2L L ù = ç ÷ +ç ÷
è 8 ø è 2 ø
dT = L ê 2 g + v ú
êë ûú
13R
Substituting dT = 0.015, L = 20 m, g = 10 ms–2, CP2 =
8
v = 300 ms–1
We get
dL 15
=
L 1600
dL 15 15 p
´ 100 = ´ 100 = % » 1%
\ L 1600 16
8. (A, B) For the charge +Q to return region 1, the radius of the
3R
circular path taken by charge should by
2
mv 2 2p p
= QvB \ = QB
(3R / 2) 3R

2p
\ B = 3QR
Thus the particle will enter region 3 through the point P1 on
2p X-axis
i.e., B should be equal or greater than 'B' is the correct option.
2QR
Change in momentum = 2p
'A' is the correct option.
Thus 'C' is incorrect
8p
When B = mv
13QR mv 2
Further = qvB \ r = \ rµm
r qB
mv 2 æ 8p ö 13 R \ 'D' is incorrect.
= Qv ç ÷ \ r=
r è 13QR ø 8
9. (A, D) The potential difference between X and Y is
Thus 'C' is the of the centre of circular path of radius
VXY = V X - VY
13R
8 VXY = (VXY )0 sin(wt + q1 )

2 2 2 2p
Y where (V XY ) 0 = V0 + V0 - 2V0 cos = 3V0
3

(VXY ) 0 3
and (V XY )rms = = V0
2 2
(A) is the correct option
C Now the potential difference between Y and Z is
5R
VYZ = VY - VZ
8
O X VYZ = (VYZ )0 sin(wt + q2 )
R P2
3R / 2 2p
2 2 2
Where (VYZ )0 = V0 + V0 - 2V0 cos = 3V0
3
P1
+Q (VYZ ) 03
and (VYZ )rms = =
V0
2 2
Thus (D) is the correct option.
JEE Advanced 2017 Solved Paper 2017-29

Now path difference


10. (A, D) P +Q p = dcos q = nl (for bright fringe)
nl
45° q \ cos q =
d
l
\ - sin q Dq = (Dn)
d
Dnl
or Dq = -
d sin q
E
Ecos q As we move from p1 to p2, q decreases and therefore
The circumference of the flat surface is an Dq increases. Therefore (D) is incorrect.
equipotential because the distance of each point on 12. (A, B, C)
the circumference is equal from + Q After a long time the current through the resistor is
(D) is the correct option. constant I will divide into two parts L1 and L2 which
The component of electric field normal to the flat are in parallel
surface is Ecosq. \ I1L1 = I2L2
Here E as well as q changes for different point on the V é L2 ù
flat surface. Therefore (C) is incorrect. Further I1 = R ê L + L ú
The total flux through the curved and flat surface ë 1 û
Q V é L1 ù
should be less than e . Therefore (B) is incorrect. I2 =
0
and R êë L1 + L2 úû
The solid angle subtended by the flat surface at Also the ratio of currents through L1 and L2 is fixed
æ 1 ö at all times At t = 0, I » 0
P = 2p ç 1 - ÷ 13. (A, C, D)
è 2ø
As Fx = 0, ax = 0. Therefore the force acting in vertical
\ Flux passing through curved surface direction will move the mid point of the bar fall
æ 1 ö vertically downwards. (A) is correct option.
2p ç 1 - ÷
Q' è 2ø Q æ 1 ö.
=- =- ç1 - ÷
e0 4p 2e 0 è 2ø Y
A (0, L)
(A) is the correct option.
11. (B, C)
Path difference at P2 is
p = S1P2 – S2P2 = d = 1.8 mm = 1.8 × 10–3m
= 3000 × 600 × 10–9 m
p = 3000 l. (x, y)
As the path difference is an integral multiple of l, P2 90 – q
B
should be a bright fringe with 300th maxima. (A) is
incorrect.
q
Further at P1, path difference = 0. Therefore a bright
fringe will be present at P1 also. Therefore total
number of fringes between P1 and P2 is 3000. (C) is a
correct option. L/2 L/2
Obviously at P2 the order of the fringe will be q L
mg cos q
maximum. Thus (B) is a correct option. 2
P1
X
P L
Rdq sin q
2

q
os When the bar makes an angle q with the vertical, the
dc
q P2 displacement of its mid point from the initial position
S1 S2
d L L
is - cos q
2 2
EBD_7801
2017-30 JEE Advanced 2017 Solved Paper
(D) is a correct option.
Instantaneous torque about the point of contact P is æ R - rö
17. (C) Here w0 (R - r ) = wR \ w = wo çè ÷
R ø
L
t = mg ´ sin q Now total kinetic energy of the ring (Kinetic rotational
2 + kinetic translational)
(C) is a correct option.
1
L K .Etotal = (2MR 2 )w2 = M w0 2 ( R - r ) 2
Now x = sin q , y = L sin(90 - q) = L cos q 2
2
2 2 2 2
18. (A) mM w2min ( R - r ) = Mg
æ 2x ö æ y ö 4x y
\ ç ÷ + ç ÷ = 1 or 2 + 2 = 1 g
è L ø è Lø L L \ wmin =
This is equation of ellipse. Therefore B is incorrect m( R - r )
14. (C) If the force is applied at P tangential than the I remains
constant and is equal to F × 2R where F is the applied
CHEMISTRY
force.
If force is applied normal to X, then as the wheels 19. (C) As T increase, V.P. increases
climbs, then the perpendicular distance of force from DTf = Kf × m
Q will go on changing initially the perpendicular is 34.5 ´ 1000
QM, later it becomes QM'. 273 – Tf¢ = 2 ´
46 ´ 500
\ T¢f = 270 K
F 20. (B) Zn ( s ) + Cu 2aq
+ +
® Zn 2aq + Cu ( s )
( ) ( )

x F
M
é Zn 2+ ù
ë û
Q DG = DG° + 2.303 RT log10Q ; Q = +
éCu ù2
ë û

[DG° = –nFE°] = –2 × F × 1.1


x
Given [Zn2+] = 10[Cu2+]
If the force is applied normal to the circumference at \ DG = –2F (1.1) + 2.303 RT log1010
point P then I is zero. So (C) is correct. = 2.303 RT – 2.2F
If the force is applied tangentially at point S then 21. (A) At eq. DG = 0
t = F ´ R and the wheel will climb. DG° = dp(DV)
15. (A) Work done by battery = q × V 2.9 kJ mol–1 = (P2 – 1) × 2 × 10–6 m3 mol–1
\ W = CVo × Vo = CVo2 or
1
Energy stored in the battery =
2
CV0 2
2.9 ´ 103
kg m2
s 2 mol -1
(
( P2 - 1) 2 ´ 10-6
=
m3
mol -1
)
\ Energy dissipated
1 2 1 2 2.9 ´ 103 ´ 106 kg
ED = W – EC = CV02 - 2 CV0 = 2 CV0 (P2 – 1) =
2 ms2
\ EC = ED
16. (C) Let Vi and Vf be the initial and final voltage in each kg
process. Then = 1.45 × 109 = 1.45 × 109 Pa
ms 2
Energy dissipated = Wbattery – DU \ P2 = 1.45 × 109 + 1Pa
1 1 = 14500 × 105 + 1 = 14501 × 105 Pa = 14501 bar
= C(Vf –Vi )Vf – C(Vf – Vi ) 2 = C(Vf – Vi ) 2
2 2 22. (C) Fe + conc. HNO3 ® Fe2O3
\ Total heat dissipated Cu + conc. HNO3 ® CuNO3 + NO2 + H2O
4Au + 8NaCN + O2 + 2H2O ® 4Na[Au (CN)2] + 4NaOH
1 éæ Vo 2V ö ù
2 2 2
ö æ 2V V ö æ Zn + NaOH ® Na2ZnO2 + H2
ED = C êç - 0÷ + ç o - o ÷ + çVo - o ÷ ú
2 ëè 3 ø è 3 3ø è 3 ø û 23. (C) Correct order :
H3PO 4 > H 4 P2 O6 > H3 PO3 > H3PO2
1
= CVo 2 ( +5) ( +4) ( +3) ( +1)
6
JEE Advanced 2017 Solved Paper 2017-31

24. (C) Lone pair is involved in aromatic sextet. Hence not


Step 1 : available.
OH OH Hence the correct order of basic strength is
IV > I > II > III
(i) NaNO 2, HCl, 0°C 26. (A, B)
(A) As adsorption is spontaneous, DG for the process is –
+H ve. Adsorption is accompained by decrease in
NH2 Nº N randomness. Therefore DS and TDS for the process is
|
Cl – also negative. As DS for the process is –ve and the
Step 2 : process is spontaneous, DH for the process has to be
–ve i.e, enthalpy of the system decreases.
OH OH (B) Under a given set of conditions of temperature and
pressure the easily liquefiable gases e.g. C2H6, NH3
(ii) aqNaOH and HCl are adsorbed more than the gases like N2, H2
and CO. The ease with which a gas can be liquefied is
+
NºN determined by its critical temperature.
N=N
| H Critical temperature is the minimum temperature above
Cl Diazocoupling which a gas can be liquified. This implies that gases
with high critical temperature values can be easily
liquified as compared to gases with low critical
temperature value.
25. (C)
+ 27. (B, D)
(IV) 28. (A, B) The Arrhenius equation is
The conjugate acid is stabilized by resonance with k = Ae–Ea/RT
two different –NH2 group. Hence electron density where A = Pre-exponential factor
increases on N of = NH A is not directly related with temperature and activation
energy.

+ Where A = Z ´ P
( Frequency factor ) (Steric factor )
(I) Hence we can say the Ea is independent of steric factor
The conjugate acid is stabilized by resonance with
one –NH2 group. Hence as compared to IV lesser K actual
P=
increase of electron density on N of = NH K theoretical

So, Aactual > Atheoretical


+ 29. (A, B, C)

(II) (A) I is Br (1° benzylic halide) and


Lone pair is not involvd in aromaticity. Hence more
available
C
|
N: C – C – Br (3° alkylhalide). Follow SN1.
|
N C
H (B) I and II follow SN2 also, as both are 1° halide.
(III) (C) Compound (IV) undergoes inversion of configuration
due to presence of chiral carbon atom.
EBD_7801
2017-32 JEE Advanced 2017 Solved Paper
30. (A, B, D)
(A) Structure of Al2(CH3)6

H
H H
C
H3C CH3
Al Al
H3C CH3
C
H H
H
3C-2e– Bond
(B) Structure of B2H6

H
H H
B B
H H
H
3C-2e– Bond

(C) Structure of Al2Cl6


Cl Cl Cl
Al Al
Cl Cl Cl
(D) BCl3 is stronger lewis acid as the bond formed with the base will involve 2p orbital overlap which is stronger than 3p orbital
overlap in the case of AlCl3.
31. (A, D) NO Þ Neutral
B2O3 Þ Acidic
CrO Þ Basic
All other oxides are amphoteric
32. (A, B)

O O
(i) O3, CH2Cl 2
(C) CH3 Q (C8H8O), i.e., CH3 C + H – C– H
(ii) Zn, H2O
H
(P)
(Q) (+ve cannizaro reaction)

O
C O
(i) O 3, CH 2Cl 2
CH3
(ii) Zn, H2O
+ H – C– H

(R) CH3 (S) (+ve haloform reaction)


CH3 O O
O3, CH2Cl2
(D) C + CH3 – C– H
Zn, H2O
H
CH3
CH3
(Q) (+ve cannizaro reaction)
(P)
JEE Advanced 2017 Solved Paper 2017-33

CH3 O
C C O
CH3 O3, CH2Cl 2
CH3 + CH3 – C– CH3
C
Zn, H2O
CH3
(R) CH3
(+ve haloform reaction)
(S)
33. (C) & 34. (B)
D
KClO3 ¾¾¾¾
® KCl + O 2
MnO2
( W)
P4 + O2 ® P4 O10
( white) ( excess) (X)
P4O10 + HNO3 ® N 2O5 + HPO3
( Y) ( Z)
35. (A) & 36. (D)
– +
O OMgBr

(CH3)3C C OEt (CH3)3C CH3 C CH3


CH 3 MgBr(excess)/(C2 H 5 )2 O
¾¾¾¾¾¾¾¾¾¾¾®

(P)
H2O
H + H
O O H

(CH3)3C C(CH3)2 (CH3)3C CH3 C CH3


H SO
¬¾¾¾
2 4¾
0°C

(Q)
–H 2O
CH3
+ CH3
(CH3)3C C(CH3)2 (CH3)3C
(Friedel Craft
alkylation)

(R)

CH3 C Cl/AlCl3
(Friedel Craft Acylation)

CH3
CH3
(CH3)3C

COCH3
(S)
EBD_7801
2017-34 JEE Advanced 2017 Solved Paper
MATHEMATICS
æ 1ö
37. (C) The required equation of plane is given by f (1) - f ç ÷
è 2ø æ1 ö
x -1 y - 1 z -1 f ' (a) = , a Î ç , 1÷
1 è2 ø
1-
2 1 -2 =0 2
3 -6 -2
Þ (x – 1) (–14) – (y – 1) (2) + (z – 1) (–15) = 0 æ1 ö
Þ f ' (a) = 1 for some a Î ç , 1÷
Þ 14x – 14 + 2y – 2 + 15z – 15 = 0 è2 ø
Þ 14 x + 2y + 15z = 31 Þ f ' (1) > 1
uuuur uuur uuuur uuur uuuur uuur uuur uuur
38. (D) OP · OQ + O R· OS = OR · OP + OQ· OS é a1 a2 a3 ù
uuuur uuur uuur uuur uuur uuur ê ú
( ) (
Þ OQ - OR · OP - OQ - OR · OS = 0 ) 41. (B) Let M = ê a 4
êë a 7
a5 a 6 ú where ai Î {0, 1, 2}
a8 a 9 úû
uuuur uuur uuur uuur
Þ ( OQ - OR ) · ( OP - OS) = 0
uuur uur é a1 a4 a 7 ù éa1 a2 a3 ù
Þ RQ ·SP = 0 Tê úê ú
Then M M = ê a 2 a5 a 8 ú êa 4 a5 a6 ú
Þ RQ ^ SP …(1)
uuur uuur uuur uuur uuur uuur uuur uuur ëê a 3 a6 a 9 ûú ëêa 7 a8 a 9 ûú
Also OR · OP + OQ · OS = OQ · OR + OP · OS T
Sum of the diagonal entries in M M = 5
uuur uuur uuur uuur uuur uuur
( ) (
Þ OR · OP - OQ - OS · OP - OQ = 0 ) Þ (a12 + a42 + a72) + (a22 + a52 + a82)
+ (a32 + a62 + a92) = 5
uuur uuur uuur uuur
Þ ( OP - OQ) · ( OR - OS) = 0 It is possible when
Case I: 5 ais are 1 and 4 ai's are zero
uuur uuur Which can be done in
Þ QP ·SR = 0
Þ QP ^ SR …(2) 9´8´7´6
9
C4 ways = = 126
From (1) and (2) S should be the orthocentre of DPQR. 4 ´ 3 ´ 2 ´1
39. (A) Given DE can be written as Case II: 1 ai is 1 and 1ai is 2 and rest.
7ai's are zero
1 It can be done in 9C1 × 8C1 = 9 × 8 = 72 ways
ò dy = ò dx
( 4+ 9+ x )( )
9+ x 8 x
42. (D)
\ Total no. of ways = 126 + 72 = 198.
N1 = 5C1 × 4C4 = 5
N2 = 5C2 × 4C3 = 40
Putting 4+ 9+ x =t N3 = 5C3 × 4C2 = 60
We get N4 = 5C4 × 4C1 = 20
N5 = 5C5 = 1
1
dx = dt \ N1 + N2 + N3 + N4 + N5 = 126
2 4+ 9 + x ·2 9+ x ·2 x 43. (B) Total number of non negative solutions of x + y + z
= 10 are = 12C2 = 66 (using n+r–1Cr–1)
\ ò dy = ò dt Þ y= t + c If z is even then there can be following cases:
z = 0 Þ No. of ways of solving x + y = 10 Þ 11C1
or y= z = 2 Þ No. of ways of solving x + y = 8 Þ 9C1
4+ 9+ x +C
z = 4 Þ No. of ways of solving x + y = 6 Þ 7C1
y(0) = 7 Þ C=0 z = 6 Þ No. of ways of solving x + y = 4 Þ 5C1
z = 8 Þ No. of ways of solving x + y = 2 Þ 3C1
\ y= 4+ 9+ x z = 10 Þ No. of ways of solving x + y = 0 Þ 1
\ Total ways when z is even = 11 + 9 + 7 + 5 + 3 + 1 = 36
\ y(256) = 3
40. (D) f " (x) > 0, " x Î R 36 6
\ Required probability = =
æ 1ö 1 66 11
f ç ÷ = , f (1) = 1
è 2ø 2 sin 2x
44. g(x) = òsin x sin -1 ( t ) dt
\ f ' is an increasing function on R.
By Lagrange's Mean Value theorem. Þ g'(x) = sin –1(sin 2x) · 2 cos 2x – sin–1(sin x) · cos x
æ pö æ pö p
g ' ç ÷ = sin –1(sin p) · 2 cos p – sin–1 ç sin ÷ cos
è 2ø è 2 ø 2
JEE Advanced 2017 Solved Paper 2017-35

=0 = –2 × (Some value oscillating between –1 and 1)


æ pö \ does not exist.
g ' ç - ÷ = sin–1(sin(–p)) · 2cos(–p) –
è 2ø
cos 2x cos 2x sin 2x
æ æ -p ö ö æ -p ö - cos x cos x - sinx
sin–1 çè sin çè 2 ÷ø ÷ø · cos çè 2 ÷ø = 0 48. (B, C) f(x) =
sin x sin x cos x
\ None of the options are matching here.
45. (A, C) If we consider tan a/2 = x and tan b/2 = y, then Operating C1 ® C1 – C2
2(cos b – cos a) + cos a cos b = 1

é1 - y2 1 - x 2 ù
Þ 2ê - ú
1 - x 2 1 - y2 ( )( ) 0 cos 2x
Þ f (x) = -2 cos x cos x
sin 2x
- sin x
2 =1–
2
ëê1 + y 1 + x ûú 1 + x 2 1 + y2 ( )( ) 0 sin x cos x
2 2 2 2
Þ 2[(1 + x )(1 – y ) – (1 – x ) (1 + y )]
= (1 + x2) (1 + y2) – (1 – x2) (1 – y2) Þ f(x) = 2 cos 3x cos x
Þ 4(x – y ) = 2(x2 + y2)
2 2
Þ f(x) = cos 4x + cos 2x
Þ x2 = 3y2 f max = 2 at x = 0
Þ x= ± 3 y f ' (x) = – 4 sin 4x – 2 sin 2x
a b = – 2sin 2x [4 cos 2x + 1]
Þ tan ± 3 tan = 0
2 2 -1
46. (A, C) f ' (x) – 2 f (x) > 0 f ' (x) = 0 Þ sin 2x = 0 or cos 2x =
4
df ( x )
Þ e–2x - 2e -2x f ( x ) > 0 p p
dx Þ x= - , 0, which is true for some x Î (–p, p)
2 2
Þ
dx
(
d -2x
e f (x) > 0 ) \ f ' (x) = 0 at more than three points in (–p, p)
Þ e–2x f(x) is an increasing function. α 1 1
ò0 (x - x )dx = 2 ò0 (x – x
3 3
49. (B, C) )dx
\ for x > 0
f (x) > f (0)
Þ e–2xf(x) > 1 Y

Þ f(x) > e2x in (0, ¥) 2


Also f '(x) > 2e2x > 0
\ f is an increasing function in (0, ¥) 1
1 - (1 - h ) [1 + h ] æ 1ö
47. (A, D) lim- f(x) = hlim
®
cos ç ÷
è hø X' X
x ®1 0 h O
-2 -1 1 2

1 -1 + h2 æ 1ö -1
= lim cos ç ÷
h ®0 h è hø
x=a
-2
æ 1ö
= lim h cos çè ÷ø = 0 Y'
h ®0 h
a 1
æ x2 x4 ö 1 æ x2 x4 ö
lim f(x) = lim 1 - (1 + h )(1 + h ) cos æ 1 ö Þ ç - ÷ = ç - ÷
x ®1+
çè ÷ø è 2 4ø 2è 2 4ø
h®0 h h 0 0

a 2 a4 1 æ 1 1 ö
-2h - h 2
æ 1ö Þ - = ç - ÷
= lim cos ç ÷ 2 4 2 è 2 4ø
h ®0 h è hø
2a 2 - a 4 1
Þ = or 4a2 – 2a4 = 1
æ 1ö 4 8
= lim ( -2 - h ) cos çè ÷ø Þ 2a4 – 4a2 + 1 = 0
h ®0 h
EBD_7801
2017-36 JEE Advanced 2017 Solved Paper
P
4 ± 16 - 8 1
Þ a2 = =1±
4 2
1
\ 0 < a < 1 Þ a2 = 1 –
2 P+Q
1
Þ a = 1 - 0.71 = 0.29 > 0.25 = 2 also a < 1 Q
R
uuur uuur uuur
1 QR uuur RP uuur PQ
Þ <a<1 uuur uuur , OY = uuur , OZ = uuur
2 \ OX = QR RP PQ
98
k +1 k +1
50. (B, D) I = å òk x ( x + 1)
dx uuur uuur
uuur uuur
OR ´ RP
k =1
\ OX ´ OY = uuur uuur
Let x – k = t Þ dx = dt QR RP
98
k +1 uuur uuur
QR RP sin ( P + Q)
1
\ I= å ò0 ( t + k )( t + k + 1) dt uuur uuur
k =1 =
QR RP
98
1 k +1 = sin (P + Q)
Þ I> å ò0 dt
k =1 ( t + k + 1) 2 52. (B) cos (P + Q) + cos (Q + R) + cos (R + P)
= cos (180 – R) + cos (180 – P) + cos (180 – Q)
98 1 = –[cos P + cos Q + cos R]
æ -1 ö
Þ I> å ( k + 1) çè t + k + 1÷ø 3
k =1 0 In any DPQR, cos P + cos Q + cos R <
2
98
æ 1 1 ö
I > å ( k + 1) çè k + 1 - k + 2 ÷ø
3
Þ Þ – (cos P + cos Q + cos R) > –
k =1 2

1 98
1 1 1 3
\ Required minimum value = –
Þ I > å k + 2 = 3 + 4 + ¼ + 100 2
k =1
53. (B) Q a, b are roots of x2 – x – 1 = 0
1 1 1 98 49 \ a2 – a – 1 = 0, b2 – b – 1 = 0
Þ I> + + ¼+ = = Þ a2 = a + 1 and b2 = b + 1
100 100 100 100 50
Also a n = pan + qbn
49 Þ a0 =p+q
Þ I>
50 a1 = pa + pb
k +1 k +1 a2 = pa2 + qb2 = p(a + 1) + q(b + 1)
Also < [Q least value of x + 1 is k + 1] = (pa + qb) + (p + q) = a1 + a0
x ( x + 1) x ( k + 1)
a 3 = pa3 + qb3 = pa (a + 1) + qb(b + 1)
k +1 1 = (pa2 + qb2) + (pa + qb)
Þ <
x ( x + 1) x = a2 + a1
Proceeding in the same manner, we get
98 98 a12 = a11 + a10
k +1 k +1 k +1 1
\ I= å òk x ( x + 1)
dx < å òk x
dx 54. (D) a 4 = a3 + a2 = a2 + a1 + a2 = 2a2 + a1
k =1 k =1
= 2a1 + 2a0 + a1 = 3a1 + 2a0
98 = 3(pa + qb) + 2(p + q)
æ k + 1ö
Þ I< å log e çè k ø
÷ é æ1+ 5 ö æ1- 5 ö ù
k =1 = 3 êp ç + qç
÷ ÷ ú + 2(p + q)
æ2 3 4 99 ö ëê è 2 ø è 2 ø úû
Þ I < loge çè ´ ´ ´¼´ ÷ø
1 2 3 98 7 3
Þ I < loge99
( p + q ) + ( p - q ) 5 = 28
=
2 2
uuur uuur uuur Þ p=q=4 \ p + 2q = 12
51. (A) OX, OY, OZ are unit vectors in the directions of
uuur uuur uuur
sides OR, RP and PQ respectively,,
JEE ADVANCED 2016
1. The question paper consists of three parts (Physics, Chemistry and Mathematics). Each part consists of three
sections.
2. Section 1 contains 5 questions. Each question has four choices (a), (b), (c) and (d). ONLY ONE of these four
options is correct.
4. Section 3 contains 5 questions. The answer to each of the questions is a single-digit integer ranging from 0 to 9
(both inclusive).
3. Section 2 contains 8 multiple choice questions. Each question has four choices (a), (b), (c) and (d) out of which
ONE OR MORE THAN ONE are correct.

PAPER - 1

PHYSICS 3. A water cooler of storage capacity 120 litres can cool water
at a constant rate of P watts. In a closed circulation system
SECTION - I (as shown schematically in the figure), the water from the
cooler is used to cool an external device that generates
This section contains 5 multiple choice questions. Each question
constantly 3 kW of heat (thermal load). The temperature of
has four choices (a), (b), (c) and (d) out of which ONLY ONE
water fed into the device cannot exceed 30°C and the entire
option is correct.
stored 120 litres of water is initially cooled to 10°C. The
1. In a historical experiment to determine Planck's constant, a entire system is thermally insulated. The minimum value of
metal surface was irradiated with light of different P (in watts) for which the device can be operated for 3 hours
wavelengths. The emitted photoelectron energies were is
measured by applying a stopping potential. The relevant
data for the wavelength (l) of incident light and the
corresponding stopping potential (V0) are given below :
l (mm) V0 (Volt)
0.3 2.0
0.4 1.0 (Specific heat of water is 4.2 kJ kg–1K–1 and the density of
0.5 0.4 water is 1000 kg m–3)
(a) 1600 (b) 2067
Given that c = 3 × 108m s–1 and e = 1.6 × 10–19 C , Planck's (c) 2533 (d) 3933
constant (in units of J s) found from such an experiment is 4. A parallel beam of light is incident from air at an angle a on
(a) 6.0 × 10–34 (b) 6.4 × 10–34 the side PQ of a right angled triangular prism of refractive
(c) 6.6 × 10 –34 (d) 6.8 × 10–34
index n = 2 . Light undergoes total internal reflection in
2. A uniform wooden stick of mass 1.6 kg and length l rests in
an inclined manner on a smooth, vertical wall of height h(< the prism at the face PR when a has a minimum value of 45°.
l) such that a small portion of the stick extends beyond the The angle q of the prism is
wall. The reaction force of the wall on the stick is
perpendicular to the stick. The stick makes an angle of 30° P
with the wall and the bottom of the stick is on a rough floor.
The reaction of the wall on the stick is equal in magnitude to q
the reaction of the floor on the stick. The ratio h/l and the a
frictional force f at the bottom of the stick are
(g = 10 m s–2)
n= 2
h 3 16 3 h 3 16 3
(a) = ,f = N (b) = ,f = N Q R
l 16 3 l 16 3

h 3 3 8 3 h 3 3 16 3 (a) 15° (b) 22.5°


(c) = ,f = N (d) = ,f = N (c) 30° (d) 45°
l 16 3 l 16 3
EBD_7801
2016-2 JEE Advanced 2016 Solved Paper
5. An infinite line charge of uniform electric charge density l (c) The plot below represents schematically the variation
lies along the axis of an electrically conducting infinite of beat frequency with time
cylindrical shell of radius R. At time t = 0, the space inside
the cylinder is filled with a material of permittivity e and v(t)
electrical conductivity s. The electrical conduction in the
material follows Ohm's law. Which one of the following P
graphs best describes the subsequent variation of the Q
magnitude of current density j(t) at any point in the material? vQ

j(t)
j(t) R
t
(a) (b)
(d) The plot below represents schematically the variation
of beat frequency with time
t
t (0, 0)
(0, 0) v(t)

j(t) j(t)
P

vQ
(c) (d) Q

t t
R
(0, 0) (0, 0)
t
SECTION - II 8. An incandescent bulb has a thin filament of tungsten that is
This section contains 8 multiple choice questions. Each question heated to high temperature by passing an electric current.
has 4 choices (a), (b), (c) and (d) out of which ONE or MORE The hot filament emits black–body radiation. The filament
is observed to break up at random locations after a
THAN ONE are correct.
sufficiently long time of operation due to non–uniform
6. Highly excited states for hydrogen–like atoms (also called evaporation of tungsten from the filament. If the bulb is
Rydberg states) with nuclear charge Ze are defined by their powered at constant voltage, which of the following
principal quantum number n, where n>>1. Which of the statement(s) is(are) true?
following statement(s) is(are) true? (a) The temperature distribution over the filament is
(a) Relative change in the radii of two consecutive orbitals uniform
does not depend on Z (b) The resistance over small sections of the filament
(b) Relative change in the radii of two consecutive orbitals decreases with time
varies as 1/n (c) The filament emits more light at higher band of
(c) Relative change in the energy of two consecutive frequencies before it breaks up
orbitals varies as 1/ n 3 (d) The filament consumes less electrical power towards
(d) Relative change in the angular momenta of two the end of the life of the bulb
9. A plano–convex lens is made of a material of refractive index
consecutive orbitals varies as 1/n
n. When a small object is placed 30 cm away in front of the
7. Two loudspeakers M and N are located 20 m apart and emit
curved surface of the lens, an image of double the size of
sound at frequencies 118 Hz and 121 Hz, respectively. A car the object is produced. Due to reflection from the convex
is initially at a point P , 1800 m away from the midpoint Q of surface of the lens, another faint image is observed at a
the line MN and moves towards Q constantly at 60 km/hr distance of 10 cm away from the lens. Which of the following
along the perpendicular bisector of MN. It crosses Q and statement(s) is(are) true?
eventually reaches a point R, 1800 m away from Q. Let v(t) (a) The refractive index of the lens is 2.5
represent the beat frequency measured by a person sitting (b) The radius of curvature of the convex surface is 45 cm
in the car at time t. Let vP , vQ and vR be the beat frequencies (c) The faint image is erect and real
measured at locations P, Q and R, respectively. The speed (d) The focal length of the lens is 20 cm
of sound in air is 330 ms–1 . Which of the following 10. A length–scale (l) depends on the permittivity (e) of a
statement(s) is(are) true regarding the sound heard by the dielectric material. Boltzmann constant (kB), the absolute
person? temperature (T), the number per unit volume (n) of certain
(a) vP + vR = 2 vQ charged particles, and the charge (q) carried by each of the
(b) The rate of change in beat frequency is maximum when particles. Which of the following expression(s) for I is(are)
dimensionally correct?
the car passes through Q
JEE Advanced 2016 Solved Paper 2016-3

æ nq 2 ö æ ek T ö
(a) l= ç ÷ (b) l = ç B2 ÷
ç ek BT ÷ ç nq ÷
è ø è ø

æ q2 ö æ q2 ö
(c) l= ç ÷ (d) l= ç ÷
ç en 2/3 k T ÷ ç en1/3 k T ÷
è B ø è B ø

11. A conducting loop in the shape of a right angled isosceles


triangle of height 10 cm is kept such that the 90° vertex is
very close to an infinitely long conducting wire (see the Which of the following statement(s) is(are) true?
figure). The wire is electrically insulated from the loop. The (a) nl sinqi = n2 sinqf
hypotenuse of the triangle is parallel to the wire. The current (b) n1 sin qi = (n 2 – n 1) sinqf
in the triangular loop is in counterclockwise direction and (c) l is independent of n2
increased at a constant rate of 10 A s–1. Which of the (d) l is dependent on n(z)
following statement(s) is(are) true?
SECTION - III

10 cm This section contains 5 questions. Each question, when worked


90°
out will result in one integer from 0 to 9 (both inclusive).

æ m0 ö 14. A metal is heated in a furnace where a sensor is kept above


(a) The magnitude of induced emf in the wire is çè ÷ø the metal surface to read the power radiated (P) by the metal.
p
The sensor has a scale that displays log 2, (P/P0 ), where P0
volt
is a constant. When the metal surface is at a temperature of
(b) If the loop is rotated at a constant angular speed about
487°C, the sensor shows a value 1. Assume that the
æ m0 ö emissivity of the metallic surface remains constant. What is
the wire, an additional emf of çè ÷ø volt is induced in the value displayed by the sensor when the temperature of
p
the wire the metal surface is raised to 2767°C?
(c) The induced current in the wire is in opposite direction 15. The isotope 12 having a mass 12.014 u undergoes
5 B
to the current along the hypotenuse
(d) There is a repulsive force between the wire and the b–decay to 12 12 has an excited state of the nucleus
6 C. 6 C
loop
12. The position vector rr of a particle of mass m is given by the
following equation
( 126 C*) at 4.041 MeV above its ground state. If 125 E decays
r 12
r (t) = at 3ˆi + bt 2ˆj , to 6 C * , the maximum kinetic energy of the b–particle in
where a = 10/3 ms–3, b = 5 ms–2 and m = 0.1 kg. At t = 1 s, units of MeV is (1 u = 931.5 MeV/c2, where c is the speed of
which of the following statement(s) is(are) true about the light in vacuum).
particle? 16. A hydrogen atom in its ground state is irradiated by light of
(a) The velocity vr is given by vr = (10iˆ + 10ˆj) ms–1 wavelength 970 Å. Taking hc/e = 1.237 × 10–6 eV m and the
r ground state energy of hydrogen atom as –13.6 eV, the
(b) The angular momentum L with respect to the origin is number of lines present in the emission spectrum is
r 17. Consider two solid spheres P and Q each of density 8 gm
given by L = – 5/3) k̂ N m s
r r cm–3 and diameters 1 cm and 0.5 cm, respectively. Sphere P
(c) The force F is given by F = (iˆ + 2ˆj) N is dropped into a liquid of density 0.8 gm cm–3 and viscosity
r h = 3 poiseulles. Sphere Q is dropped into a liquid of density
(d) The torque t with respect to the origin is given by
r 1.6 gm cm–3 and viscosity h = 2 poiseulles. The ratio of the
t = – (20/3) k̂ N m terminal velocities of P and Q is
13. A transparent slab of thickness d has a refractive index n(z) 18. Two inductors L1 (inductance 1 mH, internal resistance 3
that increases with z. Here z is the vertical distance inside
W) and L2 (inductance 2 mH, internal resistance 4 W), and a
the slab, measured from the top. The slab is placed between
resistor R (resistance 12 W) are all connected in parallel
two media with uniform refractive indices n1 and n 2 (> n1),
across a 5 V battery. The circuit is switched on at time t = 0.
as shown in the figure. A ray of light is incident with angle
qi, from medium 1 and emerges in medium 2 with refraction The ratio of the maximum to the minimum current
angle qt with a lateral displacement l. (Imax / I min) drawn from the battery is
EBD_7801
2016-4 JEE Advanced 2016 Solved Paper
CHEMISTRY (c) higher the magnitude of activation energy, stronger is
the temperature dependence of the rate constant.
SECTION - I (d) the pre–exponential factor is a measure of the rate at
which collisions occur, irrespective of their energy.
This section contains 5 multiple choice questions. Each question
25. A plot of the number of neutrons (N ) against the number of
has four choices (a), (b), (c) and (d) out of which ONLY ONE
protons (P) of stable nuclei exhibits upward deviation from
option is correct.
linearity for atomic number, Z > 20. For an unstable nucleus
19. P is the probability of finding the 1s electron of hydrogen having N/P ratio less than 1, the possible mode(s) of decay
atom in a spherical shell of infinitesimal thickness, dr, at a is(are)
distance r from the nucleus. The volume of this shell is (a) b– –decay (b emission)
4p r2dr. The qualitative sketch of the dependence of P on r is (b) orbital or K–electron capture
(c) neutron emission
P P (d) b-–decay (positron emission)
26. The crystalline form of borax has
(a) tetranuclear [B4O5(OH)4]2– unit
(b) all boron atoms in the same plane
(a) (b) (c) equal number of sp2 and sp3 hybridized boron atoms
(d) one terminal hydroxide per boron atom
27. The compound(s) with TWO lone pairs of electrons on the
0 r 0 r
central atom is(are)
(a) BrF5 (b) ClF3
P P (c) XeF4 (d) SF4
28. The reagent(s) that can selectively precipitate S2– from a
mixture of S2– and SO42– in aqueous solution is(are)
(a) CuCl2 (b) BaCl2
(c) (d) (c) Pb(OOCCH3)2 (d) Na2[Fe(CN)5NO]
29. Positive Tollen’s test is observed for
0 r 0 r H O
20. One mole of an ideal gas at 300 K in thermal contact with CHO
surroundings expands isothermally from 1.0 L to 2.0 L against (a) H (b)
a constant pressure of 3.0 atm. In this process, the change H
in entropy of surroundings (DSsurr) in JK–1 is H
(1 L atm = 101.3 J)
OH
(a) 5.763 (b) 1.013 O
(c) –1.013 (d) –5.763 Ph
(c) Ph (d)
21. The increasing order of atomic radii of the following Group
13 elements is O Ph Ph
(a) Al < Ga < In < Tl (b) Ga < Al < In < Tl 30. The product(s) of the following reaction sequence is(are)
(c) Al < In < Ga < Tl (d) Al < Ga < Tl < In
NH2
22. Among [Ni(CO) 4 ], [NiCl 4 ] 2– , [Co(NH 3 ) 4 Cl 2 ]Cl,
(i) Acetic anhydride/pyridine
Na 3[CoF 6 ], Na 2 O 2 and CsO 2, the total number of (ii) KBrO 3/HBr
paramagnetic compounds is
(iii) H 3O +, heat
(a) 2 (b) 3
(iv) NaNO 2/ HCl, 273-278 K
(c) 4 (d) 5
(v) Cu/HBr
23. On complete hydrogenation, natural rubber produces
(a) ethylene–propylene copolymer Br
(b) vulcanised rubber Br
(c) polypropylene
(d) polybutylene
(a) (b)
SECTION - II
Br Br
This section contains 8 multiple choice questions. Each question
has 4 choices (a), (b), (c) and (d) out of which ONE or MORE
THAN ONE are correct. Br
24. According to the Arrhenius equation, Br Br Br
(a) a high activation energy usually implies a fast reaction. Br Br
(b) rate constant increases with increase in temperature. (c) (d)
This is due to a greater number of collisions whose
energy exceeds the activation energy. Br
JEE Advanced 2016 Solved Paper 2016-5

31. The correct statement(s) about the following reaction the equation x2 + 2x tan q – 1 = 0. If a1 > b1 and a2 > b2, then
sequence is(are) a1 + b2 equals
(i) O CHCl / NaOH (a) 2 (sec q – tan q) (b) 2 sec q
Cumene(C9 H12 ) ¾¾¾¾
2 ¾
+
® P ¾¾¾¾¾¾
3 ® (c) –2 tanq (d) 0
(ii)H3O
38. A debate club consists of 6 girls and 4 boys. A team of 4
Q (major) + R (minor) members is to be selected from this club including the
NaOH selection of a captain (from among these 4 members) for the
Q ¾¾¾¾¾
PhCH Br
®S
team. If the team has to include at most one boy, then the
2
(a) R is steam Volatile number of ways of selecting the team is
(b) Q gives dark violet coloration with 1% aqueous FeCl3 (a) 380 (b) 320
solution (c) 260 (d) 95
(c) S gives yellow precipitate with 2, 4–dinitro- ì pü
phenylhydrazine 39. Let S = íx Î( -p, p) : x ¹ 0, ± ý . The sum of all distinct
î 2þ
(d) S gives dark violet coloration with 1% aqueous FeCl3
solutions of the equation 3 sec x + cosec x + 2(tan x – cot
solution x) = 0 in the set S is equal to
SECTION - III
7p 2p
This section contains 5 questions. Each question, when worked (a) – (b) -
9 9
out will result in one integer from 0 to 9 (both inclusive).
5p
(c) 0 (d)
32. The mole fraction of a solute in a solution is 0.1. At 298 K, 9
molarity of this solution is the same as its molality. Density 40. A computer producing factory has only two plants T1 and
of this solution at 298 K is 2.0 g cm–3 . The ratio of the T2. Plant T1 produces 20% and plant T2 produces 80% of
the total computers produced. 7% of computers produced
æ MWsolute ö in the factory turn out to be defective. It is known that
molecular weights of the solute and solvent, ç MW ÷, P (computer turns out to be defective given that it is produced
è solvent ø
in plant T1)
is = 10P
33. The diffusion coefficient of an ideal gas is proportional to (computer turns out to be defective given that it is produced
its mean free path and mean speed. The absolute temperature in plant T2),
of an ideal gas is increased 4 times and its pressure is where P(E) denotes the probability of an event E. A computer
increased 2 times. As a result, the diffusion coefficient of produced in the factory is randomly selected and it does
this gas increases x times. The value of x is not turn out to be defective. Then the probability that it is
34. In neutral or faintly alkaline solution, 8 moles of permanganate produced in plant T2 is
anion quantitatively oxidize thiosulphate anions to produce
X moles of a sulphur containing product. The magnitude of 36 47
(a) (b)
X is 73 79
35. The number of geometric isomers possible for the complex 78 75
[CoL2Cl2]– (L = H2NCH2CH2O–) is (c) (d)
93 83
36. In the following monobromination reaction, the number of
possible chiral products is 1
41. The least value of a Î ¡ for which 4ax2 + ³ 1 , for all x > 0,
x
CH2CH2CH3
is
H Br Br2 (1.0 mole)
300°C 1 1
CH3 (a) (b)
64 32
(1.0 mole)
(enantiomerically pure) 1 1
(c) (d)
27 25
MATHEMATICS
SECTION - II
SECTION - I
This section contains 8 multiple choice questions. Each question
This section contains 5 multiple choice questions. Each question has 4 choices (a), (b), (c) and (d) out of which ONE or MORE
has four choices (a), (b), (c) and (d) out of which ONLY ONE THAN ONE are correct.
option is correct.
42. Consider a pyramid OPQRS located in the first octant (x ³ 0,
p p y ³ 0, z ³ 0) with O as origin, and OP and OR along the x–axis
37. Let - < q < - . Suppose a1 and B1are the roots of the and the y–axis, respectively. The base OPQR of the pyramid
6 12
is a square with OP = 3. The point S is directly above the
equation x2 – 2x sec q + 1 = 0 and a2 and b2 are the roots of mid-point T of diagonal OQ such that TS = 3. Then
EBD_7801
2016-6 JEE Advanced 2016 Solved Paper
46. A solution curve of the differential equation (x2 +xy + 4x +2y
p
(a) the acute angle between OQ and OS is
3 dy
+ 4) – y2 = 0, x > 0, passes through the point (1, 3). Then
(b) the equation of the plane containing the triangle OQS dx
is x – y = 0 the solution curve
(c) the length of the perpendicular from P to the plane (a) intersects y = x + 2 exactly at one point
(b) intersects y = x + 2 exactly at two points
3
containing the triangle OQS is (c) intersects y = (x + 2)2
2 (d) does NOT intersect y = (x +3)2
(d) the perpendicular distance from O to the straight line 47. Let f : ¡ ® ¡ , g : ¡ ® ¡ and h : ¡ ® ¡ be
differentiable functions such that
15
containing RS is
2 f(x) = x3 + 3x + 2, g(f(x)) = x and h (g(g(x))) = x for all x Î ¡ .
Then
43. Let f : (0, ¥) ® ¡ be a differentiable function such that
1
f (x) (a) g¢(2) = (b) h¢(1) = 666
f '(x) = 2 - for all x Î (0, ¥) and f(1) ¹ 1. Then 15
x
(c) h(0) = 16 (d) h(g(3)) = 36
48. 2 2
The circle C1 : x + y = 3, with centre at O, intersects the
æ 1ö æ1ö
(a) lim f ¢ ç ÷ = 1 (b) lim xf ç ÷ = 2 parabola x2 = 2y at the point P in the first quadrant. Let the
x ®0 + è x ø x ®0 + èxø
tangent to the circle C1 at P touches other two circles C2
and C3 at R2 and R3, respectively. Suppose C2 and C3 have
(c) lim x 2 f ¢ (x) = 0 (d) |f(x)| £ 2 for all x Î (0, 2)
x ®0 + equal radii 2 3 and centres Q2 and Q3, respectively. If Q2
and Q3 lie on the y–axis, then
é3 -1 -2ù
ê2 0 a ú (a) Q2Q3 = 12
44. Let P = ê ú , where a Î ¡ . Suppose Q = [qij] is a (b) R2R3 = 4 6
êë3 -5 0 úû
(c) area of the triangle OR2R3 is 6 2
matrix such that PQ = kI, where k Î ¡ , k ¹ 0 and I is the
(d) area of the triangle PQ2Q3 is 4 2
k k2 49. Let RS be the diameter of the circle x2 + y2 = 1, where S is the
identity matrix of order 3. If q23 = – and det(Q) = ,
8 2 point (1, 0). Let P be a variable point (other than R and S) on
then the circle and tangents to the circle at S and P meet at the
(a) a = 0, k = 8 (b) 4a – k + 8 = 0 point Q. The normal to the circle at P intersects a line drawn
(c) det (P adj (Q)) = 2 9 (d) det (Q adj (P)) = 213 through Q parallel to RS at point E. Then the locus of E
45. In a triangle XYZ, let x, y, z be the lengths of sides opposite passes through the point(s)
to the angles X,Y, Z, respectively, and 2s = x + y + z.
æ1 1 ö æ 1 1ö
s-x s-y s-z (a) çè 3 , ÷ø (b) çè , ÷ø
3 4 2
If = = and area of incircle of the triangle
4 3 2
æ1 1 ö æ 1 1ö
8p (c) çè 3 , - ÷ø (d) çè , - ÷ø
XYZ is , then 3 4 2
3
SECTION - III
(a) area of the triangle XYZ is 6 6
This section contains 5 questions. Each question, when worked
35 out will result in one integer from 0 to 9 (both inclusive).
(b) the radius of circumcircle of the triangle XYZ is 6
6 50. The total number of distinct x Î ¡ for which

X Y Z 4
(c) sin sin sin = x x2 1 + x3
2 2 2 35
2x 4x 2 1 + 8x 3
= 10 is
æ X + Yö 3 3x 9x 2 1 + 27x 3
sin 2 ç =
(d) è 2 ÷ø 5
JEE Advanced 2016 Solved Paper 2016-7

51. Let m be the smallest positive integer such that the


coefficient of x2 in the expansion of (1 + x)2 + (1 + x)3 + ... + x 2 sin(bx)
53. Let a, b Î ¡ be such that lim = 1. Then
(1 + x)49 + (1 + mx)50 is (3n + 1) 51C3 for some positive x ® 0 ax - sin x
integer n. Then the value of n is 6 (a + b) equals.
52. The total number of distinct x Î [0, 1] for which
-1 + 3i
x 2 54. Let z = , where i = -1 , and r, s Î {1, 2, 3}. Let
t 2
ò1+ t 4
dt = 2x – 1 is
0 é( - z)r z 2s ù
P = ê 2s ú and I be the identity matrix of order 2.
êë z z r úû
Then the total number of ordered pairs (r, s) for which
P2 = –I is
EBD_7801
2016-8 JEE Advanced 2016 Solved Paper

PAPER - 2
1. The question paper consists of three parts (Physics, Chemistry and Mathematics). Each part consists of three
sections.
2. Section 1 contains 6 questions. Each question has four options (a), (b), (c) and (d). ONLY ONE of these four
options is correct.
3. Section 2 contains 8 multiple choice questions. Each question has four choice (a), (b), (c) and (d) out of which
ONE OR MORE THAN ONE are correct.
4. Section 3 contains 2 paragraphs each describing theory, experiment and data etc. four questions relate to two
paragraphs with two questions on each paragraph. Each question pertaining to a particular passage should have
ONLY ONE correct answer among the four given choices (a), (b), (c) and (d).

PHYSICS process at volume Vf . The amount of heat supplied to the


system in the two-step process is approximately
SECTION - I (a) 112 J (b) 294 J
This section contains 6 multiple choice questions. Each question (c) 588 J (d) 813 J
has four choices (a), (b), (c) and (d) out of which ONLY ONE 4. The ends Q and R of two thin wires, PQ and RS, are soldered
option is correct. (joined) together. Initially each of the wires has a length of 1
m at 10°C. Now the end P is maintained at 10°C, while the
1. The electrostatic energy of Z protons uniformly distributed end S is heated and maintained at 400 °C. The system is
throughout a spherical nucleus of radius R is given by thermally insulated from its surroundings. If the thermal
2 conductivity of wire PQ is twice that of the wire RS and the
3 Z (Z - 1) e coefficient of linear thermal expansion of PQ is 1.2 × 10–5
E=
5 4πε 0R K–1, the change in length of the wire PQ is
(a) 0.78 mm (b) 0.90 mm
The measured masses of the neutron 11 H, 15 15
7 N and 8 O are (c) 1.56 mm (d) 2.34 mm
1.008665 u, 1.007825 u, 15.000109 u and 15.003065 u, 5. A small object is placed 50 cm to the left of a thin convex
15 15 lens of focal length 30 cm. A convex spherical mirror of radius
respectively. Given that the radii of both the 7 N and 8 O of curvature 100 cm is placed to the right of the lens at a
nuclei are same, 1 u = 931.5 Me V/c2 (c is the speed of light) distance of 50 cm.The mirror is tilted such that the axis of
and e2/(4 pe0) = 1.44 MeV fm. Assuming that the difference the mirror is at an angle q =30° to the axis of the lens, as
between the binding energies of 15 15 shown in the figure.
7 N and 8 O is purely due
to the electrostatic energy, the radius of either of the nuclei f = 30 cm
is
(1 fm = 10–15 m)
(a) 2.85 fm (b) 3.03 fm q x
(–50,0) (0, 0) R
(c) 3.42 fm (d) 3.80 fm =
2. An accident in a nuclear laboratory resulted in deposition 10
0
50 cm cm
of a certain amount of radioactive material of half-life 18
days inside the laboratory. Tests revealed that the radiation
was 64 times more than the permissible level required for
safe operation of the laboratory. What is the minimum number (50 + 50 3, -50)
of days after which the laboratory can be considered safe
for use? If the origin of the coordinate system is taken to be at the
(a) 64 (b) 90 centre of the lens, the coordinates (in cm) of the point (x, y)
(c) 108 (d) 120 at which the image is formed are
3. A gas is enclosed in a cylinder with a movable frictionless
piston. Its initial thermodynamic state at pressure Pi = 105 (a) (0, 0) (b) (50 - 25 3, 25)
Pa and volume Vi = 10–3 m3 changes to a final state at Pf = (1/
32) × 105 Pa and Vf = 8 × 10–3 m3 in an adiabatic quasi-static (c) (25, 25 3) (d) (125 / 3,25 3)
process, such that P3V5 = constant. Consider another 6. There are two Vernier calipers both of which have 1 cm
thermodynamic process that brings the system from the same divided into 10 equal divisions on the main scale. The Vernier
initial state to the same final state in two steps: an isobaric scale of one of the calipers (C1) has 10 equal divisions that
expansion at Pi followed by an isochoric (isovolumetric) correspond to 9 main scale divisions. The Vernier scale of
JEE Advanced 2016 Solved Paper 2016-9

the other caliper (C2) has 10 equal divisions that correspond


to 11 main scale divisions. The readings of the two calipers
are shown in the figure. The measured values (in cm) by
calipers C1 and C2, respectively, are Light

2 3 4
C1
Electrons
0 5 10

2 3 4
V
C2 – +
0 5 10

(a) 2.85 and 2.82 (b) 2.87 and 2.83


(a) le decreases with increase in f and lph
(c) 2.87 and 2.86 (d) 2.87 and 2.87
(b) le is approximately halved, if d is doubled
SECTION - II (c) For large potential difference (V >> f/e), le is
approximately halved if V is made four times
This section contains 8 multiple choice questions. Each question
has 4 choices (a), (b), (c) and (d) out of which ONE or MORE (d) le increases at the same rate as lph for lph < hc/f
THAN ONE are correct. 9. In an experiment to determine the acceleration due to gravity
7. Two thin circular discs of mass m and 4m, having radii of a g, the formula used for the time period of a periodic motion
and 2a, respectively, are rigidly fixed by a massless, rigid
7 (R - r)
rod of length l = 24a through their centres. This assembly is T = 2p . The values of R and r are measured to
5g
is laid on a firm and flat surface, and set rolling without
slipping on the surface so that the angular speed about the be (60 ± 1) mm and (10 ± 1) mm, respectively. In five
axis of the rod is w. The angular momentum of the entire successive measurements, the time period is found to be
ur
assembly about the point ‘O’ is L (see the figure). Which 0.52s, 0.56s, 0.57s, 0.54s and 0.59s. The least count of the
of the following statement(s) is (are) true? watch used for the measurement of time period is 0.01s.
Which of the following statement(s) is (are) true?
4m
(a) The error in the measurement of r is 10%
(b) The error in the measurement of T is 3.75%
m (c) The error in the measurement of T is 2%
z w
l (d) The error in the determined value of g is 11%
2a
l a 10. Consider two identical galvanometers and two identical
O resistors with resistance R. If the internal resistance of the
galvanometers RC < R/2, which of the following statement(s)
about any one of the galvanometers is (are) true?
(a) The centre of mass of the assembly rotates about the
(a) The maximum voltage range is obtained when all the
z-axis with an angular speed of w/5
(b) The magnitude of angular momentum of center of mass components are connected in series
of the assembly about the point O is 81 ma2w (b) The maximum voltage range is obtained when the two
(c) The magnitude of angular momentum of the assembly resistors and one galvanometer are connected in series,
about its center of mass is 17 ma2w/2. and the second galvanometer is connected in parallel
ur
(d) The magnitude of the z-component of L is 55 ma2w. to the first galvanometer
8. Light of wavelength lph falls on a cathode plate inside a (c) The maximum current range is obtained when all the
vacuum tube as shown in the figure. The work function of components are connected in parallel
the cathode surface is f and the anode is a wire mesh of
(d) The maximum current range is obtained when the two
conducting material kept at a distance d from the cathode. A
potential difference V is maintained between the electrodes. galvanometers are connected in series and the
If the minimum de Broglie wavelength of the electrons combination is connected in parallel with both the
passing through the anode is le, which of the following resistors
statement(s) is (are) true?
EBD_7801
2016-10 JEE Advanced 2016 Solved Paper
11. In the circuit shown below, the key is pressed at time t = 0.
Which of the following statement(s) is(are) true?

Screen
z
40 mF 25 kW
O
y

v S1 S2
+ d x
D
A 50 W 20 mF (a) Straight bright and dark bands parallel to the x-axis
(b) The region very close to the point O will be dark
(c) Hyperbolic bright and dark bands with foci
+ – symmetrically placed about O in the x-direction
Key (d) Semi circular bright and dark bands centered at point.
5V
14. A rigid wire loop of square shape having side of length L
and resistance R is moving along the x-axis with a constant
(a) The voltmeter displays –5V as soon as the key is velocity v0 in the plane of the paper. At t = 0, the right edge
pressed, and displays +5V after a long time of the loop enters a region of length 3L where there is a
(b) The voltmeter will display 0V at time t = ln 2 seconds uniform magnetic field B0 into the plane of the paper, as
(c) The current in the ammeter becomes 1/e of the initial shown in the figure. For sufficiently large v0. the loop
value after 1 second eventually crosses the region. Let x be the location of the
(d) The current in the ammeter becomes zero after a long right edge of the loop. Let v(x), I(x) and F(x) represent the
time. velocity of the loop, current in the loop, and force on the
12. A block with mass M is connected by a massless spring loop, respectively, as a function of x. Counter-clockwise
current is taken as positive.
with stiffness constant k to a rigid wall and moves without
friction on a horizontal surface. The block oscillates with
small amplitude A about an equilibrium position x0. Consider
two cases: (i) when the block is at x0; and (ii) when the block
is at x = x0 + A. In both the cases, a particle with mass m(<M) R
is softly placed on the block after which they stick to each L
other. Which of the following statement(s) is (are) true about v0
the motion after the mass m is placed on the mass M?
x
(a) The amplitude of oscillation in the first case changes 0 2L 4L
L 3L
M Which of the following schematic plot(s) is (are) correct?
by a factor of , whereas in the second case it
m+M (Ignore gravity)
remains unchanged.
v(x)
(b) The final time period of oscillation in both the cases is
same. v0
(c) The total energy decreases in both the cases.
(d) The instantaneous speed at x0 of the combined masses (a)
decreases in both the cases
13. While conducting the Young’s double slit experiment, a 0 L 2L 3L 4L x
student replaced the two slits with a large opaque plate in
the x-y plane containing two small holes that act as two I(x)
coherent point sources (S1, S2) emitting light of wavelength
600 nm. The student mistakenly placed the screen parallel
to the x-z plane (for z > 0) at a distance D = 3 m from the mid-
point of S1S2, as shown schematically in the figure. The
3L 4L
distance between the sources d = 0.6003 mm. The origin O is
(b) x
at the intersection of the screen and the line joining S1S2. 0 L 2L
Which of the following is(are) true of the intensity pattern
on the screen?
JEE Advanced 2016 Solved Paper 2016-11

I(x) w

R
(c)
m
0 x
L 2L 3L 4L R/2

F(x)
15. The distance r of the block at time t is
R wt -wt R
(a)
4 (
e +e
) (b)
2
cos wt

R 2wt R
+ e -2wt
4( )
(c) e cos 2wt (d)
x 2
0 L 2L 3L 4L
16. The net reaction of the disc on the block is
(d)
1
mw2 R e2wt - e -2wt ˆj + mgkˆ
(
(a)
2 )
1
mw R (e2 wt
- e-wt ˆj + mgkˆ
(b)
2 )
(c) - mw2 R coswt ˆj - mgkˆ
SECTION - II (d) mw2 R sinswt ˆj - mgkˆ
This section contains 2 paragraphs, each describing theory, PARAGRAPH 2
experiments, data etc. four questions related to the two paragraphs Consider an evacuated cylindrical chamber of height h having
with two questions on each paragraph. Each question has only rigid conducting plates at the ends and an insulating curved
one correct answer among the four given options (a), (b), (c) and surface as shown in the figure. A number of spherical balls made
(d). of a light weight and soft material and coated with a conducting
PARAGRAPH 1 material are placed on the bottom plate. The balls have a radius r
A frame of reference that is accelerated with respect to an inertial <<h. Now a high voltage source (HV) is connected across the
frame of reference is called a non-inertial frame of reference. A conducting plates such that the bottom plate is at +V0 and the
coordinate system fixed on a circular disc rotating about a fixed top plate at –V0 . Due to their conducting surface, the balls will
axis with a constant angular velocity w is an example of non- get charged, will become equipotential with the plate and are
inertial frame of reference. The relationship between the force repelled by it. The balls will eventually collide with the top plate,
r where the coefficient of restitution can be taken to be zero due to
Frot experienced by a particle of mass m moving on the rotating the soft nature of the material of the balls. The electric field in the
r chamber can be considered to be that of a parallel plate capacitor.
disc and the force Fin experienced by the particle in an inertial
Assume that there are no collisions between the balls and the
frame of reference is
r r r ur ur r ur interaction between them is negligible. (Ignore gravity)
( ) ( )
Frot = Fin + 2m v rot ´w + m w´ r ´w. A
r
where v rot is the velocity of the particle in the rotating frame of
r –
reference and r is the position vector of the particle with respect HV
to the centre of the disc.
+
Now consider a smooth slot along a diameter of a disc of radius
R rotating counter-clockwise with a constant angular speed w
about its vertical axis through its center. We assign a coordinate 17. Which one of the following statements is correct?
system with the origin at the center of the disc, the x-axis along (a) The balls will stick to the top plate and remain there
the slot, the y-axis perpendicular to the slot and the z-axis along (b) The balls will bounce back to the bottom plate carrying
ur
the same charge they went up with
( )
the rotation axis w = wk̂ . A small block of mass m is gently
(c) The balls will bounce back to the bottom plate carrying
r
placed in the slot at r (R / 2) ˆi at t = 0 and is constrained to move the opposite charge they went up with
(d) The balls will execute simple harmonic motion between
only along the slot.
the two plates
EBD_7801
2016-12 JEE Advanced 2016 Solved Paper
18. The average current in the steady state registered by the 22. The geometries of the ammonia complexes of Ni2+ , Pt2+ and
ammeter in the circuit will be Zn2+ respectively, are
(a) zero (a) octahedral, square planar and tetrahedral
(b) proportional to the potential V0 (b) square planar, octahedral and tetrahedral
(c) tetrahedral, square planar and octahedral
(c) proportional to V01/2
(d) octahedral, tetrahedral and square planar
(d) proportional to V02 23. The correct order of acidity for the following compounds is
CO2H CO2H
CO2H CO2H
CHEMISTRY HO OH OH
SECTION - I
OH
This section contains 6 multiple choice questions. Each question OH
I II III IV
has four choices (a), (b), (c) and (d) out of which ONLY ONE
(a) I > II > III > IV (b) III> I > II > IV
option is correct.
(c) III > IV > II > I (d) I > III > IV > II
19. For the following electrochemical cell at 298 K, 24. The major product of the following reaction sequence is
Pt(s)|H2(g, 1 bar)| H+ (aq, 1 M) || M4+ (aq), M2+ (aq)|Pt(s) O
éM 2+ aq ù (i) HCHO (excess)/NaOH, heat
ê
ë ( )ûú x +
Ecell = 0.092 V when é 4+ = 10 . (ii) HCHO/H (catalytic amount)
êM (aq)ùú
ë û O O O O OH
RT (a) (b)
Given : E°M 4- /M 2+ = 0.151 V; 2.303 = 0.059V
F
The value of x is O OH
(a) –2 (b) –1 HO O O
(c) 1 (d) 2
20. The qualitative sketches I, II and III given below show the (c) (d)
variation of surface tension with molar concentration of
OH
three different aqueous solutions of KCl, CH3OH and
SECTION - II
CH3(CH2)11OSO3– Na+ at room temperature. The correct
assignment of the sketches is This section contains 8 multiple choice questions. Each question
has 4 choices (a), (b), (c) and (d) out of which ONE or MORE
I II III THAN ONE are correct.
Surface tension

Surface tension
Surface tension

25. According to Molecular Orbital Theory,


(a) C22 - is expected to be diamagnetic
(b) O22+ is expected to have a longer bond length than O2
Concentration Concentration Concentration
(c) N+2 and N -2 have the same bond order
(a) I : KCl II : CH3OH III : CH3(CH2)11OSO3– Na+
(b) I : CH3(CH2)11OSO3– Na+ II : CH3OH III : KCl (d) He+2 has the same energy as two isolated He atoms
– +
(c) I : KCl II : CH3(CH2)11OSO3 Na III : CH3OH 26. Mixture(s) showing positive deviation from Raoult’s law at
35 °C is (are)
(d) I : CH3OH II : KCl III : CH3(CH2)11OSO3– Na+ (a) carbon tetrachloride + methanol
21. In the following reaction sequence in aqueous solution, the (b) carbon disulphide + acetone
species X, Y and Z, respectively, are (c) benzene + toluene
(d) phenol + aniline
Ag+ Ag + 27. The CORRECT statement(s) for cubic close packed (ccp)
S2 O32 - ¾¾¾¾¾¾¾® X ¾¾¾¾¾¾¾® Y
clear white three dimensional structure is (are)
solution precipitate
(a) The number of the nearest neighbours of an atom
with time
¾¾¾¾¾¾¾¾¾¾¾® Z present in the topmost layer is 12
black precipitate (b) The efficiency of atom packing is 74%
(a) [Ag(S2O3)2]3–, Ag2S2O3, Ag2S (c) The number of octahedral and tetrahedral voids per
(b) [Ag(S2O3)3]5–, Ag2SO3, Ag2S atom are 1 and 2, respectively
(c) [Ag(SO3)2]3–, Ag2S2O3, Ag (d) The unit cell edge length is 2 2 times the radius of
(d) [Ag(SO3)3]3–, Ag2SO4, Ag the atom
JEE Advanced 2016 Solved Paper 2016-13

28. Extraction of copper from copper pyrite (CuFeS2) involves The standard reaction Gibbs energy, DrG°, of this reaction is
(a) crushing followed by concentration of the ore by froth- positive. At the start of the reaction, there is one mole of X 2 and
flotation no X. As the reaction proceeds, the number of moles of X formed
(b) removal of iron as slag is given by b. Thus, bequilibrium is the number of moles of X formed
(c) self-reduction step to produce ‘blister copper’ at equilibrium. The reaction is carried out at a constant total
following evolution of SO2 pressure of 2 bar. Consider the gases to behave ideally.
(d) refining of ‘blister copper’ by carbon reduction (Given R = 0.083 L bar K–1 mol–1)
29. The nitrogen containing compound produced in the reaction 33. The equilibrium constant Kp for this reaction at 298 K, in
of HNO3 with P4O10 terms of bequilibrium, is
(a) can also be prepared by reaction of P4 and HNO3
(b) is diamagnetic 8b2
equilibrium 8b2
equilibrium
(c) contains one N-N bond (a) ¾¾¾¾¾¾¾¾¾¾¾¾¾¾
2-bequilibrium
® (b) ¾¾¾¾¾¾¾¾¾¾¾¾¾¾
2
®
4 -bequilibrium
(d) reacts with Na metal producing a brown gas
30. For ‘invert sugar’, the correct statement(s) is(are) 4b2 4b2
equilibrium
equilibrium
(Given : specific rotations of (+) -sucrose, (+)-maltose, (c) ¾¾¾¾¾¾¾¾¾¾¾¾¾¾®
2 -bequilibrium
(d) ¾¾¾¾¾¾¾¾¾¾¾¾¾¾®
2
4 -bequilibrium
L-(-)-glucose and L-(+) fructose in aqueous solution are
+ 66°, +140°, –52° and +92°, respectively) 34. The INCORRECT statement among the following, for this
(a) ‘invert sugar’ is prepared by acid catalyzed hydrolysis reaction, is
of maltose (a) Decrease in the total pressure will result in formation
(b) ‘invert sugar’ is an equimolar mixture of D-(+)-glucose of more moles of gaseous X
and D-(-)-fructose (b) At the start of the reaction, dissociation of gaseous
(c) specific rotation of ‘invert sugar’ is –20° X2 takes place spontaneously
(d) on reaction with Br 2 water, ‘invert sugar’ forms (c) bequilibrium = 0.7
saccharic acid as one of the products
(d) KC < 1
31. Reagent(s) which can be used to bring about the following
PARAGRAPH 2
transformation is (are)
Treatment of compound O with KMnO4/H+ gave P, which on
heating with ammonia gave Q. The compound Q on treatment
O O O with Br2/NaOH produced R. On strong heating, Q gave S, which
O O C
C on further treatment with ethyl 2-bromopropanoate in the
presence of KOH followed by acidification, gave a compound T.
H OH
O O
COOH COOH
(a) LiAlH4 in (C2H5)2O (b) BH3 in THF
(c) NaBH4 in C2H5OH (d) Raney Ni/H2 in THF
32. Among the following, reaction(s) which gives(give)
tert-butyl benzene as the major product is(are)
Br Cl
(O)
(a) (b)
NaOC2H5 AlCl3
35. The compound R is
OH O
(c) (d) NH2
H2SO4 BF3OEt2 Br
(a) (b) Br
NH2
SECTION - III O
This section contains 2 paragraphs, each describing theory,
experiments, data etc. four questions related to the two paragraphs O O
with two questions on each paragraph. Each question has one or NHBr
NBr
more than one correct answer(s) among the four given options (c) NHBr (d)
(a), (b), (c) and (d).
O O
PARAGRAPH 1 36. The compound T is
Thermal decomposition of gaseous X2 to gaseous X at 298 K (a) glycine (b) alanine
takes place according to the following equation:
(c) valine (d) serine
X 2 (g) ƒ 2X (g)
EBD_7801
2016-14 JEE Advanced 2016 Solved Paper
MATHEMATICS SECTION - II
This section contains 8 multiple choice questions. Each question
SECTION - I
has 4 choices (a), (b), (c) and (d) out of which ONE or MORE
This section contains 6 multiple choice questions. Each question THAN ONE are correct.
has four choices (a), (b), (c) and (d) out of which ONLY ONE
option is correct. x
æ ön
ç æ nö æ nö ÷
é 1 0 0ù n n ( x + n ) ç x + ÷ ... ç x + ÷
ç è 2ø è nø ÷
ê ú 43. Let f ( x ) = lim ç , for
37. Let P = ê 4 1 0 ú and I be the identity matrix of order 3. æ 2ö æ 2 ö÷
( )
n ®¥ ç
2 2 2 n 2 n ÷
êë16 4 1 úû ç n! x + n çç x + 4 ÷÷ ... çç x + n 2 ÷÷ ÷
è è ø è øø
q 31 + q32 all x > 0. Then
If Q = [qij] is a matrix such that P50 – Q =I, then
q 21
æ1ö æ1ö æ2ö
equals (a) f ç ÷ ³ f (1) (b) f ç ÷ £ f ç ÷
(a) 52 (b) 103 è2ø è 3ø è3ø
(c) 201 (d) 205
38. Let bi > 1 for i = 1, 2, ..., 101. Suppose loge b1, logeb2, ...., loge f ' ( 3) f '(2)
(c) f ' (2) < 0 (d) ³
b101 are in Arithmetic Progression (A.P.) with the common f ( 3) f ( 2)
difference loge 2. Suppose a1, a2, ...., a101 are in A.P. such that
a1= b1 and a51= b51. If t= b1+b2 + .... + b51 and s= a1+a2+ .... + 44. Let a, b Î ¡ and f : ¡ ® ¡ be defined by
a51, then f (x) = a cos (|x3 –x|) + b |x| sin (|x3 +x|).
(a) s > t and a101> b101 (b) s > t and a101 < b101 Then f is.
(c) s < t and a101 > b101 (d) s < t and a101 < b101 (a) differentiable at x = 0 if a = 0 and b = 1
13
1 (b) differentiable at x = 1 if a = 1 and b = 0
39. The value of å
k =1 æ p ( k - 1) p ö æ p kp ö
is equal (c) NOT differentiable at x = 0 if a = 1, b = 0
sin ç + ÷ sin ç + ÷ (d) NOT differentiable at x = 1 if a = 1 and b = 1
è4 6 ø è4 6 ø 45. Let f : ¡ ® (0, ¥) and g : ¡ ® ¡ be twice differentiable
to functions such that f" and g" are continuous functions on
(a) 3- 3 ( )
(b) 2 3 - 3 ¡ . Suppose f '(2) = g(2) = 0, f"(2) ¹ 0 and g'(2) ¹ 0. If
f (x)g(x)
(c) 2 ( 3 -1 ) (d) 2 (2 - 3 ) lim
x ®2 f '(x)g '(x)
= 1 , then.

p (a) f has a local minimum at x = 2


x 2 cos x
2
(b) f has a local maximum at x = 2
40. The value of ò dx is equal to
p 1+ e
x
(c) f "(2) > f (2)
-
2 (d) f (x) – f "(x) = 0 for at least one x Î R
p2 p2 é 1 ù é 1 ù
(a) -2 (b) +2 46. Let f : ê - , 2ú ® ¡ and g : ê - , 2ú ® ¡ be functions
4 4 ë 2 û ë 2 û
p p
(c) (d) p2 + e 2 defined by f (x) = [x –3] and g(x) = |x| f (x) + |4x–7 | f (x), where
2
p2 - e 2
[y] denotes the greatest integer less than or equal to y for
41. Area of the region {( x, y ) Î ¡ : y ³
2
}
x + 3 , 5y £ x + 9 £ 15 y Î ¡ . Then

is equal to é 1 ù
(a) f is discontinuous exactly at three points in ê - , 2ú
1 4 ë 2 û
(a) (b)
6 3
é 1 ù
3 5 (b) f is discontinuous exactly at four points in ê - , 2ú
ë 2 û
(c) (d)
2 3 (c) g is NOT differentiable exactly at four points in
42. Let P be the image of the point (3,1,7) with respect to the
plane x–y+z = 3. Then the equation of the plane passing æ 1 ö
ç - , 2÷
x y z è 2 ø
through P and containing the straight line = = is.
1 2 1 æ 1 ö
(a) x + y – 3z = 0 (b) 3x + z = 0 (d) g is NOT differentiable exactly at five points in ç - , 2 ÷
è 2 ø
(c) x – 4y + 7z = 0 (d) 2x – y = 0
JEE Advanced 2016 Solved Paper 2016-15

47. Let a, b Î ¡ and a + b ¹ 0. 2 2


SECTION - III
ì 1 ü This section contains 2 paragraphs, each describing theory,
Suppose S = í Z Î C : Z = , + Î ¡, t ¹ 0 ý , where experiments, data etc. four questions related to the two paragraphs
î a + ibt þ
with two questions on each paragraph. Each question has only one
i= -1 . If z = x+ iy and z Î S, then (x, y) lies on correct answer among the four given options (a), (b), (c) and (d).
1 æ 1 ö PARAGRAPH 1
(a) the circle with radius and centre ç , 0 ÷ for a > 0,
2a è 2a ø Football teams T1 and T2 have to play two games against each
b¹ 0 other. It is assumed that the outcomes of the two games are
independent. The probabilities of T1 winning, drawing and losing
1 æ 1 ö
(b) the circle with radius - and centre ç - , 0 ÷ for 1 1 1
2a è 2a ø a game against T2 are , and respectively. Each team gets 3
2 6 3
a < 0, b ¹ 0
(c) the x-axis for a ¹ 0, b = 0 points for a win, 1 point for a draw and 0 point for a loss in a game.
(d) the y-axis for a = 0, b ¹ 0 Let X and Y denote the total points scored by teams T1 and T2
48. Let P be the point on the parabola y2 = 4x which is at the respectively after two games.
shortest distance from the center S of the circle x 2 + y2 – 4x 51. P (X > Y) is
–16y + 64 = 0.
1 5
Let Q be the point on the circle dividing the line segment SP (a) (b)
internally. Then 4 12
(a) SP = 2 5 1 7
(c) (d)
(b) SQ : QP = ( )
5 +1 : 2 2
52. P (X = Y) is
12

(c) the x-intercept of the normal to the parabola at P is 6


11 1
1 (a) (b)
(d) the slope of the tangent to the circle at Q is 36 3
2
49. Let a, l, m,Î ¡ . Consider the system of linear equations 13 1
(c) (d)
ax + 2y = l 36 2
3x – 2y = m
PARAGRAPH 2
Which of the following statement(s) is (are) correct?
(a) If a = –3, then the system has infinitely many solutions Let F1(x1, 0) and F2(x2, 0) for x1 < 0 and x2 > 0, be the foci of the
for all values of l and µ
x2 y 2
(b) If a ¹ –3, then the system has a unique solution for all ellipse + = 1 . Suppose a parabola having vertex at the
9 8
values of l and µ
(c) If l + µ = 0, then the system has infinitely many origin and focus at F2 intersects the ellipse at point M in the first
solutions for a = –3 quadrant and at point N in the fourth quadrant.
(d) If l + µ ¹ 0, then the system has no solution for
a = –3 53. The orthocentre of the triangle F1MN is
Ù Ù Ù
50. Let u = u1i + u 2 j + u 3 k be a unit vector in ¡3 and æ 9 ö æ2 ö
(a) ç - ,0÷ (b) ç , 0 ÷
Ù 1 æÙ Ù Ù ö è 10 ø è3 ø
w= ç i + j + 2k ÷ . Given that there exists a vector v in
®

6è ø æ 9 ö æ2 ö
(c) ç ,0÷ (d) ç , 6 ÷
Ù ®
æ Ù
ö Ù ® è 10 ø è3 ø
¡3 such that u´ v =1 and w . ç u ´ v ÷ =1 . Which of the
è ø 54. If the tangents to the ellipse at M and N meet at R and the
following statement(s) is (are) correct? normal to the parabola at M meets the x-axis at Q, then the
® ratio of area of the triangle MQR to area of the quadrilateral
(a) There is exactly one choice for such v MF1NF2 is
® (a) 3 : 4 (b) 4 : 5
(b) There are infinitely many choices for such v
(c) 5 : 8 (d) 2 : 3
Ù
(c) If u lies in the xy-plane then |u1| = |u2|
Ù
(d) If u lies in the xz-plane then 2 |u1| = |u3|
Telegram @unacademyplusdiscounts

Join Us on Telegram for More Such Books

https://telegram.me/unacademyplusdiscounts

Join us from the above link or search ''unacademyplusdiscounts'' in Telegram


EBD_7801
2016-16 JEE Advanced 2016 Solved Paper

SOLUTIONS
Paper - 1
PHYSICS h h
But cos 30° = \ x= … (iii)
hc f hc f x cos 30°
1. (b) - = V01 and - = V02
el1 e el 2 e
h 3l h 3 3
From (ii) and (iii) = \ =
hc é 1 1 ù cos 30° 8 l 16
\ e ê l - l ú = V01 - V02
ë 1 2û
mC DT V rC DT
3. (b) Pheater – Pcooler = =
e(V01 - V02 )l1l 2 t t
\ h=
(l 2 – l1 )c 0.12 ´ 1000 ´ 4.2 ´103 ´ 20
From the first two values given in data \ (3000 – Pcooler) =
3 ´ 60 ´ 60
1.6 ´ 10-19 [2 - 1] ´ 0.4 ´ 0.3 ´10 -6 \ Pcooler = 2067W
h=
0.1 ´ 3 ´ 108 4. (a) Applying Snell's law at A

h = 0.64 ´ 10 -33 = 6.4 ´ 10-34 J-s


1 × sin 45° = 2 ´ sin r1
Similarly if we calculate h for the last two values of \ r1 = 30° … (i)
data h = 6.4 × 10–34J-s
2. (d) Considering the normal reaction of the floor and wall
to be N and with reference to the figure.
q

Nsin 30° 90° – q


45° M
N 30° A r1 90°
+q
Ncos 30° c B

30°
n= 2
x
h

l/2 1.6g Applying Snell's law at B


N
2 sin C = 1 × sin 90°
A f B \ C = 45° … (ii)
In D AMB, 90° + q + r1 + (90° – C) = 180° (From fig.)
By vertical equilibrium. \ q = 15°

3.2 g I dV / dR
N + N sin 30° = 1.6 g Þ N = … (i) 5. (c) J= = … (i)
3 2pr l 2 pr l
By horizontal equilibrium
dr 1 dr
dR = r = ´ … (ii)
3 16 3 2pr l s 2 pr l
f = Ncos 30° = N= From (i)
2 3
dV
Taking torque about A we get Now E = -
dr
1.6g × AB = N × x
l 3.2 g 3l l
1.6 g × cos 60° = ´x \ = x … (ii) \ dV = –Edr = – dr … (iii)
2 3 8 2p Î r
JEE Advanced 2016 Solved Paper 2016-17

Relative change in the energy of two consecutive orbitals

En – En -1 E n2 -2 n + 1 - 2
= 1 - n -1 = 1 - 2
= »
En En (n - 1) (n - 1) 2 n
+
+ V
+ Ln - Ln -1 L (n - 1) 1
+ = 1 - n -1 = 1 - =
+ I Ln Ln n n
+ V + dV
l + 7. (a,b, c)
+
+
+ r P
+ vccos q q vc
+ dr
+ A
+ a Vx
+
+ 118Hz c
121Hz
+ 10 Q
+ M N
+

R
From (i), (ii), and (iii)
1 é ldr s 2p r l ù ls é v + v cos q ù
J= ê ´ ú = … (iv) n P = 121 - 118 = ê
2pr l ë 2l Î r dr û 2p Î r ë v úû

dV -l s ´ 2pr l -lsl nq = 121 - 118 = 3


Also I = = dr ´ = …(v)
dR 2 p Î r dr Î
é v - vc cos q ù
Here negative sign signifies that the current is n R = (121 –118) ê úû
decreasing ë v

d ( q ) d ( ll ) dl \ n P + n R = 2 n Q Þ ( A ) is correct option
But I = = =l … (vi)
dt dt dt In general when the car is passing through A
From (v) and (vi) é v + vc cos a ù
n = 3ê úû …(i)
dl lsl d l -s ë v
l =- Þ = dt
dt Î l Îl
dn é v sin a ù dn
On integrating \ = -3 ê c is max when sin a = 1
da ë v úû da
l t
dl s i.e., a =90° (at Q)
ò l
= - ò dt
Î0 Þ (b) is correct option.
l0
d n 3vc da
l st s
- t From (i) = (– sin a) … (ii)
\ loge =- \ l = l0 e Î dt v dt
l0 Î
Substituting this value in (iv) we get 10 2 da 10 dx
Also tan a = \ sec a =- 2
x dt x dt
s
sl0 - Î t
J= e da -10v
2p Î r \ = 2 …(iii)
6. (a, b, d)
dt x sec 2 a
From (ii) & (iii)
2
n2 E = - 13.6 Z nh
We know that r = r0 , n , Ln = dn 3v æ -10v ö 30Vc sin a
z n 2 2p = - c sin a ´ ç 2 ÷=
dt v è x sec 2 a ø x 2 sec 2 a
Relative change in the radii of two consecutive orbitals
rn – rn -1 r (n - 1) 2 dn 30vc sin a
= 1 - n -1 = 1 - \ dt = 2 2
= 0.3vc sin 3 a . At a = 90°
does not depend on Z (10 cot a ) sec a
rn rn n2

2n - 1 2 dn
= » = max
(Q n > > 1) dt
n2 n
\ (c) is the correct option
EBD_7801
2016-18 JEE Advanced 2016 Solved Paper
8. (c, d) With the use of filament and the evaporation involved, A B
m0i
the filament will become thinner thereby decreasing f= = Mi
10p x
the area of cross-section and increasing the resistance.
Therefore the filament will consume less power m0 10 cm x x dx
towards the end of life. \M =
10p
As the evaporation is non-uniform, the area of cross- Induced emf in the wire
section will be different at different cross-section. dI
Therefore temperature distribution will be non- di m 0 m = 10 AS -1
=M = ´ 10 = 0 dt
uniform. The filament will break at the point where the dt 10 p p
temperature is maximum. As the current in the triangular wire is decreasing the
æ 1ö induced current in AB is in the same direction as the
When the filament temperature is higher ç l n µ ÷ , current in the hypotenuse of the triangular wire.
è Tø
Therefore force will be repulsive.
it emits light of lower wavelength or higher band of
r
frequencies. 12. (a, b) r = at 3iˆ + bt 2 ˆj
9. (a, d) For lens
r 10
1 1 1 1 1 1 r = t 3iˆ + 5t 2 ˆjm
- = Þ - = 3
v u f v -30 f r
r dr
v= = 10t 2iˆ + 10tjˆ ms –1
v v dt
Also, m = Þ -2 =
u u r
r dv
On solving we get f = + 20 cm a= = 20tiˆ + 10 ˆjms -2
dt
and v = 60 cm.
For reflection At t = 1s
n r 10
1 1 1 2
+ = = Þ
1
+
1
=
2 rt =1 = iˆ + 5 ˆjm ;
v u f R 10 -30 R 3
r
Þ R = 30 cm vt =1 = 10iˆ + 10 ˆj ms –1
The image formed by convex side is faint erect and r
pt =1 = iˆ + ˆj kgms –1
virtual.
By lens maker formula r
at =1 = 20iˆ + 10 ˆj ms -2
1 æ nl öæ 1 1 ö
= ç - 1÷ç - ÷ iˆ ˆj kˆ
f è ns ø è R1 R2 ø r r r
L = r ´ p = 10 5 0 = kˆ é10 - 5 ù = - 5 kˆ kgms -1
3 ê ú ë3 û 3
1 æ nl öæ 1 ö
\ = ç - 1÷ç ÷ \ n = 2.5 1 1 0
20 è 1 øè 30 ø l
r r
q2 F = ma = (2iˆ + ˆj ) N
10. (b, d) We know that, dimensionally Î= ,
l2 F
RT iˆ ˆj kˆ
k BT = = PV = F ´ l
NA r r r 10 é10 ù -20 ˆ
t = r ´F = 5 0 = kˆ ê - 10 ú = k N–m
3 ë3 û 3
1/2
ek BT é q2 F ´l ù 2 1 0
Now = ê 2 ´ –3 2 ú =l
nq 2 êë l F l q úû 13. (a, c, d)
1/2 n1 sin qi = n2 sin q f [Q 1 and 2 interfaces are parallel]
q2 é l2 F ´ q2 ù
Also = ê ú =l l depends on the refractive index of transparent slab
Î n1/3 k BT ëê q 2 l -1 ´ F ´ l ûú but not on n 2 . In fact qf depends on n2.
11. (a, d) The flux passing through the triangular wire if i current 14. (9) Here P µ T 4 or P = P0T 4
flows through the inifinitely long conducting wire
0.1 P
m0 i \ log 2 P = log 2 P0 + log 2 T 4 \ log 2 = 4log 2 T
= ò 2px
´ 2pdx P0
0
JEE Advanced 2016 Solved Paper 2016-19

P CHEMISTRY
At T = 487°C = 760 K, log 2 = 4 log 2 760 = 1 … (1)
P0 19. (d) Radial probability function curve for 1s is (D).
Here P is 4pr 2R2.
At T = 2767°C = 3040K,
r
loge = 4log 2 3040 = 4 log 2 (760 ´ 4)
r0
P
= 4 éëlog 2 760 + log 2 22 ùû
= 4log 2 760 + 8 = 1 + 8 = 9
20. (c) From 1st law of thermodynamics
15. (9) Maximum kinetic energy of b-particle qsys = DU – w = 0 – [–Pext.DV]
= [mass of 12 12 = 3.0 atm × (2.0 L – 1.0 L) = 3.0 L-atm
5 B – mass of 6 C ] × 931.5 – 4.041
= [12.014 – 12] × 931.5 – 4.041] = 9MeV (q rev )surr qsys
\ DSsurr = =-
hc 1.237 ´ 10-6 T T
16. (6) E= = eV = 12.75eV
l 970 ´ 10 -10 3.0 ´ 101.3J
=- = –1.013 J/K
\ The energy of electron after absorbing this photon 300 K
= –13.6 + 12.75 = – 0.85eV 21. (b) Atomic radii increases on moving down a group.
This corresponds to n = 4 However due to poor shielding effect of d orbit, atomic
radius of Ga is smaller than Al (anomaly). Thus the
n(n - 1) 4(4 - 1) correct order is Ga < Al < In < Tl
Number of spectral line = = =6
2 2 22. (b) Compound/Ion Magnetic nature of compound
1. [Ni(CO)4] Diamagnetic
2r12 (s - r1 ) g 2. [NiCl4]2– Paramagnetic
VP 9h1 r 2 (s - r1 ) h2 3. [Co(NH3)4Cl2]Cl Diamagnetic
17. (3) = 2 = 12 ´ 4. Na3[CoF6] Paramagnetic
VQ 2 r2 (s - r 2 ) g r2 (s - r 2 ) h1
5. Na2O2 Diamagnetic
9h2 6. CsO2 Paramagnetic
So total number of paramagnetic compounds is 3.
12 [8 - 0.8] 2 23. (a)
= ´ =3
(0.5) [8 - 1.6] 3
2
CH3 CH3
| |
Polymerisation
CH 2 = C - CH = CH 2 ¾¾¾¾¾¾® -( CH 2 - C = CH - CH 2 -)
1mH 3W Isoprene Natural rubber

H2(excess)
2mH 4W catalyst

18. (8) CH3 CH3


12W |
Copolymerisation
|
CH 2 = CH 2 + CH 2 = CH ¾¾¾¾¾¾¾
® -( CH 2 - CH - CH2- CH 2-)
Ethylene Propylene Completely hydrogenated
Naturalrubber

24. (b, c, d)
5V (A) High activation energy usually implies a slow reaction.
(B) Rate constant of a reaction increases with increase in
temperature due to increase in number of collisions
5
At t = 0 Imin = whose energy exceeds the activation energy.
12
(C) k < P ´ Z ´ e, Ea RT
5 5 10 (D) So, pre-exponential factor (A) = P × Z and it is
At t = a Imax = R = 3 / 2 = 3 independent of activation energy or energy of
eq
molecules.
é 1 1 1 1 8ù 25. (b, d) When N/P ratio is less than one, then proton changes
ê R = 3 + 4 + 12 = 12 ú into neutron.
ë eq û
Position emission : 11 H ¾¾ ® 01 n + +01b
I max 10 12
\ I = ´ =8 k-electron capture : 11 H + 0
® 01 n + X-rays
-1 e ¾¾
min 3 5
EBD_7801
2016-20 JEE Advanced 2016 Solved Paper
26. (a, c, d) 30. (b)
Structure of borax
O O
HO O
– NH2 NH – C – CH3 NH – C – CH3
sp 3 B
OH Ac2O KBrO 3/HBr
O O
B Pyridine
B O B O sp 2
– Acetanilide
HO HO Br
sp 2 3 sp (Major)
+ –
Correct formula of borax is Na2[B4O5(OH)4].8H2O NH2 N NCl Br
(A) Borax has tetranuclear. [B4O5(OH)4]2– unit H 3O+ NaNO2/HCl Cu
(B) Only two 'B' atom lie in same plane 273 – 278 K HBr
(diazotisation)
(C) two Boron are sp2 and two are sp3 hybridised.
(D) one terminal hydroxide per boran atom. Br Br Br
27. (b, c) Compound Number of lone pairs
on central atom 31. (b, c)
BrF5 ® 1
OH OH OH
ClF3 ® 2 CHO
(i) O2 CHCl3/NaOH
XeF4 ® 2 +
(ii) H 3O+
SF4 ® 1
P Q
28. (a or a, c) (Major) CHO
R (Minor)

OH O – CH2Ph
S2- SO 42- CHO CHO
NaOH
Cu 2+ CuS(ppt) CuSO4 (Soluble) PhCH 2Br

Ba 2+ BaS(Soluble) BaSO 4 (ppt) S

Pb(OAc) 2 PbS(ppt) PbSO 4 (ppt) Q is steam volatile not R.


Na 2 [Fe(CN)5 NO] Na 4 [Fe(CN)5(NOS)] ____ Q and R show positive test with 1% aqueous FeCl3
Colour (not a ppt) solution.
Q, R, S give yellow precipitate with 2, 4-dinitrophenyl
29. (a, b, c) hydrazine.
Aldehydes and a-Hydroxyketones show positive 32. (9) 1 mole solution has 0.1 mole solute and 0.9 mole solvent.
Tollen's test. Let M1 = Molar mass solute
M2 = Molar mass solvent
H O
Tollen's 0.1
H ¾¾¾¾® Silver mirror ¯ Molality, m = ´ 1000 ....... (1)
H
Reagent 0.9M 2
H 0.1
Molarity, M = 0.1M + 0.9M ´ 2 ´ 1000 ....... (2)
CHO 1 2
Tollen's
¾¾¾¾® Silver mirror ¯ Q m= M
Reagent
0.1´ 1000 200
Þ =
OH 0.9M 2 0.1M1 + 0.9M 2
Ph Tollen's
Ph ¾¾¾¾® Silver mirror ¯ M1
Reagent
Þ =9
O M2
O 33. (4) Diffusion coefficient µ lµ
Ph
+ Ph T
Since l µ and m µ T
O P
Diketone
JEE Advanced 2016 Solved Paper 2016-21

MATHEMATICS
T T
\ Diffusion coefficient µ 37. (c) x2 – 2x sec q + 1 = 0 Þ x = sec q ± tan q
P
and x2 + 2x tan q – 1 = 0 Þ x = –tan q ± sec q
T T p p
Di 1 1 Q - <q< -
Thus = P = = 6 12
Df 4T 4T (4 ´ 2) 2 4
p p
2P Þ sec > sec q > sec
6 12
Df 4
or = p tan p
Di 1 and - tan < tan q < -
6 12
34. (6) 8MnO -4 + 3S2O 32 - + H 2 O ¾¾
® p p
also tan < - tan q < tan
12 6
8MnO 2 + 6SO24 - + 2OH -
a1,b1 are roots of x2 – 2x secq + 1 = 0
\ 8 moles of MnO–4 form 6 moles of SO42– and a1> b1
35. (5) [CoL2Cl2]– (L = H2NCH2CH2O–) \ a1 = secq – tanq and b1 = secq + tanq
L is unsymmetrical didentate ligand. a2, b2 are roots of x2 + 2x tan q – 1 = 0 and a2 > b2
So the complete is equivalent to [M(AB)2a2] Possible \ a2 = -tanq + secq, b2 = – tanq – secq
G..I. are \ a1 + b2 = secq – tanq – tanq – secq = – 2tanq
a a 38. (a) Either one boy will be selected or no boy will be
A B A A selected. Also out of four members one captain is to
M M be selected.
B A B B \ Required number of ways = (4C1 × 6C3+ 6C4) × 4C1
a a = (80 + 15) × 4 = 380
39. (c) 3 sec x + cosec x + 2 (tan x – cot x) = 0
a a B
A a B a A a 3 1
Þ sin x + cos x = cos2x – sin2 x
M M M 2 2
B A A B B a
æ pö
B A A Þ cos çè x - ÷ø = cos 2x
3
3 4 5
CH2CH2CH3 CH2CH2CH3 p
Þ x- = 2np ± 2x
Br2(1.0 mole)
3
2
36. (5) H Br H Br
300°C 2np p p
1 Þx= + or x = -2np -
CH
3 CH2 – Br 3 9 3
1
(1.0 mole) (Chiral) p p
(enantiomerically pure) For x Î S, n = 0 Þ x = ,-
9 3
CH2CH2CH3 CH2CH3 CH2CH3 7p
3 3 n=1Þx=
2 H Br Br H 9
+ Br Br + +
H Br H Br -5p
n = –1 Þ x =
CH3 CH3 CH3 9
(Achiral) (Chiral) (Chiral)
p p 7 p 5p
\ Sum of all values of x = - + - =0
5
9 3 9 9
CH3 CH3 CH2 – Br
4 4 20 80 7
H Br Br H H H 40. (c) P(T1) = , P(T2) = , P(D) =
100 100 100
+ H H + H H + H H
H Br H Br H Br æ Dö æ Dö
CH3 CH3 CH3 Let P ç ÷ = x, then P ç ÷ = 10x
è T2 ø è T1 ø
(Achiral) (Chiral) (Chiral)
EBD_7801
2016-22 JEE Advanced 2016 Solved Paper
length of perpendicular from P (3, 0, 0) to plane x – y = 0
æ Dö æDö
Also P(D) = P(T1) P ç ÷ + P(T2 ) P ç ÷ 3-0 3
è T1 ø è T2 ø is = =
2 2
7 20 80
Þ = ´ 10x + ´x x y -3 z x y-3 z
100 100 100 Eqn of RS : = = or = = =l
3 -3 3 1 -1 2
7 1 2 2
Þ = x or x =
280 40 If ON is perpendicular to RS, then N (l, -l + 3, 2l)
æ Dö æ Dö Q ON ^ RS Þ 1 ´ l - 1(-l + 3) + 2 ´ 2l = 0
P ç ÷ = 10 and P ç ÷ = 1
è T1 ø 40 è T2 ø 40 1 æ1 5 ö
Þl = Þ N ç , ,1÷
æ D ö 39 2 è2 2 ø
æ Dö 30
Þ Pç ÷ = and P ç T ÷ = 40
è T1 ø 40 è 2ø 1 25 15
\ ON = + +1 =
4 4 2
æ Dö
P ç ÷ P(T2 )
æT ö è T2 ø f (x)
Pç 2 ÷ = 43. (a) f ¢(x) = 2 -
è Dø æ Dö æ Dö x
P ç ÷ P ( T1 ) + P ç ÷ P(T2 )
è T1 ø è T2 ø 1
Þ f ¢(x) + f (x) = 2
x
80 39 If = elog x = x
´
100 40 156 26
= = = \ f(x).x = ò 2x dx = x2 + C
20 30 80 39 186 31
´ + ´
100 40 100 40
C
or f(x) = x + , C ¹ 0 as f(x) ¹ 1
78 26 x
Also =
93 31
æ 1ö
1 (a) lim f ¢ ç ÷ = lim (1 - Cx 2 ) = 1
2 x ®0 + è x ø x ® 0+
41. (c) Let f(x) = 4ax +
x
For x > 0, fmin = 1 æ 1ö æ1 ö 2
(b) lim xf ç ÷ = lim+ x ç + Cx ÷ lim+ 1 + Cx = 1
x®0 + è x ø x ®0 è x ø x ®0
f ¢(x) = 8ax – 1 = 0 Þ x = 1
x2 2a1/3 (c) lim x 2 f ¢ x
x ® 0+
2
æ 1 ö
fmin = 1 Þ 4a ç + 2a1/3 = 1
1/3 ÷ æ Cö 2
è 2a ø = lim x 2 ç1 - 2 ÷ = lim+ x - C = - C
x ®0 + è x ø x ® 0
1
1 (d) for C ¹ 0, f(x) is unbounded as 0 < x < 2
Þ 3a 3 = 1 or a =
27 C C C C
42. (b, c, d) Þ < <¥Þ <x+ <¥
The coordinates of vertices of pyramid OPQRS will be 2 x 2 x
æ3 3 ö P.Q Q
O(0, 0, 0), P (3, 0, 0), Q (3, 3, 0), R (0, 3, 0), S ç , , 3÷ 44. (b,c) PQ = kI Þ = I Þ P -1 =
è2 2 ø k k
dr's of OQ = 1, 1, 0 Also |P| = 12a + 20
dr's of OS = 1, 1, 2 Comparing the third elements of 2nd low on both sides,
\ acute angle between OQ and OS we get

-1 æ 2 ö æ 1 ö p æ 3a + 4 ö 1 - k
= cos ç = cos -1 ç ¹ -ç ÷= ´
è 2 ´ 6 ÷ø è 3 ÷ø 3 è 12a + 20 ø k 8
Þ 24a + 32 = 12a + 20 Þ a = –1
x y z \ |P| = 8
Eqn of plane OQS =
1 1 0
=0 Also PQ = kI Þ |P| |Q| = k3
1 1 2
k2 k2
Þ 8´ = k3 Þ k = 4 Þ |Q| = =8
Þ 2x – 2y = 0 or x – y = 0 2 2
JEE Advanced 2016 Solved Paper 2016-23

(b) 4a – k + 8 = 4 × (–1) – 4 + 8 = 0 1
(c) Now det (P adj Q) = |P| adj Q| \ Solution is : u × y = ò y ´ y2 dy = log y + C
= |P| |Q|2 = 8 × 82 = 29
(d) |Q adj P| = |Q| |P|2 = 29 -y
Þ = log y + C
s-x s- y s -z s-x +s- y+s-z s x+2
45. (a, c, d) = = = =
4 3 2 4+3+ 2 9 As it passes through (1, 3) Þ C = –1 – log 3
5s 6s 7s -y
\x= ,y= ,z= \ = log y – 1 – log 3
9 9 9 x+2
y y
D2 8p Þ log = 1- ...(1)
Area of incircle = pr2 = p 2
= 3 x+2
s 3
Intersection of (1) and y = x + 2
s(s - x)(s - y)(s - z) 8 y
Þ 2
= log = 0 Þ y= 3 Þ x = 1
s 3 3
\ (1, 3) is the only intersection point.
4 ´ 3 ´ 2s3 8 Intersection of (1) and y = (x + 2)2
Þ = or s = 9
9 ´ 9 ´ 9s 3
(x + 2)2 (x + 2)2
\ x = 5, y = 6, z = 7 log = 1 – (x + 2) or log + (x + 2) = 1
3 3
(a) Area of Dxyz = s(s - x)(s - y)(s - z)
(x + 2)2 4
9´ 4 ´ 3´ 2 = 6 6 Q > > 1, "x > 0
3 3
xyz \ LHS > 2, " x > 0 Þ no solution.
(b) Radius of circumcircle, R =
4D 47. (b, c) f(x) = x3 + 3x + 2 Þ f ¢(x) = 3x2 + 3
Also f(0) = 2, f (1) = 6, f(2) = 16, f(3) = 38, f(6) = 236
5´ 6´ 7 35 6 4 And g(f(x)) = x Þ g(2) = 0, g(6) = 1, g(16) = 2, g (38) = 2,
= = =
4´ 6 6 24 35 g (236) = 6
(a) g (f(x)) = x Þ g¢ (f(x)). f ¢(x) = 1
x y z x y z r For g¢(2), f(x) = 2 Þ x = 0
(c) r = 4 R sin sin sin Þ sin sin sin =
2 2 2 2 2 2 4R \ Putting x = 0, we get g¢(f (0)) f¢(0) = 1

2 2 ´ 24 1
= Þ g¢(2) =
3 ´ 4 ´ 35 6 3
(b) h (g(g(x))) = x Þ h¢(g(g(x))). g¢(g(x)). g¢(x) = 1
x+y
2 z For h¢(1), we need g (g(x)) = 1
(d) sin = cos2
2 2 Þ g (x) = 6 Þ x = 236
\ Putting x = 236, we get
s(s - z) 9 ´ 2 3
= = = 1
xy 5´ 6 5 h¢ [g(g(236))] =
g¢ (g(236)).g¢ (236)
dy
46. (a, d) [(x + 2)2 + y (x + 2)] = y2 1
dx Þ h¢ (g(6)) = g¢(6) g¢(236)
2 dx
Þ y - (x + 2)y = (x + 2) 2 1
dy
Þ h¢ (1) = = f ¢(1) f ¢(6)
g¢(f (1)) g¢(f (6))
1 dx 1 1
Þ - = = 6 × 111 = 666
(x + 2) 2
dy (x + 2)y y2 (c) h [g(g(x))] = x
For h (0), g (g(x)) 0 Þ g(x) = 2 Þ x = 16
-1 1 dx du \ Putting x = 16, we get
Let =uÞ =
x+2 2
(x + 2) dy dy h(g(g(16))) = 16
\ Eqn becomes Þ h(0) = 16
(d) h[g(g(x))] = x
du 1 1 For h(g(3)), we need g(x) = 3 Þ x = 38
+ u=
dy y y2 \ Putting x = 38, we get
If = elog = y = y h [g(g(38))] = 38 Þ h (g(3)) = 38
EBD_7801
2016-24 JEE Advanced 2016 Solved Paper
48. (a, b, c) \ Intersection point E of normal and y = tanq
C1 : x2 + y2 = 3 ..(i)
parabola : x2 = 2y ...(ii) tan q 1 - tan 2 q / 2
= x tan q Þ x =
Intersection point of (i) and (ii) in first quadrant 2 2
y2 + 2y – 3 = 0 Þ y = 1 (Q y ¹ -3) 1 - y2
\ Locus of E : x = or y2 = 1 – 2x
\x= 2 2
P( 2,1) æ1 1 ö æ 1 -1 ö
It is satisfied by the points çè 3 , ÷ø and çè 3 , ÷ø
3 3
Equation of tangent to circle C1 at P is 2x + y - 3 = 0
Let centre of circle C2 be (0, k); r = 2 3 x x2 1 + x3
k -3 50. (2) 2x 4x 2 1 + 8x3 = 10
\ = 2 3 Þ K = 9 or –3
3 3x 9x 2 1 + 27x 3
\ Q2 (0, 9), Q3 (0, –3)
(a) Q2 Q3 = 12
1 1 1 1 1 1
(b) R2R3 = length of transverse common tangent 3 6
Þ x 2 4 1 + x 2 4 8 = 10
= (Q2 Q3 ) 2 - (r1 + r2 ) 2 = (12)2 - (4 3)2 = 4 6 3 9 1 3 9 27
1 Operating C2 – C1, C3 – C1 for both the determinants,
(c) Area (DOR2R3) = × R2R3 × length of ^ from origin we get
2
to tangent 1 0 0 1 0 0
1 3
Þ x 2 2 -1 + x 2 2 6 = 10 6
= ´4 6´ 3 = 6 2
2 3 6 -2 3 6 24
1 Þ x3 (–4 + 6) + x6 (48 – 36) = 10
(d) Area (DPQ2Q3) = × Q2Q3 ^ distance of P from
2 Þ 2x3 + 12x6 = 10
1 Þ 6x6 + x3 – 5 = 0
y–axis = ´ 12 ´ 2 = 6 2
2 1
49. (a, c) Circle : x2 + y2 = 1 æ 5ö 3
Þ (6x3 – 5) (x3 + 1) = 0 Þ x = çè ÷ø , – 1
Equation of tangent at P(cos q, sin q) 6
x cos q + y sin q = 1 ...(1) 51. (5) (1 + x)2 + (1 + x)3 + .... + (1 + x)49 + (1 + mx)50
Equation of normal at P
y = x tan q ...(2) é (1 + x)48 - 1ù
Equation of tangent at S is x = 1 = (1 + x)2 ê (1 + x) - 1 ú + (1 + mx)50
ëê ûú
æ 1 - cos q ö æ tan q ö
\ Q ç 1, ÷ø = Q çè 1, 2 ÷ø 1é
è sin q = (1 + x)50 - (1 + x)2 ù + (1 + mx)50
xë û
2
Coeff. of x in the above expansion
Y
= Coeff. of x3 in (1 + x)50 + Coeff. of x2 in (1 + mx)50
(cos q, sin q) Þ 50C3 + 50C2 m2
P
\ (3n + 1) 51C3 = 50C3 + 50C2 m2
Q (1, tan q/2)
50 50
E C3 C2
Þ (3n + 1) = + m2
51 51
X C3 C3
R O S (1, 0)
16 1 2 m2 - 1
Þ 3n + 1 = + m Þn=
17 17 51
Least positive integer m for which n is an integer is
m = 16 and then n = 5
x t2
\ Equation of line through Q and parallel to RS is 52. (1) Let f(x) = ò0 dt - 2x + 1
1+ t4
tan q
y= x2
2 Þ f ¢(x) = - 2 < 0 " x Î [0,1]
1+ x4
JEE Advanced 2016 Solved Paper 2016-25

\ f is decreasing on [0, 1] For above to be possible, we should have


Also f(0) = 1 1
2
a – 1 = 0 and b =
1 t 3!
and f(1) = ò0 dt - 1
1 + t4 1
Þ a = 1 and b =
6
t2 1
For 0 £ t £ 1 Þ 0 £ 4
< æ 1ö
1+ t 2 \ 6 (a + b) = 6 ç1 + ÷ = 7
è 6ø
1 t2 1
\ ò0 4
dt < -1 + i 3
1+ t 2 54. (1) z= Þ z3 = 1 and 1 + z + z2 = 0
2
Þ f(1) < 0
\ f(x) crosses x-axis exactly once in [0, 1] é (- z) r z 2s ù é( - z)r z 2s ù
\ f(x) = 0 has exactly one root in [0, 1] P2 = ê úê ú
êë z 2s z r úû êë z 2s z r úû
x 2 sin bx
( )
53. (7) lim =1
x ® 0 ax - sin x é z 2r + z 4s z 2s (- z)5 + z r ù
ê ú

Þ lim
x 3b
=1
2s
(
r
êë z (-z) + z
r
) 4s
z +z 2r ú
úû
x ® 0 ax - sin x
For P2 = – I we should have
x 3b z2r + z4s = 1 and z2s ((–z)r + zr) = 0
Þ lim =1 Þ z2r + zs + 1 = 0
x ®0 æ x 3 x5 x 7 ö
ax - ç x - + - + ....¥÷ Þ r is odd and s = r but not a multiple of 3.
è 3! 5! 7! ø Which is possible when s = r = 1
\ only one pair is there.
x 3b
Þ lim =1
x ®0 x 3 x5
(a - 1)x + - + ....¥
3! 5!

Paper - 2
PHYSICS A 1
1. (c) Binding energy of nitrogen atom 2. (c) =
A 0 2n
= [8 × 1.008665 + 7 × 1.007825 – 15.000109 × 931]
n A 0 64
Binding energy of oxygen atom \2 = = = 26 Þ n = 6
= [7 × 1.008665 + 8 × 1.007825 – 15.003065] × 931 A 1
\ Difference = 0.0037960 × 931 MeV ...(I) \ time = 6 ´ t 1 = 6 ´ 18 = 108
2
3 8´ 7 e2 3 56
E
Also O = ´ ´ = ´ ´1.44MeV 5
2 R 4pÎ0 5 R 3. (c) P3V5 = constant Þ PV5/3 = constant Þ g =
3
Þ monoatomic gas
3 7´6 e2 3 42 For adiabatic process
EN = ´ ´ = ´ ´1.44MeV
5 R 4pÎ0 5 R
P PiVi 1
3 14
\ E O - E N = ´ ´1.44MeV ...(II)
5 R
2
From (i) & (ii) adiabatic
3 14
´ ´ 1.44 = 0.0037960 ´ 931 PfVf
5 R
\ R = 3.42 fm
V
EBD_7801
2016-26 JEE Advanced 2016 Solved Paper
Therefore object distance for mirror is 25 cm and object
1
´105 ´ 8 ´10-3 - 105 ´ 10-3 is virtual
Pf Vf - Pi Vi 32
W= = 1 1 1 1 1 1
1- g 5 + = \ + =
1- For minor \ v = – 50 cm
3 v u f v 25 50
The image I would have formed as shown had the mirror
25 - 100 75 ´ 3 have been straight. But here the mirror is tilted by 30°.
\ W = ( 3 - 5 ) / 3 = 2 = 112.5J Therefore the image will be tilted by 60° and will be
From first law of thermodynamics q = Du + w formed at A.
\ Du = – w Here MA = 50 cos 60° = 25 cm
\ Du = – 112.5 J and I¢A = 50 sin 60° = 25 3 cm
Now applying first law of thermodynamics for process 6. (b) For C1
1 & 2 and adding q1 + q2 = DU + Pi(Vf – Vi) L.C. = 1MSD – 1VSD
= – 112.5 + 105 (8 – 1) × 10– 3 = 587.55 = 1mm – 0.9 mm = 0.1 mm = 0.01 cm [10 VS D = 9 mm]
4. (a) The heat flow rate is same Reading = MSR + L.C × Verni scale division coinciding
T Main scale division = 2.8 + (0.01) × 7 = 2.87 cm
M 2K
P Q K For C2
10°C 400°C L.C = 1 mm – 1.1 mm [10 VSD = 11 mm]
R H S L.C = – 0.1 mm = – 0.01 cm
x H
dx Reading = 2.8 + (10 – 7) × 0.01 = 2.83 cm

KA ( 400 - T ) 2KA ( T - 10 ) 7. (a, c) In DOAM, OM = l2 + a 2 = 2ha 2 + a 2 = 5a


\ = The circumference of a circle of radius OM will be 2p(5a)
l l
\ T = 140°C = 10pa.
The temperature gradient access Pd is z
Sm Vc r^
dT 140 - 10
= \ dt = 130 dx w
dx 1
Therefore change temperature at a cross-section M A l
q c
distant ‘x’ from P is l a
DT = 130 x q
Extension in a small elemental length ‘dx’ is O M
dl = dx µ DT = dx µ (130x ) For completing this circle once, the smaller disc will

1 10pa
have to take = 5 rounds.
2pa
\ ò dl = 130 µ ò xdx
0 Therefore the C.M. of the assembly rotates about z-
axis with an angular speed of w/5.
1 The angular momentum about the C.M. of the system
\ Dl = 130 × 1.2 × 10–5 × = 78 × 10 – 5 m
2
é1 2ù
1 1 1 Lc = Icw = ê ma ú w
5. (c) For lens - = ë2 û
v u f
é1 2ù 17ma 2w
1
-
1
=
1 +ê ´ 4m ´ ( 2a ) úû ´ w =
ë2 2
v -50 30
\ v = 75 cm m ´ wa + 4m ´ 2wa 9wa
Now vc = =
5m 5
Principal axis of mirror
I¢ ml + 4m ´ 2l 9l
and r1 = =
5m 5

50 cm 5m ´ 9wa 9l 24
30° L of C.M = ´ = 81mwa 2 ´
5 5 5
30°
I A M 81mwa 2 24
LZ = cos q - Icwsin q
Principal 5
axis of
lens 24 24 17ma 2 w 1134
MIRROR = 81mwa
2
´ - = mwa 2
5 5 10 50
JEE Advanced 2016 Solved Paper 2016-27

8. (c) The wavelength of emitted photoelectron as per de 11. (a, b, c, d)


Broglie is At t=0, Capacitors act as short circuit and voltmeter
display –5v
h h
le = = At t = a , Capacitor acts as open circuit and no current
p 2m ( K.E ) flows through voltmeter (Q very high resistance of
When f increases, K.E. decreases and therefore le voltmeter)
increases so it display +5v. (A) is the correct option

( ) , i = VR e
t
When lph increases, N ph decreases , K.E decreases and -t -
q1 = 2Cv 1 - e 2CR
1
2CR

therefore le increases.

),
le is independent of the distance d.
(
t
-t V - t 2CR
q2 = Cv 1 - e 2CR
i2 = e
hc h2 é h ù 2R
Also l + eV - f = l =
2 ê e ú
ph 2mle ë 2 mk. E û
2C R
hc f h2 i1
\ +V- = ...(1)
el ph e 2mele2
V
i2 2R
f
For V >> , f << eV C
e
hc
Also << V . Then from eq (1).
el Ph 5V
1 21 é -t
2CR ù
le µ \ D V = – i2 × 2R + 2 = V êë1 - 2e =0
V C ûú
Therefore if V is made from times, le is approximately (b) is the correct option
half.
io éQ i = io e - t t ù
9. (a, b, d) At t = 1sec, i =
e ëê ûú
1 \ (c) is the correct option
% error in measurement of ‘r’ = ´ 100 = 10%
10 After a long time no current flows since both capcitor
0.52 + 0.56 + 0.57 + 0.54 + 0.59 and voltmeter does not allow current to flow.
Tmean = = 0.556 » 0.56 S 12. (a, b, d)
6
Case (i) : Applying conservation of linear momentum.
0.04 + 0 + 0.01 + 0.02 + 0.03
DT = = 0.016 » 0.02S
6 MV1 = (M + m) V2 – (1) \ V2 = æç M ö÷ V1
\ % error in the measurement of ‘T’ èM+mø

0.02
= ´ 100 = 3.57% v1 v2
0.56
% error in the value of g
From (1)
DT æ DR + Dr ö M (A1 × w1) = (M+m) (A2×w2)
=2 ´ 100 + ç ÷ ´ 100
T è R -r ø K K
\ MA1 × = (M+m) A2 ×
M M+m
= 2 ( 3.57 ) + æç 1 + 1 ö÷ ´100 » 11%
è 60 - 10 ø M
10. (a, c) The range of voltmeter ‘V’ is given by the expression \ A2 = = A1
M+m
V = Ig [Rc + (Rc + R + R)]
V is max in this case as RHS is maximum. Thus (A) is 1
correct. Also E1 = MV12
2
The range of ammeter I is given by the expression
1 1
Ig R c and E2= (M+m)V22 = (M+m)
∗ Ig 1 1 1 1 2 2
I= Where < ∗ ∗
R eq R eq R c R R
M 2 V12 2
1æ M ö 2
Here Req is minimum and therefore I is maximum. Thus ×
(M + m) 2
= ç ÷ V1
2èM+mø
(c) is the correct option.
EBD_7801
2016-28 JEE Advanced 2016 Solved Paper
Clearly E2 < E1
dv
15. (a) Force on the block along flot = m rw2 = m v
m+M dr
The new time Period T2 = 2
K
v r
R 2 dr
ò vdv = ò w2 rdr Þ v = w r - =
2
m+M \
Case (ii) : The new time Period T2 = 2 o R 4 dt
K 2

Also A2 = A1 r t
dr
Here E2 = E1 \ ò R2
= ò w dt
The instantaneous value of speed at X o of the R
4 r2 -
o

combined masses decreases in both the cases. 4


13. (b, d) Path difference at O = d = 0.6003mm
On solving we get
l 600 ´10 -6
Now, = mm = 300×10–6 mm
2 2 R2 R wt
r + r2 - = e
4 2
l 0.6003
For n = d we get n= =2001
2 300 ´10 -6 R 2 R 2 2wt 2 R wt
or r2 – = e + r - 2r e
As n is a whole number, the condition for minima is 4 4 I
satisfied.
Therefore ‘O’ will be dark. R wt
Also, as the screen is perpendicular to the plane
\ r=
4
( e + e-wt )
containing the slits, therefore fringes obtained will be
semi-circular (Top half of the screen is available) ® ®
æ Ù
ö Ù
æ Ù Ùö Ù
16. (b) Frot = Fin + 2m ç Vrot i ÷ ´ w k + m ç w k´ r i ÷ ´ w k
BLV è ø è ø
14. (a, b) i = ...(i)
R Ù ®
æ Ùö Ù
\ m rw2 i = Fin + 2m Vrot wç - j ÷ + mw2 r i
[Counter clockwise direction while entering, Zero when
è ø
completely inside and clockwise while exiting]
B 2 L2 V ®
Ù
F = iLB = ....(ii) Fin = mrV ...(i)
R rot w j
[Toward left while entering and exiting and R wt
zero when completely inside] But r = ée + e - wt ùû

dv B 2 L2 V
\ - mV = dr R
dx R \ dt = Vr = éëwe - we ùû
wt - wt
...(ii)
4
v 2 2 x
BL
\ ò dV = - mR ò dx
v0 0
From (i) and (ii)
® Rw wt - wt Ù
2 2
Fin = 2m
4
(e - e ) w j
BL
V –V0 = – x
mR
®
mRw2 wt Ù

BLx2 2 \ Fin =
2
( e - e - wt ) j
\ V= V0 – ...(iii)
mR
® mR w2 wt Ù Ù
[V decreases from x=0 to x=L, remains constant \ Freaction =
2
( e - e - wt ) j+ mg K
for x = L to x= 3L again decreases from x=3L to x=4L]
From (i) and (iii) 17. (c) After colliding the top plate, the ball will gain negative
charge and get repelled by the top plate and bounce
BL é B2 L2 x ù
i = R ê V0 - mR ú back to the bottom plate.
ë û
Q
[i decreases from x=0 to x=Li becomes zero from x = L to x = 3L 18. (d) Iav a ...(i)
i changes direction and decreases from x = 3L to x = 4L] t
These characteristics are shown in graph (a) and (b) Here Q µ V0 ...(ii)
only.
JEE Advanced 2016 Solved Paper 2016-29

1 2 So, X, Y and Z are [Ag (S2O3)2]3–, Ag2S2O3 and Ag2S


Also S = ut + at respectively.
2
22. (a) Ni2+ with NH3 shows CN = 6 forming [Ni(NH3)6]2+
1 QE 2 1 æ Q ´ 2V0 ö 2 (Octahedral)
h= t = ç ÷´ t
2 m 2 è mh ø Pt2+ with NH3 shows CN = 4 forming [Pt(NH3)4]2+
(5d series CMA, square planner)
1
\ ta – (iii) [Q Q a V0] Zn2+ with NH3 shows CN = 4 forming [Zn(NH3)4]2+
V0 (3d10 configuration, tetrahedral)
From (i), (ii) and (iii) 23. (a)
V0 –
O O
Iav µ 1 = Iav µ V02 H H
V0 COOH C
HO OH O O
CHEMISTRY
ˆˆ† 2H+ (aq) + 2e–
19. (d) At anode : H2(g) ‡ˆˆ
(I) (stabilizes by more H-bonding)
ˆˆ† M2+ (aq)
At cathode : M4+ (aq) + 2e– ‡ˆˆ

O O
COOH H
ˆˆ† 2H+ (aq) +
Net cell reaction : H2(g) + M4+ (aq) ‡ˆˆ
OH O
M2+ (aq)

Now, Ecell = E (
°
M
4+
/M 2 +
- E° +
H /H 2 ) (II) (stabilizes by H-bonding)

2
é H + ù é M 2+ ù –
0.059 log ë û ë û COOH COO
– . 4+ ù
n é
PH2 . M
ë û

2+
0.059 12 ´ éë M ùû OH OH
. log (III) (stabilizes by –I effect)
or, 0.092 = (0.151 – 0) –
2 1´ é M 4+ ù
ë û

COOH COO
é M 2+ ù
ë û
\ +
= 102 Þ x = 2
éM ù4
ë û
20. (d)
• A solution of CH3OH and water shows positive OH OH
(IV) (destabilizes by +M effect)
deviation from Raoult's law, it means by adding CH3OH
intermolecular force of attraction decreases and hence
surface tension decreases. \ acidity order is I > II > III > IV
• By adding KCl in water, intermolecular force of 24. (a)
attraction bit increases, so surface tension increases OH
by small value. O O
O CH2
• By adding surfactant like CH3(CH2)11OSO3– Na +,
surface tension decreases rapidly and after forming H–C–H / NaOH
H
micelle it slightly increases. [Cross aldol reaction]

+ 3- Ag +
S2 O32 - ¾¾¾ ® éë Ag (S2 O3 )2 ùû
Ag
21. (a) ¾¾¾
® OH
OH
(X)
clear solution O CH2
H–C–H / NaOH –
+ HCOO
on standing [Cross cannizaro
Ag 2S2 O3 ¯ ¾¾¾¾¾® Ag 2S ¯ reaction]
(Y) (Z)
white precipitate black precipitate
EBD_7801
2016-30 JEE Advanced 2016 Solved Paper

OH 4
OH O O 4 ´ pr 3
CH2 O
3 = (0.74)
3
H–C–H / H
+ (B) Packing fraction = æ 4r ö
ç ÷
Acetal formation è 2ø
(C) Each FCC unit has effective no of atoms = 4
25. (a, c) Octahedral void = 4
(A) The molecular orbital energy configuration of C2–
2 is Tetrahedral void = 8
2 *2 2
s1s , s1s , s 2s , s*2 2 2 2
2s , p2p = p2p , s2p (D) 4r = a 2
x y z
In the MO of C2– 28. (a, b, c)
2 there is no unpaired electron hence it
is diamagnetic Copper pyrite [CuFeS2]
(B) Bond order of O2+2 is 3 and O2 is 2 therefore bond length Crushing into fine powder
of O2 is greater than O2+2
(C) The molecular orbital energy configuration of N+2 is
Concentrated by froth floatation process
2 *2 2
s1s , s1s , s2s , s*2 2
2s , p2p x = p22p , s12p
y z

1
Bond order of N+2 = (9 - 4) = 2.5 Roasting take place in reverberatory furnace
2
The molecular orbital energy configuration of N–2 is ì 2CuFeS2 + O2 ¾¾ D
® Cu 2 S + 2FeS + SO 2
2 *2 2 ï (air)
s1s , s1s , s2s , s*2 2 2 2 *1 *
2s , p2p = p2p , s2p , p2p = p2p y í
x y z x ï D
î FeS + O 2 ¾¾® FeO + SO 2
1
Bond order of N–2 = (10 - 5) = 2.5
2
éIron is removed as slag of FeSiO 3 ù
(D) He+2 has less energy in comparison to two isolated He Smelting
êFeO + SiO (flux) ¾¾
ë 2
D
® FeSiO 3 ( l)(slag) úû
atoms because some energy is released during the
formation of He+2 from 2 He atoms.
Copper matte (Cu2S + FeS)
26. (a, b)
(A) H-bonding of methanol breaks when CCl4 is added so
bonds become weaker, resulting positive deviation. éSelf reduction occurs in Bessemer converter ù
(B) Mixing of polar and non-polar liquids will produce a Self reduction ê2Cu S + 3O ¾¾ ® 2Cu 2 S + 2SO 2 ( ­) ú
ê 2 2 ú
solution of weaker interaction, resulting positive êëCu 2 S + 2Cu 2 O ¾¾® 6Cu + SO 2 ( ­) úû
deviation
(C) Ideal solution (Blister copper)
(D) –ve deviation because stronger H-bond is formed.
27. (b, c, d)
CCP is ABC ABC …… type packing
(A) In topmost layer, each atom is in contact with 6 atoms Refining of blister copper is done by poling followed by
in same layer and 3 atoms below this layer electrorefining but not by carbon reduction method.

dehydration of HNO
29. (b, d) P4 O10 + 4HNO3 ¾¾¾¾¾¾¾¾
3 ® 4(HPO ) +2 N O
3 2 5
(required product)

(A) P4 + 20HNO3 ¾¾
® 4 H 3 PO 4 + 20NO 2­ + 4H 2 O
(B) N2O5 is diamagnetic in nature
O O

(C) N2O5 ® N N
O O O
N2O5 contains one N–O–N bond not N–N bond.
(D) Na + N2O5 ® NaNO3 + NO2­
(Brown gas)
JEE Advanced 2016 Solved Paper 2016-31

30. (b,c) Invert sugar is an equimolar mixture of D-(+) glucose and D(–) glucose.
H+ +
C12 H 22 O11 + H 2 O ¾¾¾
® C6 H12 O6 + H 2 O ¾¾
® C 6 H12O6
+ sucrose D(+ ) -glucose D( + ) -glucose
( +52°) ( -92°)
1444444 2444444 3
Invert sugar

-92° + 52°
• Specific rotation of invert sugar = = -20°
2
• D-glucose on oxidation with Br2-water produces gluconic acid and not saccharic acid.
31. (c, d)
LiAlH4/(C2H5)2O : Reduces to esters, carboxylic acid, epoxides and aldehydes and ketones.
BH3 in T.H.F : Reduces to –COOH and aldehydes into alcohols but do not reduce to esters and epoxides.
NaBH4 in C2H5OH : Reduces only aldehydes and ketones into alcohols but not to others.
Raney Ni in T.H.F. : Do not reduce to –COOH, –COOR and epoxide but it can reduce aldehyde into alcohols.
Br

32. (b, c, d) +
NaOEt


AlCl3 + 1, 2 H +
Shift
Cl

+
H +

BF 3 1, 2 H

+ +
O–H Shift
H

X 2 (g) ˆˆ†
‡ˆˆ 2X(g)
33. (b)
Initial mole : 1 0

æ beqbm ö
moles at equilibrium : çç 1 - ÷÷ beqbm
è 2 ø
beqbm
1- beqbm
2 P
Partial pressure : ´P æ beqbm ö
æ beqbm ö ç1 + ÷
ç1 + ÷ è 2 ø
è 2 ø

2
(Px )2 beqbm P
\ Kp = =
Px 2 æ beqbm
2 ö
ç1 - ÷
ç 4 ÷
è ø
2
4beqbm P

(
\ Kp = 4 - b 2
eqbm )
Since P = 2 bar

8b2eqbm
So, K p =
( 4 - beqbm
2
)
EBD_7801
2016-32 JEE Advanced 2016 Solved Paper
34. (c) (A) Correct statement.
As on decrease in pressure reaction move indirection where no. of gaseous molecules increase.
(B) Correct statement
At the start of reaction QP < KP so dissociation of X2 take place spontaneousely.
(C) Incorrect statement as
2
8beq 8 ´ (0.7) 2
KP = = >1
4 - b2eq 4 - (0.7) 2

35. (a)
36. (b)
O

COOH C–NH2 NH2


Å NH3 /D Br2 / NaOH
KMnO /H
¾¾¾¾¾¾
4 ® ¾¾¾¾
® ¾¾¾¾¾
®

COOH C–NH2 NH2


(O) (P) (Q) (R)
O

O O
CH3 Br O

CH3 - CH -C -OC 2H 5
N–CH–C–OC2H5 ¬¾¾¾¾¾¾¾¾ NH
KOH
O
(S)
O O

Å
H 3O
CH3–CH–COOH Alanine

NH2
(T)

MATHEMATICS Þ q32 = 50 × 4 = 200

é 1 0 0ù é 0 0 0ù q31 + q32 20600


ê ú ê ú \ = = 103
37. (b) P = 4 1 0 = I + ê 4 0 0ú = I + A
ê ú q 21 200
ê ú ê ú
ë16 4 1 úû
ê ë16 4 0úû
ê 38. (b) logc b1, logc b2,...., logcb101 are in A.P.
Þ b1, b2,...., b101 are in G.P.
é 0 0 0ù é0 0 0 ù Also a1, a2,...., a101 are in A.P.
ê2 ú 3 ê ú
A = 0 0 0 and A = ê0 0 0 ú
ê ú where a1 = b1 are a51 = b51.
ê ú ê ú
ë16 0 0úû
ê ë0 0 0 úû
ê \ b2, b3,...., b50 are GM’s and a2, a3,...., a50 are AM’s
between b1 and b51.
\ An = O, n ³ 3
Q GM < AM Þ b2 < a2, b3 < a3,...., b50 < a50
Now P50 = (I + A)50 = 50C0 I50 + 50C1 I49 A + 50C2
Q b1 + b2 +....+ b51 < a1 + a2 +....+ a51
+ I48 A2 + O
Þ t<s
= I + 50A + 25 × 49 A2.
Also a1, a51, a101 are in AP
\ Q = P50 – I = 50A + 25 × 49 A2.
b1, b51, b101 are in GP
Þ q21 = 50 × 4 = 200
Q a1 = b1 and a51 = b51
Þ q31 = 50 × 16 + 25 × 49 × 16 = 20400
\ b101 > a101
JEE Advanced 2016 Solved Paper 2016-33

The graph of given region is as follows-


13 1
39. (c) å æp p ö æp kpö
k =1 sin ç + k - 1 ÷sin ç
ç (
è4
) ÷ ç +
6ø è4 6 ø
÷
÷

(1, 2)
é ìp kp æp pöïü ù (– 4, 1)
ê sin ïí + -çç + (k - 1) ÷÷ý ú P
Q
13 1 ê ï 4 6 è 4 6 ø þï ú
î
å ê
= k =1 sin p ê æp pö æp kpö ú
ú
ê sin ç
ç + k - 1 ÷ sin
( ) 6 ÷ø çè 4 6 ÷ø úú
ç + ÷ S T R
6ëê è4 û (– 3, 0)

13 é æp pö æp kpöù
å 2 êêëcot ççè 4 + (k - 1) 6 ÷÷ø - cot ççè 4 + ÷÷ú
6 ø úû
k =1 Required area = Area (trap PQRS) – Area (PST + TQR)
éì æp pöüï æp 2pö 1 é -3 1 ù
ï p æp pö = ´(1 + 2)´5 - êò-4 - x - 3 dx +ò-3 x + 3 dx ú
= 2 êícot - cot çç + ÷÷ý + cot çç + ÷÷ - cot ç
ç + ÷÷ 2 ë û
ê
ëï
î 4 è 4 6 øï
þ è4 6 ø è4 6 ø
éæ 3/2 ö-3 æ 3/2 ö1 ù
ì
ï æp 12pö æp 13pöüïù 15 êç 2 (- x - 3) ÷ ç 2 (x + 3) ÷ ú
= - êç ÷ +ç ÷ úú
+ ...... + ícot çç 4 + 6 ÷÷ - cot çç 4 + 6 ÷÷ýúú 2 êç -3 ÷ ç 3 ÷
ï è
î ø è ø þïû ê
ëè ø -4 è ø -3 úû

é p æp 13pöù é 5pù 15 é2 16 ù 15 3
= 2 êêcot - cot çç + ÷ú = 2 ê1 - cot ú
÷ = -ê + ú = - 6 = sq.units
ë 4 è 4 6 øû
ú ê
ë 12 ûú 2 ë ê3 3 ûú 2 2
42. (c) P, the image of point (3, 1, 7) in the plane x – y + z = 3 is
é ù given by
ê1 - 3 - 1ú = 2 é1 - 2 - 3 ù = 2
2
= ê
ë 3 + 1 ûú ëê úû ( ) ( 3 -1 ) x - 3 y -1 z - 7 -2 (3 - 1 + 7 - 3)
= = =
2 1 -1 1 12 + 12 + 12
p/2 x cos x
40. (a) I =ò dx ....(i) x - 3 y -1 z - 7
-p/2
1 + ex Þ = = =–4
1 -1 1
Applying òa f (x) dx
b Þ x = –1, y = 5, z = 3
\ P(–1, 5, 3)
Now equation of plane through (–1, 5, 3) and containing
b
= òa f (a + b - x ) dx, we get
x y z
the line = = is
x 2 1 2 1
p/2 e x cos x
I =ò dx ....(ii)
-p/2
1 + ex x y z
-1 5 3 = 0 Þ – x + 4y – 7z = 0
Adding (i) and (ii)
1 2 1
p/2
2 p/2 2
2I =ò-p/2 x cos x dx = 2ò0 x cos x dx or x – 4y + 7z = 0
43. (b, c)
2 p
I= é x 2
sin x + 2x cos x - 2 sin x ù2 = p - 2 é ù x /n
ëê ûú0 4 ê n æ nö æ nö ú
ê n (x + n) ççx + ÷÷... ç
çx + ÷ ÷ ú
2 n
41. (c) y³ x + 3 Þ y2 = x + 3 f(x) = lim êê è ø è ø ú
n ®¥ ê æ 2ö æ 2 öú
2 2 n n ú
( x2 + ÷ x2 + 2 ÷
2 ïì- (x + 3) if x < -3
ên! x + n ç
ê
ë
ç
è
÷)... ç
4 ø è ç ÷
n ø úû
ú
Þ y =í ....(i)
ïî (x + 3) if x ³ -3
é ù x/n
ê 2n æ x öæ x 1 ö æ x 1 ö ú
x+9 ê n ç ç n + 1÷ç
÷ç n + 2 ÷÷.... ç
ç n + n ÷÷ ú
Also y £ and x £ 6 ....(ii) ê è ø è ø è ø ú
5 = lim ê
n ®¥ ê æ x 2 öæ x 2 1 ö æ x 2 1 ö úú
2n
Solving (i) and (ii) we get intersection points as (1, 2), ên × n! ç ÷ç ÷ ç ÷
ç n 2 + 1÷ç n 2 + 22 ÷.... ç n 2 + n 2 ÷ úú
(6, 3), (– 4, 1), (–39, –6) ê
ë è ø è ø è øû
EBD_7801
2016-34 JEE Advanced 2016 Solved Paper
44. (a, b) f(x) = a cos (|x –x|) + b |x| sin (|x3 + x|)
3 3

x /n (a) If a = 0, b = 1
é ù
ê æ x öæ x 1 ö æ x 1 ö ú
ê ç + 1÷ç
ç ÷ç + ÷÷.... ç
ç + ÷ ÷ ú Þ f(x) = |x| sin |x3 + x|
= lim ê èn ø èn 2ø èn n ø ú = x sin (x3 + x), x Î R
ê öæ x 2 1 ö æ x 2 1 ö úú
n ®¥ êæ x 2
êç1. + 1÷ç 2. + ÷ .... çn. + ÷ \ f is differentiable every where
ç 2 ÷ç 2 2÷ ç 2 n ÷ú
êè n
ë øè n ø è n ø ûú (b), (c) If a = 1, b = 0 Þ f(x) = cos3 (|x3 – x|)
= cos3(x3 – x)
which is differentiable every where.
x éê n æ x 1 ö n æç rx 2 1 ö÷ùú
Þ ln f(x) = n ®¥ n êå ln çç n + r ÷÷ - å ln ç 2 + r ÷ú
lim (d) when a = 1, b = 1, f(x) = cos(x3 – x) + x sin (x3 + x)
êër =1 è ø r =1 è n ø ûú which is differentiable at x = 1
\ Only a and b are the correct options.
é ì æ rx öüï ìï 1 æ r 2 x 2 öüïù
x ên ï 1 ú
lim êå íln + ln çç + 1÷÷ý - íln + ln çç 2 + 1÷÷ýú f ( x) g ( x ) é0 ù
= n ®¥ n êër =1 îï r è n ø þï ï î r è n ø ï
þúû 45. (a, d) lim =1 ê form ú
x ® 2 f ¢ ( x) g¢ ( x ) ë0
ê úû
én æ æ r 2 x 2 öù
x ê ln ç1 + rx ö ç1 + ÷ú f ¢ ( x ) g ( x ) + f ( x ) g¢ ( x )
= nlim
®¥ n ê å ç ÷
÷ - ln
ç 2 ÷ú Þ xlim =1
êër =1 è n ø è n ø úû ®2 f ¢¢ ( x ) g¢ ( x ) + f ¢ ( x ) g¢¢ ( x )

1 1 g¢ (2) f (2)
= xòln (1 + xy) dy - xòln 1 + x 2 y2 dy
( ) Þ f ¢¢ 2 g¢ 2 =1 Þ f ( 2 ) = f ¢¢ ( 2 )
0 0 () ()
Let xy = t Þ x dy = dt \ f(x) – f ¢¢ ( x ) = 0 for atleast one x Î R.
x x 2
\ ( )
ln f(x) = ò0 ln (1 + t ) dt -ò0 ln 1 + t dt Q Range of f(x) is ( 0, ¥ )
\ f(x) > 0, xÎ R
x æ 1+ t ö
ln f (x ) =ò
0
ln ç
ç ÷ dt
÷ Þ f(2) > 0 Þ f ¢¢ ( 2 ) > 0
è1 + t 2 ø
Þ f has a local minimum at x = 2
f ¢ (x) æ 1+ x ö 46. (b, c) f(x) = [x2 – 3] is discontinuous at all integral points in
Þ = ln çç ÷÷
f (x ) è1 + x 2 ø é 1 ù
ê - 2 , 2ú
ë û
f ¢ (2) æ3 ö
Þ = ln çç ÷÷ < 0
f (2) è5 ø Which happens when x = 1, 2, 3 ,2

Þ f ¢ (2) < 0 \ (c) is correct é 1 ù


\ f is discontinuous exactly at four points in ê - , 2ú
ë 2 û
f ¢ (3)
æ 2 ö f ¢ (2)
= ln çç ÷÷ <
( ) è 5 ø f (2) \ (d) is not correct
and f 3
Also g ( x ) = ( x + 4x - 7 ) f ( x )

æ 1+ x ö æ 1 ö
Also f ¢ (x) = f(x) ln çç ÷÷ > 0, , x Î (0, 1) Here f is not differentiable at x = 1, 2, 3 Î ç - 2 , 2 ÷
è1 + x 2 ø è ø
\ f is an increasing function. 7
and |x| + |4x – 7| is not differentiable at O and
1 æ1ö 4
\ <1 Þ f ç ÷ £ f (1)
ç ÷
2 è2ø But f(x) = 0, xÎ é ù
ê 3 , 2 ûú
ë
\ (a) is not correct
7
1 2 æ1 ö æ 2 ö \ g(x) becomes differentiable at x =
and < Þ f ç
ç ÷÷£ f ç
ç ÷
÷ 4
3 3 è3 ø è3ø
Hence g(x) is non-differentiable at four points i.e.,
\ (b) is correct
0,1, 2, 3
Hence (b) and (c) are the correct options.
JEE Advanced 2016 Solved Paper 2016-35

49. (b, c, d)
1
47. (a, c, d) z = = x + iy ax + 2y = l
a + ibt
3x – 2y = m
a - ibt
Þ x + iy = a 2
a + b2 t 2
2
For unique solution, ¹ Þ a ¹ –3
3 -2
a -bt \ (b) is the correct option.
Þ x= 2 , y= 2
2 2
a +b t a + b2 t 2 For infinite many solutions and a = – 3
1 x -3 2 l l
Þ x 2 + y2 = 2 2 2
= = = Þ =-1 or l + m = 0
a +b t a 3 -2 m m
x \ (c) is the correct option.
Þ x 2 + y2 - =0
a
-3 2 l
Also if l + m ¹ 0, then = ¹
æ 1 ö 3 -2 m
\ Locus of z is a circle with centre ç , 0 ÷ and
è 2a ø Þ system has no solution.
1 \ (d) is the correct option.
radius = 2 a irrespective of ‘a’ +ve or –ve r
50. (b, c) û ´ v = 1 Þ v sin q = 1 ...(i)
Also for b = 0, a ¹ 0, we get, y = 0
r
\ locus is x-axis
and for a = 0, b ¹ 0 we get x = 0
( )
ˆ uˆ ´ v = 1 Þ v sin q cos a = 1
w. ...(ii)

Þ locus is y-axis. where a is the angle between ŵ and a vector ^ lat to


\ a, c, d are the correct options. r r
48. (a, c, d) u&v .
Let point P on parabola y2 = 4x be (t2, 2t) From (i) and (ii) cos a = 1 Þ a = 0°
... PS is shortest distance, therefore PS should be the r r
Þ ŵ is perpendicular to the plane containing u & v
normal to parabola.
r
Þ ŵ is perpendicular to u
r
2 Clearly there can be infinite many choices for v .
P(t , 2t)
Also if û lies in xy plane i.e., û = u1iˆ + u 2 ˆj then
Q r
ŵ.u = 0

S(2, 8) Þ u1 + u2 = 0 Þ u1 = u 2

Also if û lies in xz plane, i.e., û = u1iˆ + u 3kˆ then


r
Equation of normal to y2 = 4x at P (t2, 2t) is ŵ.u = 0 Þ u1 + 2u3 = 0 Þ |u1| =2|u3|
y – 2t = – t(x – t2) Hence (b) and (c) are the correct options.
It passes through S(2, 8)
For (Qs. 51-52)
\ 8 – 2t = – t(2 – t2) Þ t3 = 8 or t = 2
\ P(4, 4) (X, Y) = {(6, 0), (4, 1), (3, 3), (2, 2), (4, 4), (0, 6)}
51. (b) P(X > Y) = P(T1 wins 2 games or T1 win one game other
-1 1
Also slope of tangent to circle at Q = = is a draw)
Slope of PS 2
1 1 æ1 1 1 1 ö 1 1 5
Equation of normal at t = 2 is 2x + y = 12 = ´ +ç
ç ´ + ´ ÷ ÷= + =
Clearly x-intercept = 6 2 2 è 2 6 6 2 ø 4 6 12
SP = 2 5 and SQ = r = 2 52. (c) P(X = Y) = P(T1 wins 1 game loses other game or both
\ Q divides SP in the ratio SQ : PQ the games draw)

= 2: 2 ( ) (
5 - 1 or 5 +1 : 4) æ 1 1 1 1 ö 1 1 1 1 13
= ç ´ + ´ ÷+ ´ = + =
Hence a, c , d are the correct options. è 2 3 3 2 ø 6 6 3 36 36
EBD_7801
2016-36 JEE Advanced 2016 Solved Paper
For (Qs. 53-54) Putting y = 0 in above equation, we get
x 2 y2 8 1 x =-
9
For ellipse + =1, e = 1 - =
9 8 9 3 10
\ F1(– 1, 0) and F2 (1, 0) æ 9 ö
Parabola with vertex at (0, 0) and focus at F2(1, 0) is y2 = 4x. \ Orthocentre ç - , 0 ÷
è 10 ø
æ3 ö 54. (c) Tangents to ellipse at M and N are
Intersection points of ellipse and parabola are M ç
ç , 6÷
÷
è 2 ø
x y 6 x y 6
+ = 1 and - =1
æ3 ö 6 8 6 8
and N ç
ç ,- 6 ÷
÷
è2 ø Their intersection point is R (6, 0)
æ3 ö
æ3 ö Also normal to parabola at M ç , 6 ÷ is
Mç , 6 ÷ è2 ø
è 2 ø
6æ 3ö
y- 6 = - çx - ÷
2 è 2ø
Q
æ7 ö
F1(–1, 0) F2(1, 0) Its intersection with x-axis is Q ç
ç , 0÷
÷
è2 ø

1 5 5 6
æ3 ö Now ar (DMQR) = ´ ´ 6=
Nç ,- 6 ÷ 2 2 4
è2 ø
Also area (MF1NF2) = 2 × Ar (F1MF2)
53. (a) For orthocentre of DF1MN, clearly one altitude is x- 1
= 2´ ´ 2´ 6 = 2 6
axis i.e. y = 0 and altitude from M to F1N is y - 6 2

5 æ 3ö ar (DMQR ) 5 6
\ = =5 : 8
= çx- ÷
ç ÷ ar (MF1NF2 ) 4´2 6
2 6 è 2ø
Telegram @unacademyplusdiscounts

Join Us on Telegram for More Such Books

https://telegram.me/unacademyplusdiscounts

Join us from the above link or search ''unacademyplusdiscounts'' in Telegram


JEE ADVANCED 2015
1. The question paper consists of three parts (Physics, Chemistry and Mathematics). Each part consists of three
sections.
2. Section 1 contains 8 questions. The answer to each of the questions is a single-digit integer ranging from 0 to 9
(both inclusive).
3. Section 2 contains 10 multiple choice questions. Each question has four choices (a), (b), (c) and (d) out of
which ONE OR MORE THAN ONE are correct.
4. Section 3 contains 2 multiple matching questions. One or more entries in Column - I may match with one or
more entries in Column - II.

PAPER - 1

4. Consider a concave mirror and a convex lens (refractive index


PHYSICS = 1.5) of focal length 10 cm each, separated by a distance of
SECTION - I 50 cm in air (refractive index = 1) as shown in the figure. An
object is placed at a distance of 15 cm from the mirror. Its
This section contains 8 questions. Each question, when worked
erect image formed by this combination has magnification
out will result in one integer from 0 to 9 (both inclusive).
M1. When the set-up is kept in a medium of refractive index
1. Two spherical stars A and B emit blackbody radiation. The
radius of A is 400 times that of B and A emits 104 times the 7 M
, the magnification becomes M2. The magnitude 2 is
6 M1
æ lA ö
power emitted from B. The ratio ç l ÷ of their wavelengths
è Bø
lA and lB at which the peaks occur in their respective
radiation curves is
2. A nuclear power plant supplying electrical power to a village 15cm
uses a radioactive material of half life T years as the fuel. The
amount of fuel at the beginning is such that the total power
requirement of the village is 12.5% of the electrical power 50cm
available from the plant at that time. If the plant is able to 5. An infinitely long uniform line charge distribution of charge
meet the total power needs of the village for a maximum period per unit length l lies parallel to the y-axis in the y – z plane at
of nT years, then the value of n is
3. A Young's double slit interference arrangement with slits S1 3
z= a (see figure). If the magnitude of the flux of the
2
4
and S2 is immersed in water (refractive index = ) as shown electric field through the rectangular surface ABCD lying in
3
in the figure. The positions of maximum on the surface of lL
the x – y plane with its centre at the origin is ne
water are given by x 2 = p2 m2l2 – d2 , where l is the 0
wavelength of light in air (refractive index = 1), 2d is the (e0 = permittivity of free space), then the value of n is
separation between the slits and m is an integer. The value of
p is z

S1 L
D 3
C a
d
Air x 2
x
a y
d O
S2 Water
A
B
EBD_7801
2015-2 JEE Advanced 2015 Solved Paper
6. Consider a hydrogen atom with its electron in the nth orbital.
w2
An electromagnetic radiation of wavelength 90 nm is used to (a) E1w1 = E2w2 (b) 2
ionize the atom. If the kinetic energy of the ejected electron is w1 = n
10.4 eV, then the value of n is (hc = 1242 eV nm)
E1 E2
7. A bullet is fired vertically upwards with velocity v from the (c) w1w2 = n2 (d) =
surface of a spherical planet. When it reaches its maximum w1 w2
11. A ring of mass M and radius R is rotating with angular speed
1 th
height, its acceleration due to the planet's gravity is of w about a fixed vertical axis passing through its centre O with
4
its value of the surface of the planet. If the escape velocity M
two point masses each of mass at rest at O. These masses
from the planet is vesc = v N , then the value of N is (ignore 8
energy loss due to atmosphere) can move radially outwards along two massless rods fixed
8. Two identical uniform discs roll without slipping on two on the ring as shown in the figure. At some instant the angular
different surfaces AB and CD (see figure) starting at A and C 8
with linear speeds v1 and v2, respectively, and always remain speed of the system is w and one of the masses is at a
9
in contact with the surfaces. If they reach B and D with the
same linear speed and v1 = 3 m/s then v2 in m/s is (g = 10 m/s2) 3
distance of R from O. At this instant the distance of the
5
v1 = 3m/s
A other mass from O is
30m w
B

v2 O
C
27m
D 2 1
(a) R (b) R
3 3
3 4
SECTION - II (c) R (d) R
5 5
This section contains 10 multiple choice questions. Each question 12. The figures below depict two situations in which two infinitely
has 4 choices (a), (b), (c) and (d) out of which ONE or MORE long static line charges of constant positive line charge
THAN ONE are correct. density l are kept parallel to each other. In their resulting
9. Planck’s constant h, speed of light c and gravitational constant electric field, point charges q and –q are kept in equilibrium
G are used to form a unit of length L and a unit of mass M. between them. The point charges are confined to move in the
Then the correct option(s) is(are) x direction only. If they are given a small displacement about
Mµ their equilibrium positions, then the correct statement(s) is(are)
(a) c (b) M µ G
(c) Lµ h (d) L µ G
l l l l
10. Two independent harmonic oscillators of equal mass are
oscillating about the origin with angular frequencies w1 and
w2 and have total energies E1 and E2, respectively. The x x
+q –q
variations of their momenta p with positions x are shown in
a a
the figures. If = n2 and = n, then the correct equation(s)
b R
is(are) (a) Both charges execute simple harmonic motion
P (b) Both charges will continue moving in the direction of
p
their displacement
Energy = E2 (c) Charge +q executes simple harmonic motion while
Energy = E1 charge –q continues moving in the direction of its
b x displacement
x
a R (d) Charge –q executes simple harmonic motion while
charge +q continues moving in the direction of its
displacement
JEE Advanced 2015 Solved Paper 2015-3

13. Two identical glass rods S1 and S2 (refractive index = 1.5)


have one convex end of radius of curvature 10 cm. They are
placed with the curved surfaces at a distance d as shown in
the figure, with their axes (shown by the dashed line) aligned. Q
When a point source of light P is placed inside rod S1 on its
axis at a distance of 50 cm from the curved face, the light rays
emanating from it are found to be parallel to the axis inside
Al 50mm
S2. The distance d is
Fe
S1 S2
P 2mm P
7mm
50 cm d

2475 1875
(a) 60 cm (b) 70 cm (a) µW (b) µW
64 64
(c) 80 cm (d) 90 cm
14. A conductor (shown in the figure) carrying constant current 1875 2475
(c) µW (d) µW
r 49 132
I is kept in the x-y plane in a uniform magnetic field B . If F is
the magnitude of the total magnetic force acting on the 17. For photo-electric effect with incident photon wavelength l,
conductor, then the correct statement(s) is(are) the stopping potential is V0. Identify the correct variation(s)
of V0 with l and 1/l.
y
R R V0 V0
l p/6 p/4 x
L R R L (a) (b)
r l
(a) If B is along $z , F µ (L + R) l

r
(b) If B is along x̂ , F = 0 V0
(c) (d) V0
r
(c) If B is along ŷ , F µ (L + R)
r 1/l 1/l
(d) If B is along $z , F = 0
15. A container of fixed volume has a mixture of one mole of 18. Consider a Vernier callipers in which each 1 cm on the main
hydrogen and one mole of helium in equilibrium at temperature scale is divided into 8 equal divisions and a screw gauge
T. Assuming the gases are ideal, the correct statement(s) is with 100 divisions on its circular scale. In the Vernier callipers,
(are) 5 divisions of the Vernier scale coincide with 4 divisions on
(a) The average energy per mole of the gas mixture is 2RT the main scale and in the screw gauge, one complete rotation
of the circular scale moves it by two divisions on the linear
(b) The ratio of speed of sound in the gas mixture to that in
scale. Then :
helium gas is 6/5 (a) If the pitch of the screw gauge is twice the least count
(c) The ratio of the rms speed of helium atoms to that of of the Vernier callipers, the least count of the screw
hydrogen molecules is 1/2 gauge is 0.01 mm
(d) The ratio of the rms speed of helium atoms to that of (b) If the pitch of the screw gauge is twice the least count
of the Vernier callipers, the least count of the screw
1
hydrogen molecules is gauge is 0.005 mm
2
(c) If the least count of the linear scale of the screw gauge
16. In an aluminium (Al) bar of square cross section, a square is twice the least count of the Vernier callipers, the
hole is drilled and is filled with iron (Fe) as shown in the least count of the screw gauge is 0.01 mm
figure. The electrical resistivities of Al and Fe are 2.7 × 10–8 (d) If the least count of the linear scale of the screw gauge
W m and 1.0 × 10–7 W m, respectively. The electrical resistance is twice the least count of the Vernier callipers, the
between the two faces P and Q of the composite bar is least count of the screw gauge is 0.005 mm
EBD_7801
2015-4 JEE Advanced 2015 Solved Paper
SECTION - III CHEMISTRY
This section contains TWO questions. Each question contains
two columns, Column I and Column II. Column I has four entries SECTION - I
(A), (B), (C) and (D). Column II has five entries (P), (Q), (R), (S)
and (T). Match the entries in Column I with the entries in Column This section contains 8 questions. Each question, when worked
II. One or more entries in Column I may match with one or more out will result in one integer from 0 to 9 (both inclusive).
entries in Column II. 1. Among the triatomic molecules/ions, BeCl2, N 3– , N2O,
19. Match the nuclear processes given in column I with the
appropriate option(s) in column II. NO+2 , O3, SCl2, ICl 2– , I3– and XeF2, the total number of
Column I Column II
linear molecule(s)/ion(s) where the hybridization of the central
(A) Nuclear fusion (P) Absorption of thermal
atom does not have contribution from the d-orbital(s) is
neutrons by 235
92 U [Atomic number : S = 16, Cl = 17, I = 53 and Xe = 54]
(B) Fission in a nuclear (Q) 60 Co nucleus 2. Not considering the electronic spin, the degeneracy of the
27
second excited state (n = 3) of H atom is 9, while the
reactor
degeneracy of the second excited state of H– is
(C) b-decay (R) Energy production in
stars via hydrogen 3. All the energy released from the reaction X ® Y, DrG° = –193
conversion to helium kJ mol–1 is used for oxidizing M+ as M+ ® M3+ + 2e–,
(D) g-ray emission (S) Heavy water E° =–0.25 V
(T) Neutrino emission Under standard conditions, the number of moles of M+
20. A particle of unit mass is moving along the x-axis under the oxidized when one mole of X is converted to Y is
influence of a force and its total energy is conserved. Four
[F = 96500 C mol–1]
possible forms of the potential energy of the particle are given
in column I (a and U0 constants). Match the potential energies 4. If the freezing point of a 0.01 molal aqueous solution of a
in column I to the corresponding statement(s) in column II. cobalt(III) chloride-ammonia complex (which behaves as a
Column I Column II strong electrolyte) is –0.0558°C, the number of chloride(s) in
the coordination sphere of the complex is
2
U0 é æ x ö2 ù [Kf of water = 1.86 K kg mol–1]
(A) U1(x) = ê1 - ç ÷ ú (P) The force acting on the
2 êë è a ø úû 5. The total number of stereoisomers that can exist for M is
particle is zero at x = a
2
H3C CH3
U0 æ x ö
(B) U2(x) = ç ÷ (Q) The force acting on the
2 èaø
particle is zero at x = 0

U 0 æ x ö2 H3C
(C) U3(x) = ç ÷ (R) The force acting on the
2 èaø
O
M
é æ x ö2 ù
exp ê- ç ÷ ú particle is zero at x = – a
a
ëê è ø ûú 6. The number of resonance structures for N is

U0 OH
(D) U4(x) = (S) The particle experiences
2 NaOH
N
éx 1æ x ö 3ù
ê – ç ÷ ú an attractive force towards
êë a 3 è a ø úû 7. The total number of lone pairs of electrons in N2O3 is
x = 0 in the region |x| < a 8. For the octahedral complexes of Fe3+ in SCN– (thiocyanato-S)
(T) The particle with total and in CN– ligand environments, the difference between the
U0 spin-only magnetic moments in Bohr magnetons (when
energy can oscillate approximated to the nearest integer) is
4
about the point x = – a [Atomic number of Fe = 26]
JEE Advanced 2015 Solved Paper 2015-5

SECTION - II 12. The major product of the reaction is


This section contains 10 multiple choice questions. Each question H3C NaNO2, aqueous HCl
has 4 choices (a), (b), (c) and (d) out of which ONE or MORE 0°C
THAN ONE are correct.
9. The major product of the following reaction is
H3C H 3C CO2H
O i. KOH, H 2O
ii. H,+ heat (a) (b)

CH3
O H3C CO2H H3C
CH3 CH3 (c) (d)
O O
(a) (b)
13. The correct statement(s) about Cr2+ and Mn3+ is(are)
[Atomic numbers of Cr = 24 and Mn = 25]
O O CH3 (a) Cr2+ is a reducing agent
CH3 (b) Mn3+ is an oxidizing agent
(c) (d) (c) Both Cr2+ and Mn3+ exhibit d4 electronic configuration
(d) When Cr2+ is used as a reducing agent, the chromium
10. In the following reaction, the major product is ion attains d5 electronic configuration
CH3 14. Copper is purified by electrolytic refining of blister copper.
CH2 The correct statement(s) about this process is(are)
HC 1 equivalent HBr (a) Impure Cu strip is used as cathode
2

(b) Acidified aqueous CuSO4 is used as electrolyte


CH3 (c) Pure Cu deposits at cathode
CH3
(d) Impurities settle as anode-mud
CH3
(a) H2C (b) H C 15. Fe3+ is reduced to Fe2+ by using
3
Br (a) H2O2 in presence of NaOH
Br
(b) Na2O2 in water
CH3 CH3 (c) H2O2 in presence of H2SO4
(c) (d) Br (d) Na2O2 in presence of H2SO4
H2C Br H 3C 16. The %yield of ammonia as a function of time in the reaction
11. The structure of D-(+)-glucose is N2(g) + 3H2(g) 2NH3(g), DH < 0 at (P, T1) is given below
CHO
H OH
% y ie ld

HO H
H OH
H OH
time
CH2OH If this reaction is conducted at (P, T2), with T2 > T1, the
The structure of L-(–)-glucose is %yield of ammonia as a function of time is represented by

T2 T1

T1
% y ie ld

T2
% y i e ld

(a) (b)
(a) (b)

time time

T2
T1
T1
% y ie ld

% y ie ld

(c) (d) T2
(c) (d)

time time
EBD_7801
2015-6 JEE Advanced 2015 Solved Paper
17. If the unit cell of a mineral has cubic close packed (ccp) array MATHEMATICS
of oxygen atoms with m fraction of octahedral holes occupied
by aluminium ions and n fraction of tetrahedral holes occupied SECTION - I
by magnesium ions, m and n, respectively, are
This section contains 8 questions. Each question, when worked
1 1 1 out will result in one integer from 0 to 9 (both inclusive).
(a) , (b) 1,
2 8 4 1. If the normals of the parabola y2 = 4x drawn at the end points
1 1 1 1 of its latus rectum are tangents to the circle (x – 3)2 + (y + 2)2
(c) , (d) , = r2, then the value of r2 is
2 2 4 8
18. Compound(s) that on hydrogenation produce(s) optically
inactive compound(s) is (are)
2.
[ x], x £ 2
Let f : R ® R be a function defined by f ( x) = 0, x > 2 {
H Br H Br where [x] is the greatest integer less than or equal to x, if
(a) (b) CH3 2
H 3C CH 3 H 2C xf ( x 2 )
I= ò dx
2 + f ( x + 1) , then the value of (4I – 1) is
H Br Br H
-1

(c) H2C (d) H C 3. A cylindrical container is to be made from certain solid


CH3 CH 3
2 material with the following constraints: It has a fixed inner
CH3 volume of V mm3, has a 2 mm thick solid wall and is open at
the top. The bottom of the container is a solid circular disc
SECTION - III of thickness 2 mm and is of radius equal to the outer radius
This section contains TWO questions. Each question contains of the container.
two columns, Column I and Column II. Column I has four entries If the volume of the material used to make the container is
minimum when the inner radius of the container is 10 mm,
(A), (B), (C) and (D). Column II has five entries (P), (Q), (R), (S)
and (T). Match the entries in Column I with the entries in Column V
then the value of is
II. One or more entries in Column I may match with one or more 250p
entries in Column II.
p
x2 +
19. Match the anionic species given in Column-I that are present 6
in the ore(s) given in Column-II. 4. Let F ( x) = ò 2cos 2 t (dt ) for all xÎR and
Column-I Column-II x
(A) Carbonate (P) Siderite
é 1ù
(B) Sulphide (Q) Malachite f : ê0, ® [0, ¥) be a continuous function. For
(C) Hydroxide (R) Bauxite ë 2 úû
(D) Oxide (S) Calamine é 1ù
(T) Argentite a Î ê0, ú , if F ¢(a) + 2 is the area of the region bounded
ë 2û
20. Match the thermodynamic processes given under Column-I
by x = 0, y = 0, y = f(x) and x = a, then f(0) is
with the expressions given under Column-II. 5. The number of distinct solutions of the equation
Column-I Column-II
5
(A) Freezing of water at (P) q = 0 cos2 2 x + cos 4 x + sin 4 x + cos6 x + sin 6 x = 2
273 K and 1 atm 4
(B) Expansion of 1 mol of (Q) w = 0 in the interval [0, 2p] is
6. Let the curve C be the mirror image of the parabola y2 = 4x
an ideal gas into a vacuum
with respect to the line x + y + 4 = 0. If A and B are the points
under isolated conditions
of intersection of C with the line y = –5, then the distance
(C) Mixing of equal volumes (R) DSsys < 0 between A and B is
of two ideal gases at 7. The minimum number of times a fair coin needs to be tossed,
constant temperature so that the probability of getting at least two heads is at
and pressure in an least 0.96, is
isolated container 8. Let n be the number of ways in which 5 boys and 5 girls can
(D) Reversible heating of (S) DU = 0 stand in a queue in such a way that all the girls stand
H2(g) at 1 atm from 300 K consecutively in the queue. Let m be the number of ways in
to 600 K, followed by which 5 boys and 5 girls can stand in a queue in such a way
that exactly four girls stand consecutively in the queue. Then
reversible cooling to 300 K
at 1 atm m
the value of is
(T) DG = 0 n
JEE Advanced 2015 Solved Paper 2015-7

SECTION - II differential equation Py ¢¢ + Qy ¢ + 1 = 0 , where P, Q are


This section contains 10 multiple choice questions. Each question
has 4 choices (a), (b), (c) and (d) out of which ONE or MORE æ dy d 2 yö
functions of x, y and y¢ ç here y ¢= , y ¢¢ = ÷ , then
THAN ONE are correct. è dx dx 2 ø
9. Which of the following values of a satisfy the equation
which of the following statements is (are) true?
(a) P = y + x (b) P = y – x
(1 + a)2 (1 + 2a) 2 (1 + 3a ) 2
(c) P + Q = 1 – x + y + y¢ + (y') (d) P – Q = x + y – y¢ –(y')2
2
(2 + a)2 (2 + 2a)2 (2 + 3a )2 = –648a ? 15. Let g : R ® R be a differentiable function with g(0) = 0, g'(0)
2 2 2
(3 + a ) (3 + 2a ) (3 + 3a)
ì x
ï g ( x ), x ¹ 0
(a) –4 (b) 9 = 0 and g'(1) ¹ 0. Let f ( x ) = í | x|
ïî0, x =0
(c) –9 (d) 4
3
10. In R , consider the planes P1 : y = 0 and P2 : x + z = 1. Let P3 and h(x) = e|x| for all x Î R . Let (foh)(x) denote f(h(x)) and
be the plane, different from P1 and P2, which passes through (hof)(x) denote h(f(x)). Then which of the following is (are) true?
the intersection of P1 and P2. If the distance of the point
(a) f is differentiable at x = 0
(0, 1, 0) from P3 is 1 and the distance of a point (a, b, g) from
(b) h is differentiable at x = 0
P3 is 2, then which of the following relations is (are) true?
(c) foh is differentiable at x = 0
(a) 2a + b +2g + 2 = 0 (b) 2a – b +2g + 4 = 0
(d) hof is differentiable at x = 0
(c) 2a + b –2g – 10 = 0 (d) 2a – b +2g – 8 = 0
3
11. In R , let L be a straight line passing through the origin. æp æp öö p
Suppose that all the points on L are at a constant distance 16. Let f ( x) = sin ç sin ç sin x ÷ ÷ for all x Î R and g(x) =
è6 è2 øø 2
from the two planes P1 : x + 2y – z + 1 = 0 and P2 : 2x – y + z
– 1 = 0. Let M be the locus of the feet of the perpendiculars sin x for all x Î R. Let (fog)(x) denote f(g(x)) and (gof)(x)
drawn from the points on L to the plane P1. Which of the denote g(f(x)). Then which of the following is (are) true?
following points lie (s) on M? é 1 1ù
(a) Range of f is ê - , ú
æ 5 2ö æ 1 1 1ö ë 2 2û
(a) ç 0, – , – ÷ (b) ç – , – , ÷
è 6 3ø è 6 3 6ø
é 1 1ù
(b) Range of fog is ê - , ú
æ 5 1ö æ 1 2ö ë 2 2û
(c) ç – ,0, ÷ (d) ç – ,0, ÷
è 6 6ø è 3 3ø
f ( x) p
12. Let P and Q be distinct points on the parabola y2 = 2x such (c) lim =
x ®0 g ( x ) 6
that a circle with PQ as diameter passes through the vertex
O of the parabola. If P lies in the first quadrant and the area (d) There is an x Î R such that ( gof )( x ) = 1
of the triangle DOPQ is 3 2 , then which of the following is r uuur r uuur r uuur
17. Let DPQR be a triangle. Let a = QR, b = RP and c = PQ . If
(are) the coordinates of P?
r r rr
(a) (4, 2 2) (b) (9,3 2) a = 12 , b = 4 3 , b . c = 24 , then which of the following
is (are) true?
æ1 1 ö
(c) ç , ÷ (d) (1, 2) r2 r2
è4 2ø c r c r
(a) - a = 12 (b) + a = 30
2 2
13. Let y(x) be a solution of the differential equation
r r r r r r
(1 + e x ) y ¢ + ye x = 1 . If y(0) = 2, then which of the following (c) a ´ b + c ´ a = 48 3 (d) a. b = –72
statement is (are) true? 18. Let X and Y be two arbitrary, 3 × 3, non-zero, skew-symmetric
(a) y(–4) = 0 (b) y(–2) = 0 matrices and Z be an arbitrary 3 × 3, non zero, symmetric
(c) y(x) has a critical point in the interval (–1, 0) matrix. Then which of the following matrices is (are) skew
(d) y(x) has no critical point in the interval (–1, 0) symmetric?
14. Consider the family of all circles whose centers lie on the (a) Y3Z4 – Z4Y3 (b) X44 + Y44
straight line y = x. If this family of circle is represented by the (c) X4Z3 – Z3X4 (d) X23 + Y23
EBD_7801
2015-8 JEE Advanced 2015 Solved Paper

SECTION - III
This section contains TWO questions. Each question contains two columns, Column I and Column II. Column I has four entries (A),
(B), (C) and (D). Column II has five entries (P), (Q), (R), (S) and (T). Match the entries in Column I with the entries in Column II. One
or more entries in Column I may match with one or more entries in Column II.
19. Match the following
Column I Column II
(A) In R2, if the magnitude of the projection vector (P) 1
of the vector a$i + bˆj on 3iˆ + ˆj is 3 and
if a = 2 + 3b , then possible value of a is/are
(B) Let a and b be real numbers such that the (Q) 2
ì -3ax - 2, x < 1
2
function f ( x) = í 2
îbx + a , x ³ 1
if differentiable for all x Î R
Then possible value of a is (are)
(C) Let w ¹ 1 be a complex cube root of unity. (R) 3
If (3 - 3w + 2w )2 4n+3 2 4n+3
+ (2 + 3w - 3w )
+(-3 + 2w + 3w2 ) 4 n +3 = 0, then possible
value (s) of n is (are)
(D) Let the harmonic mean of two positive real (S) 4
numbers a and b be 4. If q is a positive
real number such that a, 5, q, b is an arithmetic
progression, then the value(s) of | q – a | is (are)
(T) 5
20. Match the following.
Column I Column II
(A) In a triangle DXYZ, let a, b, and c be the (P) 1
lengths of the sides opposite to the angles
X, Y and Z, respectively. If 2(a2 – b2) = c2
sin( X - Y )
and l = , then possible values of
sin Z
n for which cos(npl) = 0 is (are)
(B) In a triangle DXYZ, let a, b and c be the (Q) 2
lengths of the sides opposite to the angles
X, Y, and Z respectively. If 1 + cos 2X – 2cos 2Y
a
= 2 sin X sin Y, then possible value (s) of is (are)
b
(C) In R2, let 3i + ˆj, iˆ + 3 ˆj and biˆ + (1 -b) ˆj (R) 3
be the position vectors of X , Y, and Z with
respect to the origin O, respectively. If the
distance of Z from the bisector of the
uuur uuur 3
acute angle of OX with OY is ,
2
then possible value(s) of b is (are)
(D) Suppose that F(a) denotes the area of the (S) 5
region bounded by x = 0, x = 2, y2 = 4x and
y =| ax - 1| + | ax - 2 | +ax , where a Î {0, 1} .

Then the value(s) of F (a ) + 8 2 , when


3
a = 0 and a = 1, is (are)
(T) 6
JEE Advanced 2015 Solved Paper 2015-9

PAPER - 2
1. The question paper consists of three parts (Physics, Chemistry and Mathematics). Each part consists of three
sections.
2. Section 1 contains 8 questions. The answer to each of the questions is a single-digit integer ranging from 0 to 9
(both inclusive).
3. Section 2 contains 8 multiple choice questions. Each question has four choice (a), (b), (c) and (d) out of which
ONE OR MORE THAN ONE are correct.
4. Section 3 contains 2 paragraphs each describing theory, experiment and data etc. four questions relate to two
paragraphs with two questions on each paragraph. Each question pertaining to a particular passage should have
one or more correct answer among the four given choices (a), (b), (c) and (d).

PHYSICS 4. An electron is an excited state of Li2+ ion has angular


momentum 3h/2p. The de Broglie wavelength of the electron
SECTION - I in this state is pp a0 (where a0 is the Bohr radius). The value
This section contains 8 questions. Each question, when worked of p is
out will result in one integer from 0 to 9 (both inclusive). 5. A large spherical mass M is fixed at one position and two
identical point masses m are kept on a line passing through
1. For a radioactive material, its activity A and rate of change of the centre of M (see figure). The point masses are connected
dN dA by a rigid massless rod of length l and this assembly is free
its activity R are defined as A = - and R = – , where
dt dt to move along the line connecting them. All three masses
N(t) is the number of nuclei at time t. Two radioactive sources interact only through their mutual gravitational interaction.
P (mean life t) and Q (mean life 2t) have the same activity at When the point mass nearer to M is at a distance r = 3l from
t = 0. Their rates of change of activities at t = 2t are RP and
æ M ö
RP n M, the tension in the rod is zero for m = k ç ÷ . The value
RQ, respectively. If = , then the value of n is è 288 ø
RQ e of k is
2. The monochromatic beam of light is incident at 60° on one
face of an equilateral prism of refractive index n and emerges
from the opposite face making an angle q(n) with the normal
dq
(see the figure). For n = 3 the value of q is 60° and = m. 6. The energy of a system as a function of time t is given as E(t)
dn
= A2 exp(–at,) where a = 0.2 s–1. The measurement of A has
The value of m is
an error of 1.25%. If the error in the measurement of time is
1.50%, the percentage error in the value of E(t) at t = 5 s is
7. The densities of two solid spheres A and B of the same radii
ærö
R vary with radial distance r as rA(r) = k ç ÷ and rB(r) =
60° q èRø
5
ærö
k ç ÷ , respectively, where k is a constant. The moments
è Rø
3. In the following circuit, the current through the resistor of inertia of the individual spheres about axes passing through
R (= 2 W) is I amperes. The value of I is
IB n
(=2W) their centres are IA and IB, respectively. If , I = 10 , the
A
value of n is
8. Four harmonic waves of equal frequencies and equal
p 2p
intensities I0 have phase angles 0, , and p. When they
3 3
are superposed, the intensity of the resulting wave is nI0.
The value of n is
EBD_7801
2015-10 JEE Advanced 2015 Solved Paper
SECTION - II 12. A spherical body of radius R consists of a fluid of constant
density and is in equilibrium under its own gravity. If P(r) is
This section contains 8 multiple choice questions. Each question the pressure at r(r < R), then the correct option(s) is (are)
has 4 choices (a), (b), (c) and (d) out of which ONE or MORE
THAN ONE are correct. P (r = 3R /4) 63
(a) P(r = 0) = 0 (b) =
P (r = 2R /3) 80
9. In terms of potential difference V, electric current I, permittivity
e0, permeability m0 and speed of light c, the dimensionally P (r = 3R /5) 16 P (r = R /2) 20
correct equation(s) is(are) (c) = (d) =
P (r = 2R /5) 21 P (r = R /3) 27
(a) m0I2 = e0V2 (b) m0I = m0V
13. A parallel plate capacitor having plates of area S and plate
(c) I = e0cV (d) m0cI = e0V
separation d, has capacitance C1 in air. When two dielectrics
10. Consider a uniform spherical charge distribution of radius of different relative primitivities (e1 = 2 and e2 = 4) are
R1 centred at the origin O. In this distribution, a spherical introduced between the two plates as shown in the figure,
cavity of radius R 2, centred at P with distance OP = a
= R1 – R2 (see figure) is made. If the electric field inside the C2
r uruur the capacitance becomes C2. The ratio C is
cavity at position r is E (r), then the correct statement(s) 1
is(are)

ur
(a) E is uniform, its magnitude is independent of R2 but
r
its
ur direction depends on r
(b) E is uniform, its magnitude depends on R2 and its
r
direction depends on r
ur (a) 6/5 (b) 5/3
(c) E is uniform, its magnitude is independent of a but its
r (c) 7/5 (d) 7/3
direction depends on a 14. An ideal monoatomic gas is confined in a horizontal cylinder
ur
(d) E is uniform and both its magnitude and direction by a spring loaded piston (as shown in the figure). Initially
r the gas is at temperature T1, pressure P1 and volume V1 and
depend on a
the spring is in its relaxed state. The gas is then heated very
11. In plotting stress versus strain curves for two materials P slowly to temperature T2, pressure P2 and volume V2. During
and Q,a student by mistake puts strain on the y-axis and this process the piston moves out by a distance x. Ignoring
stress on the x-axis as shown in the figure. Then the correct the friction between the piston and the cylinder, the correct
statement(s) is (are) statement(s) is (are)

(a) If V2 = 2V1 and T2 = 3T1, then the energy stored in the

(a) P has more tensile strength than Q


1
spring is PV
(b) P is more ductile than Q 4 1 1
(c) P is more brittle than Q (b) If V2 = 2V1 and T2 = 3T1, then the change in internal
(d) The Young's modulus of P is more than that of Q energy is 3P1V1
JEE Advanced 2015 Solved Paper 2015-11

(c) If V2 = 3V1 and T2 = 4T1, then the work done by the gas internal reflections at the interface of the media n1 and n2 as shown
7 in the figure. All rays with the angle of incidence i less than a
is PV particular value im are confined in the medium of refractive index
3 1 1
n1. The numerical aperture (NA) of the structure is defined as
(d) If V2 = 3V1 and T2 = 4T1, then the heat supplied to the
sin im.
17
gas is PV
6 1 1
236
15. A fission reaction is given by 92 U ® 140
54 Xe +
94
38 Sr + x + y,
236
where x and y are two particles. Considering 92 U to be at
rest, the kinetic energies of the products are denoted by KXe,
KSr, Kx(2 MeV) and Ky(2 MeV), respectively. Let the binding
236 140 94
energies per nucleon of 92 U, 54 Xe and 38 Sr be 7.5 MeV,
8.5 MeV and 8.5 MeV, respectively. Considering different 17. For two structure namely S1 with n1 = 45 / 4 and n2 = 3/2,
conservation laws, the correct option(s) is(are) and S2 with n1 = 8/5 and n2 = 7/5 and taking the refractive
(a) x = n, y = n, KSr = 129 MeV, KXe = 86 MeV index of water to be 4/3 and that of air to be 1, the correct
(b) x = p, y = e–, KSr = 129 MeV, KXe = 86 MeV option(s) is(are)
(c) x = p, y = n, KSr = 129 MeV, KXe = 86 MeV (a) NA of S1 immersed in water is the same as that of S2
(d) x = n, y = n, KSr = 86 MeV, KXe = 129 MeV
16
16. Two spheres P and Q of equal radii have densities r1 and r2, immersed in a liquid of refractive index
respectively. The spheres are connected by a massless string 3 15
and placed in liquids L1 and L2 of densities s1 and s2 and 6
viscosities h1 and h2, respectively. They float in equilibrium (b) NA of S1 immersed in liquid of refractive index is
with the sphere P in L1 and sphere Q in L2 and the string
15
being taut (see figure). If sphere P alone in L2 has terminal the same as that of S2 immersed in water
ur ur (c) NA of S1 placed in air is the same as that of S2 immersed
velocity VP and Q alone in L1 has terminal velocity V Q ,
4
then in liquid of refractive index
15
(d) NA of S1 placed in air is the same as that of S2 placed in
L1 water
18. If two structure of same cross-sectional area, but different
numerical apertures NA1 and NA2(NA2 < NA1) are joined
L2 longitudinally, the numerical aperture of the combined
structure is

NA1 NA2
(a) (b) NA1 + NA2
NA1 + NA2
ur ur
VP h VP h2
ur = 1 (c) NA 1 (d) NA 2
(a) (b) ur = h
V Q h2 VQ 1 PARAGRAPH 2
ur ur ur ur In a thin rectangular metallic strip a constant current I flows along
(c) V P .V Q > 0 (d) V P .V Q < 0
the positive x-direction, as shown in the figure. The length, width
SECTION - III and thickness of the strip are l, w and d, respectively.
su
This section contains 2 paragraphs, each describing theory, A uniform magnetic field B is applied on the strip along the
experiments, data etc. four questions related to the two paragraphs positive y-direction. Due to this, the charge carriers experience a
with two questions on each paragraph. Each question has one or net deflection along the z-direction. This results in accumulation
more than one correct answer(s) among the four given options of charge carriers on the surface PQRS and appearance of equal
(a), (b), (c) and (d). and opposite charges on the face opposite to PQRS. A potential
PARAGRAPH 1 difference along the z-direction is thus developed. Charge
Light guidance in an optical fibre can be understood by considering accumulation continues until the magnetic force is balanced by
a structure comprising of thin solid glass cylinder of refractive the electric force. The current is assumed to be uniformly
index n1 surrounded by a medium of lower refractive index n2. The distributed on the cross-section of the strip and carried by
light guidance in the structure takes place due to successive total electrons.
EBD_7801
2015-12 JEE Advanced 2015 Solved Paper
4. In dilute aqueous H2 SO 4 , the complex diaquodio-
xalatoferrate(II) is oxidized by MnO4 –. For this reaction, the
ratio of the rate of change of [H+] to the rate of change of
[MnO4–] is
5. The number of hydroxyl group(s) in Q is

H+ aqueous dilute KMnO (excess)


¾¾¾ ® P ¾¾¾¾¾¾¾¾¾¾¾
4
®Q
19. Consider two different metallic strips (1 and 2) of the same heat 0° C

material. Their lengths are the same, widths are w1 and w2


and thicknesses are d1 and d2 respectively. Two points K
and M are symmetrically located on the opposite faces parallel 6. Among the following, the number of reaction(s) that
to the x-y plane (see figure). V1 and V2 are the potential produce(s) benzaldehyde is
differences between K and M in strips 1 and 2, respectively.
Then, for a given current I flowing through them in a given
magnetic field strength B, the correct statement(s) is(are) CO, HCl
I. ¾¾¾¾¾¾¾¾
Anhydrous AlCl / CuCl
®
3
(a) If w1 = w2 and d1 = 2d2, then V2 = 2V1
(b) If w1 = w2 and d1 = 2d2, then V2 = V1
(c) If w1 = 2w2 and d1 = d2, then V2 = 2V1
H O
(d) If w1 = 2w2 and d1 = d2, then V2 = V1 II. ¾¾¾2
100°C
®
20. Consider two different metallic strips (1 and 2) of same
dimensions (length l, width w and thickness d) with carrier
densities n1 and n2, respectively. Strip 1 is placed in magnetic
H2
field B1 and strip 2 is placed in magnetic field B2, both along III. ¾¾¾¾¾
Pd - BaSO
®
4
positive y-directions. Then V1 and V2 are the potential
differences developed between K and M in strips 1 and 2,
respectively. Assuming that the current I is the same for
both the strips, the correct option(s) is(are) DIBAL –H
¾¾¾¾¾¾ ®
IV. Toluene,–78°C
(a) If B1 = B2 and n1 = 2n2, then V2 = 2V1 H 2O

(b) If B1 = B2 and n1 = 2n2, then V2 = V1


7. In the complex acetylbromidodicarbonylbis
(c) If B1 = 2B2 and n1 = n2, then V2 = 0.5V1
(triethylphosphine) iron (II), the number of Fe–C bond(s) is
(d) If B1 = 2B2 and n1 = n2, then V2 = V1 8. Among the complex ions, [Co(NH2–CH2–CH2–NH2)2Cl2]+,
[CrCl2(C2O4)2]3–, [Fe(H2O)4(OH)2]+, [Fe(NH3)2(CN)4]–,
CHEMISTRY [Co(NH2–CH2–CH2–NH2)2(NH3)Cl]2+ and
[Co(NH3)4(H2O)Cl]2+, the number of complex ion(s) that
SECTION - I show(s) cis-trans isomerism is
This section contains 8 questions. Each question, when worked SECTION - II
out will result in one integer from 0 to 9 (both inclusive). This section contains 8 multiple choice questions. Each question
1. Three moles of B2H6 are completely reacted with methanol. has 4 choices (a), (b), (c) and (d) out of which ONE or MORE
The number of moles of boron containing product formed is THAN ONE are correct.
2. The molar conductivity of a solution of a weak acid HX (0.01
9. The major product U in the following reactions is
M) is 10 times smaller than the molar conductivity of a solution
0 0
of a weak acid HY (0.10 M). If l x – » l y – the difference in CH =CH–CH , H+ radical, initiator, O
2
¾¾¾¾¾¾¾¾ 3 2®
® T ¾¾¾¾¾¾¾¾
high pressure, heat
their pKa values, pKa(HX) – pKa (HY), is (consider degree of
ionization of both acids to be <<1)
238
3. A closed vessel with rigid walls contains 1 mol of 92 U and
238
1 mol of air at 298 K. Considering complete decay of 92 U to
206 3
82 Pb, the ratio of the final pressure to the initial pressure of
the system at 298 K is (a)
JEE Advanced 2015 Solved Paper 2015-13

12. The pair(s) of ions where BOTH the ions are precipitated
upon passing H2S gas in presence of dilute HCl, is(are)
(b) (a) Ba2+, Zn2+ (b) Bi3+, Fe3+
(c) Cu2+, Pb2+ (d) Hg2+, Bi3+
13. Under hydrolytic conditions, the compounds used for
preparation of linear polymer and for chain termination,
respectively, are
(a) CH3SiCl3 and Si(CH3)4
(b) (CH3)2SiCl2 and (CH3)3SiCl
(c) (c) (CH3)2SiCl2 and CH3SiCl3
(d) SiCl4 and (CH3)3SiCl
14. When O2 is adsorbed on a metallic surface, electron transfer
occurs from the metal to O2. The true statement(s) regarding
this adsorption is(are)
(d)
(a) O2 is physisorbed
(b) Heat is released
10. In the following reactions, the major product W is
*
OH (c) Occupancy of p2r of O2 is increased
, NaOH (d) Bond length of O2 is increased
NaNO2 , HCl
0°C
15. One mole of a monoatomic real gas satisfies the equation
p(V – b) = RT where b is a constant. The relationship of
interatomic potential V(r) and interatomic distance r for the
gas is given by
(a)
N=N OH

(a)
OH
(b)
N=N

OH
(b)
N=N
(c)

(d) N=N
(c)

11. The correct statement(s) regarding, (i) HClO, (ii) HClO2,


(iii) HClO3 and (iv) HClO4, is(are)
(a) The number of Cl=O bonds in (ii) and (iii) together is
two
(b) The number of lone pairs of electrons on Cl in (ii) and
(iii) together is three (d)
(c) The hybridization of Cl in (iv) is sp3
(d) Amongst (i) to (iv), the strongest acid is (i)
EBD_7801
2015-14 JEE Advanced 2015 Solved Paper
16. In the following reactions, the product S is PARAGRAPH 2
In the following reactions
Pd - BaSO 4
C8 H6 C8 H8 ¾¾¾¾¾¾¾ (i) B2 H6
(ii)H 2 O 2 , NaOH, H 2 O
®X
H2

H2O
HgSO 4 ,H 2SO 4

(a) C8 H 8 O ¾ (i)
¾(ii)¾ ¾ ¾® Y
EtMgBr, H2 O
H + , heat

19. Compound X is

(b)
(a) (b)

(c) (d)
(c)
20. The major compound Y is

(a)

(d)

(b)
SECTION - III
This section contains 2 paragraphs, each describing theory,
experiments, data etc. four questions related to the two paragraphs (c)
with two questions on each paragraph. Each question has one or
more than one correct answer(s) among the four given options
(a), (b), (c) and (d).
PARAGRAPH 1
When 100 mL of 1.0 M HCl was mixed with 100 mL of 1.0 M NaOH (d)
in an insulated beaker at constant pressure, a temperature increase
of 5.7 °C was measured for the beaker and its contents (Expt.1).
Because the enthalpy of neutralization of a strong acid with a
strong base is a constant (–57.0 kJ mol–1), this experiment could MATHEMATICS
be used to measure the calorimeter constant. In a second
experiment (Expt. 2), 100 mL of 2.0 M acetic acid (Ka = 2.0 × 10–5) SECTION - I
was mixed with 100 mL of 1.0 M NaOH (under identical conditions This section contains 8 questions. Each question, when worked
to Expt. 1) where a temperature rise of 5.6 °C was measured. out will result in one integer from 0 to 9 (both inclusive).
1. Let m and n be two positive integers greater than 1. If
(Consider heat capacity of all solutions as 4.2 J g–1 K–1 and density
of all solutions as 1.0 g mL–1) æ cos(a n ) ö
e –e
lim ç ÷ = – æç e ö÷ then the value of m is
17. Enthalpy of dissociation (in kJ mol–1) of acetic acid obtained a ® 0 çè a m ÷ø è 2ø n
from the Expt.2 is
(a) 1.0 (b) 10.0 1
9 x +3 tan –1 x æ 12 + 9 x 2 ö
(c) 24.5 (d) 51.4 2. If a = ò (e ) ç
è 1+ x ø2 ÷
dx where tan–1x takes
0
18. The pH of the solution after Expt. 2 is
(a) 2.8 (b) 4.7 æ 3p ö
only principal values, then the value of çè log e |1 + a | – ÷ø
(c) 5.0 (d) 7.0 4
is
JEE Advanced 2015 Solved Paper 2015-15

3. Let f : ¡ ® ¡ be a continuous odd function, which 10. Let E1 and E2 be two ellipses whose centers are at the origin.
1 The major axes of E1 and E2 lie along the x-axis and the
vanishes exactly at one point and f (1) = . Suppose that y-axis, respectively. Let S be the circle x2 + (y – 1)2 = 2. The
2
x
straight line x + y = 3 touches the curves S, E1 and E2 at P, Q
x
F(x) = ò f (t )dt for all x Î [–1, 2] and G(x) = ò t | f ( f (t)) | dt for 2 2
–1 –1 and R respectively. Suppose that PQ = PR = . If e1 and
3
F ( x) 1 æ 1ö
all xÎ[–1, 2]. If lim = , then the value of f çè ÷ø is e2 are the eccentricities of E1 and E2, respectively, then the
x ®1 G ( x ) 14 2 correct expression(s) is (are)
r r
4. Suppose that p, q and rr are three non-coplanar vectors in
r r r r 43 7
¡3 . Let the components of a vector s along p, q and r (a) e12 + e22 = (b) e1e2 =
be 4, 3 and 5, respectively. If the components of this vector 40 2 10
r r r r r r r r r r
s along (– p + q + r ),( p – q + r ) and (– p – q + r ) are x, y
and z, respectively, then the value of 2x + y + z is 5 3
(c) e12 – e22 = (d) e1e2 =
æ kpö æ kpö 8 4
5. For any integer k, let ak = cos çè ÷ø + i sin çè ÷ø , where
7 7 11. Consider the hyperbola H : x2 – y2 = 1 and a circle S with
12 center N(x2, 0). Suppose that H and S touch each other at a
å | ak +1 – ak | point P(x1, y1) with x1 > 1 and y1 > 0. The common tangent to
i= –1 . The value of the expression 3 k =1 is H and S at P intersects the x-axis at point M. If (l, m) is the
centroid of the triangle PMN, then the correct expression(s)
å | a4k –1 – a4k –2 | is(are)
k =1
6. Suppose that all the terms of an arithmetic progression (A.P.) dl 1
are natural numbers. If the ratio of the sum of the first seven (a) dx1 = 1 – 3x12 for x1 > 1
terms to the sum of the first eleven terms is 6 : 11 and the
seventh term lies in between 130 and 140, then the common
difference of this A.P. is dm x1
=
( )
(b) for x1 > 1
7. The coefficient of x9 in the expansion of (1 + x) (1 + x2) (1 + x3) dx1 3 x12 – 1
... (1 + x100) is

x2 y 2 dl 1
8. Suppose that the foci of the ellipse + = 1 are (f1, 0) (c)
9 5 dx1 = 1 + 3x 2 for x1 > 1
1
and (f2, 0) where f1 > 0 and f2 < 0. Let P1 and P2 be two
parabolas with a common vertex at (0, 0) and with foci at (f1, dm 1
(d) =
0) and (2f2, 0), respectively. Let T1 be a tangent to P1 which dy1 3 for y1 > 0
passes through (2f2, 0) and T2 be a tangent to P2 which
passes through (f1, 0). If m1 is the slope of T1 and m2 is the 12. The option(s) with the values of a and L that satisfy the
following equation is(are)
æ 1 2
ö
slope of T2, then the value of ç 2 + m2 ÷ is
è m1 ø 4p

SECTION - II
òe
t
(sin 6
at + cos 4 at dt)
0
p = L?
(sin )
This section contains 8 multiple choice questions. Each question
òe
t 6 4
has 4 choices (a), (b), (c) and (d) out of which ONE or MORE at + cos at dt
0
THAN ONE are correct.

æ 6ö æ 4ö
9. If a = 3sin–1 çè ÷ø and b = 3cos–1 çè ÷ø , where the inverse e4p - 1 e4p + 1
11 9 (a) a = 2, L = (b) a = 2, L =
ep - 1 ep + 1
trigonometric functions take only the principal values, then
the correct option(s) is (are)
(a) cosb > 0 (b) sinb < 0 e4p - 1 e4p + 1
(c) a = 4, L = (d) a = 4, L =
(c) cos(a + b) > 0 (d) cosa < 0 ep - 1 ep + 1
EBD_7801
2015-16 JEE Advanced 2015 Solved Paper
13. Let f, g: [–1, 2] ® ¡ be continuous functions which are SECTION - III
twice differentiable on the interval (–1, 2). Let the values of
This section contains 2 paragraphs, each describing theory,
f and g at the points –1, 0 and 2 be as given in the following
experiments, data etc. four questions related to the two paragraphs
table:
with two questions on each paragraph. Each question has one or
more than one correct answer(s) among the four given options
x = –1 x = 0 x = 2
(a), (b), (c) and (d).
f ( x) 3 6 0
PARAGRAPH 1
g ( x) 0 1 –1
Let F : ¡ ® ¡ be a thrice differentiable function. Suppose
In each of the intervals (–1, 0) and (0, 2) the function (f – 3g)"
æ1 ö
never vanishes. Then the correct statement(s) is(are) that F(1) = 0, F(3) = –4 and F(x) < 0 for all x Î çè ,3÷ø . Let f(x)
2
(a) f ¢ ( x) – 3g ¢( x) = 0 has exactly three solutions in (–1, 0)
= xF(x) for all x Ρ .
È (0, 2)
17. The correct statement(s) is(are)
(b) f ¢ ( x) – 3g ¢( x) = 0 has exactly one solution in (–1, 0)
(a) f ¢(1) < 0
(c) f ¢ ( x) – 3g ¢( x) = 0 has exactly one solution in (0, 2) (b) f(2) < 0
(d) f ¢ ( x) – 3g ¢( x) = 0 has exactly two solutions in (–1, 0) (c) f ¢ ( x) ¹ 0 for any x Î (1, 3)
and exactly two solutions in (0, 2) (d) f ¢ ( x) = 0 for some x Î (1, 3)
æ p pö
14. Let f(x) = 7tan8x + 7tan6x – 3tan4x – 3tan2x for all x Î çè – , ÷ø . 3 2 3 3
2 2 18. If ò1 x F¢ (x)dx = –12 and ò1 x F¢¢(x)dx = 40 , then the
Then the correct expression(s) is(are) correct expression(s) is (are)
p/4 p/4 3
1
(a) ò xf ( x )dx =
12 (b) ò f ( x )dx = 0 (a) 9 f ¢ (3) + f ¢(1) – 32 = 0 (b) ò1 f ( x) dx = 12
0 0
3
p/4
1
p/4 (c) 9 f ¢ (3) – f ¢ (1) + 32 = 0 (d) ò1 f ( x ) dx = –12
(c) ò xf ( x)dx =
6 (d) ò f ( x)dx = 1
PARAGRAPH 2
0 0
Let n1 and n2 be the number of red and black balls, respectively, in
box I. Let n 3 and n 4 be the number of red and black balls,
192 x3 æ 1ö
15. Let f ¢( x ) = 4 for all x Ρ with f çè ÷ø = 0. respectively, in box II.
2 + sin px 2
19. One of the two boxes, box I and box II, was selected at
random and a ball was drawn randomly out of this box. The
1
ball was found to be red. If the probability that this red ball
If m < ò f ( x) dx £ M , then the possible values of m and M
1/ 2 1
was drawn from box II is , then the correct option(s) with
are 3
the possible values of n1, n2, n3 and n4 is(are)
1 1 (a) n1 = 3, n2 = 3, n3 = 5, n4 = 15
(a) m = 13, M = 24 (b) m = ,M=
4 2 (b) n1 = 3, n2 = 6, n3 = 10, n4 = 50
(c) m = –11, M = 0 (d) m = 1, M = 12 (c) n1 = 8, n2 = 6, n3 = 5, n4 = 20
16. Let S be the set of all non-zero real numbers a such that the (d) n1 = 6, n2 = 12, n3 = 5, n4 = 20
quadratic equation ax2 – x + a = 0 has two distinct real roots 20. A ball is drawn at random from box I and transferred to box
x1 and x2 satisfying the inequality |x1 – x2| < 1. Which of the II. If the probability of drawing a red ball from box I, after this
following intervals is(are) a subset(s) of S? 1
transfer, is , then the correct option(s) with the possible
3
æ 1 1 ö æ 1 ö
(a) çè – 2 , – ÷ (b) çè – , 0÷ values of n1 and n2 is(are)
5ø 5 ø
(a) n1 = 4 and n2 = 6
(b) n1 = 2 and n2 = 3
æ 1 ö æ 1 1ö
(c) çè 0, ÷ (d) çè , ÷ (c) n1 = 10 and n2 = 20
5ø 5 2ø
(d) n1 = 3 and n2 = 6
Telegram @unacademyplusdiscounts

Join Us on Telegram for More Such Books

https://telegram.me/unacademyplusdiscounts

Join us from the above link or search ''unacademyplusdiscounts'' in Telegram


JEE Advanced 2015 Solved Paper 2015-17

SOLUTIONS
Paper - 1
PHYSICS fm = –10cm fl = +10cm

PA AA TA4 AA l 4B
1. (2) = = ´
PB AB TB4 AB l 4A
1 1 1 15cm
l éA P ù 4 é R 2 P ù 4 é 400 ´ 400 ù 4
\ A ê A ´ B ú =ê A ´ B ú =ê ú
l B ë AB PA û 2
êë RB PA úû ë 10 4 û
l
\ A =2 30cm
lB 50cm
For convex lens u = |2fl|
2. (3) T No T No T No Therefore image will have a magnification of 1.
No ¾¾
® ¾¾
® ¾¾
®
2 4 8 When the set – up is kept in a medium
100% 50% 25% 12.5% The focal length of the lens will change
Three half life are required. Therefore n = 3
1 æ nl ö é1.5 ù
3. (3) For maxima ç –1÷ – 1ú
Path defference = ml fl n
= è s ø Þ f l¢ = ëê 1 û
\ S2A – S1A = ml 1 æ nl ö 10 é 1.5 ù
S1 ç – 1 ÷ ê – 1ú
f l¢ ç n¢ ÷ ë7 / 6 û
è s ø

d x 2 + d2 Þ fl¢ = 17.5 cm.


1 1 1
x A Applying lens formula – =
v u f l¢
1 1 1
\ – = Þ v = 140 cm.
v –20 17.5
S2 v 140
M l¢ = Magnification by lens = = = –7
é (n - 1) d 2 + x 2 + d 2 + x 2 ù – d 2 - x 2 u –20
\ êë úû = ml
M2 Mmirror ´ M l¢
\ (n – 1) (d 2 + x 2 ) = ml Now M = Mmirror ´ M = 7
1 l
æ4 ö 2 2
\ ç 3 –1÷ d + x = ml 5. (6)
cylinder
è ø
\ d 2 + x 2 = 3ml
\ d2 + x2 = 9m2l2
\ x2 = 9m2l2 – d2 line charge
\ p2 = 9 Þ p=3 +
4. (7) Applying mirror formula
q q
1 1 1
+ = 3a
v u f 2
1 1 1 1 1 a/2 a/2
= – = +
v f u –10 15
1 –15 + 10 –5 –1 a/2 1
\ = = = tanq = =
v 150 150 30 3a / 2 3
\ v = –30cm \ q = 30°
EBD_7801
2015-18 JEE Advanced 2015 Solved Paper
The flux through the dotted cylinder by Gauss’s law is Given VB = VD. Therefore from (i) and (ii)
q in lL 3 3
f cylinder = = m (3)2 + mg × 30 = m (v2)2 + mg × 27
e 0 e0 4 4
\ V2 = 7
lL 9. (a, c, d)
\ For 360° angle the flux is e L µ hx cy Gz
0
Dimensionally
lL
\ For 60° angle the flux will be 6e [M 0 L1 T 0 ] = [ML2T –1 ]x [LT –1 ]y [M –1L3T –2 ]z
0
Therefore n = 6 M 0 L1 T 0 = Mx – z L2x + y + 3z T–x – y – 2z
hc 13.6 \ x–z=0 Þ x=z
6. (2) = 2 + 10.2 \ 2x + y + 3z = 1 and –x – y – 2z = 0
l n On solving we get
1242 13.6 1 3 1
\ = 2 + 10.2 x= ,y= – ,z=
90 n 2 2 2
\ n2 = 4 \ Lµ h
\ n=2
Lµ G
7. (2) Let h be the height to which the bullet rises
C, D are correct options
–2 M µ hxcyGz
æ hö
then, g1 = g ç1 + ÷ 2 –1 x –1 y –1 3 –2 Z
è Rø M ¢L °T ° µ [ML T ] [LT ] [M L T ]
\ x – z L2x + y + 3z T– x – y –2z
M ¢L °T ° µ M
–2
g æ hö \ x–z=1
Þ = g ç1 + ÷ 2x + y + 3z = 0
4 è Rø
–x – y – 2z = 0
Þ h=R
On solving we get
2GM 1 1 1
We know that ve = = v N (given) ...(i) x= ,y= ,z=–
R 2 2 2
Now applying conservation of energy for the throw \ Mµ C
Loss of kinetic energy = Gain in gravitaional potential energy
A is the correct option.
1 2 GMm æ GMm ö 10. (b, d)
\ –ç–
R ÷ø
mv = – Maximum linear momentum in case 1 is (p1)max = mnmax
2 2R è
b = m [aw1] ...(i)
GM Maximum linear momentum in case 2 is (p2)max = mnmax
\ v= ...(ii) R = m [Rw2]
R
Comparing (i) & (ii) N=2 \ 1 = mw2 ...(ii)

1 2 1 2 b aw1
Dividing (i) & (ii) =
8. (7) Total kinetic energy of a rolling disc = mv + Iw 1 w2
2 2
w1 b 1
1 2 1 æ 1 mR 2 ö æç v ö
2
÷ \ w =a= 2 \ B is a correct option.
= mv + ç ÷ n
ø çè R 2 ÷ 2
2 2 è2 ø
1
3 Also E1 = m w12 a 2
K.E = mv2 2
4
1
For surface AB E2 = m w22 R 2
k.Ei + loss in gravitational potential energy = K.Ef 2
2 2 2 2
3 3 E1 w1 a w1 1 w1 w2 w1
m (3)2 + mg(30) = m VB2 ...(i) \ E = 2´ 2 = 2´ = 2´ =
4 4 2 w2 R w2 n w2 w1 w2
For surface CD
E1 E 2
3 3 \ w = w D is the correct option
m (v2)2 + mg (27) = m VD2 ...(ii) 1 2
4 4
JEE Advanced 2015 Solved Paper 2015-19

11. (d) Applying conservation of angular mumentum about 1 1


the axis \ =
d - 50 20
8w M 9R2 8w M 2 8w \ d = 70 cm.
MR2 × w = MR2 × + × × + r ×
9 8 25 9 8 9 B is the correct option.
14. (a, b, c)
4R ur uur ur
Þ r=
5 ( )
F = I éë ò dl ´ B ùû
D is the correct option ur r ur
12. (c) Force on change q when it is given a small displacement If B is along z then F = I éë(2 L + 2R )$i ´ B x$ ùû
x is Fnet = F1 – F2 option [A] is correct
l l
ur r ur
1 l 1 l If B is along x then F = I éë(2L + 2R )i$ ´ Bi$ùû = 0
Fnet = 2pe d – x – 2pe d + x
0 0 ur ur
ur
F1 +q F2 If B is along y then F = I éë(2L + 2R)$i ´ ˆj ùû
l éd + x – d + x ù x
\ Fnet = 2
pe0 êë d 2 - x 2 úû x d–x
Option (b) and (c) are also correct
15. (a, b, d)
l 2x d
3 5
\ Fnet = 2pe 2 2 Total energy = RT + RT = 4RT
0 d –x 2 2
2d
line charge 1 line charge 2
When x << d then 4RT
\ Average energy per mole = = 2RT
l 2
Fnet = pe x and is directed towards the mean position
0 gRT
therefore the charge +q will execute SHM. We know that Vsound =
M
l l
n1 + n2 n n
= 1 + 2
g mi x – 1 g1 - 1 g 2 –1

2 1 1
–q Þ = +
F2 g mix - 1 5 7
F1 -1 -1
3 5
2 3 5
= +
g mix - 1 2 2 = 4

1 3
\ g mix - 1 = \ g mix =
line charge 1 line charge 2 2 2
In case of charge (–q) (Vs )mix g mix M He
F2 > F1 therefore the charge –q continues to move in the = ´
(Vs ) He M mix g He
direction of its displacement.
[C] is the correct option.
3
13. (b) For refraction in S1 ´4
2 é 1´ 2 + 1´ 4 ù
= êQ M mix = = 3ú
n1 n2 n2 – n1 5 ë 2 û
– + = 3´
u v R 3
1.5 1 1 - 1.5 6
– + = =
–50 V –10 5
Þ v = 50 cm.
For refraction in S2 3RT
We know that Vrms =
M
n1 n2 n2 – n1
– + =
u v R (Vrms ) He M H2 2 1
\ (V ) = M HE
=
4
=
2
1 1.5 1.5 –1 rms H 2
– + =
–(d – 50) µ 10 \ options [A], [B] and [C] are correct.
EBD_7801
2015-20 JEE Advanced 2015 Solved Paper
Now if the least count of the linear scale of the screw gauge
rFe ´ lFe 10–7 ´ 50 ´10 –3 25
16. (b) RFe = = = ´10–4 is twice the least count of venier callipers then.
AFe 4 ´ 10 –6 2
L.C of linear scale of screw gauge = 2 × 0.025 = 0.05cm.
Then pitch = 2 × 0.05 = 0.1cm.
r Al ´ l Al 2.7 ´ 10 –8 ´ 50 ´ 10 –3 2.7 ´ 50
RAl = = = × 10–5
AAl (49 – 4) ´ 10 –6 45 0.1
Then L.C of screw gauge = cm = 0.001cm = 0.01mm.
100
= 0.3 × 10–4
(c) is a correct option.
–4 –4 19. A ® R, T ; B ® P, S ; C ® P, Q, R, T; D ® P, Q, R, T
RFe ´ RAl 12.5 ´10 ´ 0.3 ´ 10
Rtotal = = » 29µW
RFe + RAl 12.8 ´10 –4 Based on facts
20. A ® P, Q, R, T; B ® Q, S; C ® P, Q, R, S; D ® P, R, T
(B) is the correct option.
For A
17. (a, c)
2
hC hc W dU –d éU æ æ x ö2 ù
We know that – W = eV0 Þ – = V0 ê 0 ç1 - ç ÷ ú
l el e Fx = – dx = dx ç èaø ú
êë 2 è û
1
For V0 versus we should get a straight line with negative
l -2U 0
= ( x - a) x ( x + a)
slope and positive intercept. a3
For V0 vesus l, we will get a hyperbola. As l decreases V0
increases.
(a) and (c) are the correct options U U
18. (b, c) U0
Vernier callipers 2

1cm
1 MSD = = 0.125cm
8
–a +a x x
5 VSD = 4MSD U U
1
\ 5VSD = 4 × cm = 0.5cm
8
\ 1 VSD = 0.1cm
L.C = 1MSD – 1VSD –a
+a x +a x
= 0.125cm – 0.1cm
= 0.025cm
Screw gauge
One complete revolution = 2M.S.D
If the pitch of screw gauge is twice the L.C of vernier callipers
then pitch = 2 × 0.025 = 0.05cm. – dU æ xö
For B Fx = = -U 0 ç ÷
L.C of screw Gauge dx èaø
pitch
e- x / a
2 2
= Total no. of divisions of circular scale – dU
For C Fx = = U0 x( x - a)( x + a)
dx a3
0.05
= cm = 0.0005cm = 0.005 mm. dU U
100 For D Fx = - = - 03 [( x - a)( x + a)]
dx 2a
(b) is a correct option
JEE Advanced 2015 Solved Paper 2015-21

CHEMISTRY 6. (9)

1. (4) Cl – Be – Cl N 3–
Hybridization sp Hybridisation sp
Structure linear Structure linear

+
O= N®O
Hybridisation sp Hybridisation sp 2
Structure Linear Structure Trigonal planar
S
II III IV
Cl Cl
Hybridisation sp 3 Hybridisation sp 3 d
Structure Angular Structure linear

I I F Xe F
Hybridisation sp 3 d Hybridisation sp 3 d
Structure Linear Structure Linear
– +
N= N=O
Hybridisation sp
Structure Linear

Only BeCl 2 , N 3– , N 2 O and NO 2 are linear with


sp-hybridisation.
2. (3) Ground state configuration:

7. (8)
1s 2s 2p
in second excited state, electron will jump from 1s to 2p, so
degeneracy of second excited state of H– is 3.
Number of lone pairs = 8
3. (4) X ¾¾
® Y; DG° = – 193 kJ mol–1
8. (4) Fe(26) ¾¾ 6 2
® [Ar]18 3d 4s
M+ ¾¾ ® M3+ + 2e– E° = – 0.25V
Hence DG° for oxidation will be Fe3+ ¾¾ ® [Ar]18 3d 4s
5 0
DG° = – nFE°
= –2 × 96500 × (–0.25) SCN– is weak field ligand hence pairing will not occur.
= 48250 J = 48.25 kJ \ Fe3+ ¾¾
®
48.25 kJ energy oxidises one mole M+
Unpaired electrons = 5
193
\ 193 kJ energy oxidises mole M+ = 4 mole M+
48.25 Magnetic moment = 5(5 + 2) B.M.
4. (1) Given DTf = 0.0558°C
= 35 B.M. = 5.92 B.M.
as we know, DTf = i × Kf × m
Þ 0.0558 = i × 1.86 × 0.01 CN– is strong field ligand hence pairing will take place.

i =3 \ Fe3+ ¾¾
®
Therefore the complex will be [Co(NH3)5Cl]Cl2 Unpaired electrons = 1
Hence number of chloride in co-ordination sphere is 1.
5. (2) The molecule cannot show geometrical isomerism, so Magnetic moment = 1(1 + 2)B.M. = 3 B.M. = 1.732
only its mirror image will be the other stereoisomer. Difference = 5.92 – 1.732 = 4.188
Hence answer is (4).
EBD_7801
2015-22 JEE Advanced 2015 Solved Paper

9. (a)

KOH, H2O

(Minor) (Major)

10. (d)

Å Å

(Major)

11. (a) H OH

D-(+)- glucose L-(–)- glucose

12. (c)

–+
ClN 2

(Retention Product)
(Major)

13. (a, b, c) 16. (b) Initially on increasing temperature rate of reaction will
Cr2+ is a reducing agent and Mn3+ is an oxidizing agent and increase, so % yield will also increase with time. But at
both have electronic configuration d4. equilibrium % yield at high temperature (T2) would be
less than at T1 as reaction is exothermic so the graph is
E° = –0.41V E° = 1.51V
Cr3+ / Cr 2 + Mn 3+ / Mn 2 +
Above E° values explains reducing nature of Cr 2+ and T1
oxidizing behaviour of Mn3+.
14. (b, c, d) T2
%yield

In electrolytic refining of blister Cu, acidified CuSO4 is used


as electrolyte, pure Cu deposits at cathode and impurities
settle as anode mud.
time
15. (c, d)
Fe3+ is reduced to Fe2+ by H2O2 and Na2O2 in acidic medium.
JEE Advanced 2015 Solved Paper 2015-23

17. (a) In ccp, O2– ions are 4. Hence, w = 0


Hence total negative charge = –8 and qp = CpdT (Q dT = 0)
Let Al3+ ions be x, and Mg2+ ions be y. Þq=0
Total positive charge = 3x + 2y DU = CvdT (Q dT = 0)
Þ 3x + 2y = 8 DU = 0
This relation is satisfied only by x = 2 and y = 1. (C) ® P, Q, S
Hence number of Al3+ = 2. Mixing of two ideal gases at constant temperature
and number of Mg2+ = 1. Hence, DT = 0
Þ n = fraction of octahedral holes occupied by Al 3+ \q=0
2 1 DU = 0
= = also w = 0 (DU = q + w)
4 2
(D) ® P, Q, S, T
and m = fraction of tetrahedral holes occupied by Mg2+
Reversible heating and cooling of gas follows same path
also initial and final position is same.
1 q=0ü
=
8 Hence, ý Path same
w = 0þ
18. (b, d)
DU = 0 ü
ý State function
H2/Ni D G = 0þ
(A)
*
MATHEMATICS
H Br H Br
H2/Ni 1. (2) End points of latus rectum of y2 = 4x are (1, +2)
(B) Equation of normal to y2 = 4x at (1, 2) is
(Optically inactive)
y – 2 = –1(x – 1) or x + y –3 = 0
As it is tangent to circle (x – 3)2 + (y + 2)2 = r2
(C) H2C H2/Ni H3C
CH3 * CH3 3 + (–2) – 3
\ = r Þ r2 = 2
CH3 CH3 2

2
xf ( x 2 )
H 2C CH3
H2/Ni
CH3 2. (0) I = ò 2 + f ( x + 1)
dx
(D) H 3C –1

–1 < x < 2 Þ 0 < x2 < 4


19. A-(P, Q, S), B-(T), C-(Q, R), D-(R)
Also 0 < x2 < 1 Þ f(x2) = [x2] = 0
ì Siderite ® FeCO 3 1 < x2 < 2 Þ f(x2) = [x2] = 1
ï Malachite 2 < x2 < 3 Þ f(x2) = 0 (using definition of f)
® CuCO3 . Cu(OH) 2
ï 3 < x2 < 4 Þ f(x2) = 0 (using definition of f)
ï
ï Also 1 < x2 < 2 Þ 1 < x < 2
í Bauxite ® Al2 O3 .xH2 O or
ï AlOx (OH)3 - 2x 0 < x < 1 Þ 2<x+1< 2 +1
ï
ï Calamine ® ZnCO3 Þ f(x + 1) = 0
ï Argentite ® Ag 2S
î 2 2
x ´1 é x2 ù 2 1 1
20. A- (R, T), B -(P, Q, S), C -(P, Q, S), D- (P, Q, S, T) \ I= ò 2+0
dx = ê ú = – =
(A) ® R, T 1 ëê 4 ûú 1
4 4 4

ˆˆ† H O(s) Þ 4I = 1 or 4I – 1 = 0
H2O( l ) ‡ˆˆ 2 3. (4) Let r be the internal radius and R be the external radius.
It is at equilibrium at 273 K and 1 atm
Let h be the internal height of the cylinder.
So DSsys is negative
As it is equilibrium process so DG = 0 V
(B) ® P, Q, S \ pr2h = V Þ h =
pr 2
Expansion of 1 mole of an ideal gas in vacuum under isolated
Also Vol. of material = M = p[(r + 2)2 – r2]h + p(r + 2)2 × 2
condition
EBD_7801
2015-24 JEE Advanced 2015 Solved Paper
6. (4) Let (t2,
2t) be any point on y2 = 4x. Let (h, k) be the
V
or M = 4p(r + 1). + 2p (r + 2)2 image of (t2, 2t) in the line x + y + 4 = 0. Then
pr 2
h - t 2 k - 2t –2(t 2 + 2t + 4)
= =
é1 1 ù 1 1 2
Þ M = 4V ê r + 2 ú + 2p(r + 2)2
ë r û Þ h = –(2t + 4) and k = –(t2 + 4)
For its intersection with, y = –5, we have
dM é –1 2 ù –(t2 + 4) = –5 Þ t = +1
= 4V ê 2 – 3 ú + 4p(r + 2)
dr ër r û \ A(–6, –5) and B(–2, –5)
\ AB = 4.
dM 7. (8) P(x > 2) > 0.96
For min. value of M, =0
dr Þ 1 – P(x = 0) – P(x = 1) > 0.96
Þ P(x = 0) + P(x = 1) < 0.04
–4V
Þ (r + 2) + 4p(r + 2) = 0 n n
r3 æ 1ö æ 1ö
Þ çè ÷ø + n çè ÷ø < 0.04
2 2
4V V
Þ 3 = 4p or r3 = = 1000 n +1 1
r p Þ £
n 25
\ V = 1000p 2

V 2n
\ =4 Þ > 25
250p n +1
Þ minimum value of n is 8.
x 2 +p / 6
4. (3) F(x) = ò 2 cos2 t dt 8. (5) n = 5! × 6!
x
For second arrangement,
æ 2 pö 5 boys can be made to stand in a row in 5! ways, creating 6
F ¢ (a ) = 2cos2 çè a + ÷ø .2a – 2cos2a alternate space for girls. A group of 4 girls can be selected in
6 5C ways. A group of 4 and single girl can be arranged at 2
4
a places out of 6 in 6 P2 ways. Also 4 girls can arrange
F ¢ (a ) + 2 = ò0 f ( x )dx themselves in 4! ways.
\ m = 5! × 6P2 × 5C4 × 4!
Þ F ¢¢(a) = f(a)
m 5!´ 6 ´ 5 ´ 5 ´ 4!
= =5
æ 2 pö é æ 2 pö ù n 5!´ 6!
\ f(a) = 4a.2cos çè a + 6 ÷ø . ê – sin çè a + 6 ÷ø ú .2a 9. (b, c) R2 – R1 , R3 – R2
ë û

æ 2 pö (1 + a )2 (1 + 2a ) 2 (1 + 3a ) 2
+ 4 cos2 çè a + ÷ø – 4cosa (–sina) 2a + 3 4a + 3 6a + 3 = –648a
6
2a + 5 4a + 5 6a + 5
p 3
\ f(0) = 4cos2 =4× =3 R3 – R2
6 4

5 (1 + a )2 (1 + 2a )2 (1 + 3a )2
5. (8) cos22x + cos4x + sin4x + cos6x + sin6x = 2 2 2a + 3 4a + 3 6a + 3 = –648a
4
1 1 1
5 1 3
Þ cos22x + 1 – sin 22x + 1 – sin 22x = 2 C2 – C1, C3 – C2
4 2 4

5 (1 + a)2 a (3a + 2) a (5a + 2)


Þ (cos22x – sin22x) = 0 Þ cos4x = 0 2a + 3 2a 2a = –324a
4
1 0 0
p p
Þ 4x = (2n + 1) or x = (2n + 1) Þ 2a2(–2a) = –324a
2 8
Þ a3 – 81a = 0
For xÎ[0, 2p], n can take values 0 to 7
Þ a = 0, 9, –9
\ 8 solutions.
JEE Advanced 2015 Solved Paper 2015-25

10. (b, d) P3 : x + ly + z – 1 = 0
æ a2 ö
l -1 ç ,a ÷
ç 2 ÷P
Also = 1 Þ l2 – 2l + 1 = l2 + 2 è ø
2 + l2

–1
Þ l=
2
O
1
a + lb + g - 1 a – b + g –1
And =2 Þ 2 =+2 Q
2 + l2 3 æ b2 ö
ç ,b÷
2 ç 2 ÷
è ø

1 Q PQ is the diameter of circle through P, O, Q


a – b + g –1 = ±3
2
a b
Þ 2a – b + 2g – 2 = +6 \ ÐPOQ = 90° Þ ´ = –1 Þ ab = –4
2 2
Þ 2a – b + 2g – 8 = 0 or 2a – b + 2g + 4 = 0 a /2 b /2
x y z Þ b is negative.
11. (a, b) L : = = =l
a b c Also ar. DPOQ = 3 2

Where a + 2b – c = 0 ì As L is parallel 0 0 1
í
2a - b + c = 0 îto both P1 and P2 .
1 a2
Þ a 1 =3 2
a b c 2 2
Þ = =
1 –3 –5 b2
b 1
\ Any point on line L is (l, –3l, –5l) 2
Equation of line perpendicular to P1 drawn from any point
on L is 1
Þ ab (a – b) = + 3 2
x - l y + 3l z + 5l 4
= = =µ
1 2 –1 Þ a–b=+ 3 2 (using ab = –4)
\ M(µ + l, 2µ – 3l, –µ – 5l)
But M lies on P1, As a is positive and b is negative, we have a – b = 3 2
\ µ + l + 4µ – 6l + µ + 5l + 1 = 0
4
a+ = (using ab = –4)
Þ µ=
–1 a 3 2
6
Þ a2 – 3 2 a + 4 = 0
æ 1 –1 1ö
\ M çè l – , –3l , –5l + ÷ø Þ a2 – 2 2 a – 2a+4=0
6 3 6
For locus of M, Þ (a – 2 2 ) (a – 2)=0
1 1 1 Þ a = 2 2, 2
x=l– , y = –3l – , z = 5l +
6 3 6
æ
( ) ö æ
( 2) ö
2 2
x + 1/ 6 y + 1/ 3 z - 1/ 6 2 2
= = ç ,2 2 ÷ ç , 2÷
Þ =l \ Point P can be ç 2 ÷ or ç ÷
1 –3 –5 2
è ø è ø
æ –5 –2 ö
On checking the given point, we find çè 0, , ÷ø and i.e. (4, 2 2 ) or (1,
6 3 2)

æ –1 –1 1 ö dy ex 1
çè , , ÷ø satisfy the above eqn. 13. (a, c) + y=
6 3 6 dx 1 + e x
1 + ex
12. (a, d) Let point P in first quadrant, lying on parabola y2 = 2x I.F. = 1 + ex
æ a2 ö æ b2 ö \ Soln : y(1 + ex) = x + c
ç
be 2 , a ÷ . Let Q be the point ç 2 , b÷ . Clearly a > 0. y(0) = 2 Þ c = 4
è ø è ø
EBD_7801
2015-26 JEE Advanced 2015 Solved Paper

x+4 ì – g ¢( x ) , x < 0
\ y= x ï
e +1 0 , x=0
f ' (x) = í
\ y(–4) = 0 ï g ¢ ( x) , x > 0
î
dy (e x + 1) – e x ( x + 4) \ Lf ' (0) = –g'(0) = 0
Also =
dx (e x + 1) 2 Rf ' (0) = g'(0) = 0
\ f is differentiable at x = 0
dy
For critical point =0
ïìe
–x
dx , x<0
h(x) = e|x| = í x
Þ ex(x + 3) = 1 ïî e , x³0
Þ x + 3 = e–x
Its solution will be intersection point of y = x + 3 and y = e–x
ïì – e
–x
, x<0
h'(x) = í x Þ Lh'(0) = –1, Rh'(0) = 1
ïî e , x³0
3 \ h is not differentiable at x = 0
x+
y= f oh(x) = f(h(x)) = g(e|x|) as e|x| > 0

y = e–x ì g (e – x ) if x<0
ï
g (1) if x=0
(0, 1) = íï
x
î g (e ) if x>0
–1
ì – g ¢(e – x ).e – x , x<0
ï
f ' [h(x)] = í 0 , x=0
Clearly there is a critical point in (–1, 0).
ï x x
14. (b, c) Let the equation of circle be î g ¢(e )e , x>0
x2 + y2 + 2gx + 2gy + c = 0
Þ 2x + 2yy' + 2g + 2gy' = 0 Lf ' (h(0)) = –g ' (1)
Þ x + yy' + g + gy' = 0 ...(i) Rf ' (h(0)) = g ' (1)
Differentiating again, Q g ' (1) ¹ 0, \ Lf ' (h(0)) ¹ Rf ' (h(0))
1 + yy" + (y')2 + gy" = 0 \ foh is not differentiable at x = 0.

é1 + ( y ¢)2 + yy ¢¢ ù ìïe f ( x) , x¹0


Þ g=–ê ú hof(x) = í
y ¢¢ ïî 1 , x =0
ëê ûú
Substituting value of g in eqn. (i) h( f (0)) – h( f (0 – k ))
Lh'(f(0)) = lim
k ®0 k
1 + ( y ¢ )2 + yy ¢¢ æ 1 + ( y ¢ )2 + yy ¢¢ ö
x + yy' – –ç ÷ y' = 0
y ¢¢ è y ¢¢ ø g (– k ) 1- e
g (– k )
g (– k )
1- e ´
= lim = lim
Þ xy" + yy'y" – 1– (y')2 – yy" – y' – (y')3 – yy¢y¢¢ = 0 k ®0 k k ®0 g (– k ) k
Þ (x – y)y" – y'(1 + y' + (y')2) = 1
æ g (–k ) g (k ) ö
or (y – x)y" + [1 + y' + (y')2] y' + 1 = 0 = 1 × 0 = 0 çQ g ¢(0) = 0 Þ lim = lim = 0÷
Py" + Qy' + 1 = 0 è k ®0 k k ®0 k ø
\ P = y – x, Q = 1 + y' + (y')2 h( f (0 + k )) – h ( f (0))
Also P + Q = 1 – x + y + y' + (y')2 Rh'(f(0)) = lim
k ®0 k
ìx
ï g ( x ), x ¹ 0 e
g (k )
–1 e
g (k )
–1 g (k )
15. (a, d) f(x) = í x = lim = lim ´ =0
ï0 k ®0 k k ®0 g (k ) k
î ,x = 0
\ hof is differentiable at x = 0.
ì – g ( x) , x < 0 16. (a, b, c)
ï
0 , x=0 æp æp öö
= í f(x) = sin ç sin ç sin x ÷ ÷
ï g ( x) , x > 0 6 2
î è è øø
JEE Advanced 2015 Solved Paper 2015-27

p p p r2 r2 rr r2
–1< sin x <1 Þ – £ sin x £ Þ b + c + 2b.c = a
2 2 2
r2 r2 r
æp ö Þ 48 + c + 48 = 144 Þ c = 48 Þ c = 4 3
Þ –1 < sin ç sin x ÷ < 1
è 2 ø
r2
–p p æp ö p c r 48
Þ < sin ç sin x ÷ < \ –a = –12 = 12
6 6 è 2 ø 6 2 2

ép æp öù 1 r2
–1 c r
Þ < sin ê sin ç sin x ÷ ú £
2 ë 6 è 2 øû 2 + a = 24 ¹ 30
2
é –1 1 ù r r
\ Range of f = ê , ú Also b = c Þ ÐQ = ÐR
ë 2 2û
r r
ép æ p æ p b.c 1
ö öù and cos(180 – P) = r r =
fog(x) = sin ê 6 sin ç 2 sin ç 2 sin x ÷ ÷ ú b c 2
ë è è ø øû
Þ ÐP = 120° \ ÐQ = ÐR = 30°
é –1 1 ù r r r r r r r r r r r r
Range of fog = ê , ú Again a + b + c = 0 Þ a ´ (a + b + c) = 0 Þ a ´ b = c ´ a
ë 2 2û
r r r r r r
\ a ´ b + c ´ a = 2 a ´ b = 2 × 12 × 4 3 × sin150 = 48 3
æp æp öö
sin ç sin ç sin x ÷ ÷ rr
lim è6 è2 øø And a.b = 12 ´ 4 3 ´ cos150 = –72
x ®0 p 18. (c, d)
sin x
2 X ' = –X, Y ' = –Y, Z ' = Z
(Y 3Z 4 – Z 4Y 3)' = (Z 4)'(Y 3)' – (Y 3)'(Z 4)'
æp æp öö p æ p ö = (Z ')4(Y ')3 – (Y ')3(Z ')4
sin ç sin ç sin x ÷ ÷ sin ç sin x ÷
è 6 è2 øø 6 è 2 ø = –Z 4Y 3 + Y 3Z 4 = Y 3Z 4 – Z 4Y 3
= lim ´ \ Symmetric matrix.
x ®0 p æp ö p
sin ç sin x ÷ sin x Similarly X 44 + Y 44 is symmetric matrix and X 4Z 3 – Z 3X 4
6 è2 ø 2
and X 23 + Y 23 are skew symmetric matrices.
= p/6 19. [A ® Q; B ® P, Q; C ® P, Q, S, T; D ® Q, T]

p æ æp æp ööö 3a + b 2 3 -b
gof(x) = sin ç sin ç 6 sin ç 2 sin x ÷ ÷ ÷ (A) = 3 Þa=
2 è è è øøø 2 3

p æ1ö p æ1ö 2 3 -b
– sin ç ÷ £ g ( f ( x)) £ sin ç ÷ \ = 2 + 3b Þ b = 0 Þ a = 2
2 è2ø 2 è2ø 3
–0.73 < g(f (x)) < 0.73 (B) Lf '(1) = –6a and Rf '(1) = b
\ gof(x) ¹ 1 for any x Î R. –6a = b ...(i)
17. (a, c, d) Also f is continuous at x = 1,
r r r r \ –3a – 2 = b + a2
a +b+c =o Þ a2 – 3a + 2 = 0 (using (i))
Þ a = 1, 2
P (C) (3 – 3w + 2w2 ) 4n + 3 + (2 + 3w – 3w2 ) 4n + 3

+ (–3 + 2w + 3w2)4n + 3 = 0
c b 4n +3
æ 2w2 + 3 - 3w ö
Þ (3 – 3w + 2w2 ) 4n + 3 + ç ÷
ç w2 ÷
è ø
Q R
a 4n + 3
æ –3w + 2w 2 + 3 ö
+ç ÷ =0
r r2 r2 è w ø
Þ b + c = –a
Þ (3 – 3w + 2w2)4n + 3 [1 + w4n + 3 + (w2)4n + 3] = 0
EBD_7801
2015-28 JEE Advanced 2015 Solved Paper
Þ 4n + 3 should be an integer other than multiple of 3. (D) For a = 0, y = 3
\ n = 1, 2, 4, 5 For a = 1, y = |x – 1| + |x – 2| + x
Case I
2ab
(D) = 4 Þ ab = 2a + 2b ...(i) F(a) is the area bounded by x = 0, x = 2, y2 = 4x and y = 3
a+b
Also a + q = 10 or a = 10 – q 2

and b + 5 = 2q or b = 2q – 5
\ F(a) = ò0 (3 – 2 x ) dx

n
Putting values of a and b in eq (i)
y
15 5
q = 4 or Þ a = 6 or y=3
2 2
\ |q – a| = 2 or 5. x
20. [A®P, R, S; B ® P; C ® P, Q; D®S, T]
(A) 2(a2 – b2) = c2
Þ 2(sin 2x – sin2y) = sin2z
Þ 2sin(x + y) sin(x – y) = sin2z x=2
Þ 2sin(x – y) = sin z (Q sin(x + y) = sin z)
2
sin( x – y ) 1 4x x 8 2
Þ = =l = 3x – =6–
sin z 2 3 3
0
np
\ cos(npl) = 0 Þ cos = 0 Þ n = 1, 3, 5 8
2 \ F(a) + 2 =6
3
(B) 1 + cos2X – 2cos2Y = 2sinXsinY
Þ 2cos2X – 2cos2Y = 2sinXsinY Case II
Þ 1 – sin2X – 1 + 2sin 2Y = sinXsinY F(a) is the area bounded by x = 0, x = 2, y2 = 4x and
Þ sin2X + sinXsinY – 2sin2Y = 0 y = |x – 1| + |x – 2| + x
Þ (sinX – sinY) (sinX + 2sinY) = 0 ì3 – x, 0 £ x < 1

sin X î x + 1,1 £ x £ 2
Þ = 1 or –2
sin Y
1 2
\ F(a) = ò (3 - x - 2 x )dx + ò ( x + 1 – 2 x )dx
a 0 1
\ = 1.
b
y
(C) X( 3 , 1), Y(1, 3 ), Z(b, 1 – b)

y
Y(1, 3) x
1
x
y=
X( 3,1)
x=2
60°
1 2
30° æ x2 4x ö æ x2 4 ö
= ç 3x – – x÷ +ç +x– x x÷
O x ç 2 3 ÷ ç ÷
è ø0 è 2 3 ø1
By symmetry, acute angle bisector of ÐXOY is y = x.
1 4 8 2 1 4 8 2
\ Distance of Z from bisector = 3- - + 2+ 2 – – –1 + = 5 –
2 3 3 2 3 3
b -1+ b 3
= = Þ 2b – 1 = + 3 or b = 2 or –1 8 2
2 2 F(a) + =5
3
\ b = 1, 2
JEE Advanced 2015 Solved Paper 2015-29

Paper - 2
PHYSICS 3´ 6
Req = = 2W
3+ 6

1. (2) R = –
dA
=– – = =
2
d é dN ù d 2 N d N o e
–l t
( )
dt dt êë dt úû dt 2 dt 2
2 –lt –lt
\ R = No l e = (Nol) l e = Aole –lt 2W 1W
Balanced
[Q Ao = Nol] 2W
I wheatstone
8W
2t 6W 2W bridge
– l Pt lQ t
RP l P e lP e 2t 2 e 2t 6.5V 4W
= = ´ = =
\ RQ l e – lQt lQ e l Pt t 2t e
Q 10W
et
12W 4W
\ n=2
2. (2) Here ÐMPQ + ÐMQP = 60°. If ÐMPQ = r then ÐMQP
= 60 – r
Applying Snell’s law at P The equivalent resistance of balanced wheat stone
sin60° = n sin r ...(i) bridge is
Differentiating w.r.t ‘n’ we get 6 ´ 18 9
Req= = W
dr 24 2
O = sin r + n cos r × ...(ii)
dn 6.5
\ I= = 1A
2 + 4.5

60° 2W Balanced
wheat stone
P Q I bridge
60° q 6W 2W
r 60°–rr 6.5V 10W
M
12W 4W

Applying Snell’s law at Q


sin q = n sin (60° – r) ...(iii)

Differentiating the above equation w.r.t ‘n’ we get 3h


4. (2) Given mvr = Þn=3
dq é dr ù 2p
cos q
dn
= sin (60° – r) + n cos (60° – r) êë – dn úû
hr 3h é h ù
\
l
=
2p êQ l = mv ú
dq é tan r ù ë û
\cos q = sin (60° – r) – n cos (60° – r) ê –
dn ë n úû 2pr 2 é n2 ù é n2 ù
[from (ii)] \ l = 3 = 3 p ê a0 z ú êQ r = a0 ú
z úû
êë úû êë
dq 1
\ = [sin (60° – r) + cos (60° – r) tan r] ...(iv) 2 é3 ´ 3ù
dn cos q p a0 ê
\
3
l= ú = 2pa0
ë 3 û
From eq. (i), substituting n = 3 we get r = 30° \ p=2
From eq (iii), substituting n = 3 , r = 30° we get q = 5. (7) For the tension in the rod to be zero, the force on both
the masses m and m should be equal in magnitude and
60° direction. Therefore
On substituting the values of r and q in eq (iv) we get
M
dq 1
= m m
dn cos 60° [sin 30° + cos 30° tan 30°] = 2
3. (1) The equivalent resistance of balanced wheatstone
bridge is 3l l
EBD_7801
2015-30 JEE Advanced 2015 Solved Paper

GMm Gmm GMm Gmm IB 6


+ = – \ = Þn=6
2 2 2 2 I A 10
(4l) l (3l) l

é1 1 ù é p 2p ù
\ 2m = M ê – ú 8. (3) y = I 0 êsin O + sin + sin + sin pú
ë 9 16 û ë 3 3 û

7M é 3 3ù
\ m= y = I0 ê + ú = 3 I0
288 ë 2 2 û
K=7 \ Ir = y2 = 3I0 Þ n=3
6. (4) E = A2 e–0.2t 9. (a, c)We know that
\ loge E = 2 loge A –0.2t
On differentiating we get 1 m0
C= and R =
m0e 0 e0
dE dA dt
=2 – 0.2 ´ t Now, m0 I2 = e0 V2
E A t
As errors always add up therefore m0 V 2
\ = = R2 Þ Option A is correct
dE æ dA ö æ dt ö e0 I 2
´ 100 = 2 ç ´ 100 ÷ + 0.2t ç ´ 100 ÷
E è A ø è t ø Now, e0 I = m0 V
m0 I 1
dE \ = = Þ Option B is incorrect
\ × 100 = 2 × 1.25% + 0.2 × 5 × 1.5% e0 V R
E
Now, I = e0C V
dE
\ × 100 = 4% 1 V
E \ = =R
e0 C I
R
7. (6) I = ò (dm)r 2 1
\ =R
0 1
e0
m0 e0
R
m0
\ =R Þ Option C is correct
e0
r dr
Now, mo C I = e0 V
m0 V R m0 1
\ = = = ´ = µ0
e0 I C C e0 1
m 0 e0
R
Þ Option (d) is incorrect
\ I = ò r ´ 4 pr 2 dr ´ r 2
10. (d) Assume the cavity to contain similar charge distribution
0
of positive and negative charge as the rest of sphere.
R Electric field at M due to uniformly distributed charge
\ I = 4 p ò r r 4 dr of the whole sphere of radius R1
0
R R M
r 4 pK
\ I A = 4p ò k ´ r 4 dr = 5
ò r dr P
r
R R
0 0
a
4pK æ R6 ö R5 O
= ç ÷ = 4pK
R ç 6 ÷ 6
è ø
R 5
ærö
I B = 4p ò K ç ÷ r 4 dr
0
èRø
ur r r
4pK R10 R5 E= r
= ´ = 4pK 3e
R5 10 10
JEE Advanced 2015 Solved Paper 2015-31

Electric field at M due the negative charge distribution P r


in the cavity GM r
\ – ò dp = ò rdr
ur r uuur 0 R3 R
E2 = MP
3e GM r é 2 2 ù
\ The total electric field at M is \ P= R -r
ur ur ur 2 R3 ë û
r r r uuur
E = E1 + E 2 = r+ MP
3e 3e é 2 9R 2 ù 2
êR – ú 7R
ur r r r r r r uuur r P (r = 3R/ 4) êë 16 úû
\ E= r+
3e 3e
( ë )
a – r éQ r + MP = a ù
û \ = = 16 =
P (r = 2 R/ 3) é 2 4 R 2 ù 5R 2 80
63

ur r r êR – ú
ëê 9 úû 9
\ E= a
3e
(d) is the correct option é 2 9 R2 ù
11. (a, b) The maximum stress that P can withstand before êR – ú
breaking is greater than Q. Therefore (A) is a correct P (r = 3R/ 5) êë 25 úû 16
and = =
option. P (r = 2 R/ 5) é 2 4R 2 ù 21
êR – ú
Strain ëê 25 úû
B and C are correct options.
Max 13. (d)
(strain P)
d/2

Max
(strain Q)
C1 C2
e1 = 2 e2 = 4
s/2

Max + C3
stress e1 = 2
Stress of P
Maximum s/2
stress of Q

The strain of P is more than Q therefore P is more ductile.


Therefore (B) is a correct option.
2 e0 s/2 4 e0 s/2
stress C1 = C2 =
Y= d/2 d/2
strain
For a given strain, stress is more for Q. Therefore
YQ > YP.
12. (b, c) Let us consider an elemental mass dm shown in the
shaded portion.
2 e 0 s/2 e0s
C3 = =
d d
dr 2e 0 s 4 e 0 s
r ´
C1 ´ C2 d d + e0 s
p Ceq = + C3 =
p + dp C1 + C2 6e 0 s d
d
4 e 0 s e0 s
= +
3 d d
GMr 7 e0 s 7 é e0 s ù
Here P 4pr2 – (P + dP) 4pr2 = r (4pr2) dr \ Ceq = = C1 êQ C1 = d ú
R3 3 d 3 ë û
EBD_7801
2015-32 JEE Advanced 2015 Solved Paper
14. (b) Applying combined gas law
é PV
1 1 P2V2 4 P1 ù
PV
1 1 PV ê Here on applying T = T we get P2 = 3 ú
= 2 2 ê 1 2 ú
T1 T2 ê ú
2V1
If V2 = 2 V1 and T2 = 3T1 then ê and V2 = V1 + Ax Þ x = [QV2 = 3V1 ] ú
ë A û
PV
1 1 P ´ 2V1 2
= 2 Þ P1 = P2 1 P1 A 2V1 7
T1 3T1 3 \ W = 2P1V1 + ´ ´ = PV
1 1
2 3 A 3
Now change in internal energy C is correct option
f f Heat supplied
DU = [nR (T2 – T1)] = [P2V2 – P1V1] Q = W + DU
2 2
For monoatomic gas f = 3 7 PV
1 1 3
= + ( P2V2 – PV
1 1)
3 2
3 é3 ù
DU = ê P1 ´ 2V1 – PV
1 1 ú = 3P1 V1 7 PV 3 é4 ù 41
2 ë2 û =
1 1
+ ê P1 3V1 – PV PV
1 1ú = 1 1
\ (b) is the correct option. 3 2 ë3 û 6
Now assuming that the pressure on the piston on the 236
® 140 94
15. (a) 92 U 54 Xe + 38 Sr + x + y
right hand side (not considering the affect of spring) The number of proton in reactants is equal to the
remains the same throughout the motion of the piston products (leaving x and y) and mass number of product
then, (leaving x and y) is two less than reactants
kx kx \ x = p, y = e– is ruled out [B] is incorrect
Pressure of gas = P1 + Þ P2 = P1 + and x = p, y = n is ruled out [C] is incorrect
A A
Total energy loss = (236 × 7.5) – [140 × 8.5 + 94 × 8.5]
where k is spring constant and A = area of piston
= 219 MeV
1 2 The energies of kx and ky together is 4MeV
Energy stored = kx The energy remain is distributed by Sr and Xe which is
2
equal to 219 – 4 = 215 MeV
kx \ A is the correct option
P2 = P1 +
A Also momentum is conserved

3 kx 1
P1 = P1 + \ K .E. µ . Therefore K.Esr > K.Exe
2 A m
16. (a, d) From the figure it is clear that
P1 kx (a) s2 > s1
=
2 A (b) r2 > s2 [As the string is taut]
(c) r1 < s1 [As the string is taut]
P1 A \ r1 < s1 < s2 < r2
\ kx =
2 When P alone is in L2
Also,
V2 = V1 + Ax 2pr 2 (r1 – s 2 ) g
VP = is negative as r1 < s2
V1 = Ax 9h2
Where r is radius of sphere.
V
\ x= 1 When Q alone is in L1
A
2pr 2 (r2 – s1 ) g
1 P1 A V1 1 VQ = is positive as r2 > s1
\ Energy = ´ = PV
1 1 9h1
2 2 A 4
ur ur
\ A is correct Therefore V P . V Q < O option (d) is correct
Now
æ kx ö kx
W = ò PdV = ò ç P1 + ÷ dV = ò P1dV + ò dV
è Aø A s1

kx T
\ W = ò P1dV + ò ´ (dx) A T
A
s2
kx 2
\ W = P1 (V2 – V1 ) +
2
JEE Advanced 2015 Solved Paper 2015-33

VP r1 – s 2 h1 15 45 9 3 15
Also = ´ ...(i) NA = - =
VQ r2 – s1 h2 6 16 4 24
For equilibrium of Q For S2 (in water)
4 3 4 3 64 49 3 15
T+ pr s 2 g = pr 3 r 2 g ...(ii) NA = – =
3 3 4 25 25 4 5
For equilibrium of P For S2 (in air)
4 3 4 64 49 15
T+ pr r1 g = pr 3 s1 g ...(iii) NA = – =
3 3 25 25 5
(iii) – (ii) gives 4
r1 – s2 = s1 – r2 ...(iv) For S2 (in ns = )
From (i) and (iv) 15

VP h VP h1 15 64 49 3
=– 1 = NA = – =
VQ h2 \ VQ h2 4 25 25 4
\ A is also a correct option æ 16 ö
17. (a, c) For S2 ç in ns = ÷
è 3 15 ø
Applying Snell’s law at P; ns sin im = n1 sin (90° – C)
ns = Refractive index of surrounding 3 15 64 49 9
NA = – =
Q n2 16 25 25 16

ns n1 (a), (c) are correct options


C
P 90° – C 1
18. (d) NA = n12 – n22
im ns
Here
NA2 < NA1
\ the NA of combined structure is equal to the
n2 smaller value of the two numerical apertures.
Also sin C = (d) is the correct option.
n1
19. (a, d) When megnetic force balances electric force
FB = FE
q vd B = q E
n1
V
n2 \ vd B = [Q V = E × w]
w
C é I ù
\ V = wvdB = w ê ú´B
Now ë newd û
é I I ù
n1 n1 n22 êvd = neA = newd ú
NA = sin im = n cos C = n 1 – ë û
s s n12
I
\ V = ´B
ned
n12 – n22
\ NA = 1
ns \ Vµ Þ V1d1 = V2d2
d
For S1 (in air)
when d1 = 2d2, V2 = 2V1
45 9 3 and when d1 = d2, V2 = V1
NA = – = (a), (d) are correct options
16 4 4
For S1 (in water) 20. (a, d) Here
B Vn V n
3 45 9 9 Vµ Þ 11= 2 2
NA = – = n B1 B2
4 16 4 16
If B1 = B2 and n1 = 2n2 Þ V2 = 2V1
æ 6 ö and of B1 = 2B2 and n1 = n2 Þ V2 = 0.5V1
For s1 ç in ns = ÷ A and C are the correct options.
è 15 ø
EBD_7801
2015-34 JEE Advanced 2015 Solved Paper
CHEMISTRY 3. (9) Number of moles in gas phase, at start (ni) = 1
238 206 4 0
1. (6) 3B2H6 + 18CH3OH ® 6B(OCH3)3 + 18H2 92 U ®82 Pb + 82 He + 6 – b
2. (3) 1 ® HX 2 ® HY Now number of moles in gas phase, after decomposition (n F)
= 1 + 8 = 9 mole
( l m ) HX ( l m )HY at constant temperature and pressure
a1 = a2 =
l°m l °m
PF n 9
= F = =9
K a1 = C1a12 K a 2 = C2a 22 Pin n in 1

( l m )2HX ( lm ) 2 HY
4. (8) 8H+ + 5[Fe(H2O)2(OX)2]2– + MnO4– ® Mn2+ +
= 0.01
(l )
° 2
m
= 0.1
( ) l°m
2 + 5 [Fe(H2O)2(OX)2]– + 4H2O

1 d[H + ] d[MnO 4– ]
2 Rate = =–
K a1 ( ) 2
0.01 l m HX æ ( l m ) HX ö 8 dt dt
\ = = 0.1ç ÷
( ) è ( l m ) HY ø
Ka 2 2
0.1 l m HY
rate of [H + ]decay
Hence, =8
æ 1ö
2 rate of [MnO 4– ] decay
= 0.1 ç ÷ = 10–3
è 10 ø
K a1
pKa(HX) – pKa(HY) = –log K = –log10–3 = 3
a2

+
H 1, 2– Methyl Shift D
+
5. (4) H – H2O + + –H

OH CH CH
3 3
aq. dilute 0°C
KMnO4excess

OH

OH
OH
OH
CHO

6. (4)
CO, HCl
(Gatterman Koch Reaction)
Anhyd AlCl3 /CuCl

OH
CHCl 2 CH CHO
H2O OH
100°C – H 2O

COCl CHO
H2
(Rosenmund Reduction)
Pd–BaSO 4

COOMe CHO
DIBAL–H
Toluene, –78°C H2O
JEE Advanced 2015 Solved Paper 2015-35

Et3P CO
O
+2 –
7. (3) CH3– C – Fe Br

Et3P CO
8. (6) All the complexes given show cis-trans isomerism
[Co (NH2 — CH2 — CH2 — NH2) Cl2] + [CrCl2(C2O4)2]3–
en Cl ox
Cl
en Co en en Co ox Cr ox ox Cr
Cl Cl
Cl Cl Cl

[Fe(H2O)4(OH)2]+ [Fe(NH3)2(CN)4]–
OH OH CN NH3
HO OH2 HO OH
2 2 HN
3
CN NC CN
Fe Fe Fe Fe
HO
2 OH2 HO
2 OH2 HN CN NC CN
3
OH OH2 CN NH3

[Co(NH2–CH2–CH2–NH2)2(NH3)Cl]2+ (Co(NH3)4(H2O)Cl]2+
en NH3 OH2
NH3 HO NH3 H3N
2 NH3
en Co Co Co
en Co en
NH3 Cl NH3 H3N NH3
Cl NH3 Cl
Cl

+
,H Radical, initiator, O 2
9. (b)
High Pressure, heat
Cumene, T Cumene hydroperoxide, U

NH 2
10. (a)

Cl Cl
11. (b, c) Cl–O–H Cl
O O O O OH
OH O OH
(i) (ii) (iii) (iv)
Number of Cl = O bonds in (ii) and (iii) together is 3
Number of lone pairs on Cl in (ii) and (iii) together is 3
Hybridisation of Cl in all the four is sp3
Strongest acid is HClO4 (iv)
EBD_7801
2015-36 JEE Advanced 2015 Solved Paper
12. (c, d) Only group II cations precipitate as sulphide with H2S 15. (c) P(V–b) = RT
in acidic medium that is (Cu2+, Pb2+) and (Hg2+, Bi3+) Þ PV – Pb = RT
13. (b) (CH3)2SiCl2 form linear polymer on hydrolysis and PV Pb
(CH3)3SiCl is a chain terminator. Þ = +1
RT RT
14. (b, c, d) Reaction on metal surface
Pb
Þ Z =1+
® M+ + e–
M ¾¾ RT
Hence Z > 1 at all pressures.
O2 + e – ¾¾
® O2–
This means , repulsive tendencies will be dominant
This is an example of chemisorption. when interatomic distance are small.
This means, interatomic potential is never negative
but becomes positive at small interatomic distances.

16. (a)
(i) O3 NH 3
(ii) Zn, H 2O

–2H2O

17. (a) Let the heat capacity of insulated beaker be C.


Mass of aqueous content in expt. 1 = (100 + 100) × 1 19. (c) OH
= 200 g Pd-BaSO4 i. B 2H6
Þ ± Total heat capacity = (C + 200 × 4.2) J/K C8 H 6 H2 ii. H2 O2 , NaOH, H2O
Moles of acid, base neutralised in expt.
1 = 0.1 × 1 = 0.1
Þ Heat released in expt. 1 = 0.1 × 57 = 5.7 KJ
= 5.7 × 1000 J
Þ 5.7 × 1000 = (C + 200 × 4.2) × D´T. O
5.7 × 1000 = (C + 200 + 4.2) × 5.7
Þ (C + 200 × 4.2) = 1000 H2O
In second experiment, 20. (d) C8H6
HgSO4 ,
nCH COOH = 0.2, nNaOH = – 0.1 H2SO4
3
i. Et MgBr, H2O
Total mass of aqueous content = 200 g
+
Þ Total heat capacity = (C + 200 × 4.2) = 1000 ii. H , Heat
Þ Heat released = 1000 × 5.6 = 5600 J.
Overall, only 0.1 mol of CH3COOH undergo
neutralization.
–5600
Þ DHneutralization of CH3COOH =
0.1
= – 56000 J/mol
= – 56 KJ/mol. Y
Þ DHneutralization of CH3COOH = 57 – 56 = 1 KJ/mol
18. (b) Final solution contain 0.1 mole of CH3COOH and
CH3COONa each.
Hence it is a buffer solution.
[CH3COO - ]
pH = pKa log
[CH3 COOH]
0.1
= 5–log 2 + log = 4.7
0.1
JEE Advanced 2015 Solved Paper 2015-37
r
MATHEMATICS 4. (9)
r r r
s = 4 p + 3q + 5r
r r r r r r r r r r
ecos a – e
n
–e s = x(– p + q + r ) + y( p - q + r ) + z(– p - q + r )
1. (2) lim =
a® 0 a m 2 Þ –x+y–z=4
x–y–z=3
é n ù x+y+z=5
e êe cos a – 1 – 1ú n
ë û ´ cos a – 1 = –e
Þ lim 9 -7
a®0 cos a n - 1 am 2 Solving above equations x = 4, y = ,z=
2 2
2 \ 2x + y + z = 9
n æ an ö
2 a ç ÷ ipk
–2 sin ç 2 ÷ kp kp
Þ 2 è ø = –e (4) ak = cos
e lim ´ 5. + i sin =e 7
a®0 æ n ö2 a m 2 7 7
a
çç ÷÷
è 2 ø ip( k +1) ipk ipk
ak + 1 – ak = e 7 –e 7 =e 7 (eip/7 - 1)
– e 2n – m – e
Þ a = or a 2 n – m = 1
2 2 a k +1 - a k = eip / 7 - 1

m
Þ 2n – m = 0 Þ = 2 12
n
Þ å ak +1 – a k = 12 eip / 7 - 1
k =1
1 (9 x +3tan –1 x ) æ 12 + 9 x 2 ö
2. (9) a = ò0
e ç
è 1 + x2 ø
÷ dx 3
Similarly å a 4k -1 – a 4 k - 2 = 3 ei p / 7 - 1
k =1
12 + 9 x 2
Let 9x + 3tan–1x = t Þ dx = dt
1 + x2 12

3p
å a k +1 – a k
3p k =1
9+ 9+ \ =4
\ a= 4 e t dt
ò0 =e 4 -1 3
å a 4k -1 – a 4k -2
k =1
3p
9+ 3p
\ loge 1 + e 4 –1 – =9
4 7
[2a + 6d ]
2 6
= Þ a = 9d
6. (9) 11 11
x [2a + 10d ]
lim
F ( x)
=
1 ò
Þ lim x –1
f (t )dt 2
3. (7) x ®1 G ( x) 14 x®1
ò t f ( f (t )) dt
–1
a7 = a + 6d = 15d
Q 130 < 15d < 140 Þ d = 9
1 1 (Q All terms are natural numbers \ d Î N )
Q ò-1 f (t )dt = 0 and ò-1t f ( f (t )) dt = 0
7. (8) In expansion of (1 + x) (1 + x2) (1 + x3) .... (1 + x100)
f(t) being odd function x9 can be found in the following ways
\ Using L Hospital’s rule, we get
x9, x1 + 8, x2 + 7, x3 + 6, x4 + 5, x1 + 2 + 6, x1 + 3 + 5, x2 + 3 + 4
f ( x) 1 The coefficient of x9 in each of the above 8 cases is 1.
lim =
x ®1 x f ( f ( x )) 14 \ Required coefficient = 8.

f (1) 1 1/ 2 1 x2 y2
Þ = Þ = 8. (4) Ellipse: + =1
f ( f (1)) 14 æ 1 ö 14 9 5
fç ÷
è 2ø
2
Þ a = 3, b = 5 and e = 3
æ 1ö æ 1ö
Þ f ç ÷ =7 Þ fç ÷ =7 \ f1 = 2 and f2 = –2
è 2ø è 2ø
EBD_7801
2015-38 JEE Advanced 2015 Solved Paper
P1 : y2 = 8x and P2 : y2 = –16x eqn of tangent to E1 at Q is
2 5x 4y x y
T1 : y = m1x + m 2
+ 2
= 1 which is identical to + =1
1 3a 3b 3 3
It passes through (–4, 0), 4 1
Þ a2 = 5 and b2 = 4 Þ e12 = 1 – =
2 1 5 5
0 = –4m1 + Þ m12 =
m1 2 eqn of tangent to E2 at R is

4 x 8y x y
T2 : y = m2x – 2
+ 2
= 1 identical to + =1
m2 3c 3d 3 3

It passes through (2, 0) 1 7


Þ c2 = 1, d2 = 8 Þ e22 = 1 – =
4 8 8
0 = 2m2 – Þ m22 = 2
m2 43 7 27
\ e12 + e22 = , e1e2 =
2 2
, e1 - e 2 =
40 2 10 40
1
\ + m22 = 4 11. (a, b, d) H : x2 – y2 = 1 S : Circle with centre N(x2, 0)
m12
Common tangent to H and S at P(x1, y1) is
9. (b, c, d) xx1 – yy1 = 1
6 1 p x1
a = 3sin–1 > 3sin–1 or a > Þ m1 =
11 2 2 y1
\ cosa < 0
Also radius of circle S with centre N(x2, 0) through point of
4 1 contact (x1, y1) is perpendicular to tangent
b = 3cos–1 > 3cos–1 or b > p
9 2 x1 0 - y1
\ cosb < 0 and sinb < 0 \ m1m2 = –1 Þ y ´ x - x = –1
1 2 1
3p Þ x1 = x2 – x1 or x2 = 2x1
Also a + b > \ cos(a + b) > 0.
2 M is the point of intersection of tangent at P and x-axis
10. (a, b)
æ1 ö
\ M ç , 0÷
x2 y2 è x1 ø
Let E1 : + = 1 where a > b
a2 b2
Q Centroid of DPMN is (l, m)
x2 y 2 1
and E2 : 2 + 2 = 1 where c < d \ x1 + + x2 = 3l and y1 = 3m
c d x1
Also S : x2 + (y – 1)2 = 2 Using x2 = 2x1,
Tangent at P(x1, y1) to S is x + y = 3
1æ 1ö
To find point of contact put x = 3 – y in S. We get P(1, 2)
Þ ç 3 x1 + ÷ = l and y1 = m
Writing eqn of tangent in parametric form 3è x1 ø 3

x -1 y - 2 2 2 dl 1 dm 1
= =± \ =1- 2 , =
–1 1 3 dx1 3 x1 dy1 3
2 2
Also (x1, y1) lies on H, \ x12 - y12 = 1
-2 2 2 -2
x= + 1 or + 1 and y = + 2 or +2
3 3 3 3 or y1 = x12 - 1
1 5 8 4 1 2
Þ x= or and y = or \ m= x1 - 1
3 3 3 3 3
æ 5 4ö æ 1 8ö dm x1
\ Q çè , ÷ø and R çè , ÷ø \
3 3 3 3 dx1 =
3 x12 - 1
JEE Advanced 2015 Solved Paper 2015-39

12. (a, c) Let F(t) = et (sin6at + cos6at)


Then F(kp + t) = ekp + t [sin6(kp + t)a + cos6(kp + t)a] 192 x3
15. (d) f ¢(x) =
= ekp.et [sin6at + cos6at] for even values of a. 2 + sin 4 px
\ F(kp + t) = ekp F(t) ...(i)
192 x 3 192 x 3
4p p 2p 3p 4p Þ £ f ¢ ( x) £
Now ò0 F (t)dt = ò0 F (t)dt + òp F (t )dt +ò2p F (t )dt +ò3p F (t )dt 3 2
Þ 64x3 < f ' (x) £ 96x3
2p p
Also òp F (t )dt = ò F (p + x )dx (putting t = p + x) x 3 x x
96 x3 dx
0 Þ ò1/2 64 x dx £ ò
1/2
f ¢( x)dx £ ò
1/2
p p
= ò0 e F ( x )dx using eqn(i)
64 x 4 64 1 x 96 x 4 96
Þ – ´ £ ò f ¢( x )dx £ -
4 4 16 1/2 4 4 ´ 16
p
= ep ò F (t )dt
0 x 3
3p p
Þ 16x4 – 1 < ò1/ 2 f ¢( x) < 24x4 – 2
Similarly ò F (t )dt = e2p ò F (t )dt
2p 0
3
Þ 16x4 – 1 < f(x) < 24x4 –
4p p 2
ò3p F (t )dt = e3p F (t )dt
ò
0
1 1 1 æ 3ö
ò1/ 2 (16 x - 1)dx £ ò f ( x )dx £ ò
4
4p p Þ ç 24 x 4 – ÷ dx
1/ 2 è 2ø
ò0 F (t )dt = (1 + ep + e2p + e3p) ò F (t )dt 1/ 2
\
0
1 1
4p æ 16 x 5 ö 1 é 24 x 5 3 ù
ò0 F (t )dt = e –1 4p
Þ ç
è 5
- x÷ £ ò f ( x )dx £ ê
ø 1 1/ 2 êë 5
- xú
2 úû 1
Þ p p , where ‘a’ can take any even
ò0 F (t )dt e –1 2 2

1
value. Þ 2.6 < ò1/ 2 f ( x)dx < 3.9
13. (b, c) Let h(x) = f(x) – 3g(x)
h(–1) = h(0) = h(2) = 3 \ Only (d) is the correct option.
\ By Rolle’s theorem h'(x) = 0 has atleast one solution in 16. (a, d) ax2 – x + a = 0 has distinct real roots.
(–1, 0) and atleast one solution in (0, 2) But h¢¢(x) never \ D > 0 Þ 1 – 4a2 > 0
vanishes in (–1, 0) and (0, 2) therefore h'(x) = 0 should
æ 1 1ö
have exactly one solution in each interval. Þ aÎ çè – , ÷ø ...(i)
2 2
14. (a, b) f(x) = 7 tan8x + 7tan6x – 3tan4x – 3tan2x
= (7tan4x – 3) (tan4x + tan2x) Also |x1 – x2| < 1
= (7tan6x – 3tan2x) sec2x Þ (x1 – x2)2 < 1
Þ (x1 + x2)2 – 4x1x2 < 1
p/4 p/4
ò0 f ( x)dx = é tan 7 x - tan 3 x ù
ë û0 1
Þ –4<1
=1–1=0 a2

( )
p/4 p/4 1 1
\ ò0 xf ( x) dx = é x tan 7 x - tan 3 x ù Þ < 5 or a2 >
ë û0 a2 5

–ò
0
p/4
( tan 7
)
x - tan 3 x dx æ 1 ö æ 1
Þ aÎ çè – ¥, – ÷ø È çè
5
ö
, ¥÷
5 ø
...(ii)

p/4 Combining (i) and (ii)


p/4 é tan 4 x tan 6 x ù
3 2 -
= ò0 tan x (1 - tan x ) sec2x dx = ê
ëê 4 6 ûú
ú
æ 1 –1 ö æ 1 1 ö
0
S = çè – 2 , ÷ø È çè , ÷
5 5 2ø
1
=
12 æ 1 –1 ö æ 1 1ö
\ Subsets of S can be çè – 2 , ÷ø and çè , ÷.
5 5 2ø
EBD_7801
2015-40 JEE Advanced 2015 Solved Paper
17. (a, b, c) f(x) = xF(x) Þ f ' (x) = F(x) + xF'(x)
ì Using xF ¢ ( x) = f ¢( x) – F ( x )
ï
æ æ 1 öö í 3 2
\ f '(1) = F(1)+ F '(1) = F ' (1) < 0 çQ F ¢( x) < 0, x Î ç ,3 ÷ ÷ ïîand ò1 x F ¢( x )dx = –12
è è 2 øø
f(2) = 2F(2) < 0, Þ 9(f ' (3) – F(3)) – (f ' (1) – F(1)) = 4
Þ 9f ' (3) – 9 × (–4) – f ' (1) + 0 = 4
(Q F '(x) < 0 Þ F is decreasing on æ 1 ,3ö and F '(1) = 0, Þ 9f ' (3) – f ' (1) + 32 = 0
çè ÷
2 ø
19. (a,b) Let E1 º box I is selected
F(3) = –4) E2 º box II is selected
f '(x) = F(x) + x F '(x) E º ball drawn is red
For the same reason given above and F '(x) < 0 given.
F(x) < 0 " x Î(1, 3) n3 1
´
\ f '(x) ¹ 0, xÎ(1, 3). n3 + n4 2 1
P(E2/E) = n =
1 1 n3 1 3
3 2 ´ + ´
18. (c,d) ò1 x F ¢( x)dx = –12 n1 + n2 2 n3 + n4 2

3 3
é x 2 F ( x) ù – 2 x F ( x) dx = -12
û1 ò1
Þ n3
ë
n3 + n4 1
or =
3 n1 n3 3
Þ 9 F(3) – F(1) – 2 ò xF ( x) dx = -12 +
1 n1 + n2 n3 + n4
3 On checking the options we find A and B are the correct
3
Þ ò1 xF ( x) dx = -12 Þ ò f ( x)dx = -12 ...(i) options.
1 20. (c, d) E1 º Red ball is selected from box I
3 3 E2 º Black ball is selected from box I
Also ò1 x F ¢¢ ( x ) dx = 40 E º Second ball drawn from box I is red
\ P(E) = P(E1) P(E/E1) + P(E2) P(E/E2)
3
é x3 F ¢( x) ù - 3 3 x 2 F ¢( x)dx = 40
Þ ë û1 ò1 n1 n1 - 1 n2 n1
= ´ + ´
n1 + n2 n1 + n2 - 1 n1 + n2 n1 + n2 - 1
3
Þ é x 2 ( f ¢( x) - F ( x) ) ù – 3 × (–12) = 40 On checking the options, we find C and D have the correct
ë û1
values.
Telegram @unacademyplusdiscounts

Join Us on Telegram for More Such Books

https://telegram.me/unacademyplusdiscounts

Join us from the above link or search ''unacademyplusdiscounts'' in Telegram


JEE ADVANCED 2014
1. The question paper consists of three parts (Physics, Chemistry and Mathematics). Each part consists of two
sections.
2. Section 1 contains 10 multiple choice questions. Each question has four choices (a), (b), (c) and (d) out of
which ONE OR MORE THAN ONE are correct.
3. Section 2 contains 10 questions. The answer to each of the questions is a single-digit integer ranging from 0 to
9 (both inclusive).

PAPER - 1

PHYSICS n1

SECTION - I
This section contains 10 multiple choice questions. Each question Air n2
has 4 choices (a), (b), (c) and (d) out of which ONE or MORE
THAN ONE are correct.
1. Let E1 (r), E2(r) and E3(r) be the respective electric field at a
distance r from a point charge Q, an infinitely long wire with (a) f1 3R (b) f1 2.8R
constant linear charge density , and an infinite plane with (c) f2 2 R (d) f2 1.4R
uniform surface charge density . If E1(r0) = E2(r0) = E3(r0)
4. A student is performing an experiment using a resonance
at a given distance r0, then column and a tuning fork of frequency 244 s–1. He is told that
(a) Q 4 r02 the air in the tube has been replaced by another gas (assume
that the column remains filled with the gas). If the minimum
r0 height at which resonance occurs is 0.350 0.005 m, the
(b)
2 gas in the tube is
(c) E1 r0 / 2 2 E2 r0 / 2 (Useful information: 167 RT 640J1/ 2 mole 1/ 2 ;

(d) E2 r0 / 2 4 E3 r0 / 2 140 RT 590J1/ 2 mole 1/ 2 . The molar masses M in grams

2. Heater of an electric kettle is made of a wire of length L and 10


diameter d. It takes 4 minutes to raise the temperature of are given in the options. Take the values of for each
M
0.5 kg water by 40 K. This heater is replaced by a new heater gas as given there.)
having two wires of the same material, each of length L and
diameter 2d. The way these wires are connected is given in 10 7
(a) Neon M 20,
the options. How much time in minutes will it take to raise the 20 10
temperature of the same amount of water by 40 K?
(a) 4 if wires are in parallel (b) 2 if wires are in series 10 3
(c) 1 if wires are in series (d) 0.5 if wires are in parallel (b) Nitrogen M 28,
28 5
3. A transparent thin film of uniform thickness and refractive
index n1 = 1.4 is coated on the convex spherical surface of 10 9
radius R at one end of a long solid glass cylinder of refractive (c) Oxygen M 32,
32 16
index n2 = 1.5, as shown in the figure. Rays of light parallel to
the axis of the cylinder traversing through the film from air to
10 17
glass get focused at distance f1 from the film, while rays of (d) Argon M 36,
36 32
light traversing from glass to air get focused at distance f2
from the film, Then
EBD_7801
2-2014 JEE Advanced 2014 Solved Paper
5. In the figure, a ladder of mass m is shown leaning against a 8. At time t = 0, terminal A in the circuit shown in the figure is
wall. It is in static equilibrium making an angle with the connected to B by a key and an alternating current
horizontal floor. The coefficient of friction between the wall I(t) = I0cos ( t), with I0 = 1 A and = 500 rad s–1 starts
and the ladder is 1 and that between the floor and the ladder flowing in it with the initial direction shown in the figure. At
is 2. The normal reaction of the wall on the ladder is N1 and
that of the floor is N2. If the ladder is about to slip, then 7
t , the key is switched from B to D. Now onwards only
6
1
A and D are connected. A total charge Q flows from the
battery to charge the capacitor fully. If C = 20 F, R = 10
and the battery is ideal with emf of 50 V, identify the correct
statement(s).

B D
A

2 50 V
C = 20 F
mg
(a) 1 0, 2 0 and N2 tan =
2
mg R = 10
(b) 1 0, 2 0 and N1 tan
2 (a) Magnitude of the maximum charge on the capacitor
mg 7
(c) 1 0, 2 0 and N 2 before t is 1 × 10–3 C
1 1 2 6
(b) The current in the left part of the circuit just before
mg
(d) 1 0, 2 0 and N1 tan 7
2 t is clockwise
6. Two ideal batteries of emf V1 and V2 and three resistances 6
R1, R2 and R3 are connected as shown in the figure. The (c) Immediately after A is connected to D, the current in R
current in resistance R2 would be zero if is 10 A
(d) Q = 2 × 10–3 C
9. One end of a taut string of length 3 m along the x-axis is fixed
at x = 0. The speed of the waves in the string is 100 ms –1. The
V1 R1 other end of the string is vibrating in the y-direction so that
R2 stationary waves are set up in the string. The possible
waveform (s) of these stationary waves is(are)
x 50 t
V2 (a) y t A sin cos
6 3
R3
x 100 t
(b) y t A sin cos
3 3
(a) V1 = V2 and R1 = R2 = R3
(b) V1 = V2 and R1 = 2R2 = R3 5 x 250 t
(c) y t A sin cos
(c) V1 = 2V2 and 2R1 = 2R2 = R3 6 3
(d) 2V1 = V2 and 2R1 = R2 = R3
5 x
7. A light source, which emits two wavelength 1 = 400 nm and (d) y t A sin cos 250 t
2
2 = 600 nm, is used in a Young’s double slit experiment. If
recorded fringe widths for 1 and 2 are 1 and 2 and the 10. A parallel plate capacitor has a dielectric slab of dielectric
number of fringes for them within a distance y on one side of constant K between its plates that covers 1/3 of the area of
the central maximum are m1 and m2 respectively, then its plates, as shown in the figure. The total capacitance of the
capacitor is C while that of the portion with dielectric in
(a) 2> 1 between is C1. When the capacitor is charged, the plate area
(b) m1 > m2 covered by the dielectric gets charge Q1 and the rest of the
(c) Form the central maximum, 3rd maximum of 2 overlaps area gets charge Q2. The electric field in the dielectric is E1
with 5th minimum of 1 and that in the other portion is E2. Choose the correct option/
(d) The angular separation of fringes for 1 is greater options, ignoring edge effects.
than 2.
JEE Advanced 2014 Solved Paper 2014-3

Q1 E1

15. A uniform circular disc of mass 1.5 kg and radius 0.5 m is


initially at rest on a horizontal frictionless surface. Three forces
E2 of equal magnitude F = 0.5 N are applied simultaneously
Q2
along the three sides of an equilateral triangle XYZ with its
vertices on the perimeter of the disc (see figure). One second
after applying the forces, the angular speed of the disc in rad
E1 E1 1 s–1 is
(a) 1 (b) F
E2 E2 K X
Q1 3 C 2 K
(c) Q2 K (d) C1 K
O
SECTION - II
This section contains 10 questions. Each question, when worked Y F
out will result in one integer from 0 to 9 (both inclusive). Z
F
11. A galvanometer gives full scale deflection with 0.006 A 16. Consider an elliptical shaped rail PQ in the vertical plane with
current. By connecting it to a 4990 resistance, it can be OP = 3 m and OQ = 4 m. A block of mass 1 kg is pulled along
converted into a voltmeter of range 0 – 30 V. If connected to the rail from P to Q with a force of 18 N, which is always
parallel to line PQ (see the figure given). Assuming no
2n
a resistance, it becomes an ammeter of range 0 – 1.5A. frictionless losses, the kinetic energy of the block when it
249 reaches Q is (n × 10) joules. The value of n is (take acceleration
The value of n is due to gravity = 10 ms–2)
12. To find the distance d over which a signal can be seen clearly Q
in foggy conditions, a railways-engineer uses dimensions
and assumes that the distance depends on the mass density
of the fog, intensity (power/area) S of the light from the
4m
signal and its frequency f. The engineer finds that d is
proportional to S1/n. The value of n is
13. During Searle’s experiment, zero of the Vernier scale lies 90°
between 3.20 × 10–2 m and 3.25 × 10–2 m of the main scale.
O 3m P
The 20th division of the Vernier scale exactly coincides with
one of the main scale divisions. When an additional load of 17. A thermodynamic system is taken from an initial state i with
2 kg is applied to the wire, the zero of the Vernier scale still internal energy Ui = 100 J to the final state f along two different
paths iaf and ibf, as schematically shown in the figure. The
lies between 3.20 × 10–2 m and 3.25 × 10–2 m of the main scale
work done by the system along the paths af, ib and bf are
but now the 45th division of Vernier scale coincides with one
Waf = 200 J, Wib = 50 J and Wbf =100 J respectively. The heat
of the main scale divisions. The length of the thin metallic
supplied to the system along the path iaf, ib and bf are Qiaf,
wire is 2 m and its cross-sectional area is 8 × 10–7 m2. The
Qib and Qbf respectively. If the internal energy of the system
least count of the Vernier scale is 1.0 × 10–5 m. The maximum
percentage error in the Young’s modulus of the wire is Qbf
in the state b is Ub = 200 J and Qiaf = 500 J, The ratio is
14. A horizontal circular platform of radius 0.5 m and mass Qib
0.45 kg is free to rotate about its axis. Two massless spring
toy-guns, each carrying a steel ball of mass 0.05 kg are a f
attached to the platform at a distance 0.25 m from the centre P
on its either sides along its diameter (see figure). Each gun
simultaneously fires the balls horizontally and perpendicular i
to the diameter in opposite directions. After leaving the b
platform, the balls have horizontal speed of 9 ms–1 with V
respect to the ground. The rotational speed of the platform in
rad s–1 after the balls leave the platform is 18. Two parallel wires in the plane of the paper are distance X0
apart. A point charge is moving with speed u between the
EBD_7801
4-2014 JEE Advanced 2014 Solved Paper
wires in the same plane at a distance X1 from one of the wires. 3. Hydrogen bonding plays a central role in the following
When the wires carry current of magnitude I in the same phenomena
direction, the radius of curvature of the path of the point (a) Ice floats in water
charge is R1. In contrast, if the currents I in the two wires (b) Higher Lewis basicity of primary amines than tertiary
have directions opposite to each other, the radius of curvature amines in aqueous solutions
(c) Formic acid is more acidic than acetic acid
X0 R1
of the path is R2. If 3 , the value of is (d) Dimerisation of acetic acid in benzene
X1 R2 4. The correct combination of names for isomeric alcohols with
19. Airplanes A and B are flying with constant velocity in the molecular formula C4H10O is/are
same vertical plane at angles 30° and 60° with respect to the (a) Tert-butanol and 2-methylpropan-2-ol
horizontal respectively as shown in figure. The speed of A is (b) Tert-butanol and 1, 1-dimethylethan-1-ol
(c) n-butanol and butan-1-ol
100 3 m/s. At time t = 0 s, an observer in A finds B at a (d) Isobutyl alcohol and 2-methylpropan-1-ol
distance of 500 m. The observer sees B moving with a 5. The reactivity of compound Z with different halogens under
constant velocity perpendicular to the line of motion of A. If appropriate conditions is given below:
at t = t0, A just escapes being hit by B, t0 in seconds is
OH mono halo substituted
A derivative when X2 = I2
X2 di halo substituted
derivative when X2 = Br2
B Z C(CH3)3
tri halo substituted
30° 60° derivative when X2 = Cl2

20. A rocket is moving in a gravity free space with a constant The observed pattern of electrophilic substitution can be
acceleration of 2 m/s2 along +x direction (see figure). The explained by
length of a chamber inside the rocket is 4 m. A ball is thrown (a) The steric effect of the halogen
from the left end of the chamber in +x direction with a speed (b) The steric effect of the tert-butyl group
of 0.3 m/s relative to the rocket. At the same time, another ball (c) The electronic effect of the phenolic group
is thrown in –x direction with a speed of 0.2 m/s from its right (d) The electronic effect of the tert-butyl group
end relative to the rocket. The time in seconds when the two 6. The pair(s) of reagents that yield paramagnetic species is/are
balls hit each other is (a) Na and excess of NH3
(b) K and excess of O2
0.3 m/s 0.2 m/s a = 2 m/s
2 (c) Cu and dilute HNO3
x (d) O2 and 2-ethylanthraquinol
7. For the reaction
4m
I ClO3 H 2SO 4 Cl HSO4 I2
The correct statement(s) in the balanced equation is/are
CHEMISTRY
(a) Stoichiometric coefficient of HSO4 is 6
SECTION - I (b) Iodide is oxidized
This section contains 10 multiple choice questions. Each question (c) Sulphur is reduced
has 4 choices (a), (b), (c) and (d) out of which ONLY ONE or (d) H2O is one of the products
MORE THAN ONE are correct. 8. In the reaction shown below, the major product(s) formed is/
are
1. Upon heating with Cu2S, the reagent(s) that give copper NH2
metal is/are
(a) CuFeS2 (b) CuO acetic anhydride
Product(s)
(c) Cu2O (d) CuSO4 NH2 CH2Cl2
2. In a galvanic cell, the salt bridge Z O
(a) Does not participate chemically in the cell reaction H
(b) Stops the diffusion of ions from one electrode to N CH3
another O
(c) Is necessary for the occurrence of the cell reaction + CH3COOH
(a)
(d) Ensures mixing of the two electrolytic solutions NH2
O
JEE Advanced 2014 Solved Paper 2014-5

diagram. The final internal pressure, volume and absolute


NH2
temperature of the gas are P2, V2 and T2, respectively. For
this expansion,
H + CH3COOH
(b) N CH3
O O Pext = 0

H Pext = 0 Irreversible
N CH3
P2, V2, T2
O P1, V1, T1
(c) H + H2O
N CH3
O Thermal insulation
O
(a) q = 0 (b) T2 = T1
+ – (c) P2V2 = P1V1 (d) P2V2 = P1V1
NH3CH3COO
10. The correct statement(s) for orthoboric acid is/are
(a) It behaves as a weak acid in water due to self ionization.
(d) H
N CH3 (b) Acidity of its aqueous solution increases upon addition
of ethylene glycol
O O
(c) It has a three dimensional structure due to hydrogen
9. An ideal gas in a thermally insulated vessel at internal pressure bonding
= P1, volume = V1 and absolute temperature = T1 expands (d) It is a weak electrolyte in water
irreversibly against zero external pressure, as shown in the

SECTION - II
This section contains 10 questions. Each question, when worked out will result in one integer from 0 to 9 (both inclusive).
11. The total number of distinct naturally occurring amino acids obtained by complete acidic hydrolysis of the peptide shown
below is
O

O O O H
O H H N
N N N
N N N
N O
CH2 O H O H CH2
H
O

12. MX2 dissociates into M2+ and X– ions in an aqueous solution, 16. Consider the following list of reagents:
with a degree of dissociation ( ) of 0.5. The ratio of the Acidified K2Cr2O7, alkaline KMnO4, CuSO4, H2O2, Cl2, O3,
observed depression of freezing point of the aqueous solution FeCl3, HNO3 and Na2S2O3.
to the value of the depression of freezing point in the absence The total number of reagents that can oxidise aqueous iodide
of ionic dissociation is to iodine is
13. If the value of Avogadro number is 6.023 × 1023 mol–1 and 17. A list of species having the formula XZ4 is given below.
the value of Boltzmann constant is 1.380 × 10–23 J K–1, then
the number of significant digits in the calculated value of the XeF4, SF4, SiF4, BF4–, BrF4–, [Cu(NH3)4]2+, [FeCl4]2– ,
universal gas constant is [CoCl4]2– and [PtCl4]2–.
Defining shape on the basis of the location of X and Z atoms,
14. A compound H2X with molar weight of 80g is dissolved in a
the total number of species having a square planar shape is
solvent having density of 0.4 g ml–1. Assuming no change in
18. Consider all possible isomeric ketones, including
volume upon dissolution, the molality of a 3.2 molar solution
stereoisomers of MW = 100. All these isomers are
is
independently reacted with NaBH4 (NOTE: stereoisomers
15. In an atom, the total number of electrons having quantum
are also reacted separately). The total number of ketones
1 that give a racemic product(s) is/are
numbers n = 4, | ml | = 1 and ms = is
2
EBD_7801
6-2014 JEE Advanced 2014 Solved Paper
19. The total number(s) of stable conformers with non-zero (b) The second row of M is the transpose of the first column
dipole moment for the following compound is (are) of M
Cl (c) M is a diagonal matrix with non-zero entries in the main
Br CH3 diagonal
(d) The product of entries in the main diagonal of M is not
Br Cl
the square of an integer
CH3
20. Among PbS, CuS, HgS, MnS, Ag2S, NiS, CoS, Bi2S3 and 4. Let x , y and z be three vectors each of magnitude 2
SnS2, the total number of BLACK coloured sulphides is
and the angle between each pair of them is . If a is a
3
MATHEMATICS
SECTION - I non-zero vector perpendicular to x and y z and b is a
This section contains 10 multiple choice questions. Each question
non-zero vector perpendicular to y and z x , then
has 4 choices (a), (b), (c) and (d) out of which ONLY ONE or
MORE THAN ONE are correct.
(a) b b.z z x
1. Let f : [a, b] 1, be a continuous function and let
g: R R be defined as
(b) a a.y y z

(c) a.b a.y b.z


0, if x a,
x
g x f t dt , if a x b; then (d) a a. y z y
a
b 5. From a point P( , , ), perpendicular PQ and PR are drawn
f t dt , if x b. respectively on the lines y = x, z = 1 and y = – x, z = – 1. If P is
a such that QPR is a right angle, then the possible value(s)
of is/(are)
(a) 2 (b) 1
(a) g(x) is continuous but not differentiable at a
(c) – 1 (d) 2
(b) g(x) is differentiable on R
6. Let M and N be two 3 × 3 matrices such that MN = NM.
(c) g(x) is continuous but not differentiable at b
Further, if M N 2 and M 2 = N 4, then
(d) g(x) is continuous and differentiable at either (a) or (b) (a) determinant of (M 2 + MN 2) is 0
but not both (b) there is 3 × 3 non-zero matrix U such that (M 2 + MN 2)U
2. For every pair of continuous functions f, g : [0, 1] R such is the zero matrix
(c) determinant of (M 2 + MN 2) 1
that max f x :x 0,1 = max g x :x 0,1 , the (d) for a 3 × 3 matrix U, if (M 2 + MN 2)U equals the zero
correct statement(s) is (are): matrix then U is the zero matrix

(a) (f (c))2 + 3f (c) = (g(c))2 + 3g(c) for some c [0, 1] x t


1 dt
7. Let f : 0, R be given by f x t t . Then
(b) (f (c))2 + f (c) = (g(c))2 + 3g(c) for some c [0, 1] e
1
(c) (f (c))2 + 3f (c) = (g(c))2 + g(c) for some c [0, 1]
x
(d) (f (c))2 = (g(c))2 for some c [0, 1]
3. Let M be a 2 × 2 symmetric matrix with integer entries. Then M (a) f (x) is monotonically increasing on 1,
is invertible if (b) f (x) is monotonically decreasing on (0, 1)
(a) The first column of M is the transpose of the second 1
(c) f x f 0, for all x 0,
row of M x
(d) f (2x) is an odd function of x on R
JEE Advanced 2014 Solved Paper 2014-7

15. For a point P in the plane, let d1(P) and d2(P) be the distance
8. Let f : , R be given by f (x) = (log(sec x + tan x))3. of the point P form the lines x – y = 0 and x + y = 0 respectively.
2 2
Then The area of the region R consisting of all points P lying
(a) f (x) is an odd function in the first quadrant of the plane and satisfying
(b) f (x) is non-one function
(c) f (x) is an onto function 2 d1 P d2 P 4 , is
(d) f (x) is an even function 16. Let n1 < n2 < n3 < n4 < n5 be positive integers such that
9. A circle S passes through the point (0, 1) and is orthogonal n1 + n2 + n3 + n4 + n5 = 20. Then the number of such distinct
to the circles (x – 1)2 + y2 = 16 and x2 + y2 = 1. Then arrangements (n1, n2, n3, n4, n5) is
(a) radius of S is 8
1
(b) radius of S is 7 d2 5
17. The value of 4 x3 1 x2 dx is
(c) centre of S is (– 7, 1) 0 dx 2
(d) centre of S is (– 8, 1)
10. Let a R and let f : R R be given by 18. Let a , b and c be three non-coplanar unit vectors such
f (x) = x5 – 5x + a. Then
(a) f (x) has three real roots if a > 4 that the angle between every pair of them is . If
(b) f (x) has only real root if a > 4 3
(c) f (x) has three real roots if a < – 4
(d) f (x) has three real roots if – 4 < a < 4 a b b c pa qb r c , where p, q and r are scalars,

SECTION - II
p2 2q 2 r2
This section contains 10 questions. Each question, when worked then the value of is
q2
out will result in one integer from 0 to 9 (both inclusive).
11. The slope of the tangent to the curve (y – x5)2 = x(1 + x2)2 at b
19. Let a, b, c be positive integers such that is an integer. If a,
the point (1, 3) is a
12. Let f : [0, 4 ] [0, ] be defined by f (x) = cos–1(cos x). The b, c are in geometric progression and the arithmetic mean of
number of points x 0, 4 satisfying the equation a2 a 14
a, b, c is b + 2, then the value of is
10 x a 1
f x is
10 20. Let n 2 be an integer. Take n distinct points on a circle and
join each pair of points by a line segment. Colour the line
13. The largest value of non-negative integer a for which
segment joining every pair of adjacent points by blue and
1 x the rest by red. If the number of red and blue line segments
ax sin x 1 a 1 x 1 are equal, then the value of n is
lim is
x 1 x sin x 1 1 4

14. Let f : R R and g : R R be respectively given by f (x)


= | x | + 1 and g(x) = x2 + 1. Define h : R R by

max f x ,g x if x 0,
h x
min f x ,g x if x 0.

The number of points at which h(x) is not differentiable is


EBD_7801
8-2014 JEE Advanced 2014 Solved Paper

PAPER - 2
1. The question paper consists of three parts (Physics, Chemistry and Mathematics). Each part consists of three
sections.
2. Section 1 contains 10 multiple choice questions. Each questions has four choices (a), (b), (c) and (d) out of
which ONE is correct.
3. Section 2 contains 3 paragraphs each describing theory, experiment and data etc. Six questions relate to three
paragraph with two questions on each paragraph. Each question pertaining to a particular passage should have
only one correct answer among the four given choices (a), (b), (c) and (d).
4. Section 3 contains 4 multiple choice questions. Each questions has two lists (List-1: P, Q, R and S; List-2: 1,
2, 3 and 4). The options for the correct match are provided as (a), (b), (c) and (d) out of which ONLY ONE is
correct.

PHYSICS (a) 60 0.15 (b) 135 0.56


(c) 60 0.25 (d) 135 0.23
SECTION - I
5. A wire, which passes through the hole in a small bead, is
This section contains 10 multiple choice questions. Each question bent in the form of quarter of a circle. The wire is fixed vertically
has four choices (a), (b), (c) and (d) out of which ONLY ONE on ground as shown in the figure. The bead is released from
option is correct. near the top of the wire and it slides along the wire without
friction. As the bead moves from A to B, the force it applies
1. If Cu is the wavelength of K X-ray line of copper (atomic
on the wire is
number 29) and Mo is the wavelength of the K X-ray line of
molybdenum (atomic number 42), then the ratio Cu/ Mo is A
close to
(a) 1.99 (b) 2.14
(c) 0.50 (d) 0.48
2. A metal surface is illuminated by light of two different
wavelengths 248 nm and 310 nm. The maximum speeds of the
photoelectrons corresponding to these wavelengths are 90°
B
u1 and u2, respectively. If the ratio u1 : u2 = 2 : 1 and hc = 1240
eV nm, the work function of the metal is nearly (a) always radially outwards
(a) 3.7 eV (b) 3.2 eV (b) always radially inwards
(c) 2.8 eV (d) 2.5 eV (c) radially outwards initially and radially inwards later
3. Parallel rays of light of intensity I = 912 Wm–2 are incident on (d) radially inwards initially and radially outwards later
a spherical black body kept in surroundings of temperature 1
300 K. Take Stefan-Boltzmann constant = 5.7 × 10–8 6. A planet of radius R radius of Earth has the same
10
Wm–2K–4 and assume that the energy exchange with the
R
surroundings is only through radiation. The final steady state mass density as Earth. Scientists dig a well of depth on it
temperature of the black body is close to 5
(a) 330 K (b) 660 K and lower a wire of the same length and a linear mass density
(c) 990 K (d) 1550 K 10–3 kg m–1 into it. If the wire is not touching anywhere, the
4. During an experiment with a metre bridge, the galvanometer force applied at the top of the wire by a person holding it in
shows a null point when the jockey is pressed at 40.0 cm place is (take the radius of Earth = 6 × 106 m and the
using a standard resistance of 90 , as shown in the figure. acceleration due to gravity on Earth is 10 ms–2)
The least count of the scale used in the metre bridge is 1mm. (a) 96 N (b) 108 N
The unknown resistance is (c) 120 N (d) 150 N
7. A glass capillary tube is of the shape of a truncated cone
with an apex angle so that its two ends have cross sections
of different radii. When dipped in water vertically, water rises
R 90 in it to a height h, where the radius of its cross section is b. If
the surface tension of water is S, its density is , and its
contact angle with glass is , the value of h will be (g is the
acceleration due to gravity)
40.0 cm
JEE Advanced 2014 Solved Paper 2014-9

(a) 1. 21 (b) 1. 30
(c) 1. 36 (d) 1. 42
10. A tennis ball is dropped on a horizontal smooth surface. It
bounces back to its original position after hitting the surface.
The force on the ball during the collision is proportional to
h
the length of compression of the ball. Which one of the
following sketches describes the variation of its kinetic energy
K with time t most appropriately? The figure are only
illustrative and not to the scale.
K

2S 2S (a)
(a) cos (b) cos
b g b g
2S 2S t
(c) cos /2 (d) cos /2 K
b g b g
8. Charges Q, 2Q and 4Q are uniformly distributed in three
dielectric solid spheres 1, 2 and 3 of radii R/2, R and 2R
respectively, as shown in figure. If magnitude of the electric
fields at point P at a distance R from the centre of sphere 1, 2 (b)
and 3 are E1, E2 and E3 respectively, then
P t
P K
R R
2Q
Q
(c)
R/2
Sphere 1 t
Sphere 2 K

R (d)
4Q
t

SECTION - II
2R
This section contains 3 paragraphs, each describing theory,
experiments, data etc. Six questions related to the three paragraphs
Sphere 3 with two questions on each paragraph. Each question has only
(a) E1 > E2 > E3 (b) E3 > E1 > E2 one correct answer among the four given options (a), (b), (c) and
(c) E2 > E1 > E3 (d) E3 > E2 > E1 (d).
9. A point source S is placed at the bottom of a transparent
block of height 10 mm and refractive index 2.72. It is immersed Paragraph for Questions 11 and 12
in a lower refractive index liquid as shown in the figure. It is The figure shows a circular loop of radius a with two long parallel
found that the light emerging from the block to the liquid wires (numbered 1 and 2) all in the plane of the paper. The distance
forms a circular bright spot of diameter 11.54 mm on the top of each wire from the centre of the loop is d. The loop and the wire
of the block. The refractive index of the liquid is are carrying the same current I.The current in the loop is in the
counterclockwise direction if seen from above.
Liquid Q S
d d

Block Wire 1 a Wire 2

S P R
EBD_7801
10-2014 JEE Advanced 2014 Solved Paper
11. When d a but wires are not touching the loop, it is found Paragraph for Questions 15 and 16
that the net magnetic field on the axis of the loop is zero at a A spray gun is shown in the figure where a piston pushes air out
height h above the loop. In that case of a nozzle. A thin tube of uniform cross section is connected to
(a) current in wire 1 and wire 2 in the direction PQ and RS, the nozzle. The other end of the tube is in a small liquid container.
As the piston pushes air through the nozzle, the liquid from the
respectively and h a
container rises into the nozzle and is sprayed out. For the spray
(b) current in wire 1 and wire 2 in the direction PQ and SR,
gun shown, the radii of the piston and the nozzle are 20 mm and
respectively and h a 1 mm respectively. The upper end of the container is open to the
(c) current in wire 1 and wire 2 in the direction PQ and SR, atmosphere.
respectively and h 1.2a
(d) current in wire 1 and wire 2 in the direction PQ and RS,
respectively and h 1.2a
12. Consider d >> a, and the loop is rotated about its diameter
parallel to the wires by 30° from the position shown in the 15. If the piston is pushed at a speed of 5 mms–1, the air comes
figure. If the currents in the wires are in the opposite out of the nozzle with a speed of
directions, the torque on the loop at its new position will be (a) 0.1 ms–1 (b) 1 ms–1
(assume that the net field due to the wires is constant over (c) 2 ms –1 (d) 8 ms–1
the loop). 16. If the density of air is a, and that of the liquid l, then for a
2 2 2 2 given piston speed the rate (volume per unit time) at which
0I a 0I a
the liquid is sprayed will be proportional to
(a) (b)
d 2d
2 2 2 2 a
3 0I a 3 0I a (a) (b)
(c) (d) a l
d 2d l
Paragraph for Questions 13 and 14
In the figure, a container is shown to have a movable (without l
(c) (d) l
friction) piston on top. The container and the piston are all made a
of perfectly insulated material allowing no heat transfer between
outside and inside the container. The container is divided into SECTION - III
two compartments by a rigid partition made of a thermally This section contains 4 questions, each having two matching
conducting material that allows slow transfer of heat. The lower lists. Choices for the correct combination of elements from List-I
compartment of the container is filled with 2 moles of an ideal and List-II are given as options (a), (b), (c) and (d), out of which
monatomic gas at 700 K and the upper compartment is filled with one is correct.
2 moles of an ideal diatomic gas at 400 K. The heat capacities per
17. A person in lift is holding a water jar, which has a small hole
3 5 at the lower end of its side. When the lift is at rest, the water
mole of an ideal monatomic gas are CV R , CP R , and
2 2 jet coming out of the hole hits the floor of the lift at a distance
5 7
those for an ideal diatomic gas are CV R , CP R. d of 1.2 m from the person. In the following, state of the lift’s
2 2
motion is given in List-I and the distance where the water jet
hits the floor of the lift is given in List-II. Match the statements
from List-I with those in List-IIand select the correct answer
using the code given below the lists.
List - I List - II
P. Lift is accelerating 1. d = 1.2 m
vertically up
Q. Lift is accelerating 2. d > 1.2 m
13. Consider the partition to be rigidly fixed so that it does not vertically down with
move. When equilibrium is achieved, the final temperature of an acceleration less
the gases will be than the gravitational
(a) 550 K (b) 525 K acceleration
(c) 513 K (d) 490 K R. Lift is moving vertically 3. d < 1.2 m
14. Now consider the partition to be free to move without friction up with constant speed
so that the pressure of gases in both compartments is the S. Lift is falling freely 4. No water leaks out of the
same. The total work done by the gases till the time they jar
achieve equilibrium will be Code:
(a) 250 R (b) 200 R (a) P-2, Q-3, R-2, S-4 (b) P-2, Q-3, R-1, S-4
(c) 100 R (d) – 100 R (c) P-1, Q-1, R-1, S-4 (d) P-2, Q-3, R-1, S-1
JEE Advanced 2014 Solved Paper 2014-11

18. Four charges Q1, Q2, Q3 and Q4 of same magnitude are fixed 20. A block of mass m1 = 1 kg another mass m2 = 2 kg, are placed
along the x axis at x = – 2a, – a, + a and + 2a, respectively. A together (see figure) on an inclined plane with angle of
positive charge q is placed on the positive y axis at a distance inclination . Various values of are given in List-I. The
b > 0. Four options of the signs of these charges are given in coefficient of friction between the block m1 and plane is
List-I. The direction of the forces on the charge q is given in always zero. The coefficient of static and dynamic friction
List-II. Match List-I with List-II and select the correct answer
between the block m2 and the plane are equal to = 0.3. In
using the code given below the lists.
List-II expressions for the friction on block m2 are given.
q(0, b)
Match the correct expression of the friction in List-II with the
angles given in List-I, and choose the correct option. The
acceleration due to gravity is denoted by g.
[Useful information: tan (5.5°) 0.1; tan (11.5°) 0.2;
Q1 Q2 Q3 Q4 tan (16.5°) 0.3]
(– 2a, 0) (– a, 0) (+ a, 0) (+ 2a, 0)
List - I List - II
P. Q1, Q2, Q3, Q4 all 1. + x m1
positive m2
Q. Q1, Q2 positive; 2. – x
Q3, Q4 negative
R. Q1, Q4 positive; 3. + y
Q2, Q3 negative
S. Q1, Q3 positive; 4. – y List-I List-II
Q2, Q4 negative P. = 5° 1. m2g sin
Code: Q. = 10° 2. (m1 + m2)g sin
(a) P-3, Q-1, R-4, S-2 (b) P-4, Q-2, R-3, S-1 R. = 15° 3. m2g cos
(c) P-3, Q-1, R-2, S-4 (d) P-4, Q-2, R-1, S-3 S. = 20° 4. (m1 + m2)g cos
19. Four combinations of two thin lenses are given in List-I. The Code:
radius of curvature of all curved surfaces is r and the refractive
index of all the lenses is 1.5. Match lens combinations in (a) P-1, Q-1, R-1, S-3 (b) P-2, Q-2, R-2, S-3
List-I with their focal length in List-II and select the correct (c) P-2, Q-2, R-2, S-4 (d) P-2, Q-2, R-3, S-3
answer using the code given below the lists.
List - I List - II CHEMISTRY
SECTION - I
P. 1. 2r This section contains 10 multiple choice questions. Each question
has four choices (a), (b), (c) and (d) out of which ONLY ONE
option is correct.

1. For the identification of -naphthol using dye test, it is


r necessary to use
Q. 2.
2 (a) Dichloromethane solution of -naphthol
(b) Acidic solution of -naphthol
(c) Neutral solution of -naphthol
(d) Alkaline solution of -naphthol
R. 3. –r 2. Assuming 2s-2p mixing is NOT operative, the paramagnetic
species among the following is
(a) Be2 (b) B2
(c) C2 (d) N2
3. For the elementary reaction M N, the rate of disappearance
S. 4. r of M increases by a factor of 8 upon doubling the
concentration of M. The order of the reaction with respect to
M is
Code: (a) 4 (b) 3
(a) P-1, Q-2, R-3, S-4 (b) P-2, Q-4, R-3, S-1 (c) 2 (d) 1
(c) P-4, Q-1, R-2, S-3 (d) P-2, Q-1, R-3, S-4
EBD_7801
12-2014 JEE Advanced 2014 Solved Paper
4. Isomers of hexane, based on their branching, can be divided 7. Hydrogen peroxide in its reaction with KIO4 and NH2OH
into three distinct classes as shown in the figure. respectively, is acting as a
(a) Reducing agent, oxidising agent
and (b) Reducing agent, reducing agent
and
(c) Oxidising agent, oxidising agent
(d) Oxidising agent, reducing agent
(I) (II) 8. The product formed in the reaction of SOCl2 with white
phosphorous is
(a) PCl3 (b) SO2Cl2
(c) SCl2 (d) POCl3
9. Under ambient conditions, the total number of gases released
(III) as products in the final step of the reaction scheme shown
below is
The correct order of their boiling point is Complete
(a) I > II > III (b) III > II > I Hydrolysis
(c) II > III > I (d) III > I > II XeF6 P + Other product
5. The acidic hydrolysis of ether (X) shown below is fastest
when –
OH /H2O

Slow disproportionation
Acid in OH–/H2O
OR OH + ROH

Products
(a) 0 (b) 1
(c) 2 (d) 3
(a) One phenyl group is replaced by a methyl group 10. For the process
(b) One phenyl group is replaced by a para-methoxyphenyl H2O(l) H2O(g)
group at T = 100°C and 1 atmosphere pressure, the correct choice is
(c) Two phenyl groups are replaced by two para- (a) Ssystem > 0 and Ssurroundings > 0
methoxyphenyl groups (b) Ssystem > 0 and Ssurroundings < 0
(d) No structural change is made to X (c) Ssystem < 0 and Ssurroundings > 0
6. The major product in the following reaction is (d) Ssystem < 0 and Ssurroundings < 0

O 1. CH3MgBr, dry ether, 0°C


SECTION - II
Cl 2. aq. acid This section contains 3 paragraphs, each describing theory,
CH3 experiments, data etc. 6 questions related to the three paragraphs
with two questions on each paragraph. Each question has only
O one correct answer among the four given options (a), (b), (c) and
(d).
(a) H3C
CH3 Paragraph for Questions 11 and 12
X and Y are two volatile liquids with molar weights of 10 g mol–1
OH and 40 g mol–1 respectively. Two cotton plugs, one soaked in X
and the other soaked in Y, are simultaneously placed at the ends
of a tube of length L = 24 cm, as shown in the figure. The tube is
(b) H2C CH3 filled with an inert gas at 1 atmosphere pressure and a temperature
CH3 of 300 K. Vapours of X and Y react to form a product which is first
observed at a distance d cm from the plug soaked in X. Take X and
Y to have equal molecular diameters and assume ideal behaviour
for the inert gas and the two vapours.
(c) L = 24 cm
CH2
O

CH3 Cotton wool Cotton wool


soaked in X d Initial formation soaked in Y
(d) of the product
O CH3
JEE Advanced 2014 Solved Paper 2014-13

11. The value of d in cm (shown in the figure), as estimated from (d) It gives a positive iodoform test and is a geometrical
Graham’s law, is isomer of X
(a) 8 (b) 12
Paragraph For Questions 15 and 16
(c) 16 (d) 20
An aqueous solution of metal ion M1 reacts separately with
12. The experimental value of d is found to be smaller than the
estimate obtained using Graham’s law. This is due to reagents Q and R in excess to give tetrahedral and square planar
(a) Larger mean free path for X as compared to that of Y complexes, respectively. An aqueous solution of another metal
ion M2 always forms tetrahedral complexes with these reagents.
(b) Larger mean free path for Y as compared to that of X
Aqueous solution of M2 on reaction with reagent S gives white
(c) Increased collision frequency of Y with the inert gas as
compared to that of X with the inert gas precipitate which dissolves in excess of S. The reactions are
(d) Increased collision frequency of X with the inert gas as summarized in the scheme given below:
compared to that of Y with the inert gas Scheme:
Q R
Tetrahedral M1 Square planar
Paragraph For Questions 13 and 14 Excess Excess
Schemes 1 and 2 describe sequential transformation of alkynes M
Q R
and N. Consider only the major products formed in each step for Tetrahedral M2 Tetrahedral
both the schemes. Excess Excess

1. NaNH 2 (excess)
S, stoichiometric amount
2. CH3CH 2I (1 equivalent)
H X Scheme-1
3. CH3I (1 equivalent) R
HO M 4. H2, Lindlar’s catalyst White precipitate Precipitate
Excess
dissolves
1. NaNH 2 (2 equivalent) 15. M1, Q and R, respectively are
OH (a) Zn2+, KCN and HCl (b) Ni2+, HCl and KCN
2. Br (c) Cd2+, KCN and HCl (d) Co2+, HCl and KCN
Y Scheme-2 16. Reagent S is
H +
3. H 3O, (mild) (a) K4[Fe(CN)6] (b) Na2HPO4
N 4. H2, Pd/C (c) K2CrO4 (d) KOH
5. CrO3
SECTION - III
13. The product X is
H3CO This section contains 4 questions, each having two matching
lists. Choice for the correct combination of elements from List-I
and List-II are given as options (a), (b), (c) and (d), out of which
(a) one is correct.
H H 17. Match each coordination compound in List-I with an
H appropriate pair of characteristics from List- II and select the
correct answer using the code given below the lists.
H {en = H2NCH2CH2NH2; atomic numbers : Ti = 22; Cr = 24;
(b)
Co = 27; Pt = 78}
H3CO
List-IList-II
CH3CH2O P. [Cr(NH3)4Cl2]Cl 1. Paramagnetic and exhibits
ionisation isomerism
(c) Q. [Ti(H2O)5Cl](NO3)2 2. Diamagnetic and exhibits
cis-trans isomerism
H H R. [Pt(en)(NH3)Cl]NO3 3. Paramagnetic and exhibits
H cis-trans isomerism
S. [Co(NH3)4(NO3)2]NO3 4. Diamagnetic and exhibits
(d) H ionisation isomerism
Code:
CH3CH2O
P Q R S
14. The correct statement with respect to product Y is
(a) It gives a positive Tollens test and is a functional isomer
of X (a) 4 2 3 1
(b) It gives a positive Tollens test and is a geometrical (b) 3 1 4 2
isomer of X (c) 2 1 3 4
(c) It gives a positive iodoform test and is a functional (d) 1 3 4 2
isomer of X
EBD_7801
14-2014 JEE Advanced 2014 Solved Paper
18. Match the orbital overlap figures shown in List-I with the description given in List-II and select the correct answer using the code
given below the lists.
List-I List-II

P. 1. p–d antibonding

Q. 2. d–d bonding

R. 3. p–d bonding

S. 4. d–d antibonding

Code:
P Q R S
(a) 2 1 3 4
(b) 4 3 1 2
(c) 2 3 1 4
(d) 4 1 3 2
19. Different possible thermal decomposition pathways for peroxyesters are shown below. Match each pathway from List-I with an
appropriate structure from List-II and select the correct answer using the code given below the lists.
P
R +R
– CO 2
O
O Q
R +R R + X + carbonyl compound
R O R
– CO 2
(Peroxyester)
R R + X + carbonyl compound
RCO2 + R O
– CO 2

S
RCO2 + R O R +RO
– CO 2

List-I List-II
O
O
P. Pathway 1. C 6H 5CH 2 O CH3

O
O
Q. Pathway 2. C6H5 O CH3

O
O CH3
R. Pathway 3. C6H5CH2 O CH3
CH2C6H5

O
O CH3
S. Pathway 4. C6H5 O CH3
C6H5
JEE Advanced 2014 Solved Paper 2014-15

Code:
P Q R S
(a) 1 3 4 2
(b) 2 4 3 1
(c) 4 1 2 3
(d) 3 2 1 4
20. Match the four starting materials (P, Q, R, S) given in List-I with the corresponding reaction schemes (I, II, III, IV) provided in
List-II and select the correct answer using the code given below the lists.
List - I List - II
P. H H 1. Scheme I

(i) KMnO4, HO , heat (ii) H , H2O
OH (iii) SOCl2 (iv) NH3
? ————————————— C7H6N2O3

Q. 2. Scheme II
OH (ii) Sn/HCl (ii) CH3COCl (iii) conc. H2SO4

(iv) HNO3 (v) dil. H2SO4, heat (vi) HO
NO2
? ————————————— C6H6N2O2

R. 3. Scheme III
(i) red hot iron, 873 K (ii) fuming HNO3, H2SO4, heat
(iii) H2S.NH3 (iv) NaNO2, H2SO4 (v) hydrolysis
NO2
? ————————————————— C6H5NO3

S. 4. Scheme IV
(i) conc. H2SO4, 60°C
CH3 (ii) conc. HNO3, conc. H2SO4 (iii) dil. H2SO4, heat
? ————————————————— C6H5NO4

Code:
P Q R S
(a) 1 4 2 3
(b) 3 1 4 2
(c) 3 4 2 1
(d) 4 1 3 2

MATHEMATICS 2. In a triangle the sum of two sides is x and the product of the
same sides is y. If x2 – c2 = y, where c is the third side of the
SECTION - I triangle, then the ratio of the in radius to the circum-radius of
the triangle is
This section contains 10 multiple choice questions. Each question
has four choices (a), (b), (c) and (d) out of which ONLY ONE 3y 3y
(a) (b)
option is correct. 2x x c 2c x c

1. Six cards and six envelopes are numbered 1, 2, 3, 4, 5, 6 and 3y


3y
cards are to be placed in envelopes so that each envelope (c) (d)
4x x c 4c x c
contains exactly one card and no card is placed in the
envelope bearing the same number and moreover the card 3. The common tangents to the circle x2 + y2 = 2 and the parabola
numbered 1 is always placed in envelope numbered 2. Then y2 = 8x touch the circle at the points P, Q and the parabola at
the number of ways it can be done is the points R, S. Then the area of the quadrilateral PQRS is
(a) 264 (b) 265 (a) 3 (b) 6
(c) 53 (d) 67 (c) 9 (d) 15
EBD_7801
16-2014 JEE Advanced 2014 Solved Paper
4. Three boys and two girls stand in a queue. The probability, 10. The function y = f (x) is the solution of the differential equation
that the number of boys ahead of every girl is at least one
more than the number of girls ahead of her, is dy xy x4 2x
2
in (– 1, 1) satisfying f (0) = 0. Then
dx x 1 1 x2
1 1
(a) (b)
2 3
3
2 3 2
(c) (d)
3 4 f x d x is
5. The quadratic equation p(x) = 0 with real coefficients has 3
purely imaginary roots. Then the equation p(p(x)) = 0 has 2
(a) one purely imaginary roots
(b) all real roots 3 3
(a) (b)
(c) two real and two purely imaginary roots 3 2 3 4
(d) neither real nor purely imaginary roots 3 3
(c) (d)
6. For x 0, , the equation sinx + 2sin 2x – sin 3x = 3 has 6 4 6 2
(a) infinitely many solutions SECTION - II
(b) three solutions
This section contains 3 paragraphs, each describing theory,
(c) one solution
experiments, data etc. 6 questions related to the three paragraphs
(d) no solution with two questions on each paragraph. Each question has only one
correct answer among the four given options (a), (b), (c) and (d).
2
17 Paragraph for Questions 11 and 12
7. The following integral 2cosec x dx is equal to
Box 1 contains three cards bearing numbers 1, 2, 3; box 2 contains
five cards bearing numbers 1, 2, 3, 4, 5; and box 3 contains seven
4
cards bearing numbers 1, 2, 3, 4, 5, 6, 7. A card is drawn from each
log 1 2
of the boxes. Let xi be number on the card drawn from the ith box,
u 16 i = 1, 2, 3.
(a) 2 eu e du
0 11. The probability that x1 + x2 + x3 is odd, is
29 53
log 1 2 (a) (b)
105 105
u 17
(b) eu e du
0 57 1
(c) (d)
105 2
log 1 2 12. The probability that x1, x2, x3 are in an arithmetic progression,
u 17
(c) eu e du is
0
9 10
(a) (b)
105 105
log 1 2
u 16
(d) 2 eu e du 11 7
0 (c) (d)
105 105
8. Coefficient of x11 in the expansion of (1 + x2)4(1 + x3)7 (1 +
Paragraph For Questions 13 and 14
x4)12 is
Let a, r, s, t be nonzero real numbers. Let P (at 2, 2at), Q,
(a) 1051 (b) 1106
R (ar2, 2ar) and S (as2, 2as) be distinct points on the parabola y2
(c) 1113 (d) 1120
= 4ax. Suppose that PQ is the focal chord and lines QR and PK are
9. Let f : [0, 2] R be a function which is continuous on [0, 2]
parallel, where K is the point (2a, 0)
and is differentiable on (0, 2) with f (0) = 1. Let F(x) =
13. The value of r is
x2
f t dt for x 0, 2 . If F (x) = f (x) for all x 0, 2 , 1 t2 1
(a) (b)
0 t t
then F(2) equals
1 t2 1
(a) e2 – 1 (b) e4 – 1 (c) (d)
t t
(c) e – 1 (d) e 4
JEE Advanced 2014 Solved Paper 2014-17

14. If st = 1, then the tangent at P and the normal at S to the P Q R S


parabola meet at a point whose ordinate is (a) 3 2 4 1
2 2 (b) 2 3 4 1
t2 1 a t2 1
(a) (b) (c) 3 2 1 4
2t 3 2t 3 (d) 2 3 1 4
2 2 18. List - I List - II
a t2 1 a t2 2
(c) (d) 1
t3 t3 P. Let y x cos 3cos x , 1. 1
Paragraph For Questions 15 and 16
3
1 h x 1,1 , x . Then
a a 1 2
Given that for each a (0, 1), lim t (1 t ) dt exists. Let
h 0 h
this limit be g(a). In addition, it is given that the function g(a) is 1 d2y x dy x
differentiable on (0, 1). x2 1 2
x
y x dx dx
1
15. The value of g is
2 equals
(a) (b) 2 Q. Let A1, A2, ..., An (n > 2) 2. 2
be the vertices of a regular
(c) (d)
2 4 polygon of n sides with its
1
16. The value of g is
2 centre at the origin. Let ak

(a) (b) be the position vector of the


2 point Ak, k = 1, 2, ....., n.
(c) (d) 0
2 n 1
SECTION - III If ak ak 1
k 1
This section contains 4 questions, each having two matching
lists. Choice for the correct combination of elements from List-I
and List-II are given as options (a), (b), (c) and (d), out of which n 1
= ak . ak 1 ,
one is correct. k 1

17. List - I List - II


then the minimum value of n is
P. The number of polynomials 1. 8
f (x) with non-negative R. If the normal from the 3. 8
integer coefficients of point P(h, 1) on the ellipse
degree 2, satisfying f (0) = 0
x2 y2
1 1 is perpendicular
and f x dx 1, is 6 3
0
Q. The number of points 2. 2 to the line x + y = 8, then
the value of h is
in the interval 13, 13
S. Number of positive 4. 9
at which f (x) = sin(x2) + cos(x2) solutions satisfying the
attains its maximum value, is
2 1 1
3x2 equation tan
R. dx equals 3. 4 2x 1
2 1 ex
1 1 1 1 2
+ tan = tan is
2
1 x 4x 1 x2
cos 2 x log dx
1
1 x P Q R S
2 (a) 4 3 2 1
S. 4. 0
1 (b) 2 4 3 1
2
1 x (c) 4 3 1 2
cos 2 x log dx
1 x (d) 2 4 1 3
0
EBD_7801
18-2014 JEE Advanced 2014 Solved Paper
19. Let f1 : R R, f 2 : 0, R, f3 : R R and 2k 2k
20. Let zk cos i sin ; k = 1, 2, ...., 9.
10 10
x if x 0,
f4 : R 0, be defined by f1 x List-I List-II
e x if x 0; P. For each zk there exists 1. True
as zj such that zk. zj = 1
sin x if x 0,
f2(x) = x2; f3 x Q. There exists a 2. False
x if x 0;
k 1, 2,...,9 such
f 2 f1 x if x 0, that z1.z = zk has no
and f 4 x solution z in the set
f 2 f1 x 1 if x 0.
of complex numbers
List-I List-II
1 z1 1 z2 .... 1 z9
P. f4 is 1. Onto but not one-one R. 3. 1
Q. f3 is 2. Neither continuous nor 10
one-one equals
R. f2of1 is 3. Differentiable but not 9
one-one 2k
S. 1 cos equals 4. 2
S. f2 is 4. Continuous and one- k 1
10
one P Q R S
P Q R S (a) 1 2 4 3
(a) 3 1 4 2 (b) 2 1 3 4
(b) 1 3 4 2 (c) 1 2 3 4
(c) 3 1 2 4 (d) 2 1 4 3
(d) 1 3 2 4
Telegram @unacademyplusdiscounts

Join Us on Telegram for More Such Books

https://telegram.me/unacademyplusdiscounts

Join us from the above link or search ''unacademyplusdiscounts'' in Telegram


JEE Advanced 2014 Solved Paper 2014-19

SOLUTIONS
Paper - 1
PHYSICS V2
2. (b, d) H 4 ...(i)
R
r0
1. (c) 1 Q 4 l
+Q E1 = where R
4 0 r0
2
d2
r0 1 When resistances are connected in series
+ 2
+ E2 = 4 l R R
+ 4 r0
0 Total resistance = R1 + R2 = 2 2
=2
4 d 4 2
r0
++ E3 = V2
++ H t2 ...(ii)
++
0
R/2
From (i) and (ii) t2 = 2 min. Therefore (b) is correct.
When resistance are connected in parallel
E1 = E2 (Given)
1 Q 1 2 R1R 2 R12 R/4 R
Total resistance =
R1 R 2 2R1 2 8
4 0 r02 4 0 r0
Q = 2 r0 ...(1) V2
H t2 ...(iii)
E2 = E3 (Given) R /8
From (i) and (iii) t2 = 0.5 min
1 2
r0 (d) is correct
4 0 r0 2 0 3. (a, c)
(b) is incorrect There will be no effect of the transparent thin film of
uniform thickness and refractive index n 1 = 1.4
E1 = E3 (Given)
n1 n2 n2 n1
1 Q Therefore, [For case (i)]
Q 2 r02 u v R
4 2 2
0 r0 0
1 1.5 1.5 1
(a) is incorrect f1 = 3R
f1 R
1 4Q (a) is a correct option
Now E1 (r0/2) =
4 0 r02 n1 n2 n2 n1
Again applying [For case (ii)]
u v R
1 4 2 r0 1 8
1.5 1 1 1.5
4 0 r02 4 0 r0 f2 2R
f2 R
(c) is a correct option.
1 4 1 8
and 2E2 (r0/2) = 2
4 0 r0 4 0 r0 v RT 1
4. (d) Here, v = × 4l
4l 3 4l
(c) is correct M 10
v = 336.7 m/s to 346.5 m/s
1 2 1 4 For monatomic gas =1.67
E2 (r0/2) =
4 0 r0 /2 4 0 r0 0 r0 RT 10
v 100 RT
3 M
4 2 2 M 10
4E3 (r0/2) =
2 0 0 0 r0
10 10
(d) is incorrect. = 167RT 640
M M
EBD_7801
20-2014 JEE Advanced 2014 Solved Paper
O P
10 7 6. (a, b, d) i
For Neon M = 20 and
20 10
7 V1 R1
v 640 = 448 ms–1
10 R2
(a) is incorrect N Q
10 17
For Argon M = 36, V2
36 32
R3 i
17
v = 640 = 340 ms–1 M A
32
Applying KVL in MNOPQAM
(d) is the correct option.
For diatomic gas = 1.4 V1 iR1 V2 iR 3 0

10 10 V1 V2
v 140RT 590 i ... (i)
M M R1 R 3
Applying KVL in NOPQN
10 9
For Oxygen V
32 16 v1 iR1 0 i ...(ii)
R1
9
v 590 = 331.87 ms–1
16 V1 V1 V2
(c) is incorrect From (i) & (ii) R R1 R 3
1
10 3 V1R1 + V1R3 = V1R1 + V2R1
For Nitrogen V1R3 = V2R1
28 5
If V1 = V2 then R1 = R3 = R2 (a) is correct option.
3 If V1 = V2 then R1 = R3 = 2R2 (b) is correct option.
v 590 = 354 ms–1
5 (R2 can have any value as there is no current flowing
(b) is incorrect through d)
5. (c, d) When 1 0 and 2 0 If V1 = 2V2 then 2R3 = R1 (c) is incorrect option.
If 2V1 = V2 then R3 = 2R1 = R2 (d) is correct option.
7. (a, b, c)
N1 f1 = 1N1 D
We know that
d
1 As 2 > 1 2> 1 (a) is correct option.
Therefore m1 > m2 (b) is correct option.
N2
mg 2D 2 5 1 1 D
2 As 3
d 2 d
P f2 = 2N2 3 × 600 = 4.5 × 400 (c) is correct option.
N1 = 2N2 [... horizontal equilibrium]
The angular width = (d) is incorrect option.
mg = N2 + N1 [... vertical equilibrium] d
Solving the above equation we get 8. (c, d) I = cos 500 t
mg I
N2
1 1 2
(c) is the correct option. 1
When 1 = 0 7
Taking torque about P we get 6
t
l
mg cos N1 l sin 2
2
mg –1
N1 tan
2
(d) is correct
JEE Advanced 2014 Solved Paper 2014-21

B D
10. (a, d) C1
A
K
50 V A/3 Q1 E1
C = 20 F

R = 10 E2
2A/3 Q2
7
Till t , the charge will be maximum at C2 = C – C1
6 2
d
/2 /2
sin 500t
Q cos 500t dt = This is a combination of two capacitors in parallel.
500 0 Therefore
0
C = C1 + C2 C2 = C – C1
1 1
= sin 500 C kA 2A
500 2 500 500 where C1 and C C1
3 0d 3 0d
(a) is incorrect
C C1 2
7
From the graph it is clear that just before t , the C1 k
6
current is in anticlockwise direction. C 2
1
(b) is incorrect C1 k
7
At t , the charge on the upper plate of capacitor C 2
6 1
C1 k
is

7
C 2
1
6 C1 k
1 7
cos 500t dt sin 500
500 6 500 (d) is a correct option.
0
kA
1 1 Now, Q1 = C1V = ×V
10 C3 3 0d
500 2
2A
Now applying KVL (when A is just connected to D) and Q2 = (C – C1)V = V
3 0d
3
10
50 i 10 0 i = 10 A Q1 k
6
20 10 (c) is incorrect
Q2 2
(c) is the correct option.
The maximum charge on C is Q = CV = 20 × 10–6 × 50 Also V = E × d
=10–3C V
Therefore, the total charge flown = 2 × 10 –3 C E E1 E2 (a) is a correct option
d
(d) is the correct option.
9. (a, c, d) I Ig V
11. (5) S R
Clearly in the given situation a displacement node is Ig Ig
present at x = 0 and a displacement antinode is present
at x = 3 m. Therefore, y = 0 at x = 0 and y A at 1.5 0.006 2n 30
4990
x = 3 m. 0.006 249 0.006
n 5
The velocity v = 100 ms–1. x y
k 12. (3) d S fz
a, c and d are the correct options which satisfy the
above conditions. M 0 L1T 0 M x L 3x M yT yT z
EBD_7801
22-2014 JEE Advanced 2014 Solved Paper

M 0 L1T 0 M x y L 3xT y z µ0 1 1
x + y = 0, – 3x = 1 2I
R1 4 X1 X0 X1
1 1 R2 µ0 1 1
x and y = 2I
3 3 4 X1 X0 X1
n=3
X0 X1 X1 X0
FL = X
13. (4) Y 0 X1 X1 X 0 2 X1
a l
Here F, a and L are accurately known. X0
R1 X1 3
Y L 1.0 10 5 3
100 100 = 100 = 4% R2 X0 3 2
Y l 25 10 5 2
X1
14. (4) By conservation of angular momentum
2 (mvr) = I 19. (5) vA
1
2 × 0.05 × 9 × 0.25 = × 0.45 × (0.5)2 ×
2 vB cos 30°
vB
= 4 rad s–1 vB sin 30°
30°
1 30° 60°
15. (2) 3 F r I
2
Here
1 1 vA = vB cos 30°
3 0.5 0.5 = 1.5 0.5 0.5
2 2
3
= 2 rad s–1 100 3 vB
2
= 0+ t = 0 + 2 × 1 = 2 rad s–1
vB = 200 ms–1
16. (5) Work done = Increase in potential energy
+ gain in kinetic energy displacement
Time
F × d = mgh + gain in K.E. velocity
18 × 5 = 1 × 10 × 4 + gain in K.E.
500 500
Gain in K.E. = 50 J = 10n t0 5 sec
vB sin 30 200 sin 30
n=5
17. (2) Applying first law of thermodynamics to path iaf 20. (8)
Qiaf = Uiaf + Wiaf 0.3 ms –1 0.2 ms –1 a = 2 ms –2
A x
500 = Uiaf + 200 Uiaf = 300 J B
Now,
Qibf = Uibf + Wib + Wbf 4m
= 300 + 50 + 100 For ball A
Qib + Qbf = 450 J ...(1) u1 0.3 ms 1,
a1 2ms 2 , s = x, t = t
1 1
Also Qib U ib Wib
1 2
Qib = 100 + 50 = 150 J ...(2) s1 u1t1 a1t1
2
Qbf 300 x = 0.3t – t2 ...(1)
From (1) & (2) 2
Qib 150 For ball B
u2 = 0.2 ms–1, a2 = 2ms–2, s2 = 4 – x, t2 = t
mv 1
18. (3) R a 2 t 22
qB s2 = u2t2 +
2
4 – x = 0.2 t + t2 ...(2)
R1 B2
R2 B1 [... m, q, v are the same] From (1) and (2) t = 8 sec
JEE Advanced 2014 Solved Paper 2014-23

CHEMISTRY OH
1. (b, c, d) Cu2S + 2Cu2O 6Cu + SO2 c
a
Cu2S + 2CuO 4Cu + SO2
Cu2S + CuSO4 3Cu + 2SO2 C(CH3)3
b
2. (a, b) Salt bridge is introduced to keep the solutions of two
electrodes separate, so that the ions in electrodes do 6. (a, b, c) Na + NH3 (excess) Dilute solution of Na in liq.
not mix freely with each other. Salt bridge stops the NH3 Paramagnetic
diffusion of ions from one electrode to another. It is K + O2 (excess) KO2 (O2– is paramagnetic)
not necessary for occurrence of cell reaction, as we Cu + HNO3 (dil.) Cu(NO3)2 + NO
know that there is no salt bridge in lead accumulator, (NO is paramagnetic)
but still reactions take place. 2-Ethylanthraquinol + O2
3. (a, b, d) In ice, water molecules are excessively H-bonded 2-Ethylanthraquinone + H2O2
giving a cage-like structure which is lighter than water.
(H2O2 is diamagnetic)
Primary amines are more basic than tertiary amine,
because the protonated 1° amines are extensively H- 7. (a, b, d) Balanced chemical equation is
bonded and hence more stable than the corresponding 1 0
protonated 3° amines. 6I ClO3 6H 2SO 4 Cl 6HSO4 3I 2 3H 2O

H H 8. (a)
R — NH 2 R — NH3 ; R 3 N R 3 NH — NH 2 group is acetylated by acetic anhydride in
methylene chloride (solvent). Note that —CONH2
H OH 2 group does not undergo acetylation because here lone
+ pair of electrons is delocalised.
R N+ H OH 2 > R3NH OH2
H OH 2 COCH3
CH 2Cl2
More stable Less stable CH2 NH2 + (CH3CO)2O CH2 NH + CH3COOH
Acetic acid undergoes dimerisation in benzene.
O O–
O H O +
H3C C C CH3 C NH2 C NH2
O H O
4. (a, c, d) Isomeric alcohols with molecular formula C4H10O 9. (a, b, c) Since the vessel is thermally insulated, q = 0
are Further since, Pext = 0, so w = 0, hence U = 0
Since T = 0, T2 = T1, and P2V2 = P1V1
However, the process is adiabatic irreversible, so we
OH
can’t apply P2V2 = P1V1 .
n-Butanol OH 10. (b, d)H3BO3 does not undergo self ionization. However, it
(Buta-1-ol)
Butan-2-ol acts as a weak acid in water (hence it is a weak
electrolyte in water).
OH
OH H3 BO3 H 2O B(OH)4 H
lsobutyl alcohol tert-Butanol
(2-methylpropan-1-ol) (2-methylpropan-2-ol)
Addition of cis-diols (e.g., ethylene glycol) to aqueous
solution of orthoboric acid leads to complex formation,
thus acidity of aqueous solution of orthoboric acid is
5. (a, b, c) —OH group is strongly activating and o, p-directing
increased.
due to +M effect. Thus positions a, b and c are the

sites for attack by an electrophile. However, sites b CH2 — OH CH2 — O O — CH2

and c are not preferred by bulky electrophile due to B(OH)4 + 2 B + 4H2O
CH2 — OH CH2 — O O — CH2
steric crowding. Thus more bulky electrophile (like I2)
can attack only site a, which is least sterically hindered, It arranges to planner sheets due to H-bonding.
a bit smaller electrophile (Br2) can attack at sites a and 11. (1) On hydrolysis, the given peptide gives only one
also b (relatively less sterically hindered site) and the naturally occurring amino acid (glycine).
smallest electrophile (Cl2) can attack all the three sites,
viz., a, b and c (most sterically hindered site).
12. (2) MX2 M2 2X
1 2
EBD_7801
24-2014 JEE Advanced 2014 Solved Paper

obser. depression in f.pt of aq. solution –


i=
depression of f.p.t. in absence of ionic dissociation
F F
BrF4– = Br Square planar (sp3d2)
1 2 F F
= 1 2 = 1 + 2 × 0.5 = 2
1
13. (4) R = NA × k
= 6.023 × 1023 × 1.380 × 10–23 H3N NH3
2+

= 8.312 which has 4 significant figures [Cu(NH3)4]2+ = Cu


14. (8) H3N NH3
wt. of solute in 1 L of solution 1000 Square planar (dsp2)
Molality = wt. of solvent in 1 L of solution mol. wt. of solute 2–
Cl
Calculation of wt. of solvent Cl
1 mL of solvent = 0.4 g [FeCl4]2– = Fe Tetrahedral (sp3)
1000 mL of solvent = 400 g Cl Cl
Calculation of wt. of solute 2–
Cl
1000 mL of solution contain = 3.2 × 80 g solute = 256 g
Cl
256 1000 [CoCl4]2– = Co Tetrahedral (sp3)
Molality = 8 Cl
400 80 Cl

ml 1 means ml can be +1 and –1. 2–


15. (6) Cl Cl
So, for n = 4, six orbitals are possible and each has 1 [PtCl4]2– = Pt Square planar (dsp2)
Cl Cl
1
. So total number of electrons = 6.
electron with s = 18. (5) General molecular formula for ketones is CnH2nO
2
CnH2nO = 100 or 12n + 2n + 16 = 100, n = 6
16. (7) K2Cr2O7 + KI + H2SO4 K2SO4 + Cr2(SO4)3 Possible isomeric ketones with 6 carbon atoms are
+ I2 + H2O O O
2CuSO4 + KI 2CuI + I2 + 2K2SO4
H2O2 + 2KI 2KOH + I2 CH3CH2CH2CH2—C—CH3 CH3CH2CH2—C—CH2CH3
Cl2 + 2KI 2KCl + I2 (I) (II)
O3 + H2O + 2KI 2KOH + O2 + I2 O
FeCl3 + 2KI 2KCl + FeCl2 + I2
HNO3 + KI KNO3 + I2 + NO CH3 O CH3—CH—CH2—C—CH3
*
CH3CH2CH—C—CH3 CH3
(III) (IV)
F F
17. (4) XeF4 = Xe Square planar (sp3d2) CH3 O
F F CH3 O CH3—C — C—CH3
CH3—CH—C—CH2CH3 CH3
F (V) (VI)
F
Note that only isomer III has a chiral carbon so on
SF4 = S See-saw (sp3d)
F reduction with NaBH4 it will give diastereomeric
F alcohols, while all other five isomers will give racemic
mixture.
F
F 19. (3) Cl CH3
SiF4 = Si Tetrahedral (sp3)
Br CH3 Cl Br
F F
Br Cl Br Cl

F
F CH3 CH3
BF4– = B Tetrahedral (sp3)
Following three conformers (with 0) are possible
F F
JEE Advanced 2014 Solved Paper 2014-25

2. (a, d) Let f and g be maximum at c1 and c2 respectively,


CH3 CH3 CH3
Br c1 , c2 0,1
Cl CH3 Br Cl CH3
Then, f (c1) = g(c2)
Br Cl Let h(x) = f (x) – g(x)
Br Cl Br Cl
Then, h(c1) = f (c1) – g(c1) > 0
CH3 Cl Br and h(c2) = f (c2) – g(c2) < 0
h(x) = 0 has atleast one root in (c1, c2)
20. (7) All except MnS (buff coloured) and SnS2 (yellow) are
C (c1, c2)
black in colour.
i.e. for
f (c) = g(c)
MATHEMATICS which gives A and D are correct options.
1. (a,c) g(x) may be discontinuous at x = a or x = b. a b
Let us check the continuity of g(x) at x = a and x = b 3. (c,d) Let M where a, b, c are integers.
b c
lim g x 0
x a a b
M is invertible if 0 ac b2
x a
b c
lim g x lim f t dt f t dt 0
x a x a a a b
a
Then b c
a=b=c ac = b2.
a
(a) is not correct.
g a f t dt = 0
a If b c a b b=a=c
g (x) is continuous at x = a ac = b2
x b (b) is not correct.
Also lim g x lim f t dt f t dt a 0
x b– x b– a a If M = , then M ac 0
0 c
b M is invertible.
lim g x f t dt (c) is correct
x b a
2
As ac integer ac b2
b
g b f t dt (d) is correct.
a
g(x) is continuous at x = b 4. (a, b, c) x y z 2
Hence g(x) is continuous x R

0, x a Angle between each pair is


3
Now g (x) = f x , a x b
0, x b
a x y z

g a 0 and g a f a

= x z y x y z
g b– f b and g b 0

As f (a), f (b) 1, f (a), f(b) 0


= 2. 2 cos y 2. 2 cos z
3 3
Hence g a g a and g b g b

g is not differentiable at a and b. = y z


EBD_7801
26-2014 JEE Advanced 2014 Solved Paper
–( + )+ – =0
b y z x =0
dr’s of PQ are 0, 0, – 1
and dr’s of PR are , , + 1
= y.x z y. z x ( – l) ( + 1) = 0 = 1 or – 1
QPR 90
But for = 1, we get point Q itself
2. 2.cos z 2. 2.cos x we take = – 1
= 3 3 6. (a,b) Given MN = NM, M N2 and M 2 = N 4.
Then M 2 = N 4 (M + N 2) (M – N 2) = 0
= z x (i) M + N 2 = 0 and M – N 2 0
(ii) |M + N 2| = 0 and |M – N 2| = 0
In each case |M + N 2| = 0
Now b . z z.z x. z
|M 2 + MN 2| = |M | |M + N 2| = 0
= (2 – 1) = (a) is correct and (c) is not correct.
Also we know if |A| = 0, then there can be many matrices
b b.z z x is correct U, such that AU = 0
(M 2 + MN 2)U = 0 will be true for many values of U.
Hence (b) is correct.
Also a . y y. y z.y = (2 – 1) = Again if AX = 0 and |A| = 0, then X can be non-zero.
(d) is not correct.
a a.y y z is also correct x 1
t dt
t
7. (a, c, d) f x e
t
1/ x
a.b y. z y.x z.z z.x
1
x 1 1
x x 2 x
e x x e x
= (1 – 1 – 2 + 1) = – =– a. y b.z f x e =
x
x x2
(c) is correct.
For x 1, , f x 0
a.y z y z y a
f is monotonically increasing on 1,
(d) is not correct. (a) is correct.
5. (c) Lines are x = y, z = 1
For x 0,1 , f ( x ) 0
x 0 y 0 z 1
or ...(1)
1 1 0 (b) is not correct
and y = – x, z = – 1
x 1 dt 1/ x 1 dt
1 t t
x 0 y 0 z 1 t t t t
or ...(2) f x f e e =0
1 1 0 x
1/ x x
Let Q ( , , 1) and R (– , , – 1)
Direction ratios of PQ are – , – , – 1 (c) is not correct.
and direction ratios of PR are + , – , + 1
... PQ is perpendicular to line (1) 1
Replacing x by 2x in f (x) + f =0
x
P
x
We get f (2x) + f 2 =0
Q R
or f (2x) = – f (2–x)
f (2x) is an odd function.
(d) is correct.
1 2
JEE Advanced 2014 Solved Paper 2014-27

Required circle is
8. (a, b, c) f : , R x 2 + y 2 + 14x – 2y + 1 = 0
2 2
With centre (– 7, 1) and radius = 7
f (x) = [log(sec x + tan x)]3 (b) and (c) are correct options.
f (– x) = [log(sec x – tan x)]3
10. (b,d) f x x5 5 x a
3
sec x tan x sec x tan x
= log f x 0 x5 5 x a 0
sec x tan x
a = 5x – x5 = g(x)
3
1 3 g(x) = 0 when x = 0, 51/ 4 51/ 4
= log log sec x tan x
sec x tan x
and g x 0 x = 1, – 1
3 Also g (– 1) = – 4 and g(1) = 4
= – log sec x tan x f x
graph of g(x) will be as shown below.
f is an odd function. From graph
(a) is correct and (d) is not correct.
Also

2 sec x tan x sec 2 x 4


f x 3 log sec x tan x .
sec x tan x

2
= 3sec x log sec x tan x 0 x ,
2 2
1/4
1/4 –1 O 5
–5 1
f is increasing on ,
2 2
We know that strictly increasing function is one one.
–4
f is one one
(b) is correct.
3
Also lim log sec x tan x if a 4, 4
x
2
then g(x) = a or
f (x) = 0 has 3 real roots
3 If a > 4 or a < – 4
and lim log sec x tan x
then f(x) = 0 has only one real root.
x
2 (b) and (d) are the correct options.
Range of f = (– , ) = R 2 2
f is an onto function. 11. (8) y x5 x 1 x2
(c) is correct.
9. (b, c) Let the equation of circle be dy 2
2 y x5 5 x4 1 x2 2 x 1 x 2 .2 x
x2 + y2 + 2gx + 2 f y + c = 0 dx
It passes through (0, 1) At point (1, 3)
1 + 2f + c = 0 ...(i)
This circle is orthogonal to (x – 1) 2 + y 2 = 16 dy 2
2 3 1 5 1 1 2 1 1 .2
i.e. x2 + y2 – 2x – 15 = 0 dx
and x2 + y2 – 1 = 0
dy
We should have 8
dx
2g (– 1) + 2f (0) = c – 15
or 2g + c – 15 = 0 ...(ii) 12. (3) We have f : 0, 4 0,
and 2g(0) + 2f (0) = c – 1
1
or c = 1 ...(iii) f x cos cos x
Solving (i), (ii) and (iii), we get
c = 1, g = 7, f = – 1 10 x x
and g x 1
10 10
EBD_7801
28-2014 JEE Advanced 2014 Solved Paper
15. (6) Let the point P be (x, y)
x y x y
Then d1 P and d2 P
1 2 2
For P lying in first quadrant x > 0, y > 0.
Also 2 d1(P) + d2(P) 4

x y x y
2 3 10 4 2 4
2 2
The graph of y = f (x) and y = g (x) are as follows.
Clearly f(x) = g(x) has 3 solutions. x y x y
If x > y, then 2 4
2
1 x
ax sin x 1 a 1 x 1 or 2 x 2 2
13. (2) lim
x 1 x sin x 1 1 4 If x < y, then
y x x y
1 x 2 4
a 1 x sin x 1 2
lim
x 1 x 1 sin x 1
or 2 y 2 2
The required region is the shaded region in the figure
1 x given below.
sin( x 1)
a
lim x 1
x 1 sin x 1 y
1
x 1 y=x

2
a 1 1 y=2 2
2 4
a = 0 or 2
y= 2
Largest value of a is 2.
x 1, x 0
14. (3) f x x 1
x 1, x 0
g(x) = x2 + 1 x
O
x= 2 x=2 2
y = g(x)
2 2
Required area = 2 2 2 = 8 – 2 = 6 sq units.
16. (7) ...n1, n2, n3, n4 and n5 are positive integers such that
y = f(x) n1 < n2 < n3 < n4 < n5
Then for n1 + n2 + n3 + n4 + n5 = 20
If n1, n2 , n3 , n4 take minimum values 1, 2, 3, 4
respectively then n5 will be maximum 10.
Corresponding to n5 = 10, there is only one solution
n1 = 1, n2 = 2, n3 = 3, n4 = 4.
Corresponding to n5 = 9, we can have, only solution
n1 = 1, n2 = 2, n3 = 3, n4 = 5 i.e., one solution
Corresponding to n5 = 8, we can have, only solution
n1 = 1, n2 = 2, n3 = 3, n4 = 6
From graph there are 3 points at which h(x) is not
or n1 = 1, n2 = 2, n3 = 4, n4 = 4
differentiable.
i.e., 2 solution
JEE Advanced 2014 Solved Paper 2014-29

For n5 = 7, we can have 19. (4) ... a, b, c are in G.P

n1 = 1, n2 = 1, n3 = 4, n4 = 6 b = ar and c = ar2
or n1 = 1, n2 = 3, n3 = 4, n4 = 5 i.e. 2 solutions
b
For n5 = 6, we can have Also is an integer
a
n1 = 2, n2 = 3, n3 = 4, n4 = 5
r is an integer
i.e., one solution ... A.M. of a, b, c is b + 2
Thus there can be 7 solutions.
a b c
1
d2 b 2
3 2 5 3
17. (2) 4x 1 x dx
0 dx 2
a ar ar 2 3ar 6
1 1
3 d
1 x2
5 d
1 x2
5
.12 x 2 dx
a r 2 2r 1 6
= 4x
dx 0
dx
0
2
a r 1 6
2 2 51 1 5 ... a and
r are integers
= 12 x 1 x 1 x2 .24 xdx
0 0
The only possible values of a and r can be 6 and 2
respectively.
1
= 12 1 x
2 5
. 2 x dx a2 a 14 36 6 14 28
Then = =4
0 a 1 6 1 7
20. (5) Number of adjacent lines = n
6 1 Number of non adjacent lines = nC2 – n
1 x2 1
= 12 12 0 2
nC – n = n
n n 1
6 6 2 – 2n = 0
2
0
n 2 5n 0 n = 0 or 5
1
18. (4) a.b b.c c.a cos But n 2 n=5
3 2
S(4) : 1, Z1, Z2, ... Z9 are 10th roots of unity.
Z10 – 1 = 0
Given p a q b r c a b b c From equation 1 + Z1 + Z2 + .... + Z9 = 0
Re (1) + Re (Z1) + Re (Z2) + .... + Re(Z9) = 0
Taking its dot product with a , b , c , we get
Re (Z1) + Re (Z2) + ..... Re(Z9) = – 1

1 1 9
2k
p q r a b c ...(1) cos 1
2 2 K 1 10

1 1 9
2k
p q r 0 ...(2) 1 cos 2
2 2 10
K 1
1 1 Hence (c) is the correct option.
p q r a b c ...(3)
2 2
From (1) and (3), p = r Using (2) q = – p

p2 2q 2 r2 p2 2 p2 p2
4
q2 p2
EBD_7801
30-2014 JEE Advanced 2014 Solved Paper

Paper - 2
PHYSICS R l 100 l

2
R l 100 l
2 2
Cu Z Mo 1 42 1 41
1. (b) 2.14 R 0.1 0.1
2 29 1 28
Mo ZCu 1 60 40 60
R 0.25
Z b here b 1 Therefore, R 60 0.25
2
Z 1 5. (d) h N
1 2
Z 1 v
mg sin
hC 1
2. (a) W mu12 mg cos mg
1 2

hC 1
and W mu22
2 2
Dividing the above two equations, we get As the bead is moving in the circular path
hC
W mv 2
mg cos N
1 u12 R
hC u22
W mv 2
2 N mg cos ...(1)
R
1240 1 2
W By energy conservation, mv mg R R cos
248 4 2
1240 1
W v2
310 2 g 1 cos ..(2)
R
1240 4 1240 From (1) and (2)
W 4W
248 310 N mg cos m 2 g 2 g cos
W = 3.7 eV
3. (a) In steady state N mg cos 2mg 2mg cos
Energy lost = Energy gained N 3mg cos 2mg
T 4 T04 4 R2 I R2 N mg 3cos 2
Clearly N is positive (acts radially outwards) when
4
5.7 10 8 T 4 300 4 912 2
cos
T = 330 K 3
4. (c) In case of a meter bridge 2
Similarly, N acts radially inwards if cos
R X 3
Here X = 90 , l = 40.0 cm
l 100 l
6. (b) Rp
Xl Rp
R 10
100 l M 5
For finding the value of R
Rp
90 40
R 60
60
For finding the value of R
JEE Advanced 2014 Solved Paper 2014-31

Re 1 2Q 1 Q/2
Rp 6 105 m E2 · ; E3 ·
2
10 4 0 R 4 0 R2
The mass of the wire = 10–3 × 1.2 × 105 = 120 kg Clearly E2 > E1 > E3
Let gpM be the acceleration due to gravity at point M where Q/2 is the charge enclosed in a sphere of radius
which is the mid point of the wire and is at a depth of R concentric with the given sphere.
Rp
.
10 4Q Q'
Let gp be the acceleration due to gravity at the surface 4 3 4 3
2R R
of the planet. 3 3
4 9. (c)
gp GRP n2
3
11.54 mm
4 2
ge = GRE
3 P
gp Rp 1 10 mm
ge RE 10 n1

10
gp 1 ms –2 Applying Snell’s law at point P
10
n1 sin = n2sin 90°
R p /10 11.54 / 2
and g pM gp 1 1 1 0.1 0.9 ms 2 n2 2.72
Rp 2
2 11.54
10
Force = mass of wire × gpM = 120 × 0.9 = 108 N 2
n2 = 1.36
1 2 1 2 1 2
7. (d) 10. (b) K.E. = mv = m u at = m 0 gt
C 2 2 2
R
2 1
K .E mgt 2 K .E t 2 ...(1)
+ 2
M
b 2 P First the kinetic energy will increase as per eq (1). As
the balls touches the ground it starts deforming and
loses its K.E. (K.E. converting into elastic potential
energy). When the deformation is maximum, K.E. = 0.
h The ball then again regain its shape when its elastic
potential energy changes into K.E. As the ball moves
up it loses K.E. and gain gravitational potential energy.
These characteristics are according to graph B.

11. (c) B
b
In PCM cos ...(1) B2 sin
2 R B1 sin

2S B2 B1
Also P0 h g P0
R (90º – )

2S 2S B1cos + B2 cos
h cos 2 2 2
R g b g a h
h
1 Q
8. (c) E1 · ;
4 0 R2 Wire 1 a a Wire 2
EBD_7801
32-2014 JEE Advanced 2014 Solved Paper
Magnetic field due to current carrying loop = 15. (c) From principle of continuity,
Magnetic field due to straight wires a1v1 = a2v2
B = B1cos + B2cos = 2 B1cos
r12 v1 r22 v2
2
0 Ia 0I a 2 2
= 2 20 5 1 v2
2 2 3/ 2 2 a2
h 2
a 2
h2
2 a h
v2= 2000 mms–1 = 2 ms–1
h 1.2a
16. (a) A B
The current is from P to Q in wire 1 and from R to S in
wire 2. C
12. (b) We know that torque

M B
= MB sin 1 2
PA PB a va
2
0I
= I a2 2 sin 30
2 d 1 2
PC PB l vl
2
2 2
a 0I But PC = PA
=
2d
13. (d) Heat lost by monatomic gas at constant volume 1 2 1 2 a
l vl a va vl va
= Heat gained by diatomic gas at constant pressure 2 2 l

nCv1 700 T nC p2 T 400


a
Volume flow rate
3 7 l
R 700 T R T 400
2 2
17. (c)
2100 3T 7T 2800
10T = 4900
T = 490 K h2
14. (d) In this case both the gases are at constant pressure.
v = 2gh2
nC p1 700 T nC p2 T 400

5 7 h1
R 700 T R T 400
2 2
3500 – 5T = 7T – 2800
2gh2 × 2h1
12T = 6300 g
T = 525 K
= 2 h1h2
Applying first law of thermodynamics
W1 U1 Q1 If geff > g
geff = g
and W2 U2 Q2 geff < g
As the gas two system is thermally insulated, therefore
In all the three cases d 2 h1h2 1.2 m
Q1 Q2 0
If geff = 0, then no water leaks out
W1 W2 U1 U2 18. (a) If Q1, Q2, Q3 and Q4 are all positive, then the force will
be along + y-direction.
= nCv1 525 700 n2 Cv2 525 400
If Q1, Q2 are positive and Q3, Q4 are negative the force
3R 5R will act along + x-direction.
= 2 175 2 125 If Q1, Q4 are positive and Q2, Q3 are negative then
2 2
attractive force will dominate repulsive force and the
= –525R + 625R = – 100 R
force will be along – y direction.
Therefore, total work done = –100 R
JEE Advanced 2014 Solved Paper 2014-33

At = 11.5° the bodies are on the verge of moving,


1 1 1 f = 0.6 g
19. (b) For (P) 1
f R1 R2 At > 11.5° the bodies start moving and f = 0.6 g
The above relationship is true for (d).
2 1
1.5 1 f r
r r
CHEMISTRY
For the combination
1. (d) In dye test, phenolic — OH group is converted to
1 1 1 1 1 2
— O– which activates the ring towards electrophilic
F f1 f2 r r r aromatic substitution
2. (c) Be2 = 1s2 * 1s2 2s2 *2s2
r
F B2 = 1s2 * 1s2 2s2 * 2s2 2pz2
2
C2 = 1s2 *1s2 2s2 * 2s2 2pz2 2px1 2py1
1 1 1 N2 = 1s2 *1s2 2s2 *2s2 2pz2 2px2 2py2
For (Q) 1
f R1 R2 Thus only C2 will be paramagnetic
3. (b) M N
1 1 0.5 1 r = k [M]x
1.5 1
r r 2r when M = 2M; r = 8r, thus
f = 2r 8r = k[2M]x
For the combination 8 = (2)x
x= 3
1 1 1 1 1 2 1
4. (b) Greater the extent of branching, lesser is the boiling
F f1 f2 2r 2r 2r r
point of the hydrocarbon, so order of b.p is III > II > I.
F=r 5. (c) The given reaction proceeds through SN1 mechanism
Similarly, we can either find or do not find the remaining which involves carbocation as intermediate.
options (b) is the correct option.
20. (d) If (m1 + m2) sin < N2 the bodies will be at rest H
Ph 3C — O — R Ph3C — O — R Ph3C ROH
i.e., (m1 + m2)g sin < m2 g cos H
Thus, higher the stability of the carbocation, greater
will be reactivity. Presence of electron releasing group
f = N2 (e.g., — OCH3) in p-position of the phenyl group will
N2 disperse the positive charge of the carbocation by +M
effect, hence stabilizes the carbocation.
6. (d)
(m1 + m2)g cos (m1 + m2) sin (i) CH 3 MgBr
(m1 + m2)g (ii) aq. acid
O
Cl O Cl O
7. (a) KIO4 + H2O2 KIO3 + H2O + O2
m2 g Thus H2O2 is acting as a reducing agent
tan
m1 m2 g 2NH2OH + H2O2 N2 + 4H2O
Here H2O2 is acting as an oxidising agent
m2
tan 8. (a) P4 8SOCl 2 4PCl3 4SO 2 2S2 Cl 2
m1 m2
white phosphorus

0.3 2 9. (c)
tan
1 2
Complete
tan < 0.2 XeF6 Hydrolysis
XeO3 + H2F2
(P)
i.e., If the angle < 11.5° the frictional force is less than
OH –/H2O
N2 = m2g = 0.3 × 2 × g = 0.6 g OH –/H2O
and is equal to (m1 + m2)g sin HXeO4– XeO64– + Xe(g)
(Q) (Slow disproportionat
ion) + H2O + O2(g)
EBD_7801
34-2014 JEE Advanced 2014 Solved Paper
10. (b) Given conditions are boiling conditions for water due
d 40
to which 2
24 d 10
Stotal = 0
Ssystem + Ssurroundings = 0 d = 48 – 2d
Ssystem = – Ssurroundings 3d = 48
d = 16 cm
For process, Ssystem > 0
12. (d) The general formula of mean free path ( ) is
Ssurroundings < 0
11. (c) According to Graham’s law of diffusion, if all conditions RT
=
are identical, 2 d2 N A p
1 (d = diameter of molecule, p = pressure inside the vessel)
r=
M Since d and p are same for both gases, ideally their
As in this question, all conditions are identical for X are same. Hence it must be the higher drift speed of X
and Y, then due to which it is facing more collisions per second
with the inert gas in comparison to gas Y. Hence X
rx My faces more resistance from inert gas than Y and hence
covers lesser distance than that predicted by Graham’s
ry Mx
law.
CH 3CH2 I
NaNH2 + – C– Na+
H Na O (1 eq.)
13. (a) HO

CH3 I C H2
+– C H3CO H3CO
(1 eq.) Lindlar
Na O
catalyst
H H
(X), C7H14O
OH
NaNH2 Br
14. (c) H C–Na+

O–Na+ (i) H + OH CrO3


C O
(ii) H2 , pd/C

(Y), C7H17O

15. (b) Ni 2 4HCl [NiCl 4 ]2


(M1) (Q) Tetrahedral
2
Ni 4KCN [Ni(CN) 4 ]2
(M1) (R) Square planar

[Note: Co2 6CN [Co(CN)6 ]4 ]


Octahedral

OH
16. (d) Zn 2 2OH Zn(OH)2 [Zn(OH) 4 ]2
(M2) (S) excess
white ppt. so lub le

17. (b) Complex Magnetic character Isomerism


3+ 3
P, [Cr(NH 3 ) 4 Cl2 ]Cl Cr is d , hence paramagnetic cis-trans
3+ 1
Q, [Ti(H 2O)5Cl](NO 3 ) 2 Ti is d , hence paramagnetic. ionization
2+ 8
R, [Pt(en)(NH3 )Cl]NO3 Pt is d , complex is square planar, ionization
all electrons are paired, hence diamagnetic
3+ 6
S, [Co(NH3 ) 4 (NO3 ) 2 ]NO3 Co is d , all electrons are paired cis-trans
due to strong ligands, hence diamagnetic
JEE Advanced 2014 Solved Paper 2014-35

18. (c) P. d – d ( bonding)

Q. p – d( bonding)

R. p–d antibonding)

S. d–d antibonding)

19. (a)
O O
(1) C6H5CH2—C—O CH3 C 6H 5CH 2 + CO 2 + CH 3 – O
O O CH3 CH3
(3) C6H5CH2—C—O C— CH3 CO2 + C6H5CH2 + CH3—C—O
CH2 – C6H5 CH2—C6H5
O
C O CH3 O CH3
(4) C6H5 O C—CH3 C6H5— C — O + O — C — CH3
CH3 CH3
O O
C O C
(2) C 6H5 O CH3 C 6H 5 O + CH 3 — O
NO2

red hot Nitration NH4HS


20. (c) Scheme (III) : H H Fe, 873 K
Acetylene, P
NO2
NO2 NO2 NO2

NaNO2 H 2O
+ N2 + H +
H+ + boil
NH2 N2 OH
(C6H5NO3)
OH OH OH OH

Conc. H2SO4 Conc. HNO3


O2N O2N
dil. H2SO4
Scheme (IV) :
60°C Conc. H2 SO4 heat
OH OH OH OH
Q (C6 H5NO4 )
SO3H SO3H
EBD_7801
36-2014 JEE Advanced 2014 Solved Paper
NO2 NH2 NHCOCH3 NHCOCH3

Sn/HCl CH 3COCl Conc.


Scheme (II) :
H2SO4

R
SO3H
NHCOCH3 NHCOCH3 NH2
NO2 NO2 NO2
Conc. HNO3 dil. OH –
H2SO4 , heat

(C6H6N2O2)
SO3H
NO2 NO2 NO2 NO2

(i) KMnO4, OH , heat SOCl2 NH3
Scheme (I) : +
(ii) H , H2O

CH3 COOH COCl CONH2


S (C7H6N2O3)

3
2ab
MATHEMATICS 2a2 b 2 sin 2 120 4 3y
1. (c) Card numbered 1 is always placed in envelope a b c abc x c c 2c x c
numbered 2, we can consider two cases. 3. (d)
Case I: Card numbered 2 is placed in envelope numbered
1.
Y
Then it is dearrangement of 4 objects, which can be S
done in
P
1 1 1 1
4! 1 = 9 ways
1! 2! 3! 4!
Case II: Card numbered 2 is not placed in envelope X
numbered 1. (2, 0)
(– 2, 0)
Then it is dearrangement of 5 objects, which can be
done in Q
1 1 1 1 1 R
5! 1 = 44 ways
1! 2! 3! 4! 5!
Total ways = 44 + 9 = 53
Let the tangent to
2. (b) Let two sides of be a and b.
y2 = 8x be
Then a + b = x and ab = y
Also given x2 – c2 = y, where c is the third side of . 2
y mx
2
m
a b c2 ab a2 b2 c2 ab If it is common tangent to parabola and circle, then

a 2 b2 c 2 1 1 2
cos c c = 120° y mx is a tangent to x2 + y2 = 2
2ab 2 2 m

r 4 2
where = area of triangle m 4
R s abc 2 2
m2 1 m (1 m2 )
2
2
1
8 ab sin c
r 4 2
2 m4 + m2 – 2 = 0
R a b c a b c abc (m2 + 2) (m2 – 1) = 0
abc
2
JEE Advanced 2014 Solved Paper 2014-37

m = 1 or – 1 9
Required tangents are y = x + 2 and y = – x – 2 LHS and RHS 3
4
Their common point is (– 2, 0) Equation has no solution.
Tangents are drawn from (– 2, 0)
Chord of contact PQ to circle is 2
17
x. (– 2) + y.0 = 2 or x = – 1 7. (a) Let I = 2cosec x dx
and Chord of contact RS to parabola is
4
y. 0 = 4 (x – 2) or x = 2
Hence coordinates of P and Q are 2
16
(– 1, 1) and (– 1, – 1) = cosec x cot x cosec x cot x 2cosec x dx
Also coordinates of R and S are
(2, – 4) and (2, 4) 4

1 2 16
Area of trapezium PQRS is 2 8 3 15 1
2 I= 2 cosec x cot x . cosec x dx
cosec x cot x
4. (a) According to given condition, we can have the
4
following cases Let cosec x + cot x = eu
(I) G G B B B (II) B G G B B
(III) G B G B B (IV) B G B G B cosec x cot x cosec 2 x dx eu du
(V) G B B G B – cosec x dx = du
i.e., the two girls can occupy two of the first three places
(case I, II, III) or second and fourth (case IV) or first Also at x ,u ln 2 1
4
and fourth (case V) places.
Thus favourable cases are at x , u = ln 1 = 0
2
= 3 × 2! × 3! + 2 × 2! × 3! = 60 0
u 16
Total ways in which 5 persons can be seated = 5! = 120 I 2 eu e du
60 1 ln 2 1
Required probability =
120 2 ln 2 1
5. (d) Quadratic equation with real coefficients and purely u 16
= 2 eu e du
imaginary roots can be considered as 0
p(x) = x2 + a = 0 where a > 0 and a R 4 7 12
The p[p(x)] = 0 8. (c) Coeff. of x11 in exp. of 1 x2 1 x3 1 x4
(x2 + a)2 + a = 0 = (Coeff. of xa ) × (Coeff. of xb ) × (Coeff. of xc )
x4 + 2ax2 + (a2 + a) = 0 Such that a + b + c = 11
Here a = 2m, b = 3n, c = 4p
2a 4a 2 4a 2 4a 2m + 3n + 4p = 11
x2
2 Case I : m = 0, n = 1, p = 2
Case II : m = 1, n = 3, p = 0
x2 a ai Case III : m = 2, n = 1, p = 1
Case IV : m = 4, n = 1, p = 0
x a ai i where , 0
Required coeff.
p[p(x)] = 0 has complex roots which are neither purely
real nor purely imaginary. = 4 C0 7
C1 12
C2 4
C1 7
C3 12
C0
6. (d) sin x + 2sin 2x – sin 3x = 3 C2 C1 +4 7 12
C1 4
C4 7
C1 12
C0
sin x + 4sin x cos x – 3sin x + 4sin3 x = 3 = 462 + 140 + 504 + 7 = 1113
sin x (–2 + 2cos x + 4sin2 x) = 3
sin x (– 2 + 2cos x + 4 – 4cos2 x) = 3 x2
9. (b) F x f t dt for x 0, 2
3
2 + 2 cos x – 4cos2x = 0
sin x
F x f x .2 x
1 1 1 3
2 – 4cos 2 x 2 . 2cos x. + =
2 4 4 sin x Now F x f x x 0, 2
2 f x .2 x f x
9 1 3
2 cos x
4 2 sin x f x
2x
f x
EBD_7801
38-2014 JEE Advanced 2014 Solved Paper
ln f (x) = x2 + c 1 2 4 1 3 3 2 2 3 2 3 4
2
x2 =
f (x) = e x c
e .e c 3 5 7 3 5 7 3 5 7 3 5 7
As f (0) = 1 1 = ec 8 9 12 24 53
=
2 105 105
f x ex
12. (c) If x1, x2, x3 are in AP then 2x2 = x1 + x3
x2 2
LHS is even, x1 & x3 can be both even or both odd.
So F x e x dx ex 1 x1 and x3 both can be even in 1 × 3 = 3 ways
0 x1 and x3 both can be odd in 2 × 4 = 8 ways
4 Total favourable ways = 3 + 8 = 11
F 2 e 1
10. (b) Given D.E. can be written as Also one number from each box can be drawn in 3 × 5
× 7 ways
dy x x4 2x
y Total ways = 105
dx 1 x 2 1 x2 11
x Hence required probability =
If dx 1
log 1 x 2 105
2
e 1 x e 2 1 x2 a 2a
Solution is given by 13. (d) PQ is a focal chord, Q 2
,
t t
2 2 x4 2x Also QR || PK mQR = mPK
y 1 x 1 x . dx
1 x2 2a
2ar
t 0 2at
x5 a
y 1 x2 x2 c ar 2 2a at 2
5 t 2
AB f (0) = 0 At x = 0, y = 0
1
c=0 2a r
t 2at
x5 =
y 1 x2 x2 a
1
r
1
r a 2 t2
5 t t
x5 1
x2 2 – t2 = t r
or y = f (x) = 5 t
1 x2 r
1
otherwise Q will coincide with R
t
3 x5
2 x2 t2 1
I 5 dx 2 t2 1 tr r
t
3 1 x2 14. (b) Tangent at P is
2
ty x at 2 ....(i)
3 Normal at S
2
x2 x5 sx + y = 2as + as3 ....(ii)
= 2 dx is odd
1
0 1 x2 1 x2 But given st = 1 s
t
put x = sin dx = cos d
x 2a a
y
3 3 t t t3
2
I = 2 sin d 1 cos 2 d
xt 2 yt 3 2at 2 a
0 0
Putting value of x from equation (i) in above equation
we get
sin 2 3 3
= t 2 ty at 2 yt 3 2at 2 a
2 0 3 4
11. (b) x1 + x2 + x3 will be odd t 3 t 3 y at 4 2at 2 a
If two are even and one is odd
or all three are odd. 2t 3 y a t4 2t 2 1
Required probability 2
= P (EEO) + P(EOE) + P(OEE) + P(OQO) a t4 2t 2 1 a t2 1
y
2t 3 2t 3
JEE Advanced 2014 Solved Paper 2014-39

1 h or a = 3 and b = 0
a 1 There are only 2 solutions.
15. (a) g a lim t a 1 t dt
h 0 h
Q(3) f (x) = sinx2 + cosx2
1 h 9
1 1/ 2 1/ 2 f (x) is max. 2 at x2 = or
g lim t 1 t dt 4 4
2 h 0 h
3
1 h x or 13, 13
1 h 2 2
1 1
= lim dt = lim dt There are four points.
t 1 t h 0 2 2
h 0 h h 1 1
t 2 2
3x2
2 2 3x2
R(1) I dx = x
dx
x
1 h 21 e 21 e
1
t 1 h
= lim sin 1 2 lim sin 1 2t 1 b b
h 0 1 h 0 Using f x dx f a + b – x dx
h
2 h a a
2
1 1 3 x2 e x
= lim sin 1 2h sin 2h 1 = dx
h 0
21 ex

=
2 3 x2 1 e x 2
2 2 2 2I dx = 3x 2 dx
x
2 1 e 2
1 h
a 1
16. (d) g a lim t a
1 t dt 3 2
2I x 8 8 16
h 0 h 2
I=8
1 h
a a 1 1/ 2
g a lim 1 t t dt 1 x
h 0 cos 2 x log dx
h 1 x
1/ 2
b b S(4) 1/ 2
=0
using f x dx f a b x dx 1 x
cos 2 x log dx
a a 1 x
0
... Numerator
= 0, function being odd.
1 h
a
Also g 1 a lim ta 1 1 t dt Hence option (d) is correct sequence.
h 0 h 1
18. (a) P(4) y cos 3cos x
Thus g a g 1 a
y cos cos 1 4 x 3 3 x
g a g 1 a g a g 1 a 0
y = 4x3 – 3x
1 1 1
Putting a we get g g 0 dy d2y
2 2 2 12 x 2 3 and 24 x
dx dx 2
1
or g 0 1 d2y dy
2 x2 1 x
17. (d) P(2) Let f (x) = ax2 + bx + c y dx dx
where a, b, c 0 and a, b, c are integers.
1
f (0) = 0 c = 0 = 3
x 2 1 24 x x 12 x 2 3
f (x) = ax2 + bx 4x 3x
1 3x
= 8 x2 8 4 x2 1
Also f x dx 1 4 x 3 3x
0
3x 12 x 2 9 9 4 x3 3 x
3 2 1 = = 9
ax bx a b
1; 1 4 x3 3x 4 x3 3x
3 2 3 2 Q(3) ... A1, A2, A3, ... An are the vertices of a regular
0
2a + 3b = 6
a and b are integers polygon of n sides with its centre at origin and a1, a2 ,
a = 0 and b = 2
.... an are their position vectors.
EBD_7801
40-2014 JEE Advanced 2014 Solved Paper
Range of f4 = 0,
a1 a2 .... an
f4 is onto
2 From graph f4 is not one one.
2
Then ak ak 1 sin
sin x,
n x 0
Q(3) : f3 x
2 x, x 0
2
and ak ak 1 cos From graph f is differentiable
n
Hence given equation reduces to but not one one.

n 1 2 sin
2
n 1 2 cos x2 , x 0
n n R(2): f 2 of1 x
e2 x , x 0
2 2
tan 1 n 8
n n 4
x2 y2
R.(2) Normal from P(h, 1) on 1 is
6 3
x h y 1
h/6 1/ 3
2 x h h y 1 ; 2x – hy – h = 0
It is perpendicular to x + y = 8
From graph f2of1 is neither
2
1 1 h=2 continuous nor one one.
h
1 1 2 S(4): f 2 x x 2 , x 0,
S.(1) tan 1 tan 1 tan 1 It is continuous and one one.
2x 1 4x 1 x2
1 1
2k 2k
1 2x 1 4x 1 2 20. (c) P(1): zk cos i sin , k = 1 to 9
tan tan 1 10 10
1 1 x2
1 . 2k
2x 1 4 x 1 i
zk e 10
1 4x 1 2 x 1 1 2
tan tan 2k
8 x2 6x 1 1 x2 1 i
10
Now zk.zj = 1 zj e zk
1 6x 2 1 2 zk
tan tan
8x2 6 x x2 We know if zk is 10th root of unity so will be zk.
1 3x 1 1 2 For every zk, there exist zi = zk
tan tan
2
4x 3x x2 Such that zk . z j zk . z k 1
3x 1 2 Hence the statement is true.
4 x 3x x 2
2
zk
Q(2): z1 z k z for z1 0
3x2 – 7x – 6 = 0 (for x > 0) z1
2 We can always find a solution to z1.z zk
x = 3 or (rejected as x > 0)
3 Hence the statement is false.
Only one +ve solution is there
Hence (a) is the correct option. R(3): We know z10 1 z 1 z z1 .... z z9

x2 , x 0 z10 1
19. (d) P(1): f 4 x z z1 z z2 ..... z z9
z 1
e2 x 1, x 0 = 1 + z + z2 + ... z9
For z = 1 we get
1 z1 1 z2 ..... 1 z9 10
1 z1 1 z2 ..... 1 z9
1
10
Telegram @unacademyplusdiscounts

Join Us on Telegram for More Such Books

https://telegram.me/unacademyplusdiscounts

Join us from the above link or search ''unacademyplusdiscounts'' in Telegram


JEE Advanced 2013 Solved Paper 2013-1

JEE ADVANCED 2013


1. The question paper consists of 3 parts (Physics, Chemistry & Mathematics). Each part consists of 3 sections.
2. For each question in Section I, you will be awarded 2 marks if you darken the bubble corresponding to the correct
answer and zero mark if no bubbles are darkened. No negative marks will be awarded for incorrect answers in this
section.
3. For each question in Section II, you will be awarded 4 marks if you darken all the bubble(s) corresponding to only
the correct answer(s) and zero mark if no bubbles are darkened. In all other cases, minus one (–1) mark will be
awarded.
4. For each question in Section III, you will be awarded 4 marks if you darken all the bubble(s) corresponding to only
the correct answer(s) and zero mark if no bubbles are darkened. In all other cases, minus one (–1) mark will be
awarded.

PAPER - 1
5. Two non-reactive monoatomic ideal gases have their atomic
PHYSICS masses in the ratio 2 : 3. The ratio of their partial pressures,
SECTION - I when enclosed in a vessel kept at a constant temperature, is
4 : 3. The ratio of their densities is
DIRECTIONS : This section contains 10 multiple choice (a) 1 : 4 (b) 1 : 2
questions. Each question has four choices (a), (b), (c) and (d) out (c) 6 : 9 (d) 8 : 9
of which ONLY ONE is correct. 6. Two rectangular blocks, having identical dimensions, can be
1. The image of an object, formed by a plano-convex lens at a arranged either in configuration-I or in configuration-II
distance of 8 m behind the lens, is real is one-third the size of as shown in the figure. One of the blocks has thermal
conductivity k and the other 2k. The temperature difference
2
the object. The wavelength of light inside the lens is between the ends along the x-axis is the same in both the
3 configurations. It takes 9 s to transport a certain amount of
times the wavelength in free space. The radius of the curved heat from the hot end to the cold end in the configuration-I.
surface of the lens is The time to transport the same amount of heat in the
(a) 1 m (b) 2 m configuration-II is
(c) 3 m (d) 6 m Configuration-II
2. The diameter of a cylinder is measured using a Vernier callipers Configuration-I
with no zero error. It is found that the zero of the Vernier scale
lies between 5.10 cm and 5.15 cm of the main scale. The 2k
Vernier scale has 50 divisions equivalent to 2.45 cm. The 24th
division of the Vernier scale exactly coincides with one of k 2k k
the main scale divisions. The diameter of the cylinder is x
(a) 5.112 cm (b) 5.124 cm (a) 2.0 s (b) 3.0 s
(c) 5.136 cm (d) 5.148 cm (c) 4.5 s (d) 6.0 s
7. A pulse of light of duration 100 ns is absorbed completely by
1 a small object initially at rest. Power of the pulse is 30 mW
3. A ray of light travelling in the direction (iˆ 3 ˆj ) is incident
2 and the speed of light is 3×108 ms–1. The final momentum of
on a plane mirror. After reflection, it travels along the direction the object is
(a) 0.3 × 10–17 kg ms–1 (b) 1.0 × 10–17 kg ms–1
1 ˆ
(i 3 ˆj ) . The angle of incidence is (c) 3.0 × 10–17 kg ms–1 (d) 9.0 × 10–17 kg ms–1
2 8. A particle of mass m is projected from the ground with an
(a) 30° (b) 45° initial speed u0 at an angle with the horizontal. At the
(c) 60° (d) 75° highest point of its trajectory, it makes a completely inelastic
4. In the Young’s double slit experiment using a monochromatic collision with another identical particle, which was thrown
light of wavelength , the path difference (in terms of an vertically upward from the ground with the same initial speed
integer n) corresponding to any point having half the peak u0. The angle that the composite system makes with the
intensity is horizontal immediately after the collision is

(a) (2n 1) (b) (2n 1) (a) (b)


2 4 4 4
(c) (2 n 1) (d) (2 n 1) (c) (d)
8 16 2 2
EBD_7801
2-2013 JEE Advanced 2013 Solved Paper
9. The work done on a particle of mass m by a force, (a) Thedirection of themagnetic field is –z direction
(b) The direction of the magnetic field is +z direction
x y
K iˆ ˆj 50 M
2 2 32 2
(x y ) (x y 2 )3 2 (c) The magnitude of the magnetic field units
3Q
(K being a constant of appropriate dimensions), when the
particle is taken from the point (a, 0) to the point (0, a) along 100 M
a circular path of radius a about the origin in the x – y plane (d) The magnitude of the magnetic field is units
3Q
is
14. Two non-conducting solid spheres of radii R and 2R, having
2K K uniform volume charge densities 1 and 2 respectively, touch
(a) (b)
a a each other. The net electric field at a distance 2R from the
centre of the smaller sphere, along the line joining the centres
K
(c) (d) 0
2a of the spheres, is zero. The ratio 1
can be
10. One end of a horizontal thick copper wire of length 2L and 2
radius 2R is welded to an end of another horizontal thin 32
copper wire of length L and radius R. When the arrangement (a) –4 (b)
25
is stretched by applying forces at two ends, the ratio of the
elongation in the thin wire to that in the thick wire is 32
(a) 0.25 (b) 0.50 (c) (d) 4
25
(c) 2.00 (d) 4.00 15. In the circuit shown in the figure, there are two parallel plate
SECTION - II capacitors each of capacitance C. The switch S1 is pressed
This section contains 5 multiple choice questions. Each question first to fully charge the capacitor C1 and then released. The
has four choices (a), (b), (c) and (d) out of which ONE or MORE switch S2 is then pressed to charge the capacitor C2. After
are correct. some time, S2 is released and then S3 is pressed. After some
time
11. A horizontal stretched string, fixed at two ends, is vibrating
in its fifth harmonic according to the equation, S1 S2 S3
y(x, t) = (0.01 m) sin [(62.8 m–1)x] cos[(628 s–1)t]. Assuming
= 3.14, the correct statement(s) is (are)
(a) The number of nodes is 5
(b) The length of the string is 0.25 m C1 C2 C3
2V0
(c) The maximum displacement of the midpoint of the string,
from its equilibrium position is 0.01 m
(d) The fundamental frequency is 100 Hz
12. A solid sphere of radius R and density is attached to one
end of a mass-less spring of force constant k. The other end
of the spring is connected to another solid sphere of radius (a) he charge on the upper plate of C1 is 2CV0
R and density 3 . The complete arrangement is placed in a (b) The charge on the upper plate of C1 is CV0
liquid of density 2 and is allowed to reach equilibrium. The (c) The charge on the upper plate of C2 is 0
correct statement(s) is (are) (d) The charge on the upper plate of C2 is –CV0
4 R3 g SECTION - III
(a) The net elongation of the spring is This section contains 5 questions. The answer to each question
3k
is a single digit integer, ranging from 0 to 9 (both inclusive).
8 R3 g
(b) The net elongation of the spring is 16. The work functions of Silver and Sodium are 4.6 and 2.3 eV,
3k respectively. The ratio of the slope of the stopping potential
(c) The light sphere is partially submerged versus frequency plot for Silver to that of Sodium is
(d) The light sphere is completely submerged 17. A freshly prepared sample of a radioisotope of half-life
13. A particle of mass M and positive charge Q, moving with a 1386 s has activity 103 disintegrations per second. Given
constant velocity u1 4iˆ ms 1 , enters a region of uniform that ln 2 = 0.693, the fraction of the initial number of nuclei
(expressed in nearest integer percentage) that will decay in
static magnetic field, normal to the x-y plane. The region of
the first 80 s after preparation of the sample is
the magnetic field extends from x = 0 to x = L for all values of
18. A particle of mass 0.2 kg is moving in one dimension under a
y. After passing through this region, the particle emerges on
force that delivers a constant power 0.5 W to the particle. If
the other side after 10 milliseconds with a velocity
the initial speed (in ms–1) of the particle is zero, the speed
u2 2( 3iˆ ˆj ) ms 1 . The correct statement(s) is (are) (in ms–1) after 5 s is
JEE Advanced 2013 Solved Paper 2013-3

19. A uniform circular disc of mass 50 kg and radius 0.4 m is 5. In the reaction,
rotating with an angular velocity of 10 rad s–1 about its own P+Q ?R+S
axis, which is vertical. Two uniform circular rings, each of The time taken for 75% reaction of P is twice the time taken
mass 6.25 kg and radius 0.2 m, are gently placed symmetrically for 50% reaction of P. The concentration of Q varies with
on the disc in such a manner that they are touching each reaction time as shown in the figure. The overall order of the
other along the axis of the disc and are horizontal. Assume reaction is
that the friction is large enough such that the rings are at rest
relative to the disc and the system rotates about the original
[Q]0
axis. The new angular velocity (in rad s–1) of the system is
20. A bob of mass m, suspended by a string of length l1, is given
a minimum velocity required to complete a full circle in the [Q]
vertical plane. At the highest point, it collides elastically
Time
with another bob of mass m suspended by a string of length
l2, which is initially at rest. Both the strings are mass-less (a) 2 (b) 3
and inextensible. If the second bob, after collision acquires (c) 0 (d) 1
the minimum speed required to complete a full circle in the 6. Concentrated nitric acid, upon long standing, turns yellow
brown due to the formation of
l1 (a) NO (b) NO2
vertical plane, the ratio is
l2 (c) N2O (d) N2O4
7. The arrangement of X– ions around A+ ion in solid AX is
CHEMISTRY given in the figure (not drawn to scale). If the radius of X– is
250 pm, the radius of A+ is
SECTION - I
This section contains 10 multiple choice questions. Each question –
has 4 choices (a), (b), (c) and (d) out of which ONLY ONE is X
correct. +
A
1. The standard enthalpies of formation of CO2(g), H2O(l) and
glucose(s) at 25°C are –400 kJ/mol, –300 kJ/mol and –1300
kJ/mol, respectively. The standard enthalpy of combustion
(a) 104 pm (c) 183 pm
per gram of glucose at 25°C is
(b) 125 pm (d) 57 pm
(a) +2900 kJ (b) –2900 kJ
8. Upon treatment with ammoniacal H2S, the metal ion that
(c) –16.11 kJ (d) +16.11 kJ precipitates as a sulfide is
2. KI in acetone, undergoes SN2 reaction with each of P, Q, R (a) Fe(III) (c) Mg(II)
and S. The rates of the reaction vary as (b) Al(III) (d) Zn(II)
9. Methylene blue, from its aqueous solution, is adsorbed on
O
activated charcoal at 25°C. For this process, the correct
Cl statement is
H3C – Cl Cl Cl
(a) The adsorption requires activation at 25°C
(b) The adsorption is accompanied by a decrease in
P Q R S enthalpy
(c) The adsorption increases with increase of temperature
(a) P > Q > R > S (b) S > P > R > Q (d) The adsorption is irreversible
(c) P > R > Q > S (d) R > P > S > Q 10. Sulfide ores are common for the metals
3. The compound that does NOT liberate CO2, on treatment (a) Ag, Cu and Pb (c) Ag, Mg and Pb
with aqueous sodium bicarbonate solution, is (b) Ag, Cu and Sn (d) Al, Cu and Pb
(a) Benzoic acid (b) Benzenesulphonic acid SECTION - II
(c) Salicylic acid (d) Carbolic acid (Phenol)
This section contains 5 multiple choice questions. Each question
4. Consider the following complex ions, P, Q and R.
has four choices (a), (b), (c) and (d) out of which ONE OR MORE
P = [FeF6]3–, Q = [V(H2O)6]2+ and R = [Fe(H2O)6]2+ are correct.
The correct order of the complex ions, according to their
spin-only magnetic moment values (in B.M.) is 11. Benzene and naphthalene form an ideal solution at room
(a) R < Q < P (b) Q < R < P temperature. For this process, the true statement(s) is(are)
(c) R < P < Q (d) Q < P < R (a) G is positive (b) Ssystem is positive
(c) Ssurroundings = 0 (d) H= 0
EBD_7801
4-2013 JEE Advanced 2013 Solved Paper
12. The pair(s) of coordination complexes/ions exhibiting the 20. A tetrapeptide has —COOH group on alanine. This produces
same kind of isomerism is(are) glycine (Gly), valine (Val), phenyl alanine (Phe) and alanine
(a) [Cr(NH3)5Cl]Cl2 and [Cr(NH3)4Cl2]Cl (Ala), on complete hydrolysis. For this tetrapeptide, the
(b) [Co(NH3)4Cl2]+ and [Pt(NH3)2(H2O)Cl]+ number of possible sequences (primary structures) with —
(c) [CoBr2Cl2]2– and [PtBr2Cl2]2– NH2 group attached to a chiral center is
(d) [Pt(NH3)3](NO3)Cl and [Pt(NH3)3Cl]Br
13. The initial rate of hydrolysis of methyl acetate (1M) by a MATHEMATICS
weak acid (HA, 1M) is 1/100th of that of a strong acid (HX,
1M), at 25°C. The Ka of HA is SECTION - I
(a) 1 × 10–4 (b) 1 × 10–5 DIRECTIONS : This section contains 10 multiple choice
(c) 1 × 10 –6 (d) 1 × 10–3 questions. Each question has four choices (a), (b), (c) and (d) out
14. The hyperconjugative stabilities of tert-butyl cation and 2- of which ONLY ONE is correct.
butene, respectively, are due to
(a) p (empty) and * electron delocalisations 23 n
(b) * and electron delocalisations 1
1. The value of cot cot 1 2k is
(c) p (filled) and electron delocalisations n 1 k 1
(d) p (filled) * and * electron delocalisations
15. Among P, Q, R and S, the aromatic compound(s) is/are 23 25
(a) (b)
25 23
Cl 23 24
(c) (d)
24 23
AlCl3
P 2. Let PR 3iˆ ˆj 2 kˆ and SQ iˆ 3jˆ 4 kˆ determine
diagonals of a parallelogram PQRS and PT ˆi 2ˆj 3kˆ be
NaH
Q another vector. Then the volume of the parallelpiped
(NH4)2CO 3 determined by the vectors PT, PQ and PS is
R (a) 5 b) 20
100–115 °C
(c) 10 (d) 30
O O
O 1
HCl 3. Let complex numbers and lie on circles (x – x0)2 +
S
(y – y0)2 = r2 and (x – x0)2 + (y – y0)2 = 4r 2.
respectively. If z0 = x0 + iy0 satisfies the equation
2
(a) P (c) R 2 z0 r 2 2, then
(b) Q (d) S
SECTION - III 1 1
(a) (b)
This section contains 5 questions. The answer to each question 2 2
is a single digit integer, ranging from 0 to 9 (both inclusive). 1 1
16. The total number of lone-pairs of electrons in melamine is (c) (d)
7 3
17. The total number of carboxylic acid groups in the product
P is 4. For a > b > c > 0, the distance between (1, 1) and the point of
intersection of the lines ax + by + c = 0 and bx + ay + c = 0 is
O O less than 2 2 . Then
+
1.H 3O . (a) a + b – c > 0 (b) a – b + c < 0
O 2.O3 P (c) a – b + c > 0 (d) a + b – c < 0
3.H2 O2 5. Perpendiculars are drawn from points on the line
O O
x 2 y 1 z
to the plane x + y + z = 3. The feet of
2 1 3
18. The atomic masses of ‘He’ and ‘Ne’ are 4 and 20 a.m.u.,
respectively. The value of the de Broglie wavelength of ‘He’ perpendiculars lie on the line
gas at —73°C is “M” times that of the de Broglie wavelength x y 1 z 2 x y 1 z 2
of ‘Ne’ at 727°C ‘M’ is (a) (b)
5 8 13 2 3 5
19. EDTA4– is ethylenediaminetetraacetate ion. The total
number of N—Co—O bond angles in [Co(EDTA)]1–complex x y 1 z 2 x y 1 z 2
(c) (d)
ion is 4 3 7 2 7 5
JEE Advanced 2013 Solved Paper 2013-5

6. Four persons independently solve a certain problem correctly SECTION - II


1 3 1 1 DIRECTIONS : This section contains 5 multiple choice questions.
with probabilities , , , . Then the probability that the
2 4 4 8 Each question has four choices (a), (b), (c) and (d) out of which
problem is solved correctly by at least one of them is ONE or MORE are correct
11. A rectangular sheet of fixed perimeter with sides having
235 21 their lengths in the ratio 8 : 15 is converted into an open
(a) (b)
256 256 rectangular box by folding after removing squares of equal
area from all four corners. If the total area of removed squares
3 253 is 100, the resulting box has maximum volume. Then the
(c) (d)
256 256 lengths of the sides of the rectangular sheet are
7. The area enclosed by the curves y = sin x + cos x and (a) 24 (b) 32
(c) 45 (d) 60
y = cos x sin x over the interval 0, is k (k 1)
2 4n 2
12. Let Sn ( 1) k 2 . Then Sn can take value(s)
(a) 4( 2 1) (b) 2 2 ( 2 1) k 1

(c) 2( 2 1) (d) 2 2( 2 1) (a) 1056 (b) 1088


(c) 1120 (d) 1332
13. A line l passing through the origin is perpendicular to the
8. A curve passes through the point 1, . Let the slope of lines
6
l1 :(3 t)iˆ (–1 2t) ˆj (4 2t)k,
ˆ – t
y y
the curve at each point (x, y) be sec , x 0.
x x l2 :(3 2s)iˆ (3 2s) ˆj (2 s)k,
ˆ – s
Then the equation of the curve is Then, the coordinate(s) of the point(s) on l2 at a distance of

y 1 y 17 from the point of intersection of l and l1 is (are)


(a) sin log x (b) cos ec log x 2
x 2 x 7 7 5
(a) , , (b) (–1, –1, 0)
3 3 3
2y 2y 1
(c) s ec log x 2 (d) cos log x
x x 2 7 7 8
(c) (1, 1, 1) (d) , ,
9 9 9
14. Let f(x) = x sin x, x > 0. Then for all natural numbers n, f (x)
9. Let f : ,1 R (the set of all real number) be a positive, vanishes at
2
non-constant and differentiable function such that 1
(a) A unique point in the interval n, n
2
1
1
f (x) < 2f(x) and f 1. Then the value of f (x) dx lies 1
2
1/ 2 (b) A unique point in the interval n ,n 1
2
in the interval
(c) A unique point in the interval (n, n + 1)
(a) (2e – 1, 2e) (b) (e – 1, 2e – 1)
(d) Two points in the interval (n, n + 1)
e 1 e 1 15. For 3 × 3 matrices M and N, which of the following
(c) ,e 1 (d) 0,
2 2 statement(s) is (are) NOT correct?
(a) N'MN is symmetric or skew symmetric, according as
10. The number of points in (– ), for which
M is symmetric or skew symmetric
x2 – x sin x – cos x = 0, is
(b) MN – NM is skew symmetric for all symmetric matrices
(a) 6 (b) 4 M and N
(c) 2 (d) 0 (c) MN is symmetric for all symmetric matrices M and N
(d) (adj M) (adj N) = adj (MN) for all invertible matrices M
and N
EBD_7801
6-2013 JEE Advanced 2013 Solved Paper
SECTION - III 18. Consider the set of eight vectors
This section contains 5 questions The answer to each question is V aiˆ bjˆ ckˆ :a, b, c {–1,1} . Three non-coplanar
a single digit integer, ranging from 0 to 9 (both inclusive).
vectors can be chosen from V in 2p ways. Then p is
16. A vertical line passing through the point (h, 0) intersects the 19. Of the three independent events E1, E2 and E3, the probability
x2 y2 that only E1 occurs is , only E2 occurs is and only
ellipse 1 at the points P and Q. Let the tangents
4 3 E3 occurs is . Let the probability p that none of events
to the ellipse at P and Q meet at the point R. If (h) = area of E1, E2 or E3 occurs satisfy the equations ( 2 p = and
( – 3 )p = 2 . All the given probabilities are assumed to
the triangle PQR, max (h) and min (h),
1 1/ 2 h 1 2 1/ 2 h 1 lie in the interval (0, 1).
8 Pr obability of occurrence of E1
then 1 8 2 Then
5 Pr obability of occurrence of E 3
17. The coefficients of three consecutive terms of (1 + x)n+5 are 20. A pack contains n cards numbered from 1 to n. Two
in the ratio 5 : 10 : 14. Then n = consecutive numbered cards are removed from the pack and
the sum of the numbers on the remaining cards is 1224. If the
smaller of the numbers on the removed cards is k, then k – 20 =

PAPER - 2

1. The question paper consists of 3 parts (Physics, Chemistry & Mathematics). Each part consists of 3 sections.
2. For each question in Section I, you will be awarded 3 marks if you darken all the bubble(s) corresponding to only
the correct answer(s) and zero mark if no bubbles are darkened. In all other cases, minus one (–1) mark will be
awarded.
3. For each question in Section II and III, you will be awarded 3 marks if you darken the bubble(s) corresponding to
only the correct answer(s) and zero mark if no bubbles are darkened. In all other cases, minus one (–1) mark will be
awarded.

PHYSICS
SECTION - I
DIRECTIONS : This section contains 8 multiple choice questions.
Each question has four choices (a), (b), (c) and (d) out of which
ONE OR MORE arecorrect. C
1. The radius of the orbit of an electron in a Hydrogen-like
atom is 4.5 a0, where a0 is the Bohr radius. Its orbital angular
3h
momentum is . It is given that h is Planck constant and R
2
100 200 300 400 500
is Rydberg constant. The possible wavelength(s), when the
atom de-excites, is (are) (a) The rate at which heat is absorbed in the range 0 -100 K
varies linearly with temperature T.
9 9 (b) Heat absorbed in increasing the temperature from 0-
(a) (b) 100 K is less than the heat required for increasing the
32R 16R
temperature from 400-500 K.
9 4 (c) There is no change in the rate of heat absorption in the
(c) (d)
5R 3R range 400-500 K.
2. The figure below shows the variation of specific heat (d) The rate of heat absorption increases in the range 200-
capacity (C) of a solid as a function of temperature (T). The 300 K.
temperature is increased continuously from 0 to 500 K at a 3. Two non-conducting spheres of radii R1 and R2 and carrying
constant rate. Ignoring any volume change, the following uniform volume charge densities + and – , respectively,
statement(s) is (are) correct to a reasonable approximation. are placed such that they partially overlap, as shown in the
figure. At all points in the overlapping region
JEE Advanced 2013 Solved Paper 2013-7

(a) The speed of the particle when it returns to its


equilibrium position is u0
– (b) The time at which the particle passes through the
m
equilibrium position for the first time is t
R1 R2 k
(c) The time at which the maximum compression of the
4 m
spring occurs is t
3 k
(a) The electrostatic field is zero (d) The time at which the particle passes through the
(b) The electrostatic potential is constant 5 m
(c) The electrostatic field is constant in magnitude equilibrium position for the second time is t
3 k
(d) The electrostatic field has same direction 8. Two bodies, each of mass M, are kept fixed with a separation
4. Using the expression 2d sin = , one calculates the values 2L. A particle of mass m is projected from the midpoint of the
of d by measuring the corresponding angles in the range 0 line joining their centres, perpendicular to the line. The
to 90°. The wavelength is exactly known and the error in gravitational constant is G. The correct statement(s) is (are)
is constant for all values of . As increases from 0° (a) The minimum initial velocity of the mass m to escape
(a) The absolute error in d remains constant GM
the gravitational field of the two bodies is 4
(b) The absolute error in d increases L
(c) The fractional error in d remains constant (b) The minimum initial velocity of the mass m to escape
(d) The fractional error in d decreases
GM
5. Two vehicles, each moving with speed u on the same the gravitational field of the two bodies is 2
L
horizontal straight road, are approaching each other. Wind
blows along the road with velocity w. One of these vehicles (c) The minimum initial velocity of the mass m to escape
blows a whistle of frequency f1. An observer in the other 2GM
vehicle hears the frequency of the whistle to be f2. The the gravitational field of the two bodies is
L
speed of sound in still air is V. The correct statement(s) is (d) The energy of the mass m remains constant
(are)
(a) If the wind blows from the observer to the source, SECTION - II
f2 > f1
(b) If the wind blows from the source to the observer, This section contains 4 Paragraphs each describing theory,
f2 > f1 experiment, data etc. Eight questions relate to four paragraphs
with two questions on each paragraph. Each question of a
(c) If the wind blows from observer to the source,
paragraph has only one correct answer among the four choices
f2 < f1 (a), (b), (c) and (d).
(d) If the wind blows from the source to the observer,
Paragraph for Questions 9 and 10
f2 < f1
6. A steady current I flows along an infinitely long hollow The mass of a nucleus A Z X is less than the sum of the masses of
cylindrical conductor of radius R. This cylinder is placed (A-Z) number of neutrons and Z number of protons in the nucleus.
coaxially inside an infinite solenoid of radius 2R. The solenoid The energy equivalent to the corresponding mass difference is
has n turns per unit length and carries a steady current I. known as the binding energy of the nucleus. A heavy nucleus of
Consider a point P at a distance r from the common axis. The mass M can break into two light nuclei of masses m1 and m2 only
correct statement(s) is (are) if (m1 + m2) < M. Also two light nuclei of masses m3 and m4 can
(a) In the region 0 < r < R, the magnetic field is non-zero undergo complete fusion and form a heavy nucleus of mass M'
only if (m3 + m4) > M'. The masses of some neutral atoms are
(b) In the region R < r < 2R, the magnetic field is along the given in the table below:
common axis
1 1.007825 u 2 2.014102 u
(c) In the region R < r < 2R, the magnetic field is tangential 1H 1H
to the circle of radius r, centered on the axis 3 3.016050 u 4 4.002603 u
1H 2 He
(d) In the region r > 2R, the magnetic field is non-zero
6 6.015123 u 7 7.016004 u
7. A particle of mass m is attached to one end of a mass-less 3 Li 3 Li
spring of force constant k, lying on a frictionless horizontal 70 82
30 Zn 69.925325 u 34 Se 81.916709 u
plane. The other end of the spring is fixed. The particle starts
moving horizontally from its equilibrium position at time 152 151.919803 u 206 205.974455 u
64 Gd 82 Pb
t = 0 with an initial velocity u0. When the speed of the particle
209 208.980388 u 210 209.982876 u
is 0.5 u0, it collides elastically with a rigid wall. After this 83 Bi 84 Po
collision (1u = 932 MeV/c2)
EBD_7801
8-2013 JEE Advanced 2013 Solved Paper
9. The kinetic energy (in keV) of the alpha particle, when the 13. In the method using the transformers, assume that the ratio
210
nucleus 84 Po at rest undergoes alpha decay, is of the number of turns in the primary to that in the secondary
(a) 5319 (b) 5422 in the step-up transformer is 1 : 10. If the power to the
(c) 5707 (d) 5818 consumers has to be supplied at 200 V, the ratio of the
10. The correct statement is number of turns in the primary to that in the secondary in
the step-down transformer is
(a) The nucleus 63 Li can emit an alpha particle (a) 200 : 1 (b) 150 : 1
210 (c) 100 : 1 (d) 50 : 1
(b) The nucleus 84 Po can emit a proton
14. If the direct transmission method with a cable of resistance
(c) Deuteron and alpha particle can undergo complete 0.4 km–1 is used, the power dissipation| (in %) during
fusion transmission is
(d) The nuclei 70 and 82 can undergo complete (a) 20 (b) 30
30 Zn 34 Se
(c) 40 (d) 50
fusion
Paragraphs for Questions 15 and 16
Paragraph for Questions 11 and 12
A small block of mass 1 kg is released from rest at the top of a
A point charge Q is moving in a circular orbit of radius R in rough track. The track is a circular arc of radius 40 m. The block
the x-y plane with an angular velocity . This can be considered slides along the track without toppling and a frictional force acts
Q on it in the direction opposite to the instantaneous velocity. The
as equivalent to a loop carrying a steady current . A uniform work done in overcoming the friction up to the point Q, as shown
2
magnetic field along the positive z-axis is now switched on, which in the figure below, is 150 J.
increases at a constant rate from 0 to B in one second. Assume (Take the acceleration due to gravity, g = 10 ms-2)
that the radius of the orbit remains constant. The application of y
the magnetic field induces an emf in the orbit. The induced emf is
defined as the work done by an induced electric field in moving a
R P
unit positive charge around a closed loop. It is known that, for an
30°
orbiting charge, the magnetic dipole moment is proportional to
the angular momentum with a proportionality constant .
11. The change in the magnetic dipole moment associated with Q R
the orbit, at the end of the time interval of the magnetic field
change, is x
O
BQR 2
(a) – BQR2 (b) 15. The magnitude of the normal reaction that acts on the block
2 at the point Q is
(a) 7.5 N (b) 8.6 N
BQR 2
(c) (d) BQR 2 (c) 11.5 N (d) 22.5 N
2 16. The speed of the block when it reaches the point Q is
12. The magnitude of the induced electric field in the orbit at (a) 5 ms–1 (b) 10 ms–1
any instant of time during the time interval of the magnetic 1
(c) 10 3 ms (d) 20 ms–1
field change is
BR BR SECTION - III
(a) (b)
4 2
This section contains 4 multiple choice questions. Each question
(c) BR (d) 2BR has matching lists. The codes for the lists have choices (a), (b), (c)
Paragraph for Questions 13 and 14 and (d) out of which ONLY ONE is correct.
A thermal power plant produces electric power of 600 kW at 17. One mole of a monatomic ideal gas is taken along two cyclic
4000 V, which is to be transported to a place 20 km away from the processes E F G E and E F H E as shown in
power plant for consumers' usage. It can be transported either the PV diagram. The processes involved are purely isochoric,
directly with a cable of large current carrying capacity or by using isobaric, isothermal or adiabatic.
a combination of step-up and step-down transformers at the two
P
ends. The drawback of the direct transmission is the large energy F
32 P0
dissipation. In the method using transformers, the dissipation is
much smaller. In this method , a step-up transformer is used at the
plant side so that the current is reduced to a smaller value. At the
consumers' end, a step-down transformer is used to supply power
to the consumers at the specified lower voltage. It is reasonable to
assume that the power cable is purely resistive and the transformers
are ideal with power factor unity. All the currents and voltages P0 G
E H
mentioned are rms values.
V
V0
JEE Advanced 2013 Solved Paper 2013-9
Match the paths in List I with the magnitudes of the work Codes:
done in List II and select the correct answer using the codes P Q R S
given below the lists. (a) 2 3 1 4
List I List II
(b) 1 2 4 3
P. G E 1. 160 P0V0 ln2
(c) 4 1 2 3
Q. G H 2. 36 P0V0
(d) 2 3 4 1
R. F H 3. 24 P0V0
20. Match List I with List II and select the correct answer using
S. F G 4. 31 P0V0 the codes given below the lists:
Codes: List I List II
P Q R S P. Boltzmann constant 1. [ML2T-1]
(a) 4 3 2 1 Q. Coefficient of viscosity 2. [ML–1T–1]
(b) 4 3 1 2 R. Planck constant 3. [MLT–3K–1]
(c) 3 1 2 4 S. Thermal conductivity 4. [ML2T–2K–1]
Codes:
(d) 1 3 2 4
P Q R S
18. Match List I of the nuclear processes with List II containing (a) 3 1 2 4
parent nucleus and one of the end products of each process (b) 3 2 1 4
and then select the correct answer using the codes given (c) 4 2 1 3
below the lists: (d) 4 1 2 3
List I List II
P. Alpha decay 1. 15
8 O
15
7 O ... CHEMISTRY
Q. + decay 2. 138 234 SECTION - I
92 U 90 Th ...
R. Fission 3. 185 184 This section contains 8 multiple choice questions. Each question
83 Bi 82 Pb ...
has four choices (a), (b), (c) and (d) out of which ONE or MORE
S. Proton emission 4. 239 140
94 Pu 57 La ... are correct
Codes: 1. The Ksp of Ag2CrO4 is 1.1 × 10–12 at 298 K. The solubility
P Q R S (in mol/L) of Ag2CrO4 in a 0.1 M AgNO3 solution is
(A) 4 2 1 3 (a) 1.1 × 10–11 (b) 1.1 × 10–10
(B) 1 3 2 4 (c) 1.1 × 10–12 (d) 1.1 × 10–9
(C) 2 1 4 3 2. In the following reaction, the product(s) formed is(are)
(D) 4 3 2 1
19. A right angled prism of refractive index 1 is placed in a
rectangular block of refractive index 2, which is surrounded
by a medium of refractive index 3, as shown in the figure.
A ray of light 'e' enters the rectangular block at normal
incidence. Depending upon the relationships between 1, 2
and 3, it takes one of the four possible paths 'ef', 'eg', 'eh' or 'ei'.

e 45° g

1 h
i

3
2

Match the paths in List I with conditions of refractive indices in


List II and select the correct answer using the codes given below
the lists:
List I List II
P. e f 1. 1> 2 2
Q. e g 2. 2> 1 and 2> 3
R. e h 3. 1= 2 (a) P (major) (b) Q (minor)
S. e i 4. (c) R (minor) (d) S (major)
2< 1< 2 2 and 2 > 3
EBD_7801
10-2013 JEE Advanced 2013 Solved Paper
3. The major product(s) of the following reaction is(are) 5. The correct statement(s) about O3 is(are)
(a) O—O bond lengths are equal
(b) Thermal decomposition of O3 is endothermic
(c) O3 is diamagnetic in nature
(d) O3 has a bent structure
6. In the nuclear transmutation
9 8
4 Be +X 4 Be + Y
(X, Y) is(are)
(a) ( , n) (b) (p, D)
(d) (n, D) (d) ( , p)
7. The carbon-based reduction method is NOT used for the
extraction of
(a) Tin from SnO2
(b) Iron from Fe2O3
(c) Aluminium from Al2O3
(d) Magnesium from MgCO3.CaCO3
8. The thermal dissociation equilibrium of CaCO3(s) is studied
under different conditions
CaCO3(s) CaO(s) + CO2(g). For this equilibrium, the
correct statement(s) is(are)
(a) H is dependent on T
(b) K is independent of the initial amount of CaCO3
(c) K is dependent on the pressure of CO2 at a given T
(d) H is independent of catalyst, if any
SECTION - II
This section contains 4 Paragraphs each describing theory,
(a) P (b) Q experiment, data etc. Eight questions relate to four paragraphs
(c) R (d) S with two questions on each paragraph. Each question of a
4. After completion of the reactions (I and II), the organic paragraph has only one correct answer among the four choices
compound(s) in the reaction mixtures is(are) (a), (b), (c) and (d).
Paragraph for Questions 9 and 10
An aqueous solution of a mixture of two inorganic salts, when
treated with dilute HCl, gave a precipitate (P) and a filtrate (Q). The
Reaction I :
precipitate P was found to dissolve in hot water. The filtrate (Q)
remained unchanged, when treated with H2S in a dilute mineral
acid medium. However, it gave a precipitate (R) with H2S in an
ammoniacal medium. The precipitate R gave a coloured solution
(S), when treated with H2O2 in an aqueous NaOH medium.
Reaction II : 9. The precipitate P contains
(a) Pb2+ (b) Hg2 2+
(c) Ag+ (d) Hg2+
10. The coloured solution S contains
(a) Fe2(SO4)3 (b) CuSO4
(c) ZnSO4 (d) Na2CrO4
Paragraph for Questions 11 and 12
P and Q are isomers of dicarboxylic acid C4H4O4. Both decolorize
Br2/H2O. On heating, P forms the cyclic anhydride. Upon treat-
ment with dilute alkaline KMnO4, P as well as Q could produce
one or more than one from S, T and U.

(a) Reaction I : P and Reaction II : P


(b) Reaction I : U, acetone and Reaction II : Q, acetone
(c) Reaction I : T, U, acetone and Reaction II : P
(d) Reaction I : R, acetone and Reaction II : S, acetone
JEE Advanced 2013 Solved Paper 2013-11

11. Compounds formed from P and Q are, respectively 14. The pair of isochoric processes among the transformation
(a) Optically active S and optically active pair of states is
(T, U) (a) K to L and L to M (b) L to M and N to K
(b) Optically inactive S and optically inactive pair (c) L to M and M to N (d) M to N and N to K
(T, U) Paragraph for Questions 15 and 16
The reactions of Cl2 gas with cold-dilute and hot-concentrated
(c) Optically active pair (T, U) and optically active S
NaOH in water give sodium salts of two (different) oxoacids of
(d) Optically inactive pair (T, U)) and optically inactive S chlorine, P and Q, respectively. The Cl2 gas reacts with SO2 gas,
12. In the following reaction sequences V and W are respectively in presence of charcoal, to give a product R. R reacts with white
phosphorus to give a compound S. On hydrolysis, S gives an
oxoacid of phosphorus, T.
15. P and Q, respectively, are the sodium salts of
(a) Hypochlorus and chloric acids
(b) Hypochlorus and chlorus acids
(c) Chloric and perchloric acids
(d) Chloric and hypochlorus acids
16. R, S and T respectively, are
(a) SO2Cl2, PCl5 and H3PO4
(a) (b) SO2Cl2, PCl3 and H3PO3
(c) SOCl2, PCl3 and H3PO2
(d) SOCl2, PCl5 and H3PO4
SECTION - III
This section contains 4 multiple choice questions. Each question
(b) has matching lists. The codes for the lists have choices (a), (b), (c)
and (d) out of which ONLY ONE is correct.
17. The unbalanced chemical reactions given in List I show miss-
ing reagent or condition (?) which are provided in List II.
Match List I with List II and select the correct answer using
the code given below the lists :
(c)
List I List II
P. PbO2 + H2SO4 PbSO4 + O2 + other product 1. NO
Q. Na2S2O3 + H2O NaHSO4 + other product 2. I2
R. N2H4 N2 + other product 3. Warm
(d) S. XeF2 Xe + other product 4. Cl2
Codes :
Paragraph for Questions 13 and 14 P Q R S
A fixed mass 'm' of a gas is subjected to transformation of states (a) 4 2 3 1
from K to L to M to N and back to K as shown in the figure (b) 3 2 1 4
(c) 1 4 2 3
(d) 3 4 2 1
18. Match the chemical conversions in List I with the appropri-
ate reagents in List II and select the correct answer using
the code given below the lists :
List I List II

P 1. (i) Hg(OAc)2; (ii) NaBH4

Q. 2. NaOEt
13. The succeeding operations that enable this transformation
of states are R. 3. Et-Br
(a) Heating, cooling, heating, cooling
(b) Cooling, heating, cooling, heating
(c) Heating, cooling, cooling, heating S. 4. (i) BH3; (ii) H2O2/NaOH
(d) Cooling, heating, heating, cooling
EBD_7801
12-2013 JEE Advanced 2013 Solved Paper
Codes : MATHEMATICS
P Q R S
SECTION - I
(a) 2 3 1 4
(b) 3 2 1 4 DIRECTIONS : This section contains 8 multiple choice questions.
(c) 2 3 4 1 Each question has four choices (a), (b), (c) and (d) out of which
ONE or MORE are correct
(d) 3 2 4 1
19. An aqueous solution of X is added slowly to an aqueous 1. For a (the set of all real numbers), a –1,
solution of Y as shown in List I. The variation in conductiv-
(1a 2a ... n a) ) 1
ity of these reactions is given in List II. Match list I with List lim .
a 1
II and select the correct answer using the code given below n (n 1) [(na 1) (na 2) ... (na n)] 60
the lists : Then a =
List I List II (a) 5 (b) 7
P. (C2H5)3N + CH3COOH 1. Conductivity decreases and 15 17
X Y then increases (c) (d)
2 2
Q. KI (0.1M) + AgNO3(0.01M) 2. Conductivity decreases and 2. Circle(s) touching x-axis at a distance 3 from the origin and
X Y then does not change much
having an intercept of length 2 7 on y-axis is (are)
R. CH3COOH + KOH 3. Conductivity increases and
(a) x2 + y2 – 6x + 8y + 9 = 0
X Y then does not change much
(b) x2 + y2 – 6x + 7y + 9 = 0
S. NaOH + HI 4. Conductivity does not change
(c) x2 + y2 – 6x – 8y + 9 = 0
X Y much and then increases (d) x2 + y2 – 6x – 7y + 9 = 0
Codes :
P Q R S y z y z
3. Two lines L1 : x = 5, and L2 : x = ,
(a) 3 4 2 1 3 2 1 2
(b) 4 3 2 1 are coplanar. Then can take value(s)
(c) 2 3 4 1 (a) 1 (b) 2
(d) 1 4 3 2 (c) 3 (d) 4
20. The standard reduction potential data at 25°C is given below. 1
4. In a triangle PQR, P is the largest angle and cos P = .
E°(Fe3+, Fe2+) = + 0.77 V; 3
E°(Fe2+, Fe) = – 0.44 V Further the incircle of the triangle touches the sides PQ, QR
E°(Cu2+, Cu) = + 0.34 V; and RP at N, L and M respectively, such that the lengths of
PN, QL and RM are consecutive even integers. Then possible
E°(Cu+, Cu) = + 0.52 V
length(s) of the side(s) of the triangle is (are)
E°[O2(g) + 4H+ + 4e– 2H2O] = + 1.23 V; (a) 16 (b) 18
E°[O2(g) + 2H2O + 4e– 4OH–] = + 0.40 V (c) 24 (d) 22
E°(Cr3+, Cr) = – 0.74 V;
3 i
E°(Cr2+, Cr) = – 0.91 V 5. Let w and P = {wn : n = 1, 2, 3, ...}. Further H1 =
2
Match E° of the redox pair in List I with the values given in
List II and select the correct answer using the code given 1 1
z : Re z and H 2 z : Re z , where c is the
below the lists : 2 2
List I List II
set of all complex numbers. If z1 P H1, z2 P H 2 and
P. E°(Fe3+, Fe) 1. – 0.18 V
O represents the origin, then z1Oz2 =
Q. E°(4H2O 4H+ + 4OH–) 2. – 0.4 V
R. E°(Cu + Cu 2Cu+)
2+ 3. – 0.04 V (a) (b)
2 6
S. E°(Cr3+, Cr2+) 4. – 0.83 V
Codes : 2 5
(c) (d)
P Q R S 3 6
(a) 4 1 2 3 6. If 3x = 4x–1, then x =
(b) 2 3 4 1 2log3 2 2
(c) 1 2 3 4 (a) 2log3 2 1 (b) 2 log 2 3
(d) 3 4 1 2
1 2log 2 3
(c) 1 log 4 3 (d) 2log 2 3 1
JEE Advanced 2013 Solved Paper 2013-13

7. Let be a complex cube root of unity with 1 and P = [pij] 12. If chord PQ subtends an angle at the vertex of y2 = 4ax,
be a n × n matrix with pij = i+j. Then p2 0, when n = then tan =
(a) 57 (b) 55 2 2
(c) 58 (d) 56 (a) 7 (b) 7
3 3
8. The function f(x) = 2|x| + |x + 2| – | |x + 2| – 2 |x| | has a local
minimum or a local maximum at x = 2 2
(c) 5 (d) 5
3 3
2
(a) – 2 (b) Paragraphs for Questions 13 and 14
3
Let S = S1 S2 S3, where
2
(c) 2 (d) z –1 3i
3 S1 {z : | z | 4}, S2 z : Im 0
1– 3i
SECTION - II
and S3 {z : Re z 0}.
This section contains 4 Paragraphs each describing theory,
experiment, data etc. Eight questions relate to four paragraphs 13. Area of S =
with two questions on each paragraph. Each question of a 10 20
paragraph has only one correct answer among the four choices (a) (b)
3 3
(a), (b), (c) and (d).
Paragraphs for Questions 9 and 10 16 32
(c) (d)
Let f : [0, 1] (the set of all real numbers) be a function. Suppose 3 3
the function f is twice differentiable, f(0) = f(1) = 0 and satisfies min |1 3i z |
f (x) – 2f (x) + f (x) . ex, x [0, 1]. 14. z S
9. Which of the following is true for 0 < x < 1?
2 3 2 3
1 1 (a) (b)
(a) 0 f ( x) (b) f ( x) 2 2
2 2
3 3 3 3
1 (c) (d)
(c) f ( x) 1 (d) f ( x) 0 2 2
4
Paragraphs for Questions 15 and 16
10. If the function e–x f(x) assumes its minimum in the interval
1 A box B1 contains 1 white ball, 3 red balls and 2 black balls. Another
[0, 1] at x , which of the following is true? box B2 contains 2 white balls, 3 red balls and 4 black balls. A third
4
box B3 contains 3 white balls, 4 red balls and 5 black balls.
1 3
(a) f'( x) f ( x), x 15. If 1 ball is drawn from each of the boxes B1, B2 and B3, the
4 4 probability that all 3 drawn balls are of the same colour is
1 82 90
(b) f '( x) f ( x),0 x (a) (b)
4 648 648
1
(c) f '( x ) f ( x ),0 x 558 566
4 (c) (d)
648 648
3 16. If 2 balls are drawn (without replacement) from a randomly
(d) f'( x) f ( x), x 1
4 selected box and one of the balls is white and the other ball
Paragraphs for Questions 11 and 12 is red, the probability that these 2 balls are drawn from box
Let PQ be a focal chord of the parabola y2 = 4ax. The tangents to B2 is
the parabola at P and Q meet at a point lying on the line y = 2x + a, 116 126
a > 0. (a) (b)
181 181
11. Length of chord PQ is
(a) 7a (b) 5a (c) 2a (d) 3a 65 55
(c) (d)
181 181
EBD_7801
14-2013 JEE Advanced 2013 Solved Paper
SECTION - III
This section contains 4 multiple choice questions. Each question has matching lists. The codes for the lists have choices (a), (b), (c)
and (d) out of which ONLY ONE is correct.
17. Match List I with List II and select the correct answer using the code given below the lists :
List I List II
1/2
1 1
1 cos(tan y ) y sin(tan y)2 4 1 5
P. 2 1 1
y takes value 1.
y cot(sin y ) tan(sin y) 2 3

Q. If cosx + cosy + cosz = 0 = sinx + siny + sinz then 2. 2


x y
possible value of cos is
2

1
R. If cos x cos 2x + sinx sin 2 secx = cosx sin2x secx + 3.
4 2

cos x cos 2x then possible value of secx is


4

1
S. If cot sin 1 x2 sin tan 1
x 6 ,x 0, 4. 1

then possible value of x is


Codes:
P Q R S
(a) 4 3 1 2
(b) 4 3 2 1
(c) 3 4 2 1
(d) 3 4 1 2
18. A line L : y = mx + 3 meets y – axis at E(0, 3) and the arc of the parabola y2 = 16x, 0 y 6 at the point F(x0, y0). The tangent to the
parabola at F(x0, y0) intersects the y-axis at G(0, y1). The slope m of the line L is chosen such that the area of the triangle EFG has
a local maximum.
Match List I with List II and select the correct answer using the code given below the lists :
List I List II
1
P. m= 1.
2
Q. Maximum area of EFG is 2. 4
R. y0 = 3. 2
S. y1 = 4. 1
Codes:
P Q R S
(a) 4 1 2 3
(b) 3 4 1 2
(c) 1 3 2 4
(d) 1 3 4 2
19. Match List I with List II and select the correct answer using the code given below the lists :
List I List II
P. Volume of parallelepiped determined by vectors a , b and c is 2. 1. 100
Then the volume of the parallelepiped determined by vectors
2( a b ), 3(b c ) and 2(c a ) is
JEE Advanced 2013 Solved Paper 2013-15

Q. Volume of parallelepiped determined by vectors a , b and c is 5. 2. 30


Then the volume of the parallelepiped determined by vectors
3( a b ),3(b c ) and 2(c a ) is
R. Area of a triangle with adjacent sides determined by vectors a and 3. 24

b is 20. Then the area of the triangle with adjacent sides determined
by vectors (2a 3b ) and (a b ) is
S. Area of a parallelogram with adjacent sides determined by vectors 4. 60
a and b is 30. Then the area of the parallelogram with adjacent

sides determined by vectors (a b ) and a is


Codes:
P Q R S
(a) 4 2 3 1
(b) 2 3 1 4
(c) 3 4 1 2
(d) 1 4 3 2
20. Consider the lines
x 1 y z 3 x 4 y 3 z 3
L1 : , L2 and the planes P1 : 7x + y + 2z = 3, P2 =3x + 5y – 6z = 4.
2 1 1 1 1 2
Let ax + by + cz = d be the equation of the plane passing through the point of intersection of lines L1 and L2, and perpendicular
to planes P1 and P2.
Match List I with List II and select the correct answer using the code given below the lists :
List I List II
P. a = 1. 13
Q. b = 2. – 3
R. c = 3. 1
S. d = 4. – 2
Codes:
P Q R S
(a) 3 2 4 1
(b) 1 3 4 2
(c) 3 2 1 4
(d) 2 4 1 3
Telegram @unacademyplusdiscounts

Join Us on Telegram for More Such Books

https://telegram.me/unacademyplusdiscounts

Join us from the above link or search ''unacademyplusdiscounts'' in Telegram


EBD_7801
16-2013 JEE Advanced 2013 Solved Paper

SOLUTIONS
Paper - 1
PHYSICS For a phase difference of 2 the path difference is

SECTION - I For a phase difference of (2n+1) the path difference


2
1. (c) For a plano convex lens
is (2n 1) . option (b) is correct.
1 (µ 1) 1 1 4
...(i)
f R v u 5. (d) P1M1 = d1RT
P2M2 = d2RT
a a 3
Here µ 1.5 P1 M1 d1
m 2 2
a P2 M2 d2
3
Where a= wavelength of light in air 4 2 d1
m = wavelength of light in water 3 3 d2

R d1 8
d2 9 option (d) is correct.

6. (a) Given HI = HII


A T (A A) T
k1 t1 k2 t2
v = 8m
v 1 k1 t1
Also m u 24 cm. t2 ...(i)
u 3 k2 4
1.5 1 1 1 1 1 1
From (i) Where k1 and k2 are the equivalent conductivities in
R 8 24 8 24 6 configuration I and II respectively.
R = 3m option (c) is correct For configuration I :
2. (b) Reading = M.S.R + No of division of V.S matching the
2 3
main scale division (1MSD – 1VSD)
k1 k 2k k1 2k
2.45
= 5.10 24 0.05
50 4k
k1 ...(ii)
= 5.124 cm Option (b) is correct. 3
For configuration II :
iˆ 3 ˆ ˆi 3ˆ k2(A+A) = kA + 2kA
j . j
2 2 2 2 3k
3. (a) cos(180 2 ) k2 ...(iii)
2 2 2 2
1 3 1 3
2 2 2 2 4k
3 9
From (i) , (ii) and (iii) t 2 2sec
1 3k 4
cos(180 2 ) 180°-2
2 2
180° – 2 = 120° option (a) is correct
= 30°
3 9
option (a) is correct E P t 30 10 100 10 17 1
7. (b) p 10 kg ms
c c 3 108
4. (b) The intensity I is given as option (b) is correct
I Io cos 2
where Io is the peak intensity 8. (a) Activity B to M for particle thrown upwards
2
u 02 sin 2
Io Io v12 u 02 2( g)
Here I , I o cos 2 (2n 1) 2g
2 2 2 2
JEE Advanced 2013 Solved Paper 2013-17

SECTION - II
v12 u 02 (1 sin 2 ) u 02 cos 2
11. (b, c)
v1 = u0 cos ...(i)
y = [0.01 sin (62.8x)] cos (628 t).
2mv sin 2mv M
y
1
x 2mv cos Length of string = 5 5 0.25 m
2 20
u 20 sin 2
H
u0 2g 2
u0 62.8
m muo cos m
The midpoint M is an antinode and has the maximum
Applying conservation of linear momentum in displacement = 0.01 m
Y-direction /k
The fundamental frequency = 20Hz
2mv sin = mv1 = mu0cos ...(ii) [from (i)] 2l 2l
Applying conservation of linear momentum in 12. (a, d)
X-direction Consider the equilibrium of the system of both spheres
2mv cos = mu0 cos ...(iii) and the spring.
on dividing (ii) and (iii) we get

tan = 1 =
4
option (a) is correct 2
9. (d) Let us consider a point on the circle
The equation of circle is x2 + y2 = a2
The force is

xiˆ yjˆ 3
F K
(x 2 y2 )3/ 2 (x 2 y2 )3/ 2

y
4 3 4 3
The weight of system = R (3 )g R g
3 3
(0, a) ds
4 3
P(x, y) 4 R g
3
F This is to be balanced by the buoyant force.
yjˆ This can be possible only when the light sphere is
x completely submerged. In this way the buoyant force
xiˆ (a,0)
Fs B

xiˆ yjˆ
F K
(a 2 )3/ 2 (a 2 )3 / 2
W
K
F xiˆ yjˆ 4 4
a3 B R3 2 (2 ) g 4 R3 g
3 3
The force acts radially outwards as shown in the figure
and the displacement is tangential to the circular path. Now considering the equilibrium of the heavy sphere
Therefore the angle between the force and displacement Fs + B = W
is 90° and W = 0 Fs = W – B
option (d) is correct. 4 3 4 3
Kx R (3 )g R (2 )g
3 3
F / (2R) 2 F / R2
10. (c) y
1 / 2L 2 /L 4 R3 g
x
3 K
2
2 (a) and (d) are correct options.
1 13. (a, c)
option (c) is correct The magnetic field should be in the –z direction
(Fleming’s left hand rule)
EBD_7801
18-2013 JEE Advanced 2013 Solved Paper
vy 15. (b,d)
2
tan Step 1 : When S1 is pressed. The capacitor C1 gets
vx 2 3 charged such that its upper plate acquires a positive
charge + 2 CV0 and lower plate – 2 CV0.
Step 2 : When S2 is pressed (S1 open). As C1 = C2 the
6 charge gets distributed equal. The upper plates of
y C1 and C2 now take charge + CV0 each and lower plate
– CV0 each.
O z x (b) and (d) are correct option.
u2 = v
× × × × ×
vy = 2
× SECTION - III
× × × × × × 16. (1) For photoelectric effect
× × × × vx ×2 3 × Vo
× × × × × × h o
Vo
e e
Q,M×+ × ×
ui u 4iˆ
× × ×
× × × × × × The slope is
× × × × × × h
tan constant
e
arc speed time
angle The ratio will be 1.
radius radius
17. (4) For a radioactive decay
4 10 10 3 Mv
N N0 e t
radius
6 M 4 / QB QB
N t
50 M e
B N0
3Q
(a) and (c) are the correct options N t
1 1 e
14. (d) Electric field E1 due to smaller sphere at P is N0

4 0.693
1 R3 N0 N
t
1 3 t1 2
E1 1 e
4 0 (2R)2 N
0.04
1 e 1 (1 0.04)
+ + + + + 0.04 4% [ e x
1 x x <<1]
+ + + + + + +
+ +
+ + + + E2 P E1 + + + + 18. (5) Here K.E. = W = P × t
+ + + + + + + +
+ + + 1
2R R + + + + mv 2 P t
+ + 2
+ + + + + +
1
+ + + + + 2Pt 2 0.5 5
v 5ms 1
m 0.2
2
19. (8) Applying conservation of angular momentum
I 1 1 = I2 2
1 1 R 1R
E1 1
4 0 3 4 0 3 MR 2 1
Electric field E2 due to bigger sphere at P is 1 1 2
2
I2 1
MR 2 2[2mr 2 ]
E2 2R 2
3 0
1
50 0.4 0.4 10
As E1 1R 2R 2
E2
4 0 3 3 0 1
50 0.4 0.4 2[2 6.25 0.2 0.2]
2
1 40
4 8 rad / s
2 4 1
option (d) is correct.
JEE Advanced 2013 Solved Paper 2013-19

20. (5) Velocity at the highest point of bob tied to string 1 is 6. (b) The slow decomposition of HNO3 is represented by
acquired by the bob tied to string 2 due to elastic the eqn.
head-on collision of equal masses 4HNO3 4NO2 + 2H2O + O2
(yellow-brown)
Therefore g 1 5g 2 7. (a) The arrangement given shows octahedral void
arrangement-limiting radius ratio for octahedral void is
1
5 r
2 A
= 0.414
r
CHEMISTRY X–
rA+ = 0.414 × rX– = 0.414 × 250 = 103.5 104 pm.
SECTION - I 8. (d) The group reagent of fourth group is ammoniacal H2S
1. (c) The standard enthalpy of the combustion of glucose by which Zn2+ ion will be precipitated as ZnS, whereas
can be calculated by the eqn. Fe3+ ion and Al3+ ions will be precipitated as hydroxides.
C6H12O6(s) + 6O2(g) 6CO2(g) + 6H2O(l) 9. (b) The adsorption of methylene blue on activated charcoal
HC = 6 × Hf(CO2) + 6 × Hf (H2O) – Hf [C6H12O6] is an example of physiosorption which is exothermic,
multilayer and does not have energy barrier.
H° = 6 (–400) + 6(–300) – (–1300)
10. (a) Silver, copper and lead are commonly found in earth's
H° = –2900 kJ/mol
crust as Ag2S (silver glance), CuFeS2 (copper pyrites)
For one gram of glucose, enthalpy of combustion and PbS (galena)
2900 SECTION - II
H° = – 16.11kJ / gm
180
11. (b,c,d)
2. (b) For ideal solution, Ssystem > 0
O Ssurrounding = 0
Cl Hmixing = 0
Compounds : CH3 – Cl : Cl : Cl
12. (b, d) The pair of complex ions [Co(NH3 ) 4 Cl 2 ] + and
S P R Q [Pt(NH3)2(H2O)Cl]+ show geometrical isomerism.
Relative reactivities 1,00,000 : 200 : 79 : 0.02 The pair of complexes [Pt(NH3 ) 3 (NO 3 )]Cl and
towards SN 2 reaction [Pt(NH3)3Cl]Br show ionisation isomerism. The other
pairs given do not have same type of isomerism.
3. (d) Carbolic acid (Phenol) is weaker acid than carbonic acid 13. (a) As ester hydrolysis is first order with respect [H+].
and hence does not liberate CO2 on treatment with aq. RHA = K[H+]HA [ester]
NaHCO3 solution. RHX = K[H+]HX [ester]
4. (b) The electronic configuration of central metal ion in
complex ions P, Q and R are R HA [H ]HA
R HX [H ]HX
P = [FeF6]3–; Fe3+ :
3d 1
[H ]HA = 0.01
100
Q = [V(H2O)6]2+; V2+ H+ +
3d HA A–
1– 0.01 0.01 0.01
1
R = [Fe(H2O)6]2+; Fe2+ 0.01 0.01
3d 4
Ka 10
Higher the no. of unpaired electron(s), higher will be 1
magnetic moment. 14. (a)
Thus the correct order of spin only magnetic moment
is H H
Q<R<P |
CH2 CH2
5. (d) For P, if t50% = x
| ||
then t75% = 2x CH3– C CH3– C
This is true only for first order reaction.
| |
So, order with respect to P is 1. CH3 CH3
Further the graph shows that concentration of Q
decreases with time. So rate, with respect to Q, remains In tert butyl cation, carbon bearing positive charge has
constant. Hence, it is zero order wrt Q. one vacant p-orbital hence it is –p (empty) conjugation
So, overall order is 1+ 0 = 1 or hyperconjugation.
EBD_7801
20-2013 JEE Advanced 2013 Solved Paper

H
H CH2
CH2 –
CH–CH–CH3
CH=CH–CH3
In 2-butene, hyperconjugation is between bond.
15. (a, b, c, d)
Cl

AlCl3 –
NaH
AlCl4 ; Na H2

Aromatic
Aromatic
(P)
(Q)

NH3 100º 115ºC


–2H 2 O
from (NH4)2CO3
O O O NH OH HO OH
:

2 N N
| |
H H
Aromatic
(R)
(NH4)2CO3 2NH3 + CO2 + H2O

O OH
|| |
HCl

Cl

Aromatic
(S)
Note : P has 2 electrons, while Q, R & S have 6 electrons each. Hence all the 4 are aromatic.
SECTION - III
16. (6) Structure of melamine is as follows :
.. .. ..
H2N N NH2

N N
..

..

NH
.. 2
Total no. of lone pairs of electron is ‘6’.
O O O O O
COOH
17. (2) 1.H3 O O3 / H 2 O 2 HOOC
O –2CO 2 HOOC
COOH
O O O O O

No. of –COOH group is '2'


JEE Advanced 2013 Solved Paper 2013-21

18. (5) Since, ˆj 2kˆ and a – b


a b 3iˆ iˆ 3 ˆj 4kˆ
h h
(sin ce K.E. T) a 2iˆ ˆj 3kˆ, b iˆ 2 ˆj kˆ
mV 2M K.E
Let c PT iˆ 2 ˆj 3kˆ
1
Then the volume of parallelepipe determined by vectors
MT
For two gases, 2 1 3
c
a , b , and is = 1 2 1 = 10
He M Ne TNe 20 1000
1 2 3
Ne M He THe 4 200 = 5
3. (c) As lies on the circle (x – x0)2 + (y – y0)2 = r2
– | – z0|2 = r2
19. (8)
(I) ( z0 )( z0 ) = r2
O O –C=O
z0 z0 z0 z0 = r2
C CH2 CH |
2
2 2
(II) O N z0 z0 z0 = r2 (i)
(I) CH2
| 1
Co Also lies on the circle (x – x0)2 + (y – y0)2 = 4r2
(II) CH2
(III) O N 1
2

C CH2 CH z0 = 4r2
2
O O–C=O
(IV) 1 1
z0 z0 4r 2

Total no. of N – Co – O bonds are 8. 1 z0 z0


z0 z0 4r 2
20. (4) According to question C – Terminal must be alanine
and N – Terminal do have chiral carbon means it should
1 z0 z0
not be glycine. So possible sequence is : | z0 |2 4r 2
2 2 2
Val Phe Gly Ala | | | | | |
Val Gly Phe Ala 1 | |2 | z0 |2 z0 z0 4r 2 | |2 (ii)
Phe Val Gly Ala n
Subtracting eq (i) from (ii) we get
Phe Gly Val Ala
1–| |2 | z0 |2 (| |2 1) r 2 (4 | |2 1)

MATHEMATICS or (| |2 1)(| z0 |2 1) r 2 (4 | |2 1)

SECTION - I r2 2
Using | z0 |2 we get
2
r2
n (| |2 1) r 2 (4 | |2 1)
1. (b) cot 1 1 2k = cot–1 [1 + n(n+1)] 2
k 1
| |2 1 8 | |2 2
1 ( n 1) n 1
= tan = tan–1 (n + 1) – tan–1n | |
1 (n 1) n 7
23 c c
23
[tan 1 (n 1) tan 1
n] tan 1
24 tan 1 1 tan 1 4. (a) The intersection point of two lines is ,
n 1 25 a b a b
c c
23
1
n
1 23 25 Distance between (1, 1) and , 2 2
cot cot 1 2k cot tan a b a b
n 1 k 1 25 23
2
c
2 1 8
2. (c) Let a and b represent the adjacent sides PQ and a b
PS of the parallelogram PQRS then c
1 2 a+b–c>0
a b
EBD_7801
22-2013 JEE Advanced 2013 Solved Paper
5. (d) For some value of , point A on line PA is dy y y
(2 – 2, – – 1, 3 ) 8. (a) sec
dx x x
dy dv

= z
P(–2, –1, 0) Putting y = vx and v x , we get

3
–1 1
dx dx

+
= y
dv
2 2
x = sec v
x+

dx
dx
or cos v dv =
A N x
sin v = log x + c ( x 0)
x+y+z=3
y
sin = log x + c
As it lies on x + y + z = 3 x
2 – 2 – –1 + 3 =3
1
3 5 9 It passes through 1, C
= A 1, , 6 2
2 2 2
y 1
x 2 y 1 z sin= log x
Also equation of PN is x 2
1 1 1
9. (d) We have f (x) – 2 f(x) < 0
For some value of , N ( – 2, – 1, )
2x 2x
N lies on x + y + z = 3 = 2. e f '( x ) 2e f ( x) 0
N(0, 1, 2)
d 2x
7 5 (e f ( x )) 0
dr’s of AN are 1, , or 2, –7, 5 dx
2 2
2x
Equation of AN is e f ( x) is strictly decreasing function on
x y 1 z 2 1
= ,1
2 7 5 2
6. (a) P (atleast one of them solves the problem)
2x 1 1
= 1 – P (none of them solves it) e f ( x) e f
2
1 1 3 7 21 235
=1 1
2 4 4 8 256 256 or f ( x) e2 x 1
7. (b) The rough graph of y = sin x + cos x and Also given that f(x) is positive function so f(x) > 0
y = |cos x – sin x| suggest the required area is
0 f ( x) e2 x 1
2
= [(sin x cos x ) | cos x sin x |] dx 1 1
0
0 f ( x)dx e2 x 1dx
12 12
y
1
1 e2 x 1
y = sin x + cos x 0 f ( x)dx
12 2
12

1 1 e 1
y = |cos x – sin x| f ( x )dx 0,
12 2
x
0 /4 /2 10. (c) Let f(x) = x2 – x sin x – cos x
f (x) = 2x – x cos x = x(2 – cos x)
4 2
= 2sin x dx 2 cos x dx f is increasing on (0, ) and decreasing on (– , 0)
0 4
Also lim f(x) = , lim f(x) = and f(o) = –1
x x
= 2 ( cos x )0 4 (sin x) 24
y = f(x) meets x-axis twice.
= 2 2( 2 1) i.e., f(x) = 0 has two points in (– , ).
JEE Advanced 2013 Solved Paper 2013-23

SECTION - II is (2 , –3 , 2 )
11. (a, c) Let L = 8x, B = 15x and y be the length of square cut off For Intersection point P of and 1
from each corner. Then volume of box t + 3 = 2 , 2t – 1 = –3 , 2t + 4 = 2
= (8x – 2y) (15x – 2y)y t = –1, = 1 P( 2, –3, 2)
V = 120x2y – 46xy2 + 4y3 Any point Q on 2 is (2s + 3, 2s + 3, s + 2)
dV As per question PQ =
= 120x2 – 92xy + 12y2 17
dy
(2s + 1)2 + (2s + 6)2 + s2 = 17
dV 9s2 + 28s + 20 = 0
Now 0 at y = 5 for maximum value of V.
dy
10
[30x2 – 23xy + 3y2]y = 5 = 0 s = –2,
9
6x2 – 23x + 15 = 0
7 7 8
5 Point Q can be (–1, –1, 0) and , ,
x = 3, 9 9 9
6
For x = 3, sides are 45 and 24. 14. (b, c) We have f(x) = x sin x, x > 0
12. (a,d) Sn = –12 – 22 + 32 + 42 –52 – 62 + .... f (x) = sin x + x cos x
= (32 + 72 + 112 + ...) + (42 + 82 + 122 + ...) f (x) = 0 tan x = – x
–(12 + 52 + 92 + ...) – (22 + 62 + 102 + ...) y
n n n n
= (4r 1)2 (4 r ) 2 (4r 3) 2 (4 r 2) 2
r 1 r 1 r 1 r 1

n n x 0
= (4r 1)2 – (4r 3) 2 4 (2 r ) 2 (2 r 1) 2 1/2 1 3/2
r 1 r 1

n n
= 8 (2r 1) 4 (4r 1)
r 1 r 1 y
We observe, from graph of y = tan x and y = – x that
n(n 1) n(n 1)
= 8 2 n 4 4 n they intersect at unique point in the intervals
2 2
= 8n2 + 8n2 + 4n = 16n2 + 4n 1
(n, n + 1) and n ,n 1
For n = 8, 16n2 + 4n = 1056 2
and for n = 9, 16n2 + 4n = 1332 15. (c, d)
13. (b, d) The given lines are (a) (N M N) = (M N) N = N M N = N M N or –N M N
x 3 y 1 z 4 According as M is symm. or skew symm. correct
1: t
1 2 2 (b) (MN – NM) = (MN) – (NM) = N M – M N
x 3 y 3 z 2 = NM – MN = –(MN – NM)
2 : s
2 2 1 It is skew symm. Statement B is also correct.
Let direction ratios of be a, b, c then as 1 and 2 (c) (MN) = N M = NM MN
a + 2b + 2c = 0 Statement C is incorrect
2a + 2b + c = 0
(d) (adj M ) (adj N) = adj (MN) is incorrect.
a b c
= SECTION - III
2 3 2

x y z x2 y2
: 16. (9) Vertical line x = h, meets the ellipse 1 at
= 4 3
2 3 2
Any point on 1 is (t + 3, 2t – 1, 2t + 4) and any point on 3 3
P h, 4 h2 and Q h, 4 h2
2 2
EBD_7801
24-2013 JEE Advanced 2013 Solved Paper
By symmetry, tangents at P and Q will meet each other 17. (6) Let the coefficients of three consecutive terms of
at x-axis. (1 + x)n + 5 be
y n + 5C , n + 5C , n + 5C , then we have
r–1 r r+1
n + 5C n + 5C : n + 5C
r–1 : r r+1 = 5 : 10 : 14
n 5
P Cr 1 5 r 1
n 5 10 n 6 r 2
Cr
x or n – 3r + 6 = 0 ...(1)
0 (h, 0) R
n 1
Q Cr 10 r 1 5
Also n 5
Cr 1 14 n r 5 7
or 5n – 12r + 18 = 0 ...(2)
xh y 3 Solving (1) and (2) we get n = 6.
Tangent at P is 4 h2 1 18. (5) Given 8 vectors are
4 6
(1, 1, 1), (–1, –1, –1); (–1, 1, 1), (1, –1, –1); (1, –1, 1),
4 (–1, 1, –1); (1, 1, –1), (–1, –1, 1)
which meets x-axis at R ,0
h These are 4 diagonals of a cube and their opposites.
For 3 non coplanar vectors first we select 3 groups of
1 4 diagonals and its opposite in 4C3 ways. Then one
Area of PQR = 3 4 h2 h
2 h vector from each group can be selected in 2 × 2 × 2
ways.
3 (4 h 2 )3 2 Total ways = 4C3 × 2 × 2× 2 = 32 = 25
i.e., (h) = p=5
2 h
19. (6) Let P(E1) = x; P(E2) = y, P(E3) = z
d 4 h 2 (h 2 2) P(only E1) = x(1 – y) (1 – z) =
= 3 0 P(only E2) = (1 – x) y (1 – z) =
dh h 2
P(only E3) = (1 – x) ( 1 – y) z =
P(none) = (1 – x) (1 – y) (1 – z) = p.
(h) is a decreasing function.
Now given ( – 2 ) p =
1 1 x = 2y
h 1 max and min = (1)
2 2 and ( – 3r)p = 2 r
y = 3z
1
32 x = 6z
4
3 4 45 P ( E1 ) x
1 = 5 Hence = 6
2 1 8 P ( E3 ) z
2 20. (5) Let k, k + 1 be removed from pack.
(1 + 2 + 3 + ... + n) – (k + k + 1) = 1224
33 3 9
2 = n(n 1)
2 1 2 2k = 1225
2
8
1 8 2 = 45 – 36 = 9 n(n 1) 2450
5 k=
4
for n = 50, k = 25
k – 20 = 5

Paper - 2
PHYSICS n2 9
4.5 4.5 z 2
SECTION - I z z
we know that
nh 3h
1. (a, c)Angular momentum . Therefore n = 3.
2 2 1 1 1
R z2
ao n2 n12 n 22
Also rn 4.5a o
z
JEE Advanced 2013 Solved Paper 2013-25

4. (d) Given 2d sin =


1 1 1
4R
n12 n 22 d= cosec ... (i)
2
9 9 Differentiating the above equation w.r.t ' ' we get
For n2 = 3, n1 = 1 we get
8 4R 32R
d(d)
36 9 cosec cot
For n2 = 3, n1 = 2 we get d 2
5 4R 5R

4 1 d (d) = – cosec cot d ... (ii)


For n2 = 2, n1 = 1 we get 2
3 4R 3R on dividing (i) and (ii) we get
(a), (c) are correct options
2. (a, b, c, d) We know that dQ = m C dT in the range 0 to 100K d(d)
= cot d
From the graph, C increases linearly with temperature d
therefore the rate at which heat is absorbed varies
As increases from 0° to 90°, cot decreases and
linearly with temperature. Option (a) is correct
As the value of C is greater in the temperature range d(d)
400-500K, the heat absorbed in increasing the therefore decreases option (d) is correct
d
temperature from 0 - 100K is less than the heat required
for increasing the temperature from 400 - 500K option cos
(b) is correct. From (ii) |d (d)| = 2
From the graph it is clear that the value of C does not 2 sin
change in the temperature range 400-500K, therefore
there is no change in the rate of heat absorption in cos
This value of decreases as increases from
this range. Option (c) is correct. sin 2
As the value of C increases from 200-300K, the rate of
heat absorption increases in the range 200-300K. Option 0° to 90°
(d) is also correct. Source Observer
3. (c, d) Let us consider a point P on the overlapping region. 5. (a,b) S O
u u
The electric field intensity at P due to positively f1 f2
r1
charged sphere = Wind blows from observer tosource
3 0 case1:
(V w) u f 2 f1
f 2 f1
The electric field intensity at P due to negatively (V w) u

r2 Wind blows fromsource to observer


charged sphere = . The total electric field, case2:
3 0
(V w) u f 2 f1
f 2 f1
r1 r2 (V w) u
E r1 r 2
3 0 3 0 3 0 (a) and (b) are correct options
6. (a, d) In the region O < r < R, the magnetic field is present due
E r to current in solenoid.
3 0
Therefore the electric field is same in magnitude and
direction option (c) and (d) are correct.

P
r1 r2
+ – In the region r > 2 R, the magnetic field is present due to
r the current in the cylinder.
For the region R < r < 2R, the magnetic field is neither
along the common axis, nor tangential to the circle of
radius r. (a) and (d) are correct options.
EBD_7801
26-2013 JEE Advanced 2013 Solved Paper
7. (a, d) The particle collides elastically with rigid wall. Here 210 206 4
9. (a) 84 Po 82 Pb 2 He
V
e 1 V 0.5u 0 Here m = [209.982876 –(205.974455 + 4.002603)]× 932 MeV
0.5u 0
= 5.422 MeV = 5422 keV
u0 By conservation of linear momentum
Linear momentum of -particle = linear momentum of
lead
B p = plead
A 0.5u0 2m K.E = 2m lead K.E lead
C

Equilibrium position m lead K.E lead 206 K.E lead


K.E ...(1)
m 4
i.e. the particle rebounds with the same speed. Therefore
the particle will return to its equilibrium position with Also K.E + K.Elead = 5422 keV ...(2)
speed u0. option (a) is correct. On solving the above two equations we get
The velocity of the particle becomes 0.5u0 after time t.
Then using the equation V = Vmax cos wt we get K.E = 5319 keV
0.5u0 = u0 cos wt (a) is the correct option.
2 T 10. (c) Only in case of c we have m3 + m4 > M
T t (c) is the correct option.
3 t 6
In other cases of fission m1 + m2 M and in the other
m m case of fusion m3 + m4 M
The time period T 2 . Therefore t =
k 3 k 11. (b) Given M = L
The time taken by the particle to pass through the M = m R2
2 m M = m ( ) R2 ...(1)
equilibrium for the first time = 2t = . Therefore
3 k
Q×B
option (b) is incorrect But = ...(2)
The time taken for the maximum compression 2m
= tAB + tBA + tAC
QB BQR 2
m m m m 1 1 1 From (1) and (2) M = m R2
= + + 2m 2
3 k 3 k 3 k k 3 3 2
The negative sign shows that change is opposite to
7 m the direction of B.
= . Therefore option c is incorrect.
6 k (b) is the correct option.
The time taken for particle to pass through the
d d dB
equilibrium position second time 12. (b) E .dl = (B R 2 ) R2
dt dt dt
m m m 2 5 m
= 2 + = +1 =
3 k k k 3 3 k R 2B
option (d) is correct. E × 2 R = – R2B
1 GMm GM BR
8. (b) mv 2 = 2 v 2 E=
2 L L 2
The potential energy is a combined property of the three (b) is the correct option.
mass system. The kinetic energy of mass m is only its energy
which decreases as it moves. Ns Vs 10 Vs
13. (a) For step up transformer
(b) is the correct option. Np Vp 1 4000

Vs = 40,000 V

Np Vp 40,000 200
For step down transformer =
Ns Vs 200 1

(a) is the correct option.


JEE Advanced 2013 Solved Paper 2013-27

14. (b) We know that P = V × I (a) is the correct option


P 600 1000 18. (c) 15 15 0
I= 8 O 7 N 1
V 4000 particle
I = 150 A
238 234 4
Total resistance = 0.4 × 20 = 8 92 U 90 Th 2 He
particle
Power dissipated as heat = I2R = (150)2 × 8 = 180,000W
= 180 kW 185 184 1
83 Bi 82 Pb 1H
180 proton
% loss = 100 30%
600 239 140 99
(b) is the correct option. 94 Pu 57 La 37 X
(c) is the correct option.
15. (a) 19. (d) e f. For the ray to bend towards the normal at the
30° prism surface 2 > 1. The ray then moves away
40 sin 30° from the normal when it emerges out of the
40m rectangular block. Therefore 2 > 3.
e g. As there is no deviation of the ray as it emerges
N out of the prism, 2 = 1.
e h. As the ray emerges out of prism, it moves away
mg sin 60° from the normal. Therefore 2 < 1 . As the ray
v moves away from the normal as it emerges out
60°
of the rectangular block, therefore 2 > 3.
mg cos 60° mg
e i. At the prism surface, total internal reflection has
h2
mv2 taken place. For this sin 45° >
N – mg cos 60° = h1
r
1> 2 2.
2
mv (d) is the correct option.
N = mg cos 60° + ...(1)
r
Loss in P.E. = mg × 40 sin 30° = 200 J R PV ML 1T 2 L3
20. (c) Boltzmannn constant =
Work done in over coming friction = 150 J N nTN K
K.E. possessed by the particle = 50 J
ML2 T 2 K 1

1
mv2 50J F MLT 2
2 Coefficient of viscosity =
6 rv 1
mv2 = 100 J ...(2) L LT
1 100 ML 1T 1
From (1) and (2), N = 1 × 10 × = 5 + 2.5 = 7.5 N
2 40
(a) is the correct option. E ML2T 2
Planck constant = 1
ML 2T 1
16. (b) From (2), mv2 = 100 v T
v = 10 ms–1
(b) is the correct option. H ML2 T 2
L
Thermal conductivity = 2
17. (a) WGE = P0 (V0 – 32 V0) = – 31 P0V0 tA T T L K
WGH = P0 (8V0 – 32V0) = – 24 P0V0
= MLT–3K–1
P0 (8V0 ) 32P0 (V0 ) (c) is the correct option.
(WFH)adiabatic = 36P0 V0
5
1
3 CHEMISTRY
32V0 SECTION - I
(WFG) isothermal = 1(32 P0V0) loge
V0
1. (b) Ag 2 CrO 4 2Ag CrO42
= 32 P0V0 loge 25
= 160 P0V0 loge2 Ksp = 1.1 × 10–12 = [Ag+]2 [CrO 4 2 ]
EBD_7801
28-2013 JEE Advanced 2013 Solved Paper
1.1 × 10–12 = [0.1]2 [s]
s = 1.1 × 10–10
OH O OH
CHO
2. (b, d) CHCl3 +
OH

H3C CHCl2 (S)


(Minor)
CH3 CH3 (major)
(Q)
CHCl3 OH :CCl2 H 2 O Cl


OH O

OH +H2O

CH3 CH3

O O OH OH

CCl2 CHCl2 CHO
+: CCl2 H OH

CH3 CH3 CH3 CH3 (major)



O O O

H 2O


: CCl2 H3C CCl2 H3C CHCl2 (minor)
CH3

OH OH OH
3. (b)
Br Br Br Br
Aq. Br2 (3.0 equivalents)
SO 3
O
Br S
SO3H O H Br
O
O O
Br2 (1.0 mol)
4. (c) Reaction I : CH3COONa + CHBr3 +
CH3 CH3 Na OH
(T) (U) CH3 CH3
1.0 mol (unreacted)
O O
Br2 (1.0 mol)
Reaction II : CH3 CH 3COOH CH2Br
CH3 CH3
1.0 mol (P)
JEE Advanced 2013 Solved Paper 2013-29

5. (a, c, d)

O O
O O– –O O
Ozone is diamagnetic in nature (due to presence of paired electron) and both the O – O bond length are equal. It has a bent
structure.
6. (a, b) The reaction can occur by following two ways.
9 0 8 1
4 Be 0 4 Be 0n

9 1 2 8
4 Be 1H 1H 4 Be
7. (c, d) Al from Al2O3 and Mg from MgCO3.CaCO3 are separately extracted by electrolytic reduction.
8. (a, b, d)
(A) H = CP(rxn) T
Hence enthalpy depends on temperature.
(B) CaCO3(s) CaO(s) + CO2(g) p= PCO2
(C) Keq depends only on temperature and not on Pressure.
(D) Enthalpy of reaction is independent of the catalyst. Catalyst generaly changes activation energy.
9. (a) Lead salts give white precipitate of PbCl2 with dil. HCl which is soluble in hot water.
Pb++ + 2Cl– ——— PbCl2 (White ppt) soluble in hot water
10. (d) The filtrate on treatment with ammoniacal H2S gives a precipitate which dissolves in aqueous NaOH containing H2O2
giving a coloured solution. It contains Cr3+ ion.
3 H 2S in ammoniacal
Cr 3 NH 4OH Cr(OH)3
medium
(Green)

2Cr(OH)3 3H 2O 2 4NaOH 2Na 2CrO 4 8H 2


(yellow colour)
11. & 12.
COOH
11. (b)
HOOC COOH Cold alk. H OH
C=C KMnO 4
H H
'P' (Cis) (Syn addition) H OH

COOH
Meso (S), optically inactive

COOH COOH
H COOH Cold alk. H OH OH H
C=C KMnO 4 +
HOOC H
'Q' (trans) (Syn addition) OH H H OH

COOH COOH
T U
(Racemic Mixture), optically inactive

H 2 / Ni
12. (a) HOOC–CH=CH–COOH O

Q O
V
EBD_7801
30-2013 JEE Advanced 2013 Solved Paper

O O
OH
OH
Anhydrous AlCl3 Zn Hg
+ O
O HCl
O
O
V

H3PO 4

K L

13. (c) Pressure


N M

Volume
K L
V increasing at constant P
Hence T increases (Heating)
L M P decreasing at constant V
Hence T decreases (Cooling)
M N V decreasing at constant P
Hence T decreases (Cooling)
N K P increasing at constant V
Hence T increases (Heating)
14. (b) L to M and N to K, both are having constant volume therefore these processes are isochoric.
cold
Cl 2 2NaOH(dil.) NaOCl NaCl H 2O
15. (a) (P)

hot
3Cl2 6NaOH(conc.) NaClO3 5NaCl 3H 2O
(Q)
(P) and (Q) are salts of HOCl and HClO3 respectively.
Charcoal
16. (a) SO 2 Cl 2 SO2Cl 2
Catalyst
(R)

10SO 2 Cl 2 P4 4PCl 5 10SO 2


(R ) (S)

PCl5 4H 2 O H 3 PO 4 5HCl
(S) (T)

Warm
17. (d) (P) 2PbO2 2H 2SO4 2PbSO4 2H 2 O O 2

(Q) Na 2S2 O3 5H 2O 4Cl 2 2NaHSO 4 8HCl

(R) N 2 H 4 2I 2 N 2 4HI

(S) XeF2 2NO Xe 2NOF


JEE Advanced 2013 Solved Paper 2013-31

NaOEt
18. (a) (P) E2

EtBr
(Q) SN 2

(i) Hg(OAc) 2
(R) (ii) NaBH 4 OH (Markovnikov addition)

(i) BH 3
(S) (ii) H 2 O2 / NaOH
(Antimarkovnikov addition)

OH
19. (a) (p) C 2 H5 3 N CH 3COOH C2 H 5 3 NH CH 3COO
X Y
Initially conductivity increases because on neutralisation ions are created. After that it becomes practically constant
because X alone can not form ions.
(Q) KI 0.1M AgNO3 0.01M AgI KNO3
x y
Number of ions in the solution remains constant as only AgNO3 precipitated as AgI. Thereafter conductance in-
creases due to increase in number of ions.
(R) Initially conductance decreases due to the decrease in the number of OH ions as OH– is getting replaced by CH3COO–
which has poorer conductivity thereafter it slowly increases due to the increase in number of H+ ions.
(S) Initially it decreases due to decrease in H+ ions and then increases due to the increase in OH– ions.

0.77V 0.44V
20. (d) (P) Fe3 n 1
Fe2 n 2
Fe

xV n=3
Go 3 Go 3 Go
Fe / Fe Fe / Fe 2 Fe 2 / Fe

3 FE o 1 FE o 2 FE o
(Fe 3 / Fe) (Fe 3 / Fe 2 ) (Fe 2 / Fe)
3 × x = 1 × 0.77 + 2 × (–0.44)
0.11
x V 0.04 V.
3

(Q) 2H 2 O O 2 4H 4e E 1.23V
4e O 2 2H 2 O 4OH E 0.40 V
4H 2 O 4H 4OH E – 0.83V

(R) Cu 2 2e Cu E 0.34 V
2Cu 2Cu 2e E 0.52 V
Cu 2 Cu 2Cu E 0.18 V

x 0.91V
(S) Cr3 n 1
Cr 2 n 2
Cr

– 0.74V, n=3
x × 1 + 2 × (– 0.91) = 3 × (– 0.74)
x – 1.82 = – 2.22 x = – 0.4V
EBD_7801
32-2013 JEE Advanced 2013 Solved Paper
MATHEMATICS or (x – 3)2 + (y + 4)2 = 42
i.e. x2 + y2 – 6x – 8y + 9 = 0
SECTION - I or x2 + y2 – 6x + 8y + 9 = 0
3. (a, d)
1. (b, d)
x 5 y z
1a 2a na 1 L1 :
lim 0 3 2
n (n 1) a 1[(na 1) (na 2) (na n)] 60
x y z
L2 :
a a a 0 1 2
1 2 n
As L1, L2 are coplanar, therefore
na n n n 1
lim
n (n 1) a 1 n(n 1) 60 5 0 0
n 2a
2 0 3 2 0
a
1 n r 0 1 2
a 1
n nr 1 n 1
lim a 1 2
n (n 1) a 1 1 1 60 (5 )[6 5 2] 0
2 n (5 )( 1)( 4) 0
n a 1, 4,5.
a 1 1 r
1 nr 1 n 1 4. (b, d) Let PN = x, QL = x + 2, RM = x + 4
lim where x is an even integer.
n 1 1 1 60
1 a 1
n 2 n P
1 x x
x a dx 1
M
1 xa 1 1
N
0 0
1 60 1 60
2

x+
a (a 1) a
x+

2 2

4
Q x+2 L x+4 R
1 1
Then PM = PN = x, QN = QL = x + 2
1 60
(a 1) a and RL = RM = x + 4
2
So that PQ = 2x + 2, QR = 2x + 6, PR = 2x + 4
2a 2 3a 119 0 1
Now cos P
17 3
a 7 or
2 PQ2 PR 2 QR 2 1
2. (a, c)There can be two possibilites for the given circle as 2PQ.PR 3
shown in the figure
y (2x 2) 2 (2x 4) 2 (2x 6) 2 1
2.(2x 2).(2x 4) 3

(3, 4) 3[(x 1)2 (x 2) 2 (x 3) 2 ] 2(x 1)(x 2)


3
3 (x2 – 4) = 2(x + 1)(x + 2)
7
4 3x – 6 = 2x + 2
4
x=8
x PQ = 18, QR = 22, PR = 20

3 i
5. (c, d) w cos isin
2 6 6
(3, – 4)
n n
and w n cos
i sin
6 6
P contains all those points which lie on unit circle
The equations of circles can be
2 3
(x – 3)2 + (y – 4)2 = 42 and have arguments , , and so on.
6 6 6
JEE Advanced 2013 Solved Paper 2013-33

As z1 P H1 and z2 P H2, therefore z1 and z2 can Then we can write


have possible positions as shown in the figure.
2 x 4, x 2
1 1
Re z Re z 2
2 2 2 x 4, 2 x
3
f ( x) 2
4 x, x 0
3
z2 z1 4 x, 0 x 2
2 x 4, x 2
/6 /6
The graph of y = f(x) is as follows
/6 O /6

z2 z1

2 5
z1Oz2 can be or .
3 6
6. (a, b, c) –3 –2 2 0 2 3
3x = 4x–1 xlog 3 = 2(x – 1)log2 3
2 log 2
x From graph f(x) has local minimum at –2 and 0 and local
2 log 2 log 3
2
maximum at
2 log3 2 2 3
x
2log3 2 –1 2 log 2 3 9. (d) We have f (x) – 2f (x) +f(x) ex
f (x) – f (x) – f (x) – f (x) ex
1 1
Also x
1 1 log 4 3
1 log 2 3 e –x f (x) – e –x f (x) – e –x f (x) – e –x f (x) 1
2
7. (b, c, d) d –x d –x
e f (x) – e f (x) 1
2 3 4 dx dx

3 4 5 d –x
For n = 3, P e f (x) – e – x f (x) 1
4 5 6
dx

d d –x
e f (x) 1
0 0 0 dx dx
and P 2 0 0 0 Let g(x) = e–xf(x)
0 0 0 Then we have g (x) 1 0
It shows P2 = 0 if n is a multiple of 3. So g is concave upward.
So for P2 0, n should not be a multiple of 3 i.e. n can Also g(0) = g(1) = 0
take values 55, 58, 56 g(x) < 0, x (0, 1)
8. (a, b) For f (x) = 2 x x 2 | x 2| 2| x| e–x f(x) < 0
f(x) < 0, x (0, 1)
the critical points can be obtained by solving x 0, 10. (c) g(x) = e–xf(x)
g (x) = e–xf (x) – e–xf(x)
x 2 0 and | x 2 | 2 | x | 0 –x
= e (f (x) – f(x))
2 1
we get x = 0, –2, 2, As x is point of local minima in [0, 1]
3 4
EBD_7801
34-2013 JEE Advanced 2013 Solved Paper
1 13. (b) Area of shaded region
g (x) < 0 for x 0,
4
42 60 8 20
1 42 4
and g (x) > 0 for x ,1 4 360 3 3
4
14. (c) S1 : x2 + y2 < 16
1
In 0, , g (x) < 0 (x 1) i(y 3)
4 S2 : Im 0
1 i 3
e–x (f (x) – f (x)) < 0
f (x) < f(x) 3(x 1) (y 3) 0 y 3x 0
2 2 S3 : x > 0
2 a 2a Then S : S1 S2 S3 is as shown in the figure given
11. (b) PQ = at – 2at
t2 t below.
2 2 2
1 1 1
a t t– 4 t y
t t t
2
1 1
a t t– 4 (0, 4)
t t
x2+y2=16
2
1
a t
t
= 5a x
12. (d) As PQ is the focal chord of y2 = 4ax 60° (4, 0)
Coordinates of P and Q can be taken as
(1, – 3) (2, - 2 3)

2
R P (at , 2at)
y +x 3 = 0
O
s (a, 0)

Q
min 1 3i z = min distance between z and (1, –3)
z s
a 2a
, Clearly (from figure) minimum distance between z s
2 t
t
and (1, –3) from line y x 3 0
2a a
P(at 2 , 2at) and Q 2 , 3 3 3 3
t t
i.e. 2
3 1
Tangents at P and Q are
15. (a) Probability that all balls are of same colour
x a
y at and y –xt – = P (all red) + P (all white) + P (all black)
t t
1 3 3 4 1 2 3 2 4 5
which intersect each other at R –a, a t – 6 9 12 6 9 12 6 9 12
t
As R lies on the y = 2x + a, a > 0
82
1 1 648
a t– –2a a t– –1
t t
1W 2W 3W
1 16. (d) B1 3R B2 3R B3 4R
t 5
t 2B 4B 5B
2
Now, mOP and mOQ –2t Let E1, E2, E3 be the events that bag B1, B2 and B3 is
t 1
2 selected respectively.
2t 2 t
t t 2 5 Let E be the event that one white and one red ball is
tan
1– 4 –3 –3 selected.
JEE Advanced 2013 Solved Paper 2013-35

Then by baye’s theorem,


2sin sin x cos 2x 2sin x(cos x – sin x)
P(E \ E 2 )P(E 2 ) 4
P(E2 \ E)
P(E \ E1 )P(E1 ) P(E \ E 2 ) P(E 2 ) P(E \ E3 )P(E 3 )
1
2sin x (cos 2 x – sin 2 x) – (cos x – sin x) 0
2 3 2
9C 2 55
cos x sin x
1 3 2 3 3 4 181 2sin x cos x – sin x –1 0
6 9 12 2
C2 C2 C2
17. (b) sinx = 0 or tanx = 1 or cos x – 1
4
1
2
1 cos (tan 1
y) y sin (tan 1
y)
2 x
(P) y4 x = 0,
4
y2 cot (sin 1
y) tan (sin 1
y)
secx = 1 or 2
1 (R) (2)
2 2

cos c os 1 1
y sin sin 1 y (S) cos sin –1 1 – x 2 sin tan –1 x 6
2 2
1 1 y 1 y
2
y4
= y
1 1 y 2
1 y x x 6 5
cot cot tan tan x
y 1 y 2
1– x 2
1 6x 2 2 3
(S) (1)
Hence (P) (4), (Q) (3), (R) (2), (S) (1)
1
2 2 18. (a) Equation of tangent to y2 = 16x at F (x0, y0)
1 y2 yy0 = 8(x + x0)
1 1 y4 8x 0
= y2 y ( 1 y 2 ) G 0,
y0

1
Area of EFG = (3 y1) x0
1 2
= 1 y4 y4 2 1
1 8x
(P) (4) A x0 3 – 0
2 y0
(Q) we have cos x + cos y = – cos z
sin x + sin y = – sin z
1 y20 y 1 y30
Squaring and adding we get A 3 0 3y20
(cos x + cos y)2 + (sin x + sin y)2 = cos2 z + sin2 z 2 16 2 32 2
2 + 2 cos (x – y) = 1
dA 1 3y02
x y x y 1 6y0
4 cos 2 1 or cos dy0 32 2
2 2 2
Q (3) dA
(R) We have 0 y0 4 x0 1
dy0

cos – x cos2x + sinx sin2x secx 8 1


4 y1= 2
4
= cosx sin2x secx + cos x cos2x Also y0 = mx0 + 3
4 4 = m + 3 or m = 1
maximum area of EFG
cos 2x cos – x – cos x
4 4 1 43 1 1
3 42 48 32
= sin2x secx (cosx – sinx) 32 2 32 2
EBD_7801
36-2013 JEE Advanced 2013 Solved Paper

y 1
2a 3b a–b
2
E (0, 3)
1
–2a b 3b a
G (0, y1) F (x1, y0) 2
x 1
5a b
2
5
40 100 (R) (1)
2

(P) (4), (Q) (1), (R) (2), (S) (3) (S) a b 30

19. (c) (P) a b c 2 a b a b a 30

2 a b 3 b c c a (S) (2)
20. (a) Any point on L1 is (2 + 1, – , – 3)
6 a b b c c a and that on L2 is ( + 4, – 3, 2 –3)
For point of intersection of L1 and L2
2 2 + 1 = + 4, – = – 3, – 3 = 2 – 3
6 a b c = 6 × 4 = 24
= 2, = 1
(P) (3) Intersection point of L1 and L2 is (5, –2, – 1)
ax + by + cz = d is perpendicular to p 1 & p2
(Q) a b c 5
7a + b + 2c = 0 and 3a + 5b – 6c = 0
3 a b b c 2 c a a b c
16 48 32
6 a b b c c a
a b c
6 2 a b c = 6 × 2 × 5 = 60 1 3 2
Equation of plane is x – 3y – 2z = d
(Q) (4)
As it passes through (5, –2, –1)
1 5 + 6 + 2 = d = 13
(R) a b 20
2 a = 1, b = –3, c = –2, d = 13
or (P) (3)
a b 40 (Q) (2)
(R) (4)
(S) (1)
Telegram @unacademyplusdiscounts

Join Us on Telegram for More Such Books

https://telegram.me/unacademyplusdiscounts

Join us from the above link or search ''unacademyplusdiscounts'' in Telegram


TARGET
IIT MOCK TEST -1
PAPER - 1

GENERAL INSTRUCTIONS :

1. Section I : Q. No. 1 to 7, Q. No. 19 to 25, Q. No. 37 to 43 are Multiple Correct Choice Type questions. For this section, 4
marks will be awarded for correct answer, 1 mark for partial answer provided NO INCORRECT option is darkened and zero
mark for no answer. In all other cases, – 2 marks will be awarded.
2. Section II : Q. No. 8 to 12, Q. No. 26 to 30, Q. No. 44 to 48 are Integer Answer Type questions. For this section, 3 marks
will be awarded for correct answer and zero mark for all other cases.
3. Section III : Q. No. 13 to 18, Q. No. 31 to 36, Q. No. 49 to 54 are Passage cum Matching based Single Correct Choice Type
questions. For this section 3 marks will be awarded for correct answer and zero mark for no answer. In all other cases,
–1 mark will be awarded.

Time : 180 minutes Max. Marks : 183

..Part - A : Mathematics.. p
(c) Period of the function f (x) = sin 22x + cos42x + 2, is
4
1
SECTION – I - Multiple Correct Choice Type p
ò (1 + x)
1/ 2
(d) (1 - x)3/2 dx equals
This section contains 7 multiple choice questions. Each question 2
has 4 choices (a), (b), (c) and (d) for its answer, out of which ONE -1

OR MORE is/are correct. 3. Choose the incorrect statement


rr rr r r r
(a) If a.b = a.c Þ b = c (a ¹ 0)
1. Let f (x) = x3 + px2 + qx + 6, where p, q Î R and f ' (x) < 0 in r r r r r r r
(b) If a ´ b = a ´ c Þ b = c (a ¹ 0)
largest possible interval æç - , -1ö÷ then (p + q) is greater
5
rr rr r r r r r r r
è 3 ø (c) If a.b = a.c and a ´ b = a ´ c Þ b = c (a ¹ 0)
than r r r
(d) If v1 , v2 , v3 are non-coplanar vectors and
(a) –2 (b) 2 (c) 4 (d) 6 r r r r
2. Choose the correct options r v ´v r v ´v
n
k1 = r 2r 3r ; k 2 = r 3r 1r and
-1 æ 2 ö 3p v1 .(v 2 ´ v3 ) v1.(v2 ´ v3 )
(a) The sum å tan ç 2 ÷ equals r r
r =1 è r ø 4 r v ´v r r r 1
k3 = r 1r 2r then k1.(k 2 ´ k 3 ) = r r r
(b) lim n sin (2p 1 + n 2 - 2np) (n Î N) equals p v1.(v2 ´ v3 ) v1.(v 2 ´ v 3 )
n ®¥
Space for Rough Work
EBD_7801
MT-1 -2 Target IIT
4. Choose the correct options X Y Z W
(a) Locus of the feet of the perpendiculars drawn from the 0 0 0 0
foci on a variable tangent to the hyperbola
16y2 – 9x2 = 1 is x2 + y2 = 1/16 1 1 1 1
(b) A line passing through the point (21, 30) and normal to 2 2 2 2
the curve y = 2 x can have the slope equal to –5 3 3 3 3
(c) The magnitude of the gradient of the tangent at an 4 4 4 4
x2 y2
extremity of latus rectum of the hyperbola - =1 5 5 5 5
a 2 b2
6 6 6 6
is equal to e (where e is the eccentricity of the
hyperbola) 7 7 7 7
(d) TP and TQ are tangents to the parabola, y2 = 4ax at P
8 8 8 8
and Q. If the chord PQ passes through the fixed point
(–a, b) then the locus of T is y = 2a (x – a) 9 9 9 9
5. Equation of the plane containing the lines
r r
r = (1,1, 0) + t (1, -1, 2) , r = (2, 0, 2) + s ( -1,1, 0) is
(a) x + 3y + z = 4 (b) x + y – 2 = 0
8. (
If n > 3 and n Î N , then find the value of n Cr = Cr . )
C0 abc - C1 (a - 1)(b - 1)( c - 1) + C2 ( a - 2)(b - 2)( c - 2) +
(c) 5x – 3y – 4z = 2 (d) None of these
6. Given f an odd function periodic with period 2 continuous ¼ + ( -1) n C n ( a - n)(b - n )(c - n) .
x 9. Find the principle value of the argument of the complex
" x and g ( x ) = ò f (t ) dt , then number ii .
0 10. If sin q ¹ cos q and x, y, z satisfy the equations
(a) g (x) is odd function (b) g (2 n) = 1 x cos p - y sin p + z = cos q + 1
(c) g (2n) = 0 (d) g(x) is even function x sin p + y cos p + z = 1 - sin q
7. Let y(x) be a solution of the differential equation x cos ( p + q ) - y sin ( p + q ) + z = 2
(1 + e x ) y ¢ + ye x = 1 . If y(0) = 2, then which of the following then find the value of x2 + y2 + z2.
statement is (are) true? 11. A person has 6 friends and during a certain vacation he met
(a) y(–4) = 0 them during several dinners. He found that he dinned with
(b) y(–2) = 0 all the 6 exactly on one day, with every 5 of them on 2 days,
(c) y(x) has a critical point in the interval (–1, 0) with every 4 of them on 3 days, with every 3 on 4 days; with
(d) y(x) has no critical point in the interval (–1, 0) every 2 on 5 days. Furthers every friend was present at 7
dinners and every friend was absent at 7 dinners. Then find
SECTION – II - Integer Answer Type the number of dinner(s) he had alone.
This section contains 5 questions. The answer to each of the 12. A straight line cuts the x-axis at point A (1, 0), and y-axis at
questions is a single-digit integer, ranging from 0 to 9. The æ pö
point B, such that Ð OAB = a ç a > ÷ .C is a middle point
appropriate bubbles below the respective question numbers in the è 4ø
ORS have to be darkened. For example, if the correct answers to of AB, if B' is a mirror image of point B with respect to line OC
question numbers X, Y, Z and W (say) are 6, 0, 9 and 2, respectively, and C' is a mirror image of point C with respect to line BB',
then the correct darkening of bubbles will look like the following: then find the ratio of the areas of triangles ABB 'and BB ' C ' .
Space for Rough Work
MOCK IIT - 1 MT-1 -3

SECTION – III - Matching Type


This section contains 6 questions of Matching Type, contains two tables each having 3 columns and 4 rows. Based on each table, there
are three questions. Each question has four options (a), (b), (c) and (d) ONLY ONE of these four options is correct.
(Qs. 13–15): By appropriately matching the information given in the three columns of the following table, give the answer of the
questions that follows.
Column 1 gives information about definitions of relations.
Column 2 gives information about domain and co-domain in which given relations are either reflexive, symmetric, transitive or anti-
symmetric.
Column 3 gives information about types of given relations.
Column 1 Column 2 Column 3
(I) R = {(x, y) : 3x – y > 0} (i) N®N (P) Reflexive
(II) R = {(x, y) : y = x + 4 and x < 4} (ii) Z®Z (Q) Symmetric
(III) R = {(x, y) : x – y Î Z} (iii) R®R (R) Transitive
(IV) R = {(x, y) : 2x2 + 3y2 – 5xy = 0} (iv) A ® A where A = {1, 2, ....14} (S) Anti-symmetric
13. Which of the following options is the only correct combination?
(a) (II) (i) (R) (b) (III) (iv) (Q) (c) (I) (ii) (S) (d) (IV) (iii) (P)
14. Which of the following options is the only correct combination?
(a) (I) (iv) (Q) (b) (II) (i) (R) (c) (III) (ii) (P) (d) (IV) (iii) (R)
15. Which of the following options is the only incorrect combination?
(a) (I) (iv) (P) (b) (II) (i) (R) (c) (III) (ii) (S) (d) (IV) (iii) (P)
(Qs. 16–18): By appropriately matching the information given in the three columns of the following table, give the answer of the
questions that follows.
It is given that f (x) = log (ax2 + bx + c) where a ¹ 0 and a, b are the roots of ax2 + bx + c = 0.
Column 1 gives information about discriminant D = b2 – 4ac
Column 2 gives information about conditions on a and ax2 + bx + c
Column 3 gives information about domains of given function f (x).
Column 1 Column 2 Column 3
(I) b – 4ac > 0
2
(i) a < 0, ax2 + bx + c < 0 (P) f
(II) b2 – 4ac < 0 (ii) a > 0, ax2 + bx + c > 0 (Q) R
(III) b2 – 4ac = 0 (iii) a > 0, ax2 + bx + c > 0 (R) (– ¥, a) È (b, ¥)
(IV) b2 – 4ac > 0 (iv) a < 0, ax2 + bx + c > 0 (S) (a, b)
16. Which of the following options is the only correct combination?
(a) (I) (iii) (Q) (b) (II) (iv) (S) (c) (III) (i) (P) (d) (IV) (ii) (R)
17. Which of the following options is the only correct combination?
(a) (I) (iv) (S) (b) (I) (iii) (Q) (c) (III) (i) (R) (d) (IV) (ii) (R)
18. Which of the following options is the only incorrect combination?
(a) (II) (ii) (Q) (b) (III) (i) (P) (c) (I) (iv) (S) (d) (I) (iii) (Q)
Space for Rough Work
EBD_7801
MT-1 -4 Target IIT
22. A nonconducting disc having uniform positive charge Q, is
..Part - B : Physics..
rotating about its axis with uniform angular velocity w. The
magnetic field at the centre of the disc is
SECTION – I - Multiple Correct Choice Type
This section contains 7 multiple choice questions. Each question (a) directed outward
has 4 choices (a), (b), (c) and (d) for its answer, out of which ONE R
OR MORE is/are correct. m 0 Qw Q
(b) having magnitude O
4pR
19. If the first minima in a Young’s double slit experiment occurs
directly in front of one of the slits, (distance between slit (c) directed inwards
and screen D = 12cm and distance between slits d = 5cm.) m 0 Qw
then the wavelength of the radiation used can be (d) having magnitude
2 pR
(a) 2 cm (b) 4 cm 23. The coordinates of a particle moving in a plane are given by
(c) 2/3 cm (d) 4/3 cm x(t) = a cos (pt) and y (t) = b sin (pt) where a, b (< a) and p are
20. A small sphere of mass m suspended by a thread is first positive constants of appropriate dimensions. Then
taken aside so that the thread forms the right angle with the (a) the path of the particle is an ellipse
vertical and then released, then (b) the velocity and acceleration of the particle are normal
(a) total acceleration of sphere as a function of q is to each other at t = p/(2p)
(c) the acceleration of the particle is always directed
g 1 + 3cos 2 q towards a focus
(d) the distance travelled by the particle in time interval
(b) thread tension as a function of q is T =3mg cos q t = 0 to t = p/(2p) is a
(c) the angle q between the thread and the vertical at the 24. At time t = 0, terminal A in the circuit shown in the figure is
moment when the total acceleration vector of the sphere connected to B by a key and an alternating current I(t) =
I0cos (wt), with I0 = 1 A and w = 500 rad s–1 starts flowing in
-1
is directed horizontally is cos 1/ 3 7p
(d) the thread tension at the moment when the vertical it with the initial direction shown in the figure. At t = ,
6w
component of the sphere's velocity is maximum will be the key is switched from B to D. Now onwards only A and D
mg. are connected. A total charge Q flows from the battery to
21. A horizontal plank has a rectangular block placed on it. The charge the capacitor fully. If C = 20 mF, R = 10 W and the
battery is ideal with emf of 50 V, identify the correct
plank starts oscillating vertically and simple harmonically statement(s).
with an amplitude of 40 cm. The block just loses contact (a) Magnitude of the maximum charge on the capacitor
with the plank when the latter is at momentary rest. Then 7p
before t = is 1 × 10–3 C
(a) the period of oscillation is (2p/5) 6w
(b) the block weighs double its weight, when the plank is (b) The current in the left part of the circuit just before
at one of the positions of momentary rest 7p
t= is clockwise
(c) the block weighs 0.5 times its weight on the plank 6w
halfway up (c) Immediately after A is connected to D, the current in R
(d) the block weighs 1.5 times its weight on the plank is 10 A
halfway down extreme. (d) Q = 2 × 10–3 C

Space for Rough Work


MOCK IIT - 1 MT-1 -5
25. Which of the following statement(s) is (are) correct?
(a) The rest mass of a stable nucleus is less than the sum 20 kg
of the rest masses of its separated nucleons 10 kg
200 newton
(b) The rest mass of a stable nucleus is greater than the
sum of the rest masses of its separated nucleons
(c) In nuclear fission, energy is released by fusing two 27. A particle is moving on x-axis has potential energy
nuclei of medium mass (approximately 100 amu) U = 2 – 20x + 5x2 joules along x-axis. The particle is released at
(d) In nuclear fission, energy is released by fragmentation x = – 3. If the mass of the particle is 0.1 kg, then find how many
of a very heavy nucleus times of ten is the maximum velocity (in m/s) of the particle.
SECTION – II - Integer Answer Type 28. The length of a wire between the two ends of a sonometer is
This section contains 5 questions. The answer to each of the 105cm. If the sum of the distances of the positions of the two
questions is a single-digit integer, ranging from 0 to 9. The bridges from one end is expressed as (182 – x) cm so that the
appropriate bubbles below the respective question numbers in the fundamental frequencies of the three segments are in the
ORS have to be darkened. For example, if the correct answers to ratio of 1 : 3 : 15. Find the value of x.
question numbers X, Y, Z and W (say) are 6, 0, 9 and 2, respectively, 29. A brass rod of length 50 cm and diameter 3.0 mm is joined to a
then the correct darkening of bubbles will look like the following: steel rod of the same length and diameter. If the change in
X Y Z W length of the combined rod at 250 °C is (A × 10–2) cm, given the
0 0 0 0 original lengths are at 40.0 °C, what is the sum of digits of A?
The ends of the rod are free to expand (Co-efficient of linear
1 1 1 1
expansion of brass = 2.0 × 10–5 K–1, steel = 1.2 × 10–5 K–1 ).
2 2 2 2
30. Two long parallel wires carrying current 2.5 amperes and
3 3 3 3 1 ampere in the same direction (directed into the plane of
4 4 4 4 the paper ) are held at P and Q respectively such that they
are perpendicular to the plane of paper. The points P and Q
5 5 5 5
are located at a distance of 5 metres and 2 metres respectively
6 6 6 6 from a collinear point R (see figure). An electron moving
7 7 7 7 with a velocity of 4 × 105 m/s along the positive x- direction
experiences a force of magnitude 3.2 × 10–20 N at the point
8 8 8 8
R. Find the value of I (in ampere).
9 9 9 9
5m
26. The masses of 10 kg and 20 kg respectively are connected 2m
by a massless spring in fig. A force of 200 newton acts on P Q R
- -O
X - - - -OX --- - - - ............. ® x
the 20 kg mass. At the instant shown, the 10 kg mass has 2.5A 1A
acceleration 12 m/sec2. What is the acceleration (in m/s2) of
20 kg mass?
Space for Rough Work
EBD_7801
MT-1 -6 Target IIT

SECTION – III - Matching Type


This section contains 6 questions of Matching Type, contains two tables each having 3 columns and 4 rows. Based on each table, there
are three questions. Each question has four options (a), (b), (c) and (d) ONLY ONE of these four options is correct.
(Qs. 31-33): By appropriately matching the information given in the three columns of the following table, give the answer of the
questions that follows.
r
A charge particle of mass m moves at a speed of v. It enters a region of uniform magnetic field ( B ) at a point S and leaves the region of
uur )
the field at the point T as shown in column (II). Column (III) shows the corresponding length ST or arc length ST. Column (I) gives the
charge on particle.
Column I Column II Column III
T× × × × ×
× × × × × uur
× × × × × mv
I. Q= +q (i) (P) ST = 2
90° × × × × × qB
× × × × ×
v S

v S
× × × × ×
90° × × × × × ) 11 mv
II. Q=–q (ii) × × × × × (Q) Arc ST =
× × × × × 7 qB
× × × × ×
T

×T × × × ×
× × × × ×
× × × × × uur mv
III. Q = + 2q (iii) × × × × × (R) ST =
×S × × × × qB
45°
• • • • • •
T• • • • • •
• • • • • •
• • •O • • • ) 33 mv
IV. Q = – 2q (iv) 45°• • • • • • (S) Arc ST =
• • • • • • 7 qB
S• • • • • •
• • • • • •
31. If a proton is projected in the uniform magnetic field, which of the following combination best explains the path of the proton ?
(a) I (i) P (b) III (iii) Q (c) I (iii) P (d) IV (iv) Q
32. If an electron is projected in the uniform magnetic field, which of the following combination best explains the path of the electron ?
(a) III (iii) R (b) II (iv) S (c) II (ii) S (d) IV (i) P
33. Which of the following combination is wrongly matched ?
(a) III (i) R (b) IV (ii) Q (c) III (i) Q (d) I (iii) R
Space for Rough Work
MOCK IIT - 1 MT-1 -7
(Qs. 34-36): By appropriately matching the information given in the three columns of the following table, give the answer of the
questions that follows.
Displacement time graphs are given in column I and their, corresponding velocity–time graphs and acceleration –time graphs are given
in column II and III respectively.
Column I Column II Column III
a
I.
Forward (i) V (P)
Journey
Return
Journey t t
t

II. (ii) (Q) a


x v
t
–a
t t
Projectile

III. (iii) (R)


y v a

t t t
a

IV. (iv) (S)


x v

t t t

34. A body of mass m projected at an angle q with horizontal. Which of the following shows the correct matching for the position,
velocity and acceleration of the body with respect to time?
(a) I (i) P (b) II (ii) R (c) III (iv) Q (d) I (i) S
35. Which of the following matching represents the body moving with constant acceleration?
(a) I (i) P (b) III (iv) Q (c) IV (iii) P (d) a, b and c
36. Which is wrongly matched?
(a) IV (iv) R (b) III (iv) Q (c) II (ii) R (d) IV (iii) P

Space for Rough Work


EBD_7801
MT-1 -8 Target IIT
39. Two elements A and B form compounds having molecular
..Part - C : Chemistry..
formula AB2 and AB4. When dissolved in 20g of C6H6. 1g
of AB2 lowers the freezing point by 2.3 K, whereas 1.0g of
SECTION – I - Multiple Correct Choice Type
AB4 lowers it by 1.3 K. The molar depression constant for
This section contains 7 multiple choice questions. Each question
benzene is 5.1 K kg mol–1. Then –
has 4 choices (a), (b), (c) and (d) for its answer, out of which ONE
OR MORE is/are correct. (a) Atomic mass of A = 25.58 u
(b) Atomic mass of B = 42.64 u
37. Which of the following statement is false regarding follow-
ing reaction ? (c) Atomic mass of A = 42.64 u
(d) Atomic mass of B = 25.58 u
Cl 40. Choose the correct options for inferences drawn.
1 NO2
6 2 heat (a) Canary yellow precipitate with ammonium molybdate
+ NH3 pressure ® PO43–
5 3
4
(b) Brown ring test with dil. H2SO4 ® NO2–
Cl
(c) Yellow ppt. with HgCl2 solution ® SO42–
(a) No reaction is possible because —Cl is present on (d) Yellow ppt. with HgCl2 solution ® NO3–
benzene ring. 41. An unsaturated hydrocarbon on complete hydrogenation
(b) A nucleophilic substitution will take place in which gives 1-isopropyl-3 methylcyclohexane, after ozonolysis it
both —Cl will bereplaced by two —NH2 groups. gives one mole of formaldehyde, one mole of acetone and
(c) A nucleophilic substitution will take place in which one mole of 2,4-dioxohexanedial. The possible structures of
only —Cl attached on C1 will be replaced by —NH2. the hydrocarbon may be
(d) A nucleophilic substitution will take place in which
only —Cl attached on C4 will be replaced by —NH2.
38. Which of the following statement(s) is/are true ?

1 (a) (b)
(a) ionisation energy µ
Screening effect
(b) The first ionisation energies of Be and Mg are more
than ionisation energies of B and Al respectively
(c) Atomic and ionic radii of Niobium and Tantalum are
almost same (c) (d)
(d) Metallic and covalent radii of potassium are 2.3 Å and
2.03 Å respectively.

Space for Rough Work


MOCK IIT - 1 MT-1 -9
42. Pick out the correct statement(s) of the following 44. The dipole moment of KCl is 3.332 × 10–29 Coulomb meters
(a) Both Fe(II) and Fe(III) salts react with NO to give brown which indicates that it is a highly polar molecule. The
compound
interatomic distance between K+ and Cl– in this molecule is
(b) Fe(III) forms octahedral complexes but Fe(II) form either
2.6 ×10–10 m. The percentage ionic character of KCl is 10x
tetrahedral or square planar complexes
then find the value of x.
(c) Hexacyanoferrate(II) ion is diamagnetic but
hexacyanoferrate(III) is paramagnetic 45. At 27ºC, hydrogen is leaked through a tiny hole into a vessel
(d) [Fe(H2O)6]3+ has greater degree of paramagnetism than for 20 minutes. Another unknown gas at the same
[FeF6]4–. temperature and pressure as that of H2 is leaked through
43. A positive carbylamine test is given by
the same hole for 20 minutes. After the effusion of the gases
(a) N, N—dimethylaniline
the mixture exerts a pressure of 6 atmosphere. The hydrogen
(b) 2, 4–dimethylaniline
content of the mixture is 0.7 mole in a container of volume
(c) N–methyl-o-methylaniline 3 litres. If the molecular weight of the unknown gas is
(d) p-methylbenzylamine expressed as (1040 – A), then what is the value of A.
SECTION – II - Integer Answer Type 46. How many dipeptides are possible from two molecules of a
This section contains 5 questions. The answer to each of the typical a-amino acid ?
questions is a single-digit integer, ranging from 0 to 9. The
47. The edge length of unit cell of LiCl having rock salt type
appropriate bubbles below the respective question numbers in the
ORS have to be darkened. For example, if the correct answers to lattice is 5.14Å. If Li+ ions precisely fit into the octahedral
question numbers X, Y, Z and W (say) are 6, 0, 9 and 2, respectively, woids of closed packed structure of Cl– ions. If the ionic
then the correct darkening of bubbles will look like the following:
radius (in pm) of Cl– ions is expressed as 121x then what is
X Y Z W
the value of x?
0 0 0 0
48. We have taken a saturated solution of AgBr. Ksp of AgBr is
1 1 1 1
12 × 10–14. When 10–7 mole of AgNO3 are added to 1 litre of
2 2 2 2
this solution then, conductivity (specific conductance) of
3 3 3 3
this solution is find as 11x × 10–7 (S m–1 units). Find the
4 4 4 4 value of x. Given, molar conductance of Ag+, Br– and NO3–
5 5 5 5 are 6×10–3 Sm2mol–1, 8×10–3 Sm2mol–1 and 7×10–3 Sm2mol–1.
6 6 6 6

7 7 7 7

8 8 8 8

9 9 9 9

Space for Rough Work


EBD_7801
MT-1 -10 Target IIT

SECTION – III - Matching Type


This section contains 6 questions of Matching Type, contains two tables each having 3 columns and 4 rows. Based on each table, there
are three questions. Each question has four options (a), (b), (c) and (d) ONLY ONE of these four options is correct.

(Qs. 49-51): By appropriately matching the information given in the three columns of the following table, give the answer of the
questions that follows.
Column I contains reaction and Column II & Column III contains reagent and mechanism involved respectively.
Column I Column II Column III
(Reaction) (Reagent) (Stereochemistry)
H3C
(I) H3C — C º C — CH3 (i) D (P) Anti-addition
CH3
Me

(II) (ii) Me3CO - , D (Q) Syn-hydroxylation


N — Me

Cl Cl

(III) H H (iii) Aq. KMnO4 (R) Syn-elimination


OH OH

(IV) (iv) Li + Liq. NH3 + EtOH (S) Anti-elimination


Cl

49. Find the combination which follows the Hofmann’s rule


(a) (II) (ii) (P) (b) (III) (i) (Q) (c) (IV) (iii) (S) (d) (II) (i) (R)
50. Birch reduction is
(i) (I) (iii) (R) (b) (I) (iv) (P) (c) (IV) (i) (P) (d) (II) (i) (S)
51. The incorrect combination is
(a) (IV) (iii) (R) (b) (III) (iii) (Q) (c) (I) (iv) (P) (d) (II) (i) (R)

Space for Rough Work


MOCK IIT - 1 MT-1 -11
(Qs. 52-54): By appropriately matching the information given in the three columns of the following table, give the answer of the
questions that follows.
Bohr’s theory successfully explains the hydrogen spectrum. It also explains the spectrum of some other one-electron system (H-like
system) such as He+, Li2+, Be3+ etc. With the help of Bohr’s theory we find out, Radius of the orbit in which the electron is revolving
around the nucleus, Energy of electron in an orbits, velocity and wavelength.

Column I Column II Column III

2ο Ze 2 Z2
(I) Radius (i) (P) , ´ 313.6
nh n2

n 2 h2 é 1 1ù
(II) Velocity (ii) (Q) R ´ Z2 êê , úú
4ο mZe
2 2 2 2
êë n1 n 2 úû

2ο2 me 4 æ Ze2 ö÷1 2


(III) Energy (iii) (R) ççç ÷÷
ch 2 çè rm ø÷÷

2ο2 mZ2 e 4 n2
(IV) Wavelength (iv) , (S) ´ 0.0529
n2 h2 Z

52. The only correct combination of formula for the radius give in column I is
(a) (I) (ii) (P) (b) (I) (iii) (R) (c) (I) (ii) (S) (d) (I) (iv) (Q)
53. The only correct combination of formula for the velocity given in column I is
(a) (II) (iii) (P) (b) (II) (i) (R) (c) (II) (ii) (S) (d) (II) (iv) (Q)
54. The only correct combination of formula for the energy given in column I is
(a) (III) (i) (Q) (b) (III) (ii) (R) (c) (III) (iv) (S) (d) (III) (iv) (P)

Space for Rough Work


EBD_7801
MT-1 -12 Target IIT

PAPER - 2
GENERAL INSTRUCTIONS :

1. Section I : Q. No. 1 to 7, Q. No. 19 to 25, Q. No. 37 to 43 are Single Correct Choice Type questions. For this section, 3 marks
will be awarded for correct answer and zero mark for no answer. In all other cases, –1 mark will be awarded.
2. Section II : Q. No. 8 to 14, Q. No. 26 to 32, Q. No. 44 to 50 are Multiple Correct Choice Type questions. For this section, 4
marks will be awarded for correct answer, 1 mark for partial answer provided NO INCORRECT option is darkened and zero
mark for no answer. In all other cases, –2 marks will be awarded.
3. Section III : Q. No. 15 to 18, Q. No. 33 to 36, Q. No. 51 to 54 are Comprehension based Single Correct Choice Type questions.
For this section, 3 marks will be awarded for correct answer and zero mark for all other cases.

Time : 180 minutes Max. Marks : 183

..Part - A : Mathematics.. æ 2 tan -1 x ö


x
4. lim ç L
÷ equals e then L is equal to
SECTION – I - Single Correct Choice Type x ®¥ è p ø
This section contains 7 multiple choice questions. Each question
has 4 choices (a), (b), (c) and (d) for its answer, out of which ONLY 2 2 p
(a) (b) - (c) - (d) 1
ONE is correct. p p 2
1. The complex numbers 1 + i and 1 + 2i are both roots of the 5. Let C1 and C2 are circles defined by x2 + y2 – 20x + 64 = 0 and
equation x5 – 6x4 + Ax3 + Bx2 + Cx + D = 0, where A, B, C, D Î R. x2 + y2 + 30x + 144 = 0. The length of the shortest line
The value of D is segment PQ that is tangent to C1 at P and to C2 at Q is
(a) – 20 (b) 20 (c) 10 (d) 2 (a) 15 (b) 18 (c) 20 (d) 24
2. The set of values of x for which the identity
6. The sum of all positive integral values of ‘a’, a Î [1, 500] for
-1 -1 æ x
1 ö p which the equation [x]3 + x – a = 0 has solution is (where [ ]
cos x + cos ç + 3 - 3x 2 ÷ = holds good, is
è2 2 ø 3 denote the greatest integer function)
(a) 462 (b) 512 (c) 784 (d) 812
é 1ù é1 ù
(a) [0, 1] (b) ê0, (c) ê , 1ú (d) {–1,0,1} 7. Area enclosed by the graph of the function y = ln 2x – 1 lying
ë 2 úû ë2 û in the 4th quadrant is
dx 1 æ xq ö 2 4
3. Let
ò x 2008 + x = ln + C , where p, q, r Î N and (a) (b)
p èç 1 + x r ø÷ e e
need not be distinct, then the value of (p + q + r) equals æ 1ö æ 1ö
(c) 2 çe+ ÷ (d) 4 çe - ÷
(a) 6024 (b) 6022 (c) 6021 (d) 6020 è eø è eø

Space for Rough Work


MOCK IIT - 1 MT-1 -13
(a) E is divisible by exactly 2 primes
SECTION – II - Multiple Correct Choice Type (b) E is prime
This section contains 7 multiple choice questions. Each question (c) E ³ 30
has 4 choices (a), (b), (c) and (d) for its answer, out of which ONE (d) E < 35
OR MORE is/are correct.
é3 -1 -2ù
8. If b1 , b2 and b3 (b1 > 0) are three successive terms of a G.P. ê2 0 a ú
with common ratio r, the value of r for which the inequality 14. Let P = ê ú , where a Î . Suppose Q = [qij] is a
êë3 -5 0 úû
b3 > 4b2 – 3b1 holds, is given by
(a) r > 3 (b) r < 1 (c) 1 < r < 2 (d) 2 < r < 3 matrix such that PQ = kI, where k Î , k ¹ 0 and I is the
9. Let f : A ® B and g : B ® C be functions and gof : A ® C. k k2
Which of the following statement is true? identity matrix of order 3. If q23 = – and det(Q) = ,
8 2
(a) If gof is one-one then f and g both are one-one then
(b) If gof is one-one then if is also one-one (a) a = 0, k = 8 (b) 4a – k + 8 = 0
(c) If gof is bijection then f is one-one and g is onto (c) det (P adj (Q)) = 29 (d) det (Q adj (P)) = 213
(d) If f and g are both one-one then gof is one-one.
10. In the expansion of (x + y + z)20 SECTION – III - Comprehension Type
(a) coefficient of x7y8z7 is zero This section contains 2 paragraphs. Each paragraph has 2
(b) total number of distinct terms is 231 multiple choice questions based on a paragraph. Each question
has 4 choices (a), (b), (c) and (d) for its answer, out of which ONLY
20!x 20- r y r -k z k ONE is correct.
(c) every term is of the form
(20 - r)!(r - k)!k!
PARAGRAPH - 1
(d) sum of coefficient is 320 Let E : denotes the event that a student does his homework with
11. The locus of the mid point of the focal radii of a variable P (E) = p and F : denotes the event that he answer the question
point moving on the parabola, y2 = 4ax is a parabola whose
correctly.
(a) latus rectum is half the latus rectum of the original parabola
(b) vertex is (a/2, 0) 15. If p = 0.75, then the value of P (E/F) equals
(c) directrix is y-axis
8 10 12 15
(d) focus has the coordinate (a, 0) (a) (b) (c) (d)
12. For the hyperbola 9x2 – 16y2 – 18x + 32y – 151= 0 16 16 16 16
(a) one of the directrix is x = 21/5 16. The relation P (E/F) ³ P (E) holds good for
(b) length of the latus rectum = 9/2
(a) all values of p in [0, 1]
(c) focii are (6, 1) and (– 4, 1)
(d) eccentricity is 5/4 (b) all values of p in (0, 1) only
(c) all values of p in [0.5, 1] only
é1 1 ù é1 2 ù
13. Let E = ê + ú + ê + ú + ..... upto 50 terms, then (d) p = 0, 1
ë 3 50 û ë 3 50 û

Space for Rough Work


EBD_7801
MT-1 -14 Target IIT
PARAGRAPH - 2 20. A small quantity of solution containing Na24 radionuclide
Consider a polygon of sides n which satisfies the equation 3.nP4 (half life 15 hours) of activity 1.0 microcurie is injected into
= n–1P5. the blood of a person. A sample of the blood of volume
17. Rajdhani express travelling from Delhi to Mumbai has 10 1cm 3 taken after 5 hours shows an activity of 296
stations enroute. Number of ways in which a train can be disintegrations per minute. Determine the total volume of
stopped at 3 stations if no two of the stopping stations are blood in the body of the person. Assume that the radioactive
consecutive, is solution mixes uniformly in the blood of the person.
(a) 20 (b) 35 (1 curie = 3.7 × 1010 disintegrations per second).
(c) 56 (d) 85 (a) 5.91 litres (b) 0.91 litres
18. Number of quadrilaterals that can be made using the vertices (c) 3.21 litres (d) 4.12 litres
of the polygon of sides 10 if exactly two adjacent sides of 21. Consider the potentiometer circuit arranged as in figure.
The potentiometer wire (resistance 15r) is 600 cm long. If the
the quadrilateral are common to the sides of the n-gon, is
jockey touches the wire at a distance of 560 cm from A, what
(a) 50 (b) 60
will be the current in the galvanometer ?
(c) 70 (d) None of these
E r
..Part - B : Physics..

SECTION – I - Single Correct Choice Type


This section contains 7 multiple choice questions. Each question A
has 4 choices (a), (b), (c) and (d) for its answer, out of which ONLY N
ONE is correct.
r
19. Two blocks each of mass m lie on a smooth table. They are G
attached to two other masses as shown in the figure. The E/2
pulleys and strings are light. An object O is kept at rest on (a) 3E/11r (b) 5E/22r
the table. The sides AB and CD of the two blocks are made (c) 3E/22r (d) 5E/11r
reflecting. Find the acceleration of two images formed in 22. Three identical positive charges Q are arranged at the vertices
those two reflecting surfaces w.r.t. each other. of an equilateral triangle. The side of the triangle is a. Find
m A C m the intensity of the field at the vertex of a regular tetrahedron
of which the triangle is the base.
KQ KQ
O (a) 6 2 (b) 2 2
B D
a a
KQ
(c) 3 2 (d) None of these
a
3m 2m 23. A point source is emitting sound in all directions. Find the
ratio of distance of two points from the point source where
the difference in loudness levels is 3 dB. (log10 2 = 0.3)
(a) 17g/6 (b) 7g/6
(a) 2 (b) 3
(c) 11g/6 (d) 5g/6
(c) 1/2 (d) 1/ 2
Space for Rough Work
MOCK IIT - 1 MT-1 -15
24. 0.5 mole of an ideal gas at constant temperature 27°C kept a 3a
inside a cylinder of length L and cross-section area A closed (a) l (b) l
2g 2g
by a massless piston.
a 2a
(c) l (d) l
g 3g

SECTION – II - Multiple Correct Choice Type


This section contains 7 multiple choice questions. Each question
has 4 choices (a), (b), (c) and (d) for its answer, out of which ONE
The cylinder is attached with a conducting rod of length L , OR MORE is/are correct.
cross-section area (1/9) m2 and thermal conductivity K, 26. A double star is a system of two stars of masses m and 2m,
whose other end is maintained at 0°C. If piston is moved rotating about their centre of mass only under their mutual
such that rate of heat flow through the conducing rod is gravitational attraction. If r is the separation between these
constant then velocity of piston when it is at height L/2 two stars then their time period of rotation about their centre
from the bottom of cylinder is : of mass will be proportional to
[Neglect any kind of heat loss from system] (a) r3/2 (b) r
(c) m1/2 (d) m–1/2
æ Kö æ K ö 27. In the figure shown, the plates of a parallel plate capacitor
(a) çè ÷ø m / sec (b) çè ÷ m / sec
R 10R ø have unequal charges. Its capacitance is C. P is a point
outside the capacitor and close to the plate of charge –Q.
æ K ö æ K ö The distance between the plates is ‘d’.
(c) çè ÷ m / sec (d) çè ÷ m / sec
100R ø 1000R ø 2Q –Q

25. A U-tube of base length l filled with same volume of two


liquids of densities r and 2r is moving with an acceleration
a on the horizontal plane. If the height difference between P
the two surfaces (open to atmosphere) becomes zero, then
the height h is given by –

(a) A point charge at point 'P' will experience electric force


due to capacitor
(b) The potential difference between the plates will be
3Q/2C
h (c) The energy stored in the electric field on the region
a between the plates is 9Q2/8C
(d) The force on one plate due to the other plate is
Q2
2pe 0 d 2

Space for Rough Work


EBD_7801
MT-1 -16 Target IIT
28. Suppose the potential energy between electron and proton 31. The figure shows a system consisting of (i) a ring of outer
radius 3R rolling clockwise without slipping on a horizontal
Ke 2 surface with angular speed w and (ii) an inner disc of radius
at a distance r is given by - . Application of Bohr’ss 2R rotating anti-clockwise with angular speed w/2. The ring
3r 3
and disc are separated by frictionless ball bearings. The
theory to hydrogen atom in this case shows that point P on the inner disc is at a distance R from the origin,
(a) energy in the nth orbit is proportional to n 6 where OP makes an angle of 30° with the horizontal. Then
(b) energy is proportional to m –3 (m : mass of electron) with respect to the horizontal surface,
(c) energy in the nth orbit is proportional to n –2 z
(d) energy is proportional to m 3 (m = mass of electron) w
29. During an experiment, an ideal gas is found to obey a
P2 w/ 2
condition = constant [r = density of the gas]. The gas 3R
r x
O 2R
is initially at temperature T, pressure P and density r. The
gas expands such that density changes to r /2
(a) The pressure of the gas changes to 2P
(b) The temperature of the gas changes to 2T (a) the point O has linear velocity 3 Rw î
(c) The graph of the above process on the P-T diagram is 11 3
(b) the point P has linear velocity Rwiˆ + Rwkˆ .
parabola 4 4
(d) The graph of the above process on the P-T diagram is 13 3
hyperbola (c) the point P has linear velocity Rwiˆ - Rwkˆ
4 4
30. A parallel beam of light (l = 5000 Å) is incident at an angle a
(d) the point P has linear velocity
= 30° with the normal to the slit plane in a Young’s double
slit experiment. Assume that the intensity due to each slit at æ 3ö ˆ 1 ˆ
çè 3 - ÷ Rwi + Rwk
any point on the screen is I0. Point O is equidistant from S1 4 ø 4
and S2. The distance between slits is 1mm. 32. Two thin convex lenses of focal lengths f1 and f2 are separated
by a horizontal distance d (where d <f1, d< f2) and their centres
are displaced by a vertical separation D as shown in the fig.
S1
y

O D
o x
S2
3m d
(a) the intensity at O is 4I0 Taking the origin of coordinates O, at the centre of the first
(b) the intensity at O is zero lens the x and y coordinates of the focal point of this lens
(c) the intensity at a point on the screen 1m below O is 4I0 system, for a parallel beam of rays coming from the left, are
(d) the intensity at a point on the screen 1m below O is given by:
zero
Space for Rough Work
MOCK IIT - 1 MT-1 -17
f1 f 2 PARAGRAPH - 2
(a) x= ,y= D
f1 + f 2 Superposition of waves results in maximum and minimum of
f ( f + d) D intensities such as in case of standing waves. This phenomenon
(b) x= 1 2 ,y=
f1 + f2 - d f1 + f2 is called as interference. Another type of superposition result in
interference in time which is called as beats. In this case waves are
f1 f 2 + d ( f1 - d ) D( f1 - d )
(c) x= ,y= analyzed at a fixed point as a function of time. If the two waves are
f1 + f 2 - d f1 + f2 - d
of nearby same frequency are superimposed, at a particular point,
f1 f2 + d ( f1 - d ) intensity of combined waves gives a periodic peak and fall. This
(d) x= ,y = 0
f1 + f 2 - d phenomenon is beats. If w1 and w2 are the frequencies of two
waves then by superimposed y = y1 + y2, we get at
SECTION – III - Comprehension Type
This section contains 2 paragraphs. Each paragraph has 2 é æ w - w 2 ö ù é æ w1 + w 2 ö ù
x = 0, y = 2A cos ê ç 1 ÷ .t sin ç ÷ .t
multiple choice questions based on a paragraph. Each question
ëè 2 ø úû êëè 2 ø úû
has 4 choices (a), (b), (c) and (d) for its answer, out of which ONLY
ONE is correct. Thus amplitude frequency is small and fluctuates slowly. A
beat i.e., a maximum of intensity occurs, also intensity depends
PARAGRAPH - 1
on square of amplitude. The beat frequency is given by
The figure shows the interference pattern obtained in double-slit
experiment using light of wavelength 600nm. 1, 2, 3, 4 and 5 are w beat = | w1 - w 2 | .
marked on the five fringes. Number of beats per second is called as beat frequency.
Central bright fringe A normal ear can detect only upto 15 Hz of frequency because of
persistence of ear.
35. If two sound sources of frequency difference 25 Hz are
sounded together. Then which of the following is correct ?
(a) A normal human ear will hear 25 Hz beat frequency
12 3 4 5
33. Which fringe results from a phase difference of 4p between (b) A normal human ear will hear only 10 Hz beat frequency
the light waves emanating from two slits (c) A normal human ear can hear this frequency
(a) 2 (b) 3 (d) A normal human ear cannot hear this beat frequency
(c) 5 (d) 4
36. The frequency of beats produced when two sources of
34. Let DXA and DXC represent path differences between waves
sound are activated, one emitting wavelength 32 cm, other
interfering at 1 and 3 respectively then
32.2 cm is (Take vsound = 350 m/s)
( | DXC | – | DXA | ) is equal to
(a) 0 (b) 300nm (a) 14 (b) 18
(c) 600 nm (d) 900nm (c) 7 (d) 10

Space for Rough Work


EBD_7801
MT-1 -18 Target IIT
(iii) No precipitate of CHI3 is observed with I2 / OH-.
..Part - C : Chemistry.. (iv) Oxidation with CrO3 / pyridine gives a chiral compound.
OH H
SECTION – I - Single Correct Choice Type
This section contains 7 multiple choice questions. Each question (a) Ph (b) CH3
has 4 choices (a), (b), (c) and (d) for its answer, out of which ONLY Ph
H CH3 OH
ONE is correct.
CH 3
37. The bond dissociation enthalpies of H2(g) and N2(g) are
+ 435.95 kJ mol–1 and + 941.8 kJ mol–1 and enthalpy of (c) Ph (d) Ph OH
formation of NH3(g) is – 46.024 kJ mol–1. What is the CH3
enthalpy of atomization of NH3(g) OH
(a) 0.170 MJ mol–1 (b) 1.70 MJ mol–1 41. Consider the cell Ag(s) | AgBr(s)|Br– (aq)|| AgCl(s) | Cl– (aq)
(c) 1.170 MJ mol–1 (d) 2.130 MJ mol–1 | Ag(s) at 25ºC. The solubility product constants of AgBr &
38. In Ostwald’s process for the manufacture of nitric acid, the AgCl are respectively 5 × 10–13 & 1 × 10–10. For what ratio
first step involves the oxidation of ammonia gas by oxygen of the concentrations of Br – & Cl– ions would the emf of the
gas to give nitric oxide gas and steam. What is the maximum cell be zero ? [Given : 0.059 log 5 × 10–13 = –0.7257]
weight of nitric oxide that can be obtained starting only (a) 1 : 200 (b) 1 : 100
with 10.00 g of ammonia and 20.00 g of oxygen ? (c) 1 : 500 (d) 200 : 1
(a) 15g (b) 20g 42. Which of the following complex will show geometrical as
(c) 10g (d) 25g well as optical isomerism. [en = ethylene diamine]
39. The molar volume of liquid benzene (density=0.877 g mL–1) (a) [Pt(NH3)2Cl2] (b) [Pt(NH3)Cl4]
increases by a factor of 2750 as it vapourizes at 20ºC. At (c) [Pt(en3)]4+ (d) [Pt(en)2Cl2]
27ºC when a non-volatile solute (that does not dissociate) O
is dissolved in 54.6 cm3 of benzene, vapour pressure of this 43. O NaOCH3
solution, is found to be 98.88 mm Hg. Calculate the freezing Product
point of the solution.
Given : Enthalpy of vapourization of benzene (l) = 394.57 Jg–1. Ph CH2CH2 Br
Enthalpy of fusion of benzene (l) = 10.06 kJ mol–1 Product is/are
Molal depression constant for benzene = 5.0 K kg mol –1. O
[Given : log 100.2 = 2.00086, log 74.63 = 1.8729] O
(a) O (b)
(a) 177.65 K (b) 277.65 K
(c) 517.65 K (d) 237.15 K
Ph CH2CH2OCH3 Ph
40. An unknown compound (A) with the M.F. C9H12O does COOCH3
not decolorize Br 2 in CCl4 and is oxidised by hot KMnO4 to
O O
give PhCO2H. The compound reacts with Na to give a
colourless and odourless gas. From the following results (c) (d)
deduce the correct structure for (A).
(i) he colour of Cr2O72- changes from orange to blue- C – OCH3
Ph Ph COCH3
green. O
(ii) The compound can be resolved.

Space for Rough Work


MOCK IIT - 1 MT-1 -19
47. An aromatic chiral compound G (C8H8Br2) on treatment
SECTION – II - Multiple Correct Choice Type
with aqueous NaOH gives H (C8H9BrO). On heating ‘G’
This section contains 7 multiple choice questions. Each question
with potassium tert-butoxide I (C8H7Br) is formed. With one
has 4 choices (a), (b), (c) and (d) for its answer, out of which ONE
equivalent of methyl magnesium bromide in ether
OR MORE is/are correct. p-bromoisopropylbenzene is formed which observation/s
44. A solution of 0.2g of a compound containing Cu2+ and is/are correct about these reactions.
C2O42– ions on titration with 0.02M KMnO4 in presence of (a) (I) is an optically inactive aromatic alkene
H2SO4 consumes 22.6 mL of the oxidant. The resultant (b) H is an optically active phenol
solution is neutralized with Na2CO3, acidified with dil. acetic
acid and treated with excess KI. The liberated iodine requires
11.3 mL of 0.05 M Na2S2O3 solution for complete reduction. (c) G =
Then the correct options are
(a) Amount of C2O42– in the solution = 1.13 × 10–3 mol (d) Benzylic halide gives nucleophilic substitution with
(b) Amount of Cu2+ in the solution = 0.56 × 10–3 mol faster rate than aryl halide
(c) Amount of C2O42– in the solution = 0.56 × 10–3 mol
48. Choose the correct options for
(d) Amount of Cu2+ in the solution = 1.13 × 10–3 mol
45. Choose the correct statements CH3
(a) BeO is insoluble but BeSO4 is soluble in water. Na Me – Cl
(b) The carbon hydride of the type CnH2n+2 act as Lewis H OH a I
acid or base Red P MeONa
14CH b II
(c) Due to its high bond enthalpy, dihydrogen is not 3 I2
particularly reactive at room temperature
(d) The s-block elements are very reactive (a) I and II are identical
46. Which of the following is (are) correct statement(s) (b) I and II are different
(assuming oxidation number of metal does not affect crystal (c) Mechanism of formation of I and II are same
field energy)
(d) Mechanism of formation of I and II are different
(a) Considering H2O to be a weak ligand then on the basis
of CFSE only, we can say that [Co(H2O)6]2+ is more 49. Identify the correct statement(s) –
stable than [Co(H2O)6]3+ (a) | y | is the probability of finding the electron in an
(b) On the basis of CFSE only [Fe(NH3)6]2+ is more stable orbital
than [Fe(NH3)6]+3 (b) p-orbital is directional in nature
(c) All octahedral complexes of Ni(II) are bound to be outer (c) d has dumb bell shape along x and y-axis
x2 - y2
d-complex
(d) The type of d-orbital involved in the hybridisation for (d) d has dumb bell shape along x and y-axis
z2
a square planar complex (CN = 4) is d 2 .
z

Space for Rough Work


EBD_7801
MT-1 -20 Target IIT
50. Buffer solution A of a weak monoprotic acid and its sodium The incorrect increasing order of dipole moment of given
salt in the concentration ratio x : y has pH = (pH)1. Buffer species is –
solution B of the same acid and its sodium salt in the (a) I < II < III (b) II < I < III
concentration ratio y : x has pH = (pH)2. (c) III < II < I (d) III < I < II
If (pH)2 – (pH)1 = 1 unit and (pH)1 + (pH)2 = 9.5 units, then PARAGRAPH - 2
(a) pKa = 4.75 (b) x/y = 2.36
Consider the inter conversion of nitrosotriacetoamine into nitrogen
(c) x/y = 3.162 (d) pKa = 5.25
phorone and water.
SECTION – III - Comprehension Type
This section contains 2 paragraphs. Each paragraph has 2
multiple choice questions based on a paragraph. Each question
has 4 choices (a), (b), (c) and (d) for its answer, out of which ONLY (aq) N2(g) + H2O(l)
ONE is correct.
PARAGRAPH - 1
Paragraph for question nos. 51 to 52
The shapes of molecules can be predicted by VSEPR theory,
hybridization and dipole moment. Total number of hybrid orbitals
(H) on the central atom of a molecule can be calculated by using
the following relation :
H = [Total no. of valence electron pairs (P)–3 × (no. of atoms + (aq) – 20 kJ/ mol
surrounding the central atom, excluding Hydrogen atoms)]
One can also calculate total no. of bond pairs (n) around central
atom as n = total number of atoms surrounding the central atom
also, total no. of lone pairs (m) = H – n
The reaction is 1st order in each direction, with an equilibrium
Thus, VSEPR notation of a molecule can be written as AXnEm.
constant of 104, the activation energy for the forward reaction is
Where, A denotes central atom of the molecule.
57.45 kJ/ mol. Assuming arrhenius preexponetial factor of 1012 s–1.
X denotes bond pairs on central atom of the molecule.
E denotes lone pairs on central atom of the molecule. 53. What is the expected forward rate constant at 300K , if we
In a polar molecule, the net dipole moment of the molecule initiate this reaction starting with only reactant
µm (a) 102 (b) 106
(c) 10 8 (d) 104
51. VSEPR notation of chlorine trifluoride molecule is
(a) AX5 (b) AX3 54. If the change in entropy of the reaction is 0.07 kJ. K–1 mol–1
(c) AX2E3 (d) AX3E2 at 1 atm pressure. Calculate up to which temperature the
52. Some molecules are given below reaction would not be spontaneous. (For forward reaction)
CO2, SO2, H2O (a) T < 285.7 K (b) T > 250 K
I II III (c) T < 340.2 K (d) T > 200 K

Space for Rough Work


MOCK IIT - 1
RESPONSE SHEET MT-1 -21

Name : ..................................
MOCK TEST - 1 Date : .........................
PAPER 1 PAPER 2

1 a b c d 28 1 a b c d 28 a b c d

2 a b c d 29 2 a b c d 29 a b c d

3 a b c d 30 3 a b c d 30 a b c d

4 a b c d 31 a b c d 4 a b c d 31 a b c d

5 a b c d 32 a b c d 5 a b c d 32 a b c d

6 a b c d 33 a b c d 6 a b c d 33 a b c d

7 a b c d 34 a b c d 7 a b c d 34 a b c d

8 35 a b c d 8 a b c d 35 a b c d

9 36 a b c d 9 a b c d 36 a b c d

10 37 a b c d 10 a b c d 37 a b c d

11 38 a b c d 11 a b c d 38 a b c d

12 39 a b c d 12 a b c d 39 a b c d

13 a b c d 40 a b c d 13 a b c d 40 a b c d

14 a b c d 41 a b c d 14 a b c d 41 a b c d

15 a b c d 42 a b c d 15 a b c d 42 a b c d

16 a b c d 43 a b c d 16 a b c d 43 a b c d

17 a b c d 44 17 a b c d 44 a b c d

18 a b c d 45 18 a b c d 45 a b c d

19 a b c d 46 Space for Rough


19 Work
a b c d 46 a b c d

20 a b c d 47 20 a b c d 47 a b c d

21 a b c d 48 21 a b c d 48 a b c d

22 a b c d 49 a b c d 22 a b c d 49 a b c d

23 a b c d 50 a b c d 23 a b c d 50 a b c d

24 a b c d 51 a b c d 24 a b c d 51 a b c d

25 a b c d 52 a b c d 25 a b c d 52 a b c d

26 53 a b c d 26 a b c d 53 a b c d

27 54 a b c d 27 a b c d 54 a b c d
EBD_7801
TEST ASSESSMENT AND ANALYSIS SHEET
Mock Test - 1
Name :……………………................… Test Code : ........…………….. Date & Time of test:…………….............

Marks per PAPER 1 PAPER 2


questions Total Qs. Attempted Correct Net Score Total Qs. Attempted Correct Net Score
PHYSICS
M CQ'S
M CQ > 1 correct
Passage Cum M atching
Integer Answer
Passage Based
Physics Net Score
Sectional % Score
CHEMISTRY
M CQ'S
M CQ > 1 correct
Passage Cum M atching
Integer Answer
Passage Based
Chemistry Net Score
Sectional % Score
MATHEMATICS
M CQ'S
M CQ > 1 correct
Passage Cum M atching
Integer Answer
Passage Based
Maths Net Score
Sectional % Score
STRIKE RATE (Correct Answers/
Questions attempted):
TO TAL NET S CO RE
COMBINED NET S CORE (Paper 1 + Paper 2)

1) Analysis of wrong questions

Reasons for wrong questions No. of ques.


( to be filled after you have attempted wrong questions on your own after the test)
Knew the question and solved after test but did wrong because of calculation mistake (A)
Knew the question and solved after test but did wrong because got confused and applied wrong concept (B)
Did not knew the question and couldn’t solve even after exam (C)
Total Number of questions attempted wrong
Note : If some of (A) & (B) is high then you need more practice and also read instructions more carefully whereas if (C) is very high
it means the coverage of topic is not sufficient and you need to improve on it.

2) Analysis of not attempted questions : Divide the questions not attempted in 3 categories
Reasons for unattempted questions No. of ques.
(fill after you have tried unattempted questions on your own after the test)
Easy questions (A)
Average questions (B)
Difficult questions (C)
Total Number of ques. not attempted
Telegram @unacademyplusdiscounts

Join Us on Telegram for More Such Books

https://telegram.me/unacademyplusdiscounts

Join us from the above link or search ''unacademyplusdiscounts'' in Telegram


TARGET
IIT MOCK TEST -2
PAPER - 1

GENERAL INSTRUCTIONS :

1. Section I : Q. No. 1 to 7, Q. No. 19 to 25, Q. No. 37 to 43 are Multiple Correct Choice Type questions. For this section, 4
marks will be awarded for correct answer, 1 mark for partial answer provided NO INCORRECT option is darkened and zero
mark for no answer. In all other cases, – 2 marks will be awarded.
2. Section II : Q. No. 8 to 12, Q. No. 26 to 30, Q. No. 44 to 48 are Integer Answer Type questions. For this section, 3 marks
will be awarded for correct answer and zero mark for all other cases.
3. Section III : Q. No. 13 to 18, Q. No. 31 to 36, Q. No. 49 to 54 are Passage cum Matching based Single Correct Choice Type
questions. For this section 3 marks will be awarded for correct answer and zero mark for no answer. In all other cases, –
1 mark will be awarded.

Time : 180 minutes Max. Marks : 183

4. Let x, y be real variable satisfying the x2 + y2 + 8x – 10y – 40 = 0.


..Part - A : Mathematics.. Let a = max {(x + 2)2 + (y – 3)2} and b = min {(x + 2)2 +
(y – 3)2} then
SECTION – I - Multiple Correct Choice Type
(a) a + b = 18 (b) a+b= 4 2
This section contains 7 multiple choice questions. Each question
has 4 choices (a), (b), (c) and (d) for its answer, out of which ONE (c) a - b = 8 2 (d) a . b = 73
5. Which of the following is/are correct(s)
OR MORE is/are correct.
(sin nx) [(a - n)nx - tan x]
(a) If lim = 0 , then the value
4
æ ( log x 2) ö
2 x ®0 x2
1. The value of ò2 çç log x 2 - ln 2 ÷÷ dx is not equal to: of a is n +
1
è ø n
(a) 0 (b) 1
ìï x 2 + 2, x ³ 1
(c) –2 (d) 8
(b) If f (x) = í , then lim f (x) = 3
2. If C0, C1, ..........., Cn all binomial coefficients in expansion of ïî 2x + 1, x < 1 x ®1
(1 + x)n and a1, a2, ..........., an are in G.P. with common ratio
2x
r(¹ 0) , then C1a1 + C2a 2 + C3a 3 + ........ + Cn a n = æ a bö
(c) If lim ç 1 + + 2 ÷ = e2 , then a = 1 and b Î R
a1 x ®¥ è x x ø
(a) [(1 + r) n - 1] (d) None of these
r
(b) a1 + a1 (1 + r) + a1 (1 + r) 2 + ...... + a 1 (1 + r) n -1 é –1 2 5 ù
(c)
a1
[r n - 1] 6. The rank of the matrix êê 2 –4 a – 4 úú is
(r - 1) êë 1 –2 a + 1 úû
(d) a1[1 + r + r 2 + .......... + r n - 1] (a) 1 if a = 6 (b) 2 if a = 1
3. Let a, b > 0 then (c) 3 if a = 2 (d) 1 if a = – 6
a b 1 1 7. f ( x) is defined for x ³ 0 and has a continuous derivative.
(a) | a - b |£ |a-b| (b) 2
+ 2
³ +
b a a b It satisfies f (0) = 1, f '(0) = 0 and (1 + f ( x) ) f "( x) = 1 + x.
a b 2 2 The values f (1) can’t take is (are)
a b
(c) + ³ a+ b (d) + £1 (a) 2 (b) 1.75
b a 1 + a4 1 + b4
(c) 1.50 (d) 1.35
EBD_7801
MT-2 -2 Target IIT
8. Given
SECTION-II- Integer Answer Type
( ))
n
This section contains 5 questions. The answer to each of the ( ) (
f ( x ) = lim 2 - ( n +1) å 2 - r cosec 2 - r x cot 2 - r x then find
n ®¥
questions is a single-digit integer, ranging from 0 to 9. The r =1

appropriate bubbles below the respective question numbers in the


the value of lim x2 f ( x ) .
ORS have to be darkened. For example, if the correct answers to x® 0

question numbers X, Y, Z and W (say) are 6, 0, 9 and 2, respectively, 9. Let p (x) be a real polynomial function given by
then the correct darkening of bubbles will look like the following: p (x)= ax3 + bx2 + cx + d, such that if | p( x) | £ 1 for all x such
that | x | £ 1 then find the greatest value of
X Y Z W
| a|+|b|+ |c|+|d|.
0 0 0 0
10. The sum of the three positive numbers a , b , g is equal to
1 1 1 1
p
2 2 2 2 and if tan a , tan b , tan g form harmonic progression, find
2
3 3 3 3
the value of cot a cot g .
4 4 4 4
11. Find the positive value of m, for which area of the region
5 5 5 5 bounded by the curve y = x - x 2 and the line y = mx is equal
6 6 6 6
9
7 7 7 7 to .
2
8 8 8 8
n2
9 9 9 9 12. For a positive integer n, let f ( n) = .
n3 + 200

Find the value of n for which f ( n ) is the greatest.

SECTION-III- Matching Type


This section contains 6 questions of matching type, contains two tables each having 3 columns and 4 rows. Based on each table, there
are three questions. Each question has four options (A), (B), (C) and (D) only ONE OF these four option is correct.
(Qs. 13-15) By appropriately matching the information given in the three columns of the following table, give the answer of the
questions that follows.
Column 1 and 2 give information about the equations of straight lines and column 3 gives information about the shortest distances
between these straight lines.

Space for Rough Work


MOCK IIT - 2 MT-2 -3

Column 1 Column 2 Column 3


x-3 y-5 z-7 3 y - 1 4z - 1 2 x - 1
(I) = = (i) = = (P) 6
1 -2 1 3 6 1
1 1 1
x- y- z-
2= 3= 4 x+2 y z-7
(II) (ii) = = (Q) 9
1 2 3 -4 1 1
x -1 y - 2 z - 3 x +1 y +1 z +1
(III) = = (iii) = = (R) 0
2 3 4 7 -6 -1
x+3 y-6 z x-2 y-4 z-5
(IV) = = (iv) = = (S) 6 3
-4 3 2 3 4 5
13. Which of the following options is the only correct combination?
(a) (I) (iii) (P) (b) (II) (i) (S) (c) (III) (iv) (R) (d) (IV) (ii) (Q)
14. Which of the following options is the only correct combination?
(a) (I) (iii) (P) (b) (II) (i) (R) (c) (III) (iv) (S) (d) (IV) (ii) (R)
15. Which of the following options is the only incorrect combination?
(a) (II) (i) (R) (b) (III) (iv) (P) (c) (IV) (ii) (Q) (d) (I) (iii) (R)
(Qs. 16-18) By appropriately matching the information given in the three columns of the following table, give the answer of the
question that follows.
Column 1 gives information about the functions f (x) and g (x).
Column 2 gives information about the domain or range of fog (x) or gof (x)
Column 3 gives information about the interval which is either domain or range of fog (x) or gof (x).
Column 1 Column 2 Column 3
(I) f (x) = cot x
–1
(i) Domain of gof (x) (P) (0, ¥)
g (x) = e – x
(II) f (x) = 1 + x (ii) Domain of fog (x) (Q) R
g (x) = (ln x)2
(III) f (x) = sin x (iii) Range of gof (x) (R) (e– p, 1)
g (x) = cos– 1 x
(IV) f (x) = cot– 1 x (iv) Range of fog (x) (S) [0, 1]
g (x) = ln x
16. Which of the following options is the only correct combination?
(a) (I) (iii) (P) (b) (II) (i) (Q) (c) (III) (iv) (S) (d) (IV) (ii) (R)
17. Which of the following options is the only correct combination?
(a) (II) (iii) (R) (b) (I) (ii) (S) (c) (IV) (i) (Q) (d) (III) (iv) (P)
18. Which of the following options is the only incorrect combination?
(a) (II) (ii) (Q) (b) (III) (iv) (S) (c) (IV) (i) (Q) (d) (I) (iii) (R)

Space for Rough Work


EBD_7801
MT-2 -4 Target IIT
(a) equivalent resistance of the circuit decreases
..Part - B : Physics..
(b) ammeter reading will increase
(c) voltmeter reading will increase
SECTION – I - Multiple Correct Choice Type (d) power dissipated across R in right branch will become
This section contains 7 multiple choice quesitons. Each question zero.
has 4 choices (a), (b), (c) and (d) for its answer, out of which ONE 23. A composite block is made of slabs A, B, C, D and E of
OR MORE is/are correct. different thermal conductivities (given in terms of a constant
19. In displacement method, the distance between object and K and sizes (given in terms of length, L) as shown in the
screen is 96 cm. The ratio of length of two images formed by figure. All slabs are of same width. Heat ‘Q’ flows only from
a convex lens placed between them is 4.84. left to right through the blocks. Then in steady state
(a) Ratio of the length of object to the length of shorter
image is 11/5.
(b) Distance between the two positions of the lens is 36cm.
(c) Focal length of the lens is 22.5 cm.
(d) Distance of the lens from the shorter image is 30 cm.
20. A particle is projected vertically upwards with a velocity u
from a point O. When it returns to the point of projection
(a) its average velocity is zero (a) heat flow through A and E slabs are same.
(b) its average speed is u/2 (b) heat flow through slab E is maximum.
(c) its displacement is zero (c) temperature difference across slab E is smallest.
(d) its average speed is u. (d) heat flow through C = heat flow through B + heat flow
21. In a resonance tube experiment, a close organ pipe of length through D.
120 cm. resonates when tune with a turning fork of frequency 24. Two metallic rings A and B, identical in shape and size but
340 Hz. If water is poured in the pipe then having different resistivities rA and rB, are kept on top of
(given vair = 340 m/sec) two identical solenoids as shown in the figure. When current
(a) minimum length of water column to have the resonance I is switched on in both the solenoids in identical manner,
is 45 cm. the rings A and B jump to heights hA and hB, respectively,
(b) the distance between two successive nodes is 50 cm. with h A > hB. The possible relation(s) between their
(c) the maximum length of water column to create the reso- resistivities and their masses mA and mB is(are)
nance is 95 cm. A B
(d) None of these
22. The diagram shows a circuit with two identical resistors.
The battery has a negligible internal resistance. When switch
S is closed,

(a) rA > rB and mA = mB (b) rA< rB and mA = mB


(c) rA > rB and mA > mB (d) rA < rB and mA < mB

Space for Rough Work


MOCK IIT - 2 MT-2 -5
25. Two solid spheres A and B of equal volumes but of different 26. In the arrangement shown in figure mA = 1 kg and mB = 2 kg,
densities dA and dBare connected by a string. They are fully while all the pulleys and strings are massless and frictionless.
immersed in a fluid of density dF. They get arranged into an At t = 0, a force F = 10 t starts acting over central pulley in
equilibrium state as shown in the figure with a tension in the vertically upward direction. If the velocity of A (in m/s) when
string. The arrangement is possible only if B loses contact with floor is 2x, find the value of x
(a) dA < dF
A F=10t
(b) dB > dF
(c) dA > dF B
(d) dA + dB = 2dF

SECTION – II - Integer Answer Type


This section contains 5 questions. The answer to each of the
questions is a single-digit integer, ranging from 0 to 9. The A B
appropriate bubbles below the respective question numbers in the
ORS have to be darkened. For example, if the correct answers to 27. A spring is stretched by 0.20 metre when a mass of 0.50 kg is
question numbers X, Y, Z and W (say) are 6, 0, 9 and 2, respectively, suspended. The period of the spring when a mass of 0.25 kg
then the correct darkening of bubbles will look like the following: 11x
is suspended and put to oscillation is given by , find the
35
X Y Z W
value of x (g = 10 m/sec2).
0 0 0 0 28. An inductor of inductance 2.0 mH is connected across a
charged capacitor of capacitance 5.0 mF, and the resulting
1 1 1 1 LC circuit is set oscillating at its natural frequency. Let Q
2 2 2 2
denote the instantaneous charge on the capacitor, and I the
current in the circuit. It is found that the maximum value of Q
3 3 3 3 is 200 mC. When Q = 100 mC, the value of |dI/dt| is 10 y A/s.
Find y ?
4 4 4 4
29. Two insulated metal spheres of radii 10 cm and 15 cm charged
5 5 5 5 to a potential of 150 V and 100 V respectively, are connected
6 6 6 6 by means of a metallic wire. What is the charge on the first
sphere (in e.s.u.) ?
7 7 7 7
30. A whistling train approaches a junction. An observer
8 8 8 8 standing at junction observes the frequency to be 2.2 kHz
and 1.8 kHz of the approaching and the receding train
9 9 9 9
respectively. If the speed (in ms–1) of the train (speed of
sound = 300 m/s) is 6 B, what is the value of B ?

Space for Rough Work


EBD_7801
MT-2 -6 Target IIT

SECTION – III - Matching Type


This section contains 6 questions of Matching Type, contains two tables each having 3 columns and 4 rows. Based on each table, there
are three questions. Each question has four options (a), (b), (c) and (d) ONLY ONE of these four options is correct.
(Qs. 31-33) By appropriately matching the information given in the three columns of the following table, give the answer of the
questions that follows.
According to Bohr’s model, electron revolves in circular orbits around the nucleus under the influence of coulombic force of attraction
nh
in defined stationary orbits, for which angular momentum, mvr = . Column I, II & III give different relation between Z = atomic

number, n = orbit number, and different physical quantities like angluar velocity, energy, current, ionization energy. (Here A0, B0, C0 and
D0 are constants)
Column I Column II Column III
Z2
I. Ionisation energy of an (i) Inversely proportional to n (P) A0 (sec –1 )
n3
electron in nth Bohr’s orbit
Z
II. Current developed due to (ii) Inversely proportional to n 3 (Q) B0
n
motion of an electron
in n th orbit
Z2
III. Velocity of an electron (iii) Directly proportional to Z2 (R) C0
n3
in nth Bohr’s orbit
IV. Angular speed of an
Z2
electron in nth Bohr’s orbit (iv) Directly proportional to Z (S) D0
n2
31. If the value of current developed due to motion of an electron in 3 rd Bohr’s orbit (for Z = 3) is C0 /3 then correct matching
satisfying the above condition will be:
(a) II (iii) R (b) IV (iii) P (c) II (ii) P (d) I (iv) S
32. Which of the following shows the correct matching?
(a) II (ii) Q (b) III (i) Q (c) IV (ii) Q (d) I (i) R
33. Which of the following does not show the correct matching?
(a) IV (ii) P (b) II (iii) R (c) II (ii) R (d) I (iv) Q

Space for Rough Work


MOCK IIT - 2 MT-2 -7
(Qs. 34-36) By appropriately matching the information given in the three columns of the following table, give the answer of the
questions that follows.
Two or more resistors are said to be connected in parallel if the same potential difference exists across all resistors. Column I shows
different combination of resistances. Column II shows the equivalent resistance of the combination. A cell of emf 60 volts and internal
resistance 1 W is connected across M and N.

Column I Column II Column III


7W

I. 3W 5W (i) 2W (P) 20 A
10 W

M 10 W N

3W
3W
3W 6W
II. 6W (ii) 5W (Q) 10 A
3W

M 3W N

N
r
r r 4r 180
III. M (iii) (R) A
r 3 4r + 3

r r
M N
60
IV. (iv) r (S) A
r +1
r r

34. If resistance r is 3 W, in the combination shown in Column I (III), then the equivalent resistance of the combination and current
through the cell respectively are (4 W, 12 A); Which of the following is correct matching satisfying the condition?
(a) III (iv) S (b) III (iii) R (c) I (iv) P (d) III (ii) Q
35. Which of the following shows the correct matching?
(a) I (ii) Q (b) II (i) P (c) IV (ii) S (d) Both (a) and (b)
36. Which of the following shows the incorrect matching?
(a) III (iv) S (b) IV (iii) R (c) I (ii) Q (d) Both (a) and (b)
Space for Rough Work
EBD_7801
MT-2 -8 Target IIT

..Part - C : Chemistry.. (a) Work done A ® B is zero


(b) In path A ® B work will be done on the gas by the
surroundings
SECTION – I - Multiple Correct Choice Type (c) Volume of gas at C = 2 × Volume of gas at A
This section contains 7 multiple choice quesitons. Each question (d) Volume of gas at B is 32.8
has 4 choices (a), (b), (c) and (d) for its answer, out of which ONE
OR MORE is/are correct. Cl

37. 0.2 mol of Na3PO4 and 0.5 mol of Ba(NO3)2 are mixed in 1L of CH2 – C – Cl
solution. Which of the following is/are correct about this OH
system ? NaOH O
40. P
(a) 0.2 mol of barium phosphate precipitate is obtained
(b) 0.1 mol of barium phosphate precipitate is obtained OH
(c) Molarity of Ba2+ ions in the resulting solution is 0.2 NaOH LiAlH4
(d) Molarities of Na + and NO3– ions are 0.6 and 1.0 Q R S
respectively O O
38. Which of the following statement is/are correct
(a) R=
(a) The relative stability of carbocations follows
+ + + + O
(CH 3 )3 C > (CH3 ) 2 C H > CH3 C H 2 > C H 3
(b) The relative stability of carbocations follows O
+ + + + CH2 – C – Cl
(CH3 )3 C > CH3 C H 2 > C H3 > (CH3 ) 2 C H (b) Q =
(c) The stability of alkyl radical follows the order O
OH
& & & &
CH 3 < CH3 CH 2 < (CH3 ) 2 CH < (CH3 )3 C
(d) The stability of alkyl radical follows the order O
C=O
& & & & (c) Q=
CH 3 < (CH3 )3 C < CH3CH 2 < (CH3 ) 2 CH
39. One mole of an ideal gas is subjected to a reversible process CH2Cl
OH
that involves two steps ( A ® B and B ® C ). The pressure
OH
at A and C is same. Consider the graph and choose correct
statements. (d) S =
O – CH2 – CH2 – OH
B (2,800)
(1,200) 41. Which of the following statement(s) is/are correct?
A (a) B2O3 behaves as a base when reacted with strongly
C (1,400)] acidic oxides such as P2O5
P (atom)
(b) Orthoboric acid is a weak monobasic acid
(c) Orthoboric acid contains triangular BO33– units, hy-
drogen bonded together into two dimensional sheets
(d) Orthoboric acid is a strong dibasic acid
T (k)
Space for Rough Work
MOCK IIT - 2 MT-2 -9
42. Point out the correct statement(s) amongst the following 44. Vapour pressure (in torr) of an ideal solution of two liquids A
(a) [ Cu (CN ) 4 ]3- has square planar geometry and d 2sp and B is given by : P = 52XA + 114
hybridization
where XA is the mole fraction of A in the mixture. If the vapour
(b) [ Ni( CN ) 6 ] 4- is octahedral and Ni has d 2sp3
hybridization pressure (in torr) of equimolar mixture of the two liquids is
(c) [ZnBr4 ]2 - is tetrahedral and diamagnetic expressed as 7x + 126, then find the value of x.

(d) [Cr(NH3 )6 ]3+ has octahedral geometry and sp 3d 2 45. A metal (Atomic mass = 75 g mol –1) crystallizes in cubic
hybridization lattice, the edge length of unit cell being 5Å. The density of
43. Aryl halides are less reactive towards nucleophilic the metal is given 2g cm–3. If the radius (pm) of the metal
substitution reaction as compared to alkyl halides due to atom is expressed as 30 x + 7, then find the value of x.
(a) the formation of less stable carbonium ion
(b) resonance stabilization 46. Electrolysis of NaCl solution with inert electrodes for certain
(c) longer carbon-halogen bond
(d) sp2 hybridized carbon attached to the halogen. period of time gave 600 cm3 of 1.0 M NaOH in the electrolytic
cell. During the same period 31.80 g of copper was deposited
SECTION – II - Integer Answer Type
in a copper voltameter in series with the electrolytic cell. If
This section contains 5 questions. The answer to each of the
questions is a single-digit integer, ranging from 0 to 9. The the percent current efficiency in the electrolytic cell is
appropriate bubbles below the respective question numbers in the expressed as 10 x then find the value of x (At. wt. of Cu =
ORS have to be darkened. For example, if the correct answers to 63.6)
question numbers X, Y, Z and W (say) are 6, 0, 9 and 2, respectively,
then the correct darkening of bubbles will look like the following: 47. For the reaction R – X + OH– ® ROH + X– , the rate is given
X Y Z W as :
0 0 0 0
Rate = 5.0×10–5[R–X][OH–] + 0.20 × 10–5 [R–X].
1 1 1 1
If the percentage of R–X react by SN2 mechanism when
2 2 2 2
[OH–] = 1.0 × 10–2 M is 10 x, then find the value of x.
3 3 3 3

4 4 4 4 CH3
5 5 5 5 H+ Br , CCl
48. ¾¾¾®[F] ¾¾2¾ ¾
¾4 ® C H Br
4284 .
- H 2O 14 32
6 6 6 6 5 such products
H3C OH are possible
7 7 7 7
How many structures for F are possible?
8 8 8 8

9 9 9 9

Space for Rough Work


EBD_7801
MT-2 -10 Target IIT

SECTION – III - Matching Type


This section contains 6 questions of Matching Type, contains two tables each having 3 columns and 4 rows. Based on each table, there
are three questions. Each question has four options (a), (b), (c) and (d) ONLY ONE of these four options is correct.
(Qs. 49-51) By appropriately matching the information given in the three columns of the following table, give the answer of the
questions that follows.
Column I contains Reactant and Column II & Column III contains condition required for reaction and product respectively.

Column I Column II Column III


(Reactant) (Reaction condition) (Product)
s
(I) HC º CH (i) (i) KMnO4, OH, heat (P) C6H5NO4
Å
(ii) H , H 2O (iii) SOCl2
(iv) NH3
OH
(II) (ii) (i) Sn/HCl (ii) CH3COCl (Q) C6H5NO3
(iii) Conc. H2SO4 (iv) HNO3
s
OH (v) Dil H2SO4, heat (vi) OH
NO2
(III) (iii) (i) Red hot iron, 873K (R) C6H6N2O2
(ii) Fuming HNO3, H2SO4, heat
(iii) H2S. NH3 (iv) NaNO2,
H2SO4 (v) Hydrolysis
NO2

(IV) (iv) (i) Conc. H2SO4, 60°C (S) C7H6N2O3


(ii) Conc. HNO3, Conc. H2SO4
(iii) Dil. H2SO4, heat
CH3

49. Find suitable combination which gives m-nitro phenol as a final product.
(a) (III) (i) (Q) (b) (II) (iv) (S) (c) (I) (iii) (Q) (d) (I) (ii) (P)
50. o-nitro aniline is given by which combination
(a) (IV) (iii) (S) (b) (III) (ii) (R) (c) (II) (iv) (Q) (d) (I) (iv) (S)
51. Find correct combination
(a) (IV) (iv) (S) (b) (II) (i) (P) (c) (I) (iv) (R) (d) (IV) (i) (S)

Space for Rough Work


MOCK IIT - 2 MT-2 -11
(Qs. 52-54) By appropriately matching the information given in the three columns of the following table, give the answer of the
questions that follows.
Column I, II & III contains, cell representation, type of reaction and electrode potential respectively.

Column I Column II Column III

(I) Pt, H2 (1 atm) | H Å (10–3 M) || H Å (i) Spontaneous (P) Ecell = – 0.13V


(10–6 M) | H2 (1 atm) | Pt
(II) Pt, F2(g) (1 atm) | F s (10–2 M) (ii) Non-spontaneous (Q) Ecell = – 0.177 V
|| F s (10–3 M) | F (g) (2 atm) | Pt
2
(III) Hg, Hg2Cl2(s) | KCl saturated solution (iii) Exergonic (R) Ecell = 0.582 V
|| H Å (pH = 10) | Q, QH2 | Pt

æ E Q|H 2O = 0.7V ö
çç ÷÷
è ESCE = 0.24V ø

(IV) Q + 2 H Å (pH = 2) + 2e– ¾¾


® H2O(aq) (iv) Endergonic (S) Ecell = 0.069 V

E ocell = 0.7

52. For the cell given in column I, the only correct combination is
(a) (I) (ii) (P) (b) (I) (ii) (Q) (c) (I) (iv) (R) (d) (I) (iv) (S)
53. For the cell given in column I, the only correct combination is
(a) (III) (iv) (P) (b) (III) (iv) (Q) (c) (III) (ii) (R) (d) (III) (ii) (S)
54. For the cell given in column I, the only correct combination is
(a) (IV) (iii) (S) (b) (IV) (iii) (R) (c) (IV) (i) (Q) (d) (IV) (ii) (P)

Space for Rough Work


EBD_7801
MT-2 -12 Target IIT

PAPER - 2
GENERAL INSTRUCTIONS :

1. Section I : Q. No. 1 to 7, Q. No. 19 to 25, Q. No. 37 to 43 are Single Correct Choice Type questions. For this section, 3 marks
will be awarded for correct answer and zero mark for no answer. In all other cases, –1 mark will be awarded.
2. Section II : Q. No. 8 to 14, Q. No. 26 to 32, Q. No. 44 to 50 are Multiple Correct Choice Type questions. For this section, 4
marks will be awarded for correct answer, 1 mark for partial answer provided NO INCORRECT option is darkened and zero
mark for no answer. In all other cases, –2 marks will be awarded.
3. Section III : Q. No. 15 to 18, Q. No. 33 to 36, Q. No. 51 to 54 are Comprehension based Single Correct Choice Type questions.
For this section, 3 marks will be awarded for correct answer and zero mark for all other cases.

Time : 180 minutes Max. Marks : 183

..Part - A : Mathematics.. (a) 2n - 2 [na1 + Sn ] (b) 2n [a1 + Sn ]

SECTION – I - Single Correct Choice Type (c) 2[na1 + Sn ] (d) 2n -1[a1 + Sn ]


This section contains 7 multiple choice questions. Each question 4. Find the coefficient of x50 in the expression :
has 4 choices (a), (b), (c) and (d) for its answer, out of which ONLY (1 + x)1000 + 2x(1 + x)999 + 3x2 (1 + x)998 + . . . + 1001. x1000.
ONE is correct. (a) 1008 C55 (b) 1002 C52
1. The function f : R ® R, defined by f (x) = [x], " x Î R, is (c) 1000 C40 (d) 1002 C50
(a) one-one 5. If a, b and c are distinct positive real numbers and
(b) onto a2 + b2 + c2 = 1, then ab + bc + ca is
(c) both one-one and onto (a) less than 1 (b) equal to 1
(d) neither one-one nor onto (c) greater than 1 (d) any real number
2. If x = a + b, y = aw + bw2 and z = aw2 + bw, then x3 + y3 is equal 6. Number of integers satisfying either
to log3 | x | < 2 or | log3 x | < 2 are
(a) 6 (a3 + b3) (b) 3 (a3 + b3) (a) 18 (b) 16
3
(c) a + b 3 (d) None of these (c) 20 (d) 23
7. If the straight line ax + by = 2 ; a, b ¹ 0 touches the circle
3. If a1, a2 , a3 ,..., an are in A.P. with Sn as the sum of first ‘n’ x2 + y2 – 2x = 3 and is normal to the circle x2+y2–4y= 6,then
n the values of a and b are respectively
terms, then å
k =0
n
Ck S k is equal to (a) 1, – 1
(c) –4/3, 1
(b) 1, 2
(d) 2, 1

Space for Rough Work


MOCK IIT - 2 MT-2 -13

SECTION – II - Multiple Correct Choice Type (a) 2n In+1 = 2–n + (2n – 1) In (b) I 2 = p + 1
This section contains 7 multiple choice questions. Each question 8 4
has 4 choices (a), (b), (c) and (d) for its answer, out of which ONE p 1 p 5
OR MORE is/are correct. (c) I2 = - (d) I3 = -
8 4 16 48
8. The values of x between 0 and 2p, which satisfy the equation r r r r
13. If 2a , -3b , 2(a ´ b ) are position vectors of the vertices
2
sin x 8 cos x = 1 are in A.P. Choose the correct options r r uur uuur
A, B, C of DABC and | a | = 1,| b | = 1, OA ·OB = -3 (where
(a) common difference of A.P. is p/4
(b) x = p/8 O is the origin), then
(c) x = 3p/8 (a) triangle ABC is right angled triangle
(d) x = 5p/8 (b) angle B is 90°
9. The straight lines x + y = 0, 3x + y – 4 = 0 and x + 3y – 4 = 0 æ 7ö
from a triangle which is (c) A = cos-1 ç ÷
(a) isosceles (b) right-angled è 19 ø
(c) obtuse-angled (d) equilateral (d) The position vector of orthocentre is 2(a ´ b ).
| x - 1|
10. The function f(x) = 14. The quadratic expression 21 + 12x – 4x2 takes
x2 (a) the least value 5 (b) the greatest value 30
(a) increases in (– ¥ ,0) È (1, 2) (c) the greatest value 21 (d) none of these
(b) increases in (0, 1) È (2, ¥)
(c) decreases in (0, 1) È (2, ¥) SECTION – III - Comprehension Type
(d) decreases in (–¥, ¥) È (1, 2) This section contains 2 paragraphs. Each paragraph has
2 multiple choice questions based on a paragraph. Each question
d2 x
11. The differential equation
2
+ y + cot 2 x = 0 must be has 4 choices (a), (b), (c) and (d) for its answer, out of which ONLY
dy
ONE is correct.
satisfied by
PARAGRAPH - 1
(a) 2 + c1 cos x + c 2 sin x
The sides of a triangle ABC satisfy the relations a+b– c = 2 and
æ xö 2ab – c2 = 4 and f (x) = ax2 + bx + c.
(b) cos x.l n ç tan ÷ + 2 15. Area of the triangle ABC in square units, is
è 2ø
3
æ xö (a) 3 (b)
(c) 2 + c1 cos x + c2 sin x + cos x ln ç tan ÷ 4
è 2ø
(d) all of the above 9 3
(c) (d) 4 3
1
dx 4
12. If In = ò 2 n
, n Î N , then which of the following 16. If x Î [0, 1] then maximum value of f (x) is
0 (1 + x ) (a) 3/2 (b) 2
statements hold good (c) 3 (d) 6

Space for Rough Work


EBD_7801
MT-2 -14 Target IIT
PARAGRAPH - 2 20. In the figure a charged sphere of mass m and charge q starts
A jar contains 2n throughly mixed balls, n white and n black balls. sliding from rest on a vertical fixed circular track of radius R
n persons each of whom draw 2 balls simultaneously from the bag from the position shown. There exists a uniform and constant
without replacement. horizontal magnetic field of induction B. The maximum force
17. If the probability that each of the n person draw both balls exerted by the track on the sphere.
of different colours is 8/35, then the value of n equals m
(a) 3mg - qB 2gR q
(a) 3 (b) 4 q
(c) 5 (d) 6
18. If n = 4 the probability that each of the 4 persons draw both (b) 2mg - qB 2gR × B
balls of the same colour is equal to (c) mg - 2qB 2gR
(a) 1/35 (b) 2/35
(c) 3/35 (d) 4/35 (d) mg - qB 3gR
21. The displacement of a particle is given at time t, by:
Part - B : Physics..
x = A sin(-2wt) + Bsin 2 wt . Then
SECTION – I - Single Correct Choice Type (a) the motion of the particle is SHM with an amplitude of
This section contains 7 multiple choice questions. Each question
B2
has 4 choices (a), (b), (c) and (d) for its answer, out of which ONLY A2 +
4
ONE is correct.
(b) the motion of the particle is not SHM, but oscillatory
19. The following figure represents a wavefront AB which with a time period of T = p/w
passes from air to another transparent medium and produces (c) the motion of the particle is oscillatory with a time period
a new wavefront CD after refraction. The refractive index of of T = p/2w
the medium is (PQ is the boundary between air and the (d) the motion of the particle is a periodic.
22. A sphere of radius r and mass m has a linear velocity v0
medium) B
directed to the left and no angular velocity as it is placed on
a belt moving to the right with a constant velocity v1. If after
air sliding on the belt the sphere is to have no linear velocity
relative to the ground as it starts rolling on the belt without
P
sliding, in terms of v1 and the coefficient of kinetic friction
A 1 3
µk between the sphere and the belt, determine the required
4 D medium Q
2 value of v0

v0
C
cos q1 cos q 4
(a) (b) v1
cos q4 cos q1
sin q1 sin q 2 (a) (1/5) v1 (b) (3/5) v1
(c) sin q 4 (d) sin q3 (c) (2/5) v1 (d) 3v1

Space for Rough Work


MOCK IIT - 2 MT-2 -15
23. When 0.50 Å X-rays strike a material, the photoelectrons (c) If the rays are incident at the curved surface (figure (ii))
from the K- shell are observed to move in a circle of radius 23 then these are focussed at distance 18 cm from point P
mm in a magnetic field of 2 × 10–2 T. What is the binding to the right.
energy of K-shell electrons? (d) If the rays are incident at the curved surface (figure (ii))
(a) 6.2 keV (b) 0.2 keV then these are focussed at distance 14 cm from point P
(c) 4.2 keV (d) 3.2 keV to the right.
24. A potentiometer wire of length L and resistance 10 W has a 27. Let vr , vrms and vp respectively denote the mean speed, the
battery of 2.5V and a resistance in series in its primary circuit. root-mean-square speed, and the most probable speed of
The null point for a cell of emf 1 volt comes at L/2 distance the molecules in an ideal monoatomic gas at absolute tem-
from one end. If the series resistance in the primary circuit is perature T. The mass of a molecule is m.
(a) No molecule can have speed greater than vrms.
doubled, then position of the new null point is
vp
(a) 0.5 L (b) 0.6 L (b) No molecule can have speed less than .
(c) 1.0 L (d) None of the above 2
r
25. Two infinitely long straight parallel wires are joined at equal (c) v p < v < vrms
intervals of distance 2l by cross wires of length l each. The 3
wires all have the same resistance per unit length equal to (d) The average kinetic energy of a molecule is mv2p
4
R/l. A current enters and leaves the network at the ends of 28. Two infinite, parallel, non-conducting sheets carry equal
one of the cross wires. The equivalent resistance of the positive charge density s. One is placed in the yz plane and
network is the other at distance x = a. Take potential V = 0 at x = 0
(a) R (b) R / 2 (a) For 0 £ x £ a , potential Vx = 0
(c) R / 3 (d) 2R s
(b) For x ³ a , potential Vx = - (x - a)
Î0
SECTION – II - Multiple Correct Choice Type s
(c) For x ³ a , potential Vx = Î (x - a)
This section contains 7 multiple choice questions. Each question 0
has 4 choices (a), (b), (c) and (d) for its answer, out of which ONE s
(d) For x £ 0 potential Vx = Î x
OR MORE is/are correct. 0 ×
29. In the figure shown R is a fixed B
26. A parallel beam of light is incident normally on the flat surface conducting ring of negligible P × Q
of a hemisphere of radius 6 cm and refractive index 1.5, placed resistance and radius ‘a’. PQ is a r
in air as shown in figure (i). Assume paraxial ray uniform rod of resistance r. R
approximation. It is hinged at the centre of the ring and rotated about this
point in clockwise direction with a uniform angular velocity
w. B is a uniform magnetic field of strength B pointing
6cm P P inwards. ‘r’ is a stationary resistance.
(a) Current through r is zero
figure(i) figure(ii) (b) Current through r is 2Bwa2/5r
(a) The rays are focussed at 12 cm from the point P to the (c) Direction of current in external r is from centre to
right, in the situation as shown in figure (i). circumference
(b) The rays are focussed at 16 cm from the point P to the (d) Direction of current in external r is from circumference
right, in the situation as shown in figure (i). to centre.

Space for Rough Work


EBD_7801
MT-2 -16 Target IIT
30. A cylindrical vessel of 90 cm height is kept filled upto the (a) The gravitational
y
brim. It has four holes 1, 2, 3, 4 which are respectively at force due to this
heights of 20 cm, 30 cm, 40 cm and 50 cm from the horizontal object at the origin is
floor PQ. zero.
(b) the gravitational force A
-m
x
O B
at the point B (2, 0 ,0)
4 is zero.
z
3
(c) the gravitational potential is the same at all points of
2 circle y2 + z2 = 36.
1 (d) the gravitational potential is the same at all points on
the circle y2 + z2 = 4.
P Q
The water falling at the maximum horizontal distance from SECTION – III - Comprehension Type
the vessel comes from This section contains 2 paragraphs. Each paragraph has
(a) hole number 4 (b) hole number 3 2 multiple choice questions based on a paragraph. Each question
(c) hole number 2 (d) hole number 1 has 4 choices (a), (b), (c) and (d) for its answer, out of which ONLY
31. In an experiment to determine the acceleration due to gravity ONE is correct.
g, the formula used for the time period of a periodic motion is
PARAGRAPH - 1
7 R r If an object is placed at a distance u from the pole of a spherical
T 2 . The values of R and r are measured to be
5g mirror and its image is formed at a distance v (from the pole) then
(60 ± 1) mm and (10 ± 1) mm, respectively. In five successive 1 1 1 2
measurements, the time period is found to be 0.52s, 0.56s, + = =
v u f R
0.57s, 0.54s and 0.59s. The least count of the watch used for
the measurement of time period is 0.01s. Which of the follow- In this formula to calculate unknown, known quantities are
ing statement(s) is (are) true? substituted with proper sign.
(a) The error in the measurement of r is 10% If a thin object linear size O situated vertically on the axis of a
(b) The error in the measurement of T is 3.75% mirror at a distance u from the pole and its image of size I is formed
(c) The error in the measurement of T is 2% at a distance v (from the pole) magnification (transverse) is defined
as the ratio of the size of image to the size of object
(d) The error in the determined value of g is 11%
32. A solid sphere of uniform density and radius 4 units is located (+ve Erect image)
with its centre at the origin O of coordinates. Two spheres of
equal radii 1 unit, with their centres at A (–2, 0 ,0) and éIù é vù (–ve inverted image)
m= ê ú =– êë u úû
B (2, 0, 0) respectively, are taken out of the solid leaving ë Oû (|m| >1 large image)
behind spherical cavities as shown in fig. (|m| < 1 Small image)
Then :

Space for Rough Work


MOCK IIT - 2 MT-2 -17
Here -ve magnification implies that image is inverted with respect 34. As shown in figure, an object O is at the position (–10, 2)
to object while +ve magnification means that image is erect with with respect to the origin P. The concave mirror M1 has
respect to object. radius of curvature 30 cm. A plane mirror M2 is kept at a
However, if the 1-D object is placed with its length along the distance 40 cm infront of the concave mirror.
principal axis, the so called longitudinal magnification becomes Considering first reflection on the concave mirror M1 and
second on the plane mirror M2. Find the coordinates of the
I (v - v ) dv
mL = =- 2 1 =- second image w.r.t. the origin P.
O (u 2 - u1 ) du
Relation between Object and Image Velocity : 40cm
y
1 1 1
Differentiating equation + =
v u f O
x
45° P
1 dv 1 du 1 1
Þ -
2 dt
- 2 = 0 Þ - 2 VIM - 2 VOM = 0
v u dt v u
M2 M1
v2
Þ VIM = - VOM Þ V = – m2V
(a) (– 46, – 70) (b) (– 30, – 60)
u2 IM OM
(c) (– 20, – 50) (d) (– 35, – 65)
where, VIM = velocity of image w.r.t. mirror and
VOM = velocity of object w.r.t. mirror PARAGRAPH - 2
The PV diagram for a cyclic process performed on an
33. A particle revolves in clockwise direction (as seen from point
ideal monoatomic gas is shown in figure. The curve AC is one
A) in a circle C of radius 1 cm and completes one revolution
fourth of circle .
in 2 sec. The axis of the circle and the principal axis of the
mirror M coincide. Call it AB.
The radius of curvature of the mirror is 20 cm. Then the
direction of revolution (as seen from A) of the image of the
particle and its speed is –

35. Work done by the gas in process A to B is :


(a) –100 J (b) +100 J
(c) – 200 J (d) + 200 J
10cm M 36. Heat absorbed by gas in cyclic process is :
(a) Clockwise, 3.14 m/s (b) Clockwise, 1.57 cm/s (a) –25p J (b) 25p J
(c) Anticlockwise, 1.57 m/s (d) Anticlockwise, 3.14 m/s (c) 100p J (d) –100p J

Space for Rough Work


EBD_7801
MT-2 -18 Target IIT
40. Calculate the overall complex dissociation equilibrium
Part - C : Chemistry..
constant for the [Cu(NH3)4]2+ ion, given that b4 for this
complex is 2.1 × 1013
SECTION – I - Single Correct Choice Type (a) 4.7 × 10–14 (b) 1.7 × 10–16
This section contains 7 multiple choice questions. Each question –11
(c) 0.7 × 10 (d) 8.7 × 10–14
has 4 choices (a), (b), (c) and (d) for its answer, out of which ONLY
41. An excess of granular zinc was added to 500 mL of 1M nickel
ONE is correct.
nitrate till equilibrium was established. Find out the
37. A gas bulb of 1 litre capacity contains 2.0 × 1021 molecules concentration of nickel at the equilibrium, if the standard
of nitrogen exerting a pressure of 7.57 × 103 Nm–2. If the ratio electrode potential of Zn 2+ / Zn and Ni2+/Ni are
of most probable speed to the root mean square speed is – 0.75 and – 0.24 V, respectively. [Given : log 1.8 = 0.25]
0.84, calculate the most probable speed for these molecules (a) 5.56 × 10–18 M (b) 0.56 × 10–18 M
at this temperature. –18
(c) 2.16 × 10 M (d) 4.12 × 10–18 M
(a) 425.14 m s–1 (b) 405.14 m s–1
42. When metal X is treated with sodium hydroxide, a white
(c) 415.14 m s–1 (d) 515.14 m s–1 precipitate (A) is obtained, which is soluble in excess of
38. What happens when 2-butyne reacts with H2 in presence of NaOH to give soluble complex (B). Compound (A) is soluble
Nickle Boride or lindlar’s catalyst (Pd/CaCO3—PbO)- in dilute HCl to form compound (C). The compound (A)
(a) CH3 — C — H when heated strongly gives (D), which is used to extract
|| metal. Identify (X).
CH3 — C — H (a) Aluminium (b) Gallium
(c) Indium (d) Thallium
(b) CH3 — C — H
|| 43. It has been observed that writing on paper with an invisible
H — C — CH3 ink containing ammonium thiocyanate becomes visible as
red colour when the paper is brushed with an aqueous FeCl3.
(c) CH3—CH2—CH2—CH3 If ammonium thiocyanate is first made alkaline instead of red
(d) CH 2 = CH colour, you get an orange colour which is less clear.
| The above observation can be explained best by which of
CH = CH 2 the following combinations
39. The following data were obtained during the first order (a) Fe – CNS complex is formed in case of ammonium
thermal decomposition of SO2Cl2 at a constant volume. thiocyanate and Fe (OH)3 is formed in case of alkaline
SO2Cl2(g) ® SO2(g) + Cl2(g) ammonium thiocyanate
Experiment Time/s Total pressure/atm (b) Fe – CNS complex is formed in case of ammonium
1 0 0.5 thiocyanate and Fe(OH)2 is formed in case of alkaline
2 100 0.6 ammonium thiocyanate
Calculate the rate of the reaction when total pressure is 0.65 (c) In both cases same complex is formed but some of it
atm. [Given : log 5 = 0.699. log 4 = 0.602] gets dissolved in case of alkaline ammonium
(a) 7.8 × 10–4 s–1 atm. (b) 0.8 × 10–4 s–1 atm. thiocyanate
–2 –1
(c) 2.4 × 10 s atm. (d) 6.1 × 10–8 s–1 atm. (d) None of the above is correct

Space for Rough Work


MOCK IIT - 2 MT-2 -19
46. The following statement is/are true for Cannizzaro reaction
SECTION – II - Multiple Correct Choice Type
(a) The aldehyde is oxidised as well as reduced
This section contains 7 multiple choice questions. Each question (b) The aldehydes not containing a-Hydrogen atoms give
has 4 choices (a), (b), (c) and (d) for its answer, out of which ONE the reaction
OR MORE is/are correct. (c) The reaction is not given by aldehydes containing
a-Hydrogen atoms.
44. Given following series of reactions (unbalanced) :
(d) In crossed Cannizzaro reaction of H2CO and PhCHO,
(I) NH3 + O2 ¾¾
® NO + H 2 O H2CO does not oxidized.
47. An aromatic compound ‘A’ on treatment with aqueous
(II) NO + O2 ¾¾
® NO 2 ammonia and heating forms compound ‘B’ which on heating
with Br2 and KOH forms a compound ‘C’ of molecular formula
(III) NO 2 + H 2 O ¾¾
® HNO3 + HNO 2
C6H7N. Then choose the correct options
(IV) HNO 2 ¾¾
® HNO3 + NO + H 2 O (a) A is Benzoic acid (b) B is Benzamide
(c) C is aromatic amine (d) C is aniline
Select the correct option (s) – 48. Choose the correct statements
(a) Moles of HNO3 obtained is half of moles of Ammonia used (a) In Lassaigne’s method the appearance of a bluish green
if HNO2 is not used to produce HNO3 by reaction (IV) or a blue colouration, confirms the presence of nitrogen
100 in the organic compound.
(b) % more HNO3 will be produced if HNO2 is used to (b) In Beilstein test appearance of green or bluish green
6
colour of the flame indicates the presence of halogens.
produce HNO3 by reaction (IV) than if HNO2 is not
(c) If organic compound is fused with sodium peroxide
used to produce HNO3 by reaction (IV)
(Na2O2) and the mass is extracted with water and
(c) If HNO2 is used to produce HNO3 then (1/4)th of total extract is boiled with conc. HNO3 and then ammonium
HNO3 is produced by reaction (IV) molybedate solution is added. The appearance of yellow
(d) Moles of NO produced in reaction (IV) is 50% of moles precipitate or colouration shows the presence of
of total HNO3 produced phosphorus.
45. Anhydrous barium nitrate when heated decomposes and (d) In sodium nitroprusside test the appearance of violet
oxygen and NO2 gas is evolved. Similarly magnesium nitrate colouration indicates nitrogen.
when heated decomposes to give out NO2 gas and oxygen. 49. Which of the following trends of ionisation energy are
In both cases corresponding oxides are also formed. Select correct?
the correct answer(s) : (a) O > N > F (b) F > N > O
(a) The lattice energy value is higher for magnesium nitrate (c) Na < Al < Mg (d) P > S > Si
than that of barium nitrate 50. Amongst the following, the correct statement(s) is/are:
(b) NO2 will be evolved at a lower temperature in case of (a) NO has one unpaired electron in the antibonding
Mg(NO3)2 as compared to that Ba(NO3)2 molecular orbital
(c) NO2 will be evolved at a lower temperature in case of (b) NO+ is more stable than O + 2
Ba (NO3)2 as compared to that of Mg(NO3)2
(d) In both cases [i.e., heating of Mg(NO3)2] the ratio of (c) OF+ is more paramagnetic than Ne+ 2
volume of NO2 and O2 evolved is 4 : 1 (d) A pi bond is concentrated along the bond axis.

Space for Rough Work


EBD_7801
MT-2 -20 Target IIT
Ketonic hydrolysis
SECTION – III - Comprehension Type (1) KOH (dil.)
This section contains 2 paragraphs. Each paragraph has CH3COCHRCOOC2H5 ¾¾¾¾¾¾¾¾¾
®
2 multiple choice questions based on a paragraph. Each question (2) H 2SO 4
has 4 choices (a), (b), (c) and (d) for its answer, out of which ONLY CH3 - C - CH 2 - R + CO 2 + EtOH
ONE is correct. ||
O
PARAGRAPH - 1 Acid hydrolysis
There are set of 4 quantum numbers : conc. KOH
(i) Principal quantum number is denoted by n. The values of n CH3COCHRCOOC2H5 ¾¾¾¾¾ ®
ranges from 1 to n. It gives information about the size and CH3COOK + RCHCOOK + EtOH
energy level of major energy shells. The above names are in agreement to the type of products ob-
For one electron species, the mathematical expression of tained.
53. What is the final product S in the given reaction
-2 p 2 me4 z 2
energy is E n = CH3COCH2COOC2H5 ¾¾¾¾¾¾
®P
EtONa (1mole)
n 2h 2
(ii) Azimuthal or angular quantum number is denoted by l . EtI
¾¾® Q (1) KOH
¾¾¾¾¾¾¾¾
® S
Its value ranges from 0 to (n – 1) for s, p, d, f sub-shells (2) H 3PO 4
respectively. It gives information about shape, energy level (a) CH3COOH
of sub-shell and orbital angular momentum of the electron, (b) CH3 - C - CH 2 - CH 2 - CH3
é h ù ||
ê l(l + 1) 2p ú . O
ë û (c) CH3 - C - CH - CH3
(iii) Magnetic quantum number is denoted by m. The values of
|| |
m ranges from – l to + l . It gives information about possible O CH3
number of orientations of sub-shells. (d) CH3 - C - Et
(iv) Spin quantum number is denoted by s. The values of s are ||
1 1 O
and - . It signifies the direction of spin of electrons in 54. Which reaction sequence can prepare succinic acid as
2 2
an orbital. final product
51. For an electron in f sub-shell, the orbital angular momentum EtONa (1eq)
(a) CH3COCH2COOEt ¾¾¾¾¾¾ ®
is
Cl-CH2COOEt KOH
(a) 2 h / 2p (b) 3 h / 2p ¾¾¾¾¾¾¾¾ ® ¾¾¾¾®

(c) 12 h / 2p (d) 2 h/2p EtONa (1eq)
52. The angular momentum l of an electron in a Bohr’s orbital (b) CH3COCH2COOEt ¾¾¾¾¾¾ ®
is given as Cl -CH2COOH KOH
¾¾¾¾¾¾¾¾ ® ¾¾¾¾®
nh h HÅ
(a) L = (b) L = l(l + 1)
2p 2p (c) CH3COCH2COOEt ¾¾¾¾¾¾
EtONa (1eq)
®
h h
(c) L = l(l + 2) (d) L = Cl-CH2COOMe Conc. KOH
2p 4p ¾¾¾¾¾¾¾¾® ¾¾¾¾¾ ®
EtONa (1eq)
PARAGRAPH - 2 (d) CH3COCH2COOEt ¾¾¾¾¾¾ ®
Alkyl derivatives of aceto acetic ester can undergo two types of Conc. KOH
CH3 - C - Cl ¾¾¾¾¾ ®
hydrolysis, ketonic and acid hydrolysis. The scheme of these ||
hydrolysis reactions are as follows : O
Space for Rough Work
MOCK IIT - 2
RESPONSE SHEET MT-2 -21

Name : ..................................
MOCK TEST - 2 Date : .........................
PAPER 1 PAPER 2

1 a b c d 28 1 a b c d 28 a b c d

2 a b c d 29 2 a b c d 29 a b c d

3 a b c d 30 3 a b c d 30 a b c d

4 a b c d 31 a b c d 4 a b c d 31 a b c d

5 a b c d 32 a b c d 5 a b c d 32 a b c d

6 a b c d 33 a b c d 6 a b c d 33 a b c d

7 a b c d 34 a b c d 7 a b c d 34 a b c d

8 35 a b c d 8 a b c d 35 a b c d

9 36 a b c d 9 a b c d 36 a b c d

10 37 a b c d 10 a b c d 37 a b c d

11 38 a b c d 11 a b c d 38 a b c d

12 39 a b c d 12 a b c d 39 a b c d

13 a b c d 40 a b c d 13 a b c d 40 a b c d

14 a b c d 41 a b c d 14 a b c d 41 a b c d

15 a b c d 42 a b c d 15 a b c d 42 a b c d

16 a b c d 43 a b c d 16 a b c d 43 a b c d

17 a b c d 44 17 a b c d 44 a b c d

18 a b c d 45 18 a b c d 45 a b c d

19 a b c d 46 Space for Rough


19 Work
a b c d 46 a b c d

20 a b c d 47 20 a b c d 47 a b c d

21 a b c d 48 21 a b c d 48 a b c d

22 a b c d 49 a b c d 22 a b c d 49 a b c d

23 a b c d 50 a b c d 23 a b c d 50 a b c d

24 a b c d 51 a b c d 24 a b c d 51 a b c d

25 a b c d 52 a b c d 25 a b c d 52 a b c d

26 53 a b c d 26 a b c d 53 a b c d

27 54 a b c d 27 a b c d 54 a b c d
EBD_7801
TEST ASSESSMENT AND ANALYSIS SHEET
Mock Test - 2
Name :……………………................… Test Code : ........…………….. Date & Time of test:…………….............

Marks per PAPER 1 PAPER 2


questions Total Qs. Attempted Correct Net Score Total Qs. Attempted Correct Net Score
PHYSICS
M CQ'S
M CQ > 1 correct
Passage Cum M atching
Integer Answer
Passage Based
Physics Net Score
Sectional % Score
CHEMISTRY
M CQ'S
M CQ > 1 correct
Passage Cum M atching
Integer Answer
Passage Based
Chemistry Net Score
Sectional % Score
MATHEMATICS
M CQ'S
M CQ > 1 correct
Passage Cum M atching
Integer Answer
Passage Based
Maths Net Score
Sectional % Score
STRIKE RATE (Correct Answers/
Questions attempted):
TO TAL NET S CO RE
COMBINED NET S CORE (Paper 1 + Paper 2)

1) Analysis of wrong questions

Reasons for wrong questions No. of ques.


( to be filled after you have attempted wrong questions on your own after the test)
Knew the question and solved after test but did wrong because of calculation mistake (A)
Knew the question and solved after test but did wrong because got confused and applied wrong concept (B)
Did not knew the question and couldn’t solve even after exam (C)
Total Number of questions attempted wrong
Note : If some of (A) & (B) is high then you need more practice and also read instructions more carefully whereas if (C) is very high
it means the coverage of topic is not sufficient and you need to improve on it.

2) Analysis of not attempted questions : Divide the questions not attempted in 3 categories
Reasons for unattempted questions No. of ques.
(fill after you have tried unattempted questions on your own after the test)
Easy questions (A)
Average questions (B)
Difficult questions (C)
Total Number of ques. not attempted
Telegram @unacademyplusdiscounts

Join Us on Telegram for More Such Books

https://telegram.me/unacademyplusdiscounts

Join us from the above link or search ''unacademyplusdiscounts'' in Telegram


TARGET
IIT MOCK TEST -3
PAPER - 1

GENERAL INSTRUCTIONS :

1. Section I : Q. No. 1 to 7, Q. No. 19 to 25, Q. No. 37 to 43 are Multiple Correct Choice Type questions. For this section, 4
marks will be awarded for correct answer, 1 mark for partial answer provided NO INCORRECT option is darkened and zero
mark for no answer. In all other cases, – 2 marks will be awarded.
2. Section II : Q. No. 8 to 12, Q. No. 26 to 30, Q. No. 44 to 48 are Integer Answer Type questions. For this section, 3 marks
will be awarded for correct answer and zero mark for all other cases.
3. Section III : Q. No. 13 to 18, Q. No. 31 to 36, Q. No. 49 to 54 are Passage cum Matching based Single Correct Choice Type
questions. For this section 3 marks will be awarded for correct answer and zero mark for no answer. In all other cases, –
1 mark will be awarded.

Time : 180 minutes Max. Marks : 183

(b) integers n ¹ 1
..Part - A : Mathematics.. (c) integers n ¹ 1 since f (n –) ¹ f (n)
(d) integers n ¹ 1 since f (n+) ¹ f (n)
SECTION – I - Multiple Correct Choice Type 3. If [x] denotes the greatest integer less than or equal to x
n
This section contains 7 multiple choice questions. Each question
has 4 choices (a), (b), (c) and (d) for its answer, out of which ONE
then lim å [kx ] is not equal to
n ® ¥ k =1
OR MORE is/are correct. (a) x/2 (b) x/3 (c) x (d) 0
1. Two sides of a triangle are lies along the lines (a + b)x + [x] + 1 5 1
4. f (x) = for f : [0, ) ® ( , 3] , where [ . ] represents
(a – b)y – 2ab=0 and (a – b) x + (a + b) y – 2ab = 0. If the {x} + 1 2 2
triangle is isosceles and the third side passes through the greatest integer function and { . } represents fractional part
point (b – a, a – b), then the equation of third side can be of x, then which of the following is/are true?
(a) x + y = 0 (b) x = y + 2 (b – a) (a) f (x) is injective discontinuous function
(c) x – b + a = 0 (d) y – a + b = 0 (b) f (x) is surjective non differentiable function
2. With the usual notation for the greatest integer function æ ö
(c) m in ç lim f (x ), lim f (x ) ÷ = f (1)
f (x) = [x]3 – [x3] is discontinuous at all è x ® 1- x ® 1+ ø
(a) integers n (d) max (x values of point of discontinuity)= f (1)
Space for Rough Work
EBD_7801
MT-3 -2 Target IIT
5. A function f (x) satisfies the relation f(x + y) X Y Z W
= f (x) + f (y) + xy (x + y) x, y Î R . If f ' (0) = – 1, then 0 0 0 0

(a) f (x) is a polynomial function 1 1 1 1


(b) f (x) is an exponential function 2 2 2 2
(c) f (x) is twice differentiable for all x Î R
3 3 3 3
(d) f ' (3) = 8
6. Let P(X = r) = pqr and P(Y = r) =pqr, where r = 1, 2, .............., 4 4 4 4
0 < p < 1, q = 1– p. Suppose X and Y are independent. Let Z 5 5 5 5
= max (X, Y). Then
6 6 6 6
(a) P(Z £ m) = (1 – qm)2
7 7 7 7
(b) P(Z = m) = 2 pq m -1 – p(1 + q) q 2 m - 2
8 8 8 8
1
(c) å P(Z = m) = 9 9 9 9
m³ 1 p
(d) P( X £ m ) = 1 - q m 8. If 2 tan -1 ( 2 x + 1) = cos -1 x , then find the value of x.
7. If the conics whose equations are 9. Consider the curve C1 : y = sin 2 x - 3 sin x , C1 cuts the
2 2 2 2
S1 : (sin q) x + (2h tan q) xy + (cos q) y x - axis at (a, 0), a Î( -p , p ) . A1: The area bounded by the
+ 32 x + 16 y + 19 = 0 curve C1 and the positive x-axis between the origin and the
ordinate x = a,
S 2 : (cos2 q) x 2 – (2h ¢ cot q) xy + (sin 2 q) y 2 A2: The area bounded by the curve C1 and the negative
+ 16 x + 32 y + 19 = 0 x-axis between the ordinate x = a and the origin. Find the
value of A1 + A2 + 8A1A2.
p
intersect in four concyclic point, where q Î æç 0, ö÷ , then
è 2ø tan x - sin(tan -1 (tan x ))
10. Find the value of lim .
the correct statement(s) can be x®
p tan x + cos 2 (tan x )
2
(a) h + h ¢ = 0 (b) h - h¢ = 0

(c) q =
p
(d) q =
p æ 4 ( log x 2)2 ö÷ dx .
4 6 11. Find the value of ò2 ç x
ç log 2 -
ln 2 ÷ø
è
SECTION – II - Integer Answer Type 12. Let f ( x), x ³ 0 be a non-negative continuous function
This section contains 5 questions. The answer to each of the x
questions is a single-digit integer, ranging from 0 to 9. The
appropriate bubbles below the respective question numbers in the ò
and let F (x) = f (t )dt, x ³ 0 .
0
ORS have to be darkened. For example, if the correct answers to
question numbers X, Y, Z and W (say) are 6, 0, 9 and 2, respectively, If for some c > 0, f ( x) £ c F ( x), then find the value of
then the correct darkening of bubbles will look like the following: f (c).

Space for Rough Work


MOCK IIT - 3 MT-3 -3
SECTION – III - Matching Type
This section contains 6 questions of Matching Type, contains two tables each having 3 columns and 4 rows. Based on each table, there
are three questions. Each question has four options (a), (b), (c) and (d) ONLY ONE of these four options is correct.
(Qs. 13–15): By appropriately matching the information given in the three columns of the following table, give the answer of the
question that follows.
Column 1 gives information about first order non-linear differential equations.
Column 2 gives information about general solutions of the given differential equations.
Column 3 gives information about singular solutions of the given differential equations.
Column 1 Column 2 Column 3
2
æ dy ö 1
(I) 1 + ç ÷ = 2 (i) y = Cx + C2 + x2 (P) y=2
è ø
dx y

2
dy æ dy ö 1 2
(II) y = x +ç ÷ (ii) (x + C)2 + y2 = 1 (Q) y= - x
dx è dx ø 4

2
(III) y = çæ ö÷ - 3x
dy dy
+ 3x 2 (iii) 4 (2 – y) (y + 1)2 = 9 (x + C)2 (R) y=1
è dx ø dx

2
æ dy ö 2 3 2
(IV) ç ÷ (1 - y ) = 2 - y (iv) y = Cx + C2 (S) y= x
dx
è ø 4

[Note : Singular solution of a differential equation does not have any arbitrary constant.]
13. Which of the following options is the only correct combination?
(a) (I) (ii) (R) (b) (II) (iii) (P) (c) (III) (iv) (S) (d) (IV) (i) (Q)
14. Which of the following options is the only correct combination?
(a) (I) (ii) (P) (b) (IV) (iii) (P) (c) (III) (iv) (S) (d) (II) (i) (S)
15. Which of the following is the only incorrect combination?
(a) (I) (ii) (R) (b) (IV) (iii) (P) (c) (III) (i) (S) (d) (III) (iv) (Q)
(Qs. 16–18): By appropriately matching the information given in the three columns of the following table, give the answer of the
questions that follows.
Columns 1 and 2 gives information about inequalities.
Column 3 gives information about the solutions of given system of inequalities.
Space for Rough Work
EBD_7801
MT-3 -4 Target IIT
Column 1 Column 2 Column 3

5x
(I) 2 (x – 1) < x + 5 (i) x– <-8 (P) (– 7, 11)
3
(II) 3x – 7 > 2 (x – 6) (ii) 2x + 19 < 6x + 47 (Q) (– 1, 7)

3x x
(III) +5< (iii) 3 (x + 2) > 2 – x (R) (5, ¥)
2 2
(IV) 5 (2x – 7) – 3 (2x + 3) < 0 (iv) 6 – x > 11 – 2x (S) No solution
16. Which of the following options is the only correct combination?
(a) (I) (ii) (Q) (b) (II) (i) (S) (c) (III) (iv) (R) (d) (IV) (ii) (P)
17. Which of the following options is the only correct combination?
(a) (IV) (ii) (R) (b) (II) (iv) (Q) (c) (III) (i) (S) (d) (I) (iii) (P)
18. Which of the following options is the only incorrect combination?
(a) (II) (iv) (Q) (b) (III) (i) (S) (c) (IV) (ii) (P) (d) (I) (iii) (Q)

..Part - B : Physics.. 20. Graph shows a hypothetical speed distribution for a sample
dN
SECTION – I - Multiple Correct Choice Type of N gas particle (for v > v0 ; =0)
dv
This section contains 7 multiple choice quesitons. Each question
has 4 choices (a), (b), (c) and (d) for its answer, out of which ONE
OR MORE is/are correct.
19. Figure shows the variation of frequency of a characteristic a
X-ray and atomic number. dN
dv

v0
speed v
(a) The value of (av0) is 2N
(a) The characteristic X-ray is Kb
(b) The ratio vavg /v0 is equal to 2/3
(b) The characteristic X-ray is Ka
(c) The energy of photon emitted when this X-ray is emitted (c) The ratio vrms /v0 is equal to 1/ 2
by a metal having Z = 101 is 204 keV.
(d) The energy of photon emitted when this X-ray is emitted (d) Three fourth of the total particle has a speed between
by a metal having Z = 101 is 102 keV. 0.5v0 and v0.

Space for Rough Work


MOCK IIT - 3 MT-3 -5
21. Standing waves are produced on a stretched string of length 24. An ideal monoatomic gas is confined in a horizontal cylinder
L with fixed ends . When there is a node at a distance L/3 by a spring loaded piston (as shown in the figure). Initially
from one end, then : the gas is at temperature T1, pressure P1 and volume V1 and
the spring is in its relaxed state. The gas is then heated very
(a) minimum and next higher number of nodes excluding
slowly to temperature T2, pressure P2 and volume V2.
the ends are 2, 5 respectively
During this process the
(b) minimum and next higher number of nodes excluding piston moves out by a
the ends are 2, 4 respectively distance x. Ignoring the
(c) frequency produced may be V/3L friction between the piston
(d) frequency produced may be 3V/2L and the cylinder, the correct
statement(s) is (are)
[ V = Velocity of waves in the string ]
(a) If V2 = 2V1 and T2 = 3T1, then the energy stored in the
22. From a cylinder of radius R, a cylinder
I 1
of radius R/2 is removed, as shown. spring is P1V1
4
Current flowing in the remaining (b) If V2 = 2V1 and T2 = 3T1, then the change in internal
cylinder is I. Magnetic field strength energy is 3P1V1
is (c) If V2 = 3V1 and T2 = 4T1, then the work done by the gas
(a) Zero at point A 7
(b) Zero at point B is P1V1
A B 3
R/2 (d) If V2 = 3V1 and T2 = 4T1, then the heat supplied to the
m0I
(c) at point A R 17
3pR gas is PV
6 1 1
m0I 25. A parallel plate capacitor has a dielectric slab of dielectric
(d) at point B constant K between its plates that covers 1/3 of the area of
3pR
its plates, as shown in the figure. The total capacitance of
23. The magnitudes of the gravitational field at distance r1 and
the capacitor is C while that of the portion with dielectric in
r2 from the centre of a uniform sphere of radius R and mass between is C1. When the capacitor is charged, the plate area
m are F1 and F2 respectively. Then: covered by the dielectric gets charge Q1 and the rest of the
F1 r1 area gets charge Q2. The electric field in the dielectric is E1
(a) = if r1 < R and r2 < R and that in the other portion is E2. Choose the correct option/
F2 r2
options, ignoring edge effects.
F1 r22 E1
= (a) E = 1 Q1 E1
(b) F2 r12 if r1 > R and r2 > R 2
E1 1
F1 r1 (b) E = K
(c) = if r1 > R and r2 > R
2
F2 r2 Q1 3
(c) Q = K E2
2 Q2
F1 r12
= C 2+ K
(d) F2 r22 if r1 < R and r2 < R (d) C = K
1

Space for Rough Work


EBD_7801
MT-3 -6 Target IIT

SECTION – II - Integer Answer Type


L y
This section contains 5 questions. The answer to each of the
questions is a single-digit integer, ranging from 0 to 9. The I M
appropriate bubbles below the respective question numbers in the x
ORS have to be darkened. For example, if the correct answers to N z
question numbers X, Y, Z and W (say) are 6, 0, 9 and 2, respectively, K
I
then the correct darkening of bubbles will look like the following:
A particle of charge q is released at the origin with a velocity
ur uur
X Y Z W v = -v0 $i. Find the magnitude of instantaneous force F
0 0 0 0
on the particle if µ0 qv0 I =8 R. Assume that space is gravity
free.
1 1 1 1 28. Consider a parallel plate capacitor of capacity 10 µF with air
2 2 2 2 filled in the gap between the plates. Now one half of the
space between the plates is filled with a dielectric of dielectric
3 3 3 3 constant K = 4 as shown in figure. If the new capacitance of
4 4 4 4
the capacitor (in µF) is 5x, find the value of x?

5 5 5 5

6 6 6 6
K=4
7 7 7 7

8 8 8 8

9 9 9 9
29. 0.05 kg steam at 373 K and 0.45 kg of ice at 253K are mixed in
an insulated vessel. Find the equilibrium temperature (in
degree celcius) of the mixture. Given, Lfusion = 80 cal/g = 336
26. Find the self inductance (in henry) of a coil in which an J/g, Lvaporization = 540 cal/g = 2268 J/g, Sice = 2100 J/kg
e.m.f. of 10 V is induced when the current in the circuit K = 0.5 cal/gK and Swater = 4200 J/kg K = 1 cal/gK
changes uniformly from 1 A to 0.5 A in 0.2 sec. 30. In a resonance tube experiment to determine speed of sound,
27. A circular loop of radius R is bent along a diameter and air column in the pipe is made to resonate with a given
given a shape as shown in the figure. One of the semicircles tuning fork of frequency 480 Hz. The diameter of the pipe is
(KNM) lies in the x-z plane and the other one (KLM) in the 5 cm and it is open at one end. The smallest resonating
length is observed to be 16 cm. The speed of sound in m/s
y-z plane with their centres at the origin. Current I is flowing
from the given experimental data is found to be (330 + z).
through each of the semi-circles as shown in figure. Find the value of z.

Space for Rough Work


MOCK IIT - 3 MT-3 -7
SECTION – III - Matching Type
This section contains 6 questions of Matching Type, contains two tables each having 3 columns and 4 rows. Based on each table, there
are three questions. Each question has four options (a), (b), (c) and (d) ONLY ONE of these four options is correct.
(Qs. 31-33): By appropriately matching the information given in the three columns of the following table, give the answer of the
question that follows.
When a body performed translatory as well as rotatory motion we say it is rolling motion. Figure in column I shows the different types
of motion while column II and III gives the corresponding velocity of topmost point P and contact point O of the sphere respectively.
Column I Column II Column III
I. P (i) V p = Vcm + wR (P) V 0¹O
R
Vcm
O
Pure translational
II. P (ii) Vp ¹ 2 wR (Q) V0 = O
R
w
V=
cm O

O
Pure rotational
III. P (iii) Vp = Vcm (R) V0 = – Rw
R
Vcm
O
Rolling without slipping
IV. P (iv) Vp = Rw (S) V0 = Vcm
R
Vcm
O
Rolling with slipping
31. If w = 2sec–1, R = 4cm, then the velocity of point P and velocity of contact point O for the sphere in pure rotational motion are
equal in magnitude but opposite in direction. The correct matching for pure rotational motion is :
(a) II (iii) Q (b) II (iv) R (c) IV (i) Q (d) II (ii) Q
32. Which of the following is the correct matching for the body rolling without slipping?
(a) III (i) Q (b) I (iii) P (c) III (iv) S (d) III (iii) R
33. Which of the following is incorrect matching?
(a) I (iii) S (b) II (iv) R (c) IV (iii) Q (d) IV (ii) P
(Qs. 34-36): By appropriately matching the information given in the three columns of the following table, give the answer of the
question that follows.
M
Time period of spring-mass system executing S.H.M is given by T = 2p Column I shows the spring-mass system executing
K eff
S.H.M. column II shows the force constant (spring constant) of the combination, while column III represents the time period of the oscillation.
Space for Rough Work
EBD_7801
MT-3 -8 Target IIT
Column I Column II Column III

T = 2π [ M K1 + K 2 ]
12
I. K1
K2
(i) K = K1 + K2 (P)

T = 2π [ M(K 1 +K 2 ) K1K 2 ]
12
II. (ii) K = K1 + 2 K2 (Q)
K1

K2

K1 2K2 K1K 2
T = 2π [ M(2K1 + K 2 ) 2K1K 2 ]
12
III. M (iii) K= (R)
K1 + K 2

2K1K 2
T = 2π [ M K 1 + 2K 2 ]
12
IV. (iv) K= (S)
2K1 + K 2
K1 K1

K2

1
34. If the time period of the combination of spring-mass system shown in column I (III) [if K1 = K2 = K1] is 2p M/3K . The correct
matching for the system is:
(a) III (i) P (b) III (ii) S (c) III (iv) R (d) IV (iii) Q
35. The spring constant for the combination shown in the column I (II) becomes twice if K1 and K2 becomes twice. Then correct
matching shown for this spring-mass system is
(a) II (iii) Q (b) I (ii) S (c) II (i) P (d) II (iv) R
36. Which of the following shows the wrong matching?
(a) I (i) P (b) II (iii) Q (c) III (ii) R (d) IV (iv) R

Space for Rough Work


MOCK IIT - 3 MT-3 -9
(b) Bond enthalpy of C = C bond = 631.72 kJ mol–1
..Part - C : Chemistry..
(c) resonance energy compared to Kekule structure
= – 53.68 kJ mol–1
SECTION – I - Multiple Correct Choice Type (d) Bond enthalpy of C – C bond = 631.72 kJ mol–1
This section contains 7 multiple choice quesitons. Each question 39. Choose the correct statement.
has 4 choices (a), (b), (c) and (d) for its answer, out of which ONE (a) The nodal surface of 2s-orbital exists at a distance
OR MORE is/are correct. 1.058 Å from the nucleus.
37. Which of the following is wrong (b) in the plots of radial probability versus distances from
the nucleus, the number of peaks equal to n - l .
CH 3 CH 3 CH 3 (c) The opposite lobes of p-orbital and d-orbital have the
| | |
Na / ether same sign.
(a) CH 3 — C — Cl ¾¾¾¾® CH 3 — C — C — CH 3
| | | (d) Orbitals of a particular type will have different angular
CH3 CH 3 CH 3 wave function, depending upon the value of n.
40. In a reaction between A and B, the initial rate of reaction was
(b)
measured for different initial concentrations of A and B as
given below : [Given log 1.5 = 0.17609, log 2 = 0.3010]
CH 3 CH3 CH3
| - + | | A/M 0.20 0.20 0.40
+
H
CH 3 — C — CO O N a ¾¾® CH 3 — C — C — CH3 B/M 0.30 0.10 0.05
| | | R0/M s–1 5.07 × 10–5 5.07 × 10–5 7.6 × 10–5
CH 3 CH 3 CH 3
(a) The order of reaction with respect to A is 0.5
(b) The order of reaction with respect to B is 0.5
CH3 C2H5 (c) The order of reaction with respect to B is 0
(d) The order of reaction with respect to A is 1.5
C2H5 C2 H5 C2H5 41. The following conversion reaction can be carried out by
C=C H 2 / Ni CH3
(c) ¾¾¾® H using reaction sequence/s.
CH3 CH3
H

O
H 2 /Ni/ D
(d) CH3—C—CH2—CH3 ¾¾¾¾ ® (a) Zn / Hg / HCl,D Br2 / hn KCN H3O+ , D
¾¾¾¾¾¾¾®¾¾¾¾ ¾®¾¾¾¾ ®¾¾¾¾¾ ®
CH3—CH2 —CH2—CH3
38. The enthalpy of formation of ethane, ethylene and benzene NaBH 4 Al 2 O3 , D O3 / H 2 O(oxidative)
(b) ¾¾¾¾¾ ®¾¾¾¾¾ ®¾¾¾¾¾¾¾¾¾®
from the gaseous atoms are – 2839.2, – 2275.2 and –5536
kJ mol–1 respectively. The bond enthalpy of C – H bond is
(c) Heat I / NaOH, D
2 H+
given as equal to 410.87 kJ mol–1. ¾¾¾® ¾¾¾¾¾¾ ® ¾¾¾ ®
Correct options are
(d) KMnO 4 / OH - / heat
(a) Bond enthalpy of C – C bond = 373.98 kJ mol–1

Space for Rough Work


EBD_7801
MT-3 -10 Target IIT
42. Which of the following statement(s) is/are true? 44. One molal solution of a carboxylic acid in benzene shows the
(a) [PtCl2(NH3)4]Br2 will exhibit ionization isomerism elevation of boiling point of 1.518 K. If the degree of association
(b) [Co(NO2)(NH3)5]2+ will display linkage isomerism of the acid in benzene in percent is expressed as 40A then find
(c) [Co(NH3)6] [Cr(CN)6] will exhibit coordination the value of ‘A’. (Kb for benzene = 2.53 K kg mol–1) :
isomerism
(d) The oxidation number of Pt in the coordination 45. A compound of metal ion M x+ (Z = 24) has a spin only
compound Na[Pt(C2H4)Cl3] is +3.
43. Which of the following statement(s) is/are correct? magnetic moment of 15 B.M. Find the number of unpaired
(a) TeCl4 has trigonal bipyramidal structure with one po- electrons in the compound.
sition occupied by a lone pair
46. Sulfuryl chloride (SO2Cl2) reacts with water to give a mixture
(b) TeCl4 reacts with HCl to form [TeCl6]2– complex ion of H2SO4 and HCl. How many moles of baryta would be
(c) [TeCl6]2– is isomorphous with [SiF6]2– and [SbCl6]2– required to neutralize the solution formed by adding 1 mol
(d) None is correct of SO2Cl2 to excess of water?
SECTION – II - Integer Answer Type 47. H2 and D2 gases at a pressure of 1 atm each at 25ºC are in
This section contains 5 questions. The answer to each of the equilibrium with a solution containing H+ and D+ ions. If
questions is a single-digit integer, ranging from 0 to 9. The
E° + = -0.296 V , calculate log [D + ] /[H + ] .
appropriate bubbles below the respective question numbers in the D /D 2
ORS have to be darkened. For example, if the correct answers to
48. The density of solid water (ie. ice) at 25ºC is 0.98 g cm–2
question numbers X, Y, Z and W (say) are 6, 0, 9 and 2, respectively,
while that of liquid mercury at the same temperature is
then the correct darkening of bubbles will look like the following:
13.60 g cm–3. If the percentage of height of the ice above
X Y Z W
the surface of a container filled with mercury is expressed
0 0 0 0
as 100 – A then find the value of ‘A’.
1 1 1 1

2 2 2 2 SECTION – III - Matching Type


3 3 3
This section contains 6 questions of Matching Type, contains
3
two tables each having 3 columns and 4 rows. Based on each
4 4 4 4 table, there are three questions. Each question has four options
5 5 5 5 (a), (b), (c) and (d) ONLY ONE of these four options is correct.
6 6 6 6 (Qs. 49-51): By appropriately matching the information given in
7 7 7 7 the three columns of the following table, give the answer of the
question that follows.
8 8 8 8
Column I contains compound and Column II & III contains their
9 9 9 9
CFSE value and dipule moment respectively.

Space for Rough Work


MOCK IIT - 3 MT-3 -11
Column I Column II Column III
CFSE (D°) m spin (B.M)
(I) d3 (high spin) (i) –1.6 (P) 1.73
(II) d4 (low spin) (ii) –1.8 (Q) 4.90
(III) d6 (high spin) (iii) –1.2 (R) 2.83
(IV) d7 (low spin) (iv) –0.4 (S) 3.87

49. Find suitable combination which has highest dipole moment


(a) (I) (ii) (Q) (b) (IV) (iii) (S) (c) (III) (iv) (Q) (d) (II) (i) (Q)
50. e0g configuration is shown by which combination
(a) (III) (i) (P) (b) (III) (iii) (R) (c) (IV) (i) (P) (d) (I) (iii) (S)
51. Correct combination is
(a) (I) (ii) (S) (b) (I) (iii) (Q) (c) (IV) (ii) (P) (d) (IV) (iv) (R)

(Qs. 52-54): By appropriately matching the information given in the three columns of the following table, give the answer of the
question that follows.
Column I, II & III contains starting material, factors responsible for reaction & type of products / intermediate respectively.
Column I Column II Column III
(I) CH3CH2CH = CHCH3 + HCl (i) Rearrangement (P) 1, 2 and 1, 4 addition product
(II) CH2 = CH – CH = CH2 + HCl (ii) Inductive effect (Q) Stable carbocation
(III) C6H5CH = CH CH3 + HBr (iii) Hyper conjugation (R) 1, 2-hydride shift
(IV) C6H5 CH2 CH = CH2 + HBr (iv) Resonance (S) Position isomers

52. For the reaction given in column I, the only correct combination is
(a) (I) (ii) (P) (b) (I) (iv) (S) (c) (I) (ii) (S) (d) (I) (iii) (P)
53. For the reaction given in column I, the only correct combination is
(a) (II) (i) (P) (b) (II) (iv) (P) (c) (II) (iv) (R) (d) (II) (iii) (S)
54. For the reaction given in column I, the only correct combination is
(a) (IV) (i) (R) (b) (IV) (ii) (Q) (c) (IV) (i) (P) (d) (IV) (iv) (R)

Space for Rough Work


EBD_7801
MT-3 -12 Target IIT

PAPER - 2
GENERAL INSTRUCTIONS :

1. Section I : Q. No. 1 to 7, Q. No. 19 to 25, Q. No. 37 to 43 are Single Correct Choice Type questions. For this section, 3 marks
will be awarded for correct answer and zero mark for no answer. In all other cases, –1 mark will be awarded.
2. Section II : Q. No. 8 to 14, Q. No. 26 to 32, Q. No. 44 to 50 are Multiple Correct Choice Type questions. For this section, 4
marks will be awarded for correct answer, 1 mark for partial answer provided NO INCORRECT option is darkened and zero
mark for no answer. In all other cases, –2 marks will be awarded.
3. Section III : Q. No. 15 to 18, Q. No. 33 to 36, Q. No. 51 to 54 are Comprehension based Single Correct Choice Type questions.
For this section, 3 marks will be awarded for correct answer and zero mark for all other cases.

Time : 180 minutes Max. Marks : 183

4. The exponent of 7 in 100C50 is


..Part - A : Mathematics.. (a) 0 (b) 2
(c) 4 (d) None of these
SECTION – I - Single Correct Choice Type
5. The equation 3sin2x + 10 cos x – 6 = 0 is satisfied if ( n Î I )
This section contains 7 multiple choice questions. Each question
(a) x = np + cos–1(1/3) (b) x = np – cos–1(1/3)
has 4 choices (a), (b), (c) and (d) for its answer, out of which ONLY np
ONE is correct. (c) x = 2np ± cos–1 (1/3) (d) x = – cos–1 (1/3)
2
1. A ray of light travels along a line y = 4 and strikes the surface 6. Let the coordinates of the two points A and B be (1, 2) and
of a curve y2 = 4 (x + y) then equation of the line along (7, 5) respectively . The line AB is rotated through 45º in
reflected ray travel anti clockwise direction about the point of trisection of AB
which is nearer to B . The equation of the line in new
(a) x = 0 (b) x = 2
position is
(c) x + y = 4 (d) 2x + y = 4
(a) 2x - y - 6 = 0 (b) x - y - 1 = 0
2. The conic represented by
(c) 3x - y - 11 = 0 (d) None of these
x = 2 (cos t + sin t), y = 5 (cos t – sin t) is 7. A line intersects the straight lines 5x – y – 4 = 0 and
(a) a circle (b) a parabola 3x – 4y – 4 = 0 at A and B respectively. If a point P (1, 5) on
(c) an ellipse (d) a hyperbola the line AB is such that AP : PB = 2 : 1 (internally), find the
3. If the straight line ax + by = 2 ; a, b ¹ 0 touches the circle point A.
x2 + y2 – 2x = 3 and is normal to the circle x2 + y2 – 4y = 6,then æ 75 307 ö æ 65 304 ö
the values of a and b are respectively (a) çè 17 , 17 ÷ø (b) ç ,
è 17 17 ÷ø
(a) 1, – 1 (b) 1, 2
(c) –4/3, 1 (d) 2, 1 æ 75 104 ö æ 75 180 ö
(c) çè , ÷ (d) çè , ÷
17 17 ø 17 17 ø
Space for Rough Work
MOCK IIT - 3 MT-3 -13
y = t2 + t | t |,
(b) If y = f (x) where x = 2t – | t |, t Î R then it
SECTION – II - Multiple Correct Choice Type
is differentiable in [– 1, 1].
This section contains 7 multiple choice questions. Each question
(c) A function f is defined by f(x2) = x3 for all x > 0 then f is
has 4 choices (a), (b), (c) and (d) for its answer, out of which ONE not differentiable at 4.
OR MORE is/are correct. (d) Every differentiable function is continuous, but the
converse is not true.
3 3x 3x 2 + 2a 2
dy
8. Let D (x) = 3x 3x 2 + 2a 2 3x 3 + 6a 2 x 12. The solution of x2y12 + x yy1 – 6y2 = 0 are (y1 means )
dx
3x 2 + 2a 2 3x 3 + 6a 2 x 3x 4 + 12a 2 x 2 + 2a 4 (a) y = cx2 (b) x2y = c
then 1
(c) log y = c + log x (d) x3y = c
(a) D' (x) = 0 2
(b) D (x) is independent of x
13. The graph of the function cos x cos( x + 2) - cos 2 ( x + 1) is
1
(a) A straight line passing through (0, 0)
ò D (x) dx = 16a
6
(c)
0 æp ö
(b) A straight line passing through ç , - sin 2 1÷ and
(d) y = D (x) is a straight line of infinite slope è2 ø
9. Indicate the relation which is/are true? paralles to x-axis
(a) tan | tan–1 x | = | x | (b) cot | cot–1 x | = x (c) A straight line passing through (0, – sin 21)
(c) tan–1 | tan x | = | x | (d) sin | sin–1 x | = | x | (d) Not a straight line
10. Choose the correct statements 14. Let w be a complex cube root of unity with w ¹ 1 and
n2
P = [pij] be a n × n matrix with pij = wi+j. Then p2 ¹ 0, when
(a) The largest term in the sequence a n = is a7. n=
n 3 + 200 (a) 57 (b) 55
(c) 58 (d) 56
2(y - 1) y2 - 1
(b) log y lies between and for all y > 0. SECTION – III - Comprehension Type
y +1 y
(c) If a, b > 0; 0 £ p < 1 then (a + b)p £ ap + bp. This section contains 2 paragraphs. Each paragraph has 2
(d) The radius of the right circular cylinder of greatest multiple choice questions based on a paragraph. Each question
curved surface which can be inscribed in a given cone has 4 choices (a), (b), (c) and (d) for its answer, out of which ONLY
ONE is correct.
is half that of the cone.
11. Choose the correct statements PARAGRAPH - 1
(a) If f (x) = x3 – x2 + x + 1 and
Let f be an even function integrable everywhere and periodic with
ìmax .f (t) ; 0 £ t £ x for 0 £ x £ 1 x
g(x) = í 3 - x ;
î 1< x £ 2 period 2. Let g (x) = ò f (t)dt and g (1) = a
then g(x) is not differentiable at x = 1. 0

Space for Rough Work


EBD_7801
MT-3 -14 Target IIT
15. Function g(x) is (c) To find image of a line in a plane consider the line
(a) odd (b) even
x -a y-b z- g
(c) neither even nor odd (d) none of these = = =r.
l m n
16. Which of the following statement is correct for all x?
A variable point on this line be taken as
(a) g (x + 2) + g (x) = g (2) (b) g (x + 2) – g (x) = g (2)
(c) g (x + 2) – g (x) = 2g (2) (d) none of these Q (lr + a, mr + b, nr + g ) .
If this lies on the plane, then
PARAGRAPH - 2
a (lr + a ) + b(mr + b) + c(nr + g ) + d = 0
Three concepts related to lines are :
(a) Let equation of the line be aa + bb + cg + d
or r = -
al + bm + cn
x-a y-b z-c
= = =r (say) ..........(1) The co-ordinates of Q can be obtained by the substituting
l m n
the value of r.
and (a, b, g) be the point, Let l,m,n, denote the actual d.c.'s
Now, find the image R of (a, b, g ) in the plane. Find the
of the line. Any point on the line (1) is
P (lr + a, mr + b, nr + c) ...........(2) equation of the line RQ which is the image of the PQ.
If it is the foot of the perpendicular, from A on the line, then 17. A line with direction cosines proportional to (2, 7, –5) is
AP is ^ to the line, so x-5 y-7 z+2
drawn to intersect the lines = = and
l(lr + a – a) + m (mr + b – b) + n (nr + c – g) = 0 3 -1 1
i.e. r = (a – a) l + (b – b) m + (g – c)n x+3 y-3 z-6
Since l2 + m2 + n2 = 1 = = . The intercepted length is
-3 2 4
Putting this value of r in (2), we get the foot of the
perpendicular from point A to the line (a) 58 (b) 78
Since foot of perpendicular P is known, length of
(c) 65 (d) 62
perpendicular,
r
2 2 2
18. Let A, B, C be points with position vectors r1 = 2iˆ - ˆj + kˆ ,
AP = [(lr + a - a ) (mr + b - b) + (nr + c - g ) ]
r ˆ ˆ ˆ r
r r r r2 = i + 2j + 3k and r3 = 3iˆ + ˆj + 2kˆ relative to the origin
(b) Shortest distance between the lines r = a1 - lb1 and
O. The shortest distance between point B and plane OAC is
r r r r (a) 10 (b) 5
r r r (a 2 - a1 ).(b1 ´ b 2 )
r = a 2 - lb2 is given by S.D. = r r
b ´b 1 2
(c) 5/7 (d) 2 5/7

Space for Rough Work


MOCK IIT - 3 MT-3 -15
m0I m0 I
..Part - B : Physics.. (a) (b)
4 pd 2pd
m 0I 2m 0 I
SECTION – I - Single Correct Choice Type (c) (d)
This section contains 7 multiple choice questions. Each question pd pd
has 4 choices (a), (b), (c) and (d) for its answer, out of which ONLY 21. The molar heat capacity C for an ideal gas going through a
ONE is correct. given process is given by C = a/T , where 'a' is a constant.
If g = CP/CV , the work done by one mole of gas during
19. As shown in arrangement, waves with identical wavelengths heating from T0 to h T0 through the given process will be
and amplitudes that are initially in phase travel through æ h - 1ö
1
different media, ray 1 travels through air and ray 2 through (a) ln h (b) a ln h – ç RT
T0
a transparent medium for equal length L, in four different a è g - 1÷ø
situations. In each situation the two rays reach a common (c) a ln h – (g – 1) RT0 (d) none of these
point on the screen.
L l3
22. Let y = l 2 - where l = 2.0 ± 0.1, z = 1.0 ± 0.1 then the
z
value of y is given by
Ray 2
(a) + 2 ± 0.8 (b) – 4 ± 1.6
Ray 1 (c) – 4 ± 0.8 (d) None of these
The number of wavelengths in length L is N2 for ray 2 and 23. An electric field is given by Ex = – 2x3 kN/C. The value of
N1 for ray 1. In the following table, values of N1 and N2 are
given for all four situations. The order of the situations the potential at the point (1, –2), if potential of the point
according to the intensity of the light at the common point (2, 4) is taken as zero, is
in descending order is (a) 7.5 × 103 V (b) 2.5 × 103 V
Situations 1 2 3 4
N1 2.25 1.80 3.00 3.25 (c) 15 × 103 V (d) 5 × 103 V
N2 2.75 2.80 3.25 4.00 24. A wall is moving with velocity u and a source of sound
(a) I3 = I4 > I2 > I1 (b) I1 > I3 = I4 > I2 moves with velocity u/2 in the same direction as shown in
(c) I1 > I2 > I3 > I4 (d) I2 > I3 = I4 > I1
the figure. Assuming that the sound travels with velocity
20. If the magnetic field at P can be written as K tan (a/2) then K
is 10u, the ratio of incident sound wavelength on the wall to
I the reflected sound wavelength by the wall, is equal to

(a) 9 : 11
u
P (b) 11 : 9
d (c) 4 : 5 u/2
I
S
(d) 5 : 4

Space for Rough Work


EBD_7801
MT-3 -16 Target IIT
25. Which of the following statement(s) is/are correct ? 1
r (a) Total charge flow through the conductor is i0 t 0
(a) A particle has initial velocity u = 2iˆ + 3jˆ and 2
r
acceleration a = 3iˆ - 2ˆj always. It’s resultant path will
æ tö
be a circle. (b) Expression of current in terms of time is i 0 ç1 - t ÷
(b) The potential energy of a particle moving along x-axis è 0ø
in a conservative force field is U = 2x2 – 5x + 1 in S.I.
units. No other forces are acting on it. It performs SHM. (c) If resistance of conductor is R then total heat dissipated
(c) A standing wave pattern is formed on a string. The
power transfer through a point (other than node and Rt 0i 02
across resistance R is
antinode) is zero always. 3
(d) None of these (d) If resistance of conductor is R then total heat dissipated
SECTION – II - Multiple Correct Choice Type 2Rt 0i02
This section contains 7 multiple choice questions. Each question across resistance R is
3
has 4 choices (a), (b), (c) and (d) for its answer, out of which ONE
29. In the circuit shown, resistance R = 100W, inductance
OR MORE is/are correct.
2 8
26. Suppose the potential energy between electron and proton L = H and capacitance C = mF are connected in series
p p
Ke 2 with an ac source of 200 volt and frequency f. If the readings
at a distance r is given by - . Application of Bohr’ss
3r 3 of the hot wire voltmeters V1 and V2 are same then
theory to hydrogen atom in this case shows that
(a) energy in the nth orbit is proportional to n6
(b) energy is proportional to m –3 (m : mass of electron) V1 V2

(c) energy in the nth orbit is proportional to n –2


(d) energy is proportional to m 3 (m = mass of electron)
27. Which of the following is/are conservative force(s) ? R L

r r
(a) F = 2r 3 rˆ (b) F = - 5 ˆr
r
r ˆ ˆ
3 (xi + yj) r 3 (yiˆ + xj)
ˆ
(c) F = 2 2 3/2
(d) F =
(x + y ) (x 2 + y2 )3 / 2 ~
28. Relation between current in conductor and time is shown in
figure then correct options are
i (a) f = 250 p Hz (b) f = 125 Hz
(c) current through R is 2A (d) V1 = V2 = 1000 volt
i0 30. The dimensions of energy per unit volume are the same as
those of
(a) work (b) stress
(c) pressure (d) modulus of elasticity

Space for Rough Work


MOCK IIT - 3 MT-3 -17
31. A parallel plate capacitor having plates of area S and plate
separation d, has capacitance C1 in air. When two dielectrics SECTION – III - Comprehension Type
of different relative primitivities (e1 = 2 and e2 = 4) are This section contains 2 paragraphs. Each paragraph has 2
introduced between the two plates as shown in the figure, multiple choice questions based on a paragraph. Each question
the capacitance becomes C2. Which is not the correct ratio has 4 choices (a), (b), (c) and (d) for its answer, out of which ONLY
of C2/C1 :- ONE is correct.
PARAGRAPH - 1
If the current flowing through the coil is i and the flux linked with
each turn is f , then the total flux linkage is Nf.
Nf µ i or Nf = Li
where L is a constant called the 'coefficient of self-induction' or
'self-inductance' of the coil. By the above equation, we have
L = Nf/i.
In this equation, if i = 1, then L = Nf. Hence the coefficient of
self-induction of a coil is equal to the number of flux-linkages in
the coil when unit current is flowing in the coil.
Alternatively, the coefficient of self inductance can be
defined as twice of the work done in establishing a flow of one
ampere current in the circuit. This energy is stored in the magnetic
field of the inductor.
(a) 6/5 (b) 5/3 33. A long coaxial cable consists of two concentric cylinders of
(c) 7/5 (d) 7/3 radii a and b. The central conductor of the cable carries a
32. Two spheres P and Q of equal radii have densities r1 and steady current i and the outer conductor provides the return
r2, respectively. The spheres are connected by a massless path of the current. The self inductance of this length l of
string and placed in liquids L1 and L2 of densities s1 and s2 the cable is
and viscosities h1 and h2, respectively. They float in
equilibrium with the sphere P in L1 and sphere Q in L2 and m0l æ bö m 0l æ bö
(a) log e ç ÷ (b) log e ç ÷
the string being taut (see figure). If sphere P alone in L2 has 2p è aø p èaø
ur
terminal velocity VP and Q alone in L1 has terminal velocity m0l æ aö m 0l æaö
ur (c) log e ç ÷ (d) log e ç ÷
V Q , then 2p è bø p è bø
ur 34. Calculate the self inductance per unit length of a current
VP h1
L1
= loop formed by joining the ends of two long parallel wires of
(a) ur
V Q h2 radius r separated by a distance d between their axes,
ur L2 neglecting the end effects and magnetic flux within the wires.
VP h m0 d-r m0 d+r
2
(b) ur = (a) loge (b) loge
VQ h1 2p r 2p r
ur ur
(c) V P . V Q > 0 m0 d-r
ur ur (c) loge (d) None of these
(d) V P .V Q < 0 p r

Space for Rough Work


EBD_7801
MT-3 -18 Target IIT
PARAGRAPH - 2
If the container filled with liquid gets accelerated horizontally or nRT æ P2 ö nRT æ P1 ö
(a) çè1 + P ÷ø (b)
gh çè1 + P ÷ø
vertically, pressure in liquids gets changed. In case of horizontally gh 1 2
ax
accelerated liquid (ax), the free surface has the slope . In case
g nRT æ P2 ö nRT æ P1 ö
ç1 - ÷ 1-
gh çè P2 ÷ø
of vertically accelerated liquid (ay) for calculation of pressure, (c) (d)
gh è P1 ø
effective g is used. A closed box with horizontal base 6m by 6m and
a height 2m is half filled with liquid. It is given a constant horizontal 39. Which of the following is correct set of physical properties
acceleration g/2 and vertical downward acceleration g 2. of the geometrical isomers –
35. The angle of the free surface with the horizontal is equal to
CH3 CH3
(a) 30° (b) tan–1 (2/3)
–1
(c) tan (1/3) (d) 45°
36. Length of exposed portion of top of box is equal to
(a) 2m (b) 3m Dipole moment Boiling point Melting point Stability
(c) 4m (d) 2.5m (a) I > II I > II II > I I > II
(b) II > I II > I II > I II > I
..Part - C : Chemistry.. (c) I > II I > II I > II I > II
(d) II > I II > I I > II I > II
SECTION – I - Single Correct Choice Type 40. The percentage void space per unit volume of unit cell in
This section contains 7 multiple choice questions. Each question zinc fluoride structure is
has 4 choices (a), (b), (c) and (d) for its answer, out of which ONLY
(a) 15.03% (b) 22.18%
ONE is correct.
(c) 18.23% (d) 25.07%
37. One gram of commercial AgNO3 is dissolved in 50 mL of 41. Electrolysis is carried out in three cells : (a) – 1.0 M CuSO4,
water. It is treated with 50 mL of a KI solution. The silver
Pt electrodes; (b) – 1.0 M CuSO4, Copper electrodes; (c) –
iodide thus precipitated is filtered off. Excess of KI in the
1.0 M KCl, Pt electrodes. If volume of electrolytic solution is
filtrate is titrated with M/10 KIO3 solution in presence of 6
maintained constant in each of the cells, which is correct set
M HCl till all I– ions are converted into ICl. It requires 50 mL
of pH changes in (a), (b) and (c) cells respectively?
of M/10 KIO3 solution. Twenty millilitres of the same stock
solution of KI requires 30 mL of M/10 KIO3 under similar (a) increase in all the three
conditions. Calculate the percentage of AgNO 3 in the (b) decrease in all the three
sample. (c) increase, constant, increase
(Reaction : KIO3 + 2KI + 6HCl ® 3ICl + 3KCl + 3H2O.) (d) decrease, constant, increase
(a) 85 (b) 80 42. The value of DfGº for formation of Cr2O3 is –540 kJ mol–1 and
(c) 75 (d) 90 that of Al2O3 is –827 kJ mol–1. Is the reduction of Cr2O3
38. An ideal gas undergoes a single-stage expansion against a possible with Al ?
constant opposing pressure P2 from P1, V1, T to P2, V2, T. (a) Yes (b) No
What is the largest mass m which can be lifted through a
(c) data insufficient (d) None of these
height h in this expansion ?

Space for Rough Work


MOCK IIT - 3 MT-3 -19
43. Which of the following statements is/are correct? 46. Which of the following statements are not correct
(1) OF2, Cl2O and Br2O are tetrahedral with two positions (a) A meso compound has chiral centres but exhibits no
occupied by two lone pairs of electrons optical activity
(2) In OF2, the bond angle is less than 109°28' (b) A meso compound has no chiral centres and thus are
(3) In Cl2O, the bond angle is greater than 109°28' optically inactive
(c) A meso compound has molecules which are
(4) In Br2O the bond angle is greater than 111°
superimposable on their mirror images even though
(a) 1, 2, 3, 4 (b) 1, 3, 4 they contain chiral centres
(c) 1, 2, 4 (d) 1, 3 (d) A meso compound is optically inactive because the
rotation caused by any molecule is cancelled by an
SECTION – II - Multiple Correct Choice Type equal and opposite rotation caused by another mol-
This section contains 7 multiple choice questions. Each question ecules that is the mirror image of the first.
has 4 choices (a), (b), (c) and (d) for its answer, out of which ONE 47. Choose the correct statements.
OR MORE is/are correct. (a) Vacancy defects lower the density of the substance
44. Which of the following statement/s with regard to quantum (b) Interstitial defects increase the density of the
number is/are correct? substance
(c) Schottky defects, preserve the electrical neutrality of
(a) The azimuthal quantum number gives the contribution
the crystal.
of energy due to angular momentum towards the total
(d) Frenkel defects do not affect the density of the crystal.
energy of the electron
48. In which of the following pairs hybridisation of the central
(b) The azimuthal quantum number gives the relative of
atom is same ?
energies of subshells belonging to the same shell
(a) ClF3, ClF3O (b) ClF3O, ClF3O2
(c) The orbital angular momentum is given by azimuthal (c) (ClF2O)+, (ClF4O)– (d) (ClF4O)– (XeOF4)
h 49. One mole of an ideal gas is subjected to a reversible process
quantum number which is equal to l(l + 1) that involves two steps ( and ). The pressure at A and C is
2p
same. Consider the graph and choose correct statements.
(d) The orbital angular momentum depends on the value
of ‘n’
B (2,800)
45. PCl5 dissociates into PCl3 and Cl2 thus (1,200)
PCl5(g) PCl3(g) + Cl2(g) A
C (1,400)]
P (atm.)
If the total pressure of the system in equilibrium is P at a
density r and temperature T. The vapour density of the gas
mixture at equilibrium has the value of 62 when the
temperature is 230ºC.
T (K)
Correct options are
(a) Work done is zero
(a) Degree of dissociation = 0.68 (b) In path, work will be done on the gas by the
(b) Value of P/r = 0.3327 surroundings
(c) Degree of dissociation = 0.3327 (c) Volume of gas at C = 2 × Volume of gas at A
(d) Value of P/r = 0.68 (d) Volume of gas at B is 32.8

Space for Rough Work


EBD_7801
MT-3 -20 Target IIT
50. Among the following compounds, which will react with (iii) Cation with pseudo noble gas configuration shows highest
acetone to give a product containing > C = N –bond ? polarisation power.
(a) C6H5NH2 (b) (CH3)3N 51. Which is most covalent in nature
(c) C6H5NHC6H5 (d) C6H5NHNH2. (a) NaCl (b) MgCl2
(c) AlCl3 (d) CaCl2
SECTION – III - Comprehension Type 52. Which is having highest melting point?
This section contains 2 paragraphs. Each paragraph has 2 (a) LiF (b) LiCl
multiple choice questions based on a paragraph. Each question (c) LiBr (d) LiI
has 4 choices (a), (b), (c) and (d) for its answer, out of which ONLY PARAGRAPH - 2
ONE is correct.
A hydrocarbon (A) of molecular weight 54 reacts with excess of
PARAGRAPH - 1 Br2 in CCl4 to give a compound (B) whose molecular weight is
593% more than that of (A) However, on catalytic hydrogenation
When anions and cation approach each other, the valence shell
with excess of hydrogen, (A) forms (C) whose molecular weight is
of anions are pulled towards a cation, is known as polarisation
only 7.4% more than that of (A) (A) reacts with CH3CH2Br in the
and ability of the cation to polarize the anion is called as polarising
presence of NaNH2 to give another hydrocarbon (D), which on
power of cation. Due to polarisation, sharing of electrons occurs
ozonolysis yields diketone (E),(E) on oxidation gives propanoic
between two ions to some extent and bond shows some covalent
acid.
character. Fajan's suggested following factors on which
53. Structure of C is
polarisation depend.
(a) CH3–CH2–C ºCH (b) CH3–CH2– CH2 – CH3
(i) As the charge on cation or anion increases polarisation
(c) CH3– CH2–C º C–C2H5 (d) CH3CH2COOH
increases.
54. Structure of D is
(ii) Size of cation decreases or size of anion increases,
(a) CH3–CH2–C ºCH (b) CH3–CH2– CH2 – CH3
polarisation increases.
(c) CH3– CH2–C º C–C2H5 (d) CH3CH2COOH

Space for Rough Work


MOCK IIT - 3
RESPONSE SHEET MT-3 -21

Name : ..................................
MOCK TEST - Date : .........................
PAPER 1 PAPER 2

1 a b c d 28 1 a b c d 28 a b c d

2 a b c d 29 2 a b c d 29 a b c d

3 a b c d 30 3 a b c d 30 a b c d

4 a b c d 31 a b c d 4 a b c d 31 a b c d

5 a b c d 32 a b c d 5 a b c d 32 a b c d

6 a b c d 33 a b c d 6 a b c d 33 a b c d

7 a b c d 34 a b c d 7 a b c d 34 a b c d

8 35 a b c d 8 a b c d 35 a b c d

9 36 a b c d 9 a b c d 36 a b c d

10 37 a b c d 10 a b c d 37 a b c d

11 38 a b c d 11 a b c d 38 a b c d

12 39 a b c d 12 a b c d 39 a b c d

13 a b c d 40 a b c d 13 a b c d 40 a b c d

14 a b c d 41 a b c d 14 a b c d 41 a b c d

15 a b c d 42 a b c d 15 a b c d 42 a b c d

16 a b c d 43 a b c d 16 a b c d 43 a b c d

17 a b c d 44 17 a b c d 44 a b c d

18 a b c d 45 18 a b c d 45 a b c d

19 a b c d 46 Space for Rough


19 Work
a b c d 46 a b c d

20 a b c d 47 20 a b c d 47 a b c d

21 a b c d 48 21 a b c d 48 a b c d

22 a b c d 49 a b c d 22 a b c d 49 a b c d

23 a b c d 50 a b c d 23 a b c d 50 a b c d

24 a b c d 51 a b c d 24 a b c d 51 a b c d

25 a b c d 52 a b c d 25 a b c d 52 a b c d

26 53 a b c d 26 a b c d 53 a b c d

27 54 a b c d 27 a b c d 54 a b c d
EBD_7801
TEST ASSESSMENT AND ANALYSIS SHEET
Mock Test - 
Name :……………………................… Test Code : ........…………….. Date & Time of test:…………….............

Marks per PAPER 1 PAPER 2


questions Total Qs. Attempted Correct Net Score Total Qs. Attempted Correct Net Score
PHYSICS
M CQ'S
M CQ > 1 correct
Passage Cum M atching
Integer Answer
Passage Based
Physics Net Score
Sectional % Score
CHEMISTRY
M CQ'S
M CQ > 1 correct
Passage Cum M atching
Integer Answer
Passage Based
Chemistry Net Score
Sectional % Score
MATHEMATICS
M CQ'S
M CQ > 1 correct
Passage Cum M atching
Integer Answer
Passage Based
Maths Net Score
Sectional % Score
STRIKE RATE (Correct Answers/
Questions attempted):
TO TAL NET S CO RE
COMBINED NET S CORE (Paper 1 + Paper 2)

1) Analysis of wrong questions

Reasons for wrong questions No. of ques.


( to be filled after you have attempted wrong questions on your own after the test)
Knew the question and solved after test but did wrong because of calculation mistake (A)
Knew the question and solved after test but did wrong because got confused and applied wrong concept (B)
Did not knew the question and couldn’t solve even after exam (C)
Total Number of q uestions attempted wrong
Note : If some of (A) & (B) is high then you need more practice and also read instructions more carefully whereas if (C) is very high
it means the coverage of topic is not sufficient and you need to improve on it.

2) Analysis of not attempted questions : Divide the questions not attempted in 3 categories
Reasons for unattempted questions No. of ques.
(fill after you have tried unattempted questions on your own after the test)
Easy questions (A)
Average questions (B)
Difficult questions (C)
Total N umber of qu es. not a ttempted
Telegram @unacademyplusdiscounts

Join Us on Telegram for More Such Books

https://telegram.me/unacademyplusdiscounts

Join us from the above link or search ''unacademyplusdiscounts'' in Telegram


TARGET
IIT MOCK TEST -4
PAPER - 1

GENERAL INSTRUCTIONS :

1. Section I : Q. No. 1 to 6, Q. No. 21 to 26 and Q. No. 41 to 46 are Single Correct Choice Type questions. For this section, 3 marks
will be awarded for correct answer and zero marks for no answer. In all other cases, –1 mark will be awarded.
2. Section II : Q. No. 7 to 10, Q. No. 27 to 30 and Q. No. 47 to 50 are Multiple Correct Choice Type questions. For this section, 4
marks will be awarded for correct answer and zero marks for no answer. In all other cases, –1 mark will be awarded.
3. Section III : Q. No. 11 & 12, Q. No. 31 & 32 and Q. No. 51 & 52 are Multiple Matrix-Match Type questions. Each question
contains statements given in two columns, which have to be matched. The statements in Column-I are labelled A, B, C and D,
while the statements in Column-II are labelled p, q, r, s and t. Any given statement in Column -I can have correct matching with
ONE OR MORE statement(s) in Column-II. 2 marks each will be awarded for darkening the correct bubble in each row. Hence
the maximum marks of each question is 8 marks. No negative mark will be awarded for an incorrectly bubbled answer.
4. Section IV : Q. No. 13 to 18, Q. No. 33 to 38 and Q. No. 53 to 58 are Comprehension Type questions. For this section, 4 marks
will be awarded for correct answer and zero marks for no answer. In all other cases, –1 mark will be awarded.
5. Section V : Q. No. 19 & 20, Q. No. 39 & 40 and Q. No. 59 & 60 are Reasoning [Statement-1(Assertion) and Statement-
2(Reason)] Type questions. In this section, 3 marks will be awarded for correct answer and zero marks for no answer. In all
other cases, –1 mark will be awarded.

Time : 180 minutes Max. Marks : 240

1. At the end points A, B of the fixed segment of length L, lines


..Part - A : Mathematics.. are drawn meeting in C and making angles q and 2q
respectively with the given segment. Let D be the foot of
the altitude CD and let x represents the length of AD. The
SECTION – I - Single Correct Choice Type value of x as q tends to zero i.e. lim x equals
This section contains 6 multiple choice questions. Each question q®0
has 4 choices (a), (b), (c) and (d) for its answer, out of which ONLY (a) L/2 (b) 2L/3
ONE is correct. (c) 3L/4 (d) L/4

Space for Rough Work


EBD_7801
MT-4 -2 Target IIT
2. Number of positive integral values of ‘a’ for which the curve 7. Let a > 2, a Î N be a constant . If there are just 18 positive
y = ax intersects the line y = x is integers satisfying the inequality (x – a) (x – 2a) (x – a2) < 0,
(a) 0 (b) 1 then which of the option (s) is/are correct
(c) 2 (d) More than 2 (a) ‘a’ is composite
3. Let S º (3, 4) and S' º (9, 12) be two foci of an ellipse. If the (b) ‘a’ is odd
coordinates of the foot of the perpendicular from focus S to
(c) ‘a’ is greater than 8
a tangent of the ellipse is (1, – 4) then the eccentricity of the
(d) ‘a’ lies in the interval (3, 11)
ellipse is
r r r
(a) 5/13 (b) 4/5 8. If a, b, c be three non-zero vectors satisfying the condition
(c) 5/7 (d) 7/13 r r r r r r
4. If A is a non-singular matrix satisfying AB – BA = A, then a ´ b = c & b ´ c = a then which of the following always
which one of the following holds true hold(s) good ?
(a) det. B = 0 (b) B = 0 r r r
(a) a, b, c are orthogonal in pairs
(c) det. A = 1 (d) det. (B + I) = det. (B – I)
5. If H represent the harmonic mean between the abscissa, rrr r
(b) [a b c] = | b |
and K that between the ordinates of the points, in which a
circle x2 + y2 = c2 is cut by a chord lx + my = d, where l and rrr r
(c) [a b c] = | c |2
m are the direction cosines of the unit vector in the xy plane,
r r
then lH + mK has the value equal to (d) | b | = | c |
c2 c2
(a) 2d - (b) d - n
d 2d 9. Consider the binomial expansion of æç x + 1 ö÷ , n Î N
è 2.4 x ø
2c 2 c2
(c) d- (d) 2d - where the terms of the expansion are written in decreasing
d 2d
powers of x. If the coefficient of the first three terms form an
6. Number of ordered pair(s) (a, b) of real numbers such that
arithmetic progression then the statement(s) which hold good
(a + ib)2008 = a – ib holds good, is
is/are
(a) 2008 (b) 2009
(a) Total number of terms in the expansion of the binomial
(c) 2010 (d) 1
is 8
(b) Number of terms in the expansion with integral power
SECTION – II - Multiple Correct Choice Type of x is 3
This section contains 4 multiple choice questions. Each question (c) There is no term in the expansion which is independent
has 4 choices (a), (b), (c) and (d) for its answer, out of which ONE of x
OR MORE is/are correct.
(d) Fourth and fifth are the middle terms of the expansion

Space for Rough Work


MOCK IIT - 4 MT-4-3
10. Let f (x) and g (x) are differentiable function such that
1 cos 2x
x (a) sin 2x, sin 2x (b) , cos 2x
2 2
f (x) + ò g (t) dt = sin x (cos x - sin x) , and
0 1
(c) sin 2x, - sin 2x (d) – sin2x, cos 2x
(f '(x))2 + (g (x))2=1, then f (x) and g (x) respectively, are 2

SECTION – III - Matrix-Match Type


This section contains 2 questions. Each question contains statements given in two columns, which have to be p q r s t
matched. The statements in Column-I are labelled A, B, C and D, while the statements in Column-II are A p q r s t
labelled p, q, r, s and t. Any given statement in Column -I can have correct matching with ONE OR MORE B p q r s t
statement(s) in Column-II. The appropriate bubbles corresponding to the answers to these questions have to be C p q r s t
darkened as illustrated in the following example: D p q r s t
If the correct matches are A–p, s and t; B–q and r; C–p and q; and D–s and t; then the correct darkening
of bubbles will look like the given.

11. Normal to parabola y 2 = 4 x at points P and Q of parabola meet at R (x2, 0) and tangents at P and Q meets at T (x1, 0). Let x2 = 3
Match the entries of two columns.
Column – I Column – II
(A) The area of quadrilateral PTQR is p. 3
(B) If the quadrilateral PTQR can be inscribed in a circle q. 4

then the value of circumferecnce is


4p
(C) The number of normals that can be drawn to the parabola from R is r. 1

(D) The square of the length PT is s. 8


2 2
12. Let the circle (x – 1) + (y – 2) = 25 cuts a rectangular hyperbola with transverse axis along y = x at four points A,B,C and D having
coordinates (xi,yi), i = 1,2,3,4 respectively. O being the centre of the hypeprbola. Now match the entries from the following two
columns :
Column – I Column – II
(A) x1 + x2 + x3 + x4 is equal to p. 2
(B) x21 + x22 + x23 + x24 is equal to q. 4
(C) y21 + y22 + y23 + y24 is equal to r. 44
(D) OA2 + OB2 + OC2 + OD2 is equal to s. 56
t. 100

Space for Rough Work


EBD_7801
MT-4 -4 Target IIT
16. y satisfies the differential equation
SECTION – IV - Comprehension Type
This section contains 2 groups of questions. Each group dy
(a) + y = e x (cos x - sin x) - e - x (cos x + sin x)
has 3 multiple choice questions based on a paragraph. Each dx
question has 4 choices (a), (b), (c) and (d) for its answer, out of
which ONLY ONE is correct. dy
(b) - y = e x (cos x - sin x) + e - x (cos x + sin x)
dx
Paragraph for question nos. 13 to 15
dy
(c) + y = e x (cos x + sin x) - e- x (cos x - sin x)
Urn-I contains 5 red balls and 1 blue ball, dx
Urn-II contains 2 red balls and 4 blue balls. dy
A fair die is tossed. If it results in an even number, balls are (d) - y = e x (cos x - sin x) + e - x (cos x - sin x)
dx
repeatedly drawn one at a time with replacement from urn-I.
If it is an odd number, balls are repeatedly drawn one at a 17. The value of f ' (0) + f (0) equals
time with replacement from urn-II. Given that the first two (a) –1 (b) 2
draws both have resulted in a blue ball. (c) 1 (d) 0
18. f (x) as a function of x equals
13. Conditional probability that the first two draws have resulted
in blue balls given urn-II is used is ex 2
(a) e - x (cos x - sin x) + (3cos x + sin x) + e- x
(a) 1/2 (b) 4/9 5 5
(c) 1/3 (d) None of these
ex 2
14. If the probability that the urn-I is being used is p, and q is (b) e - x (cos x + sin x) + (3cos x - sin x) - e - x
the corresponding figure for urn-II then 5 5
(a) q = 16p (b) q = 4p
ex 2
(c) q = 2p (d) q = 3p (c) e - x (cos x - sin x) + (3cos x - sin x) + e - x
15. The probability of getting a red ball in the third draw, is 5 5
(a) 1/3 (b) 1/2
ex 2
(c) 37/102 (d) 41/102 (d) e - x (cos x + sin x) + (3cos x - sin x) - e - x
5 5

Paragraph for question nos. 16 to 18


Section - V - Reasoning Type
Let y = f (x) satisfies the equation This section contains 2 reasoning type questions. Each question
x has 4 choices (A), (B), (C) and (D), out of which ONLY ONE is
f (x) = (e–x + ex) cos x – 2x – ò (x - t) f ¢ (t) dt correct.
0

Space for Rough Work


MOCK IIT - 4 MT-4-5
19. Let h (x) = f1(x) + f2(x) + f3(x) + ........ + fn(x) where f1(x), f2(x), ..Part - B : Physics..
f3(x) ,........ , fn(x) are real valued functions of x.
Statement 1 : f (x) = | cos | x | | + cos–1 (sgn x) + | ln x | is not SECTION – I - Single Correct Choice Type
differentiable at 3 points in (0, 2p) This section contains 6 multiple choice questions. Each question
because has 4 choices (a), (b), (c) and (d) for its answer, out of which ONLY
Statement 2 : Exactly one function fi (x), i = 1, 2, ........, n not ONE is correct.
differentiable and the rest of the function differentiable at x
= a makes h (x) not differentiable at x = a. 21. A soap bubble of radius R is surrounded by another soap
(a) Statement-1 is true, statement-2 is true and statement- bubble of radius 2R, as shown. Take surface tension = S.
2 is correct explanation for statement-1 Then the pressure inside the smaller soap bubble, in excess
(b) Statement-1 is true, statement-2 is true and statement- of the atmospheric pressure, will be
2 is NOT correct explanation for statement-1
(c) Statement-1 is true, Statement-2 is false.
(d) Statement-1 is false, Statement-2 is true.

1 3

20. Statement 1 : If f (x)= ò (x f (t) + 1) dt , then ò f (x) dx = 12


0 0 Atmosphere

because
(a) 4S/R (b) 3S/R
Statement 2 : f (x) = 3x + 1
(c) 6S/R (d) None of these
(a) Statement-1 is true, statement-2 is true and statement- 22. Vernier callipers has 20 divisions on its vernier scale which
2 is correct explanation for statement-1 coincide with 19 divisions on the main scale. Least count of
(b) Statement-1 is true, statement-2 is true and statement- the instrument is 0.1mm. The main scale division is
2 is NOT correct explanation for statement-1 (a) 1 mm (b) 3mm
(c) Statement-1 is true, Statement-2 is false. (c) 2 mm (d) 6mm
(d) Statement-1 is false, Statement-2 is true. 23. A radioactive substance with decay constant of 0.5s–1 is
being produced at a constant rate of 50 nuclei per second.
If there are no nuclei present initially, the time (in second)
after which 25 nuclei will be present is
(a) 1 (b) ln 2
(c) ln (4/3) (d) 2 ln (4/3)

Space for Rough Work


EBD_7801
MT-4 -6 Target IIT
24. Consider six wires into or out of the page, all with the same
SECTION – II - Multiple Correct Choice Type
current. Rank the line integral of the magnetic field (from
This section contains 4 multiple choice questions. Each question
most positive to most negative) taken counterclockwise
has 4 choices (a), (b), (c) and (d) for its answer, out of which ONE
around each loop shown.
OR MORE is/are correct.
loop C
loop B
27. Which of the following are not dependent on the intensity
of the incident radiation in a photoelectric experiment
(a) Amount of photoelectric current
(b) Stopping potential to reduce the photoelectric current
to zero
(c) Work function of the surface
(d) Maximum kinetic energy of photoelectrons
loop D 28. A particle is projected vertically upwards in absence of air
loop A
resistance with a velocity u from a point O. When it returns
(a) B > C > D > A (b) B > C = D > A to the point of projection
(c) B > A > C = D (d) C > B = D > A (a) its average velocity is zero
25. Resonance occurs in a series L-C-R circuit when the (b) its displacement is zero
frequency of the applied emf is 1000 Hz. Then
(c) its average speed is u/2
(a) when f = 900 Hz, the circuit behaves as a capacitative
(d) its average speed is u
circuit.
29. How does the total energy stored in the capacitors in the
(b) the impedance of the circuit is maximum at f = 1000 Hz
circuit shown in the figure change when first switch K1 is
(c) at resonance the voltage across L and voltage across closed (process-1) and then switch K 2 is also closed
C differ in phase by 180° (process-2). Assume that all capacitors were initially
(d) if the value of C is doubled resonance occurs at uncharged
f = 2000 Hz.
26. Two identical thin planoconvex lenses of refractive index n
are silvered, one on the plane side and the other on the C C C
convex side. The ratio of their focal lengths is

n n -1 C K2
(a) (b)
n -1 n

n +1 K1
V
(c) (d) n
n

Space for Rough Work


MOCK IIT - 4 MT-4-7
(a) Increases in process-1
(b) Increases in process-2 A
(c) Decreases in process-2 Red
White P2
(d) Magnitude of change in process-2 is less than that in P1 Violet
process-1
A
30. P1 and P2 are identical prisms arranged as shown in figure.
A ray of white light incident on face of P1 undergoes
(a) Light emerging from P2 will be white
dispersion and falls on one face of P2. The facing surfaces
(b) In the light emerging from P2 dispersion will be greater
of the prisms are parallel to each other. Then
(c) The direction of light emerging from P2 will be parallel
to the direction of ray incident on P1
(d) The ray emerging from P2 will be white even if prisms
P1 and P2 have identical geometry but different material

SECTION – III - Matrix-Match Type


This section contains 2 questions. Each question contains statements given in two columns, which have to be p q r s t
matched. The statements in Column-I are labelled A, B, C and D, while the statements in Column-II are A p q r s t
labelled p, q, r, s and t. Any given statement in Column -I can have correct matching with ONE OR MORE B p q r s t
statement(s) in Column-II. The appropriate bubbles corresponding to the answers to these questions have to be C p q r s t
darkened as illustrated in the following example: D p q r s t
If the correct matches are A–p, s and t; B–q and r; C–p and q; and D–s and t; then the correct darkening
of bubbles will look like the given.

31. Column – I Column – II


(A) Solid sphere p. Rotational Kinetic Energy £ (Translational Kinetic Energy)/2
(B) Hollow sphere q. Rotational Kinetic Energy > (Translational Kinetic Energy)/2

MR 2
(C) Circular disc r. Moment of inertia about diameter <
2

MR 2
(D) Circular ring s. Moment of inertia about diameter ³
2

Space for Rough Work


EBD_7801
MT-4 -8 Target IIT
32. A simple telescope used to view distant objects has eyepiece and objective lens of focal lengths fe and f0 respectively. Then
Column – I Column – II
(A) Intensity of light received by lens p. Radius of aperture
(B) Angular magnification q. Dispersion of lens
(C) Length of telescope r. Focal length of objective lens and eyepiece lens
(D) Sharpness of image s. Spherical aberration

33 Just after closing the switch


SECTION – IV - Comprehension Type
(a) 2mA (b) 3mA
This section contains 2 groups of questions. Each group
has 3 multiple choice questions based on a paragraph. Each (c) 0mA (d) None of these
question has 4 choices (a), (b), (c) and (d) for its answer, out of 34. A long time after the switch was closed
which ONLY ONE is correct. (a) 2mA (b) 3mA
(c) 6mA (d) None of these
Paragraph for question nos. 33 to 35 35. Just after reopening the switch
In the connection shown in the figure, initially the switch K (a) 2mA (b) 3mA
is open and the capacitor is uncharged. Then the switch is closed (c) 6mA (d) None of these
and the capacitor is charged up to the steady state and the switch
is opened again. Determine the values indicated by the ammeter.
Paragraph for question nos. 36 to 38
[Given : V0 = 30V, R1 = 10kW, R2 = 5kW]
C
A thin, homogeneous stick of mass m and length l may
rotate in the vertical plane around a horizontal axle pivoted at one
end of the stick. A small ball of mass m and charge Q is attached to
R1 R2 the opposite end of this stick. The whole system is positioned in
A
mg
a constant horizontal electric field of magnitude E = . The
2Q
stick is held horizontally at the beginning.

K
× Q,m
V0
E

Space for Rough Work


MOCK IIT - 4 MT-4-9
36. What is the acceleration of the small ball at the instant of Section - V - Reasoning Type
releasing the stick ?
This section contains 2 reasoning type questions. Each question
(a) 3g/2 (b) 3g/4
has 4 choices (A), (B), (C) and (D), out of which ONLY ONE is
(c) 9g/8 (d) None of these correct.
37. What is the speed of ball when rod becomes vertical
39. Statement 1 : For a non-uniformly charged thin circular ring
3gl
(a) (b) 2gl with net charge zero, the electric potential at each point on
2
axis of the ring is zero.
Statement 2 : For a non-uniformly charged thin circular ring
3gl
(c) (d) None of these with net charge zero, the electric field at any point on axis of
4
the ring is zero.
38. Magnitude of acceleration of the end of the stick when it (a) Statement- 1 is True, Statement-2 is True, Statement-2
swings through the vertical position is a correct explanation for Statement -1
(b) Statement -1 is True, Statement -2 is True; Statement-2
3 5g 3 17 g
(a) (b) is NOT a correct explanation for Statement - 1
2 8
(c) Statement - 1 is True, Statement- 2 is False
3 2g (d) Statement -1 is False, Statement -2 is True
(c) (d) None of these
4 40. Statement 1 : Temperature of real gases often changes when
undergoing a free expansion under adiabatic condition
whereas that of an ideal gas does not.
Statement 2 : First law of thermodynamics holds for ideal
gases only.
(a) Statement-1 is true, statement-2 is true and statement-
2 is correct explanation for statement-1
(b) Statement-1 is true, statement-2 is true and statement-
2 is NOT correct explanation for statement-1
(c) Statement-1 is true, Statement-2 is false.
(d) Statement-1 is false, Statement-2 is true.

Space for Rough Work


EBD_7801
MT-4 -10 Target IIT

..Part - C : Chemistry.. O O
|| ||
(c) EtO - C - CH = CH - C - OEt
SECTION – I - Single Correct Choice Type
This section contains 6 multiple choice questions. Each question
has 4 choices (a), (b), (c) and (d) for its answer, out of which ONLY O
ONE is correct. COOEt

41. During mixed acid nitration of benzene concentrated nitric (d)


acid behaves like O

(a) acid (b) base COOEt


(c) catalyst (d) electrophile
42. Which of the following species will have highest bond length 44. If in the [Ma2bcde]n± complex ‘c’ is replaced by ‘b’ then
(a) O2 – (b) O2 2– number of geometrical isomers will be reduced by

(c) F2 (d) O2+ (a) 6 (b) 3


(c) 2 (d) 4
O O
|| || 45. Determine the amount of heat (in kcal) given off at constant
43. Et - O - C - CH 2 - CH 2 - C - O - Et OEtQ
¾¾¾® ? volume when 0.5 mol of N2 and 1.5 mol of H2 reacted
according to equation at 300K.
Major product of this reaction is
N2(g) + 3H2(g) ® 2NH3(g) ; DrH300 = – 380 kcal/mol
O (Given : R = 2 cal/mol/K)
(a) 378.8 kcal (b) 190 kcal
O
(c) 1888.8 kcal (d) 189.4 kcal
(a) C – O – Et
46. In Fe-extraction, the roasting is adopted though the ore is
not having any sulphide because
O (a) haematite is to be decomposed
COOEt
(b) all FeO is to be converted into Fe2O3
(c) all Fe2O3 is to be converted into FeO
(b) EtOOC (d) slag formation is encouraged
O

Space for Rough Work


MOCK IIT - 4 -11
MT-4

49. Correct statements about the plot is/are


SECTION – II - Multiple Correct Choice Type
This section contains 4 multiple choice questions. Each question
0
has 4 choices (a), (b), (c) and (d) for its answer, out of which ONE
–100
OR MORE is/are correct. 2C + O
–200
2 2CO
–300
47. The enthalpy of neutralization of 1M solution of HCl with –400
1M NaOH is –57.3 kJ/mol. The enthalpy of solution of a –500
weak base XOH (1M) with same HCl solution is –600

G (kJ/mol)
–54.6 kJ/mole. If molar conductivity of 1M solution of XOH –700
gO
is –800 2M
200 × 10–4 Sm2 mol–1 and molar conductivity of XOH at –900 O2
g+
infinite dilution is 0.20 Sm2 mol –1 then select correct –1000 2M
statement(s) [Assume all statements at 298 K] –1100

(a) DHionization of XOH is 3.0 kJ/mole –1200

(b) pH of 1M XOH solution is 12 T1 T2 T3


(c) pKa of X+ is 1 Temperature
(d) At equivalent point resultant solution of HCl and XOH (a) T1 and T2 are melting point and boiling point of Mg
is acidic respectively
48. 80.0gm. salt of weak base and strong acid XY is dissolved in (b) T1 and T2 are melting point and boiling point of MgO
water and formed 2 litre of aqueous solution. The pH of the respectively
resultant solution was found to be 5 at 298K. If XY forms (c) Reduction of MgO by coke is possible above T 3
(d) Mg can be extracted from gaseous products by rapid
CsCl type crystal having rX + (radius of X+) = 1.6Å
cooling
50. Propane reacts with chlorine in sunlight to give two prod-
and rY - (radius of Y– ) = 1.864 Å then select write
ucts. 1-chloropropane is obtained in 44% yield and 2-
statement(s) (Given Kb (XOH) = 4 × 10–5, NA = 6 × 1023) chloropropane is obtained in 56% yield of the total product.
(a) Molar mass of salt is 100g/mol 2-methylpropane reacts with chlorine under same condi-
tions to produce 1-chloro-2-methylpropane 66% and 2-
(b) % Degree of dissociation of salt is 0.25
chloro-2-methylpropane 33%. The percent yield (X) of the
(c) Edge length of AB is 4Å major monochloro product obtained when 3-ethylpentane
(d) Density of solid salt XY is 2.6 in gm/cc is treated with Cl2 in similar conditions.

Space for Rough Work


EBD_7801
MT-4 -12 Target IIT
The percent yield of the monochloro product obtained in Choose the correct options
minimum amount is (Y). Total number of different type of (a) X = 63 (b) Y = 12
carbonyl compounds formed (Z) during ozonolysis of mix-
(c) Z = 2 (d) X + Y + Z = 79
ture of all the benzene derivatives of formula C8H10.

SECTION – III - Matrix-Match Type


This section contains 2 questions. Each question contains statements given in two columns, which have to be p q r s t
matched. The statements in Column-I are labelled A, B, C and D, while the statements in Column-II are A p q r s t
labelled p, q, r, s and t. Any given statement in Column -I can have correct matching with ONE OR MORE B p q r s t
statement(s) in Column-II. The appropriate bubbles corresponding to the answers to these questions have to be C p q r s t
darkened as illustrated in the following example: D p q r s t
If the correct matches are A–p, s and t; B–q and r; C–p and q; and D–s and t; then the correct darkening
of bubbles will look like the given.

51. Column I Column II


(A) 1.5 mole of CO2(g) p. 33600 ml at STP
(B) 3.0 g of H2 q. Total number of atoms = 4.5 × NA
(C) 1.5 mole of O3 (g) r. Weighs 72 g
(D) 1 mole of oxygen s. Weighs 32 g

52. Column I Column II


(A) H2S p. Decolourises acidified sol. of KMnO4
(B) SO2 q. Disproportionation reaction
(C) NO2 r. Bleaching action
(D) HNO2 s. V-shaped structure

Space for Rough Work


MOCK IIT - 4 MT-4-13

SECTION – IV - Comprehension Type


This section contains 2 groups of questions. Each group has 3 multiple choice questions based on a paragraph. Each question
has 4 choices (a), (b), (c) and (d) for its answer, out of which ONLY ONE is correct.

Paragraph for question nos. 53 to 55

Red hot Mg
H2O NiCl2 solution
(I) (J) (K)
White Deep blue
solid solution
(When excess
Anhydrous J is passed)
(A) (B) + (C) + (D) (E)
Orange Green CoCl2 Pink
solid coloured solid solid

conc. HCl
H2O2/OH H2O2/H +
(F) (G) (H)
Green Yellow Et2O Blue
solution solution

53. Which of the following gas is turning the filter paper dipped into (MnSO4 + H2O2) solution brownish black
(a) gas C (b) gas D (c) gas J (d) None of these
54. Type of hybridisation in compound (K)
(a) d 2 sp 3 (b) sp 3 d 2 (d) dsp 2 (d) sp 3
55. Which of the following set of reagent is suitable to get (F) from (G)
(a) Neutral H2O2 solution (b) H2O2 /H+ + amyl alcohol
(c) H2O2/OH – (d) H2O2/H+

Paragraph for question nos. 56to 58 56. If MCl2 salt is coloured compound, then comment on colour
of its iodide salt
Colour of compounds occurs due to phenomenon of (a) blue
polarisation, charge transfer, d-d transition and H.O.M.O – (b) white
L.U.M.O. transition. Usually complex compounds are coloured (c) may be white or coloured
due to d-d transition and ionic compounds are coloured due to (d) may be coloured or black
polarisation of anion.

Space for Rough Work


EBD_7801
MT-4 -14 Target IIT
57. Which of the following species is coloured not due to d-d (a) Statement-1 is true, statement-2 is true and statement-
transition – 2 is correct explanation for statement-1
(a) [Cr(H2O)6]Cl3 (b) K3[Cu(CN)4] (b) Statement-1 is true, statement-2 is true and statement-
(c) [Mn(H2O)6]Cl3 (d) KMnO4 2 is NOT correct explanation for statement-1
58. Choose incorrect statement – (c) Statement-1 is true, Statement-2 is false.
(a) During charge transfer, oxidation state of atoms remains (d) Statement-1 is false, Statement-2 is true.
same 60. Statement 1 : The Friedel-Crafts alkylation of nitro-benzene
(b) Ag 2 CO 3 is yellow coloured compound due to gives metal alkylated nitrobenzene.
polarisation Statement 2 : Nitro group is meta directing group in
(c) Higher the polarisation intensity of colour is more electrophilic aromatic substitution reactions as it decreases
(d) Halogens are coloured due to HOMO – LUMO electron density at o & p-position.
transition (a) Statement-1 is true, statement-2 is true and statement-
2 is correct explanation for statement-1
(b) Statement-1 is true, statement-2 is true and statement-
Section - V - Reasoning Type 2 is NOT correct explanation for statement-1
This section contains 2 reasoning type questions. Each question (c) Statement-1 is true, Statement-2 is false.
has 4 choices (A), (B), (C) and (D), out of which ONLY ONE is
(d) Statement-1 is false, Statement-2 is true.
correct.

59. Statement 1 : Result of Victor meyer test


1° ROH ® Red colour
2° ROH ® Blue colour
3° ROH ® White or no colour
Statement 2 : Victor Meyer test is a method for separations
of 1°, 2° and 3° alcohol.

Space for Rough Work


MOCK IIT - 4 MT-4-15

PAPER - 2

GENERAL INSTRUCTIONS :

1. Section I : Q. No. 1 to 5, Q. No. 20 to 24 and Q. No. 39 to 43 are Single Correct Choice Type questions. For this section, 3 marks
will be awarded for correct answer and zero marks for no answer. In all other cases, –1 mark will be awarded.
2. Section II : Q. No. 6 to 8, Q. No. 25 to 27 and Q. No. 44 to 46 are Multiple Correct Choice Type questions. For this section, 4
marks will be awarded for correct answer and zero marks for no answer. In all other cases, –1 mark will be awarded.
3. Section III : Q. No. 9 & 10, Q. No. 28 & 29 and Q. No. 47 & 48 are Multiple Matrix-Match Type questions. Each question
contains statements given in two columns, which have to be matched. The statements in Column-I are labelled A, B, C and D,
while the statements in Column-II are labelled p, q, r, s and t. Any given statement in Column -I can have correct matching with
ONE OR MORE statement(s) in Column-II. 2 marks each will be awarded for darkening the correct bubble in each row. Hence
the maximum marks of each question is 8 marks. No negative mark will be awarded for an incorrectly bubbled answer.
4. Section IV : Q. No. 11 to 15, Q. No. 30 to 34 and Q. No. 49 to 53 are Integer Answer Type questions. In this section, 4 marks
will be awarded for correct answer and –1 mark for each wrong answer. Marks will be awarded only if you have darken the
appropriate bubble.
5. Section V : Q. No. 16 to 19, Q. No. 35 to 38 and Q. No. 54 to 57 are Reasoning [Statement-1(Assertion) and Statement-
2(Reason)] Type questions. In this section, 3 marks will be awarded for correct answer and zero marks for no answer. In all
other cases, –1 mark will be awarded.

Time : 180 minutes Max. Marks : 225

2. In the quadratic equation


..Part - A : Mathematics..
B
A ( 3 - 2) x 2 + x + C = 0 with a, b as its
SECTION – I - Single Correct Choice Type ( 3 + 2)
This section contains 5 multiple choice questions. Each question roots, if A = (49 + 20 6)1/ 4 ,
has 4 choices (a), (b), (c) and (d) for its answer, out of which ONLY
ONE is correct. 8 6 16
B = sum of the infinite G.P. as 8 3 + + + .....¥
3 3
dy
1. If solution of differential equation - y = 1 - e- x and and | a - b | = (6 6)k , where
dx
k = log6 10 - 2log 6 5 + log 6 (log 6 18 + log 6 72),
y (0) = y0 has a finite value. When x ® ¥, then y0 is equal to
then find the value of C.
(a) –1/2 (b) 0 (a) 136 (b) 164
(c) 1 (d) –1 (c) 128 (d) 64
Space for Rough Work
EBD_7801
MT-4 -16 Target IIT

x +1
6. Let f (x) is a derivable function satisfying
x
æ 1ö æ 1ö
3. Let f (x) = ç1 + ÷ and g(x) = ç1 + ÷ , both f and g
è xø è xø x
f (x) = ò e t sin (x - t) dt and g (x) = f '' (x) – f (x)
being defined for x > 0, then which one of the following 0
statements is correct
then the possible integers in the range of g (x) is
(a) Both f (x) and g (x) are increasing
(b) f (x) is increasing and g (x) is decreasing (a) – 1 (b) 0

(c) f (x) is decreasing and g (x) is increasing (c) 1 (d) 2


(d) Both (f) and g (x) are decreasing 7. Let P1 denotes the equation of the plane to which the vector
4. Let m denote the number of four digit numbers such that the (iˆ + ˆj) is normal and which contains the line L whose
left most digit is odd, the second digit is even and all four
r
digits are different and n denotes the number of four digit equation is r = iˆ + ˆj + kˆ + l (iˆ - ˆj - k)
ˆ , P denotes the
2
numbers such that the left most digit is even, an odd second equation of the plane containing the line L and a point with
digit and all four different digits. If m = nk then the value of
k equals position vector ĵ . Which of the following holds good
(a) 6/5 (b) 5/4 (a) Equation of P1 is x + y = 2
(c) 4/3 (d) 3/2 r
(b) Equation of P2 is r.(iˆ - 2jˆ + k)
ˆ =2
5. If A is an involutory and diagonal matrix of non-positive
entries and order 3, then
(c) The acute angle between P1 and P2 is cot -1 ( 3)
(a) there may exist some diagonal element in A, which is
zero (d) The angle between the plane P2 and the line L is
(b) value of | A | is –1 tan -1 3
(c) A–1 does not exist 8. Let z1, z2, z3 be three complex number such that | z1 | = | z2 | =
(d) | 3adj 2A | = – 1728
z12 z2 z2
| z3 | = 1 and + 2 + 3 + 1 = 0 then
z 2 z3 z1z 3 z1z 2
SECTION – II - Multiple Correct Choice Type
This section contains 3 multiple choice questions. Each question | z1+ z2 + z3 | can take the value equal to
has 4 choices (a), (b), (c) and (d) for its answer, out of which ONE (a) 1 (b) 2
OR MORE is/are correct. (c) 3 (d) 4

Space for Rough Work


MOCK IIT - 4 MT-4 -17

SECTION – III - Matrix-Match Type


This section contains 2 questions. Each question contains statements given in two columns, which have to be p q r s t
matched. The statements in Column-I are labelled A, B, C and D, while the statements in Column-II are A p q r s t
labelled p, q, r, s and t. Any given statement in Column -I can have correct matching with ONE OR MORE B p q r s t
statement(s) in Column-II. The appropriate bubbles corresponding to the answers to these questions have to be C p q r s t
darkened as illustrated in the following example: D p q r s t
If the correct matches are A–p, s and t; B–q and r; C–p and q; and D–s and t; then the correct darkening
of bubbles will look like the given.

9. Column-I Column-II
(A) if the medians from B and C are perpendicular to each other, the p. 0
minimum value of cot B + cot C is

2
(B) if R = 13 , r = 2 and r1 = 3, then its area in square units is q.
2 3

1
(C) if a (bc2 + ca2 + ab2) = b2c + c2a + a2b + 3abc then cos A is equal to r.
2

(D) if sin 4A + sin 4B + sin 4C + 8 cos A = 0, then cos A is equal to s. 30

10. Column-I Column-II


(A) The equation cos 2x + a sin x = 2a – 7 p. [1, ¥)
possess a solution if a belongs to the interval
(B) The equation 7cos x + 5 sin x = 2k + 1 has q. [2, 6]
a solution if k belongs to the interval
(C) If cos4x + a cos2x + 1 = 0 has at least one solution then a r. (–¥, –2]
belongs to the interval
(D) If the equation |sin 2x| – |x| – a = 0 does not s. [–4, 3]
have any real solution then a belongs to the interval

Space for Rough Work


EBD_7801
MT-4 -18 Target IIT
13. Evaluate:
SECTION – IV - Integer Answer Type
This section contains 5 questions. The answer to each of the æ
( ) ( ) ( )
2 3 kö
1/ k e1/ k e1/ k e1/ k
questions is a single-digit integer, ranging from 0 to 9. The çe ÷
lim + 2× + 3× + ... + k ×
appropriate bubbles below the respective question numbers in the k ®¥ ç k 2 k 2
k 2
k 2 ÷
çè ÷ø
ORS have to be darkened. For example, if the correct answers to
question numbers X, Y, Z and W (say) are 6, 0, 9 and 2, respectively,
then the correct darkening of bubbles will look like the following: 14. Tangents at points P and Q of parabola y 2 = 4 x intersect
X Y Z W
each other at point R on parabola y 2 = - x. The normal at P
0 0 0 0
and Q intersect at right angle at S. Then find the diameter of
1 1 1 1 circumcircle of qudrilateral PQRS.
2 2 2 2
b -c c - a a -b
15. In triangle ABC, find the value of + +
3 3 3 3 r1 r2 r3
4 4 4 4

5 5 5 5 Section - V - Reasoning Type


6 6 6 6 This section contains 4 reasoning type questions. Each question
7 7 7 7
has 4 choices (A), (B), (C) and (D), out of which ONLY ONE is
correct.
8 8 8 8

9 9 9 9
2x + 2
16. Let f (x) = ln (2 + x) –
x+3
x2 y 2 Statement-1 : The equation f (x) = 0 has a unique solution in
11. C is the centre of the hyperbola - = 1, and ' A ' is
4 1 domain of f (x) because
any point on it. The tangent at A to the hyperbola meets the Statement-2 : If f (x) is continuous in [a, b] and is strictly
line x - 2 y = 0 and x + 2 y = 0 at Q and R respectively.. monotonic in (a, b) then f has a unique root in (a, b).
Then find the value of CQ.CR. (a) Statement-1 is true, statement-2 is true and statement-
2 is correct explanation for statement-1
12. The point ( a , b ) ( b < a ) on the curve 4 x 2 + 3 y 2 = 12 , in (b) Statement-1 is true, statement-2 is true and statement-
the first quadrant is such that the area enclosed by the lines 2 is NOT correct explanation for statement-1
y = x, y = b , x = a and the x- axis is minimum. Find the value (c) Statement-1 is true, Statement-2 is false.
(d) Statement-1 is false, Statement-2 is true.
of b .

Space for Rough Work


MOCK IIT - 4 MT-4-19
(a) Statement- 1 is True, Statement-2 is True, Statement-2
é 1 -3 -4 ù is a correct explanation for Statement -1
17. Statement-1 : A = êê -1 3 4 úú is a Nilpotent. (b) Statement -1 is True, Statement -2 is True ; Statement-
êë 1 -3 -4 úû 2 is NOT a correct explanation for Statement - 1
(c) Statement - 1 is True, Statement- 2 is False
Statement-2 : A matrix A is said to be a Nilpotent matrix ifAk = 0,
for k Î N. (d) Statement -1 is False, Statement -2 is True

(a) Statement- 1 is True, Statement-2 is True, Statement-2 19. Statement 1 : The product of slopes of all the common
is a correct explanation for Statement -1 tangents of circles

(b) Statement -1 is True, Statement -2 is True ; Statement- S1 º x2 + y2 – a2 = 0 and S2 º (x – 2a)2 + (y – 2a)2 – 4a2 = 0 is 1.
2 is NOT a correct explanation for Statement - 1 Statement 2 : Slope of line joining centre of S1 º x2 + y2 – a2 = 0
(c) Statement - 1 is True, Statement- 2 is False and S2 º (x – 2a)2 + (y – 2a)2 – 4a2 = 0 is 1. Direct common
tangents make equal angles with the line joining centres of
(d) Statement -1 is False, Statement -2 is True
the circles.
18. Statement 1 : There exists no solution to
(a) Statement-1 is true, statement-2 is true and statement-
(sin x + cos x + 2)4 = 128 sin 2x. 2 is correct explanation for statement-1
Statement 2 : Let a, b, c, d be positive numbers, then (b) Statement-1 is true, statement-2 is true and statement-
2 is NOT correct explanation for statement-1
a +b+c+d 4
³ abcd . (c) Statement-1 is true, Statement-2 is false.
4
(d) Statement-1 is false, Statement-2 is true.
a +b+c+d 4
Further = abcd only if a = b = c = d.
4

Space for Rough Work


EBD_7801
MT-4 -20 Target IIT
with initial upward velocity v1 that will also rise to height h,
..Part - B : Physics..
but this time with air resistance ?
SECTION – I - Single Correct Choice Type v
This section contains 5 multiple choice questions. Each question
has 4 choices (a), (b), (c) and (d) for its answer, out of which ONLY
ONE is correct. (a) t

20. A ball of mass m collides horizontally with a stationary


wedge on a rough horizontal surface in the two orientations
as shown. Neglect friction between ball and wedge. Two
v
students comment on system of ball and wedge in these
situations.
(b) t
y
m m
m m
x
rough rough
(I) (II) v

Saurav : Momentum of system in x-direction will change by


significant amount in both cases. (c) t
Rahul : There are no impulsive external forces in y-direction
in both cases hence the total momentum of system in
y-direction can be treated as conserved in both cases.
(a) Saurav is wrong and Rahul is correct v
(b) Saurav is correct and Rahul is wrong
(c) Both are correct
(d) Both are wrong (d) t
21. You calculate that to throw an object vertically to a height h
it needs to be launched with an initial upward velocity v0,
assuming no air resistance. The dashed lines in figure show
the motion according to this calculation. Which of the 22. In a standing wave formed as a result of reflection from a
surface, the ratio of the amplitude at an antinode to that at
velocity-time graphs shows the motion of an object tossed
node is x. The fraction of energy that is reflected is

Space for Rough Work


MOCK IIT - 4 MT-4-21

2 2
25. A battery of emf e0 = 5V and internal resistance 5W is
é x - 1ù é x ù connected across a long uniform AB of length 1m and
(a) êë x úû (b) êë x + 1úû
resistance per unit length 5Wm–1. Two cells of e1 = 1V and e2
= 2V are connected as shown in the figure.
2 2
é x - 1ù é1ù
(c) êë x + 1úû (d) êë x úû
5V 5
23. Behind a thin converging lens having both the surfaces of
the same radius 10cm, a plane mirror has been placed. The
image of an object at a distance 40cm from the lens is formed at
the same position. What is the refractive index of the lens?
P
A B
1V 1
O
G

2
(a) 1.5 (b) 5/3 2V

(c) 9/8 (d) None of these


24. When a resistance R is connected in series with an element (a) The null point is at A
A, the electric current is found to be lagging behind the (b) If jockey is touched to point B the current in the
voltage by angle q1. When the same resistance is connected galvanometer will be going towards B
in series with element B, current leads voltage by q2. When (c) When jockey is connected to point A no current is
R, A, B are connected in series, the current now leads voltage flowing through 1V battery
by q. Assume same AC source is used in all cases, then (d) The null point is at distance of 8/15m from A
(a) q = q2 – q1 (b) tan q = tan q2 – tan q1 26. Choose the correct options
q1 + q2
(c) q= (d) None of these z
2

L
SECTION – II - Multiple Correct Choice Type
This section contains 3 multiple choice questions. Each question
has 4 choices (a), (b), (c) and (d) for its answer, out of which ONE d
x y
OR MORE is/are correct.

Space for Rough Work


EBD_7801
-22
MT-4 Target IIT
(a) Three point masses m1, m2 and m3 are located at the 27. When a sample of a gas is taken from state i to state f along
vertices of an equilateral triangle of side ‘a’ then moment the path iaf, heat supplied to the gas is 50 cal and work done
of inertia of the system about an axis along the altitude by the gas is 20 cal. If it is taken by path ibf, then heat
supplied is 36 cal. Then choose the correct options
a2
for the triangle passing through m1 is (m2 + m3 )
4
(b) A thin plate of mass M, length L, and width 2d is
mounted vertically on a frictionless fixed axle along the
z-axis, as shown. Initially the object is at rest. It is then a f

Pressure
tapped with a hammer to provide a torque t , which
produces an angular impulse H about the z-axis of

magnitude H = ò t dt then the angular speed w of i b

3H
the plate about the z-axis after the tap is .
Md2 Volume
(c) If a particle of mass m1 is located at (x, y, z) = (0, a, 0) (a) Work done by the gas along path ibf is 6 cal
and a second particle of mass m2 is located at (x, y, z) = (b) If work done upon the gas is 13 cal for the return path
(b, c, 0), then the location of their centre of mass is ‘fi’, then heat rejected by the gas along path ‘fi’ is 43
cal
æ m 2 b m1a + m 2 c ö
çè m + m , m + m , 0÷ø (c) If internal energy of the gas at state i is 10 cal, then
1 2 1 2
internal energy at state ‘f’ is 40cal
(d) Two bodies, A and B, collide as shown in figures a and (d) If internal energy at state ‘b’ is 22 cal and at ‘i’ is 10
b, they exert equal and opposite forces on each other cal then heat supplied to the gas along path ‘ib’ is 18
in (a) but not in (b). cal

VA VB
(i) A B

VA VB
(ii) A
B

Space for Rough Work


MOCK IIT - 4 MT-4 -23

SECTION – III - Matrix-Match Type


This section contains 2 questions. Each question contains statements given in two columns, which have to be p q r s t
matched. The statements in Column-I are labelled A, B, C and D, while the statements in Column-II are A p q r s t
labelled p, q, r, s and t. Any given statement in Column-I can have correct matching with ONE OR MORE B p q r s t
statement(s) in Column-II. The appropriate bubbles corresponding to the answers to these questions have to be C p q r s t
darkened as illustrated in the following example: D p q r s t
If the correct matches are A–p, s and t; B–q and r; C–p and q; and D–s and t; then the correct darkening
of bubbles will look like the given.

28. Column I Column II


(A) Orbital velocity of satellite p. radius of orbit of the satellite
(B) Escape velocity q. radius of earth
(C) Time period of satellite r. gravitational constant
(D) Value of gravitational acceleration due to earth s. density of the earth

29. Column I Column II


(A) Transformer core getting heated. p. Lenz law
(B) Alternating current produced in generators q. Change in flux
(C) Electromagnetic brakes r. Ohm’s law
(D) Heat produced in a wire due to alternating current s. Faraday’s right hand rule

Space for Rough Work


EBD_7801
MT-4 -24 Target IIT

SECTION – IV - Integer Answer Type


This section contains 5 questions. The answer to each of the
questions is a single-digit integer, ranging from 0 to 9. The
appropriate bubbles below the respective question numbers in the L R
ORS have to be darkened. For example, if the correct answers to
question numbers X, Y, Z and W (say) are 6, 0, 9 and 2, respectively,
then the correct darkening of bubbles will look like the following:
m
X Y Z W
0 0 0 0

1 1 1 1 31. In the figure masses m1, m2 and M are 20 kg, 5 kg and 50 kg


respectively. The coefficient of friction between M and ground
2 2 2 2
is zero. The coefficient of friction between m1 and M and that
3 3 3 3 between m2 and ground is 0.3. The pulleys and the strings are
4 4 4 4 massless. The string is perfectly horizontal between P1 and
m1 and also between P2 and m2. The string is perfectly vertical
5 5 5 5
between P1 and P2. An external horizontal force F is applied
6 6 6 6 to the mass M. Take g = 10 m/s2.
7 7 7 7

8 8 8 8
P1 m1
9 9 9 9

30. A pair of parallel horizontal conducting rails of negligible


M F
resistance shorted at one end is fixed on a table. The distance P2
between the rails is L. A conducting massless rod of m2
resistance R can slide on the rails frictionlessly. The rod is
tied to a massless string which passes over a pulley fixed to
the edge of the table. A mass m, tied to the other end of the
string hangs vertically. A constant magnetic field B exists Let the magnitude of the force of friction between m1 and
perpendicular to the table. If the system is released from M be f1 and that between m2 and ground be f2. For a particular
rest, calculate the acceleration (in ms–2) of the mass at the
2
instant when the velocity of the rod is half the terminal F it is found that f1 = 2f2. Find
5
( f1 - f2 ) in newton.
velocity.

Space for Rough Work


MOCK IIT - 4 MT-4-25
32. A column of air and a tuning fork produce 4 beats per second Section - V - Reasoning Type
when sounded together. The tuning fork gives the lower
This section contains 4 reasoning type questions. Each question
note. The temperature of air is 15ºC. When the temperature
has 4 choices (A), (B), (C) and (D), out of which ONLY ONE is
falls to 10ºC, the two produce 3 beats per second. If the
frequency (in Hz) of the fork be n then find the value of correct.

æ n ö
ç - 4÷ . 35. Statement 1 : The mutual inductance of two concentric
è 10 ø conducting rings of different radii is maximum if the rings
33. An electric heater is used in a room of total wall area 137 m2 are also coplanar.
to maintain a temperature of + 20°C inside it, when the outside Statement 2 : For two coaxial conducting rings of different
temperature is – 10°C. The walls have three different layers radii, the magnitude of magnetic flux in one ring due to current
materials. The innermost layer is of wood of thickness in other ring is maximum when both rings are coplanar.
2.5 cm, the middle layer is of cement of thickness 1.0 cm and
(a) Statement-1 is true, statement-2 is true and statement-
the outermost layer is of brick of thickness 25.0 cm. Find the
2 is correct explanation for statement-1
power (in kilowatt) of the electric-heater. Assume that there
is no heat loss through the floor and the ceiling. The thermal (b) Statement-1 is true, statement-2 is true and statement-
conductivities of wood, cement and brick are 0.125, 1.5 and 2 is NOT correct explanation for statement-1
1.0 watt/m/°C respectively. (c) Statement-1 is true, Statement-2 is false.
34. A circuit is shown in fig. (d) Statement-1 is false, Statement-2 is true.
36. Statement 1 : Sum of work done by the Newton’s 3rd law
pair internal forces, acting between two particles may be
A B zero.
2m F
3 mF Statement 2 : If two particles undergo same displacement
then work done by Newton’s 3rd law pair forces on them is
5 F of opposite sign and equal magnitude.
(a) Statement-1 is true, statement-2 is true and statement-
4m F
2 is correct explanation for statement-1
+ –
(b) Statement-1 is true, statement-2 is true and statement-
6V
2 is NOT correct explanation for statement-1
Find the charge (in µC) on the condenser having a capacity (c) Statement-1 is true, Statement-2 is false.
of 5 mF. (d) Statement-1 is false, Statement-2 is true.

Space for Rough Work


EBD_7801
MT-4 -26 Target IIT
37. Statement 1 : Electric field of a dipole can’t be found using ..Part - C : Chemistry..
only Gauss’s law. (i.e. without using superposition principle)
Statement 2 : Gauss’s law is valid only for symmetrical charge SECTION – I - Single Correct Choice Type
distribution.
This section contains 5 multiple choice questions. Each question
(a) Statement-1 is true, statement-2 is true and statement- has 4 choices (a), (b), (c) and (d) for its answer, out of which ONLY
2 is correct explanation for statement-1 ONE is correct.
(b) Statement-1 is true, statement-2 is true and statement- 39. Which of the following molecule has the longest N – N
2 is NOT correct explanation for statement-1 bond length
(c) Statement-1 is true, Statement-2 is false. (a) N2H4 (b) N2O
(d) Statement-1 is false, Statement-2 is true. (c) H2N2O2 (d) N2O4
38. Statement 1 : When an inductor coil is connected to a cell, alc. KOH
initial current in it is zero. 40. CH3 - CH 2 - CH - CH3 ¾¾¾¾® X (major)
D
|
Statement 2 : When an inductor coil is connected to a cell, Br
the initial emf induced in it is zero.
Br
(a) Statement-1 is true, statement-2 is true and statement- ¾¾¾

Products.
CCl4
2 is correct explanation for statement-1
Last products of the reaction.
(b) Statement-1 is true, statement-2 is true and statement-
2 is NOT correct explanation for statement-1 (a) Racemic mixture (b) Meso compound

(c) Statement-1 is true, Statement-2 is false. (c) Diastereomers (d) Optically active

(d) Statement-1 is false, Statement-2 is true. 41. For first order parallel reaction k1 and k2 are 8 and 2 min –1
respectively at 300K. If the activation energies for the
formation of B and C are respectively 20 and 28.314 kJ/mol
respectively, find the temperature at which B and C will be
obtained in molar ratio of 2 : 1. [Given ln 4 = 1.4]

k1 B
A
k2 C

(a) 379.75 K (b) 300 K


(c) 479.75 K (d) 579.75 K

Space for Rough Work


MOCK IIT - 4 MT-4-27

CH2 – I SECTION – II - Multiple Correct Choice Type


NaH
This section contains 3 multiple choice questions. Each question
42. CH3 - C º C - H ¾¾¾
®(X) (Y)
has 4 choices (a), (b), (c) and (d) for its answer, out of which ONE
OR MORE is/are correct.
O3 H2O2

(Z) 44. The pair(s) of compounds which can not exist together in
aqueous solution is/are
Final products (Z) of reaction are (a) NaH2PO4 and Na2HPO4

COOH (b) NaH2PO4 and NaOH

(a) Et – COOH and (c) Na2CO3 and NaHCO3


(d) NaHCO3 and NaOH
45. Which of the following names is/are correct for the
CH2 – COOH compound Na[CoCl2(NO2)(s-C3H5) (NH3)2]
(b) CH3 – COOH and (a) Sodium allyldiamminedichloronitrito-N-cobaltate(III)
(b) Sodium diamminedichloroallylnitrito-N-cobaltate(III)
(c) Sodium diamminedichlorocyclopropylnitrito-N-
COOH
cobaltate(III)
(c) CH3 – COOH and
(d) Sodium diamminecyclopropylnitrito-N-dichloro-
cobaltate(III)
COOH 46. A 2.5 gm. impure sample containing weak monoacidic base
(Mol wt. = 45) is dissolved in 100ml water and titrated with
(d) CH3 – COOH and 0.5 M HCl. When (1/5)th of the base was neutralized the pH
was found to be 9 and at equivalent point pH of solution is
4.5. Given : All data at 25°C and log 2 = 0.3.
43. The freezing point depression of a 0.10M solution of formic
acid is –0.2046°C. What is the equilibrium constant for the Select correct statement(s)
reaction at 298 K ? (a) Kb of base is less than 10–6
HCOO– (aq) + H2O (l) ƒ HCOOH (aq) + OH– (aq) (b) Concentration of salt at equivalent point is 0.25M
(Given Kf (H2O) = 1.86 K kg mol–1, Molarity = molality) (c) Volume of HCl is used at equivalent point is 100ml.
(a) 1.1 × 10–3 (b) 9 × 10–12 (d) Weight percentage of base in given sample is 80%.
(c) 9 × 10–13 (d) 1.1 × 10–11

Space for Rough Work


EBD_7801
MT-4 -28 Target IIT

SECTION – III - Matrix-Match Type


This section contains 2 questions. Each question contains statements given in two columns, which have to be p q r s t
matched. The statements in Column-I are labelled A, B, C and D, while the statements in Column-II are A p q r s t
labelled p, q, r, s and t. Any given statement in Column-I can have correct matching with ONE OR MORE B p q r s t
statement(s) in Column-II. The appropriate bubbles corresponding to the answers to these questions have to be C p q r s t
darkened as illustrated in the following example: D p q r s t
If the correct matches are A–p, s and t; B–q and r; C–p and q; and D–s and t; then the correct darkening
of bubbles will look like the given.

47. Column I Column II

(A) •• p. p-bond involved in conjugation


N

••
(B) H 2N CH = CH - C º CH q. Every lp present in molecule involved in resonance

••
(C) CH 2 = CH - O- CH3 r. 2p bonds involved in conjugation
••

(D) CH3CH = CHCH = CH2 s. p, s bond resonance

48. Column I Column II

(A) Projectiles successfully used for nuclear disintegration p. a-particles

(B) Natural agents used for nuclear disintegration q. Protons, deutron, neutron
(C) Artificial agents used for nuclear disintegration r. Cosmic rays

(D) Used in transmutation, known as photo-disintegration s. g-rays

Space for Rough Work


MOCK IIT - 4 MT-4-29

SECTION – IV - Integer Answer Type 51. The composition of a sample of Wustite is Fe0.93O1.00. If
This section contains 5 questions. The answer to each of the percentage of the iron is present in the form of Fe (III) is
questions is a single-digit integer, ranging from 0 to 9. The expressed as 90/x then what will be the value of x?
appropriate bubbles below the respective question numbers in the 52. A monobasic acid is dissociated in 0.1 M solution. When
ORS have to be darkened. For example, if the correct answers to 200ml of the acid solution is neutralized by 0.1 M NaOH,
question numbers X, Y, Z and W (say) are 6, 0, 9 and 2, respectively, heat evolved is 244 cal. If the heat of neutralization of a
then the correct darkening of bubbles will look like the following: strong acid with a strong base is –13.7 kcal then calculate
the amount of the molar heat of dissociation (kcal) of the
X Y Z W
acid.
0 0 0 0
53. What will be the total number of cyclic structure as well as
1 1 1 1
stereoisomers possible for a compound with the molecular
2 2 2 2 formula C5H10?
3 3 3 3

4 4 4 4 Section - V - Reasoning Type


5 5 5 5 This section contains 4 reasoning type questions. Each question
6 6 6 6 has 4 choices (A), (B), (C) and (D), out of which ONLY ONE is
7 7 7 7
correct.

8 8 8 8
54. Statement 1 : The boiling point of C2F6 is less than C2H6.
9 9 9 9
Statement 2 : The molecular weight of C2F6 is more than
C2H6.
49. An element X (Atomic mass = 25) exists as X4 is benzene. (a) Statement-1 is true, statement-2 is true and statement-
51g of saturated solution of X in benzene was added to 2 is correct explanation for statement-1
50.0 g of pure benzene. The resulting solution showed a (b) Statement-1 is true, statement-2 is true and statement-
depression of freezing point of 0.55 K. Calculate the solubility
2 is NOT correct explanation for statement-1
of X per 100 g of benzene. (Kf for benzene = 5.5 K kg mol –1)
50. Kp for the process (c) Statement-1 is true, Statement-2 is false.

CuSO4.5H2O(s) CuSO4.3H2O(s) + 2H2O(g) (d) Statement-1 is false, Statement-2 is true.


55. Statement 1 : Froth floatation method is used for
is 1.40 × 10–6 atm2 at certain temperature. If aqueous tension
benefication of galena through ZnS is present as impurity.
at that temperature is 30 torr, then at what relative humidity
of air will CuSO4.5H2O efforsce? Statement 2 : NaCN or KCN solution is very good
depressant to suppress the floating property of PbS.

Space for Rough Work


EBD_7801
MT-4 -30 Target IIT
(a) Statement-1 is true, statement-2 is true and statement- 57. Statement 1 : D0 increases in the order of
2 is correct explanation for statement-1 [CrCl6]3– < [Cr(CN)6]3– < [Cr(C2O4)3]3–
(b) Statement-1 is true, statement-2 is true and statement- Statement 2 : Stronger the ligand field higher will be D0
2 is NOT correct explanation for statement-1 value.
(c) Statement-1 is true, Statement-2 is false. (a) Statement-1 is true, statement-2 is true and statement-
(d) Statement-1 is false, Statement-2 is true. 2 is correct explanation for statement-1
56. Statement 1 : (b) Statement-1 is true, statement-2 is true and statement-
2 is NOT correct explanation for statement-1
(c) Statement-1 is true, Statement-2 is false.
+
O (d) Statement-1 is false, Statement-2 is true.
major

Statement 2 : 2-butene is more stable than 1-butene as it


having more a-H.
(a) Statement-1 is true, statement-2 is true and statement-
2 is correct explanation for statement-1
(b) Statement-1 is true, statement-2 is true and statement-
2 is NOT correct explanation for statement-1
(c) Statement-1 is true, Statement-2 is false.
(d) Statement-1 is false, Statement-2 is true.

Space for Rough Work


RESPONSE SHEET

Name : ..................................
MOCK TEST - 4 Date : .........................
PAPER 1 PAPER 2
1 31 A p q r s t u 1 30 31 32 33 34
2 B p q r s t u 2
3 C p q r s t u 3
4 D p q r s t u 4
32 A p q r s t u 5
5
B p q r s t u 6
6
p q
7
C r s t u
7 8
D p q r s t u
8 9 A p q r s t u
33
9 B p q r s t u 35
34
10 C p q r s t u 36
35 p q r s t u
D 37
36 p q r s t u 38
10 A
p q r s t u
37 B p q r s t u 39
11 A
B p q r s t u 38 C p q r s t u 40
C p q r s t u 39 D p q r s t u 41
p q 40 11 12 13 14 15 42
D r s t u
43
41
44
p q r s t u
42
12 A 45
B p q r s t u 43 46
C p q r s t u 44
47 A p q r s t u
D p q r s t u 45
B p q r s t u
46
C p q r s t u
47
D p q r s t u
48
16 p q
13 49 48 A r s t u
17
14 B p q r s t u
50 18
15 C p q r s t u
51 A p q r s t u 19
D p q r s t u
16 B p q r s t u 20
17 C p q r s t u 21 49 50 51 52 53
18 D p q r s t u 22
19 p q r s t u 23
52 A
p q
24
20 B r s t u
p q
25
21 C r s t u
26
22 D p q r s t u
27
23 53
28 A p q r s t u
24 54
B p q r s t u
25 55
C p q r s t u
26 56
D p q r s t u
27 57 p q r s t u
29 A
28 54
58 B p q r s t u
55
29 59 C p q r s t u
56
30 60 D p q r s t u
57
Telegram @unacademyplusdiscounts

Join Us on Telegram for More Such Books

https://telegram.me/unacademyplusdiscounts

Join us from the above link or search ''unacademyplusdiscounts'' in Telegram


EBD_7801
TEST ASSESSMENT AND ANALYSIS SHEET
Mock Test - 4
Name :……………………................… Test Code : ........…………….. Date & Time of test:…………….............

Marks per PAPER 1 PAPER 2


questions Total Qs. Attempted Correct Net Score Total Qs. Attempted Correct Net Score
PHYSICS
M CQ'S
M CQ > 1 correct
Passage Cum M atching
Integer Answer
Passage Based
Physics Net Score
Sectional % Score
CHEMISTRY
M CQ'S
M CQ > 1 correct
Passage Cum M atching
Integer Answer
Passage Based
Chemistry Net Score
Sectional % Score
MATHEMATICS
M CQ'S
M CQ > 1 correct
Passage Cum M atching
Integer Answer
Passage Based
Maths Net Score
Sectional % Score
STRIKE RATE (Correct Answers/
Questions attempted):
TO TAL NET S CO RE
COMBINED NET S CORE (Paper 1 + Paper 2)

1) Analysis of wrong questions

Reasons for wrong questions No. of ques.


( to be filled after you have attempted wrong questions on your own after the test)
Knew the question and solved after test but did wrong because of calculation mistake (A)
Knew the question and solved after test but did wrong because got confused and applied wrong concept (B)
Did not knew the question and couldn’t solve even after exam (C)
Total Number of q uestions attempted wrong
Note : If some of (A) & (B) is high then you need more practice and also read instructions more carefully whereas if (C) is very high
it means the coverage of topic is not sufficient and you need to improve on it.

2) Analysis of not attempted questions : Divide the questions not attempted in 3 categories
Reasons for unattempted questions No. of ques.
(fill after you have tried unattempted questions on your own after the test)
Easy questions (A)
Average questions (B)
Difficult questions (C)
Total N umber of qu es. not a ttempted
TARGET
IIT MOCK TEST -5
PAPER - 1

GENERAL INSTRUCTIONS :

1. Section I : Q. No. 1 to 5, Q. No. 23 to 27 and Q. No. 45 to 49 are Single Correct Choice Type questions. For this section, 3 marks
will be awarded for correct answer and zero marks for no answer. In all other cases, –1 mark will be awarded.
2. Section II : Q. No. 6 to 8, Q. No. 28 to 30 and Q. No. 50 to 52 are Multiple Correct Choice Type questions. For this section, 4
marks will be awarded for correct answer and zero marks for no answer. In all other cases, –1 mark will be awarded.
3. Section III : Q. No. 9 & 10, Q. No. 31 & 32 and Q. No. 53 & 54 are Multiple Matrix-Match Type questions. Each question
contains statements given in two columns, which have to be matched. The statements in Column-I are labelled A, B, C and D,
while the statements in Column-II are labelled p, q, r, s and t. Any given statement in Column -I can have correct matching with
ONE OR MORE statement(s) in Column-II. 2 marks each will be awarded for darkening the correct bubble in each row. Hence
the maximum marks of each question is 8 marks. No negative mark will be awarded for an incorrectly bubbled answer.
4. Section IV : Q. No. 11 to 14, Q. No. 33 to 36 and Q. No. 55 to 58 are Integer Answer Type questions. In this section, 4 marks
will be awarded for correct answer and –1 mark for each wrong answer. Marks will be awarded only if you have darken the
appropriate bubble.
5. Section V : Q. No. 15 to 20, Q. No. 37 to 42 and Q. No. 59 to 64 are Comprehension Type questions. For this section, 4 marks
will be awarded for correct answer and zero marks for no answer. In all other cases, –1 mark will be awarded.
5. Section V : Q. No. 21 & 22, Q. No. 43 & 44 and Q. No. 65 & 66 are Reasoning [Statement-1(Assertion) and Statement-
2(Reason)] Type questions. In this section, 3 marks will be awarded for correct answer and zero marks for no answer. In all
other cases, –1 mark will be awarded.

Time : 180 minutes Max. Marks : 267

Space for Rough Work


EBD_7801
MT-5 -2 Target IIT
5. The value of the definite integral
..Part - A : Mathematics..
p /2
ì x xü
SECTION – I - Single Correct Choice Type I= ò e x ícos (sin x) cos 2 + sin (sin x)sin 2 ý dx , is
î 2 2þ
0
This section contains 5 multiple choice questions. Each question
has 4 choices (a), (b), (c) and (d) for its answer, out of which ONLY
ONE is correct. 1 p /2 e p /2
(a) [e (cos 1 + sin 1) - 1] (b) (cos 1 + sin 1)
2 2
1. Define ak = (k2 + 1) k! and bk = a1 + a2 + a3 + ........ + ak. Let
1 p /2 e p /2
a100 m (c) (e cos 1 - 1) (d) (cos 1 + sin1 - 1)
= 2 2
b100 n where m and n are relatively prime natural
numbers. The value of (n – m) is equal to
(a) 99 (b) 100 SECTION – II - Multiple Correct Choice Type
(c) 101 (d) 102 This section contains 3 multiple choice quesitons. Each question
2. Assume that f is continuous on [a, b], a > 0 and differentiable has 4 choices (a), (b), (c) and (d) for its answer, out of which ONE
OR MORE is/are correct.
f (a) f (b)
on an open interval (a, b). If = , then there exist
a b
x0 Î (a, b) such that 6. Let e be the eccentricity of a hyperbola and f (e) be the
(a) x0 f ' (x0) = f (x0) (b) f ' (x0) + x0 f (x0) = 0 eccentricity of its conjugate hyperbola then
(c) x0 f ' (x0) + f (x0) = 0 (d) f ' (x0) = x02 f (x0) 3
3. If a, b and c are positive and 9a + 3b + c = 90 then the ò 1fff424
...... f (e)de is equal to
3
maximum value of (log a + log b + log c) is (base of the 1 n times
logarithm is 10)
(a) 3 (b) 2 (a) 4 if n is even (b) 4 if n is odd
(c) 3/4 (d) does not exist (c) 2 if n is even (d) 2 2 if n is odd
4. Given f (z) = the real part of a complex number z. For example,
f (3 – 4i) = 3. If a Î N, n Î N then the value of 7. Given that the two curves arg (z) = p and | z - 2 3i |= r
6
6a
( )
f æç 1 + i 3 ö÷ has the value equal to
n
å log 2 è ø
intersect in two distinct points then ([r] represents integral
n =1 part of r)
(a) 18a2 + 9a (b) 18a2 + 7a (a) r > 3 (b) r = 6
(c) 18a2 – 3a (d) 18a2 – a (c) 0 < r < 3 (d) [r] ¹ 2

Space for Rough Work


MOCK IIT - 5 -3
MT-5

8. Let PM be the perpendicular from the point (1, 2, 3) to x– y


5 2
plane. If OP makes an angle q with the positive direction of (a) tan q = (b) sin q sin f =
3 14
the z-axis and OM makes an angle f with the positive
direction of x-axis, where O is the origin then ( q and f are 1
(c) tan f = 2 (d) cos q cos f =
14
acute angles)

SECTION – III - Matrix-Match Type


This section contains 2 questions. Each question contains statements given in two columns, which have to be p q r s t
matched. The statements in Column-I are labelled A, B, C and D, while the statements in Column-II are A p q r s t
labelled p, q, r, s and t. Any given statement in Column -I can have correct matching with ONE OR MORE B p q r s t
statement(s) in Column-II. The appropriate bubbles corresponding to the answers to these questions have to be C p q r s t
darkened as illustrated in the following example: D p q r s t
If the correct matches are A–p, s and t; B–q and r; C–p and q; and D–s and t; then the correct darkening
of bubbles will look like the following.

9. Column-I Column-II

é 5ù
(A) Domain of f (x) = loge {(ax3 +(a + b) x2 + (b + c)x + c} if b2 – 4ac < 0, a > 0 p. êë –1, 4 úû

ì1 ü
(B) Domain of f (x) = ln tan–1 {(x3 –6x2 + 11x – 6) x (ex – 1)} q. R – í ,1ý
î5 þ

x 2 – 3x + 2
(C) Range of f (x) = r. (–1, ¥ )
x2 + x – 6

2 x x
(D) Range of f (x) = sin + cos s. (1, 2) È (3, ¥ )
4 4

Space for Rough Work


EBD_7801
MT-5 -4 Target IIT
10. Column-I Column-II
é tan -1 x ù
lim
(A) x ® 0 ê x ú , where [.] denotes the greatest integer function, is equal to p. –1
ëê ûú
sin 2 x - a sin x
(B) If the xlim be a finite number then a can be equal to q. 0
®0 x3

e1/ x - e -1/ x
(C) If f (x) =g (x) 1/ x -1/ x , x ¹ 0 , where g(x) = xnh (x), h(x) being a continous function, r. 1
e +e
then n can be equal to
(D) Let f (x) = –1 + x - 1 , g (x) = 2 – x + 1 , then go f (x) is continous if x is equal to s. 2

3
t.
2

11. Find the number of solutions of the equation


SECTION – IV - Integer Answer Type
This section contains 4 questions. The answer to each of the æ ö æ p pö
questions is a single-digit integer, ranging from 0 to 9. The sin -1 ( sin x ) = cosec -1 ç - log x 2÷ in ç - , ÷
è p ø è 2 2ø
appropriate bubbles below the respective question numbers in the
ORS have to be darkened. For example, if the correct answers to 12. A conical vessel is to be prepared out of a circular sheet of
question numbers X, Y, Z and W (say) are 6, 0, 9 and 2, respectively, metal of unit radius. In order that the vessel has maximum
then the correct darkening of bubbles will look like the following: volume, the sectorial area that must be removed from the
sheet is A 1 and the area of the given sheet is A. If
X Y Z W
0 0 0 0 A
= m + n , then find the value of m + n.
1 1 1 1 A1
2 2 2 2 13. ABCD is a regular tetrahedron; A is the origin; AB is the
3 3 3 3 x-axis; ABC lies in the xy-plane; AB = d. Under these
conditions, find the number of possible tetrahedra.
4 4 4 4

5 5 5 5 14. ( )
If ò cos ec 2 x ln cos x + cos 2 x dx

( )
6 6 6 6
= f ( x) ln cos x + cos 2 x + g ( x ) + f ( x ) - x + c,
7 7 7 7

8 8 8 8 æ pö
then, find the value of f 2(x) – g2(x) ç 0 < x £ ÷ .
9 9 9 9 è 2ø

Space for Rough Work


MOCK IIT - 5 MT-5-5
18. The orthogonal trajectories of the family of curves y = cx2 is
SECTION – V- Comprehension Type
This section contains 2 groups of questions. Each group has 3 x2
(a) - y2 = k (b) x + y2 = k
multiple choice questions based on a paragraph. Each question 2
has 4 choices (a), (b), (c) and (d) for its answer, out of which ONLY
x2
ONE is correct. (c) + y2 = k (d) x2 = y + k
2
19. The orthogonal trajectories of the family of curves
Paragraph for question nos. 15 to 17
x2 + y2 = c2 is

From the point P (h, k) three normals are drawn to the x2


(a) + y2 = k (b) x2 – y2 = k
parabola x2 = 8y and m1, m2 and m3 are the slopes of three normals. 2

y2
15. Algebraic sum of the slopes of these three normals is (c) x2 - =k (d) y = kx
2
k-4 20. The orthogonal trajectories of family of curves
(a) Zero (b) x2 + y2 = cx is
h
(a) x2 – y2 = ky (b) x2 + y2 = ky.
k-2 2- k 2 2
(c) –x + y = ky (d) – x2 – y2 = ky.
(c) (d)
h h
16. If two of the three normals are at right angles then the locus SECTION – VI - Reasoning Type
of point P is a conic whose latus rectum is This section contains 2 reasoning type questions. Each question
(a) 1 (b) 2 has 4 choices (A), (B), (C) and (D), out of which ONLY ONE is
(c) 3 (d) 4 correct.
17. If the two normals from P are such that they make
complementary angles with the axis then the locus of point 21. Let f (x) = x3 + ax2 + bx + c be a cubic polynomial with real
P is a conic, then a directrix of conic is coefficients and all real roots. Also | f (i) | = 1 where i = -1 .
(a) 2y – 3 = 0 (b) 2y + 3 = 0 Statement 1 : All 3 roots of f (x) = 0 are zero.
(c) 2y – 5 = 0 (d) 2y + 5 = 0 because
Statement 2 : a + b + c = 0
(a) Statement-1 is true, statement-2 is true and statement-
Paragraph for question nos. 18 to 20
2 is correct explanation for statement-1
(b) Statement-1 is true, statement-2 is true and statement-
Any curve which cuts every member of a given family of 2 is NOT correct explanation for statement-1
curves at right angle, is called an orthogonal trajectory of the family. (c) Statement-1 is true, Statement-2 is false.
(d) Statement-1 is false, Statement-2 is true.

Space for Rough Work


EBD_7801
MT-5 -6 Target IIT
24. A small mass slides down a fixed inclined plane of inclination
x - 4 y + 5 z -1 x - 2 y +1 z
22. Given lines = = and = = q with the horizontal. The coefficient of friction is µ = µ0x
2 4 -3 1 3 2 where x is the distance through which the mass slides down
Statement 1 : The lines intersect and µ0 is a constant. Then the speed is maximum after the
because mass covers a distance of
Statement 2 : They are not parallel.
cosq sin q
(a) Statement-1 is true, statement-2 is true and statement- (a) (b)
m0 m0
2 is correct explanation for statement-1
(b) Statement-1 is true, statement-2 is true and statement- tan q 2 tan q
2 is NOT correct explanation for statement-1 (c) (d)
m0 m0
(c) Statement-1 is true, Statement-2 is false.
(d) Statement-1 is false, Statement-2 is true. 25. Water at temperature 20°C flows from a tap T into a
heated container C. The container has a heating element
(a resistor R) which is generating heat at the rate of P, that
..Part - B : Physics.. may be varied. The rate of water inflow from tap is F = 0.021
litres per minute. The heat generated is sufficient so that the
water in the container is boiling and getting converted into
SECTION – I - Single Correct Choice Type steam at a steady rate. What is the minimum power P
This section contains 5 multiple choice questions. Each question (in watt) that must be generated as heat in the steady state
has 4 choices (a), (b), (c) and (d) for its answer, out of which ONLY in resistor R so that the amount of liquid water in the
ONE is correct. container neither increases nor decreases with time ?
(Neglect other losses of heat, such as conduction from the
23. A bar magnet was pulled away from a hollow coil A as shown. container to the air and heat capacity of container)
As the south pole came out of the coil, the bar magnet next For water, specific heat c = 4.2 kJ kg–1K–1, latent heat of
to hollow coil B experienced a magnetic force vapourisation Lvap = 2.3 MJkg–1, density r = 1000 kg m–3.
T
Up

N S Left Right

A Down

C
N S R
B
(a) to the right (b) to the left (a) 922.6 watt (b) 450.6 watt
(c) upwards (d) equal to zero (c) 606.1 watt (d) 505.2 watt

Space for Rough Work


MOCK IIT - 5 -7
MT-5

26. A capillary tube with inner cross-section in the form of a 28. The diagram shows a modified meter bridge, which is used
square of side ‘a’ is dipped vertically in a liquid of density r for measuring two unknown resistances at the same time.
and surface tension s which wet the surface of capillary When only the first galvanometer is used for obtaining the
tube with angle of contact q. The approximate height to balance point, it is found at point C. Now the first
which liquid will be raised in the tube is (Neglect the effect galvanometer is removed and the second galvanometer is
of surface tension at the corners of capillary tube) used, which gives balance point at D. Using the details
2s cos q 4s cos q given in the diagram, find out the values of R1 and R2.
(a) a rg (b) a rg
R R1 R2
8s cos q
(c) arg (d) None of these

27. A box of mass 1 kg is mounted with two cylinders each of G G


mass 1 kg and moment of inertia 0.5 kg m2 and radius 1m as
shown in figure. Cylinders are mounted on their control axis A B
C D
of rotation and this system is placed on a rough horizontal
surface. The rear cylinder is connected to battery operated
motor which provides a torque of 100N-m to this cylinder AB = L, AC = L/4 and AD = 2L/3
via a belt as shown. If sufficient friction is present between
cylinder and horizontal surface for pure rolling, find (a) R1 = 5R/3 (b) R2 = 4R/3
acceleration of the vehicle in m/s2. (Neglect mass of motor, (c) R1 = 4R/3 (d) R2 = 5R/3
belt and other accessories of vehicle). 29. One mole of a gas is subjected to two processes AB and BC,
one after the other as shown in the figure. BC is represented
Electric motor by PV = constant. We can conclude that (where
T = temperature, W = work done by gas, V = volume and
U = internal energy)

P
////////////////////////////////////// A B
(a) 20 m/s2 (b) 5 m/s2 P0
(c) 15 m/s2 (d) 10 m/s2
C
V
SECTION – II - Multiple Correct Choice Type V0 2V0 3V0
This section contains 3 multiple choice quesitons. Each question
has 4 choices (a), (b), (c) and (d) for its answer, out of which ONE (a) TA = TB = TC (b) VA < VB, PB < PC
OR MORE is/are correct. (c) WAB > WBC (d) TA < TB

Space for Rough Work


EBD_7801
MT-5 -8 Target IIT
30. When two blocks of different masses connected by a (a) their velocities are equal and opposite
stretched spring (as shown) start moving from rest towards (b) their accelerations are equal and opposite
each other under mutual interaction then pickup the correct (c) the force acting on them are equal and opposite
alternative or alternatives. (d) their momenta are equal and opposite
k
m1 m2
/////////////////////////////////////////
smooth horizontal floor

SECTION – III - Matrix-Match Type


This section contains 2 questions. Each question contains statements given in two columns, which have to be p q r s t
matched. The statements in Column-I are labelled A, B, C and D, while the statements in Column-II are A p q r s t
labelled p, q, r, s and t. Any given statement in Column -I can have correct matching with ONE OR MORE B p q r s t
statement(s) in Column-II. The appropriate bubbles corresponding to the answers to these questions have to be C p q r s t
darkened as illustrated in the following example: D p q r s t
If the correct matches are A–p, s and t; B–q and r; C–p and q; and D–s and t; then the correct darkening
of bubbles will look like the following.

31. In each situation of column-I, some charge distributions are given with all details explained. In column-II, the electrostatic potential
energy and its nature is given in colunm II. Then match situation in column I with the corresponding results in column II.
Column I Column II –Q
1 Q2
(A) A thin shell of radius a and having a charge –Q p. in magnitude
8p Î0 a 2
a
uniformly distributed over its surface as shown.
5a 3 Q2 –Q
(B) A thin shell of radius and having a charge –Q q. in magnitude
2 20p Î0 a –Q
uniformly distributed over its surface and a point charge 5a
–Q placed at its centre as shown. 2
–Q

2 Q2
(C) A solid sphere of radius a and having a charge –Q r. in magnitude
5p Î0 a a

uniformly distributed throughout its volume as shown.

Space for Rough Work


MOCK IIT - 5 MT-5-9
(D) A solid sphere of radius a and having a charge –Q s. Positive in sign –Q
uniformly distributed throughout its volume. The solid
sphere is surrounded by a concentric thin uniformly –Q
charged spherical shell of radius 2a and carrying charge
–Q as shown a 2a

27Q2
t. in magnitude
80p Î0 a
32. Column I Column II
(A) A charged capacitor is connected to the ends p. A constant current flows through the wire
of the wire
(B) A wire is moved perpendicular to its length with q. Thermal energy is generated in the wire
a constant velocity in a uniform magnetic field
perpendicular to the plane of motion
(C) A wire is placed in a constant electric field r. A constant potential difference develops between the
that has a direction along the length of the wire ends of the wire
(D) A battery of constant emf is connected to the s. Charge of constant magnitude appear at the ends of
ends of the wire the wire

33. Two coils are wound on the same iron rod so that the flux
SECTION – IV - Integer Answer Type generated by one also passes through the other. The primary
This section contains 4 questions. The answer to each of the has 100 loops and secondary has 200 loops. When a current
questions is a single-digit integer, ranging from 0 to 9. The of 2 A flows through the primary, the flux in it is 25 x 10–4
appropriate bubbles below the respective question numbers in the Wb. If the value of mutual inductance (in henry) between
ORS have to be darkened. For example, if the correct answers to the coils is (0.025 K), find the value of K.
question numbers X, Y, Z and W (say) are 6, 0, 9 and 2, respectively, 34. A block A of mass m1(= 10 kg) rests on a block B of mass
then the correct darkening of bubbles will look like the following: m2 (=20 kg). B rests on fixed surface. The coefficient of friction
X Y Z W
between any two surfaces is µ (=0.3). A and B are connected
0 0 0 0
by a massless string passing around a frictionless pulley
1 1 1 1 fixed to the wall as shown in fig. The force(F) by which A is
2 2 2 2 dragged so as to keep both A and B moving with uniform
3 3 3 3 speed is related to a constant h as F = 10(h + 6), find the
value of h.
4 4 4 4

5 5 5 5
6 6 6 6
T
F A
7 7 7 7 f2 f1
8 8 8 8
B
9 9 9 9 T
f3

Space for Rough Work


EBD_7801
MT-5 -10 Target IIT
35. A point particle of mass 0.1 kg is executing S.H.M. of spring (spring constant k) is connected to the connector ab and is
amplitude of 0.1 m. When the particle passes through the in a relaxed state. The whole system is placed in a uniform magnetic
mean position, its kinetic energy is 8 × 10–3 joule. Obtain field of strength B directed into the plane of rails (figure). Now at
the magnitude (in rad/s) of angular frequency. time t = 0, connector is suddenly given velocity v0 in rightward
36. An ideal monoatomic gas is confined (in mega pascal) in direction. If resistance and self inductance of circuit is negligible
a cylinder by a spring-loaded piston of cross-section then
8.0 × 10–3 m2. Initially the gas is at 300 K and occupies a
volume of 2.4 × 10–3 m3 and the spring is in its relaxed a
(unstretched, uncompressed) state in fig. The gas is heated
by a small electric heater until the piston moves out slowly B
by 0.1 m. The final temperature (in kelvin) of the gas is found spring
to be 100T. What is the value of T? The force constant of C L
the spring is 8000 N/m, atmospheric pressure is 1.0 × 105
Nm–2. The cylinder and the piston are thermally insulated. v0
The piston is massless and there is no friction between the b
piston and the cylinder. Neglect heat loss through lead wires x=0
of the heater. The heat capacity of the heater coil is negligible.
Assume the spring to be massless. 37. The magnitude of acceleration of connector as a function
of x
Open atmosphere kx kx
(a) (b)
m m - B2 L2C
Rigid
Heater support kx
(c) (d) None of these
m + B2 L2 C
38. The maximum compression in the spring

m - B2 L2C m + B2 L2C
(a) v0 (b) v0
SECTION – V- Comprehension Type k k
This section contains 2 groups of questions. Each group has 3
multiple choice questions based on a paragraph. Each question m
(c) v0 (d) None of these
has 4 choices (a), (b), (c) and (d) for its answer, out of which ONLY k
ONE is correct. 39. The rod will execute (after projecting it at t = 0)
(a) SHM
Paragraph for question nos. 37 to 39 (b) Harmonic motion but not SHM
The conducting connector of mass m and length L can freely (c) The rod will come at rest at certain position then
slide on horizontal long conducting parallel rails connected by afterward it will not move
capacitor C at one end as shown in figure. A non-conducting light
(d) None of these

Space for Rough Work


MOCK IIT - 5 -11
MT-5

Paragraph for question nos. 40 to 42 Statement 2 : In YDSE set up magnitude of electromagnetic


The circuit shown is placed in vacuum. Both the capacitors field at central bright fringe is not varying with time.
are identical and they have the same capacitance C. Light is (a) Statement- 1 is True, Statement-2 is True, Statement-2
incident on the left plate of the upper capacitor. When all the is a correct explanation for Statement -1
switches are open then the hf versus KEmax is shown by the (b) Statement -1 is True, Statement -2 is True ; Statement-
straight line (1). In all cases, we are measuring the KEmax when the 2 is NOT a correct explanation for Statement - 1
electron reaches the opposite plate. (c) Statement - 1 is True, Statement- 2 is False
When only the switches S1 and S2 are closed, the graph (d) Statement -1 is False, Statement -2 is True
becomes (2). When only S3 and S4 are closed then the graph
44. Statement 1 : Peak voltage across the resistance can be
becomes (3).
greater than the peak voltage of the source in an series LCR
circuit.
(3) Statement 2 : Peak voltage across the inductor can be greater
hf than the peak voltage of the source in an series LCR circuit.
E1 (a) Statement-1 is true, statement-2 is true and statement-
(1) 2 is correct explanation for statement-1
S3 S2 S4
S1 (2) (b) Statement-1 is true, statement-2 is true and statement-
V 2 is NOT correct explanation for statement-1
–E KEmax (c) Statement-1 is true, Statement-2 is false.
(d) Statement-1 is false, Statement-2 is true.
40. What is the work function of the cathode ?
(a) E (b) E1
(c) E + E1 (d) None of these ..Part - C : Chemistry..
41. What is the value of eV ?
(a) E (b) E1 SECTION – I - Single Correct Choice Type
(c) E + E1 (d) None of these
42. What is the value of E1 ? This section contains 5 multiple choice questions. Each question
(a) 3E (b) 3E/2 has 4 choices (a), (b), (c) and (d) for its answer, out of which ONLY
(c) E/2 (d) None of these ONE is correct.

O Conc. NaOH
SECTION – VI - Reasoning Type 45. ¾¾®
3 (P) ¾¾¾¾¾® (Q)
Zn
This section contains 2 reasoning type questions. Each question
has 4 choices (A), (B), (C) and (D), out of which ONLY ONE is
LiAlH CH COCl
correct. ¾¾¾¾
4 ®(R) ¾¾¾¾
3 ¾
®(S)
excess
Change in molecular weight from (R) to (S) is
43. Statement 1 : In standard YDSE set up with visible light, the
position on screen where phase difference is zero appears (a) 43 (b) 42
bright. (c) 86 (d) 84

Space for Rough Work


EBD_7801
MT-5 -12 Target IIT
46. Which of the following statement is correct regarding the Here X and Y stands for mole fraction in liquid and vapour
complex [CuL6]2+, one solution is blue and another is green. phase respectively.
(a) The blue solution will have higher value of D0 (a) Xbenzene = 0.5 and Ytoluene = 0.2
(b) The green solution will have higher value of D0 (b) Xtoluene = 0.3 and Ybenzene = 0.6
(c) Both have same value of D0 (c) Xbenzene = 0.3 and Ytoluene = 0.4
(d) No prediction can be done regarding D0 (d) if Xbenzene = 0.7 than Ytoluene < 0.3
47. Which of the following will form tri-bromo derivative of 49. The degree of hydrolysis of 0.1 M solution if conjugate
phenol ? base of HA is 0.01. Find the H+ concentration in 0.4 M solution
of A–.
OH
OH (a) 5 × 10–11 M (b) 5 × 10–12 M
–3
(c) 2 × 10 M (d) 2 × 10–4 M
Br2
Br2
(a) H2O (b)
H2O
SECTION – II - Multiple Correct Choice Type
This section contains 3 multiple choice quesitons. Each question
OH OH has 4 choices (a), (b), (c) and (d) for its answer, out of which ONE
OR MORE is/are correct.
Br2 Br2
(c) (d)
CS2 H2O 50. Assume ideal gas behaviour for all the gases considered
and vibrational degrees of freedom to be active. Separate
48. The following graph represents variation of boiling point equimolar samples of Ar, H2, SO2 were subjected to a two
with composition of liquid and vapours of binary liquid step process as mentioned. Initially all are at same state of
mixture. The graph is plotted at constant pressure. temperature and pressure.
Which of the following statement(s) is incorrect ? Step 1 : All undergo reversible adiabatic expansion to attain
same final volume, which is double of the original volume
thereby causing the decreases in their temperature.
Step 2 : After step I all are given appropriate amount of heat
isochorically to restore the original temperature.
Mark the correct option(s) –
Boiling point

(a) Due to step I only, the decrease in temperature will be


maximum for Ar
(b) During step II, heat given will be maximum for SO2
(c) There will be no net change in internal energy for any
of the gas after both the steps of process are completed.
0.0
(d) The P–V graph of Ar and H2 will be different for overall
0.1 0.2 0.3 0.4 0.5 0.6 0.7 0.8 0.9 1.0
Xbenzene=1 process.
Xtoluene=1

Space for Rough Work


MOCK IIT - 5 MT-5 -13
51. Choose the correct options – 52. Choose the correct options for
(a) Volume of air at 8.21 atm and 300 K containing 21% of
CH3
oxygen by volume is required to completely burn 100g
of carbon sample containing 16% inert material, is 100 Na Me – Cl
H OH a I
litre
(b) Among C – C, Si – Si, Ge – Ge and Sn – Sn, C – C has 14CH Red P MeONa
3 b II
the highest bond energy I2
(c) The change in entropy of 2 moles of an ideal gas upon
(a) I and II are identical
isothermal expansion at 243.6 K from 20 litre until the
(b) I and II are different
pressure becomes 1 atm, is 2.77 cal/K
(c) Mechanism of formation of I and II are same
(d) H2SO4 solution (80% by weight and specific gravity
1.75 g/ml) is used to prepare 2 litre of 0.25 M H2SO4 (d) Mechanism of formation of I and II are different
(aq). The volume of H2SO4 solution (original) which
must be used is 35ml

SECTION – III - Matrix-Match Type


This section contains 2 questions. Each question contains statements given in two columns, which have to be p q r s t
matched. The statements in Column-I are labelled A, B, C and D, while the statements in Column-II are A p q r s t
labelled p, q, r, s and t. Any given statement in Column -I can have correct matching with ONE OR MORE B p q r s t
statement(s) in Column-II. The appropriate bubbles corresponding to the answers to these questions have to be C p q r s t
darkened as illustrated in the following example: D p q r s t
If the correct matches are A–p, s and t; B–q and r; C–p and q; and D–s and t; then the correct darkening
of bubbles will look like the following.

53. Column A Column B


(Reaction) (Reaction characteristics or
order of reaction)
(A) Hydrodysis of an ester p. Pseuo-first order
0.693
(B) Saponification of ester q. t1/ 2 =
k

1
(C) Inversion of cane sugar r. t1/ 2 µ
initial conc.
(D) Decomposition of N2O5 in CCl4 solution s. Rate constant k is expressed in units of litre mol–1 sec–1.

Space for Rough Work


EBD_7801
MT-5 -14 Target IIT
54. Column - I Column - II
(A) H4P2O7 p. Reducing agent
(B) H3PO4 q. Contains a minimum of one P – H bond
(C) H3PO3 r. Contains P – O – P linkage
(D) H3PO2 s. Gives metaphosphoric acid on being heated to
a temperature above 600°C

SECTION – IV - Integer Answer Type 56. Find out the number of waves made by a Bohr electron in
one complete revolution in its 3rd orbit.
This section contains 4 questions. The answer to each of the
57. A spherical balloon of 24 cm diameter is to be filled up with
questions is a single-digit integer, ranging from 0 to 9. The
hydrogen at N.T.P. from a cylinder containing the gas at 20
appropriate bubbles below the respective question numbers in the
atmospheres at 27ºC. If the cylinder can hold 2.82 litres of
ORS have to be darkened. For example, if the correct answers to
water, calculate the number of balloons that can be filled up.
question numbers X, Y, Z and W (say) are 6, 0, 9 and 2, respectively,
then the correct darkening of bubbles will look like the following: 58. 20% of surface sites are occupied by N2 molecules. The
density of surface site is 6.023 × 1014 cm–2 and total surface
X Y Z W area is 1000 cm2. The catalyst is heated to 300 K while N2 is
0 0 0 0 completely desorbed into a pressure of 0.001 atm and volume
1 1 1 1 of 2.46 cm3. Find the number of active sites occupied by
each N2 molecule.
2 2 2 2

3 3 3 3

4 4 4 4 SECTION – V- Comprehension Type


5 5 5 5 This section contains 2 groups of questions. Each group has 3
6 6 6 6 multiple choice questions based on a paragraph. Each question
has 4 choices (a), (b), (c) and (d) for its answer, out of which ONLY
7 7 7 7
ONE is correct.
8 8 8 8

9 9 9 9

Paragraph for question nos. 59 to 61


Electrode potential depends on ion concentrations. It is
55. A compound contains 28 percent of nitrogen and 72 percent possible to construct a cell from two half cells composed of the
of metal by weight. 3 atoms of metal combine with 2 atoms same material but differing in ion concentrations. Such a cell is
of N. If the atomic weight of metal is expressed as 20+A. called a concentration cell.
Then find the value of A.

Space for Rough Work


MOCK IIT - 5 MT-5-15
59. Calculate the EMF of the cell at 25°C
Pt | H2 (g, 1 bar) | HCl (aq) || HCl (aq) | H2 (g, 1 bar) | Pt
Ag
pH = 3.95 pH = 1.45
(a) 0.08865V (b) 0.319 V
(c) – 0.14775 V (d) 0.14775 V AgNO3
0.05 M
60. Carefully observe the given figure and using data provided
Beaker (A)
find the EMF (in volt) of shown Galvanic cell :

Ag
H2 (g) H2(g)
1 bar Salt 1 bar
bridge
0.01 M AgNO3

Beaker (B)
Solution - A Solution - B
(a) 9.6 × 10–2 (b) 7.8 × 10–2
Solution A is 0.1 M each in CH3COOH and CH3COONa and
(c) 5.8 × 10–3 (d) None of these
solution B is 0.1 M NH4Cl.
[Given Ka (CH3COOH) = 10–5, Kb (NH4OH) = 10–5 and Paragraph for question nos. 62 to 64
2.303RT Arushi, a student of class XI is working in the chemistry lab
= 0.06 volt ] of her school. She is provided with 4 containers of large capacity
F
(a) 0.06 V (b) 0.12 V by the lab assistant.
(c) 0.24 V (d) 0 V Container 1 contains 2L of 5.6V H2O2.
61. Two beakers A and B contains AgNO 3 solutions of Container 2 contains 2L of 33.6V H2O2.
concentrations 0.05 M and 0.01 M respectively as shown in
figures. Pure silver electrodes are now lowered in each beaker. Container 3 contains sufficient amount of water.
Now NH3 is added in each beaker till conc. of NH3 in beaker Container 4 is empty
A and B become 0.1 M and 0.2 M respectively. If two beakers
She has been asked by her teacher to prepare H2O2 solution
are now connected in Galvanic cell arrangement, find the
EMF (in volts) of the resulting cell at 298 K. using the components of container 1, 2 or 3 (partially or
completely) and store it in container 4.
RT
[Use : 1og10 2 = 0.3, (2.303) = 0.06 at 298 K] , Given : 62. If water can be used, then the volume of water required to
F
prepare maximum volume of 16.8 V H2O2 solution is
E0 = 0.8 volt, K f [Ag(NH3 ) 2+ ] = 108 (a) 0.33 L (b) 0.5 L
Ag + /Ag
(c) 0.67 L (d) 1 L

Space for Rough Work


EBD_7801
MT-5 -16 Target IIT
63. The maximum volume of O2 gas at STP that can be liberated 65. Statement 1 : Carbon reduction for Al2O3 is very difficult.
from the prepared solution is (Water may or may not be
Statement 2 : Al forms carbide the Al4C3 which produces
used)
propyne when reacts with H2O.
(a) 67.2 L (b) 39.2 L
(a) Statement- 1 is True, Statement-2 is True,Statement-2
(c) 19.6 L (d) 78.4 L is a correct explanation for Statement -1

64. If water is not be used, then maximum volume of 16.8V H2O2 (b) Statement -1 is True, Statement -2 is True ; Statement-
solution that can be prepared is – 2 is NOT a correct explanation for Statement - 1

(a) 4.67 L (b) 3.33 L (c) Statement - 1 is True, Statement- 2 is False

(c) 2..67 L (d) 4 L (d) Statement -1 is False, Statement -2 is True

66. Statement 1 : d-bonds are formed by four lobes interaction


of d-orbitals.
SECTION – VI - Reasoning Type
This section contains 2 reasoning type questions. Each question Statement 2 : d z2 orbital does not have nodal plane.
has 4 choices (A), (B), (C) and (D), out of which ONLY ONE is
(a) Statement- 1 is True, Statement-2 is True, Statement-2
correct.
is a correct explanation for Statement -1

(b) Statement -1 is True, Statement -2 is True ; Statement-


2 is NOT a correct explanation for Statement - 1

(c) Statement - 1 is True, Statement- 2 is False

(d) Statement -1 is False, Statement -2 is True

Space for Rough Work


Telegram @unacademyplusdiscounts

Join Us on Telegram for More Such Books

https://telegram.me/unacademyplusdiscounts

Join us from the above link or search ''unacademyplusdiscounts'' in Telegram


MOCK IIT - 5 MT-5-17

PAPER - 2

GENERAL INSTRUCTIONS :

1. Section I : Q. No. 1 to 5, Q. No. 23 to 27 and Q. No. 45 to 49 are Single Correct Choice Type questions. For this section, 3 marks
will be awarded for correct answer and zero marks for no answer. In all other cases, –1 mark will be awarded.
2. Section II : Q. No. 6 to 8, Q. No. 28 to 30 and Q. No. 50 to 52 are Multiple Correct Choice Type questions. For this section, 4
marks will be awarded for correct answer and zero marks for no answer. In all other cases, –1 mark will be awarded.
3. Section III : Q. No. 9 & 10, Q. No. 31 & 32 and Q. No. 53 & 54 are Multiple Matrix-Match Type questions. Each question
contains statements given in two columns, which have to be matched. The statements in Column-I are labelled A, B, C and D,
while the statements in Column-II are labelled p, q, r, s and t. Any given statement in Column -I can have correct matching with
ONE OR MORE statement(s) in Column-II. 2 marks each will be awarded for darkening the correct bubble in each row. Hence
the maximum marks of each question is 8 marks. No negative mark will be awarded for an incorrectly bubbled answer.
4. Section IV : Q. No. 11 to 14, Q. No. 33 to 36 and Q. No. 55 to 58 are Integer Answer Type questions. In this section, 4 marks
will be awarded for correct answer and –1 mark for each wrong answer. Marks will be awarded only if you have darken the
appropriate bubble.
5. Section V : Q. No. 15 to 20, Q. No. 37 to 42 and Q. No. 59 to 64 are Comprehension Type questions. For this section, 4 marks
will be awarded for correct answer and zero marks for no answer. In all other cases, –1 mark will be awarded.
5. Section V : Q. No. 21 & 22, Q. No. 43 & 44 and Q. No. 65 & 66 are Reasoning [Statement-1(Assertion) and Statement-
2(Reason)] Type questions. In this section, 3 marks will be awarded for correct answer and zero marks for no answer. In all
other cases, –1 mark will be awarded.

Time : 180 minutes Max. Marks : 267

..Part - A : Mathematics.. (x + y - 3)2


1. If (x – 1)2 + (y – 1)2 = represents the equation
2
SECTION – I - Single Correct Choice Type of a parabola, then
(a) equation of its axis is y = x + 1
This section contains 5 multiple choice questions. Each question
has 4 choices (a), (b), (c) and (d) for its answer, out of which ONLY æ 1 1ö
(b) its vertex is ç - , ÷
ONE is correct. è 2 2ø
(c) length of latus rectum is 2
(d) None of these
Space for Rough Work
EBD_7801
MT-5 -18 Target IIT
2. Let a, b are real number such that a + b = 1, then the minimum
SECTION – II - Multiple Correct Choice Type
p This section contains 3 multiple choice questions. Each question
value of the integral ò (a sin x + bsin 2x) 2 dx is equal to has 4 choices (a), (b), (c) and (d) for its answer, out of which ONE
0 OR MORE is/are correct.
(a) p/2 (b) p/4
(c) p/8 (d) 3p/4 6. If a, b, c are in H.P., then which of the following is true
3. Triangle ABC is isosceles with AB = AC & Ð CAB = 106°.
æ bö b æ bö
Point M is the interior of the triangle so that Ð MBA =7° and (a) ln ç a - ÷ , ln , ln ç c - ÷ are in H.P..
è 2ø 2 è 2ø
Ð MAB = 23°. The number of degrees in Ð AMC is equal to
(a) 87° (b) 67° a b c
(c) 74° (d) 83° (b) , , are in H.P..
1 - 2a 1 - 2b 1 - 2c
4. If the system of equations
b b b
x - ly - z = 0 (c) c - , , a - are in G.P..
lx - y - z = 0 2 2 2
x+y-z = 0 (d) e1/a, e1/b, e1/c are in G.P.
7. Let z1, z2, z3, z4 be the vertices A, B, C, D respectively of a
has a unique solution, then the value of l can be
square on the argand plane taken in anticlockwise direction,
(a) 0 (b) 2 then
(c) Both (a) and (b) (d) 1 (a) 2z4 = (1 – i) z1 + (1 + i) z3 (b) 2z2 = (1 – i) z1 + (1 + i) z3
5. L1 and L2 are two lines whose vector equations are : (c) 2z4 = (1 + i) z1 + (1 – i) z3 (d) 2z2 = (1 + i) z1 + (1 – i) z3
r The function f (x) = x1/3 (x – 1)
L1 : r = l [(cos q + 3) ˆi + ( 2 sin q) ˆj + (cos q - 3) k]
ˆ , 8.
(a) has 2 inflection points
r
L2 : r = m(aiˆ + bjˆ + ck)
ˆ (b) is strictly increasing for x > 1/4 and strictly decreasing
where l and m are scalars and a is the acute angle between for x < 1/4.
L1 and L2. If the angle a is the independent of q then the (c) is concave down in (–1/2, 0)
value of a is (d) area enclosed by the curve lying in the fourth quadrant
(a) p/6 (b) p/4 (c) p/3 (d) p/2 is 9/28.

SECTION – III - Matrix-Match Type


This section contains 2 questions. Each question contains statements given in two columns, which have to be p q r s t
matched. The statements in Column-I are labelled A, B, C and D, while the statements in Column-II are A p q r s t
labelled p, q, r, s and t. Any given statement in Column -I can have correct matching with ONE OR MORE B p q r s t
statement(s) in Column-II. The appropriate bubbles corresponding to the answers to these questions have to be C p q r s t
darkened as illustrated in the following example: D p q r s t
If the correct matches are A–p, s and t; B–q and r; C–p and q; and D–s and t; then the correct darkening
of bubbles will look like the following.

Space for Rough Work


MOCK IIT - 5 MT-5-19
9. Column-I Column-II
(A) The intercept of the common tangent to the curves y2 = 8x and xy = –1 on the axis of p. –2
y is equal to
(B) Let f be a real function whose derivatives upto third order exist and for some pair q. 0

f (a ) + f '(a) + f "(a) f "'(c )


a,b Î R, a < b; log = a – b,then $c Î (a, b) for which
f (b) + f '(b) + f "(b) f (c )

is equal to
(C) Let f (x) = (x2 – 1) (x2 – 4), anda, b, g be the roots of the equation f '(x) = 0 then [a] + [b] + [g] r. 1
is equal to ([t] represents the integral part of t)

5
(D) If three normals can be drawn to the curve y2 = x from the point (c, 0) then c can be s.
4
equal to t. 2
10. A bag contains some white and some black balls, all combinations being equally likely. The total number of balls in the bag is 12.
Four balls are drawn at random from the bag at random without replacement. Now match the entries from the following two
columns :
Column-I Column-II

14
(A) Probability that all the four balls are black is equal to p.
33

1
(B) If the bag contains 10 black and 2 white balls then the probability that all four balls are q.
5
black is equal to

70
(C) If all the four balls are black then the probability that the bag contains 10 black balls r.
429
is equal to

13
(D) Probability that two balls are black and two are white is equal to s.
165

Space for Rough Work


EBD_7801
MT-5 -20 Target IIT

SECTION – IV - Integer Answer Type 13. Let S1 , S2 ,... squares such that for each n ³ 1 , the length of a

This section contains 4 questions. The answer to each of the side of S n equals the length of a diagonal of S n+1 . If the length
questions is a single-digit integer, ranging from 0 to 9. The of a side of S1 is 10cm, then find the number of values of n, for
appropriate bubbles below the respective question numbers in the
which the area of S n less than or equal to 2 sq. cm.
ORS have to be darkened. For example, if the correct answers to
14. P is a point on the plane ax + by + cz = d. A point Q is taken
question numbers X, Y, Z and W (say) are 6, 0, 9 and 2, respectively,
on the line OP such OP. OQ = d2. If the locus of Q satisfies
then the correct darkening of bubbles will look like the following:
d (ax + by + cz )
X Y Z W = k then find the value of k .
x2 + y2 + z2
0 0 0 0

1 1 1 1
SECTION – IV - Comprehension Type
2 2 2 2 This section contains 2 groups of questions. Each group has 3
3 3 3 3 multiple choice questions based on a paragraph. Each question
has 4 choices (a), (b), (c) and (d) for its answer, out of which ONLY
4 4 4 4
ONE is correct.
5 5 5 5
Paragraph for question nos. 15 to 17
6 6 6 6
A conic C satisfies the differential equation,
7 7 7 7 (1 + y2) dx – xy dy = 0 and passes through the point (1, 0).
8 8 8
An ellipse E which is confocal with C having its eccentricity equal
8
to 2/3.
9 9 9 9
15. Length of the latus rectum of the conic C, is
(a) 1 (b) 2
(c) 3 (d) 4
n n 16. Equation of the ellipse E is
Cr
If n Î N, then find the value of (n + 3) å (-1)
r
11. r +3
Cr x2 y2 x2 y2
r =0 (a) + =1 (b) + =1
3 1 1 3

12. ABCD is a tetrahedron and O is any point. If the lines joining x2 y2 x2 y2


(c) + =1 (d) + =1
O to the vertices meet the opposite faces in P, Q, R, S 4 9 9 4
17. Locus of the point of intersection of the perpendicular
OP OQ OR OS tangents to the ellipse E, is
respectively, then find the value of + + + .
AP BQ CR DS (a) x2 + y2 = 4 (b) x2 + y2 = 10
2 2
(c) x + y = 8 (d) x2 + y2 = 13

Space for Rough Work


MOCK IIT - 5 -21
MT-5

Paragraph for question nos. 18 to 20 22. A triangle is inscribed in a circle of radius R. The length of
Consider the binomial expansion R = (1 + 2x)n = I + f where I the sides of the triangle are 7, 8 and 9 units.
is the integral part of R and f is the fractional part of
Statement 1 : The radius R has an irrational value.
R, n Î N. Also the sum of the coefficients of R is 6561.
Statement 2 : Area of the triangle has an irrational value.
1
18. The value of (n + R – Rf) for x = equals (a) Statement-1 is true, statement-2 is true and statement-
2
2 is correct explanation for statement-1
(a) 7 (b) 8
(b) Statement-1 is true, statement-2 is true and statement-
(c) 9 (d) 10
2 is NOT correct explanation for statement-1
19. If ith terms is the greatest term for x = 1/2, then ‘i’ equals
(a) 4 (b) 5 (c) Statement-1 is true, Statement-2 is false.
(c) 6 (d) 7 (d) Statement-1 is false, Statement-2 is true.
20. If kth terms has greatest coefficient then sum of all possible
value(s) of k is
(a) 6 (b) 7 ..Part - B : Physics..
(c) 11 (d) 13
SECTION – I - Single Correct Choice Type
SECTION – V - Reasoning Type This section contains 5 multiple choice questions. Each question
has 4 choices (a), (b), (c) and (d) for its answer, out of which ONLY
This section contains 2 reasoning type questions. Each question
ONE is correct.
has 4 choices (A), (B), (C) and (D), out of which ONLY ONE is
correct.

23. Two identical pulses move in opposite directions with same


x uniform speed on a stretched string. The width and kinetic
21. Statement 1 : Let f (x) = ò 1 + t 2 dt is an odd function energy of each pulse are L and K respectively. At the instant
0 they completely overlap, the kinetic energy of the width L
and g (x) = f ' (x) is an even function. because of the string is
Statement 2 : For a differentiable function f (x), if f ' (x) is an
even function then f (x) is an odd function. L
/////////////

/////////////
(a) Statement-1 is true, statement-2 is true and statement-
2 is correct explanation for statement-1 L
K
(b) Statement-1 is true, statement-2 is true and statement-
2 is NOT correct explanation for statement-1 (a) K (b) 2K
(c) Statement-1 is true, Statement-2 is false. (c) 4K (d) 8K
(d) Statement-1 is false, Statement-2 is true.
Space for Rough Work
EBD_7801
MT-5 -22 Target IIT
24. A bead of mass m is sliding down the fixed inclined rod 26. One gram of a radioactive sample of half-life 10 min is sealed
without friction. It is fixed to a point P on the horizontal in a capsule at time t = 0. Amount of sample decayed upto 5
surface with a light spring of spring constant k. The bead is min is
initially released from rest and the spring is initially (a) 0.293g (b) 0.5g
unstressed and vertical. The bead just stops at the bottom
of the inclined rod. Find the angle which the inclined rod (c) 0.25g (d) 0.707g
makes with horizontal. 27. A source emitting sound of frequency f0 is moving in a
m circle of radius R, having centre at the origin, with a uniform
speed = c/3, where c = speed of sound. Find the maximum
and minimum frequencies heard by a stationary listener at
h k the point (R/2, 0).

//////////////////////////////////////
P 6f 0 6f 0 2 3f0 2 3f0
(a) , (b) ,
5 7 2 3 -1 2 3 + 1
-1 æ 2mg ö -1 æ 2mg ö
(a) cot ç1 + kh ÷ (b) tan ç1 + kh ÷
è ø è ø 3f 0 3f 0
(c) , (d) None of these
2 5

æ mg ö æ mg ö
cot -1 ç 1 + tan -1 ç1 +
kh ÷ø kh ÷ø
(c) (d) SECTION – II - Multiple Correct Choice Type
è è
This section contains 3 multiple choice questions. Each question
25. The state of an ideal gas is changed through an isothermal
has 4 choices (a), (b), (c) and (d) for its answer, out of which ONE
process at temperature T0 as shown in figure. The work
OR MORE is/are correct.
done by gas in going from state B to C is double the work
done by gas in going from state A to B. If the pressure in the
state B is P0/2 then the pressure of the gas in state C is 28. Two point charges (Q each) are placed at (0, y) and (0, –y).
P A point charge q of the same polarity can move along X-
axis. Then
P0 A
(a) the force on q is maximum at x = ± y / 2
P0 B
C (b) the charge q is in equilibrium at the origin.
2
T0 (c) the charge q performs an oscillatory motion about the
origin
V (d) for any position of q other than origin, the force is
(a) P0/2 (b) P0/4
directed away from the origin
(c) P0/6 (d) P0/8
Space for Rough Work
MOCK IIT - 5 MT-5 -23
29. A block of mass m is kept on a horizontal platform of mass
M. The platform is doing SHM in horizontal plane with
angular frequency w. There is no slipping between the block
and the platform due to friction. Then

m
M

(a) The frictional force on the block is directly proportional


to the displacement of the platform from mean position.
(b) The contact force on the block is directly proportional
to the displacement of the platform from mean position.
(c) The net contact force on the block is directly ///////////////////////////////////////////////
proportional to mass of block (m) (a) Speed of the image of fish formed after reflection from
the mirror as seen by the bird is 3 cm/s
(d) The net contact force on the platform due to the block
is directly proportional to mass of plank (M) (b) Speed of image of bird relative to the fish looking
upwards is 12 cm/s
30. A bird in air is diving vertically over a tank with speed
6 cm/s. Base of the tank is silvered. A fish in the tank is (c) Speed of image of bird relative to the fish looking
rising upward along the same line with speed 4 cm/s. Choose downwards in the mirror is 4 cm/s
the correct options [Take µwater = 4/3] (d) Speed of image of bird relative to the fish looking
downwards in the mirror is 3 cm/s

SECTION – III - Matrix-Match Type


This section contains 2 questions. Each question contains statements given in two columns, which have to be p q r s t
matched. The statements in Column-I are labelled A, B, C and D, while the statements in Column-II are A p q r s t
labelled p, q, r, s and t. Any given statement in Column -I can have correct matching with ONE OR MORE B p q r s t
statement(s) in Column-II. The appropriate bubbles corresponding to the answers to these questions have to be C p q r s t
darkened as illustrated in the following example: D p q r s t
If the correct matches are A–p, s and t; B–q and r; C–p and q; and D–s and t; then the correct darkening
of bubbles will look like the following.

Space for Rough Work


EBD_7801
MT-5 -24 Target IIT
31. A small block lies on a rough horizontal platform above its centre C as shown in figure. The plank is moved in vertical plane such
that it always remains horizontal and its centre C moves in a vertical circle of centre O with constant angular velocity w. There is
no relative motion between block and the plank and the block does not loose contact with the plank anywhere. P, Q, R and S are
four points on circular trajectory of centre C of platform. P and R lie on same horizontal level as O. Q is the highest point on the
circle and S is the lowest point on the shown circle. Match the statements in column I with points in column II.
Q
block
C plank
R P
O

Column I S Column II
(A) Magnitude of frictional force on block is maximum p. when block is at position P
(B) Magnitude of normal reaction on block is equal to mg q. when block is at position Q
(C) Magnitude of frictional force is zero r. when block is at position R
(D) Net contact force on the block is directed towards the centre s. when block is at position S
32. Regarding a conducting wire, match the following
Column I Column II
(A) With increase in temperature p. Drift velocity increases
(B) With increase in length q. Resistance increases
(C) With increase in area of cross-section r. Resistance decreases
(D) With increase in volume s. Number of conduction electrons increases

33. Two parallel vertical metallic rails AB


SECTION – IV - Integer Answer Type and CD are separated by 1 m. They
This section contains 4 questions. X Y Z W are connected at two ends by
0 0 0 0
The answer to each of the questions resistances R1 and R 2 as shown in
is a single-digit integer, ranging from 1 1 1 1 figure. A horizontal metallic bar L of
0 to 9. The appropriate bubbles below 2 2 2 2 mass 0.2 kg slides without friction
the respective question numbers in 3 3 3 3
vertically down the rails under the
A R1 C
the ORS have to be darkened. For action of gravity. There is a uniform
4 4 4 4 L
horizontal magnetic field of 0.6 tesla
example, if the correct answers to 5 5 5 5 perpendicular to the plane of the rails.
question numbers X, Y, Z and W (say) 6 6 6 6 It is observed that when the terminal
are 6, 0, 9 and 2, respectively, then 7 7 7 7 velocity is attained, th e powers
R2
the correct darkening of bubbles will dissipated in R1 and R2 are 0.76 watt
8 8 8 8 B D
look like the following: and 1.2 watt respectively. Find the
9 9 9 9
terminal velocity (in m/s) of the bar L.

Space for Rough Work


MOCK IIT - 5 MT-5 -25
34. A circular disc with a groove along its diameter is placed shown is semicircular in shape. Two identical smooth spherical
horizontally on a rough surface. A block of mass 1 kg is placed balls (of small size) each of mass m and of diameter slightly less
as shown. The co-efficient of friction between the block and than the inner diameter of tube enter into the tube simultaneously
all surfaces of groove and horizontal surface in contact is with a velocity u as shown. (Assume no less of energy anywhere
and all collisions to be elastic).
2
m= . The disc has an acceleration of 25 m/s2 towards left.
5
u
The acceleration (in m/s2) of the block with respect to disc is
4 3
5a, find a. Given cos q = , sin q = .
5 5
u

25 m/s
2
37. Speed of each spherical ball, just before their collision
u 2u
q (a) (b)
3 3 3
35. AB is a cylinder of length 1m fitted with a thin flexible diaphram u 3u
(c) (d)
C at middle and two other thin flexible diaphram A and B at the 2 2
ends. The portions AC and BC contain hydrogen and oxygen 38. The angle between velocity vectors of both balls just before
gases respectively. The diaphrams A and B are set into collision, as observed in the ground frame is
vibrations of the same frequency. The minimum frequency (in
Hz) of these vibrations for which diaphram V is a node is (a) 2 tan -1 2 (b) 2 tan -1 1 / 2
given by 50(30 + n). What is the value of n? Under the
(c) tan -1 2 (d) p
condition of the experiment the velocity of sound in hydrogen
is 1100 m/s and oxygen 300 m/s. 39. The maximum speed of U tube is
36. A parallel plate capacitor is maintained at a certain potential (a) u/2 (b) 2u
difference. When a 3 mm slab is introduced between the plates (c) u (d) None of these
in order to maintain the same potential difference, the distance
between the plates is increased by 2.4 mm. Find the dielectric Paragraph for question nos. 40 to 42
constant of the slab. A loudspeaker system uses alternating current to amplify
sound of certain frequencies. It consists of 2 speakers.
Tweeter-which has smaller diameter produces high
SECTION – V- Comprehension Type frequency sounds. Woffer-which has larger diameter produces
This section contains 2 groups of questions. Each group has 3 low frequency sound. For purpose of circuit analysis, we can take
multiple choice questions based on a paragraph. Each question both speakers to be of equal resistance R. The equivalent circuit
has 4 choices (a), (b), (c) and (d) for its answer, out of which ONLY is shown in the figure. The 2 speakers are connected to the
ONE is correct. amplifier via capacitance and inductance respectively. The
capacitor in tweeter branch blocks the low frequency sound but
Paragraph for question nos. 37 to 39
passes the high frequency. The inductor in woffer branch does
A narrow U shaped tube of mass 2m is placed at rest on a
the opposite.
smooth fixed horizontal surface. The curved surface of tube as
Space for Rough Work
EBD_7801
MT-5 -26 Target IIT
R C SECTION – VI - Reasoning Type
Tweeter This section contains 2 reasoning type questions. Each question
R has 4 choices (A), (B), (C) and (D), out of which ONLY ONE is
correct.
Woofer L
43. When two charged concentric spherical conductors have
~ electric potential V1 and V2 respectively.
V0sin(w t) Statement 1 : The potential at centre is V1 + V2.
40. Which plot correctly represents rms current against Statement 2 : Potential is scalar quantity.
frequency (a) Statement- 1 is True, Statement-2 is True, Statement-2
Tweeter is a correct explanation for Statement -1
I Woofer I (b) Statement -1 is True, Statement -2 is True ; Statement-
2 is NOT a correct explanation for Statement - 1
(a) Tweeter (b) Woofer
(c) Statement - 1 is True, Statement- 2 is False
f f (d) Statement -1 is False, Statement -2 is True
Woofer
44. STATEMENT-1 : A direct uniformly distributed current flows
I I Tweeter through a solid long metallic cylinder along its length. It
produces magnetic field only outside the cylinder.
(c) Tweeter (d) Woofer STATEMENT-2 : A thin long cylindrical tube carrying
f f uniformly distributed current along its length does not
41. What is the frequency which is sounded equally loud by produce a magnetic field inside it. Moreover, a solid cylinder
both speakers can be supposed to be made up of many thin cylindrical
tubes.
1 R2 1 1 4R 2 1 (a) Statement- 1 is True, Statement-2 is True, Statement-2
(a) - (b) -
2p L2 LC 2p L2 LC is a correct explanation for Statement -1
(b) Statement -1 is True, Statement -2 is True ; Statement-
1 1 R2 1
2 is NOT a correct explanation for Statement - 1
(c) - 2 (d)
2p LC 4L 2p LC (c) Statement - 1 is True, Statement- 2 is False
42. For a combination of L, R and C the current in woofer and (d) Statement -1 is False, Statement -2 is True
tweeter are always found to have a phase difference of p/2.
What is the relation between L, R and C.
..Part - C : Chemistry..
(a) L = 2R2C (b) L = 2R 2 C

R 2C
SECTION – I - Single Correct Choice Type
(c) L = R2C (d) L= This section contains 5 multiple choice questions. Each question
2
has 4 choices (a), (b), (c) and (d) for its answer, out of which ONLY
ONE is correct.

Space for Rough Work


MOCK IIT - 5 MT-5-27
45. How many grams of sucrose (mol. wt. = 342) should be
dissolved in 100gm water in order to produce a solution
SECTION – II - Multiple Correct Choice Type
This section contains 3 multiple choice quesitons. Each question
with 105°C difference between the freezing point and boiling
has 4 choices (a), (b), (c) and (d) for its answer, out of which ONE
point temperatures ?
OR MORE is/are correct.
(Unit : kf = 1.86 K.kg mol–1, kb = 0.51 K.kg mol–1)
(a) 34 gm (b) 46 gm 50. Two moles of an ideal gas (Cv,m = 3R/2) is subjected to
(c) 72 gm (d) 342 gm following change of state.
46. In FCC unit cell, what fraction of edge is not covered by A (500K, 5.0 bar) B C (250K, 1.0 bar)
Reversible Isochoric
atoms isothermal cooling Single stage
(a) 0.134 (b) 0.24 expansion adiabatic
(c) 0.293 (d) None of these compression
D
OH (3 bar)

The correct statement is/are


(a) The pressure at B is 2.0 bar
P.C.C.
¾¾¾® (P) HO OH
47. (b) The temperature at D is 450 K
(2mole) (1 mole) (c) DHCD = 1000 R
(d) DUBC = 375 R
CH2OH 51. Which of the following exhibits the phenomenon of
tautomerism ?
(i) CH MgBr
3 Lucas reagent
(Q) ¾¾¾¾¾® (R) ¾¾¾¾¾¾
® (S) CHO O
(ii) HÅ / H 2O
The product(s) show white ppt.
(a) Immediate (b) After 5 min. (a) (b)
(c) No ppt. at room temperature
(d) No reaction
O
Et
O
Keq
48.
Et
Me Me (c)
Equilibrium constant for above reaction is –
(a) K = 1 (b) K > 1
(c) K < 1 (d) K = 0
(d) O O
49. Which isomeric form of complex compound [Ma4bc]n± gives
three isomers as product by replacing (a) by (b)
[where a, b, c are monodentate simple ligands] 52. Which of the following compounds on direct heating cannot
(a) Trans (b) Cis produce anhydrous form of it
(c) Cis-trans both (d) None of these (a) FeCl3.6H2O (b) CuSO4.5H2O
(c) MgCl2.6H2O (d) ZnCl2.2H2O
Space for Rough Work
EBD_7801
MT-5 -28 Target IIT

SECTION – III - Matrix-Match Type


This section contains 2 questions. Each question contains statements given in two columns, which have to be p q r s t
matched. The statements in Column-I are labelled A, B, C and D, while the statements in Column-II are A p q r s t
labelled p, q, r, s and t. Any given statement in Column -I can have correct matching with ONE OR MORE B p q r s t
statement(s) in Column-II. The appropriate bubbles corresponding to the answers to these questions have to be C p q r s t
darkened as illustrated in the following example: D p q r s t
If the correct matches are A–p, s and t; B–q and r; C–p and q; and D–s and t; then the correct darkening
of bubbles will look like the following.
53. Column-I Column-II
O COOH
|| NaBH4
(A) CH3 – C – COOH H OH p. Stereospecific
CH3
(+) – (100%)

CH3 CH3
H
Br2 H Br
(B) C=C q. Stereoselective
H CH3 H Br
CH3
meso (100%)

Br
C6H5 H
(C) C6H5 H alc.
C=C r. anti-Addition
H C6H5 KOH
H C6H5
CH3
CH3 CH3
H2 / Pd.
(D) H3CC º C CH3 BaSO4
C=C s. anti-Elimination
H H
(95%)
54. Column I Column II
(A) Hydrogen bonding p. Depend on temperature
(B) Dispersion forces q. Do not depend on temperature
(C) Dipole-dipole force r. This type of forces are
involved in solidification of I2
(D) Dipole-induced dipole forces s. All molecular forces
involved in solution of HBr in carbon tetrachloride

Space for Rough Work


MOCK IIT - 5 MT-5 -29

SECTION – IV - Integer Answer Type SECTION – V- Comprehension Type


This section contains 4 questions. The answer to each of the This section contains 2 groups of questions. Each group has 3
questions is a single-digit integer, ranging from 0 to 9. The
multiple choice questions based on a paragraph. Each question
appropriate bubbles below the respective question numbers in the
ORS have to be darkened. For example, if the correct answers to has 4 choices (a), (b), (c) and (d) for its answer, out of which ONLY
question numbers X, Y, Z and W (say) are 6, 0, 9 and 2, respectively, ONE is correct.
then the correct darkening of bubbles will look like the following:
X Y Z W
Paragraph for question nos. 59 to 61
0 0 0 0
Aromatic compounds are those that meet the following
1 1 1 1
criteria :
2 2 2 2
(i) The structure must be cyclic, containing (4n + 2) p electrons
3 3 3 3
(ii) Each atom in the ring must have unhybridized p-orbitals.
4 4 4 4
(iii) Unhybridized p-orbital must overlap orbitals.
5 5 5 5
(iv) Structure must be planar.
6 6 6 6
In anti-aromatic compound (4np) electron are delocalization
7 7 7 7
but delocalization of pi electrons over the ring increases the
8 8 8 8
electronic energy.
9 9 9 9
59. Which of the following compound is aromatic ?

55. A mixture of HCOOH and H2 C 2 O 4 is heated with Å H Å Å H


concentrated H2SO4. The gas produced is collected and on N N H–N N
(a) H (b) H
treating with KOH solution, the volume of gas decreases by
one-sixth. Calculate the molar ratio of the two acids
(HCOOH : H2C2O4) in the original mixture.
56. Find the orbital angular momentum, of 2s-orbital of hydrogen Å
atom in units of h/2p. H–N N–H H–N N–H
(c) (d)
57. An intimate mixture of ferric oxide, Fe2O3, and aluminium,
Al, is used in solid fuel rockets. Calculate the fuel value per
cc of the mixture. Heats of formation and densities are as 60. Which of the following is anti aromatic
follows :
H f (Al2O3) = 399 kcal/mole; H f (Fe2O3) = 199 kcal/mole;
Density of Fe2O3 = 5.2 g/cc; Density of Al = 2.7 g/cc. (a) (b)
58. Substances A and B react to form C and D and the reaction
is found to be second order overall and second order in A.
The rate constant at 300 K is 0.500 L mol–1 min–1. What is (c) (d) All of these
the half-life of A (in min.) when 0.4 M of A is mixed with O
excess of B ?
Space for Rough Work
EBD_7801
MT-5 -30 Target IIT
61. In which of following reaction product formed is aromatic–
SECTION – VI - Reasoning Type
OH This section contains 2 reasoning type questions. Each question
has 4 choices (A), (B), (C) and (D), out of which ONLY ONE is

HÅ correct.
(a) (b) D
D
65. Statement 1 : Electrical conductance of aqueous solution
of NaCl is greater than that of LiCl if both solutions have
same concentration of salts.
(c) KH (d) All of these
D Statement 2 : Mobility of Na+(aq) is greater than mobility of
Li+(aq) due to smaller size of Na +(aq).
(a) Statement- 1 is True, Statement-2 is True, Statement-2
Paragraph for question nos. 62to 64 is a correct explanation for Statement -1
Using dil. H2SO4 gas evolution takes place from several
(b) Statement -1 is True, Statement -2 is True ; Statement-
acid radicals. If we consider their smell, colour and other
2 is NOT a correct explanation for Statement - 1
observation if any, several acid radicals can be detected in this
step. (c) Statement - 1 is True, Statement- 2 is False
62. Using dil. H2SO4, which of the acid radical cannot be (d) Statement -1 is False, Statement -2 is True
confirmed.
(a) NO2– (b) CH3CO2– O
||
(c) HSO3– (d) S2– 66. NaOH
Statement 1 : Cl3C - C - H ¾¾¾® Cl3C – CH2OH +
63. When the aqueous suspension of Ag2SO3 and Ag2CO3 are Cl3C – COONa
heated, the respective ppts. are obtained as –
Statement 2 : There are no a-H in this compound, so it can’t
(a) Ag2O and Ag2O (b) Ag2O and Ag
give aldol.
(c) Ag and Ag2O (d) Ag and Ag
(a) Statement- 1 is True, Statement-2 is True, Statement-2
64. Which of the following radical(s) is/are producing same gas
is a correct explanation for Statement -1
on treated with (Zn + dil. H2SO4)
I. SO32– II. HSO3– (b) Statement -1 is True, Statement -2 is True ; Statement-
2 is NOT a correct explanation for Statement - 1
III. S2– IV. Cl–
(a) I and II only (b) I, II and III only (c) Statement - 1 is True, Statement- 2 is False
(c) I, II, III and IV (d) II, III and IV only (d) Statement -1 is False, Statement -2 is True

Space for Rough Work


RESPONSE SHEET

Name : ..................................
MOCK TEST - 5 Date : .........................
PAPER 1 PAPER 2
1 33 34 35 36 1 33 34 35 36
2 2
3 3
4 4
5 5
6 6
7 7
8 8
9 A p q r s t u p q r s t u
9 A
B p q r s t u B p q r s t u
C p q r s t u 37 C p q r s t u 37
D p q r s t u 38 D p q r s t u 38
10 A p q r s t u 39 10 A p q r s t u 39
B p q r s t u 40 B p q r s t u 40
C p q r s t u 41 p q 41
C r s t u
42 42
D p q r s t u D p q r s t u
43 43
11 12 13 14 11 12 13 14
44 44
45 45
46 46
47 47
48 48
49 49
50 50
51 51
52 52
53 A p q r s t u 53 A p q r s t u
16 B p q r s t u 16 B p q r s t u
C p q r s t u C p q r s t u
17 17
D p q r s t u D p q r s t u
18 18
54 A p q r s t u 54 A p q r s t u
19 19
B p q r s t u B p q r s t u
20 20
C p q r s t u C p q r s t u
21 D p q r s t u 21 p q
D r s t u
22 22
55 56 57 58 55 56 57 58
23 23
24 24
25 25
26 26
27 27
28 28
29 29
30 30
31 A p q r s t u A p q r s t u
31
B p q r s t u B p q r s t u
59 59
C p q r s t u 60 C p q r s t u 60
D p q r s t u 61 D p q r s t u 61
32 A p q r s t u 62 p q r s t u 62
32 A
63 63
B p q r s t u B p q r s t u
64 64
C p q r s t u 65 C p q r s t u 65
D p q r s t u 66 D p q r s t u 66
EBD_7801
TEST ASSESSMENT AND ANALYSIS SHEET
Mock Test - 5
Name :……………………................… Test Code : ........…………….. Date & Time of test:…………….............

Marks per PAPER 1 PAPER 2


questions Total Qs. Attempted Correct Net Score Total Qs. Attempted Correct Net Score
PHYSICS
MCQ'S
MCQ > 1 correct
Multiple Matching
Integer Answer
Passage Based
Assertion Reason
Physics Net Score
Sectional % Score
CHEMISTRY
MCQ'S
MCQ > 1 correct
Multiple Matching
Integer Answer
Passage Based
Assertion Reason
Chemistry Net Score
Sectional % Score
MATHEMATICS
MCQ'S
MCQ > 1 correct
Multiple Matching
Integer Answer
Passage Based
Assertion Reason
Maths Net Score
Sectional % Score
STRIKE RATE (Correct Answers/
Questions attempted):
TOTAL NET SCORE
COMBINED NET SCORE (Paper 1 + Paper 2)
1) Analysis of wrong questions

Reasons for wrong questions No. of ques.


( to be filled after you have attempted wrong questions on your own after the test)
Knew the question and solved after test but did wrong because of calculation mistake (A)
Knew the question and solved after test but did wrong because got confused and applied wrong concept (B)
Did not knew the question and couldn’t solve even after exam (C)
Total Number of questions attempted wrong
Note : If some of (A) & (B) is high then you need more practice and also read instructions more carefully whereas if (C) is very high
it means the coverage of topic is not sufficient and you need to improve on it.

2) Analysis of not attempted questions : Divide the questions not attempted in 3 categories
Reasons for unattempted questions No. of ques.
(fill after you have tried unattempted questions on your own after the test)
Easy questions (A)
Average questions (B)
Difficult questions (C)
Total Number of ques. not attempted
Telegram @unacademyplusdiscounts

Join Us on Telegram for More Such Books

https://telegram.me/unacademyplusdiscounts

Join us from the above link or search ''unacademyplusdiscounts'' in Telegram


SOLUTIONS - MOCK TESTS 1
SOLUTIONS - MOCK IITs
MOCK IIT - 1
ANSWER KEY - PAPER 1
1 (a, b, c, d) 13 (d) 25 (a,d). 37 (a,b,d) 49 (d)
2 (a, b, c, d) 14 (b) 26 4 38 (a,b,c,d) 50 (b)
3 (a, b) 15 (c) 27 5 39 (a,b) 51 (a), (b)
4 (a, b, c, d) 16 (c) 28 7 40 (a,b,c) 52 (c)
5 (b) 17 (a) 29 7 41 (a, c) 53 (b)
6 (c, d) 18 (d) 30 4 42 (c, d) 54 (d)
7 (a, c) 19 (a,c) 31 (c) 43 (b, d)
8 (0) 20 (a,b,c) 32 (b) 44 8
9 (0) 21 (a,b,c,d) 33 (d) 45 7
10 (2) 22 (a,d) 34 (c) 46 1
11 (1) 23 (a,b,c) 35 (d) 47 3
12 (2) 24 (c,d) 36 (a) 48 5

EXPLANATORY NOTES

PAPER - 1 3p
S = T1 + T2 + ...... + Tn = p – tan–1 (1) =
1. (a, b, c, d) 4

f ' (x) = 3x2 + 2px + q < 0,


(b) l = lim n sin (2 p 1 + n 2 - 2np)
n ®¥
æ 5 ö –1
x Î ç - , -1÷ –5/3
è 3 ø
æ 2p( 1 + n 2 - n) ö
= lim n sin ç ´ ( 1 + n 2 + n)÷
Þ 3x2 + 2px + q = (3x + 5) (x + 1) n ®¥ çè ÷ø
1 + n 2 + n)
Þ 2p = 8 ; p = 4 & q = 5 Þ p + q = 9
2. (a, b, c, d)
æ æ 2p ö ö
æ 2ö æ 2 ö ç n sin ç ÷ ÷
(a) Tn = tan -1 ç 2 ÷ = tan -1 ç ÷ ç è 1+ n2 + n ø æ 2p ö÷
èn ø è 1 + (n 2 - 1) ø = lim ç ç ÷
n ®¥ æ 2p ö è 1 + n2 + n ø ÷
ç ÷
çè çè 1 + n 2 + n ÷ø ÷ø
æ 2 ö
= tan -1 ç
è 1 + (n - 1)(n + 1) ø÷
2np 2p
-1 æ (n + 1) - (n - 1) ö = nlim = =p
= tan çè 1 + (n - 1)(n + 1) ø÷
®¥ æ 1 ö 2
n ç 1 + 2 + 1÷
è n ø
= tan–1 (n + 1) – tan –1 (n – 1)
(c) f(x) = sin22x + (1 – sin22x)2 + 2
T1 = tan–1 (2) – tan–1 (0)
= sin22x + 1 + sin42x – 2 sin22x + 2
T2 = tan–1 (3) – tan–1 (1)
= 3 + sin42x – sin22x = 3 – sin22x (1 – sin22x)
T3 = tan–1(4) – tan–1 (2)

M M M sin 2 4x
= 3 – sin22x . cos2 2x = 3 -
4
Tn–1 = tan–1(n) – tan–1 (n – 2)
Tn = tan–1 (n + 1) – tan –1 (n – 1) p
Þ period of f(x) is
–––––––––––––––––––––––––– 4
EBD_7801
2 Target IIT

1
4. (a, b, c, d)
ò (1 + x)
1/ 2 3/ 2
(d) I = (1 - x) dx
-1 y2 x2 (0,1/4)
(a) - =1
1/16 1/ 9
1
1/ 2
(1 - x )1/ 2 (1 - x ) dx Locus will be the
= ò (1 + x ) (0,–1/4)
-1 auxillary circle

1 x2 + y2 = 1/16
= ò 1 - x (1 - x ) dx
2
(b) The point with slope 2 and 3 are normal at (4, – 4) ; (9, –6)
-1
where there is no curve, point of normal (am 2, –2am)
Put x = sin q Þ dx = cos q dq
xx1 yy1 x.ae y.b 2 ex y
p/2
(c) T: - ; - = 1 or - =1
a2 b2 a2 a.b 2 a a
\ I= ò cos q (1 - sin q ) cos qdq
-p / 2
or ex – y = a Þ m = e

p/2 p/2 (d) Chord of contact of (h, k)


= ò cos 2 q dq - ò cos 2 q sin q dq
-p / 2 -p / 2 ky = 2a (x + h). It passes through (–a, b)

Þ bk = 2a (– a + h) Þ Locus is y = 2a (x – a).
p/2 p/2
sin 2q q cos3 q
= + +
4 2 -p / 2 3 -p / 2
(h,k)
T
p p 2p p
= + +0= = Q
4 4 4 2

3. (a, b) P
(–a,b)
r r r F
(a) a.(b - c) = 0
r r r r r
Þ Either b = c or angle between a and b - c is 90°
Þ (a) is not correct.
r r r
(b) a ´ (b - c) = 0
r r r r r 5. (b) P(x,y,z)
Þ Either b = c or a and b - c are collinear
Þ (b) is not correct. ^ ^ ^
p = i – j + 2k
(1,1,0) A
r r r r r r
(c) a.(b - c) = 0 and a ´ (b - c) = 0 B (2,0,2)
r r
Either b = c r r ù
Þ r and
or angle between a & (b - c) is 90° úû ^ ^ ^
q = –i + j + 0k
r r
Either b = c r r ù
r Lines intersect at (2, 0, 2). Equation of the plane is
or angle between a & (b - c) is 0 úû

r r r r r x-2 y z-2
a and b - c are collinear. Hence, b = c
-1 1 0 =0 Þ x + y– 2 = 0
(c) is correct. 1 -1 2
(d) (d) is true, refer reciprocal system of vectors.
SOLUTIONS - MOCK TESTS 3

x+ 2
dy (e x + 1) – e x ( x + 4)
6. (c, d) g ( x + 2) = ò f (t )dt Also dx =
(e x + 1) 2
0

dy
2 x+ 2 x For critical point =0
dx
ò f (t ) dt + ò ò
f (t ) dt = g (2) + f (t ) dt
0 2 0 Þ ex(x + 3) = 1
Þ x + 3 = e–x
\ g ( x + 2) = g (2) + g ( x )
Its solution will be intersection point of y = x + 3 and y = e–x
Þ g ( x ) is periodic with period 2

3
2 1 2
x+
Also, g (2) = ò f (t )dt = ò f (t )dt + ò f (t )dt y=
0 0 1
y = e–x
1 0 (0, 1)
= ò f (t )dt + ò f (t )dt [putting t = u + 2]
0 -1
–1

1
= ò f (t ) dt = 0 [Q f(x) is odd]
Clearly there is a critical point in (–1, 0).
-1

8. Ans : 0
\ g (2n) = 0
Here, tr+1 = n Cr (a - r )(b - r )(c - r )( -1) r
[Q g(x) is periodic with period 2]

n n

-x u
Now, å tr +1 = å n Cr (-1)r (a - r ) (b - r ) (c - r )
r =0 r =0
Also, g ( - x ) = ò f ( t ) dt = ò f ( -u ) ( -du )
0 0
n
Þ S = å n Cr (-1)r {abc + (a + b + c) r 2 - ( ab + bc + ca )r - r 3 }
x
= ò f ( u ) du (Q f ( x ) is odd )
r =0

0
ìï n üï
= g ( x)
Þ å
S = abc í (-1) r n Cr ý
îïr =0 ïþ

dy ex 1 ìï n üï
+ y=
7. (a, c)
dx 1 + e x
1 + ex å
+ (a + b + c ) í ( -1) r r 2 n Cr ý
ïî r = 0 ïþ
I.F. = 1 + ex
\ Soln : y(1 + ex) = x + c ìï n üï n
y(0) = 2 Þ c = 4
å
- (ab + bc + ca ) í ( -1) r r n Cr ý - å
( -1) r r 3 n Cr
îï r =0 ïþ r =0

x+4
\ y= x n
e +1 Since, å (-1)r nCr = 0
r =0
\ y(–4) = 0
EBD_7801
4 Target IIT
11. Ans : 1
n
d
Also, å (-1) r Cr = (1 - x) n |x =1 = 0
r
dx
n Let At, t = 1, 2, ...... 6 be the set of days on which the friend is
present at dinner and Bt be the set of days on which the
r =0
friend is absent at dinner. Then | At | = | Bt | = 7
Similarly,
Also | Ai Ç A j | = 7, | Ai Ç A j Ç Ak | = 4
n
d
å (-1) r 2 n
r Cr = value of
dx
(nx(1 - x )n -1 |x =1 = 0
| Ai Ç A j Ç Ak Ç Al | = 3, | Ai Ç A j Ç Ak Ç Al Ç Am | = 2
r =0

Hence, S = abc (0) + (a + b + c) (0) – (ab + bc + ca) (0) – 0 and | A1 Ç A2 Ç A3 Ç A4 Ç A5 Ç A6 | = 1


\ S = 0. where i, j, k, l, m, vary from 1 to 6 and are distinct. Now the
9. Ans : 0 number of dinners at which at least one friend was present
Let z = i i = | A1 È A2 È ........ È A6 |
p -p
Þ ln z = i ln i = - or z = e 2 > 0 Þ arg ( z ) = 0 = å | Ai | - å | Ai Ç A j | + å | Ai Ç A j Ç Ak |
2
10. Ans : 2
- å | Ai Ç A j Ç Ak Ç Al | + å | Ai Ç A j Ç Ak
cos p - sin p 1
Ç Al Ç Am | - | A1 Ç A2 Ç ...... Ç A6 |
Let D = sin p cos p 1
cos ( p + q ) - sin ( p + q ) 1 = 6 C1 ´ 7 - 6 C2 ´ 5 + 6 C3 ´ 4 - 6 C4
Applying R3 : R3 - cos qR1 + sin qR2
´ 3 + 6 C5 ´ 2 - 6 C6 ´ 1 = 13
cos p - sin p 1 Total number of dinners is | At | + | Bt | = 7 + 7 = 14
D = sin p cos p 1
\ Number of dinners the person had alone = 14 – 13 = 1
0 0 1 + sin q - cos q
12. Ans : 2

On expanding we get D = 1 + sin q – cos q (Q sin q ¹ cos q )


To solve for x, y, z Y

cos q + 1 - sin p 1
B
Dx = - sin q + 1 cos p 1
C'
2 - sin ( p + q ) 1

Apply C1 : C1 - C3 first, then R3 : R3 - cos qR1 + sin R2 and D


C
expanding we get Dx = (1 + sin q - cos q ) cos ( p + q )
B'
Similarly, Dy = (1 + sin q - cos q ) sin ( p + q ) and a a X
O A (1, 0)
cos p - sin p cos q + 1
Dz = sin p cos p - sin q + 1
sin ( p + q ) - sin ( p + q ) 2
From figure OB = tan a and Ð COA = a
Applying R3 : R3 - cos qR1 + sin qR2 and expanding we get
So, equation of line OC is y = x tan a and AB = sec a
Dz = 1 + sin q - cos q
So, in DBDC angle BCD = (180° - 2a) ( Q ÐCOA = a )
D Dy D
\x = x ; y = ; z = z x = cos (p + q); y = –sin (p + q);
x D D Þ BD = BC sin(180 - 2a )

z=1 Þ x 2 + y 2 + z 2 = 2.
SOLUTIONS - MOCK TESTS 5

1 1
= BC ´ sin 2a = ´ sec a ´ 2sin a ´ cos a = sin a = tan a (1 - x1 ) - y1
2 2

Þ BD = B ' D ' = sin a


1 æ 2 tan 2 a ö tan a ( tan 2 a - 1)
= tan a ç1 - ÷-
Clearly D BDC and D BDC ' are congruent 2 è 1 + tan 2 a ø 1 + tan 2 a

1
BC ' = BC = sec a and ÐABB ' = ÐB ' BC ' ...(2) tan a (1 - tan 2 a )
2 =
1 + tan 2 a
area of D ABB ' AB ´ BB ' ´ sin ÐABB '
Now, = area of D BB ' C '
area of D BB ' C ' BB '´ BC ' ´ sin ÐB ' BC '

0 tan a 1 0 tan a 1
AB sec a 1 1
= = =2 (Since ÐABB ' = ÐB ' BC ' ) = x1 y1 1 = x1 y1 - tan a 0
BC ' 1 2 2
sec a
2 x2 y2 1 x2 y2 - tan a 0

ALTERNATE METHOD :
1
= é x1 ( y2 - tan a ) - x2 ( y1 - tan a ) ûù
From figure OB = tan a. So, the coordinates of B are 2 ë
(0, tan a) and the equation of OC is y = x tana
The equation of BB' is 1 2 tan 2 a ìï 3 tan 3 a - tan a üï
= í - tan a ý
2 1 + tan 2 a îï 2 (1 + tan a )
2
y - tan a = - cot a ( x - 0 ) Þ x cot a + y = tan a þï

Now B' ( x1, y1 ) is given by 3 tan 2 a - 1 ìï tan 3 a - tan a üï


- í - tan a ý
2 (1 + tan 2 a ) ïî 1 + tan a
2
ïþ
x1 - 0 y1 - tan a -2 ( - tan a )
= =
tan a -1 1 + tan 2 a
1 tan 3 a ( tan 2 a - 3 ) tan a ( 3 tan 2 a - 1)
= +
\ x1 =
2 tan 2 a
, y1 =
tan 3 a - tan a 2 (1 + tan 2 a )2 (1 + tan 2 a )2
1 + tan 2 a 1 + tan 2 a

æ1 1 ö 1 tan a ( tan 4 a - 1) 1 tan a (1 - tan 2 a )


= =
Further C is ç , tan a ÷ , so C ' ( x2 , y2 ) is given by 2 (
1 + tan 2 a )
2
è2 2 ø 2 1 + tan 2 a

1 1 æ1 1 ö 1
x2 - y2 - tan a -2 ç cot a + tan a - tan a ÷ = ( area of D ABB ')
2= 2 = è 2 2 ø 2
cot a 1 2
cot a + 1
area of DABB'
\ = 2
area of DBB'C'
3 tan 2 a - 1 tan a ( 3 tan 2 a - 1)
\ x2 = , y2 = 13. (d) SinceR : {(x, y) : 2x2 + 3y2 – 5xy = 0}
2 (1 + tan 2 a ) 2 (1 + tan 2 a )
For R ® R
Now area of D ABB ' x Rx Û 2x2 + 3x2 – 5x2 = 0 (True) and so reflexive.
14. (b) Since R : N ® N; R = {(x, y) : y = x + 4 and x < 4}
1 0 1 1 0 0
1 1 Þ R = {(1, 5), (2, 6), (3, 7)}
= 0 tan a 1 = 0 tan a 1
2 2 Þ R is not reflexive, not symmetric, but transitive and
x1 y1 1 x1 y1 1 - x1
anti-symmetric by default.
EBD_7801
6 Target IIT
15. (c) Since, R : Z ® Z; R = {(x, y) : x – y Î Z}
mv 2B
Qx–x=0ÎZ Now, at B : TB - mg cos q =
L
Þ R is reflexive and x – y Î Z
Put VB Þ TB = 3mg cos q
Þ y – x Î Z Þ R is symmetric.
When total acceleration vector directed horizontally
Also x – y, y – z Î Z Þ (x – y) + (y – z) Î Z
Þ (x – z) Î Z Þ R is transitive. at g sin q 1
tan(90° - q) = = = tan q
Q (2, 4), (4, 2) Î Z, but 2 ¹ 4 a r 2g cos q 2

Þ R is not anti-symmetric.
1
On solving, q = cos -1
16. (c) If D = b2 – 4ac = 0, a < 0, 3
then ax2 + bx + c < 0 " x Î R
21. (a, b, c, d)
Þ f (x) is not defined at any real x.
As the block loses contact with the plank at extreme
Þ Domain = f position
17. (a) If D = b2 – 4ac > 0, a < 0 a=g
N

then ax2 + bx + c > 0 " x Î (a, b) A a = g/2


Mean A/2
Þ Domain = (a, b) position
a=0
A/2
18. (d) If D = b2 – 4ac > 0, a > 0 A a = g/2
mg
a=g
then ax2 + bx + c > 0 " x Î (– ¥, a) È (b, ¥)
Þ Domain = (– ¥, a) È (b, ¥) i.e. normal force becomes zero, it has to be the upper
extreme, where acceleration of the block will be g
19. (a, c)Path difference = D 2 + d 2 - D = 1 cm. downwards.

10
2 2 l 2(1) \ w2A = g ; w2 = = 25
Also, [ D + d - D] = (2n - 1) Þ l = 0.4
2 2n - 1

For n = 1, 2, 3.................... 2p 2p
or w = 5 rad/s \ T = = s
w 5
2 2
l = 2cm, cm, cm,................
3 5 Acceleration in SHM is given by, a = w 2 x

20. (a, b, c) From the figure we can see that, at lower extreme,
acceleration is g upwards
1
Between A and B, mgL cos q = mv2B \ N – mg = ma or N = m (a + g) = 2mg
2
At halfway up, acceleration is g/2 downwards
\ v 2B = 2gL cos q
A 1
\ mg – N = ma or N = m(g - g / 2) = mg = 0.5 mg
2
v 2B ar
Now, a r = = 2g cos q
At halfway down, acceleration is g/2 upwards
L L

and a t = g sin q B 3
\ N – mg = ma or N = m(g + g / 2) = mg
2
C
at At mean position, velocity is maximum and acceleration
\ a = a 2t + a r2 = g 1 + 3cos 2 q is zero. \ N = mg
SOLUTIONS - MOCK TESTS 7
22. (a, d)Consider a ring of radius x and the thickness dx. p
Hence, at t = , velocity and acceleration of the
Equivalent current in the ring 2p
dx particle are normal to each other.
w r
= ´ charge on the ring x
2p At t = t, position of the particle r (t ) = xi$ + y $j

= a cos pt $i + b sin pt ˆj and acceleration of the particle


w Q
= ´ (2pxdx) is
2p pR 2
r
a(t ) = ax $i + a y $j = - p 2 [a cos pt $i + b sin pt $j ]
m 0 æ w 2xQ ö
dx ÷
2x çè 2p R 2
dB (due to this ring) =
ø r
= – p2 [ xi$ + y $j ] = - p 2 r (t )
R
m 0w Q m wQ m wQ Therefore, acceleration of the particle is always directed
\ B= ò 2p R 2
dx = 0 2 .R = 0
2pR 2pR
(Outward)
towards origin.
0
p
At t = 0, particle is at (a, 0) and at t = , particle is at
x 2p
23. (a, b, c) x = a cos pt Þ cos (pt) = ... (1)
a (0, b). Therefore, the distance covered is one fourth of
y the elliptical path and not a.
y = b sin pt Þ sin (pt) = ... (2)
b 24. (c, d) I = cos 500 t
Squaring and adding (1) and (2), we get,
I
x2 y2
+ =1
a2 b2 1
7p
\ The path of the particle
6w
p t
is an ellipse. p 2p
w w
2w
From the given equations
we can find, –1

dx d2x
= v x = - ap sin pt ; = ax = - ap 2 cos pt B D
dt dt 2
A
dy 50 V
= v y = pb cos pt
dt C = 20 m F

d2y
and = a y = -bp 2 sin pt R = 10 W
2
dt 7p p
Till t = , the charge will be maximum at
6w 2w
p p
At time t = or pt = p / 2w p / 2w
2p 2 é sin 500t ù
Q¢ = ò cos 500t dt = ê
ë 500 û 0
ú
p 0
ax and v y become zero (because cos = 0 ). Only
2
1 æ p ö 1
vx and ay are left, = sin ç 500 ´ ÷= C
500 è 2 ´ 500 ø 500
or we can say that velocity is along negative x-axis and
acceleration along negative y-axis. \ (a) is incorrect
EBD_7801
8 Target IIT
28. 7
7p
From the graph it is clear that just before t = , the
6w From the law of length of stretched string, we have
current is in anticlockwise direction. n1l1 = n2l2 = n3l3

\ (b) is incorrect Here n1 : n2 : n3 = 1 :3 : 15

7p l1 n2 3 l1 n3 15
At t = , the charge on the upper plate of capacitor \ l = n = 1 and l = n = 1
6w 2 1 3 1

is
l1 l1
l2 = and l 3 =
7p 3 15
6w
1 æ 7p ö
ò cos 500t dt = 500 sin çè 500 ´ 6 ´ 500 ÷ø The total length of the wire is 105 cm
0
Therefore, l1 + l2 + l3 = 105

1 1 l1 l1 21l1
=- ´ = -10 -3 C or l1 + + = 105 or = 105
500 2 3 15 15

Now applying KVL (when A is just connected to D) 105 ´ 15


l1 = = 75 cm
15
10-3
50 + - i ´10 = 0 Þ i = 10 A
20 ´10-6 \ l2 =
l1
=
75
= 25 cm ; l3 =
l1
=
75
= 5 cm
3 3 15 15
\ (c) is the correct option.
Hence the bridge should be placed at 75 cm and (75 + 25)
The maximum charge on C is Q = CV = 20 × 10–6 × 50 = 100 cm from one end.
=10–3C \ Required sum = (75 + 100) cm = 175 cm = (182 – 7) cm
Therefore, the total charge flown = 2 × 10 –3 C \ x = 7.
\ (d) is the correct option. 29. 7

25. (a,d) are correct options. Here l1 = 50cm, a1 = 2.10 ´ 10-5 / K


26. 4
-5
Dt = (250 - 40) = 210°C , a 2 = 1.2 ´ 10 / K
Force on 10 kg mass = 10 × 12 = 120 N
Change in length
The mass of 10 kg will pull the mass of 20 kg in the backward
direction with a force of 120 N. DL1 = L1a1Dt = 50 ´ 2.10 ´ 10 -5 ´ 210 = 0.22 cm.
\ Net force on mass 20 kg = 200 – 120 = 80 N
Similarly, DL2 = L2 a 2 Dt = 50 ´ 1.2 ´ 10-5 ´ 210 = 0.12 cm.
80 N
Its acceleration = = 4m/s2 \ Total change in length
20 kg
DL = DL1 + DL2 = 0.22 + 0.12 = 0.34 cm.
27. 5
= 34 × 10–2 cm
F = 20 – 10x = – 10 (x – 2)
Þ A = 34. Hence sum of digits of A = 3 + 4 = 7.
Hence, force constant is k = 10 ; m = 0.1 kg
30. 4
k The magnetic field (due to current in wire P) at R
Angular frequency, w = = 10 rad/s
m
µ0 2I p µ0 2 ´ 2.5
vmax = w A = 50 m/s = (5×10) m/s = ´ = ´
4p rPR 4p 5
SOLUTIONS - MOCK TESTS 9

µ0 33 mv
= [in the plane of paper downwards] = .
4p 7 qB
2.5Amp 1Amp 5
e v = 4 × 10 m/s 33. (d) For the combination,
P Q R +X (a) III (i) R
2m
uur mv mv
rPR = 5m ST = 2 ´ r = 2 ´ =
2qB qB
Similarly the magnetic field (due to current in wire Q) at R
(b) IV (ii) Q
µ 2 ´ I µ0
= 0´ = I [in the plane of paper downward] ) 1 1 22 mv
4p 2 4p Arc ST = ´ 2pr = ´ 2 ´ ´
2 2 7 2qB
The total magnetic field at R [due to P and Q]
11 mv
µ0 µ0 µ =
B= + I = 0 (1 + I ) [in the plane of paper downwards] 7 qB
4p 4p 4p
(c) III (i) Q
The force experienced by the electron
Same as explained above.
F = qvB sin q
34. (c) during the upward journey of projectile velocity of the
body decreases and finally becomes zero at the top
µ0
= evB sin 90° = 1.6 × 10–19× 4 × 105 × (1 + I) but then after the direction of velocity changes and
4p goes on increasing while acceleration remains constant
But F = 3.2 × 10–20 N (Given) throughout the journey a = – g.

\ 3.2 × 10–2 = 1.6 × 10–19 × 4 × 105 × 10–7 (1 + I) 35. (d) When velocity-time graph is straight line the
corresponding acceleration-time graph will be
Þ I = 4 amp. straight line parallel to time-axis i.e. constant
uur acceleration.
31. (c) ST = 2OS cos ÐOST
36. (a) For case IV displacement time graph shows that body
×T × ×
mvcos 45° × × × is in rest.
= 2´ O × × ×
qB × × × 37. (a,b,d)
×S × ×
v Recall that the —Cl group present in the o- and p- positions
45°
mv 1 to the electron-withdrawing group is activated toward
= 2´ ´
qB 2 nucleophilic substitution, hence only —Cl present on the
o- and/or p-position to the —NO2 group will be replaced.
mv 38. (a,b,c,d)
= 2´
qB
(a) Screening effect ­, effective nuclear charge ¯ thus
32. (b) ÐOST = ÐOTS = 45° valence shell electron is loosely bound. Hence I.E ¯
(b) Be and Mg has ns2 configuration (stable configuration)
So, ÐSOT = 90°
) (c) Due to lanthanide contraction
Arc length ST • • • • •
T• • • • • (d) rmetallic > rcovalent (covalent bond formation involves
• • • • •
• • •O • • the overlapping of orbitals)
3 45°• • • • •
= ´ 2p OS • • • • •
4 39. (a,b)
S• • • • •
• • • • •
Let a and b be the atomic masses of A and B respectively.
3 22 mv Then
= ´2´ ´
4 7 qB
Molar mass of AB2 = (a + 2b) g/mol
EBD_7801
10 Target IIT
Molar mass of AB4 = (a + 4b) g/mol
From the given data,
41. (a, c)
Molality of AB2 in solution O O
1g / (a + 2b)g mol -1
1´ 50 or +
= = (a + 2b) mol kg–1
-1 H H
(20g / 1000g kg )
and molality of AB4 in solution
O
-1
1g / (a + 4b)g mol 1´ 50
= -1 = mol kg–1
(20g / 1000g kg ) (a + 4b) + OHC
OHC
50 O
Then, DTf (AB2) = Kf ×
(a + 2b)
mol kg–1

50 42. (c, d)
2.3 K = 5.1 K kg mol–1 × (a + 2b) mol kg–1 ........(1)
[Fe(CN)6]4–

50 3d 4s 4p
andDTf(AB4) = Kf × mol kg–1 d2sp3
(a + 4b)

50
No unpaired electron(s)
1.3 K = 5.1 K kg mol–1 × (a + 4b) mol kg–1 [Fe(CN)6]3–

From the above equations, one can write, 3d 4s 4p


d2sp3
2.3 (a + 4b)
= (a 2b)
1.3 + One unpaired electron
This gives, a = 0.6 b, [Fe(H2O)6]3+
Put in eq. (1), b = 42.64
Hence a = 0.6 × 42.64 = 25.58
sp3d2 hybridization
Therefore, Atomic mass of A = 25.58 u
Atomic mass of B = 42.64 u [FeF6]4–
40. (a,b,c)
(a) PO34- + 3HNO3 ¾¾
D
® H3 PO4 + 3NO3-
sp3d2 hybridization
43. (b, d)
D
H 3 PO 4 + 12(NH 4 ) 2 MoO 4 + 21HNO 3 ¾¾¾¾¾®
Not exceeding 40° C NOTE : Only primary amines give carbylamine test.
(NH 4 )3 PO 4 .12MoO 3 ¯ +21NH 4 NO 3 + 12H 2 O Hence 2,4-dimethylaniline and p-methylbenzylamine both
give this test.
or (NH 4 )3 [P(Mo3 O10 ) 4 ] ¯
Amm. Phospho molybdate
(Canary yellow ppt.

NO -2 + H + ¾¾¾
R.T.
(b) ® HNO 2
(dil.)

R.T.
3HNO 2 ¾¾¾
® 2NO ­ + HNO3 + H 2 O

® [Fe(H 2 O)5 (NO)]2 + + SO 42 -


R.T.
FeSO4 (aq) + NO ­ ¾¾¾
44. Ans : 8
Brown ring complex
Dipole moment, µ = e × d coulombs metre
(c) 3HgCl2 + SO 42- + 2H 2 O ¾¾
® HgSO4 .2HgO ¯ +4H + + 6Cl -
Yellow ppt. For KCl, d = 2.6 × 10–10 m
SOLUTIONS - MOCK TESTS 11
For complete separation of unit charge (electronic charge)

(e) = 1.602 × 10–19 C Cl
Hence µ = 1.602 × 10–19 × 2.6 × 10–10 = 4.1652 × 10–29 Cm
µKCl = 3.332 × 10–29 Coulomb meter (given) y
x

3.332 ´ 10 -29 + –
\ % Ionic character of KCl = × 100 Li x
Cl
4.1652 ´ 10 -29
= 80% Cl– ions occupy the corners of the face of cube as well as
Given % ionic character of KCl = 10x the centre of the face.
\ 80 = 10x Þ x = 8. y = distance between the centres of two Cl– ions.
45. Ans : 7
Using gas equation; PV = nRT Hence, y = x 2 + x 2 = x 2 = 2.57 × 1.414 = 3.634 Å
Total no. of moles of gases in the mixture (n)
= 363 pm
PV 6 ´3
= = = 0.7308 mol. Given, 363 = 121x.
RT 0.0821 ´ 300
Thus no. of moles of unknown gas = 0.7308 – 0.7 363
\x = = 3.
= 0.0308 mol. 121
Now we know that
48. Ans : 5
r1 moles of hydrogen gas 0.7
r2
=
moles of unknown gas
=
0.0308 Given : Ù¥ + -3 ¥ - -3
m (Ag ) = 6 ´ 10 ; Ù m (Br ) = 8 ´ 10 ;

r1 M2 Ù¥ -
m (NO3 ) = 7 ´10
-3
and K sp (AgBr) = 12 ´10 -14
Also we know that =
r2 M1
To find the specific conductivity (k) of the final solution of
AgBr in which AgNO3 (10–7 M) is mixed we must find the
2 2
æ r1 ö æ 0.7 ö individual k of the ions.
\ M2 = çç ÷÷ M1 or M2 = ç ÷ × 2 = 1033
è r2 ø è 0.0308 ø
or k so ln = k +k +k
Ag + Br - NO3-
Given 1033 = 1040 – A.
\ A = 7.
Again, k = Ù ¥
m ´ molar concentration
46. Ans : 1
Two molecules of an a-amino acid will form only one Calculation of molar concentration of ions :
dipeptide, recall that four different dipeptides are formed
Concentration,
when two a-amino acids are different.
47. Ans : 3
[ NO3- ] = 10 -7 moles / l º 10 -4 moles / m 3
Cl– ions in LiCl similar to rock salt (NaCl) constitute ccp
which has face centred cubic lattice. Li+ ions fit exactly into Let x be the molar concentration of Ag+ from AgBr
the octahedral voids.
Li+ Þ (x + 10-7 )x = 12 ´ 10-14
Cl– Cl

or x 2 + 10 -7 x - 12 ´ 10-14 = 0

or, ( x + 4 ´ 10 -7 )(x - 3 ´10 -7 ) = 0 Þ x = 3 ´10 -7 M

Edge length of unit cell = 5.14 Å


Þ [Br - ] = 3 ´ 10-7 M º 3 ´10 -4 moles / m 3 and
5.14
Distance between Li+ and Cl– ions (x) = = 2.57Å
2 [Ag + ] = 3 ´ 10 -7 + 10-7 = 4 ´10 -7 M
EBD_7801
12 Target IIT

= 4 ´10-4 moles / m3
Na

k = 6×10 –3×4×10–4 Na
Ag + minor product trans isomer, more stable,
major product
= 24 ´10-7 (Sm 2 mol -1 ´ mol / m3 ) = 24 ´10 -7 S / m

– NaNH2
Similarly, k = 8 ´ 10 -3 ´ 3 ´ 10-4 =24×10 –7 S/m and H — NH2
Na
Br -
Na H
k = 7 ´ 10 -3 ´10 -4 = 7 ´10 -7 S / m
NO3-

Na
H
H – NH2
Þ k = (24 + 24 + 7) ´10 -7 S / m = 55 ´ 10 -7 S / m + NaNH2
H
H
Given 55 × 10–7 = 11x × 10–7 \ x = 5.

Passage 6
Me3CO
51. (a)
49. (d) Anti elimination
Cl Hofmann's product

Me
H H
N — Me Aq. KMnO4
(b) syn hydroxylation
s syn-elimination H OH H
O OH

The option (a) is not matching, thus answer is (a)


Cl Cl
Less substituted product Passage - 2
(Hofmann’s product)

n2 h 2 n2
52. (c) radius (rn) = < ´ 0.529Å
50. (b) Birch reduction (anti-addition) 4ο 2 m Ze 2 Z

CH3 n2
Li - Liq NH3
H3C — C º C — CH3 < < 0.0529 nm
EtOH Z
CH3
(Anti-addition)

2ο Ze2 æç Ze 2 ö÷÷
12
53. (b) Vn < < çç ÷
nh çè rm ø÷÷
s Nar
H3C — C º C — CH3 Na CH3 — C = C — CH3

2ο 2 m Z2 e 4 Z2
or 54. (d) E n < , <, ´ 313.6 kcal
n2 h 2 n2
SOLUTIONS - MOCK TESTS 13
ANSWER KEY PAPER - 2
1 (a) 14 (b,c) 27 (a,b,c) 40 (c) 53 (b)
2 (c) 15 (d) 28 (a,b) 41 (a) 54 (a)
3 (c) 16 (a) 29 (b,d) 42 (d)
4 (b) 17 (c) 30 (a,c) 43 (b)
5 (c) 18 (a) 31 (a,b) 44 (a,b)
6 (d) 19 (a) 32 (c) 45 (a,c,d)
7 (b) 20 (a) 33 (d) 46 (a,b,c)
8 (a, b) 21 (c) 34 (b) 47 (a,c,d)
9 (b, c, d) 22 (a) 35 (d) 48 (a,c)
10 (a, b, c, d) 23 (d) 36 (c) 49 (b, c)
11 (a, b, c, d) 24 (c) 37 (c) 50 (a, c)
12 (a, b, c, d) 25 (b) 38 (a) 51 (d)
13 (b, d) 26 (a,d) 39 (b) 52 (a)

PAPER - 2 ln x 2007 - ln (1 + x 2007 ) 1 æ x 2007 ö


= ln ç +C
1. (a) Obviously roots are 1 + i, 1 – i, 1+ 2i & 1 – 2i and r (say), = 2007 2007 è 1 + x 2007 ÷ø
where r is real
Now sum of the roots = 6 (from the given equation) Comparing from the given, we get
\ 4+r= 6Þr=2 p = 2007, q = 2007, r = 2007
Product of the roots = (1 – i 2) (1 – 4i2) · 2 \ p + q + r = 6021
\ – D = 2 · 5 · 2 Þ D = – 20
æ 2 tan -1 x ö
1 lim x ç -1÷
2. (c) Case 1 : If 0 £ x £ , then x ®¥ çè p ÷ø
2 4. (b) Given e = eL

æ 1 3ö æ 2 tan -1 x - p ö
-1 æ
x 1 ö where L = lim x ç
cos ç + 3 - 3x 2 ÷ = cos-1 ç x. + 1 - x 2 . ÷ x ®¥ è p ÷
è2 2 ø è 2 2 ø ø

1 2 [(p / 2) - cot -1 x] - p
= cos -1 x - cos -1 L = lim x
2 x ®¥ p
1 p 2.x.cot -1 x 2 tan -1 (1 / x)
\ Equation is: cos -1 x + cos -1 x - cos -1 = 2
2 3 = lim - = lim - =-
x ®¥ p x ®¥ p (1 / x) p
1
Þx=
2
5. (c) C2
1 C1 r1 Q
Case 2 : If £ x £ 1, then
2

æx 1 ö 1
cos-1 ç + 3 - 3x2 ÷ = cos-1 - cos-1 x A B
è2 2 ø 2 (10,0) r
d
(–15,0)
1
p P
\ Equation is cos -1 -1 1 -1
x + cos - cos x= ,
2 3
which is identity
Centres of C1 and C2 are (10, 0) and (–15, 0) respectively.
é1 ù
Hence the identity holds good for x Î ê , 1ú .
2 ë û r1 = 6; r2 = 9, d = 25, where d is the distance between
the centres.
dx x 2007 + 1 - x 2007
3. (c) I=ò x (x 2007 + 1)
=ò x (x 2007 + 1) dx Now, r1 + r2 < d Þ Circles are separated.

æ1 x 2006 ö PQ = l = d 2 - (r1 + r2 ) 2 = 625 - 225 = 20


= òç x - 2007 ÷
dx = ln x - 1 ln (1 + x 2007 )
è 1 + x ø 2007
EBD_7801
14 Target IIT
6. (d) x = a – [x]3 Þ xÎI (b) If gof is one-one then f is also one-one,
\ a = x3 + x If f is many-one then gof can not be one-one

7 7 2
æ 7 ´ 8ö 7 ´8
å a = å r3 + å r = çè 2
÷ø +
2
= 784 + 28 = 812
f g
r =1 r =1 B
A C
7. (b) y = ln2x – 1 a a 4
y
b b 5
2 ln x
y¢ = =0
x
gof
Þ x=1
1 x
x > 1, y ­ O 1/e e (c) and (d) are obviously true.
and 0 < x < 1, y is ¯ –1
10. (a, b, c, d)

e 7 + 8 + 7 = 22 ¹ 20 Þ coefficient of x7 y8 z7 is zero.
ò (ln
2
A= x - 1) dx 22C = 231
Total number of terms is 2
1/e

20!
e General term = xp yq zk,
e æ ln x ö æ 1ö p!q!k!
= [ x ln 2 x ùû -2ò ç ÷ .x dx - ç e - ÷
1/e
1/e è
x ø è eø
where p + q + k = 20
If we choose p = 20 – r and q = r – k
e
æ 1ö æ ln x ö æ 1ö
= çè e - e ÷ø - 2 ò çè x ÷ø .x dx - çè e - e ÷ø
then we obtain the required form.
1/ e
For sum of coefficients, put x = y = z = 1
Þ sum of coefficients = 320.
é e
e ù
= - 2 ê[ x In x ûù1/e - ò dx ú 11. (a, b, c, d)
êë 1/e úû
Any point on the parabola is (at2, 2at)

éæ 1ö æ 1ö ù -4 4 æ a + at 2 ö
= -2 ê ç e + ÷ - ç e - ÷ ú = = \ mid point of (a, 0) and (at2, 2at) is ç 2 ,at ÷
ë è e ø è eø û e e è ø

8. (a, b)We have b3 > 4b2 – 3b1 Þ b1 r2 > 4b1 r – 3b1


a + at 2
Þ r2 > 4r – 3 [Q b1 > 0] \ locus is given by x = , y = at
2
Þ r2 – 4r + 3 > 0 Þ (r – 3)(r – 1) > 0 Þ r > 3 or r < 1
9. (b, c, d) æ y2 ö
i.e., 2x = a ç1 + 2 ÷
As shown gof is one-one but g is many-one è a ø

f g i.e., 2ax = a2 + y2 i.e., y2 = 2a (x – a/2)


A B C
It is a parabola with vertex at (a/2, 0), latus rectum = 2a
3
a 4 directrix is x – a/2 = – a/2 i.e., x = 0
1
b 5 At focus, x – a/2 = a/2 i.e., x = a
2
Hence, focus = (a, 0).
Þ (a) is not correct.
SOLUTIONS - MOCK TESTS 15
12. (a, b, c, d)
(b) 4a – k + 8 = 4 × (–1) – 4 + 8 = 0
Given hyperbola can be written as
(c) Now det (P adj Q) = |P| adj Q|
(x - 1)2 (y - 1)2 = |P| |Q|2 = 8 × 82 = 29
- =1
16 9 (d) |Q adj P| = |Q| |P|2 = 29
For Qs. 15-16.
X2 Y2
Þ - = 1 {where X = x – 1, Y = y – 1} P(E) = p
16 9
P (F) = P (E Ç F) + P (E Ç F)
b2 9 5
e = 1 + 2 = 1+ =
a 16 4 P (F) = P (E) P (F/E) + P (E) P(F / E)

a 1 4p 1
Directries are X = ± = p.1 + (1 – p). = +
e 5 5 5

16 21 11 E
Þ x -1 = ± Þ x= and x = - F
5 5 5
E
2b2 9
Length of latus rectum = =
a 2 15. (d) If p = 0.75
At Focii, X = ± ae, Y = 0
1 1
P (F) = (4p + 1) = (4) = 0.8
\ focii are (6, 1) and (– 4, 1). 5 5
13. (b, d)
P (E Ç F) 0.75 15
\ P (E/F) = = =
1 1 x é1 x ù P(F) 0.80 16
For 1 £ x £ 33, < + <1 Þ ê + ú = 0
3 3 50 ë 3 50 û
5p
16. (a) Now P(E / F) = ³p
1 x 4 é1 x ù (4p + 1)
For 34 £ x £ 50, 1 < + < Þ ê + ú =1
3 50 3 ë 3 50 û
Equality holds for p = 0 or p = 1
\ E = 17 For all others value of p Î (0, 1), L.H.S. > R.H.S.

P.Q Q For Qs. 17-18


14. (b,c) PQ = kI Þ = I Þ P -1 =
k k
n! (n - 1)!
3.nP4 = n–1P5 Þ 3. =
Also |P| = 12a + 20 (n - 4)! (n - 6)!
Comparing the third elements of 2nd low on both sides,
Þ 3n = (n – 4) (n – 5) Þ 3n = n2 – 9n + 20
we get
Þ n2 – 12n + 20 = 0
æ 3a + 4 ö 1 - k Þ (n – 10) (n – 2) = 0 Þ n = 10 as n ¹ 2
-ç ÷= ´
è 12a + 20 ø k 8
17. (c) From 10 stations train to be stopped at 3.
Þ 24a + 32 = 12a + 20 Þ a = –1 \ 7 remains. In between these 7 stations i.e. there are 8
\ |P| = 8 gaps.

Also PQ = kI Þ |P| |Q| = k3 \ out of 8 gaps select any 3 in 8C3 i.e. 56 ways.
18. (a) Any 3 consecutive vertices can be selected in 10 ways
k2 k2
Þ 8´ = k3 Þ k = 4 Þ |Q| = =8 (1, 2, 3) or (2, 3, 4) or .............. or (10, 1, 2)
2 2
EBD_7801
16 Target IIT
say e.g. (1, 2, 3)
\ N 0¢ = (2)5/15 ´ N1 = (2)1/3 ´ 3.844 ´ 105
Now 4 and 10 cannot be selected from the remaining
five vertices from 5 to 9, any one vertex can be selected = 1.269 ´ 3.844 ´ 105 = 4.878 ´ 105
in 5C1 ways.
\ number of such quadrilaterals = 10 . 5C1 = 50 [Q (2)1/3 = 1.269 ]

3mg 3 Volume of blood


19. (a) Acceleration of block AB = = g
3m + m 4
N0 2.883 ´ 109
V= = = 0.5910 ´ 104 cm3
2mg 2g N 0¢ 4.878 ´ 105
Acceleration of block CD = =
2m + m 3
= 5.91 litres.
Acceleration of image in mirror AB = 2 acceleration of

æ -3g ö -3 15r
mirror = 2 ç = g 21. (c) Resistance of 560cm. wire = ´ 560 = 14r
è 4 ÷ø 2 600

Let potential at A is zero. Then apply Kirchoff’s law,


æ 2g ö 4g
Acceleration of image in mirror CD = 2 ç ÷ = E
è 3ø 3
x - 0 x - 2 - 0 (x - E - 0) 14E
+ + =0Þ x =
\ Acceleration of the two image w.r.t. each other 14r r 2r 22

4g æ -3g ö 17g E æ 14E ö - E


= -ç ÷= x- ç ÷
3 è 2 ø 6 2 = è 22 ø 2 = 3E
Þ I0 =
r r 22r
0.693
20. (a) We know that T1/2 =
l 22. (a) Each charge creates at point D a field intensity of
0.693 0.693 KQ
or l= = = 1.283 × 10–5 sec. E1 =
T1/2 15 ´ 3600 . The total intensity will be the sum of three
a2
dN vectors (fig.). The sum of the horizontal components
Now activity A = = lN 0 where A = 1 microcurie of these vectors will be zero, since they are equal in
dt
magnitude and form angles of 120° with each other.
= 3.7 × 104 disintegrations/sec The vectors from angles of 90° – a with the vertical,
where a is the angle between the edge of the tetrahedron
\ 3.7 ´ 104 = 1.283 ´ 10 -5 N0 and the altitude h of triangle ABC.

4
Þ N 0 = 3.7 ´ 10 = 2.883 ´ 109
1.283 ´ 10 -5

Let the number of radioactive nuclei present after 5


hours be N1 in 1cc sample of blood. Then

dN 296 0.693
= lN1 or = N1
dt 60 15 ´ 3600

296 ´ 15 ´ 3600
or N1 = = 3.844 ´ 105
60 ´ 0.693
Let N0' be the number of radioactive nuclei per cc of
sample, then

t /T The vertical components are identical each being equal


N1 æ1ö
=ç ÷ or N 0¢ = (2) t/T ´ N1
N 0¢ è 2 ø
SOLUTIONS - MOCK TESTS 17
KQ l l
to sin a . It follows from triangle ADE that PB = PA + ra = Patm + rgh + ra
a2 2 2
2 l 3
sin a = . PC = PB + (2r)a = Patm + rgh + ral
3 .... (1)
2 2
Therefore, the sought intensity of the field is But from left limb :
KQ PC = Patm + (2r)gh .... (2)
E= 6 .
a2 From (1) and (2)

æ Iö æ K / r2 ö 3 3a
23. (d) dB = 10 log ç ÷ = 10log ç Patm + rgh + ral = Patm + 2rgh Þ h = l
÷ 2 2g
è I0 ø è I0 ø
2mr 2r
= 10 [log (K ¢ ) - 2 log r] 26. (a, d) r2 = =
m + 2m 3
dB1 = 10 (log K' – 2 log r1), dB2 = 10 (log K' – 2 log r2) r2 r1
3 = dB1 – dB2 4p2 r23
T22 =
Gm m 2 1 2m
ær ö ær ö 1 r C.M.
= 20 log ç 2 ÷ Þ ç 1 ÷ = 32p 2 r3
è r1 ø è r2 ø 2 T22 =
27Gm
DQ DW Kaq T2 µ r3 / 2 ; T2 µ m -1/ 2
24. (c) = = work done per unit time =
Dt Dt L 27. (a, b, c) (a) Due to net electric field at P produced by
nRT capacitor charge will experience force
Power = F × velocity = PAV' = AV ¢
V 2Q Q 3Q
(b) E= + ÞE=
where V ® volume, V' ® velocity 2Ae 0 2Ae0 2Ae 0 2Q –Q

0.5 R(300) Kaq


Þ AV ¢ =
V L E
3 Q 3Q
0.5 R(300) Kaq E= Þ Ed = =V
Þ AV ¢ = 2 Cd 2C
L L
A.
2
2
1 1 æ 3Q ö 9 Q2
K(1/ 9)æ 27 ö K (c) Energy = e 0 E 2 Ad = e 0 ç ÷ Ad =
Þ V¢= ç ÷= 2 2 è 2Cd ø 8 C
R è 300 ø 100R
(d) Force on plate,
25. (b) For the given situation, liquid of density 2r should be
behind that of r. æ 2Q ö æ -Q ö Q2 Q2
F=ç ´ ç ÷ = - F =
è 2Ae 0 ø÷ è 1 ø Ae 0 ; Ae 0

dU Ke 2
a 28. (a, b) | F | = = 4 .............. (1)
dr r

Ke 2 mv 2
= .............. (2)
r4 r

nh
and mvr = .............. (3)
2p
C B A
2 By (2) and (3),
From right limb :
Ke 2 4p 2 m m
r= 2 2
= K1 .............. (4)
PA = Patm + rgh h n n2
EBD_7801
18 Target IIT
2 velocities
From (2) KE = 1 mv2 = 1 Ke
2 2 r3 (i) 3Rw iˆ (due to translational motion)

Ke 2 Ke 2 R
Total energy = KE + PE = - (ii) w making an angle of 30º with the vertical due
2r 3 3r 3 2
to rotation
Ke 2 Ke 2 Ke 2 n 6
= 3
= 3
=
6r æ K mö 6K13 m3 uur é ˆ Rw ù 3 Rw ˆ
6 ç 12 ÷ \ vP = ê 3 Rw i - iˆ ú + k
è n ø ë 4 û 4

Total energy µ n6
11 3
= Rw iˆ + Rw kˆ
Total energy µ m–3 4 4
29. (b, d) 32. (c) The image I1 of parallel rays formed by lens 1 will act as
virtual object.
P2 P 2 RT æ kM ö
=k Þ = k Þ PT = ç ........... (1) 2
r PM è R ÷ø

1
P2 P¢ 2 P
= Þ P¢ =
r r/ 2 2
I1
I
Hence from (1), T ¢ = T 2 .
PT = constant, hence P-T curve is a hyperbola. d
f1
30. (a, c)Path difference at point O is d sin a = 0.5 mm,
Corresponding phase difference, Applying lens formula for lens 2

1 1 1 f ( f - d)
2p 2p (0.5 ´ 10 -3 ) Þ - = Þ v2 = 2 1
Df = ´ Dx = = 2000 p = 2p ´ 1000 v2 f1 - d f 2 f 2 + f1 - d
l 5000 ´ 10 -10
\ The horizontal distance of the image I from O is
Þ O is a point corresponding to maxima.
The point at 1m below O corresponds to central maxima. f 2 ( f1 - d ) f f + d ( f1 - d )
x=d+ = 1 2
31. (a, b) For rolling motion, the velocity of the point of contact f 2 + f1 - d f1 + f 2 - d
with respect to the surface should be zero. For this
To find the y-coordinate, we use magnification formula for
r r lens 2
3Rw( -iˆ) + vo = 0 \ vo = 3Rw iˆ

f 2 ( f1 - d )
v f + f -d f2
R R m= 2 = 1 2 =
w w cos30º kˆ u2 f1 - d f1 + f 2 - d
2 2

R D ´ f2
w sin 30º ( -iˆ ) 3R w iˆ h2
2 Also m = Þ h2 =
P D f1 + f2 - d

\ The y-coordinate y = D – h2
30º
Df 2 D ( f1 - d )
= D- =
f1 + f 2 - d f1 + f 2 - d
A shown in the figure, the point P will have two
SOLUTIONS - MOCK TESTS 19
33. (d) Phase difference of 4p is for second order bright fringe. PV = nRT
i.e. fringe no. 4.
34. (b) DXC = l, DXA = l/2 æ 78n1 ö æ 2750 ö
Pç ÷ ç ÷ = n1 × 0.082 × 293
è 0.877 ø è 1000 ø
DXC – DXA = l/2 = 300nm.
35. (d) Normal human ear cannot hear beat frequency (25Hz) PºBenzene = 0.0982 atm. = 74.63 mm Hg
as it is greater than 15Hz. PºBenzene at 27ºC can be calculated as

v 350 350 æ P2 ö DH vap é T2 - T1 ù


36. (c) f= ; f1 - f 2 = - 7
log ç P ÷ = ê ú
l (32 /100) (32.2 /100)
è 1 ø 2.303 R ë T1T2 û
37. (c) The desired equation is
NH3(g) ® N(g) + 3H(g) æ P2 ö 394.57 ´ 78 é 7 ù
log ç ÷= ê ú
è 74.63 ø 2.303 ´ 8.314 ë 300 ´ 293 û
we can consider the reaction as
PºBenzene (at 27ºC) = 100.2 mm Hg
DH1
NH3(g) ¾¾¾ ® N(g) + 3H(g)
Xsolute ´ 1000
1 3 Molality of the solution = X
DH 2 solvent ´ 78
¾¾¾ ® 2 N2(g) + 2 H2(g)

\ DH1 + DH2 = + 46.024 kJ mol–1 æ 100.2 - 98.88 ö


ç ÷ ´1000
è 98.88 ø
3 1 = = 0.17
Now, DH2 = – (435.95) – (941.8) 0.98 ´ 78
2 2
DTf = Kf m = 5 × 0.17 = 0.85
or DH1 = 46.024 + 653.925 + 470.9
or DH1 = 1170.849 kJ mol–1 é RTf2 ù
We know, that, Kf = ê1000 DH ú M
or DH1 = 1.170849 kJ mol–1 ëê f ú
û
38. (a) The reaction is
4NH3(g) + 5O2(g) ® 4NO(g) + 6H2O(g) æ 8.314 ´ Tf2 ö
5 = çç 1000 ´ 10060 ÷÷ 78
4 × 17 g 5 × 32 g 4 × (14 + 16)g è ø

= 68 g = 160 g = 120 g Tf = 278.5 K


10.0 g 20.0 g ? \ Freezing point of solution = 278.5 – 0.85 = 277.65 K
Inspection of the given data shows that in the given 40. (c) Compound A is a monosubstituted Ph-derivative. It
reaction, oxygen is the limiting reagent. Therefore, has an OH in the side chain, as indicated by the reaction
maximum mass of nitric oxide formed of Na releasing H2. The possible structures are

120g ´ 20g OH H CH3


= = 15g
160g
Ph CH3 Ph
39. (b) Let the moles of benzene vapourizes at 20ºC = n 1 H CH3 Ph OH CH3
OH
Volume of n 1 mole of benzene when it vaporises (l) (1) (2) (3)

78n1
=
0.877 CH3

Ph OH Ph OH
æ 78n1 ö
Volume of n1 mole of benzene (g) = 2750 ç ÷
è 0.877 ø CH3
(4) (5)
EBD_7801
20 Target IIT
(i) 3o
Structure (5) is a alcohol and is discounted on the 44. (a,b)
basis of chromic acid test. The chemical equation involving the titration of C2O42–
(ii) Structure (4) is eliminated; it is not chiral. with MnO4– is
(iii) Structure (2) would give a precipitate with I2/OH-
because of the -CH(OH)CH3 group and is therefore ® 2Mn +2 + 10CO 2 + 8H 2 O
2MnO -4 + 5C2 O 42- + 16H + ¾¾
eliminated. Only structure (1) and (3) are possible at The chemical equation involving Cu2+ ions are
this point.
(iv) The compound (A) is structure (3) because on 2Cu 2 + + 4I- ¾¾
® Cu 2 I2 + I2
oxidation it forms the chiral aldehyde,
PhCH(CH3)CH=O. I2 + 2S2 O32 - ¾¾
® 2I - + S4 O62 -
0.059 The stoichiometry of the above reactions give
41. (a) E° = E° + log KSP
Br - / AgBr/ Ag Ag + / Ag 1
2mol MnO 24 - º 5 mol C 2 O42 -
AgBr = E ° – 0.7257
Ag + / Ag
2mol S2 O32 - º 1 mol of I 2 º 2 mol Cu 2 +
0.059 Therefore,
and E° = E° + log KSP
Cl- / AgCl/Ag Ag + / Ag 1 Amount of C2O42– in the solution
AgCl = E° – 0.59 0.02 mol 5
Ag + / Ag -3
= 1000 mL ´ 22.6 mL ´ 2 = 1.13 ´ 10 mol
Now cell reaction is
Ag + Br – ¾¾¾
® AgBr + 1e– and amount of Cu2+ in the solution
® Ag + Cl–
AgCl + e– ¾¾¾ 0.05 mol -3
= 1000 mL ´ 11.3 mL = 0.56 ´ 10 mol

e -
Br– + AgCl ¾¾® Cl– + AgBr 45. (a,c,d)
(a) BeO is essentially covalent. Therefore, it does not
0.059 [Br - ] dissolve in water. BeSO4 in soluble in water due to its
0 = (0.7257 – 0.59) + log
1 [Cl- ] very high hydration enthalpy.
(b) The carbon hydrides of the type CnH2n + 2 are electron
[Br - ] precise compounds. This compound will neither act as
Þ = 0.005 = 5 : 1000 = 1 : 200
[Cl - ] Lewis acid or Lewis base because all electrons are
involved in bond formation.
42. (d) [Pt(en)2Cl2] will exhibit geometrical as well as optical
(c) Due to its high bond enthalpy, dihydrogen is not
isomerism. Geometrical isomerism is found in
particularly reactive at room temperature. It shows
compounds having co-ordination number 4 (having reactivity only at higher temperatures particularly in
square planar structure) and co-ordination number 6. the presence of catalyst.
(d) The s-block elements are highly electropositive in
O CH3O nature. So, these are very reactive.
43. (b) O
46. (a,b,c)

Ph CH2 (a) [Co(H2O)6]2+ ¾¾¾


® t2g2, 2, 1 eg1, 1
\ CFSE = – 0.8 D0
CH2
Br [Co(H2O)6]3+ ¾¾¾
® t2g2, 1, 1 eg1, 1
\ CFSE = – 0.4 D0
Br
O (b) [Fe(NH3)6]2+ ¾¾¾
® t2g2, 2, 2 eg0, 0
O CH2
\ CFSE = – 2.4 D0
CH2
[Fe(NH3)6]3+ ¾¾¾
® t2g2, 2, 1 eg0, 0
Ph C – OCH3 Ph C – OCH3
\ CFSE = – 2.0 D0
O O
SOLUTIONS - MOCK TESTS 21
(c) For co-ordination no six, two empty d-orbitals are not 9.5
available for d2sp3 as Ni(II) has 3d8 4s0 configuration. so, pKa = = 4.75
2
(d) d-orbital involved is d not d .
x2 - y2 z2 51. (d) Number of hybrid orbitals, H in ClF3
47. (a,c,d)
1
CH = CH2 = (valence electrons of Cl +
2
+
(CH3)3CO K
3 × valence electron of F) – 3 × 3 F-atoms)

1
= (7 + 3 × 7) – 9 = 14 – 9 = 5
3 2

3 Number of bond pairs (n) = 3


3 Total number of lone pairs (m) = H – n = 5 – 3 = 2
3
CH3MgBr (1 eq) / ether Hence, VSEPR notation of ClF3 is AX3E2.

1
48. (a,c) 52. (a) CO2 : H = (4 + 2 × 6) – 3 × 2 = 2 ;
2

Dipole moment (DM) = H – n


Me Me Me
Na CH3 – Cl where n = no. of bond pairs
H OH H ONa H OMe
SN2
14 Me 14 Me 14 Me n = 2; d.m. = H – n = 2 – 2 = 0
Me Me
MeONa
1
Red P/I2 SO2 : H = (6 + 2 × 6) – 3 × 2 = 3 ; n = 2; d.m. = 3 – 2 = 1
I H H OMe 2
SN2 SN2
14 Me 14 Me

1
H2O : H = (2 × 1 + 6) – 3 × 0 = 4 ; n = 2; d.m. = 4 – 2 = 2
49. (b, c) 2

|Y| has no physical significance. However, |Y|2 is the Ea


probability of finding the electron in an orbital, dz2 orbital 53. (b) log Kb = log A –
2.303 RT
has dumb bell shape along z-axis and has a doughnut shaped
electron cloud at the centre.
57450
log Kb = 12 –
50. (a, c) 2.303 ´ 8.314 ´ 300

(pH)1 = pKa + log (y/x) log Kb = 2 ; Kb = Antilog 2 ; Kb = 102

(pH)2 = pKa + log (x/y) KF KF


Kc = K Þ 104 = 2 ; KF = 106
æ x2 ö b 10
æ xö æ yö
(pH)2 – (pH)1 = log ç ÷ – log çè ÷ø = 1 Þ log ç 2 ÷ = 1
è yø x èy ø 54. (a) DG = DH – TDS ; DG = 20 – T × 0.07

x2 x For non-spontaneous process DG > 0


so = 10 Þ = 3.162
y2 y hence 0 < 20 – T × 0.07

æ yö æ xö 20
(pH)2 + (pH)1 = 2pKa + log çè ÷ø + log ç ÷ = 2 pKa = 9.5 T< Þ T < 285.7 K
x è yø
0.07
Telegram @unacademyplusdiscounts

Join Us on Telegram for More Such Books

https://telegram.me/unacademyplusdiscounts

Join us from the above link or search ''unacademyplusdiscounts'' in Telegram


EBD_7801
22 Target IIT
SOLUTIONS - MOCK IITs
MOCK IIT - 2
ANSWER KEY - PAPER 1
1 (b, c, d) 13 (d) 25 (a, b, d) 37 (b, c, d) 49 (c)
2 (a, b) 14 (b) 26 5 38 (a, c) 50 (b)
3 (a, b, c, d) 15 (d) 27 2 39 (c, d) 51 (d)
4 (a, d) 16 (c) 28 4 40 (a, c, d) 52 (b)
5 (a, b, c) 17 (c) 29 4 41 (a, b, c) 53 (a)
6 (b, d) 18 (a) 30 5 42 (a, c) 54 (b)
7 (a, b, c, d) 19 (a, b, d) 31 (a) 43 (b, d)
8 (1) 20 (a, b, c) 32 (b) 44 2
9 (7) 21 (a, b, c) 33 (d) 45 7
10 (3) 22 (a, b, c, d) 34 (b) 46 6
11 (4) 23 (a, c, d) 35 (d) 47 2
12 (7) 24 (b, d) 36 (d) 48 3

EXPLANATORY NOTES

PAPER - 1 a1 æ (1 + r)n - 1ö
= [(1 + r) n
- 1] = a 1ç
1. (b, c, d) r è (1 + r) - 1 ÷ø

= a1 [1 + (1 + r) + (1 + r)² + .......... + (1 + r)n–1 ]


é ln2
4 ( ln 2) 2 ùú dx
I= ò ê - 3. (a, b, c, d)
2 ê ln x
ë ( ln 2)( ln x )2 úû
Assume a = b, we have nothing to prove.
(a) for a > b
4é 1 1 ù
= ( ln 2) ò ê - ú dx
2 êë ln x ( ln x ) 2 úû a-b a-b a-b
a- b= £ <
a+ b a a -b
t t
Put ln x = t Þ x = e Þ dx = e dt for x = 2, t = ln 2 Hence option (a) is correct
x = 4, t = ln 4 = 2 ln 2 (b) The given inequality is equivalent to

2ln 2 a 3 + b3 ³ ab(a + b)
2ln 2 é1 1 ù t é et ù
= ( ln 2) ò ê - 2 ú e dt = ( ln 2) ê ú For a < b or a > b
ln 2 ët t û êë t úû n 2
Þ a 2 + b 2 - ab ³ ab
é 4 2 ù
= ( ln 2) ê - = 0.
ë 2 ln 2 ln 2 úû Þ (a - b)2 ³ 0 for all a > 0, b > 0

Hence option (b) is also correct.


2. (a, b)
(c) The given inequality is equivalent to
a2 = a1r, a3 = a1r², .............., an = a1 rn–1.

a a 3 / 2 + b3 / 2 ³ ab( a + b)
Given expression = 1 [C1r + C2 r + ....... + C n r ]
2 n

r For a < b or a > b


SOLUTIONS - MOCK TESTS 23
Þ n2 (a – n) – n = 0 Þ an – n2 – 1 = 0
Þ (a1/ 2 )2 + (b1/ 2 )2 - a1/ 2 b1/ 2 ³ ab

n2 + 1 1
( ) Þa= =n+
2
Þ a- b ³0 n n

Hence option (c) is also correct. (b) lim f (x) = lim [2(1– h) + 1] = 3
h® 0
x ®1-
(d) Given expression is equivalent to

lim f (x) = hlim [(1+ h)2 + 2] = 3


2a 2 2b2 x ®1+
®0
+ £2
1 + a4 1 + b4
\ LHL = RHL, so lim f(x) = 3
x ®1
2 2
2a 2b
Þ - 1+ -1 £ 0
1 + a4 1 + b4 æ a bö
2x
(c) e 2 = lim ç 1 + + 2 ÷ [1¥ form]
x ®¥ è x x ø

(1 - a ) + (1 - b )
2 2
2 2
Þ ³0 é a b ù æ bö
4 4 lim 2x ê1+ + -1ú lim 2ç a + ÷
1+ a 1+ b e x®¥ ë
x x2 û
e x ®¥ è xø
= e 2a
= =

\ a = 1 and b Î R
( )
2
As 1 - a 2 ³ 0 for all a > 0

é –1 2 5 ù é –1 2 5 ù
(1 - a 2 )2 6. (b, d) Let A = ê 2 –4 a – 4ú ~ ê 0 0 a + 6úú
ê ú ê
So ³ 0 for all a > 0
1+ a4 ëê 1 –2 a + 1ûú ëê 0 0 a + 6 ûú

(R2 ® R2 + 2R1, R3 ® R3 + R1)


(1 - b 2 )2
Similarly ³ 0for all b > 0 Clearly rank of A is 1, if a = – 6
1 + b4

Hence option (d) is also correct. –1 2 5


4. (a, d) Also, for a = 1, | A |= 2 –4 –3 = 0
1 –2 2
x2 + y2 + 8x – 10y – 40 = 0
centre of the circle is (– 4, 5)
2 5
its radius = 9 and –4 –3 = –6 + 20 = 14 ¹ 0

Distance of the centre (– 4, 5) (–2,3) \ Rank of A is 2, if a = 1.


from the point (– 2, 3) is 4+4 =2 2 7. (a,b,c,d) 1 + x is never zero, so 1 + f ( x ) is never zero. It is 1
\ a = 2 2 + 9 and b = -2 2 + 9 for x = 0, so it is always positive.

Hence f "( x ) is always positive. f '(0) = 0, so


\ a + b = 18, a - b = 4 2 , a.b = 81 – 8 = 73
f '(0) > 0 for all x > 0 and hence f is strictly increasing.
5. (a, b, c)
So, in particular, 1 + f ( x) ³ 2 for all x.
(sin nx) [(a - n)nx - tan x]
(a) lim =0
x ®0 x2 (1 + x)
We have f "( x) £ .
2
æ n 3 x3 öé ìï x3 üïù
ç nx - + ...÷ ê n(a - n)x - íx + + ...ýú
è 3! ø êë ïî 3 ïþúû x x2 x x 2
Integrating , f '( x ) £ f '(0) + + = + .
Þ lim 2 =0 2 4 2 4
x ®0 x
EBD_7801
24 Target IIT
Finally : d < 0, c < 0 then
x 2 x3
Integrating , again, f ( x) £ f (0) + + . Hence |a|+|b|+|c|+|d|=a+b–c–d
4 12

1 1 4 5 æ 1ö 4 æ 1ö 5 4
f (1) £ 1 + + = . = p(1) - 4 p ç ÷ + p ç - ÷ £ + 4 + = 7
4 12 3 3 è 2ø 3 è 2ø 3 3
8. Ans : 1 \ |a|+|b|+|c|+|d| £ 7
n
æ xö æ xö 10. Ans : 3
f ( x) = lim 2 - (n +1) å 2 - r cosec ç ÷ cot ç ÷
n ®¥ r =1
è2 ø
r è 2r ø The sum of the three positive numbers a, b, g is equal
p p
Consider to i.e., a + b + g = ...(1)
2 2

æ xö æ æ x ö ö cot a, cot b, cot g form an A. P.


n cos ç r ÷ç 2cos2 ç (r +1) ÷ -1
n ÷
è2 ø è ø
å 2-r 2 æ x ö =å 2-r çç 2 æ x 2ö 2 æ x ö ÷÷ 2 cot b = cot a + cot g ... (2)
r =1 sin ç r ÷ r=1 ç 4sin ç (r+1) ÷ cos ç (r +1) ÷ ÷
è2 ø è2 ø è2 øø From (1),
è
1 cot b cot a cot g - cot b
cot ( a + g ) = Þ =1
n
æ cot b cot a + cot g
æ x ö æ x öö
å çè 2-( r +1) cosec2 çè 2n+1 ÷ø - 2-r cosec 2 çè 22 ÷ø ÷ø
r =1 i.e. cot b cot a cot g = cot b + 2 cot b, using (2)
Þcot a cot g = 3.
- ( n +1) 2æx ö æ xö
=2 cosec ç ÷ - 2 - ( n + 2) cosec 2 ç ÷ 11. Ans : 4
è2 ø
n +1 è 2ø
Solving y = x - x 2 with y = mx, we get x = 0 or 1 - m
2
æ æ x ö ö 1 æ xö
ç çè (n +1) ÷ø ÷ 1 cosec 2 ç ÷ 1- m é x 2 x3 mx 2 ù
1- m
Þ f ( x ) = lim ç
x ®¥ ç
2
æ x ö÷ x
÷ ×
2
- lim
n ®¥
2
(2 ( n +
è
2)
)
2ø 9
\± =
2 ò ( 2
x - x - xm dx = ê -
ëê 2
)
3
-
2 úû
ú
çè sin çè ( n +1) ÷ø ÷ø
0 0
2

(1 - m) 2 (1 - m )3 m (1 - m )
2

x f ( x) = 1
2 = - -
\ lim
x®0 2 3 2

9. Ans : 7 3
9 (1 - m )
\± =
Without loss of generality we can assume that a, b ³ 0 . 2 6
Now
Þ 1 - m = ±3 Þ m = -2 or 4
If c, d ³ 0 then p (1) = a + b + c + d £ 1
Hence positive value of m = 4.
Þ | a | + | b | + | c | + | d | £1 12. Ans : 7

If c ³ 0, d < 0 then | a | + | b | + | c | + | d | Consider the function ,

= a + b + c –d = (a + b + c + d) – 2d x2
f ( x) = on [1, ¥ )
= p (1) –2p (0) £ 1 + 2 = 3 x3 + 200

If c < 0, d ³ 0 then | a | + | b | + | c | + | d | = a + b – c + d

4 1 8 æ 1ö 8 æ 1 ö 4 1 8 8 f '( x ) =
(
x 400 - x 3 )
= p (1) - p (-1) - p ç ÷ + p ç - ÷ £ + + + = 7
(x )
3 2
3 3 3 è 2ø 3 è 2ø 3 3 3 3 + 200
SOLUTIONS - MOCK TESTS 25
line. So, shortest distance will be zero.
f ' ( x ) > 0 for 0 < x < 3 400
x-3 y-5 z-7
f ' ( x ) < 0 for x > 3 400 15. (d) Since, = = ... (1)
1 -2 1

f ( x ) increases in 0 < x < 3 400 and f ( x ) decreases in


x +1 y +1 z +1
and = = ... (2)
neighbourhood of x > 3 400 7 -6 -1

Now 7 < 3 400 < 8 , which follows that the largest term for Point A (3, 5, 7) is a point on line (1) and B (– 1, – 1, – 1) is a point
on line (2). Also direction ratios of line (1) are 1, – 2, 1 and
f ( x ) can be either f ( 7) or f ( 8) those of line (2) are 7, – 6, – 1.
We use the cartesian formula for the shortest distance.
49 8
f ( 7) = > f ( 8) =
543 89 S.D.

f ( 7) is the greatest. x2 - x1 y2 - y1 z2 - z1
l1 m1 n1
x+3 y-6 z
13. (d) Since, = = l2 m2 n2
-4 3 2 =
( m1n2 - m2 n1 ) + ( n1l2 - n2 l1 ) + (l1m2 - l2 m1 ) 2
2 2

x+2 y z-7
and = =
-4 1 1 Here, (x1, y1, z1) = (3, 5, 7)

Let l, m, n be the direction cosines of the line MN which is (x2, y2, z2) = (– 1, – 1, – 1)
perpendicular to each of the given lines. l1 : m1 : n1 = 1 : – 2 : 1
\ – 4l + 3m + 2n = 0 and – 4l + m + n = 0
l2 : m2 : n2 = 7 : – 6 : – 1
l m n
Solving, = =
3 - 2 - 8 + 4 - 4 + 12 \ (m1n2 - m2 n1 ) 2 + (n1l2 - n2 l1 ) 2 + (l1m2 - l2 m1 ) 2

l m n l 2 + m2 + n2 1 = (2 + 6) 2 + (7 + 1) 2 + ( - 6 + 14) 2 = 8 3
Þ = = = =
1 -4 8 1 + 16 + 64 9
x2 - x1 y2 - y1 z2 - z1
1 -4 8 Also, l1 m1 n1
Þ l = ,m= ,n=
9 9 9 l2 m2 n2

It is obvious that the points P (– 3, 6, 0) and Q (– 2, 0, 7) are


situated on the given lines. -1 - 3 -1- 5 -1 - 7
\ Length of shortest distance = Projection of PQ on the = 1 -2 1
common perpendicular MN 7 -6 -1

[(- 2) - (- 3)] + æç - ö÷ [0 - 6] + [7 - 0] = 9
1 4 8
=
9 è 9ø 9 - 4 -6 -8
= 1 -2 1 = 144
14. (b) Since, 3 y - 1 = 4 z - 1 = 2 x - 1 and 7 - 6 -1
3 6 1

144 18
1 1 1 Hence, the shortest distance = = = 6 3
x- y- z- 8 3 3
2 = 3= 4 are equations of same straight
1 2 3
EBD_7801
26 Target IIT
16. (c) Since, f (x) = sin x and g (x) = cos– 1 x Distance between two position = v – u = 66 – 30
Here, range of g (x) is [0, p] where sin x is non-monotonic but = 36 cm
continuous.
1 1 1 30 + 66
(c) = + =
é pù ép ù f 66 30 30 ´ 66
If we divide [0, p] into two subintervals ê0, ú and ê , pú .
ë 2û ë2 û
30 ´ 66 330
é pù Þ f= = = 20.625 cm
In the interval ê 0, ú . 96 16
ë 2û
(d) u = 30 cm
sin x increases and belongs to [0, 1] whereas in the interval
20. (a, b, c)
ép ù
ê 2 , p ú . sin x decr eases from 1 to 0 and so, (a) Total displacement is zero hence its average velocity is
ë û
zero.
sin x belongs to [0, 1].
(b) Total distance travelled is
Hence, the range of fog is [0, 1].
2s and total time taken is 2t. V1=0
17. (c) Since f (x) = cot– 1 x, g (x) = ln x
0² = u² – 2gs
Domain of f (x) is (– ¥, ¥) and its range is (0, p) whereas
domain of g (x) is (0, ¥) and its range is R u2 u2 s,t
\ s= Þ 2s =
Range of f (x) should be in domain of g (x) for gof (x) to be 2g g
defined. V1=u
Also, 0 = u - gt Þ t = u / g
Therefore, (0, p) should be in domain of g (x) and so (0, ¥)
Hence, domain of gof (x) is R. 2u u2 / g u
\ 2t = < speed > = =
18. (a) Here f (x) = 1 + x, g (x) = (ln x)2 g 2u / g 2
Since, domain of f (x) = R, (c) Displacement is zero
range of f (x) = R 21. (a, b, c)
Domain of g (x) = (0, ¥) : range of g (x) = [0, ¥)
As V = nl
Domain of fog (x) = {x Î (0, ¥) : g (x) Î R}
V 340
= {x Î (0, ¥) : (ln x)2 Î R} l= = = 1m
n 340
= (0, ¥)
25cm.

first resonance light


19. (a, b, d) 75cm.
l 1 120cm.
R1 = = m = 25 cm.
I2 4 4
Given, D = 96, = 4.84 45cm.
I1
3l 3
Let I 2 = 4.84 a & I1 = a ; O= I1 I2 = 2.2 a \ R2 = = m = 75 cm.
4 4
2.2 a 11
(a) Required ratio = = 5l 5
a 5 \ R3 = = m = 125 cm.
4 4
v 11
(b) = ........... (1) i.e., third resonance does not establish.
u 5
v + u = 96 ............ (2) Now H2O is poured, \ Minimum length of H2O column to
have the resonance = 120 – 75 = 45 cm.
5v 16 v
\ v+ = 96 Þ = 96 Þ v = 66 & u = 30 \ Distance between two successive nodes
11 11
SOLUTIONS - MOCK TESTS 27
Also
l 1
= = m = 50cm
2 2 DTC DTC
QC = = = 2 KZ ( DTC )
and maximum length of H2O column to create resonance RC 1/ 2 KZ
i.e., 120 – 25 = 95 cm.
DTB DTC 3KZ ( DTC )
22. (a, b, c, d) QB = = = [Q DTB = DTC]
RB 4 / 3KZ 4
When S is closed resistance parallel to S become ineffective
(no current, no power) hence equivalent resistance of the DTD DTC 5KZ ( DTc )
QD = = = [Q DTD = DTC ]
circuit decreases, ammeter and voltmeter reading increases. RD 4 / 5KZ 4
23. (a, c, d)
3 KZ ( DTC ) 5 KZ ( DTC )
QB + QD = +
It is given that heat Q flows only from left to right through 4 4
the blocks. Therefore heat flow through A and E slabs are
the same. 8 KZ ( DTC )
= = 2 KZ ( DTC ) = QC
4
\ [a] is correct option
\ (d) is the correct option.

A B 24. (b, d)
3K QB RB E
2K 6K When current I is switched on in both the solenoids in
C RC identical manner, eddy currents are setup in metallic rings A
4K QC and B in such a way that rings A and B are repelled.
QA QE

RA D RE
5K QD RD
QA = QB + QC + QD = QE
hA
rA r B hB
A B
Since heat flow through slabs A and E is same,

[b] is not correct.

l
We know that resistance to heat flow is R =
KA

Let the width of slabs be Z. Then

L 1 4L 4 Given hA > hB . This shows that eddy currents produced in


RA = = ,R = =
2 K (4L) Z 8KZ B 3K ( LZ ) 3KZ
A are greater than in B. This is possible when r A < rB (the
rate of change of flux is same in both the rings, therefore
4L 1 4L 4 induced emf is same).
RC = = ,R = =
4 K (2 LZ ) 2 KZ D 5K ( LZ ) 5 KZ
25. (a, b, d)
Let V be the volume of spheres. VdF g
L 1
RE = = A
6 K (4 LZ ) 24 KZ For equilibrium of A : VdA g

Now, DT = QR T + VdAg = Vdfg T


\ T = Vg (df – dA) ...(1)
As RE is least, DTE is also smallest ie since the resistance to T
VdF g
heat flow is least for slab E, the temperature difference across For T > 0, df > dA or dA < df
B
is smallest.
(a) is the correct option
VdB g
\ Option (c) is the correct answer. For equilibrium of B :
EBD_7801
28 Target IIT
T + Vdfg = VdBg 28. 4
\ T = Vg (dB – df) ...(2) This is a problem of L-C oscillations.
For T > 0, dB > df Here Q0 = maximum value of

(b) is the correct option Q = 200 µ C = 2 × 10–4 C

From (1) & (2) Vg (df – dA) = Vg (dB – df) 1 1


w= = = 10+4 s -1
\ LC -3 -6
df – dA = dB – df (2 ´ 10 H ) (5.0 ´ 10 F)
\ 2df = dA + dB
Let at t = 0, Q = Q0 then
(d) is the correct option. Q (t) = Q0 cos wt … (1)
26. 5
dQ
10 t = 2T Þ T = 5t I (t ) = = -Q0w sin wt … (2)
dt
Block A will lose contact when
F=10t L=2.0mH
T = mAg; 5t = mAg

m
Þ t1 = sec = 2sec
5

While block B will lose contact,

then A B C=5.0mF

T ' = mB g Þ 5t = 2mBg
dI (t )
= -Q0w2 cos (wt ) … (3)
2g dt
or t2 = sec = 4 sec
5
æ Q0 ö
At t1 £ t for block A For Q = 100 µC çè or ÷ø
2
T – mg = ma
From (1), 100 = 200 cos wt

v t2
1
mdv Þ m dv =
5t – mg = ò ò (5t – mg )dt or cos (wt) =
2
,
dt 0 t1
From eqution (3) :
v = 10 m/s = (2 × 5) m/s \ x=5
dI æ 1ö
27. 2 = (2.0 ´10-4 C ) (10+4 s -1)2 ç ÷
dt è 2ø
The force constant k of the spring is given by
dI
F 0.5 kg Wt 0.5 ´ 10 Newton = 104 A/s \ y=4
k= = = = 25 Newton/metre dt
y 0.20 m 0.20 m
29. 4

æ mö æ 0.25 ö æ 22 1 ö Here, r1 = 10 cm, r2 = 15cm


Now T = 2p ç ÷ = 2 ´ p ç ÷ = ç2´ ´ ÷ sec
è kø è 25 ø è 7 10 ø V1 = 150 V, V2 = 100 V
Common potential
11
= 2 ´ sec \ x=2
35 C1V1 + C 2V2 4 p Î0 ( r1 V1 + r2 V 2 )
V = = = 120 volt.
C1 + C 2 4 p Î0 ( r1 + r2 )
SOLUTIONS - MOCK TESTS 29

10 -1 Z Z2
q1 = C1V = 4 p Î0 r1 V = ´ 12 C Vn = V0 m/s , E n = E 0 2 J
n n
9 ´ 109
Z2
12 9 33. (d) Ionization energy, E n = E 0
= ´ 3 ´ 10 esu = 4esu n2
9 ´ 109
30. 5
N
Let v be the actual frequency of the whistle. By Doppler's r
r r M N
effect 34. (b) M
r
vS
v' = v
v S - vt
r 4r 4 ´ 3
r+ = = = 4W
where vs = speed of sound = 300 m/s (given) 3 3 3
v' = 2.2 k Hz = 220 Hz (given) Now if battery of emf 60 V and resistance 1 W is con-
nected
300
\ 2200 = v ... (i)
300 - vt 4W
M N

While the train is receding

vs 60 V 1W
v'' = v
v s + vt
60
I= = 12 A
Here, v' = 1.8 kHz = 1800 Hz (given) 4 +1

7W
300 W
\ 1800 = v ... (ii) 10
300 + vt 35. (d) 5W
3W W 5W 10 W
10
M N
Dividing (i) and (ii) 10 W
M N
10 W
2200 300 300 + vt
= ´
1800 300 - vt 300
5W
5W
11 300 + vt =5W
Þ =
9 300 - vt M 10 W N

Þ 3300 – 11vt = 2700 = 9vt 60


So current I = = 10A
5 +1
Þ 600 = 20 vt
Þ vt = 30 m/s = (6 × 5) m/s \ B=5 3W 6W
3W
3W
6W
6W

3W 6W 3W
3W 6W
31. (a) Current developed due to motion of an electron in n th N
M 3W
M N
Z2
Bohr’s orbit = I0 3 (unit ampere) here in column III
n
6W
Z 2
32 C 6´3
option (r) C0 3 , for z = 3 and n = 3 I = C0 × 3 = 0 º = 2W
n 3 3 6+3
M 3W N
2 2
z Z
32. (b) ω n = ω0 sec -1 , I n = I 0 3 A , 60
n 3
n So current I = = 20A
2 +1
EBD_7801
30 Target IIT
Due to + I effect stability of free radicals depends
r
r M M N upon the presence of electron donating group. More
36. (d) r r the number of electron donating group attached to
free radicals more is the stability i.e. 3° > 2° > 1°.
r 4r 39. (c, d)
ºr+ =
3 3
Volume of A,
Now if battery of emf 60v and resistance 1W is con-
nected across M and N. Then 1 ´ R ´ 200
VA = = 200 R [Q PV = nRT ]
1
4r/3
M N = 200 × 0.082 = 16.4 L
60 180
I= =
4r 4r + 3 gives work in isothermal and reversible expansion of an ideal
+1
1W 3 gas
60v
0.082 ´ 800
VB = = 32.8L
r r r r 2
M
N M r r N 6.082 ´ 400
r r VC = = 32.8L
1
Since VB > VA, expansion of gas occurs along A and B and
2r work is done by the gas.
r
=r 40. (a,c,d)
2r
O
O
60 + C
Now if battery is connected then I = Cl CH2 – Cl
r+1
OH
(P)
37. (b, c, d)

2Na3PO4 (aq) + 3Ba(NO3 ) 2(aq) O


C=O
® Ba 3 (PO 4 )2 (s) + 6NaNO3 (aq)
O CH2 Cl
Na3PO4 is the limiting reactant and is completely consumed. (Q)

0.2 O O
Mol. of Ba3(PO4)2 formed = = 0.1 .
2

3 O
Mol. of Ba(NO3)2 reacted = ´ 0.2 = 0.3 (R)
2

Mol. of unreacted Ba(NO3)2 = 0.5 – 0.3 = 0.2 OH


= mol of Ba2+ ion. LiAlH 4
¾¾¾¾
®
Mol. of Na+ in solution = 0.2 × 3 = 0.6 O – CH2 – CH2 – OH
(S)
Mol. of NO3– in solution = 0.5 × 2 = 1
38. (a, c) The stability of a carbocation increases as the number 41. (a, b, c)
of alkyl substituents bonded to the positively charged (a) B2O3 + P2O5 ® 2BPO4
carbon increases hence the correct order of the
stability of carbocations is 3° > 2° > 1°. Further ethyl (b) H3BO3 H+ + H2BO3–
carbocation is more stable then methyl carbocation. H2BO3– H2O + BO2–
SOLUTIONS - MOCK TESTS 31
(c) In borates, each B atom is bonded to three oxygen Given 140 = 7 x + 126
atoms, arranged at the corners of an equilateral triangle
7 x = 14
(sp2-hybridisation)
x=2
10
4s 45. Ans : 7
42. (a, c) [Cu (CN) 4 ] 3-
:
3d
.. ..
ZM
Density of the crystal d (g cm–3) =
.. ..4p
dsp 2 hybridization
N A ´ a3

[ Ni(CN) 6 ]2 - :
.. .. .. .. (
d ´ N A ´ a 3 2 ´ 6 ´10 23 ´ 5 ´10 –8 )
3
Z= = =2
M 75
.. .. sp3d 2 hybridization
Thus, the unit cell of cubic lattice will be body centred.
[Zn Br4 ]2 - :
.. .. .. .. For bcc lattice, 4r (radius of atom) = Diagonal of cube
sp3 hybridization
= 3a
[Cr(NH3 )6 ] 3+
: .. .. ..
.. .. .. d 2sp3 hybridization (always) r=
3
´ a ´ 102 =
1.732 ´ 5 ´10 2
= 216.5 = 217 pm
4 4
43. (b, d) Aryl halides are stable due to resonance stabilization.
(1Å = 10–8 cm = 102 pm)
The resonating structures
Given 217 pm = 30 x + 7

+ + + 30 x = 210
Cl Cl Cl
x=7
– –
46. Ans : 6

stabilise the aryl halide. These structures include a NaCl(aq) (cathode) : 2H 2 O + 2e - ¾


¾® H 2( g ) + 2OH (-aq )
double bond between C and Cl which is shorter and
thus stronger than the usual C – Cl single bond. The + -
CuSO4(aq) (cathode) : Cu 2(aq ) + 2e ¾¾® Cu (s)
sp2 hybridised carbon, being electronegative, makes
the C – Cl bond shorter and stronger.
44. Ans : 2 600 ´ 1
Equivalents of OH– = mole of OH– formed = = 0. 6
1000
Total V.P., P = PAº X A + PBº X B = PAº X A + PBº ( 1 - X A )
31.8
= (PAº - PBº )X A + PBº Equivalents of Cu deposited = = 1. 0 ;
63.6 / 2

Thus, PBº = 114 torr ; PAº - PBº = 52


0.6 ´100
Current efficiency = % = 60%
1
or PAº = 166 torr
Given 10 x = 60
1 1
Hence P = 166 ´ + 114 ´ = 140 torr
2 2 x=6
EBD_7801
32 Target IIT
47. Ans : 2
Re d hot
-5 -2
49. (c) HC º CH ¾¾¾¾¾¾ ®
Rate ( S N 2 ) = 5.0 ´ 10 ´ 10 [ R - X ] iron,873K

= 5.0 ´ 10-7 [ R - X ]
NO2

Rate ( S ) = 0.20 ´ 10 -5 [ R - X ]
N1 HNO3 + H2SO4

NO2
5 ´ 10 -7 [ R - X ] ´ 100
% of S = = 20
N2 5 ´ 10 -7 [ R - X ] + 0.20 ´ 10 -5 [ R - X ]
NH2
Given 20 = 10 x \ x=2

H2S.NH3 NaNO2 + H2SO4

48. Ans : 3
NO2
CH3
H+
¾¾¾
®
CH3 OH
N2 HSO4 OH

CH3 CH 2 CH = CH 2 + CH3CH = CHCH3 H3O


Butene-1 Butene-2
E55555555555555555555555F (cis - and trans )
F NO2 NO2
[C6H5NO3]

CH3CH 2 CH =
Br2
CH 2 ¾¾¾ * HCH Br
® CH3 CH 2 C 2
Butene-1 | 50. (b) NO2 NH2
Br
(+ ) - and (–) enantiomer
Sn/HCl CH3COCl

Br
¾¾¾

NHCOCH3 NHCOCH3

Conc. H2SO4

SO3H

CH3 NHCOCH3
® H Br
Br2
¾¾¾ NO2
H Br Conc. HNO3 dil H2SO4 /
CH3 (desulphonation)
meso

SO3H
So F can have three possible structures.
SOLUTIONS - MOCK TESTS 33

NHCOCH3 NH2 éH+ ù


52. (b) Ecell = -
0.059 ë ûa
NO2 NO2 log
1 éH+ ù
OH ë ûc
hydrolysis = - 0.059(pHc – pHa )

(C6H6N2O2) = - 0.059 ( 6 - 3) = - 0.177V

Q Ecell is – ve, so reaction is Non-spontaneous.


s
KMnO4 + OH/
51. (d) O2N CH3 53. (a) Ecell = E (Q,2HÅ | H — ESCE
2O)

Where,

H / H2O E = E° - 0.059 pH
O2N COO (Q,2H + /H 2O) (Q,2H + /H 2O)

é ° ù
\ Ecell = ê (E(Q,2H + / H O) – 0.059 pH) - ESCE ú
ë 2 û
O2N COOH
= (0.7 – 0.059 pH) – (0.24V)

= (0.7 – 0.059 × 10) – (0.24V) = – 0.13 V


SOCl2
SOCl Q Ecell is – ve, so reaction is endergonic (i.e DG = +ve)
O2N COCl
54. (b) Ecell = E° – 0.059 pH

= 0.7 – 0.059 × 2 = 0.582 V


O
NH
NH3 Since Ecell is + ve, so reaction is exergonic
O2N C NH2

(C7H6N2O3)
EBD_7801
34 Target IIT
ANSWER KEY PAPER - 2
1 (d) 14 (b) 27 (c, d) 40 (a) 53 (b)
2 (b) 15 (a) 28 (a, b, d) 41 (a) 54 (c)
3 (a) 16 (d) 29 (b, d) 42 (a)
4 (d) 17 (b) 30 (a, b) 43 (a)
5 (a) 18 (c) 31 (a, b, d) 44 (a, c, d)
6 (b) 19 (c) 32 (a, c, d) 45 (a, b, d)
7 (c) 20 (a) 33 (b) 46 (a, b, c)
8 (a, b, c, d) 21 (a) 34 (a) 47 (a, b, c, d)
9 (a, c) 22 (c) 35 (b) 48 (a, b, c)
10 (a, c) 23 (a) 36 (a) 49 (b, c, d)
11 (b, c) 24 (b) 37 (c) 50 (a, b, c)
12 (a, b) 25 (b) 38 (a) 51 (c)
13 (a, c, d) 26 (a, c) 39 (a) 52 (a)

PAPER - 2 é æ a + a öù
= 2n -3 ê 2na1 + 2n ç 1 n ÷ ú
1. (d) Let f (x1) = f (x2) Þ [x1] = [x2] this not implies that x1 = x2 ë è 2 øû
[For example, if x1 =1.4 and x2 = 1.5,
then [1.4] = [1.5] = 1] = 2n -2 [na1 + Sn ].

\ f is not one-one. 4. (d) Let S = (1 + x)1000 + 2x (1 + x)999 + 3x2(1 + x)998


Also, f is not onto as its range I (set of integers) is a + . . . 1000. x999 (1 + x)+ 1001. x1000 ... (i)
proper subset of its co-domain R. Above series is a arithmetic-geometric series with
2. (b) Q x + y + z = a (1 + w + w2 ) + b (1 + w + w2) = 0
x
common ratio of G.P. equal to
(Q 1 + w + w2 = 0) (1 + x )
Þ x3 + y3 + z3 = 3xyz
x
= 3 (a + b) (aw + bw2) (aw2 + bw) So multiplying both sides by , we get
(1 + x )
= 3 (a + b) [a2w3 + b2w3 + ab (w2 + w4)]
= 3 (a + b) [a2 + b2 + ab (w2 + w)] xS 1001.x1001
= x (1 + x ) + 2x 2 (1 + x ) + . . . +
999 998

= 3 (a + b) (a2 + b2 – ab) = 3 (a3 + b3) (1 + x ) (1 + x )


n n . . . (ii)
k
3. (a) å
k =0
n
Ck Sk = å
k =0
n
Ck
2
[2a + (k - 1)d ] From (i) – (ii), we get

æ x ö
÷ S = (1 + x ) + x (1 + x ) + x 2 (1 + x )
1000 999 998
çè 1 -
éæ dö
n
d
n ù 1+ x ø
= êç a1 - ÷
êëè 2 ø k =0åk n Ck +
2 k =0 å
k 2 n Ck ú
úû
1001.x1001
+ . . . + x1000 -
(1 + x )
æ dö d
= ç a1 - ÷ n.2n-1 + [n.2n-1 + n(n - 1)2n- 2 ]
è 2ø 2
é æ x ö 1001 ù
ê1 - ç ú
= a1.n.2n -1 + dn(n - 1)2n - 3. S 1000 ê è 1 + x ÷ø ú - 1001. x
1001
Þ = (1 + x )
(1 + x ) ê æ x ö ú (1 + x )
ê 1 - çè ÷ø ú
= n.2n - 3 [4a1 + an - a1 ] = n.2 n -3 [3a1 + an ] ëê 1 + x ûú
SOLUTIONS - MOCK TESTS 35

é a – 2 = 2 a 2 + 1 or (a – 2)2 = 4 (a2 + 1)
S x1001 ù
= (1 + x ) ú .(1 + x )
1000
Þ ê1 -
(1 + x ) êë (1 + x )1001 úû or a2 + 4 – 4a = 4a2 + 4 or 3a2 + 4a = 0
or a (3a + 4) = 0 or a = 0, – 4/3
x1001 \values of a and b are – 4/3, 1 respectively according
- 1001.
(1 + x ) to the given choices.
8. (a, b, c, d)
Þ S = (1 + x)1002 – x1001 (1 + x) – 1001. x1001
\ coefficient of x50 in above expression = 1002C50 We have sin x 8 cos 2 x = 1
5. (a) Since a and b are unequal,
1
2
a +b 2 Þ sinx |cosx | =
> a 2 b 2 (A.M. > G.M. for unequal numbers) 2 2
2
Case –I : When cos x > 0
Þ a2 + b2 > 2ab
Similarly b2 + c2 > 2bc and c2 + a2 > 2ca 1
In this case sinx cos x =
Hence 2 (a2 + b2 + c2) > 2 (ab + bc + ca) 2 2

Þ ab + bc + ca < 1
1 p 3p 9p 13p
6. (b) log3 | x | < 2 Þ | x | < 3² = 9, x ¹ 0 Þ sin 2x = + Þ 2x = , , ,
2 4 4 4 4
\ – 9 < x < 9, x¹0
p 3p 9p 13p
\ set of integral values of x Þ x= , , ,
8 8 8 8
= {–8, –7, ..........–1, 1, 2, 3, ..........8}
As x lies between 0 and 2p and cos x > 0
Þ –2 < log3x < 2 , x ¹ 0 Þ 3–2 < x < 32, x ¹ 0
Þ 1/9 < x < 9, x ¹ 0 p 3p
\x = ,
8 8
\ set of integral values of x = {1, 2, 3, .........., 8}
Case–II :When cosx < 0 .
Þ set of integral values of x satisfying either
log3 | x | < 2 or | log3 x | < 2 is 1
In this case sin x |cos x | =
{–8, –7, ..............–1, 1, 2, .........., 8} 2 2

\ number of values of x is 16.


1 1
7. (c) Given x2 + y2 – 2x = 3 Þ sinx cosx = – of sin 2x = –
2 2 2
\ centre is (1, 0) and radius is 2 and x2 + y2 – 4y = 6
5p 7p 13p 15p
\ centre is (0, 2) and radius is 10 . Þ x= , , ,
8 8 8 8
Since line ax + by = 2 touches the first circle.
5p 7 p
Þ x= , as cos x < 0
a(1) + b(0) - 2 2 2 8 8
\ = 2 or (a – 2) = [2 a + b ] ...(i)
2 2
a +b Thus the values of x satisfying the given equation which lie

Also the given line is normal to the second circle. Hence p 3p 5p 7p


between 0 and 2p are , , ,
it will pass through the centre of the second circle. 8 8 8 8

\ a (0) + b (2) = 2 or 2b = 2 Þ b =1
p
Putting this value in equation (i) we get These are in A.P. with common difference
4
EBD_7801
36 Target IIT
9. (a, c)
dy sec 2 (x / 2) æ xö
Let the given lines be represented by AB, BC and CA, = cos x - sin x l n ç tan ÷
dx 2 tan (x / 2) è 2ø
respectively. Then A (–2, 2), B (2, – 2) and C (1, 1) are the
vertices of the triangle ABC. Also, note that
dy æ xö
AB = 4 2, AC = 10 and BC = 10 . This shows that D = cot x - sin x l n ç tan ÷
dx è 2ø
ABC is isosceles, it is clearly not right-angled or equilateral.

d2 y æ xö
BC2 + AC2 - AB2 10 + 10 - 32 = - cot 2 x - 2 - cos x l n ç tan ÷
Since cos C = = <0 dx 2 è 2 ø
2 (BC) (AC) 2 10 10

Þ D ABC is obtuse-angled d2 y
2
+ y + cot 2 x = 0
10. (a, c) dx

x
ì x -1 (c) y = 2 + c1 cos x + c 2 sin x + cos x l n tan
, x ³1 2
| x - 1 | ïï x 2
f(x) = =í
x2 ï1 - x , x < 1, x ¹ 0
dy d æ æ xöö
îï x 2 = - c1 sin x + c 2 cos x + cos x l n ç tan ÷ ÷
dx dx çè è 2øø

ì x-2 d2 y d2 æ æ x öö
ï , x < 1, x ¹ 0 = - (c1 cos x + c2 sin x) + cos x l n ç tan ÷ ÷
x3 2ç
è è 2øø
ïï dx2 dx
\ f ' (x) = ídoes not exist , x =1
ï 2-x
ï , x >1 d2 y æ xö
îï x3 Þ = -c1 cos x - c2 sin x - cot 2 x - 2 - cos xl n ç tan ÷
dx 2 è 2ø

Clearly, f ' (x) > 0 for x < 0 or 1 < x < 2


d2 y
Þ + y + cot 2 x = 0
and f ' (x) < 0 for 0 < x < 1 or x > 2 dx 2
Thus, f(x) is increasing for (–¥, 0) È (1, 2) 12. (a, b)
and decreasing for (0,1) È (2, ¥).
1 1
dx 2 -n
11. (b, c) In = ò 2 n = ò (1 + x ) dx
0 (1 + x ) 0
(a) y = 2 + c1 cos x + c 2 sin x
1
x 1
dy = - ò (-n)(1 + x 2 )- n -1.2x.x dx
= - c1 sin x + c 2 cos x (1 + x 2 ) n 0 0
dx

1
d2 y 1
= - c1 cos x - c 2 sin x = 2 - y = + 2n ò (1 + x 2 ) - n -1.x.x dx
dx 2 2n 0

d2 y 1
1
x 2dx 1
1
1 + x2 - 1
+ y-2 =0
dx 2 = + 2n ò = + 2n ò dx
2n 0 (1 + x 2 ) n +1 2n 0 (1 + x 2 ) n +1

æ xö
(b) y = cos x l n ç tan ÷ + 2 1
è 2ø = + 2nI n - 2nI n +1
2n
SOLUTIONS - MOCK TESTS 37

\ 2n In +1 = 2- n + (2n - 1) I n (2n)! n! (2n)!


n(S) = = ,
n
(2!) .n! 2n
1 1 1 p
\ 2I 2 = + I1 = + tan -1 x |10 \ I2 = +
2 2 4 8 where S denotes the sample space

uur uuur [n equal groups each of 2, distributed in n persons]


13. (a, c, d) OA . OB = -3
n (E) = (nC1)2 . (n–1C1)2 . (n–2C1)2 ....... (1C1)2 = (n!)2 ,
2.3cos q = -3 where E: denotes the event that each of the n persons
draw balls of different colour.
1 2p
cos q = - Þ q =
2 3 (n!)2 .2n 8 (n!) 2 .2 n 8
P(E) = = = n =
r r r r r r (2n)! 35 2 .n![1.3.5....(2n - 1)] 35
(2(a ´ b ) - 2a ).(2(a ´ b ) - 3b )

n! 8
r r r r 3 1 = = which hold true for n = 4.
= 4 | a |2 | b |2 sin 2 q + 6 a. b = 4. - 6. = 0. [1.3.5....(2n - 1)] 35
4 2

Angle C is 90°. W W W W
18. (c) If n = 4 we have 8 ;
B B B B
14. (b) Since it is downward parabola
P1 P2 P3 P4 4 persons
D
Þ - is greatest value
4a 8!.4! 7!
n(S) = 4
=
(2!) .4! 2
[(12) 2 - 4 (21)( -4)]
= - = 30
4( -4) 4!
Now 4 white can be grouped, 2 in each in
2!2!2!2!
For Qs. 15 to 16
ways and similarly 4 black can also be grouped in
Given a + b – c = 2 ....(1)
4!
ways. Hence number of ways in which we
and 2ab – c2 = 4 ....(2) 2!2!2!2!
a2 + b2 + c2 + 2ab – 2bc – 2ca = 4 = 2ab – c2 4! 4!
have 4 groups each of two balls of same colour, is . .
(b – c)2 + (a – c)2 = 0 Þ a = b = c 8 8

Þ triangle is equilateral These groups can be distributed in 4 person in 4! ways.

3 4! 4!
.4 = 3 Hence n (A) = . .4! = 6.6.6
Also a = 2 from (1) Þ area of D ABC = 8 8
4
Where A denotes the event that 4 persons have draw
Also f (x) = 2 (x2 + x + 1)
both balls of same colour.
Þ f is increasing in [0, 1] Þ f (x)|max = 6
6.36.2 72.6 3
\ P (A) = = =
D 7! 7.720 35
and r1 = = 3
s-a
19. (c) By Snell’s law
15. (a) 16. (d)
sin i m 2
=
n white P1 P2 P3 ......Pn sin r m1
17. (b) 2n 1442443
n black n persons
EBD_7801
38 Target IIT
the bottom-most point.
B Li = Lf (L = angular momentum)
i= 1
2 v 2
air or mrv 0 = Iw = (mr 2 ) 1 Þ v0 = v1
5 r 5
A 1 3
P
4 D
Q 23. (a) The velocity of the photoelectrons is found from
2
F = ma
r = q4
C
v2 e
evB = m or v= BR
R m

For i = q1 , r = q 4 and µ1 = 1 The kinetic energy of the photoelectrons is then

1 2 1 e2 B2 R 2
sin q1 K= mv =
m2 = 2 2 m
sin q 4

1 (1.6 ´ 10-19 C)2 (2 ´ 10-2 T) 2 (23 ´ 10 -3 m) 2


20. (a) Fm = qvB , and directed radially inward. =
2 (9.1 ´ 10-31 kg)
mv2
\ N - mg sin q + qvB =
R = 2.97 ´ 10 -15 J

1keV
mv 2 or K = (2.97 ´10 -15 J ) = 18.6keV
Þ N= + mg sin q - qvB 1.6 ´ 10 -16 J
R

Hence at q = p/2 The energy of the incident photon is

hc 12.4 keV
2mgR En = = = 24.8 keV
Þ Nmax = + mg - qB 2gR = 3mg - qB 2gR l 0.50
R
The binding energy is the difference between these
21. (a) The displacement of the particle is given by
two values:
x = A sin(-2wt) + Bsin 2 wt BE = E n - K = 24.8keV - 18.6 eV = 6.2 keV

B
= - A sin 2wt + (1 - cos 2wt) Ep RW
2 24. (b) For the first case x = ´
L RW + R
B B
= -(A sin 2wt + cos 2wt) +
2 2 where R is the series
resistance. Thus for the cell of 1 volt emf with balancing
This motion represents SHM with an amplitude length L/2,

2.5 10 L
B2 1 = xL/2 or 1= ´ ´
A2 + L 10 + R 2
4
22. (c) In ground reference frame the sphere has no relative or 20 + 2R = 25 or R = 2.5 W
velocity, but relative to belt it performs pure rolling For second case series resistance
motion, i.e., it has only angular velocity w when pure
rolling starts. No slipping condition yields 2.5 10
R' = 2R = 5 W \ x¢ = ´
L 10 + 5
v1
wr = v1 or w = 1 = x'l' where the new balancing length l' is
r
15L
Now we apply angular momentum conservation about l¢ =1 ´ = 0.6 L
25
SOLUTIONS - MOCK TESTS 39
25. (b) The network is simplified as follws : For figure (ii), image created by curved part acts as object
for the flat part.

1.5 1 0.5
R For curved part , - = Þ v = 18 cm
v ¥ 6
2R 2R
R
From flat surface, object is at 12 cm to the right.
2R 2R
A R B 1 3/ 2 1 1
For flat part, - =0 Þ - = 0 Þv=8
2R R 2R v 12 v 8
2R 2R 27. (c, d)
RX R
where X = 2R + + 2R .....(1)
R+X 3RT 8RT RT
v rms = ; v= = 2.55
M pM M
For the upper half,
2RT
vp =
M

1 1 1 1 1 1 æ3 ö 3
= + + Average KE = mv2rms = m ç vp2 ÷ = mvp2
R AB X R X 2 2 è2 ø 4

28. (a, b, d)

where RAB is the equivalent resistance between A and é x ù


B 0 £ x £ a ; ê - ò E x dx ú + V(0) = 0 as (Ex = 0)
êë 0 úû
From eq. (1)
4R (R + X) + RX = (R + X) X é x ù é x s ù s
x ³ a ; ê - ò E x dx ú + V(a) = ê-ò dx ú + V(a ) = - (x - a)
4R (R + X) = X2 êë a Î0 úû Î0
ëê 0 ûú
X2 – 4RX – 4R2 = 0
x
æ s ö s
4R ± 16R + 16R 2 2
x £ 0 ; – ò E x dx + V(0) = - ç - x ÷ + V(0) = x
X= = 2R (1 + 2 ) 0
è Î0 ø Î0
2

(negative sign is not possible) 29. (b, d)


Equivalent circuit,
1 2 1 2 1 2
= + = + = ;
R AB X R 2R(1 + 2) R R

R
R AB = eO e
2 r/2 r/2 Oe r/4
X Y

26. (a, c)
r r
As in figure (i), rays are parallel incident on curved part.
(ii)

6cm P P
figure(i) figure(ii) æ Bwa 2 ö
Induced emf e = ç ÷
1 3 / 2 1- 3 / 2 è 2 ø
Therefore - = Þ v = 12 cm
v ¥ -6
EBD_7801
40 Target IIT
y2 z2
Now, + = 36 represents the equation of a circle with
æ X - eö æ X - 0ö
By nodal equation : 4 ç + =0 centre (0, 0, 0) and radius 6 units the plane of the circle is
è r ÷ø çè r ÷ø
perpendicular to x-axis.

4e 2Bwa 2 X 2Bwa 2
Þ 5X = 4e Þ X = = and I = = Y
5 5 r 5r

also direction of current in r will be towards negative termi-


nal i.e. from rim to origin.
Alternatively by equivalent of cells (figure (ii)) : A B
(-2,0,0)
O (2,0,0)
X
4
e 4e
I= =
r 5r
r+ Z
4

30. (a, b)
Note : Since the spherical mass distribution behaves as if
The maximum horizontal distance from the vessel comes from the whole mass is at its centre (for a point outside on the
hole number 3 and 4. sphere) and since all the points on the circle is equidistant
from the centre of the sphere, the circle is a gravitational
v = 2gh ® h is height of hole from top. equipotential.
Horizontal distance The same logic holds good for option (d).
33. (b) By mirror formula :
2 (H - h)
x = vt = 2gh = 2 h(H - h)
g 1 1 1 1
+ = Þ v = +5 cm \ m = +
v -10 10 2
1
31. (a, b, d) % error in measurement of ‘r’ = ´ 100 = 10%
10 1
the image revolves in circle of radius cm .
2
0.52 + 0.56 + 0.57 + 0.54 + 0.59
Tmean = = 0.556 » 0.56 S Image of a radius erect Þ particle will revolve in the
6
same direction as the particle. The image will complete
one revolution in the same time 2s.
0.04 + 0 + 0.01 + 0.02 + 0.03
DT = = 0.016 » 0.02S Velocity of image,
6

\ % error in the measurement of ‘T’ 2p 1 p


v = wr = ´ = cm / s = 1.57 cm / s
2 2 2
0.02
= ´ 100 = 3.57%
0.56 34. (a) For M1, u = – 10, f = – 15, h = 2.

% error in the value of g

O I
DT æ DR + Dr ö 10
=2 ´ 100 + ç ÷ ´ 100
T è R -r ø 2 6
45°
30
= 2 ( 3.57 ) + æ 1 + 1 ö ´100 » 11%
ç 60 - 10 ÷
è ø
M2 M1
32. (a,c,d) The gravitational field intensity at the point O is zero
(as the cavities are symmetrical with respect to O). Now the
40cm
force acting on a test mass m0 placed at O is given by
F = m0 E = m0 × 0 = 0 Using mirror formula
SOLUTIONS - MOCK TESTS 41
39. (a) If the reaction is assumed to proceed to completion,
1 1 1 1 1 1
+ = Þ + = the final pressure will be doubled the initial pressure.
v u f v -10 -15
Then for a first order reaction, one can write.

1 1 1 3- 2
Þ = - = Þ v = 30cm. 2.303 pf - pi 2.303 (1 - 0.5) atm
v 10 15 30 k= log = log (1 - 0.6) atm
t pf - pi t

h2 v For t = 100 s
and h = - u Þ h 2 = 6 cm.
1
2.303 0.5 atm
The image formed by the plane mirror is at 70 below the k= log
0.4 atm
= 2.23 × 10–3 s–1
100s
principal axis and 70 + 6 – 30 = 46 of the concave mirror.
\ coordinates of I2 w.r.t. P = (– 46, – 70) For total pressure of 0.65 atm,

35. (b) WAB = PDV = 10 × 10 = 100 J Rate the reaction = k × p (SO2Cl2)

36. (a) DQ = DU + DW From the reaction stoichiometry,

For cyclic process DU = 0 p(SO2Cl2) = 2pi – pt = 1 atm – 0.65 atm = 0.35 atm
So, Rate of reaction = 2.23 × 10–3 s–1 × 0.35 atm
1 1
DQ = DW = – p r1r2 = - p (10) (10) = -25pJ = 7.8 × 10–4 s–1 atm.
4 4

DW ® –ve for anticlockwise cycle 40. (a) The overall dissociation constant is the reciprocal of
overall stability constant i.e. 1/b4 = 4.7 × 10–14
37. (c) We are given that
41. (a) The equation for cell reaction is :
V = 1 L º 1 dm3 º 10–3 m3 ; N = 2.0 × 1021
Zn(s) + Ni(NO3)2 ® Zn(NO3)2 + Ni(s)
p = 7.57 × 103 N m–2
or Zn(s) + Ni2+ (aq) ® Zn2+ (aq) + Ni(s)
cmp/Crms = 0.84; cmp = ?
In this cell zinc acts as anode while nickel acts as cathode
so Ecell = Ecathode – Eanode
2.0 ´ 1021
Now, amount of gas =
6.023 ´ 1023 mol-1 (on the basis of SRP)
\ Ecell = – 0.24 – (– 0.75) = + 0.51 V
pV Nernst equation for above reaction
Using ideal gas equation, we get T =
nR
0.0591 [Zn 2+ ]
Ecell = Eºcell – log
(7.57 ´ 103 N m-2 )(10-3 m3 ) n [Ni 2 + ]
=
(2.0 ´ 1021 / 6.023 ´ 1023 mol-1)(8.314 J K-1 mol-1 )
At equilibrium Ecell = 0 and n = 2
= 274.2 K
0.0591 [Zn 2 + ]
So, 0 = 0.51 – log
3RT 2 [Ni 2 + ]
Now, crms =
M
[Zn 2+ ] 0.51 ´ 2
1/ 2 or log = = 17.25
-1
é 3(8.314 J K mol) (274.2 K) ù [Ni 2 + ] 0.0591
=ê ú = 494.22 m s–1
êë (28 ´10-3 kg mol-1 ) úû
[Zn 2 + ]
or = 1.8 × 1017
cmp = (0.84) (494.22 ms–1) = 415.14 m s–1. [Ni 2 + ]

38. (a) 2-butyne forms cis-2-butene with Lindlar’s catalyst. Let x mol L–1 be the concentration of Ni2+(aq) at
equilibrium.
EBD_7801
42 Target IIT
So Zn2+ (aq) ion conc. = (1 – x) mol L–1 » 1 mol L–1
(II) 2NO + O 2 ¾¾
® 2NO 2

1 1
So = 1.8 × 1017 or x = = 5.56 × 10–18 M (III) 2NO 2 + H 2 O ¾¾
® HNO3 + HNO2
x 1.8 ´1017

Hence, [Ni2+] = 5.56 × 10–18 M (IV) 3HNO2 ¾¾


® HNO3 + 2NO + H 2 O
42. (a) The scheme of reaction is Let the moles of NH3 = x moles

4 2 1 x
NaOH NaOH (excess) \ n HNO3 = (III reaction) = x ´ ´ ´ =
X A Soluble complex B 4 2 2 2
(metal) (White ppt)

4 2 1 1 x
Heated strongly n HNO3 = (IV reaction) = x ´ ´ ´ ´ =
+HCl(dil.) 4 2 2 3 6

D 4 2 1 2 x
C (used to extract metal)
n NO = x ´ ´ ´ ´ =
(Soluble) 4 2 2 3 3

x x 2x
From the given information, it can be concluded that, Total n HNO3 = + = moles
2 6 3
X is aluminium
\ Moles of HNO3 obtained in III reaction (if HNO3 is not
A is Al(OH)3
x n NH3
B is Na+ [Al(OH)4]– used to produce HNO3) = =
2 2
C is AlCl3 (aq)
x/6 100
D is Al2O3 \ n HNO3 = (IV reaction) = 2x / 3 ´ 100 = 3 %
of n HNO3 in III reaction
The reactions are given below.
x/6 1
n HNO3 = (IV reaction) = = (Total n HNO3 )
Al + 3NaOH ¾¾
® Al(OH)3 + 3Na+ 2x / 3 4
limited (A) white ppt n NO x/3
´ 100 = ´ 100 = 50%
Total n HNO3 2x / 3
Al(OH)3 + NaOH ¾¾
® Na+[Al(OH)4]–
45. (a, b, d)
(excess) Sodium tetrahydroxoaluminate
The nitrates of Mg and Ba decompose as follows
(soluble) (B)
heat
2M(NO3 ) 2 ¾¾¾ ® 2MO + 4NO 2 (g) + O 2 (g) [M = Mg
Al(OH)3 ¾¾¾ heat
® Al2O3(s) + 3H2O or Ba]
(O)
The option (c) is incorrect because of the fact that Mg(NO3)2
is more covalent than Ba(NO3)2. Due to this Mg(NO3)2
Al(OH)3 + 3HCl(dil.) ¾¾
® AlCl3(aq) + 3H2O decomposes more readily.
(C) (soluble)
46. (a, b, c)
All aldehydes, not containing a-Hydrogen atoms, react with
43. (a) Fe3+ (aq) + SCN - (aq) ¾¾
® [Fe(SCN)]2 + (aq)
Red cold concentrated. alkali to form a corresponding alcohol
and a salt of the corresponding acid. The aldehyde gets
oxidised as well as reduced.
Fe3+ (aq) + 3OH - (aq) ¾¾
® [Fe(OH)3 ](s)
Brown
HCHO + NaOH ¾¾
® HCOO– Na+ + CH3OH
44. (a, c, d) Methanal (sodium methanoate) (methanol)

(I) 4NH 3 + 5O 2 ¾¾
® 4NO + 6H 2 O H2CO always oxidized in crossed-Cannizzaro reaction.
SOLUTIONS - MOCK TESTS 43
47. (a, b, c, d) 2 unpaired electrons

NH3 (aq), D
A ¾¾¾¾¾ Br2 + KOH Ne+2 (15) : (s 2s)2 (s* 2s)2 (s 2pz)2 (p 2px)2 (p 2py)2 (p* 2px)2
® B ¾¾¾¾¾ ® C (C6H7N)
(p* 2py)2(s* 2pz)1
NH2
1 unpaired electron
C is an aromatic amine. So its structure is (Aniline)
51. (c) Orbital angular momentum for f orbital ( l = 3)

h h
3(3 + 1) = 12
Based on the reactions, 2p 2p
h
CONH2 52. (a) The angular momentum must be a multiple of .
2p
nh
B is an amide. So its structure is (Benzamide) L= [where n = 1, 2, 3, 4, ........ for K, L, M, N shell
2p
respectively]

COOH Q
EtO
53. (b) CH3 - C - CH - COOEt ¾¾¾®
and A is an acid. So its structure is (Benzoic acid) || |
O H

48. (a, b, c)

In Lassaigne’s method, the appearance of a bluish green or


a blue colouration, confirms the presence of nitrogen in the
organic compound. In Beilstein test appearance of green or
bluish green colour of the flame indicates the presence of
halogens. If organic compound is fused with sodium
peroxide (Na2O2) and the mass is extracted with water and CH3 - C - CH - COOEt
|| |
extract is boiled with conc. HNO3 and then ammonium O CH 2 - CH3
molybedate solution is added. The appearance of yellow
precipitate or colour ation sh ows the presence of
phosphorus. In sodium nitroprusside test the appearance CH3 - C - CH 2 - CH 2 - CH3
of violet colouration indicates sulphur. ||
O
49. (b, c, d)

I. E, of O (Z = 8) is less than that of N (Z = 7) because of EtOQ


54. (c) CH3 - C - CH 2 - COOEt ¾¾¾®
presence of half-filled orbitals in case of N (1s 2 2s 2 ||
2px12py12pz 1). It is an exception. In general I.E. increases on O
moving from left to right in a period.

50. (a, b, c)
2

NO (11) : (s 2s)2 (s* 2s)2 (s 2pz)2 (p 2px)2 (p 2py)2 (p* 2px)1

NO+ (10) : (s 2s)2 (s* 2s)2 (s 2pz)2 (p 2px)2 (p 2py)2; B.O. = 3


Acidic hydrolysis
CH3 - C - CH - COOEt ¾¾¾¾¾¾¾ ®
O+
2 (11) : B.O. = 2.5 || |
O CH 2 - COOMe
Higher the bond order, higher is the stability
CH 2 - COOH
|
OF+ (12) : (s 2s)2 (s* 2s)2 (s 2pz)2 (p 2px)2 (p 2py)2 (p* 2px)1 CH 2 - COOH
(p* 2py)1 ; CH3 COOH + succinic acid
Telegram @unacademyplusdiscounts

Join Us on Telegram for More Such Books

https://telegram.me/unacademyplusdiscounts

Join us from the above link or search ''unacademyplusdiscounts'' in Telegram


EBD_7801
44 Target IIT
SOLUTIONS - MOCK IITs
MOCK IIT - 3
ANSWER KEY - PAPER 1
1 (a, b) 13 (a) 25 (a, d) 37 (a, c) 49 (c)
2 (b, c) 14 (b) 26 4 38 (a, b, c) 50 (d)
3 (b, c, d) 15 (d) 27 2 39 (c, d) 51 (c)
4 (a, b, d) 16 (d) 28 2 40 (a, c) 52 (c)
5 (a, c, d) 17 (c) 29 0 41 (b, c, d) 53 (b)
6 (a, b, d) 18 (a) 30 6 42 (a, b, c) 54 (d)
7 (b, c) 19 (b, d) 31 (b) 43 (a, b, c)
8 (0) 20 (a, b, c, d) 32 (a) 44 2
9 (4) 21 (a, d) 33 (c) 45 3
10 (1) 22 (c, d) 34 (b) 46 2
11 (0) 23 (a, b) 35 (a) 47 5
12 (0) 24 (a, b, c) 36 (c) 48 7

EXPLANATORY NOTES

PAPER - 1 n n n

1. (a, b)
Þ å (kx - 1) < å [kx] < å (kx + 1)
k =1 k =1 k =1
(a + b) x + (a – b) y – 2ab = 0
n
and (a – b) x + (a + b) y – 2ab = 0 xn (n + 1) xn (n + 1)
- n < å [kx] < +n
Equation of the angle bisectors are 2 k =1
2
(a + b) x + (a – b) y – 2ab = 0 = ± ((a – b) x + (a + b) y – 2ab)
2bx – 2by = 0 i.e., x = y x æ 1ö 1 1 n x æ 1ö 1
and 2ax + 2ay – 4ab = 0 i.e., x + y = 2b çè 1 + ÷ø - < 2 å [kx] < çè1 + ÷ø +
2 n n n k =1 2 n n
\ Equation of third side is given by
Taking limit as n ® ¥ , we have
(i) x – y = k satisfying the point (b – a, a – b)
\ k = 2b – 2a n
x
\ the line is x – y = 2 (b – a)
lim
n ®¥
å [kx] = 2
k =1
(ii) x + y – 2b = k passing through the point (b – a, a – b)
\ k = – 2b 4. (a, b, d) ì 1
\ the line is x + y = 0 ï x +1 , 0 £ x < 1
2. (b, c)f (n) = [n]3 – [n3] = n3 – n 3 = 0 ï
ïï 2
f (x) = í , 1£ x < 2
f (n + ) = lim{[n + h]3 - [(n + h)3 ]} ï x
h ®0
ï 3
ï , 2£ x < 5/2
= hlim {n 3 - [n3 + h (h 2 + 3n 2 + 3nh)]} ïî x - 1
®0

= n3 – n3 = 0 = f (n)

f (n - ) = lim{[n - h]3 - [(n - h)3 ]}


h ®0 3
3 3 2 2
= lim {(n - 1) - [n - h (h + 3n - 3nh)]} 2
h ®0

= (n – 1)3 – (n3 – 1) ¹ f (n) except when n = 1. 1


3. (b, c, d) For any integer k, ½
kx – 1 < [kx] < kx + 1 O 1 2 5/2
SOLUTIONS - MOCK TESTS 45
Clearly f (x) is discontinuous and bijective (both injective 7. (b, c) Curve through the intersection of S1 and S2 is given
1 lim f (x) = 2 by S1 + lS2 = 0
and surjective) function lim- f (x) = ;
x ®1 2 x ®1+
Þ x 2 (sin 2 q + l cos 2 q) + 2(h tan q - lh¢ cot q) xy
æ ö 1
min ç lim f (x), lim f (x)÷ = ¹ f (1)
è x ®1- x ®1+ ø 2 +(cos 2 q + l sin 2 q) y 2 + (32 + 16l) x + (16 + 32l) y

max (1, 2) = 2 = f (1). + 19(1 + l) = 0


5. (a, c, d) The above equation will represent a circle if
f (x + y) = f (x) + f (y) + xy (x + y)
sin 2 q + l cos 2 q = cos 2 q + l sin 2 q
f (h)
f (0) = 0 \ lim = -1
h®0 h Þ (1 - l)(sin 2 q - cos 2 q) = 0

f (x + h) - f (x) f (x) + f (h) + xh (x + h) - f (x) Þ l = 1 or q = p


\ lim = lim 4
h®0 h h®0 h
Also h tan q - l h 'cot q = 0
f (h)
= lim + lim x (x + h) = -1 + x 2
h®0 h h®0 Þ h tan q = lh ¢ cot q which is satisfied if
p
l = 1and q =
x3 4
\ f ' (x) = – 1 + x2 \ f (x) = -x+c
3
Þ h = h¢
\ f (x) is a polynomial function, f (x) is twice differentiable
for all x Î R and f ' (3) = 32 – 1 = 8 8. Ans : 0

6. (a, b, d) Note that P(Z £ m) = P{ X £ m, Y £ m} 2 tan -1 ( 2 x + 1) = cos -1 x

= P{X £ m} P{Y £ m} Þ cos éë 2 tan -1 ( 2 x + 1) ùû = x

But P{Y £ m} = P{X £ m}


1 - ( 2 x + 1)
2
4x2 + 4x
= P(X = 0) + P(X = 1) + .................. + P(X = m) or = x, or =x
1 + ( 2 x + 1) 4 x2 + 4 x + 2
= p + pq + pq2 + ..................
1 1
m Þ 2 x3 - x = 0 or x = 0, ,-
+ pqm -1 = p(1 - q ) = 1 - q m 2 2
1- q
Let us verify whether these roots satisfy the parent equation.
m 2
\ P(Z £ m) = (1 - q ) p p
x = 0 Þ L. H. S. = ; R. H. S. = x is a root …(1)
Now, P(Z = m) = P(Z £ m) – P (Z £ m – 1) 2 2

m 2 m -1 2 1
= (1 - q ) – (1 - q ) x=
2
Þ L. H. S. = 2 tan - 1 ( 2 +1 )
= 1 - 2q + q
m 2m
- (1 - 2q m - 1 + q 2 m - 2 )
æ ö
2 2 +2
= 2 pq m
-1
– p(1 + q) q 2 m - 2 tanp]-1 2
[Q 1 – q = 2 + 1 = p + tan -1 ç ÷
è (
ç1- 2 + 2 2 +1 ) ÷
ø
Clearly, å P(Z = m)

( )
m ³1
æ 2 2 +1 ö
p 3p
2m - 2 = p + tan
-1 ç ÷ =p - =
= å [ 2 pq m -1 – p (1 + q) q ] çè -2 - 2 2 ÷ø 4 4
m ³1

2p p (1 + q) 1 ö p
= - = 2 – 1 = 1. -1 æ
1 - q 1 - q2 R. H. S. = cos çè ÷=
2ø 4
EBD_7801
46 Target IIT

1
x= is not a root … (2) é 1 ù ìï 1/ 2 æ 3 ö ïü
2 = ê - - 3 ú - í- - 3ç- ÷÷ ý
ë 2 û ïî 2 ç
è 2 ø ïþ
1
x=–
2
Þ L. H. S. = 2 tan -1 2 - 2 ( ) 1 1 3 7
= - - 3+ - = + 3
2 4 2 4
= -2 tan -1 ( )
2 -1 < 0
7 æ 49 ö 7 1
Thus, A1 + A2 + 8 A1 A2 = + 8 ç - 3÷ = + = 4
1 ö 3p 2 è 16 ø 2 2
-1 æ
R. H. S. = cos çè - ÷= 4
2ø 10. Ans : 1
We have
1
\x=– is not a root. … (3)
2 tan x - sin tan -1 (tan x )
LHL = lim
Thus from (1), (2) and (3), x = 0 is the required solution. p
x® - tan x + cos 2 (tan x)
2
9. Ans : 4

The given curve is C1 : y = sin 2 x - 3 | sin x | tan x - sin x


= lim
x ®p / 2- tan x + cos 2 (tan x )
5p
Now, C1 cuts the x-axis i.e, when y = 0 at x = – and x
6
sin x
1-
p tan x 1- 0
= for x Î (–p, p ) i.e. = lim = =1
6 x ®p / 2-
2
cos (tan x) 1+ 0
1+
We have two values of a . tan x

5p p -
a= -
6
and a =
6 (As x ® p , 0 < x < p \ tan -1 (tan x ) = x
2 2

[ y = sin 2 x - 3 sin x for x Î ( 0, p ) p


Further as , x ® -, tan x ® ¥ and cos 2 (tan x) is a real
2
and y = sin 2 x + 3 sin x for x Î ( -p , 0 )]
number between 0 and 1)
Thus,
p /6
tan x - sin tan -1 (tan x)
RHL = lim
A1 = ò
0
p /6
( sin 2 x - ) é cos 2 x
3 sin x dx = ê -
ë 2
ù
+ 3 cos x ú
û0
p
x® +
2
tan x + cos 2 (tan x )

é 1/ 2 æ 3 öù é 1 ù tan x + sin x
= ê- + 3ç - - + 3 (1) ú
2 ç 2 ÷÷ ú êë 2 û
= lim
p tan x + cos 2 x(tan x )
ëê è ø ûú x® +
2

1 3 1 7
=- + + - 3= - 3 sin x
4 2 2 4 1+
tan x 1+ 0
= lim = =1
p 2
x ® + 1 + cos (tan x )
1- 0
0 2
tan x
A2 = ò ( sin 2 x + )
3 sin x dx
5p
- p+ p
6 (As x ® , x > Þ tan–1 tan x
2 2

0 = tan -1 tan( x - p) = x - p
é cos 2 x ù
= ê- - 3 cos x ú
ë 2 û - 5p \ sin tan -1 (tan x) = sin( x - p ) = - sin x
6
SOLUTIONS - MOCK TESTS 47
p Hence, f (x) = 0 " x ³ 0. So, f (c) = 0
Further as x ® +; tan x ® -¥ and cos 2 (tan x) is a real
2
number between 0 and 1) 1
13. (a) Since, 1 + (y¢)2 = ... (1)
LHL = RHL = 1 \ required limit = 1. y2
11. Ans : 0 Differentiating above equation with respect to y¢ gives,
2y¢ = 0 Þ y¢ = 0
4 é ln 2 ( ln 2) 2 ùú
I=
2 ò ê -
êë ln x ( ln 2 )( ln x ) 2 úû
Putting y¢ = 0 in equation (1), we get

1
1+0=
y2
1 4é 1 ù
dx = ( ln 2) ò ê - ú dx
Þ y=+1
2 ê ln x
ë ( ln x) 2 úû
which will be a singular solution. To find general solution
t t of differential equation.
Put ln x = t Þ x = e Þ dx = e dt
For x = 2, t = ln 2 , x = 4, t = ln 4 = 2 ln 2 1 1 - y2
(y¢)2 = -1=
y2 y2
2ln 2
2ln 2 é 1 1ù t é et ù
\ I = ( ln 2 ) ê t - 2 ú e dt = ( ln 2 ) ê t ú
ln 2 ò ë t û 1 - y2
ëê ûú ln 2 Þ y¢ = ±
y
é 4 2 ù
= ( ln 2 ) ê - ú = 0.
ë 2 ln 2 ln 2 û y dy
Þ = + dx
12. Ans : 0 1 - y2

Given that, for x ³ 0, After integrating, on both sides, we get

x 0 1 - y 2 = + (x + C)
F (x) = ò f (t )dt Þ F (0) = ò f (t )dt = 0
0 0 where C is an arbitrary constant.
Hence, (x + C)2 + y2 = 1
As f (x) £ cF ( x)"x ³ 0, we get f (0) £ cF (0) Þ f (0) £ 0 14. (b) Since, y¢ (1 – y2) = 2 – y... (B)
Since f ( x ) ³ 0 "x ³ 0, we get Differentiating with respect to x, we get
2y¢ (1 – y)2 = 0 ... (C)
f (0) ³ 0 \ f (0) = 0
Eliminating y¢ from (B) and (C), we get
Since, f is continuous on [0, ¥], F is differentiable on
2- y
[0, ¥], and F'(x) = f (x) " x ³ 0. (y¢)2 =
(1 - y )2
Since f ( x) £ c F ( x) £ 0 " x ³ 0, Multiplying both sides
2- y
by e–cx (the integrating factor) we get Þ ·(1 - y )4 = 0
2
(1 - y )
d - cx
e- cx F '( x) - ce -cx F ( x) £ 0 Þ [e F ( x)] £ 0
dx Þ (1 – y)2 (2 – y) = 0
So, g (x) = e–cx F(x) is a decreasing function on 2
æ dy ö 2- y
[0, ¥)i.e., g ( x) £ g (0) for each x ³ 0. Here, ç ÷ =
è dx ø (1 - y )2
o
But g (0) = e F (0) = 0
dy 2- y
\ g ( x) £ 0"x ³ 0 Þ =±
dx 1- y
Þ e- c x F ( x) £ 0 " x ³ 0 Þ F ( x) £ 0 " x ³ 0
(1 - y ) dy
But it is given that f ( x ) ³ 0 "x ³ 0. Þ = + dx
2- y
EBD_7801
48 Target IIT
On integrating both sides, we get 3x – 5x < – 24
– 2x < – 24
(1 - y ) dy
ò 2- y
= ± ò dx + C x > 12
Hence, there is no solution of the given system of
2 inequalities.
Þ 2 - y (2 - y - 3) = m x - C
18. (a) Since, 3x – 7 > 2 (x – 6)
3
3x – 2x > – 12 + 7
4 x>–5
Þ (2 – y) (y + 1)2 = (x + C)2
9
and 6 – x > 11 – 2x
Þ 4 (2 – y) (y + 1)2 = 9 (x + C)2 – x + 2x > 11 – 6
Differentiating with respect to C, we get x>5
0 = 18 (x + C) Hence, x Î (5, ¥)
x+C=0 19. (b, d)
So, (y + 1)2 (2 – y) = 0
From Moseley’s law n = a(Z - b)
Þ y=2
n = 0 at Z = b From graph n = 0 at Z = 1
which will be the singular solution.
\b=1 \ Ka
15. (d) Since, y = (y¢)2 – 3xy¢ + 3x2 ... (A)
The general solution of above differential equation is given 3RC
For K a , n= (Z - 1)
by 4
y = Cx + C2 + x2
3RC
Differentiating with respect to x, we get Put Z = 101 Þ n= 100
4
– x – 2C = 0
Photon energy = hn
-x 3 3
Þ C= = hRC 100 2 = ´ (13.6 eV) × 1002 = 102 keV
2 4 4
-x 20. (a, b, c, d)
Putting C = in the equation (A), we get Area under the curve is equal to number of molecules of the
2
gas sample.
2
æ xö æ- xö 2 3 2 1
y = ç - ÷· x - ç ÷ -x = x Hence N = av0 Þ av0 = 2N
è 2ø è 2 ø 4 2
16. (d) Since, 5(2x – 7) – 3 (2x + 3) < 0 dN a
Also, =
Þ 4x – 44 < 0 vdv v 0
Þ x < 11 v0
and 2x + 19 < 6x + 47 ò vdN v
1 0 v 2a 2
– 4x < 28 v avg = 0 = ò dv = 3 v0 (using av0 = 2N)
N N 0 v0
x>–7
Hence, x Î (– 7, 11) vavg 2
Þ =
3x x v0 3
17. (c) Since, +5<
2 2 v- v0
1 1 a v2
3x x
v 2rms =
N ò v2 dN =
N ò v2
v0
v dv = 0
2
- +5<0 0 0
2 2
v 1
2x < – 10 Þ rms =
v0 2
x<–5
3
5x Area under the curve from 0.5 v0 to v0 is of total area.
and, x - <-8 4
3
SOLUTIONS - MOCK TESTS 49

3l PV PV
1 1
21. (a, d) =L = 2 2
2 T1 T2

If V2 = 2 V1 and T2 = 3T1 then

PV
1 1 P ´ 2V1 2
= 2 Þ P1 = P2
T1 3T1 3
3 Now change in internal energy
Minimum nodes = 2, Fundamental n = V
2L
f f
(excluding ends) DU = [nR (T2 – T1)] = [P2V2 – P1V1]
2 2
For monoatomic gas f = 3

3 é3 ù
DU =
2 ê 2 P1 ´ 2V1 – PV
1 1 ú = 3P1 V1
ë û
Next higher nodes = 5
22. (c, d) \ (b) is the correct option.
For cylinder : Now assuming that the pressure on the piston on the right
hand side (not considering the affect of spring) remains the
m0 Ir same throughout the motion of the piston then,
B= ; r<a
2pR 2
kx kx
Pressure of gas = P1 + Þ P2 = P1 +
m0 I A A
B= ; r³a
2pr where k is spring constant and A = area of piston
r r r
Bdue to remaining = Bdue to whole - Bdue to removed 1 2
portion cylinder portion Energy stored = kx
2
Current that can be consider in whole cylinder = 4I/3
kx 3 kx
and in removed part = I/3. P2 = P1 + Þ P1 = P1 +
A 2 A
m 0 (I / 3) m I
At point A : B = 0 - =- 0 P1 kx
2p (R / 2) 3pR =
2 A
m 0 æ 4I / 3 ö æ R ö m I PA
At point B : B = ç ÷ ç ÷ -0 = 0 \ kx = 1
2 è pR ø è 2 ø
2 3pR 2
GM Also,
23. (a, b) For r > R, the gravitational field is F =
r2 V2 = V1 + Ax
V1 = Ax
GM GM F1 r22
\ F1 = and F2 = Þ = V1
r12 r22 F2 r12 \ x=
A
GM 1 P1 A V1 1
For r < R, the gravitational field is F = ´r \ Energy = ´ = PV
R3 2 2 A 4
1 1

GM GM \ A is correct
\ F1 = 3
´ r1 and F2 = 3
´ r2
R R Now
F1 r1 æ kx ö kx
Þ = W = ò PdV = ò ç P1 + ÷ dV = ò P1dV + ò A dV
F2 r2 è Aø
24. (a, b, c) kx
\ W = ò P1dV + ò ´ (dx) A
Applying combined gas law A
EBD_7801
50 Target IIT

kx 2 2A
\ W = P1 (V2 – V1 ) + and Q2 = (C – C1)V = ´V
2 3Î0 d

Q1 k
é PV P2V2 4 P1 ù \ = \ (c) is incorrect
1 1 Q2 2
ê Here on applying T = T we get P2 = 3 ú
ê 1 2 ú Also V = E × d
ê 2V1 ú
ê and V2 = V1 + Ax Þ x = [QV2 = 3V1 ] ú V
ë A û \ E= = E1 = E2 \ (a) is a correct option
d
1 P1 A 2V1 7
\ W = 2P1V1 + ´ ´ = PV
1 1
2 3 A 3 26. 4
C is correct option dI 1 - 0.5 0.5
Given : e = 10 V and = = = 2.5 A/s
Heat supplied dt 0.2 0.2
Q = W + DU
e 10
7 PV
1 1 + 3 ( P V – PV )
Self inductance of coil L= = = 4H
= 2 2 1 1 dI / dt 2.5
3 2
27. 2
7 PV 3 é4 ù 41 ur
1 1
= + ê P1 3V1 – PV
1 1ú = PV
1 1 Magnetic field ( B) at the origin = Magnetic field due to
3 2 ë3 û 6
semicircle KLM + Magnetic field due to other semicircle
KNM.
25. (a, d) C1 ur µ I µ I
K Therefore, B = 0 (-$i) + 0 ( $j )
A/3 Q1 E1 4R 4R

ur µ I µ I µ I
Þ B = - 0 $i + 0 $j = 0 ( -i$ + $j )
4R 4R 4R
E2
ur µ I
2A/3 Q2 [ B due to a circular current carrying loop is 0
2R
C2 = C – C1
d \ For semicircle it is half]
Therefore, magnetic force acting on the particle,
This is a combination of two capacitors in parallel. ur r ur ì µ Iü
Therefore F = q (v ´ B) = q í( -v0 i$) ´ ( -$i + $j ) ´ 0 ý
î 4R þ
C = C1 + C2 \ C2 = C – C1
kA 2A -µ0 qv0 I $
where C1 = and C - C1 = = k
3Î0 d 3Î0 d 4R

C - C1 2 ur µ qv I 8R
\ = \ F = 0 0 = = 2 units
C1 k 4R 4R

C 2 28. 2
\ -1 = The arrangement is equivalent to three capacitors in parallel
C1 k

C 2 e o A / 4 10
\ = +1 C1 = = = 2.5 mF ;
C1 k d 4

C 2 K eo A / 2 10
= +1 C2 = = 4 ´ = 20 mF ;
C1 k d 2
\ (d) is a correct option. e o A / 4 10
C3 = = = 2.5 mF
kA d 2
Now, Q1 = C1V = ×V
3Î0 d
SOLUTIONS - MOCK TESTS 51

\ C p = C1 + C2 + C3 + = 2.5 + 20 + 2.5 = 25 µF = 52 µF
35. (a)
\ x=2
29. 0 K1
(1) Heat lost by steam at 100°C to change to 100°C water
mLvap = 0.05 × 2268 × 1000 = 1,13,400 J K2
(2) Heat lost by 100°C water to change to 0°C water
M
= 0.05 × 4200 × 100 = 21,000 J
(3) Heat required by 0.45 kg of ice to change its temperature
K1 K 2
from 253 K to 273 K = m × Sice × DT = 0.45 × 2100 × 20 = K= but condition given as K1 and K2
18,900 J K1 + K 2
(4) Heat required by 0.45 kg ice at 273 K to convert into becomes twice
0.45 kg water at 273 K = mLfusion = 0.45 × 336 × 1000 = 2K1 × 2K 2 2K1 K 2
151,200 J so K = =
2K1 + 2K 2 K1 + K 2
From the above data it is clear that the amount of heat
required by 0.45 kg of ice at 253 K to convert into 0.45 kg of 36. (c) in case III, K1 and 2K2 are parallel so K = K1 + 2K2,
water at 273 K (1,70,100 J) cannot be provided by heat lost T = 2π M K1 + 2K 2
by 0.05 kg of steam at 273 K to convert into water at 273 K.
Therefore, the final temperature will be 273 K or 0ºC. 37. (a, c)
30. 6 (a) is wrong because Wurtz’s reaction is not applicable for
3º alkane.
The smallest resonating length l1 , corresponds to the
(c) is wrong because the addition on cis form gives meso
fundamental mode. The diameter D, of the pipe is not very product in suitable condition.
small compared to the resonating length l1 . So one should
38. (a, b, c)
account for the end correction which is generally taken to
be in the range of 0.29 D to D/3. Taking the end correction to Bond enthalpy of C – C bond
be 0.3 D. = Enthalpy required to break C2H6 into gaseous atoms
l – 6 × bond enthalpy of C – H bond
l1 + 0.3 D = ; where l is the wavelength of the
4 = 2839.2 KJ mol–1 – 6 × 410.87 kJ mol–1= 373.98 kJ mol–1
fundamental mode.
Bond enthalpy of C = C bond = Enthalpy required to break
Then l= 70 cm and the velocity of sound = 336 m/s = 330 + 6 C2H4 into gaseous atoms – 4 × bond enthalpy of C – H
\ z=6 bond = 2275.2 kJ mol–1 – 4 × 410.87 kJ mol–1
31. (b) For pure rotational motion = 631.72 kJ mol–1
Vp = Rw = 8cm sec–1 and Vo = – Rw = –8cm sec–1 For the formation of benzene having Kekule structure, we
32. (a) For body rolling without slipping have to form 3C – C bonds, 3C = C bonds and 6C – H bonds
for which enthalpy released is
Vp = Vcm + wR and Vo = 0
[3 (–373.98) + 3 (–631.72) + 6(–410.87)] =– 5482.32 kJ mol–1
33. (c) For body in pure rotational motion
But the given value of DfH is DfH (actual) = – 5536 kJ mol–1
Vp = Vo = Vcm
Hence resonance energy compared to Kekule structure
For body rolling with slipping
= DfH (actual) – DfH (Kekule structure)
Vp ¹ 2wR and Vo ¹ 0
= (–5536 + 5482.32) = – 53.68 kJ mol–1
34. (b) K1 2K2 39. (c, d)
M The opposite lobes of p-orbital have different sign but
opposite lobes of d-orbitals have the same sign (two lobes
have +ve sign while other two lobes have -ve sign). Orbitals
K = K1 + 2K2 = K1 + 2K1 = 3K1
of a particular type will have the same angular wave function
Þ T = 2π M 3K1 irrespective of the value, n.
EBD_7801
52 Target IIT
40. (a, c)
Let the rate equation be KMnO4 /OHs /D®
(d) ¾¾¾¾¾¾¾¾
Rate, R0 = k [A]n [B]m
Using the given data, one can write
42. (a, b, c)
5.07 × 10–5 = k (0.2)n (0.3)m ............. (1)
(a) [PtCl2(NH3)4]Br2 is isomeric with [PtBr2(NH3)4]Cl2
5.07 × 10–5 = k (0.2)n (0.1)m ............. (2)
(b) [Co(NO 2 )(NH 3 ) 5 ] 2+ is isomeric with
7.6 × 10–5 = k (0.4)n (0.05)m ............. (3)
[Pt(ONO)(NH3)5]2+
From equation (1) and (2)
(c) [Co(NH3)6] [Cr(CN)6] is isomeric with [Cr(NH3)6]
-5 n m m
5.07 ´ 10 k (0.2) (0.3) æ 0.3 ö [Co(CN)6]
-5 = n m =ç ÷ = 3m
5.07 ´ 10 k(0.2) (0.1) è 0.1 ø (d) O.N. of Pt in the complex is +2.
1 = 3m; This gives, m = 0 43. (a, b, c)
(a) Four unpaired electrons can form bonds with four Cl
From eqs. (2) and (3),
atoms giving rise to sp3d hybridisation.
7.6 ´10 -5 k (0.4) n (0.05)m n m (b) TeCl4 + 2HCl ® H2[TeCl6]
æ 0.4 ö æ 0.05 ö
= n m =ç ÷ ç ÷ 44. Ans : 2
5.07 ´ 10-5 k(0.2) (0.1) è 0.2 ø è 0.10 ø
D Tb(normal) = Kb m = 2.53 × 1 = 2.53 K;
= 2n × 2–m = 2n – m = 2n
ΔTb (obs) 1.518
1.5 = 2n i= = = 0.6
ΔTb (nor) 2.53
n log 2 = log 1.5
n = log 1.5 / log 2 = 0.17609 / 0.3010 = 0.58 0.5
æ 1ö æ 1ö
Thus the order of reaction with respect to A is 0.5, and with i = 0.6 = 1 - ç1 - ÷a = 1 - ç1 - ÷a ; a = 0.8 = 80%
è nø è 2ø
respect to B is 0.
41. (b, c, d) Given 80 = 40A
\ A= 2
45. Ans : 3
NaBH
(b) ¾¾¾¾

Magnetic moment = n( n + 2) = 15 B.M. (given)
Þ n=3
46. Ans : 2
Al2O3 D
¾¾¾¾¾ ®
-H2 O SO 2Cl 2 + 2H 2O ® H 2SO 4 + 2HCl

H 2SO 4 + Ba (OH ) 2 ® BaSO 4 + 2 H 2 O


Baryta
O3 /H 2O + HCOOH
¾¾¾¾® 2HCl + Ba (OH) 2 ® BaCl 2 + 2H 2O
47. Ans : 5
2H +(aq) + 2e - ¾¾
® H 2(g) , E º = 0.0V ......(i)
[O]
+
2D(aq) + 2e- ¾¾
® D2(g) , E º = -0.296V ......(ii)
(i) – (ii), 2H +(aq) + D 2(g) ¾¾ +
® H 2(g) + 2D (aq) ,
D 3
(c) ¾¾¾¾
®
º
-CO 2 Ecell = 0.296V
+
º 0.0592 PH ´ [D (aq) ]2
COO – Ecell = 0 = Ecell - log 2 +
;
OH
s
+ CH3X 2 PD [H (aq)
2
]2
Haloform reaction

H
+ + º
COOH [D (aq) ] Ecell 0.296
log = = =5
+ 0.0592 0.0592
[H (aq) ]
SOLUTIONS - MOCK TESTS 53
48. Ans : 7
3
Suppose x% of height of ice immerses in mercury. 50. (d) d = high spin m spin = n(n + 2) B.M
– – e 0g
For floating condition, weight of ice 3(3 + 2)
t 32g
= weight of displaced Hg
(n = 3) 15
or 100 × dice × g = x × dHg × g CFSE = – 0.4 × 3 = 3.87 B.M
or 100 × 0.98 × g = x × 13.60 × g Þ x = 7.3 = – 1.2
Hence, % height remaining above the surface of Hg
= 100 – 7.3 = 92.3 » 93 51. (c) d7 (low spin)
1
– e g CFSE = – 0.4 × 6 + 0.6 × 1
Given : 93 = 100 – A = – 1.8
6
t 2g
\ A=7
49. (c) d6 (high spin) n=1

2
e g CFSE = – 0.4 × 4 + 0.6 × 2 m spin = n(n + 2) B.M
= 0.4
t42g = 1(1 + 2)
(n = 4) = 3
= 1.73 B.M
m spin = n(n + 2) B.M
= 4(4 + 2) = 24 = 4.90 B.M

+ H
+
H
+ +
52. (c) CH3CH2CH2CHCH3 CH3CH2CH = CHCH3 CH3CH2CHCH2CH3
(Five hyperconjugative Highly favoured due to
structures) inductive of ethyl group
(4 hyperconjugative structures)

– –
Cl Cl

Cl

CH3CH2CH2CHCH3 CH3CH2CHCH2CH3

Cl

é + + ù
53. (b) H+
CH 2 = CH - CH = CH 2 ¾¾¾ ® êCH 3 C HCH = CH 2 ¬¾® CH 3CH = CH C H 2 ú
ë û
2°Carbocation, stable due to resonance

Cl Cl
- | |
Cl
¾¾¾® CH3CHCH = CH 2 + CH3 CH = CHCH 2
(1, 2 - addition) (1, 4 - addition)

+ + 1, 2–
54. (d) — CH 2CH = CH 2 + H — CH 2CHCH 3
hydride shift
2º carbocation

Br
+
Br - |
- C HCH 2 CH 3 ¾¾
¾® - CHCH 2 CH 3

More stable
benzyl carbocation
EBD_7801
54 Target IIT
ANSWER KEY PAPER - 2
1 (a) 14 (b, c, d) 27 (a, b, c) 40 (c) 53 (b)
2 (c) 15 (a) 28 (a, b, c) 41 (a) 54 (a)
3 (c) 16 (b) 29 (b, c, d) 42 (d)
4 (a) 17 (b) 30 (b, c, d) 43 (b)
5 (c) 18 (d) 31 (a, b, c) 44 (a,b)
6 (c) 19 (d) 32 (a, d) 45 (a,c,d)
7 (a) 20 (b) 33 (a) 46 (a,b,c)
8 (a, b, c) 21 (b) 34 (c) 47 (a,c,d)
9 (a, b, d) 22 (b) 35 (d) 48 (a,c)
10 (a, b, c, d) 23 (a) 36 (c) 49 (b, c)
11 (a, b, d) 24 (a) 37 (c) 50 (a, c)
12 (a, c, d) 25 (b) 38 (a) 51 (d)
13 (b,c) 26 (a, b) 39 (b) 52 (a)

PAPER - 2 or a2 + 4 – 4a = 4a2 + 4, or 3a2 + 4a = 0


or a (3a + 4) = 0, or a = 0, – 4/3
1. (a) Given curve is
\ values of a and b are – 4/3 and 1 respectively according
y2 – 4y = 4x
to the given choices.
(y – 2)2 = 4 (x + 1)
100 100!
Focus : x + 1 = 1 4. (a) We have C50 =
50!50!
Þ x= 0
(0,2) The exponent of 7 in 50! is
y–2 =0 (–1,2)
Þ y= 2 é 50 ù é 50 ù
ê 7 ú + ê 2 ú = 7 +1 = 8,
Point of intersection of ë û ë7 û
the curve and y = 4 is and the exponent of 7 in 100! is
(0, 4) from the reflection
property of parabola é 100 ù é 100 ù
reflected ray passes ê 7 ú + ê 2 ú = 14 + 2 = 16
ë û ë7 û
through the focus.
Thus, exponent of 7 in 100C50 is 16 – 2 (8) = 0
2. (c) From given equations
5. (c) The given equation is equivalent to
x/2 = cos t + sin t ; y/5 = cos t – sin t
3(1 – cos2x) + 10 cos x – 6 = 0
Eliminating t from (1) and (2), we have
Þ 3cos2x – 10 cosx + 3 = 0
2 2 2 2
x y x y
+ =2 Þ + = 1, which is an ellipse. Þ (3cosx –1) (cos x – 3) = 0
4 25 8 50
Therefore cos x = 1/3 (because cos x ¹ 3).
3. (c) Given x2 + y2 – 2x = 3
Hence x = 2np ± cos–1 (1/3), n Î I.
\ Centre is (1, 0) and radius is 2 and x2 + y2 – 4y = 6
m1 - m 2
\ Centre is (0, 2) and radius is 10 . 6. (c) tan q =
1 + m1m 2
Since line ax + by = 2 touches the first circle.
m - (1 / 2)
=1
a(1) + b(0) - 2 1 + (m / 2)
\ = 2 or (a – 2) = [2 a 2 + b2 ] ...(i)
a 2 + b2 1 m
Þ m- = +1
2 2
Also the given line is normal to the second circle. Hence
it will pass through the centre of the second circle. m 3
Þ = Þm=3
\ a (0) + b (2) = 2 or 2b = 2 Þ b =1 2 2
Putting this value in equation (i), we get Trisection point (5, 4)
So, equation of line in new position will be
a – 2 = 2 a 2 + 1 , or (a – 2)2 = 4 (a2 + 1)
y – 4 = 3(x – 5) or 3x – y – 11 = 0
SOLUTIONS - MOCK TESTS 55
7. (a) Any point A on the first line is (t, 5t – 4). Any point B 0<x< 3
400 and negative for x > 3
400 , the function
æ 3r - 4 ö f(x) increases in 0 < x < 3 400 < 8. It follows that the
on the second line is ç r, ÷.
è 4 ø largest term in the sequence can be either a 7 or a8.
3r - 4 Since a7 = 49/543 > a8 = 8/89, the largest term in the
2r + t + 5t - 4
Hence, 1 = and 5 = 2 49
3 3 given sequence is a7 = .
543
Þ 2r + t = 3 and 3r + 10t = 42. 2(y - 1)
75 (b) If f1(y) = y + 1 – log y,,
On solving, we get t = .
17
(y - 1) 2
æ 75 307 ö f '1(y) = – < 0 for all y > 0
Hence A is çè , ÷ y(y + 1) 2
17 17 ø
8. (a, b, c) Applying C3 ® C3 – x C2, C2 ® C2 – xC1 Thus f1(y) is a decreasing function for all y > 0 and as
we obtain f (1) = 0, f1(y) < 0 for all y > 1.
y2 - 1
3 0 2a 2 If f2(y) = – log y,,
y
D (x) = 3x 2a 2 4a 2 x
1 1
f '2(y) = 1 – + 2 > 0 for all y > 1.
3x 2 + 2a 2 4a 2 x 6a 2 x 2 + 2a 4 y y
Apply C3 ® C3 – x C2, we get f2 (1) = 0 and f2(y) is increasing for all y > 1 so that
f2 (y) > 0, y > 1.
3 0 1
2(y - 1) y2 - 1
D (x) = 4a 4 3x 1 x \ for all y > 1, < log y <
y +1 y
3x 2 + 2a 2 2x x 2 + 2a 2
(c) We have to check if (a + b)p £ ap + bp
Apply C1 ® C1 – 3C3, we get By dividing both sides by bp , we have to prove
0 0 1 p p
æ aö æaö
ç1 + ÷ £ ç ÷ + 1 i.e., (1 + x)p £ xp + 1,
4
D (x) = 4a 0 1 x = 16a 6
è bø èbø
-4a 2 2x x 2 + 2a 2
a
\ D' (x) = 0, Dx is independent of x, where x= >0
1 b
Let f (x) = xp + 1 – (1 + x)p , x > 0
ò D (x) dx = 16a , y = 16a6 is a straight line of zero slope.
6

0 \ f ' (x) = pxp – 1 – p(1 + x)p – 1


9. (a, b, d)
ïì 1 1 ïü
ì tan -1 x, if 0 £ tan -1 x < p / 2
= p í 1- p - 1- p ý
> 0 always as 1 – p > 0 and
ïî x (1 + x) ïþ
Since | tan x | = ïí
-1
-1 -1 x >0
îï - tan x, if - p / 2 < tan x < 0 Hence the function increases in (0, ¥)
ì -1 \ f (x) = 1 + xp – (1 + x)p > f (0) = 0, which proves the
ï tan x, if x ³ 0 proposition.
í Þ | tan -1 x | = tan -1 | x | "x Î R
ïî - tan -1 x, if x < 0 (d) Let ‘b’ be the height of the cone and a be its semivertical
angle.
Þ tan | tan -1 x | = tan tan -1 | x | = | x | LD = x = radius of the inscribed cylinder and LM = h be
its height, LM = OM – OL = b – x cot a
Likewise sin | sin -1 x | = sin sin -1 | x | = | x | " | x |£ 1
cot | cot -1 x | as 0 < | cot -1 x | < p, " x Î R O

Þ cot | cot -1 x | = cot cot -1 x = x a a


| tan x | is not necessarily always equal to tan | x | OL=xcota
L
10. (a, b, c, d) C D
x2
(a) Consider the function f (x) = LM= b–xcota
x 3 + 200
in the interval [1, ¥).
A M B
x(400 – x 3 )
Since the derivative f '(x) = is positive for
(x 3 + 200)2
EBD_7801
56 Target IIT
Now, S = 2prh = curved surface = lim [ 3 + 0 + ...] = 3 ,
S = 2px (b – x cot a) or S = 2p (bx – x2 cot a) h®0

\ dS/dx = 2p (b – 2x cot a) = 0 which is a finite number.


Hence f is differentiable at 4.
1 1
\ x = (b/2) tan a or x = (b tan a) = (r1) Hence option (c) is not correct.
2 2
Option (d) is obviously correct.
æ 1ö 12. (a, c, d) x2y12 + x yy1 – 6y2 = 0
or radius of cylinder = çè ÷ø . (radius of cone)
2
- xy ± x 2 y 2 + 24x 2 y 2 - xy ± 5xy 2y 3y
Thus all statements (a, b, c, d) are correct. y1 = = = ,-
2 2 x x
11. (a, b, d) 2x 2x
(a) Here f (x) = x3 – x2 + x + 1 dy 2y dy 3y
Þ f ¢(x) = 3x2 – 2x + 1, which is strictly increasing = or =-
dx x dx x
in (0, 2)
1 dy dx dy dx
ì f (x); 0 £ x £ 1 i.e., = or ò = -3ò
\ g (x) = í3 - x; 1 < x £ 2 [as f(x) is increasing so, 2 y x y x
î i.e., ln y = ln cx2 or ln y = – ln cx3
f(x) is maximum, when 0 £ t £ x] or y = cx2 or x3y = c

ïì x 3 - x 2 + x + 1; 0 £ x £ 1 13. (b, c) y = cos x cos( x + 2) - cos2 ( x + 1)


So, g(x) = í 1
ïî 3 - x; 1< x £ 2 = [cos(2 x + 2) + cos 2 - (1 + cos(2 x + 2))]
2
ìï3x 2 - 2x + 1; 0 £ x £ 1 1
also, g¢(x) = í = (cos 2 - 1) = - sin 2 1
ïî -1; 1< x £ 2 2
which clearly shows g (x) is not differentiable at x = 1. \ Graph is a straight line y = - sin 2 1, which is
(b) When t ³ 0; | t | = t parallel to x-axis
y
\ x = 2t – t = t Þ x ³ 0 éw2
ê w3 w4 ùú
and y = t2 + t2 = 2t2 ê ú
(1, 2) 14. (b, c, d) For n = 3, P = êw3 w4 w5 ú
i.e. y = 2x2, when x ³ 0 ê ú
êw4 w5 w6 ú
When t < 0; | t | = – t –1 O ë û
x¢ 1 x
\ x = 2t – (– t) = 3t Þ x < 0
é0 0 0ù
and y = t2 + t (– t) = 0 ê ú
2
and P = ê0 0 0ú
i.e. y = 0, when x < 0 y¢ ê ú
êë0 0 0úû
ìï 2x 2 ; x ³ 0 Þ 0 £ x £ 1 It shows P2 = 0 if n is a multiple of 3.
Thus f (x) = í
ïî 0 ; x < 0 Þ -1 £ x < 0 So for P2 ¹ 0, n should not be a multiple of 3 i.e. n can
take values 55, 58, 56
Hence, the function is differentiable in [– 1, 1].
-x x
(c) We have
f (4 + h) - f (4)
15. (a) g (– x) = ò f (t)dt = -ò f (- y)dy ( where t = – y )
f ¢(4) = lim 0 0
h ®0 h x

= lim
( 2
)
f ( 4 + h ) - f (2 ) 2 = -ò f (y)dy = -g(x)
0
h® 0 h
so g (x) is an odd function.
(4 + h)3/2 - 8 8[(1 + h / 4)3/ 2 - 1]
= lim = lim x+2 2
h®0 h h®0 h
16. (b) g (x + 2) – g (2) = ò f (t)dt - ò f (t)dt
é 3h ù 0 0
8 ê1 + + .... - 1ú
ë 24 û x
= lim
h ®0 h = ò f (y + 2)dy ( where t = y + 2 )
0
é3 3 æ h2 ö ù
8ê h + ç ÷ + ...ú x
ëê 8 8 è 16 ø ûú = ò f (y)dy = g(x) Þ g (x + 2) – g (x) = g (2) for all x.
= lim
h®0 h 0
SOLUTIONS - MOCK TESTS 57
17. (b) The given equations 20. (b) Let us compute the magnetic field due to any one
segment :
x-5 y-7 z+2
= = ..........(1) m0 I
3 -1 1 B= (cos 0° + cos (180° - a))
4p (d sin a)
x+3 y-3 z-6
= = ...........(2) m0 I m I a
-3 2 4 = (1 - cos a ) = 0 tan
4 p (d sin a) 4 pd 2
Any point P on (1) is (3r1 + 5, – r1 + 7, r1 – 2) and any
point Q on (2) is (–3r2 – 3, 2r2 + 3, 4r2 + 6)
The direction ratios of PQ are I
(3r1 + 3r2 + 8, – r1 – 2r2 + 4, r1 – 4r2 – 8) ...........(3)
The line with d.r's 2, 7, -5 will be proportional to the
P
d.r's given by (3)
d
3r1 + 3r2 + 8 - r1 - 2r2 + 4 r1 - 4r2 - 8

ds
\ = = ...........(4)

in
2 7 -5
Solving (4), we get r1 = r2 = – 1
So point of intersection are P (2, 8, – 3) and Q (0, 1, 2)
and intercepted length
Resultant field will be
= PQ = (2 - 0)2 + (8 - 1)2 + (-3 - 2) 2 = 78 m0I a m I
Bnet = 2B = tan Þ K = 0
2 pd 2 2 pd
18. (d) Shortest distance between B and plane OAC is
hT0
uuur uuur uuur
21. (b) hT0
OA ´ OC × OB
h = uuur uuur
Q= ò CdT = a ln
T0
= a ln h
| OA ´ OC | T0

R
uuur uuur uuur 1 2 3 DU = CV DT = (h - 1)T0
g -1
Here, OA ´ OC × OB = 2 -1 1
3 1 2
é h -1ù
W = Q - DU = a ln h - ê ú RT0
= 1 (–2 – 1) + 2 (3 – 4) + 3 (2 + 3) = 10 ë g -1 û

ˆi ˆj kˆ l3
uuur uuur 22. (b) y = l2 -
OA ´ OC = 2 -1 1 z
3 1 2
æ z.3l2dl - l3dz ö æ 3l2 ö l3
dy = 2ldl - ç =
÷ ç 2l - dl + dz
= ˆi(-2 - 1) + ˆj(3 - 4) + k(2 z ÷ø
ˆ + 3) = -3iˆ - ˆj + 5kˆ
è z2 ø è z2

uuur uuur 10 5 æ 3 ´ 22 ö 8
| OA ´ OC | = 35 ; h = =2
=ç 2 ´ 2 - ÷ (± 0.1) + (± 0.1)
35 7 1 ø 1
è
19. (d) In cases 1, 2, 3, 4 the path difference are respectively = – 8 (± 0.1) + 8 (± 0.1) = ± 1.6
l l 3l l3 23
, l , and y = l2 - = 22 - = 4 - 8 = -4
2 4 4 Z 1
Þ phase difference are respectively \ y = – 4 ± 1.6
r r
2 æ fö
23. (a) dV = - Edr = - ( -2x 3ˆi).(dxiˆ + dyjˆ + dzk)
ˆ = 2x 3 dx
p, 2p, p / 2,3p / 2 and I = I0 cos ç ÷
è 2ø v 1

Þ ò dV = ò (2x ) ´ 10 dx Þ V = -7.5 ´ 10 V
3 3 3
I0 I0
\ the intensity in the four cases are 0, I0 , , 0 2
2 2
respectively.
EBD_7801
58 Target IIT

24. (a) li = wavelength of the incident sound Total energy µ n6


Total energy µ m–3
10u - (u / 2) 19u 27. (a, b, c)
= =
f 2f u r r
fi = frequency of the incident sound Since, W = ò F.dr

u/2 Clearly for forces (A) and (B) the integration do not require
10u - u 18
= f = f = fr S any information of the path taken.
10u - (u / 2) 19
3 (xiˆ + yj)
ˆ
ˆ = 3 xdx + ydy
= frequency of the reflected sound For (C) : Wc = ò (dxiˆ + dyj) ò (x 2 + y2 )3/ 2
2 2 3/ 2
(x + y )
l r = wavelength of the reflected sound
Taking : x2 + y2 = t
10u + u 11u 11 ´ 19 u dt
= = ´ 19 = . 2x dx + 2y dy = dt Þ x dx + y dy =
fr 18f 18 f 2
dt / 2 3 dt
Þ Wc = 3ò
2 ò t 3/ 2
l i 19u 18f 9 = which is solvable
= ´ = t 3/2
lr 2f 11 ´ 19u 11
Hence (a), (b) and (c) are conservative forces.
25. (b) (i) Q ar = constant and q ¹ 0, 180° But (d) requires some more information on path. Hence
\ resultant path is parabola non-conservative.
28. (a, b, c)
dU
(ii) F = - = -4x + 5 \ SHM 1
dx (a) q = ò idt = area of given curve q = i 0 t 0
2
(iii) At node v = 0, at antinode tension ^ to velocity
rr i t æ tö
(b) + = 1 Þ i = i0 çè1 - t ÷ø
\ at these points power = 0 (P = F.v) i0 t 0 0

At other points P ¹ 0 t0 2
2 æ tö Rt 0i 02
26. (a, b) (c) Heat = ò i 2 R dt = ò 0 çè t 0 ÷ø
i R 1 - dt ; H =
3
0
dU Ke 2 29. (b, c, d)
|F|= = 4 .............. (1)
dr r 1
V1 = V2 Þ X L = X C Þ f = = 125 Hz
2p LC
Ke 2 mv 2
= .............. (2) V0 200
r4 r I0 = = (X = 0 Q Z = R) = 2A
R 100
nh Q V1 = V2 = IXL = I.(wL) = 2 × 2p × 125 × 2/p = 1000 volt
and mvr = .............. (3) 30. (b, c, d)
2p
Dimensions of energy per unit volume are
By (2) and (3),
= dimensions of energy/dimensions of volume
Ke 2 4p 2 m m ML2T–2/L3 = ML–1T–2.
r= = K1 .............. (4)
h 2
n 2
n2 Stress, pressure and modulus of elasticity all have the
dimensions of ML–1T–2. The dimensions of work are ML2T–2.

1 1 Ke 2 31. (a, b, c) d/2


From (2) KE = mv2 =
2 2 r3

C1 C2
Ke 2 Ke 2 e1 = 2 e 2 = 4 s/2
Total energy = KE + PE = -

2r 3 3r 3
+ C3
2 2 2 6 e1= 2
Ke Ke Ke n s/2
= 3
= 3
=
6r æ K mö 6K13 m3
6 ç 12 ÷
è n ø
SOLUTIONS - MOCK TESTS 59
For equilibrium of Q
2 e0 s/2 4 e0 s/2
C1 = C2 = 4 3 4
d/2 d/2 T+ pr s 2 g = pr 3 r 2 g ...(ii)
3 3
For equilibrium of P

4 3 4
T+ pr r1 g = pr 3 s1 g ...(iii)
3 3
2 e 0 s/2 e0s (iii) – (ii) gives
C3 = =
d d
r1 – s2 = s1 – r2 ...(iv)
From (i) and (iv)
2e 0 s 4 e0 s
´ VP h VP h1
C1 ´ C2
+ C3 = d d + e0 s =– 1 \ =
Ceq = VQ h2 VQ h2
C1 + C2 6e 0 s d
d \ A is also a correct option
4 e0 s e0 s 33. (a) The magnetic field B
= +
3 d d in the space between the
two conductors is given by dr
7 e0 s 7 é e0 s ù
\ Ceq = = C1 êQ C1 = d ú m0 i
3 d 3 ë û B= …(i)
2 pr a r
32. (a, d) From the figure it is clear that O
(a) s2 > s1 b
(b) r2 > s2 [As the string is taut]
where , i = current in either
(c) r1 < s1 [As the string is taut]
of two conductors
\ r1 < s1 < s2 < r2
r = distance of the point from the axis.
When P alone is in L2 r r
Because ò B.d l = m 0 ´ current enclosed by the path
2pr 2 (r1 – s 2 ) g
VP = is negative as r1 < s2 (Ampere’s law)
9h2
or B.2p r =m 0i
Where r is radius of sphere.
The energy density in the space between the
When Q alone is in L1
conductors
2pr 2 (r2 – s1 ) g 2
VQ = is positive as r2 > s1 B2 1 é m 0i ù m 0 i 2 joule
9h1 u= = =
2m 0 2m 0 êë 2pr úû 8p 2 r 2 metre3
ur ur
Therefore V P . V Q < O option (d) is correct Consider a volume element dV in the form of a cylindrical
shell of radii r and (r + dr) as shown in the figure.

m 0i2 m 0i 2l æ dr ö
s1 Energy dW = u dV = ´ 2pr ldr =
8p 2 r 2 4p çè r ÷ø
T
T Total magnetic energy can be obtained by integrating
s2 this expression between the limits r = a to r = b.
Hence,

m 0i2l
b
æ dr ö m 0i2 l æ bö
W = ò dW = =
Also
VP r1 – s 2 h1
= ´ ...(i) 4p ò çè r ÷ø 4p
log e ç ÷
è aø
VQ r2 – s1 h2 a
EBD_7801
60 Target IIT
If L be the self inductance of length l of the cable, then As the slope of free surface is 45°. Thus free surfaces
2
the energy in magnetic field will be (1/ 2)Li . Hence passes through centre of box and having co-ordinates
(2, 2) at top of box. Thus length of exposed top part
1 2 m 0i2 l æ bö m0l æ bö = 6 – 2 = 4m.
Li = log e ç ÷ ; L = log e ç ÷
2 4p è a ø 2 p è aø 37. (a) It is given that 20 mL of stock solution of KI requires
34. (c) Let A and B are the two long parallel wires as shown 30 mL of M/10 KIO3 solution to convert I– ions into ICl
which are formed to form a current loop. The two wires according to the reaction
would obviously be carrying equal currents in opposite KIO3 + 2KI + 6HCl ® 3ICl + 3KCl + 3H2O
direction. Consider a space length l and thickness dx
For 50 mL of KI solution, the required volume of KIO3
as shown in figure.
solution would be
The magnetic flux through the area ldx due to the
currents in the two wires is df = Bldx where B is the 30 mL of KIO3 solution
intensity of magnetic field due to current carrying wires × 50 mL of KI
20 mL of KI solution
at a distance x from A.
solution = 75 mL of KIO3 solution
i After treating 50 mL of KI solution with 1 g AgNO3
A sample, the volume of KIO3 solution used is 50 mL.
x P This means KI equivalent to 25 mL of KIO3 solution is
d dx
used in precipitating out Ag+ ions from 50 mL of KI
d–x
solution.
B
i Amount of KIO3 in 25 mL of M/10 KIO3 solution (25 mL)
æ 1 ö æ 25 ö æ 1 ö
B=
m0 i m0
+ .
i ç M÷ = ç L ÷ ç mol L-1 ÷ = 0.0025 mol
2p x 2p d - x è 10 ø è 100 ø è 10 ø
m 0i æ 1 1 ö Amount of KI equivalent to this amount
df = Bldx = çè + ÷ ldx = 2 × 0.0025 mol
2p x d - x ø
Mass of AgNO3 precipitated out
d-r
m0 i æ 1 1 ö = 2 × 0.0025 × 170 g = 0.85 g
\ f= ò çè +
2p x d - x ø
÷ ldx
r 0.85g
Percent of AgNO3 in the sample = × 100 = 85.
m 0 il m il 1.0 g
d-r
[log e x - log e (d - x)]dr - r = 0 log e
2p p r 38. (c) The maximum work done by the system = P2(V2 – V1)
Magnetic flux linked with unit length of the loop æ nRT nRT ö
m i d-r \ P2(V2 – V1) = mgh or P2 ç P - P ÷ = mgh
f = 0 loge è 2 1 ø
p r
But f = Li where L is self-inductance. æ P2 ö nRT æ P2 ö
m0 d-r or mgh = nRT ç1 - ÷ or m = gh ç1 - P ÷
\ L= loge è P1 ø è 1ø
p r
39. (c) In compounds
dy ax g/2
35. (d) = = =1
dx a y + g - g / 2 + g CH3 CH3
first has
(effective g will be g – a = g/2]
q = 45°
more dipole moment than second.
36. (c) y (0,4) Therefore its boiling point will be higher. Melting point
depends on symmetry therefore I has higher melting
(2,2) point than II. Steric crowding in II is more than I there-
fore I is more stable than II.
2m 40. (d) Anions occupy fcc positions and half of the tetrahedral
1m 45° holes are occupied by cations. Since there are four
x anions and 8 tetrahedral holes per unit cell, the fraction
(4,0)
of volume occupied by spheres per unit volume of the
6m
unit cell is
SOLUTIONS - MOCK TESTS 61

æ4 ö 1 æ4 ö Repulsion between bond pairs now exceeds that


4 ´ ç pra3 ÷ + ´ 8 ´ ç prc3 ÷ ìï æ r ö3 üï
ø = p c between lone pairs. Hence the bond angle is greater
= è 3 ø 2 è 3 í1 + ç ÷ ý
3 2 r
ïî è a ø ïþ (111°) than 109°28'.
16 2ra3
(4) The difference between electronegativities of Br and O
Q for tetrahedral holes, is greater than that between Cl and O, so bonding
rc electrons are even closer to O in Br2O. Thus repulsion
= 0.225 between bond pairs should be even greater, and a bond
ra
angle greater than 111° is expected.
p 44. (a, b, c)
= {1 + (0.225)3} = 0.7493 The orbital angular momentum depends on the value of l
3 2
and not n.
\ Void volume = 1 – 0.7493
45. (a, b)
= 0.2507/unit volume of unit cell.
PCl5(g) PCl3(g) + Cl2(g)
% void space = 25.07%
41. (d) w
Since PV = nRT = RT
T
M
+ -
(a) Cathode : Cu 2(aq ) + 2e ¾¾® Cu (s) w RT r RT
M= =
PV P
1
¾® 2H + +
Anode : H 2 O ¾ O 2 + 2e - r RT
2 \ V.D.(d) =
2P
pH decreases.
PCl5(g) PCl3(g) + Cl2(g)
+ -
(b) Cathode : Cu 2(aq ) + 2e ¾
¾® Cu (s) Initial C 0 0
+ -
At eq.m. C(1 – a) Ca Ca
¾® Cu (2aq
Anode : Cu (s) ¾ ) + 2e
Total moles at equilibrium Initial vapour density
pH remains unchanged. =
Total initial moles Vapour density at equilibrium

(c) Cathode : 2H 2 O + 2e - ¾
¾® H 2 + 2OH (-aq ) C (1 + a ) D
=
C d
Anode : 2Cl - ¾
¾® Cl 2 (aq ) + 2e -
M/2 PM é PM ù
pH increases. Þ 1 + a = rRT / 2P = rRT ê\ a = rRT - 1ú
ë û
42. (a) Reduction of Cr2O3 with Al can be described by the
heat
equation Cr2O3(s) + 2Al(s) ¾¾¾ M 208.5
® 2Cr(s) + Al2O3(s) If, d = 62, = = 104.25
2 2
Then, DrGº = SDfGº (products) – SDfGº (reactants)
DrGº = (DfGº (Al2O3) + 2 DfGº(Cr) – (DfGº (Cr2O3) 104.25 - 62
a= = 0.68
+ 2DfGº (Al)) 62
DrGº=(–827 kJ mol +0)–(–540 kJmol + 0)=– 287kJ mol–1
–1 –1
P (1 + a) RT
Since DrGº for the reduction of Cr 2O3 with aluminium \ = = 0.3327 atm / (gm/L)
r M
is negative, hence the reduction of Cr 2 O 3 with
aluminium is possible. 46. (b, c)
43. (a) A meso compound has minimum two chiral centres and it
(1) Due to sp3 hybridisation has a plane of symmetry and it is optically inactive.
47. (a, b, c, d)
(2) Bond angles are distorted due to lone pairs. In OF2,
the bonding electrons are nearer the F atom due to its (a) Vacancy defects lower the density of the substance
greater electronegativity. Thus repulsion between lone (b) Interstitial defects increase the density of the substance
pairs is greater than that between bond pairs. Hence (c) Schottky defects, preserve the electrical neutrality of
bond angle is less than 109°28'. the crystal.
(3) In Cl2O, the bonding electrons are nearer the oxygen (d) Frenkel defects do not affect the density of the crystal.
atom, because it is more electronegative than chlorine. Hence all statements are correct.
EBD_7801
62 Target IIT
48. (a, b, d) Steric number of ClF3 = 3 + 2 = 5 To 100 gm of (A), add Br2 and the new weight is 593 gm
Steric number of ClF3O = 4 + 1 = 5
\ To 54 gm of (A), add Br2 and the new weight is
Steric number of ClF3O2 = 5 + 0 = 5
Steric number of (ClF2O)+ = 3 + 1 = 4
593 ´ 54
Steric number of (ClF4O)– = 5 + 1 = 6 = 320.2 gm
100
Steric number of XeOF4 = 5 + 1 = 6
So answer are (abd) as steric no. are same for
these molecules and hybridisation is same in each Weight 320.2
\ No of moles of Br2 = =
pair. Molecular weight 160
49. (c, d) Volume of A,
= 2 moles
1 ´ R ´ 200
VA = = 200 R [Q PV = nRT ]
1 (i) As two moles of Br2 were added, compound has two
= 200 × 0.082 = 16.4 L p bond hence general formula of A should be :
gives work in isothermal and reversible expansion of an
(CnH2n–2 )
ideal gas
0.082 ´ 800 CnH2n–2= 54
VB = = 32.8L
2
12n + 2n – 2 = 54
6.082 ´ 400
VC = = 32.8L
1 n = 4 \ molecular formula = C4H6
Since VB > VA, expansion of gas occurs along A and B and
(ii) (A) reacts with CH3CH2Br in presence of NaNH2
work is done by the gas.
thus, compound (A) should be terminal alkyne,
50. (a, d) C6H5NH2 + O = C(CH 3)2 –H O C 6H 5N = C(CH 3)2
2
i.e. 1-butyne
C 6H 5NHNH 2 + O = C(CH 3)2
–H2O

C 6H 5NHN = C(CH 3)2 Br


Br2
|
51. (c) Higher charge density on Al leads to higher polarising CH3 - CH 2 - C º CH ¾¾¾ ® CH3 CH 2 – C – CHBr2
CCl 4
power of Al3+ cation. So AlCl3 is most covalent in (A) |
nature. Br
52. (a) LiF is most ionic (F– is small in size therefore it has
least polarisability). Therefore, it has higher melting
point. CH3CH2Br in
Catalytic
CH3–CH2–CH2–CH3
NaNH2 Hydrogenation
53. (b) 54. (c) (C)

(A) (B)
Compound Br2/CCl4 Molecular weight
CH3– CH2 – C º C – C2H5
Molecular weight 54 excess (593 % more than A)
CH3CH2Br (D)
in
NaNH2
Catalytic
(D) Hydrogenation
(C) O3
Molecular weight
O3
(7.4 % more than A)
Diketone (E) [O ]
CH3 - CH 2 - C - C - C 2 H 5 ¾¾¾ ® 2CH3CH 2 COOH
[O]
|| ||
Propanoic acid O O
Telegram @unacademyplusdiscounts

Join Us on Telegram for More Such Books

https://telegram.me/unacademyplusdiscounts

Join us from the above link or search ''unacademyplusdiscounts'' in Telegram


SOLUTIONS - MOCK TESTS 63

MOCK IIT - 4
EXPLANATORY NOTES

ANSWER KEY - PAPER 1


1 (b) 11 A-s; B-r; C-p; D-s 21 (c) 31 A-p,r; B-q,s; C-p,r; D-q,s 41 (b) 51 A-p,q; B-p; C-p,q,r; D-s
2 (b) 12 A-p; B-r; C-s; D-t 22 (c) 32 A-p; B-r; C-r; D-p,q,s 42 (b) 52 A-p,s; B-p,q,r,s; C-p,s; D-p,s
3 (a) 13 (b) 23 (d) 33 (c) 43 (b) 53 (c)
4 (d) 14 (a) 24 (c) 34 (a) 44 (b) 54 (b)
5 (a) 15 (c) 25 (a) 35 (a) 45 (d) 55 (d)
6 (c) 16 (a) 26 (a) 36 (c) 46 (b) 56 (d)
7 (b,d) 17 (d) 27 (b,c,d) 37 (a) 47 (a,d) 57 (d)
8 (a,c) 18 (c) 28 (a,b,c) 38 (b) 48 (a,c) 58 (a)
9 (b,c) 19 (a) 29 (a,b,d) 39 (c) 49 (a,c,d) 59 (c)
10 (c,d) 20 (c) 30 (a,c) 40 (c) 50 (a,b,d) 60 (d)

PAPER - 1 y

h h
1. (b) CD = h, tan q = , tan 2q =
x L-x a=1
C

x
O
h f ' (x) = ex – 1
f ' (x) > 0 for x > 0 and f ' (x) < 0 for x < 0
q 2q
\ f (x) is increasing ( ­ ) for x > 0
A x D L–x B
and decreasing ( ¯ ) for x < 0
L
Now, x tan q = (L – x) tan 2q I I
x (tan q + tan 2q) = L tan 2q

æ tan 2q ö
2q x
æ tan 2q ö ç 2q ÷
x=0
x=ç ÷ L ; x =ç L
è tan q + tan 2q ø tan q tan 2q ÷ y = ex always lies above y = x i.e. ex – x ³ 1 for a > 1
çq + 2q ÷
è q 2q ø Hence never cuts = a = (0, 1] Þ B
3. (a) SS' = 2ae, where a and e are length of semi-major axis
2L and eccentricity respectively.
lim x =
q® 0 3
2. (b) For 0 < a £ 1 the line always cuts y = ax. (9 - 3)2 + (12 - 4)2 = 2ae , \ ae = 5
\ Centre is mid point of SS'
\ Center º (6, 8)
Let the equation of auxiliary circle be (x – 6)2 + (y – 8)2
= a2
We know that the foot of the perpendicular from the
focus on any tangent lies on the auxiliary circle
\ (1, – 4) lies on auxiliary circle.
i.e. (1 – 6)2 + (– 4 – 8)2 = a2 Þ a = 13
Q ae = 5 Þ e = 5/13
4. (d) A is non singular. \ det A ¹ 0
Given AB – BA = A, hence AB = A + BA = A(I + B)
For a > 1, say a = e det. A · det. B = det. A · det. (I + B)
Consider f (x) = ex – x det. B = det. (I + B) ....(1) (as A is non singular)
EBD_7801
64 Target IIT
Again AB – A = BA 8. (a, c)
A(B – I) = BA rr rr rr
Clearly a.c = 0 & b.c = 0 . Also a.b = 0 Þ (A)
(det. A) · det.(B – I) = det. B · det. A
r r r r r
Þ det. (B – I) = det. (B) ....(2) |a ||b| = | c| ù |a| |c| r r r
úÞ r = r Þ|a |= |c|&|b|=1
From (1) and (2) Again, r r r ú | c| |a |
| b|| c | = |a | û
det. (B – I) = det. (B + I)
5. (a) Solving line and circle r rr r r r r r
Þ a ´ b.c = | a | | b | | c | = | a |2 = | c |2
m2x2 + (d – lx)2 = m2c2 r r r
x1 (Student will assume a = ˆi, b = ˆj, c = kˆ but in this case all
(m2 + l2) x2 – 2ldx + (d2 – m2c2) = 0 2 2
x2 (l + m = 1) the four will be correct which will be wrong)
9. (b, c)
2x1x 2 2 (d 2 - m2 c2 ) n n -r
Given H = = æ 1/2 1 -1/ 4 ö 1
x1 + x 2 2ld çè x + x ÷ø
n
; Tr +1 = Cr x 2 . .x - r/ 4
2 2r
d 2 - m 2c2 1 n 1
lH = ......... (1) n n
Coefficient of the first 3 terms are C0 , C1 , C2 . 2
d 2 2

n 1 1
lx+my = d C 0 + n C 2 . = 2. n C1 .
B 4 2
(x2,y2) n(n - 1)
A 1+ =n ;
(x1,y1) 8
n(n - 1)
d 2 - c2 l 2 = (n - 1) Þ n = 8 (as n ¹ 1)
Similarly mK = ......... (2) 8
d
8- r æ 3r ö
1 1 çè 4 - ÷ø
2d 2 - c 2 (l 2 + m 2 ) c2 Tr +1 = Cr8
x 2 . .x - r/4
= Cr . r .x 8 4
(1) + (2) Þ lH + mK = = 2d - 2r 2
d d
where (l2 + m2 = 1) Number of terms with integer power of x occur when
r = 0, 4, 8 Þ 3 terms. Hence b and c are correct.
6. (c) Let z = a + i b Þ z = a - ib 10. (c, d)
Hence, we have z 2008 = z Given : (f '(x))2 + (g (x))2 = 1
x
\ | z |2008 = | z | = | z | f (x) + ò g (t) dt = sin x (cos x - sin x)
0
| z | [ | z |2007 -1] = 0
Differentiating both sides
| z | = 0 or | z | = 1, if | z | = 0 Þ z = 0 Þ (a, b) = (0, 0) f ' (x) + g (x) = cos 2x – sin 2x ....... (1)
if | z | = 1 ; z 2009 = zz = | z |2 = 1 Squaring (1),
(f '(x))2 + (g (x))2 + 2 f '(x). g (x) = 1 – sin 4x
Þ 2009 values of z 1 + 2 f '(x) . g (x) = 1 – sin 4x
Þ Total number of ordered pairs = 2010 \ 2 f ' (x) g (x) = – sin 4x
7. (b, d)
sin 4x
As a > 2 hence a2 > 2a > a > 2 Now, substituting g (x) = - in eq. (1)
2f ¢ (x)
Now (x – a)(x – 2a)(x – a2) < 0 Þ the solution set is as shown
sin 4x
f ¢(x) - = cos 2x - sin 2x
0 a 2a 2f ¢(x)
a2
Put f ' (x) = t
Between (0, a) there are (a – 1) positive integers 2t2 – 2 (cos 2x – sin 2x) t – sin 4x = 0
Between (2a, a2) there are (a2 – 2a – 1) positive integers
\ a2 – 2a – 1 + a – 1 = 18 Þ a2 – a – 20 = 0 2 (cos 2x - sin 2x) ± 4 (1 - sin 4x) + 8sin 4x
Þ t=
(a – 5)(a + 4) = 0 4
\ a = 5 and a = –4 but a > 2 \ a = 5 \ 4t = 2 (cos 2x – sin 2x) ± 4 (1 - sin 4x) + 8sin 4x
SOLUTIONS - MOCK TESTS 65
Þ 2t = (cos 2x – sin 2x) ± 1 + sin 4x (B) Clearly, RT will be the diameter of circle
\ Circumference = (p × diameter)
Taking (+) ve sign, 2t = cos 2x – sin 2x + cos 2x + sin 2x
Þ t = cos 2x p´ 4
= p ´ RT = = 1.
Taking (–) ve sign, t = – sin 2x 4p
Hence f ' (x) = cos 2x or f ' (x) = – sin 2x
(C) Since in the part (A), we have found x2 ( 2 at + at 2 ) > 2a ,
1 cos 2x
Integrating, f (x) = sin 2x + C1 or f (x) = + C2
2 2
f (0) = 0 Þ C1 = 0 and C2 = –1/2 ( if t ¹ 0 )
1 cos 2x - 1 \ For three real normals, x2 > 2a = 2 ´ 1 = 2
\ f (x) = sin 2x or f (x) =
2 2 i.e. x2 > 2.
If f ' (x) = cos 2x then g (x) = – sin 2x
If f ' (x) = – sin 2x then g (x) = cos 2x p
(D) Equation of PT is y = x + a Þ Ð PTM =
4
1
i.e. f (x) = sin 2x and g (x) = – sin 2x p PM 2
2 Þ sin = =
4 PT PT
cos 2x - 1
f (x) = and g (x) = cos 2x
2 \ PT = 2 2
11. A - s; B - r; C - p; D - s 12. A - p; B - r; C - s; D - t
2
(A) y = 4 x ..... (1) The circle is x2 + y2 – 2x – 4y – 20 = 0 and let the hyperbola
æ cö
Clearly, if P = (at 2 , 2at ) then by symmetry,, be xy = c2. If çè ct , ÷ø be the points of intersection then
t
Q = (at 2 , - 2at )
c2 4c
2
c2t 2 + - 2ct - - 20 = 0
2 t
Equation of tangent is ty = x + at t

For T, y = 0, x1 = - at 2 Þ c 2 t 4 - 2ct 3 - 20t 2 - 4ct + c 2 = 0


and equation of normal is If t1, t2, t3, t4 be its roots then,
2 20
y = -tx + 2at + at 3 å t1 = c ; å t1t2 = - c 2 ;
For R, y = 0, x2 = 2at + at 2 ..... (2)
4
Here, a = 1 Þ x1 = -t 2 and x2 = 2 + t 2 å t1t2t3 = c and t1t2 t3t4 = 1

(A) x1 + x2 + x3 + x4 = ct1 + ct2 + ct3 + ct4 = 2


x2 = 3 = 2 + t 2 Þ t 2 = 1 Þ t = ±1
(B) Sx1 x2 = c 2 St1t2 = -20
Take t = 1, then x1 = -1,
\ PM = 2at = 2 ´ 1 ´ 1 = 2 \ Sx12 = (Sx1 )2 - 2Sx1 x2 = 44

RT = x1 + x 2 = (1 + 3) = 4 c St1t 2t3
(C) Sy1 = S t = c t t t t = 4
\ Area of quadrilateral PTQR 1 1234

æ1 ö 1
= 2 ´ ç ´ 4 ´ 2÷ = 8 sq. units Sy1 y2 = c 2 S = -20
è2 ø t1t2

(at 2 , 2at ) \ Sy12 = (Sy1 )2 - 2 Sy1 y2 = 56


P (D) OA2 + OB 2 + OC 2 + OD 2 = S ( x12 + y12 ) = 100

T For Qs. 13 to 15.


O M R( x2 , 0)
(x1 , 0 ) 5R 2R
Urn-I ; Urn-II
1B 4B
Q A : first two draws resulted in a blue ball.
(at 2 , -2at )
EBD_7801
66 Target IIT

B1 : urn-I is used, P(B1 ) =


1 Let I = ò e2x (cos x - sin x) dx = e2x (Acos x + Bsin x)
2
On solving, A = 3/5 and B = –1/5 and C = 2/5
1
B2 : urn-II is used, P(B2 ) = x æ3 1 ö -x 2 -x
2 \ y = e çè cos x - sin x÷ø - (sin x - cos x) e + e
5 5 5
1 1 1 4 4 16 4 19. (a) y = | ln x | not differentiable at x = 1
P(A / B1 ) = . = , P(A / B2 ) = . = =
6 6 36 6 6 36 9
p 3p
y = | cos | x | | is not differentiable at x =,
1 1 2 2
.
2 36 1 y = cos–1 (sgn x) = cos–1 (1) = 0 differentiable
P(B1 / A) = =
1424 3 1 1 1 16 17 x Î (0, 2p)
E1 . + .
2 36 2 36 1 1
20. (c) Let ò f (t) dt = k , so f (x) = kx + 1, now ò (kt + 1) dt = k
1 16 E1
. 0 0
2 36 16
P(B2 / A) = = E
1424 3 1 16 1 1 17 k
E2 . + . E2 Þ + 1 = k, so k = 2
2 36 2 36 2
E : third ball drawn is red \ f (x) = 2x + 1,
P (E) = P (E Ç E1) + P (E Ç E2) 3

1 5 16 2
. + . =
5
+
32
=
37 Also ò f(x) dx = 12
= 0
17 6 17 6 102 102 102
13. (b) 14. (a) 15. (c) 4S
21. (c) P1 - P0 = ......... (1)
16. (a) f (0) = 2, 2R
P2
é x x ù 4S
f (x) = (e x + e - x ) cos x - 2x - ê x ò f ¢(t)dt - ò t{ f{
¢(t) dt ú P2 - P1 = ......... (2)
êë 0 úû R P1
0 I II
Add (1) and (2)
P0
f (x) = (e x + e- x ) cos x - 2x 6S
P2 - P0 =
R
é ìï x
x üï ù
- ê xf (x) - xf (0) - ít.f (t) 0 - ò f (t)dt ý ú 1 (main scale)
êë ïî 0 ïþ úû 22. (c) Least count =
N

f (x) = (e x + e- x ) cos x - 2x - x f (x) + 2x 1 (main scale)


0.1 mm = ;
20
é x ù 1 Main scale division = 2mm.
+ ê x f (x) - ò f (t)dt ú
êë 0 úû N t
dN N dN
x
23. (d)
dt
= 50 -
0.5
; ò 50 - 2N = ò dt
0 0
f (x) = (e x + e - x ) cos x - ò f (t)dt ........... (1) N = (100 (1 – e–t/2)) ; N = 25 ; t = 2 ln (4/3)
0
ur uur
Differentiating eq. (1) 24. (c) For loop B, µ0 (2i – i) = Ñò B.dl
f ' (x) + f (x) = cos x (ex – e–x) – (ex + e–x) sin x ........ (2) ur uur
For loop C, µ0 (i – 2i) = Ñ
ò lB.d
dy
Hence + y = e x (cos x - sin x) - e - x (cos x + sin x) ur uur
dx For loop A, µ0 (3i – 3i) = Ñò B.dl
ur uur
17. (d) f ' (0) + f (0) = –2 + 2 = 0 For loop D, µ0 (0 – i) = Ñò B.dl
18. (c) I.F. of DE (1) is ex.
So, B > A > C = D
y.e x = ò e 2x (cos x - sin x) dx - ò (cos x + sin x) dx 25. (a) At resonance XL = XC and Z = Zmin = R
1
y.e x = ò e 2x (cos x - sin x) dx - ò (sin x - cos x) + C XL = wL and = XC
wC
SOLUTIONS - MOCK TESTS 67
If ‘f’ is decreased then w will decrease and hence XC 30. (a, c)
will increase therefore at f < fr, circuit behaves as
capacitative.
VL and VC always differ in phase by 180° at any
frequency
1 2 1
26. (a) = + (1) (2)
F FL Fm

/////////////////
////////////////

Deviation, d = i + e – A
Dispersion, q = (dV – dR) = (nV – n R) A
d1 = –d2

///
d1 + d2 = 0
R \ Final ray will be parallel to initial ray.
Fm = ¥ Fm =
2 But q1 ¹ q2 Þ q1+ q2 ¹ 0
1 n -1 1 n -1 31. A - p, r; B - q, s; C - p, r; D - q, s
= =
FL R FL R A - p,r; Rotational Kinetic Energy (Ek)r = 40% of Translation
Kinetic Energy (Ek)t
1 2(n - 1) 1 1 2(n - 1) 2
\ = + \ = + æ2ö
F1 R ¥ F2 R R About diameter : I = ç ÷MR 2
è5ø
R R
Þ F1 = Þ F2 =
2 (n - 1) 2n æ2ö
B - q,s; (Ek)r = 66% of (Ek)t ; About diameter : I = ç ÷MR 2
è3ø
F1 n
Ratio = =
F2 n - 1 MR 2
27. (b, c, d) C - p,r; (Ek)r = 50% of (Ek)t; About diameter : I =
4
Intensity increases no. of photon coming on plates therefore
it wouldn’t change stopping potential, work function and MR 2
D - q,s; (Ek)r = (Ek)t; About diameter: I =
maximum K.E. 2
28. (a, b, c) 32. A - p; B - r; C - r; D - p, q, s
When return to same point displacement is zero therefore
A - p; More the radius of aperture more is the amount of light
average velocity is also zero.
entering the telescope.
Total distance 2 (u 2 / 2g) u f0
Average speed = = = B - r; M =
Total time 2 (u / g) 2
fe
29. (a, b, d)
Equivalent circuit C - r; L = f 0 + f e
C
D - p, q, s; Depends on dispersion of lens, spherical
aberration and radius of aperture.
C
C K2 33. (c) Just after closing, capacitor behaves as short circuit
and all current flows through it hence ammeter reads
C zero.
34. (a) After long time capacitor behaves as open circuit and
K1 no current flows through it.
V
1 V0 30
When K1 is closed, energy stored = CV 2 Therefore i = = = 2mA
2 R1 + R 2 10 + 5
5C 35. (a) Just after reopening, potential difference across R2
When K2 is also closed, equivalent capacitance =
3 remains same initially as charge on capacitor does not
change initially, hence current remains same.
1 1 5C 2 5CV 2
Energy stored E = Ceq V 2 = V = a
2 2 3 6 36. (c) E
2 2
5CV CV 2 1
Change = - = CV 2 = CV 2 mg mg
6 2 6 3
EBD_7801
68 Target IIT
43. (b)
l æ ml 2 ö
t = mg + mgl = ç + ml 2 ÷ a –
2 è 3 ø O O O EtO O
– EtOOC
EtO – EtO EtO
9g 9g 9g + EtO
a= a= ´l = OEt
8l 8l 8
O
OEt
O
OEt
O EtO O OEt O
l 1 æ ml 2 ö
37. (a) EQl + mg + mgl = ç + ml 2 ÷ w 2
2 2è 3 ø
O O O
E EtOOC EtOOC EtOOC
Q EtO –

m COOEt COOEt
EtO COOEt
O OEt
O –O

44. (b) By replacement of 'c' by 'b' in the complex [Ma 2bcde]n±


it becomes [Ma2b2de]n±
Geometrical isomers of [Ma2b2de]n±
b b a b d
mgQ 3mgl 2ml2 2 a b a b b d a d a b
- l+ = w
2Q 2 3 M M M M M
a a a e b e a e a b
2ml 2 2 3gl e d a e
mgl = w ; v= cis cis
b
3 2
Total number of geo. isomers = 6 Trans
4ml 2
38. (b) EQl = a ; Geometrical Isomers of [Ma2bcde]n±
3
a a a
mgl 4ml 2 3g e a e a d a
= a ; =a
2 3 8l
M M M
3g d c c d e c
at =
8 b b b
3g EQ a a a
a r = w2 r = a a a
2 e d c
2 2 M M M
æ 3g ö æ 3g ö 3g 1 3g b b b
a = ç ÷ +ç ÷ = +1 = 17 d e d
è 8ø è 2ø 2 16 8
c c e
39. (c) Electric field also depends upon charge distribution.
40. (c) In free expansion pressure outside is zero. So no work
cis
is done by ideal gas but in real gas work may be done
against internal force existing between molecules.
a a a
O O e e b d b
.. M M M
41. (b) H – O – S – O – H + H – .O. – N O
d c c d e c
O base
a a a
acid
Trans
H O Total number of geometrical isomers = 9
– +
HSO4 + H – O – N O H2O + NO2 Hence, decrease in number of geometrical isomers = 3
+
(-1) ´ 2 ´ 300
42. (b) O2– B.O. = 1.5 ; O22– B.O. = 1.0 ; F2 B.O. = 1.0 ; 45. (d) -190 = DE +
O2+ B.O. = 2.5 1000
DE = – 189.4 kcal/mol
O22– and F2 have same bond order and they are different
46. (b) In this case, all FeO is to be converted into Fe2O3
molecular species, hence bond length is decided by
otherwise FeO can combine with SiO2 present as
size of atoms bonded.
gangue in the haematite ore.
Bond length in O22– > Bond length in F2 because size FeO + SiO2 ¾¾ ® FeSiO3
of O– > size of F. gangue slag
SOLUTIONS - MOCK TESTS 69
47. (a, d) It has 9 primary H = 66 % given product
-4 It has 1 tertiary H= 33% given product
Ùm 200 ´ 10
a= = = 0.1
Ù¥ 0.2 66 33 4.5 reactivity with 3°H
m so = 7.33 ; = =
Acid is 10% ionised so enthalpy of ionisation of 9 7.33 1 reactivity with 1°H

2.7 33
XOH = = 3 kJ / mol = 33
0.9 1
XOH (aq) ƒ X+(aq) + OH– (aq) \Reactivity ratio for monochlorination with
c–x x x 1°H : 2°H : 3°H = 1 : 3.82 : 4.5
(0.1C)2 10-14 H
Kb = = 0.011 ; K a = = 9.1 ´ 10-13 |
0.9C 0.011
(a) CH3 - CH - CH - CH 2 - CH3
Due to cationic hydrolysis resultant solution is acidic.
|
48. (a, c) CH 2 - CH3
XY (aq) ® X+ (aq) + Y–(aq)
X+ (aq) + H2O (l) ƒ XOH (aq) + H+(aq)
Cl
At equilibrium : C (1 – a) – Ca Ca |
Kw -14 ¾¾
® CH3 - CH - CH - CH 2 - CH3
[H+] = Ca = ´ C ; (10 -5 ) 2 = 10 80 1
´ ´ |
Kb -5 M 2 CH 2 - CH3
4 ´ 10
M = 100 gm/mol (Major product)
Kh 1° H = 9 × 1 = 9
%a = ´ 100 Þ 2.5 ´ 10-3 2° H = 6 × 3.82 = 22.92 (major) % of 2-chloro-3-ethyl
C
pentane
a for XY : 3a = 2 (r + + r - )
22.92
1.732 × a = 2 (1.6 + 1.864) 3° H = 1 × 4.5 = 4.5 (minor) = = 62.93%
36.42
a = 4Å = 4 × 10–8 cm.
Effective formula unit of XY = 1 Total Þ 36.42 » 63% \ X = 63

Z.M Cl
d= 3 |
a ´ NA (b) CH3 - CH 2 - C - CH 2 - CH3 (minor)
|
1 ´ 100 1´ 1000 CH 2 - CH3
= -8 3 23
= = 2.6 gm / cc
(4 ´ 10 ) ´ 6 ´10 384
% of 3-chloro-3-ethyl pentane
49. (a, c, d)
(a) At T1 and T2 the value of DGf for MgO becomes more 4.5
= ´ 100 = 12.36% » 12 \ Y = 12
and more positive due to decrease in entropy in the 36.42
formation of MgO.
T1 CH3
2 Mg (l) + O2 (g) ¾¾ ® MgO(s) (c) (i) O3 CHO
T2 (ii) Zn, H2O 2 H–C–C +
2 Mg (g) + O2 (g) ¾¾ ® 2MgO(s) CHO
O O
(c) Beyond T3, DGf of CO is more negative than that of MgO
T3
Mg(g) + CO ­
(d) MgO (s) + C(s) 144244 3 (i) O3
CH3
CHO
gaseous product CH3 – C – C +2
¯ Rapid cooling (ii) Zn, H2O
CHO
Mg(s) O O
50. (a, b, d)
CH3 – CH2 – CH3
It has 6 primary H = 44% given product and 2 secondary (i) O3 CHO
H = 56% given product. 2 CH3 – C – CH +
(ii) Zn, H2O
CHO
28.0 3.82 reactivity with 2°H O O
44 = =
So = 7.33 ;
6 7.33 1 reactivity with 1°H

In CH3 - CH - CH3 (i) O3


| 2 CH3 – C – CH + CHO
(ii) Zn, H2O
CH3
O O CHO
EBD_7801
70 Target IIT

C2H5 C ® NO2 undergoes decolourisation of KMnO4


2KMnO 4 + 3H 2 SO 4 + 10NO 2 + 2H 2 O
(i) O3 C2H5 – C – CH CHO ¾¾
® K 2 SO 4 + 2MnSO4 + 10HNO 3
+2 NO2 molecule has angular structure (V-shape) with O–N–O
(ii) Zn, H2O
O O CHO bond angle of about 132º and N–O distance of about 1.19Å
So we obtain four types of product (given in bracket) which is intermediate between a single and a double bond.
\Z=4 D ® HNO2 decolourises KMnO4 :
(d) X + Y + Z = 63 + 12 + 4 = 79 2KMnO 4 + 3H 2 SO 4 + 5HNO 2 ¾¾
®
51. A - p, q; B - p; C - p, q, r; D - s
K 2 SO 4 + 2MnSO 4 + 3H 2 O + 5HNO3
1. 5 mole of CO2 (g) = 1.5 × 22400 mL at NTP
= 33600 ml at NTP X-ray and Raman spectral studies indicate an angular
Total number of atoms in one molecule of CO2 = 1 + 2 = 3 structure for nitrite ion (NO2–) with O – N – O angle of 132º
Total number of atoms in 1.5 mole of CO2= 1.5 × 3 × NA and bond length (N – O) of 1.13 Å.
= 4.5 × NA 53. (c)

3 (NH4)2Cr2O7 Cr2O3(s) + N2 + H2O (g)


3.0 g of H2 = ´ 22400 mL of H2 at NTP = 33600 ml at NTP
2 (A) (B) (C) (D)
Orange solid Green solid
3 Mg CoCl2
Number of mole of H2 in 3.0 g H2 = or 1.5 moles Conc.
2
HCl
Number of atoms in 1 molecule of H2 = 2 Mg3N2
Number of atoms in 1.5 mole H2 = 2 × 1.5 × NA = 3.0 × NA CrCl3 (I)
(F) CoCl2.6H2O
Volume of 1.5 moles of ozone at NTP = 22400 × 1.5 ml Green sol. Pink
= 33600 ml H2O (E)
Number of atoms in one molecule of O3 = 3 H2O2 /OH
Total number of atoms in 1.5 mole of O3= 3 × 1.5 × NA NH3
= 4.5 × NA 2– (J)
Weight of 1 mole of O3 = 48 g CrO4
(G) NiCl2
Weight of 1.5 mole of O3 = 48 × 1.5 g = 72 g Yellow sol. sol.
Weight of 1 mole of oxygen (O2) = 32g
[Ni(NH3)6] 2+
52. A-p, s; B-p, q, r, s; C-p, s; D-p, s
Et2 O H2O2/H + Deep blue
A ® H2S decolourises acidified solution of KMnO4 (K)
2KMnO 4 + 3H 2 SO 4 + 5H 2 S ¾¾
® K 2 SO 4 + 2MnSO 4 + 8H 2 O + 5S
The shape of the H2S molecule is similar to that of the water CrO5.OEt2
molecule i.e. V-shaped structure with bond length (H–S) (H)
Blue
1.35Å and bond angle (H–S–H) 92.5º involving sp3
hybridization of S atom. Mn 2 + + NH3 + 2H 2O ¾¾
® Mn(OH) 2 ¯ +2NH 4+
B ® SO2 undergoes disproportionation (oxidation of two
molecules of the same compound at the expense of the third
H2O2
which is reduced) reaction.
Heat
3SO 2 ¾¾¾
® 2SO 3 + S MnO.(OH) 2 ¯ or MnO 2
Decolourises KMnO4 Brownish black
5SO2 + 2KMnO4 + 2H2 O ¾¾
® K 2SO4 + 2MnSO4 + 2H2SO4 54. (b) Ni2+ in [Ni(NH3)6]2+
Sulphur dioxide acts as bleaching agent in the following way: 3d
In the presence of water, it is oxidised with the liberation of
nascent hydrogen which reduces the colouring matter to
colourless.
4s 4p 4d
SO 2 + 2H 2 O ¾¾ ® H 2SO 4 + 2[H]
ˆˆˆ†
Colouring matter + 2[H] ‡ˆˆˆ Colourless compound
air
Sulphur dioxide has a planar triangular (V-shaped) structure Hyb. sp3d2
involving sp2 hybridisation of the sulphur atom (sp2 µeff = 2.8 B.M.
hybridisation occurs between 3s23px1 and 3py1 electrons).
SOLUTIONS - MOCK TESTS 71
55. (d) levels of polarizing agent and the anion to be polarized,
O eg. BiI 3 is black.
O O
2– + 57. (d) [Cr(H2O)6]Cl3 and [Mn(H2O)6]Cl3 are coloured due
CrO4 + 2H2O2 + 2H Cr + 3H2O
to d-d transition. K3[Cu(CN)4] colourless. Due to
O O charge transfer KMnO4 is coloured.
deep blue
(unstable in the absence 58. (a) During charge transfer the atom which transfers
of organic solvent) electron is oxidized and the atom which gains the
® CrO5 is stabilized in presence of ether, Amyl alcohol, electron is reduced ; therefore oxidation state of atoms
amylacetate. change.
56. (d) If MCl2 is coloured due to polarization, then MI2 will 59. (c) Victor Meyer test is a identification test.
either be coloured or black because higher the 60. (d) Nitrobenzene is a deactivated ring & deactivated rings
polarization less will be energy gap between energy do not give F.C. reaction.

ANSWER KEY - PAPER 2


1 (a) 11 5 21 (b) 31 6 41 (a) 51 6
2 (c) 12 1 22 (c) 32 7 42 (b) 52 2
3 (b) 13 1 23 (c) 33 9 43 (b) 53 7
4 (b) 14 5 24 (b) 34 9 44 (b, d) 54 (b)
5 (b) 15 0 25 (a, b) 35 (b) 45 (a, c) 55 (c)
6 (a,b,c) 16 (c) 26 (a, b, c) 36 (a) 46 (b, c) 56 (d)
7 (a, c) 17 (a) 27 (a, b, c, d) 37 (c) 47 A -p; B-p,q; C-p; D-p,r,s 57 (d)
8 (a, b) 18 (a) 28 A -q,r,s; B-q,r,s ; C-p; D-q,r,s 38 (c) 48 A -p,q,r,s ; B-p,r,s; C-q; D- s
9 A-q; B-s; C-r,q; D-p 19 (a) 29 A -q,r; B-q,r,s ; C-p,q; D-r 39 (d) 49 2
10 A-q; B-s; C-r; D-p,q 20 (d) 30 4 40 (b) 50 3

PAPER - 2 Now, | a - b | = (6 6)k


1
dy k = log6 10 – log6 5 + log ((log6 (18.72))
1. (a) - y = 1 - e- x , I.F. = e–x 2 6
dx
1
-x -2x = log6 2 + log (log6 1296)
\ y . e–x = ò (e - e ) dx 2 6
1 1
y.e - x = - e - x + e-2x + C = log6 2 + log 4 = 2 log6 2 = log6 4
2 6
2
If x = 0, y = y0 \ | a – b | = (6 6)log 6 4 = ((6)3/ 2 ) 2 log6 2 = 6log6 8 = 8
1 1 Hence, (a – b)2 = 64
y 0 = -1 + + C Þ C = y0 + (a + b)2 – 4 ab = 64
2 2
576 – 4C = 64 Þ 4C = 512 Þ C = 128
-x -x 1 -2x 1 3. (b) f (x) is increasing.
\ y.e = - e + e + y0 +
2 2 é x +1 ù
x +1 x +1êln
If x ® ¥ , then y0 = –1/2 æ 1ö ë x úû
g(x) = ç1 + ÷ =e
2. (c) A = ((5 + 2 6) 2 )1/4 = (5 + 2 6)1/2 = [( 3 + 2)2 ]1/2 è xø
x +1
8 6 16 æ 1ö é æ 1 1ö æ x + 1ö ù
Hence, A = 3 + 2 , B = 8 3 + + + ........¥ , g ¢(x) = ç1 + ÷ ê (x + 1) çè x + 1 - x ÷ø + ln çè x ÷ø ú
3 3 è xø ë û

8 3 (8 3) 3 x +1
8 6 1 2 æ 1ö é æ x + 1ö x +1 ù
r= . = , \ B= = = çè 1 + ÷ø êln çè x ÷ø + 1 - x ú
3 8 3 3 1 - ( 2 / 3) 3- 2 x ë û
B = 24 ( 3 + 2) x +1
Hence, quadratic equation is Let = u Î (1, ¥) as x > 0
x
24 ( 3 + 2) Consider h (u) = ln u + 1 – u
( 3 + 2) ( 3 - 2) x 2 + x+C =0
( 3 + 2) 1
h'(u) = – 1 < 0 in (1, ¥)
x2 + 24x + C = 0 ........ (1) u
EBD_7801
72 Target IIT
Hence, g (u) is a decreasing function in (1, ¥) r ˆ -1) Þ nr = ˆi + 2jˆ - kˆ
n 2 = iˆ (0 + 1) - ˆj(-1 - 1) + k( 2
h (u) < h (1)
h (u) < 0 n2 = AB × V
\ g ' (x) < 0 Þ g is decreasing
\ f is increasing and g is decreasing
4. (b) m = 5 . 5. 8. 7 = 1400
^ ^ ^
V=i –j–k
^
5 5 8 7 B(j)
n = 1400 – (5 . 8. 7) = 1400 – 280 = 1120 P2
0 r
Hence equation of P2 is (r - ˆj) . (iˆ + 2ˆj - k)
ˆ =0
1400 5 r ˆ ˆ ˆ
Þ k= = r . (i + 2j - k) = 2 Þ (b) is not correct.
1120 4
5. (b) If A is an involutory and diagonal matrix, then A2 = I. If q is the acute angle between P1 and P2 then

éa 0 0ù r r
ê ú n .n (iˆ + ˆj).(iˆ + 2ˆj - k)
ˆ 3 3
Let A = ê0 a 0ú then A2 = I Þ a2 = 1 Þ a = –1 cos q = r 1 2r = = =
| n1 | . | n 2 | 2. 6 2. 6 2
ëê0 0 a úû
(Q a is non-positive) p
q = cot -1 3 = Þ (c) is correct
Thus | A | = – 1, A–1 = A, | 2A | = – 8 6
and | 3 adj 2A | = 27 | adj (2A) | = 27 (– 8)2 = 1728 As L is contained in P2 Þ q = 0
6. (a,b,c) 8. (a, b)
x x
x-t
1
f (x) = ò e t sin (x - t) dt = ò e sin (t) dt Given | z1 | = | z2 | = | z3 | = 1 Þ z1 = z
1
0 0

x z12 z 22 z 32
-t Also, + + +1 = 0
òe
x
f (x) = e sin t dt z 2 z3 z1z 3 z1z 2
0
Þ (z1)3 + (z2)3 + (z3)3 + z1z2z3 = 0
æx ö Þ (z1)3 + (z2)3 + (z3)3 – 3z1z2z3 = – 4z1z2z3
f ' (x) = ex. e–x sin x + ç ò e - t sin t dt ÷ e x (z1 + z2 + z3) [(z1)2 + (z2)2 + (z3)2 – S z1z2] = – 4z1z2z3
è0 ø S z1 [ (S z1)2 – 3 S z1 z2] = – 4z1z2z3
f ' (x) = sin x + f (x) .......... (1) Let z1 + z2 + z3 = z
f '' (x) = cos x + f ' (x) = cos x + sin x + f (x) z [z2 – 3 S z1 z2] = – 4z1z2z3
[Using (1)] z3 = 3z S z1 z2 – 4z1z2z3
f '' (x) – f (x) = sin x + cos x .......... (2)
é æ1 1 1ö ù
g (x) = sin x + cos x Þ g(x) Î[- 2, 2] z3 = z1z 2 z3 ê3z ç + + ÷ - 4ú
7. (a, c) ë è z1 z 2 z3 ø û
r r r
(r - a).n1 = 0 = z1z 2 z3 [3z (z1 + z2 + z3 ) - 4]
rr rr z3 = z 2
r.n1 = a.n1 1 z2 z3 =[3 | z | – 4] ........... (1)
2
(xiˆ + yjˆ + zk).(i
ˆ ˆ + ˆj) = 2 Now, if | z | ³
3
^ ^
n1 = i + j then | z |3 = 3 | z |2 – 4 Þ | z |3 – 3 | z |2 + 4 = 0
2
| z | ( | z | – 2) – | z | ( | z | – 2) – 2 ( | z | – 2) = 0
P1 L ( | z | – 2) ( | z |2 – | z | – 2) = 0
A ( | z | – 2) ( | z | – 2) ( | z | + 1) = 0 Þ| z | = 2
r=(1,1,1)+l (1,–1,–1) or | z | = – 1 (rejected)
(1,1,1)
2
x + y = 2 Þ (a) is correct Now, if 0 < | z | < then equation (1) becomes
uuur 3
Now, AB = ˆi + kˆ | z |3 = 4 – 3 | z |2 Þ | z |3 + 3 | z |2 – 4 = 0
ˆi ˆj | z |2 ( | z | – 1) + 4 | z | ( | z | – 1) + 4 ( | z | – 1) = 0

uuur r r ( | z | – 1) ( | z |2 + 4 | z | + 4) = 0
Now AB ´ V = n 2 = 1 0 1 ( | z | – 1) ( | z | + 2)2 = 0 Þ| z | = +1 or | z | = – 2 (rejected)
1 -1 -1 Hence, | z | = {1, 2} where | z | = z1 + z2 + z3
SOLUTIONS - MOCK TESTS 73
9. A - q; B - s; C - r, q; D - p p
(A) Let the medians BE and CF meet at P. Let EP = y Þ either cos A = 0 i.e., A =
Þ PB = 2y 2
Let FP = x Þ PC = 2x p p p
or sin A sin 2B sin 2C = 1 Þ A = , B = ,C =
x 2x 2 4 4
Now, tan ÐFBP = and tan ÐCPB = 10. A - q; B - s; C - r; D - p, q
2y 2y
(A) The given equation can be written as
Also B = ÐFBP + ÐCPB 1 – 2 sin2 x + a sin x = 2a – 7
x x Þ 2 sin2 x – a sin x + 2a – 8 = 0
+
2y y 3xy
Hence tan B = = a ± a 2 - 8 (2a - 8) a ± (a - 8)
x 2 2
2y – x 2 Þ sin x = =
1– 4 4
2
2y
a-4
Þ sin x = which is possible if
2
2 y 2 – x2
Þ cot B = a-4
3xy -1 £ £ 1 or 2 £ a £ 6.
2
A
(B) 7 cos x + 5sin x = 2k + 1 Þ sin( x + a) =
2k + 1 ,
74

F E 7 5
where sin a = and cos a = .
x P y 74 74
2y 2x 2k + 1 - 74 - 1 74 - 1
So, -1 £ £1 Þ £k£
B C 74 2 2
2
2x – y 2 (C) Let t = cos x Þ t Î [0, 1] and equation is
2
Similarly, cot C = so that t2 + at + 1 = 0.
3xy
This quadratic has atleast one root in [0, 1] if
x 2 + y 2 2 xy 2 a2 - 4 ³ 0 and 1 + a + 1 £ 0 Þ a £ -2
cot B + cot C = ³ =
3xy 3xy 3 (D) The equation is |sin 2x| = |x| + a. In the limiting situation
(B) Since r1 + r2 + r3 = r + 4R, the lines y = |x| + a must be tangents to
y = |sin 2x|.
13 At the point of contact P,
r2 + r3 = 4 ´ + 2 – 3 = 25
2
d p
(sin 2 x) = 1 Þ 2 cos 2 x = 1 Þ x = .
Also, 1 = 1 + 1 + 1 dx 6
r r1 r2 r3
æ p 3ö
r +r So, P is ç , ÷.
Þ 1 = 2 + 2 3 Þ r2r3 = 150 Þ r2 = 10, r3 = 15 è6 2 ø
2 3 r2 r3

r1r2 r3 3 ´ 15 ´ 10 3 3 -p
The point also lies on y = x + a Þ a =
Moreover, s2 = = = 225 Þ s = 15. 6
r 2 y
Hence, D = rs = 30 sq. units y = |x| + a
(C) The given equation can be written as
(a + b – c) (b – c)2 + (b + c – a) (c – a)2 P y = |sin 2x|
+ (c + a – b) (a – b)2 = 0
Since all the terms are non negative,
(a + b + c) (b – c)2 = 0, (b + c – a) (c – a)2 = 0,
p 0 p
(c + a – b) (a – b)2 = 0 2 2
Þ b = c = a Þ the triangle is equilateral.
(D) We have \ The lines neither cut nor touch the curve if
0 = sin 4 A + sin 4B + sin 4C + 8 cos A
= – 4 sin 2A sin 2B sin 2C + 8 cos A 3 3-p
a> .
= 8 [– sin A sin 2B sin 2C + 1] cos A 6
EBD_7801
74 Target IIT
11. Ans : 5
Þ 3 ( 2 - b ) = 12b 2 - 3b 4 (as b ¹ 0 )
2

The tangent at any point A(2sec q, tan q) is given by


x sec q y tan q On squaring, 9 éë b 2 - 4b 2 + 4 ùû = 12b 2 - 3b 4
- = 1.
2 1 Þ 12b 4 - 48b 2 + 36 = 0
It meets the line x - 2 y = 0 or b 4 - 4b 2 + 3 = 0
x sec q x tan q 2 3 9
Þ - =1 Þ x=
2 2 sec q - tan q ( 2
)( )
Þ b - 3 b - 1 = 0 Þ b 2 = 3 or 1 Þ a =
2 2
or
4 4
æ 2 1 ö as b < a , we consider b 2 = 1 (only)
Þ Q ºç , ÷ ...(1)
è sec q - tan q sec q - tan q ø
d 2A
Also, the tangent meets the line x + 2 y = 0 at R, so < 0 for b = 1 ( b = +1 only as b > 0, P Î Q1 )
db2
x x 2 (Verify yourself)
Þ sec q + tan q = 1 Þ x =
2 2 sec q + tan q \ A is maximum for b = 1
æ 2 -1 ö 13. Ans: 1
Þ Rºç , ÷ ...(2)
è sec q + tan q sec q + tan q ø é
( ) ( ) ( )
2 3 kù
ê e1/ k e1/ k e1/ k e1/ k ú
2 2 + 12 2 2 + 12 lim ê + 2. + 3. + .... +
Now, CQ.CR = k ®¥ k2
ê k2 k2 k úú
(sec q - tan q) 2 (sec q + tan q) 2 ë û

= 22 + 12 1 é e1/ k 2 2 / k 3 3 / k k ù
Þ C Q . C R =5 lim ê + e + e + .... + e k / k ú
k ® ¥ k ëê k k k k ûú
12. Ans : 1
Let P º ( a , b ) ; Q º ( a , a ) ; and R º ( b , b ) 1 k r r/k
As b < a \ P lies below y = x in the first quadrant.
= lim å e
k ® ¥ k r =1 k

Let A º ( a , 0) , B º ( b , 0) 1
Let kh = 1, h = , k ® ¥ , so h ® 0
Now we have to find P º ( a , b ) such that A(OAPR) is k
maximum. k 1
1
lim h å ( rh ) erh = ò x e x dx = é xe x - e x ù
Further, P ( a , b ) lies on 4 x + 3 y = 12
2 2
h ® ¥ r =1 0 ë û0

\ 4a + 3b = 12 ….(1)
2 2
= e - ( e - 1) = 1.
1 14. Ans : 5
A(OAPR) = A = OB . RB + BA . PA
A
2 Let coordinates be P (t12 , 2t1 ) and Q (t22 , 2t2 ).
1 1 Coordinates of points of intersection R is (t1t2 , (t1 + t2 ))
\A = b ( b ) + ( a - b ) .b = b 2 + ab - b 2
2 2
R lies on y 2 = - x, then(t1 + t2 )2 = -t1t2 ...(i)
b 2
b 12 - 3b b
2 2
Coordinates of the point of intersection of normal at P and
= ab - = - (Using (1))
2 2 2
Q are S (2 + (t12 + t22 + t1t2 ), -t1t2 (t1 + t2 ))
1 From equation (i), coordinates of S are
= é 12b 2 - 3b 4 - b 2 ù
2ë û
(2 - 2t1t2 , -t1t2 (t1 + t2 ))
dA 1 é 1 ù P, Q, R, S are concyclic points. Hence centre of circle is
\ db = 2 ê
êë 2 12b 2 - 3b 4
(
. 24b - 12b 3 - 2b ú
úû
) the point of intersection of PQ and RS .

æ t12 + t22 2(t1 + t2 ) ö


b é 12 ( 2 - b ) ù
2

= ê - 2ú Co-ordinates of mid-point of PQ çç 2 , 2
÷
÷
2 ê 2 12b 2 - 3b 4 úû è ø
ë
and the co-ordinates of mid-point of RS
dA æ t1t2 + 2 - 2t1t2 (t1 + t 2 ) - t1t2 (t1 + t2 ) ö
For maximum or minimum =0 ,
db çè 2 2 ÷ø
SOLUTIONS - MOCK TESTS 75
Þ sin x = cos x = 1 (not possible)
(t12 + t22 ) 2 - t1t2 2(t1 + t2 ) (t1 + t2 )[1 - t1t2 ]
So, = and = Statement-1 is correct and statement-2 is correct
2 2 2 2 explanation to statement-1.
Þ -3t1t2 = 2 - t1t2 and (t1 + t2 )[1 - t1t2 - 2] = 0 19. (a) Let q be the angle between the line joining the centres
and a common tangent, then slope of the tangents are
Þ t1t2 = -1 ...(ii) tan (45° + q) and tan (45° – q)
Þ From equation (i), t1 + t 2 = ±1 ...(iii) Further tan (45° + q) tan (45° – q) = 1
\ Statement-1 is true.
Hence coordinates of centre are æç , ±1ö÷ and
3 Statement-2 says slope of the line joining the centre
è2 ø
2a - 0
coordinates of R are (-1, ±1). = =1
2a - 0
2 \ Statement-2 is true.
æ3 ö 2 5 20. (d) In case-I no horizontal external impulse on ball and
The radius of required circle is ç + 1÷ + (±1 m 1) = .
è2 ø 2 mass system while in case-II friction is impulsive as
normal with ground is impulsive.
P \ Both are wrong.
R 21. (b) Initial velocity v1 must be greater than v0 if it has to
reach same height and finally while coming back it will
approach a terminal velocity.
A1 + A 2 A x -1
S 22. (c) =x ; 2 =
A1 - A 2 A1 x + 1
Q 2
æ x - 1ö
Energy µ A2 Þ ç
è x + 1÷ø
15. Ans. : 0
b -c c - a a -b
Given expression, + +
r1 r2 r3
23. (c) O
s-a ( s - b) ( s - c)
= (b - c) + ( c - a) + ( a - b)
S S S
s 1 “O” acts as focal point.
=
S
[b - c + c - a + a - b] - S éëa ( b - c ) + b ( c - a ) + c ( a - b) ùû
1 æ 1 1 ö 1 æ 1 1ö
=0 = (m - 1) ç - = (m - 1) ç + ÷
è R1 R 2 ÷ø
f ; è 10 10 ø
40
2x + 2
16. (c) f (x) = ln (2 + x) – is continuous in (–2, ¥)
x+3 1 æ 2ö 9
= (m - 1) ç ÷ Þ m =
1 4 2
(x + 3) - 4(x + 2) 40 è 10 ø 8
f ¢(x) = - =
x + 2 (x + 3) 2
(x + 2) (x + 3)2 XL XC
24. (b) tan q1 = ; tan q2 =
R R
x 2 + 2x + 1 (x + 1)2
= 2
= >0 XC - XL
(x + 2) (x + 3) (x + 2) (x + 3) 2 tan q =
R
(f ' (x) = 0 at x = – 1)
Þ f is increasing in (–2, ¥) tan q = tan q2 – tan q1
25. (a, b)
Also, lim f (x) ® -¥ and lim f (x) ® ¥
x ®¥
x ® 2+
Þ unique root
é 1 -3 -4ù é 1 -3 -4 ù é0 0 0 ù
17. (a) A 2 = ê -1 3 4 ú ê -1 3 4 ú = ê0 0 0 ú
ê úê ú ê ú
êë 1 -3 -4úû êë 1 -3 -4 úû êë0 0 0 úû
A B
Þ A is nilpotent i
sin x + cos x + 1 + 1
18. (a) ³ (sin x cos x)1/4 G
4 C D
(sin x + cos x + 2)4 ³ 128 sin 2x.
i
EBD_7801
76 Target IIT
For null point current flows in the loop CD only. B - q,r,s ; A.C current produced in generators related to
change in flux, Ohm’s law. Its direction is determined by
3V
i= = 1A Faraday’s right hand rule.
2W + 1W C - p,q ; Electromagnetic brake works on the principle of
VCD = 1V – 1 (1) = 0 \ option (A) is correct electromagnetic induction, so it is related to Lenz’s law &
VA > VB change in flux.
When Jockey touches B current flows from A to B to increase D - r ; Heat produced in a wire due to A.C will relate only to
the P.D. across the secondary circuit. Ohm’s law.
\ option (b) is correct. 30. Ans : 4
26. (a, b, c) If v is the velocity of the rod at any time t, induced emf = BvL
m1 and so induced current in the rod
a2
(a) I = (m 2 + m3 ) Induced e.m.f. BvL
4 I= =
R R
H 3H Due to the this current, the rod in the field B will experience
(b) H = Iw, w = = a/2 a/2
2
(Md / 3) Md 2 a force
m2 m3
B 2 L2 v
m x + m2 x 2 m y + m2 y 2 F = BIL = (opposite to its motion) … (1)
(c) x CM = 1 1 ; yCM = 1 1 R
m1 + m 2 m1 + m 2
So equation of motion of rod will be,
m1z1 + m 2 z 2 T – F = O × a i.e., T = F [as rod is massless]
z CM = T
m1 + m 2 mg – T = ma i.e.a = g -
m
(d) According to Newton’s third law they exert equal and
opposite forces on each other in both (i) and (ii). F B 2 L2 v
27. (a, b, c, d) or a=g- = g- … (2)
m mR
Wiaf + Uif = Hiaf So rod will acquire terminal velocity when its acceleration is
20 + Uij = 50 zero i.e.,
Uif = 30 and Wibf + Wif = Hibf
B 2 L2 vT mgR
Wibf + 30 = 36 g- = 0 i.e. vT = 2 2 ;
mR B L
Wipf = 6
(a) Wibf = 6 vT
For the case when velocity is
(b) Hfi = Wfi + Ufi = 13 + 30 = 43 2
(c) Uif = 30, Ui = 10, Uf = 40 vT mgR
(d) Hib = Wib + Uib = 6 + (22 – 10) = 18 v= = 2 2
2 2B L
28. A - q, r, s; B - q, r, s; C - p; D - q, r, s
Substituting this value of velocity in eq. (2) we get
A - q,r,s; orbital velocity of satellite
B2 L2 1 mgR 1 g 10
4
G ´ pR 3r a =g- ´ 2 2
= g- g = = = 4ms –2
GM 3 mR 2 B L 2 2 2
v0 = gR = =
R R 31. Ans : 6
So v0 depends upon G, R & r. Given m1 = 20 kg, m2 = 5 kg, M = 50 kg, µ = 0.3 and
B - q,r,s; Escape velocity g = 10 m/s2
(a) Free body diagram of mass M is :
4
GM 2G pR 3r T N1
= 2gR = 2 3
R = R T f1
T
So ve µ G, r and R Mg F
C - p ; (Time period of satellite)2 µ (radius of orbit)3 T
4 N
G ´ pR 3r
D - q,r,s; Value of g = GM 3
= (b) The maximum value of f1 is
R R (f1)max = (0.3) (20) (10) = 60 N
or g depends upon G, R and r The maximum value of f2 is
29. A - q, r; B - q, r, s; C - p, q; D - r (f2)max = (0.3) (5) (10) = 15 N
A - q, r ; Transformer core gets heated up due to induced Forces on m1 and m2 in horizontal direction are as
current so it will be related to change in flux & Ohm’s law. follows :
SOLUTIONS - MOCK TESTS 77
34. Ans : 9
T m1 f1 f2 m2 T Potential difference between A and B = 6 volts. The
condensers 2 mF and 5 mF are in parallel. Their effective
Now, there are only two possibilities. capacitance, C = 2 + 5 = 7 mF.
(1) either both m1 and m2 will remain stationary (w.r.t. ground) The capacitance between A an d B is given by
or (2) both m1 and m2 will move (w.r.t. ground). First case is
possible when C ´ 3 7 ´ 3 21
C' = = = mF
T < (f1)max or T < 60 N C + 3 7 + 3 10
and T < (f2)max or T < 15 N
21 63
These conditions will be satisfied when T < 15 N say T = 14 Total charge Q = CV = ´ 6 = mC
then f1 = f2 = 14 N. 10 5
Therefore the condition f1 = 2f2 will not be satisfied. Thus Total potential difference across 3 mF is
m1 and m2 both can't remain stationary. Q 63 1 21
In the second case, when m1 and m2 both move V1 = = ´ = volts
3 5 3 5
f2 = (f2)max = 15 N
Hence the common potential difference across the
Therefore, f1 = 2f2 = 30 N
condensers in parallel is
\ f1 – f2 = 15 N
21 9
2 V2 = 6 - = V
Þ (f1 - f 2 ) = 6N 5 5
5
So, the charge on 5 µF condenser is
32. Ans : 7
Let the frequency of the tuning fork be n Hz 9
Q = 5 ´ V2 = 5 ´ = 9μC
Then frequency of air column at 15ºC = n + 4 5
Frequency of air column at 10ºC = n + 3
According to v = nl, we have 35. (b) M = K L1L 2 , K = 1 if rings are coplanar..
v15 = (n + 4)l and v10 = (n + 3)l 36. (a) Work done by action reaction force may be zero only if
disp. of both bodies are same.
v15 n+4 37. (c) As net charge of dipole is zero. So, statement-1 is true.
\ =
v10 n+3 Gauss law is valid for all charge distributions, so
The speed of sound is directly proportional to the square- statement-2 is false.
root of the absolute temperature. 38. (c) i = i 0 (1 - e - t / t ) at t = 0, i = 0
v15 15 + 273 288
\ = = di i 0 L - t / t
v10 10 + 273 283 e = -L = e at t = 0, e ¹ 0
dt t
1/ 2
n+4 288 æ 5 ö
\ = = ç1 + ÷ H
n+3 283 è 283ø
N H
1 5 5 39. (d) (a) H x N ; x = 1.46 Å
Þ 1+ = 1 + 1/2 × =1+ sp3
n+3 283 566 H
1 5 y y +
Þ = Þ n + 3 = 113 Þ n = 110 Hz (b) N N O N=N=O
n + 3 566
n HO
\ -4 =7 N
10
z
33. Ans : 9 (c)
N
Heat flow per second OH
K1 K2 K3 O
O
QA = 20°C Wood Cement Brick q B (= -10°C)
N v N sp²
d1 d2 d3 (d)
O O
v=1.64 Å
(q A - q B ) A [20 - ( -10)] ´ 137
H= = = 9000 W v>x>z>y
d1 d 2 d é 2.5 1 25 ù -2 N2H4 exists in gauche conformation hence repulsion
+ + 3 + +
êë 0.125 1.5 1 úû ´ 10
k1 k 2 k 3 between l.p. and l.p. is reduced while in N2O4 all atoms
= 9kW being is same plane there is repulsion between two
This rate of heat has to be supplied by the heater to keep bonds having B.O. = 1.5 each, hence N–N bond length
the temperature constant. in N2O4 is greater than N – N bond length in N2H4.
EBD_7801
78 Target IIT
43. (b) DTf = Kf.m.i
H 0.2406 = 1.86 × 0.1 (1 + a) ; a = 0.1
O HCOOH(aq) ƒ H+(aq) + OH– (aq)
O N H
N N H Ca 2 0.1 ´ (0.1) 2 1
N Ka = = = ´ 10 -2
O O (1 - a ) 0.9 9
H
Kw 10-14
40. (b) Kh = = = 9 ´ 10-12
Ka 1
´ 10-2
– 9
OH
H 44. (b, d)
NaH 2 PO 4 + NaOH ¾¾
® Na 3 PO 4 + H 2 O
CH3 – CH – CH – CH3 CH3 – CH = CH – CH3 (Cis/trans)
Trans is major NaHCO 3 + NaOH ¾¾ ® Na 2 CO 3 + H 2 O
Br 45. (a, c)
Br2/CCl4
s – C3H5 can exist in two structures :

CH3 (i) H 2C = CH – CH 2
H Antiaddition at
H3 C H Br trans obtained
Br Br
H
H Br erythreo product C
H
CH3 CH3
(ii) H2C H2C
k1¢ E1 é 1 1 ù 46. (b, c)
41. (a) ln = ê - ú ........ (1) When base is (1/5)th neutralized
k1 R ë T1 T2 û
RNH2 + HCl ® RNH3Cl
k 2¢ E 2 é 1 1ù a x
ln = ê - ú
k2 R ë T1 T2 û ........ (2) 4
a a

Solving we get eq. (2) – eq. (1) 5 5

k 2¢ k ¢ æ E - E1 ö é 1 1ù æ a /5 ö
- ln 1 = ç 2 pOH = pKb + log çè ÷
ln
è ÷ø ê - ú 4a / 5 ø
k2 k1 R ë T1 T2 û
5 = pKb + log (1/4) [ Q pDH = 14 – pH = 14 – 9 = 5]
k1 2k '1 pKb = 5.6
Q for 2 : 1 molar ratio =
k 2 k '2 1
pH = [pKw – pKb – log C]
æ k ¢ k ö æ 8314 ö (T2 - 300) 2
ln ç 2 ´ 1 ÷ = ç ÷ 4.5 × 2 = (14 – 5.6 – log C)
è k1¢ k 2 ø è 8.314 ø 300 ´ T2 log C = – 0.6
C = 0.25
æ T2 - 300 ö 0.7 ´ 3 æ 300 ö Let V ml. of HCl is used
ln 2 = (1000) ç 300 ´ T ÷ ; 10 = ç1 - T ÷
è 2ø è 2 ø 0.5 ´ V
= 0.25
300 æ 0.7 ´ 3ö V + 100
= ç1 - ÷ V = 100 ml.
T2 è 10 ø
M. Moles of acid = M. Moles of base = 0.5 × 100 = 50
300 ´ 10
T2 = = 379.746 K = 379.75 K 50 ´ 10-3 ´ 45
(10 - 0.7 ´ 3) Wt. % of base in sample = ´ 100 = 90
2.5
47. A-p; B-p, q; C-p; D-p, r, s
+ – CH2 – I (A) In pyridine the lone pair of electrons on N is present
42. (b)
NaH – in sp2 orbital, hence not involved in aromatic sextet,
CH3 – C C – H CH3 – C C
while p electrons are involved in sextet and
(X) conjugation.
O (B) The lp is conjugated with p bond. Further, the p
bond of C = C is in conjugation with only one p
CH2 – C C – CH3 O CH2 – C – OH bond of C º C because its only one p bond is in the
3
O same plane of the p bond of C = C while the other
H2O2
p bond of C º C is not in the same plane as that of
+ CH3 – C – OH
the p bond of C = C.
SOLUTIONS - MOCK TESTS 79
(C) When an atom has more than one lp, its only one 52. Ans : 2
(not all) lp is in conjugation with the p bond.
(D) CH3CH = CH – CH = CH2 has three hyperconjugative 200
Moles of the acid neutralized = ´ 0.1 = 0.02
hydrogens, so the compound will also show 1000
hyperconjugation (p, s bond resonance). Moles of acid remaining undissociated
48. A-p, q, r, s; B-p, r, s; C-q; D- s
All the given particles (i.e. cosmic rays, g-rays, a-particles, æ 25 ö
= 0.02ç1 - ÷ = 0.015
protons, neutrons and deutrons) have been successfully è 100 ø
used as projectiles for nuclear disintegration.
Atomic nuclei can also be disintegrated by bombarding them Heat evolved during neutralization for 100% ionization =
with high energy photons. This process is usually known 13700 × 0.02 = 274 cal
as photo disintegration. Heat used up for dissociation of 0.015 mole
49. Ans : 2 = 274 – 244 = 30 cal
Let x g be the mass of element in 51.0 g of saturated Heat of dissociation per mole
solution.
Mass of benzene in 51.0 g of saturated solution 30
= = 2000 cal = 2.0 k cal
= 51.0 – x g 0.015
Total mass of benzene containing x g of solute
53. Ans : 7
= 50 + 51 – x = (101 – x) g
The seven possible cyclic structural and stereoisomers are
1000 K f WB 1000 ´ 5.5 ´ x
DT f = = = 0.55 ( given) CH3
M BWA 4 ´ 25 ´ (101 – x )
Þ x = 1.0g
W B ´100 1
Hence, solubility = = ´ 100 = 2.0 g
WA (51 – 1) C2H5 H3C CH3
50. Ans : 3
2
K p = pH = 1.40 ´ 10 -6
2O

-2 H3C CH3 H3C H H3C H


Þ pH 2O = 1.183 ´10 atm = 760 × 1.183 × 10–3 = 0.9 torr
CuSO 4 .5H 2O will efforsce when partial pressure of H 2 O
H H H CH3 H CH3
vapour in air falls below 0.9 torr. cis, (meso) trans, enantiomers
0.9
Hence relative humidity of air < ´ 100 = 3% 54. (b) There is large d– charge developed (due to F-atom) on
30
the periphery of C2F6 molecule, hence fluoro-carbons
51. Ans : 6
have higher boiling point than corresponding
In pure iron oxide (FeO), iron and oxygen are present in the
hydrocarbons.
ratio 1 : 1.
However, here number of Fe2+ present = 0.93 55. (c) Froth floatation method is used for benefication of
or No. of Fe2+ ions missing = 0.07 sulphide ore like PbS, Cu2S etc.
Since each Fe2+ ion has 2 positive charge, the total number NaCN or KCN solution is very good depressant to
of charge due to missing (0.07) Fe2+ ions = 0.07 × 2 = 0.14 suppress the floating property of ZnS not for PbS.
To maintain electrical neutrality, 0.14 positive charge is
compensated by the presence of Fe3+ ions. Now since, (Pyrolysis of ester)
56. (d)
replacement of one Fe2+ ion by one Fe3+ ion increases one +
positive charge, 0.14 positive charge must be compensated O D major
by the presence of 0.14 Fe3+ ions.
In short, 0.93 Fe2+ ions have 0.14 Fe3+ ions O
0.14
100 Fe2+ ions have = ´ 100 = 15.05% » 15% Hoffmann alkene is major product.
0.93
57. (d) According to spectrochemical series CN– is stronger
90 field ligand, so it has more D0 value,
Given : 15 =
x so correct order of D0 for these complex ions.
[CrCl6]3– < [Cr(C2O4)3]3– < [Cr(CN)6]3–
90
x= =6
15
EBD_7801
80 Target IIT

MOCK IIT - 5
ANSW ER KEY PAPER - 1
1 (a) 15 (b) 29 (c, d) 43 (c) 57 7
2 (a) 16 (b) 30 (c,d) 44 (d) 58 5
3 (a) 17 (a) 31 A -p, s ; B-q, s ; C-q, s; D-s, t 45 (d) 59 (d)
4 (d) 18 (c) 32 A -q; B-r, s; C-s ; D-p, q, r 46 (a) 60 (d)
5 (a) 19 (d) 33 1 47 (d) 61 (b)
6 (a,d) 20 (b) 34 9 48 (b) 62 (c)
7 (a,b,d) 21 (b) 35 4 49 (b) 63 (d)
8 (a,b,c) 22 (d) 36 8 50 (a, b, c, d) 64 (b)
9 A -r; B- s; C-q; D-p 23 (a) 37 (c) 51 (a, b, c, d) 65 (c)
10 A -q; B-s; C-r,s ,t; D-p,q,r,s,t 24 (c) 38 (b) 52 (a, c) 66 (b)
11 0 25 (a) 39 (a) 53 A - p, q ; B - r, s ; C - p, q ; D - p, q
12 9 26 (b) 40 (a) 54 A -r, s ; B-s ; C-p, q; D-p, q
13 4 27 (a) 41 (a) 55 4
14 1 28 (a, b) 42 (b) 56 3

EXPLANATORY NOTES

PAPER - 1 3. (a) 9a + 3b + c = 90

c
1. (a) ak = (k2 + 1) k! = (k (k + 1) – (k – 1)) k ! 3a + b + = 30
= k (k + 1) ! – (k – 1) k ! 3
So, a1 = 1 . 2! – 0 Now using GM £ AM for 3 numbers 3a, b and c/3
a2 = 2 . 3! – 1 . 2!
a3 = 3 . 4! – 2 . 3!
c
M 1/3 3a + b +
æ cö 3
ak = k (k + 1) ! – (k – 1) k ! çè 3a.b. ÷ø £
3 3
––––––––––––––––––––
a1 + a2 + ...... + ak = k (k + 1) !
Hence bk = k (k + 1) ! 30
(abc)1/ 3 £ = 10
2 2 2 3
a k (k + 1)k! (k + 1) k + 1
\ = = =
bk k(k + 1)! k(k + 1) k 2 + k \ abc £ 1000
a100 10001 m log a + log b + log c £ 3
\ b = =
10100 n
100 Þ log a + log b + log c |max = 3
\ (n – m) = 99
f (x) é æ p pö ù næ np
n
np ö
2. (a) Consider a function g(x) = as f (x) and x are 4.
n
(d) (1 + i 3) = ê 2 çè cos + isin ÷ø ú = 2 çè cos + i sin ÷ø
x 3ë 3 3 û 3
differentiable hence g (x) is also differentiable.
f (a) f (b) np
Now g(a) = and g(b) = f ((1 + i 3)n ) = real part of z = 2 n cos
a b 3
f (a) f (b)
Since = \ g (a) = g (b) 6a 6a
a b np é np ù
Hence Rolle’s theorem is applicable for g (x) \ å log 2 2n cos
3
= å ên + log 2 cos
3 úû
n =1 n =1 ë
\ $ some x0 Î (a, b) where g ' (x) = 0
xf ¢ (x) - f (x)
But g ¢(x) = , 6a(6a + 1)
x2 = + (-1 - 1 + 0 - 1 - 1 + 0)
2 144424443
x 0 f ¢ (x 0 ) - f (x 0 ) a such term
g ¢ (x 0 ) = =0
x 20
= 3a (6a + 1) – 4a = 18a2 – a
\ x0 f ' (x0) = f (x0)
SOLUTIONS - MOCK TESTS 81
p /2 8. (a,b,c)
ì æ 1 + cos x ö æ 1 - cos x ö ü
5. (a) I= ò ex ícos (sin x) ç
î è 2 ÷ø + sin (sin x) çè 2 ÷ø þ
ý dx Here, P be (x, y, z)
0
Then,
p/ 2
1 x = r sin q.cos f , y = r sin q sin f, z = r cos q
=
2 ò e x [{cos (sin x) + sin(sin x)}
14444244443
0 f (x) Þ 1 = r sin q.cos f, 2 = r sin q sin f , 3 = r cos q ...(i)

+ cos x {cos (sin x) - sin(sin x)}]dx Þ 12 + 2 2 + 3 2


1444442444443
f ¢ (x)
= r 2 sin 2 q cos 2 f + r 2 sin 2 q sin 2 f + r 2 cos 2 q
p/2 p/2
1 1 x ù
= 2 ò e x [ f (x) + f ¢ (x) ] dx = e f (x) ú z
2 û0
0 P (1, 2, 3)
1é x p/2 q r
= e {cos (sin x) + sin (sin x)}ù
2 ë û 0 O y
1 p/ 2 f
= [e (cos 1 + sin 1) - 1]
2
x M
6. (a,d)
The eccentricities of a hyperbola and its conjugate e1 and
= r 2 sin 2 q (cos2 f + sin 2 f ) + r 2 cos 2 q
e2 are related by
= r 2 sin 2 q + r 2 cos2 q = r 2
1 1 e1
+ = 1 Þ e2 = Þ r = ± 14
e12 e22 e12 - 1 \ From (i), we have

e 1 2 3
\ f (e) = Þ ff (e) = e sin q cos f = , sin q sin f = , cos q =
14 14 14
e2 - 1
(neglecting –ve sign assuming acute angles)
ì e sin q sin f 2 sin q 5
ï 2 , if n is odd \ = and tan q = =
\ 14243 ( ) = í e - 1
f f f ..... f e sin q cos f 1 cos q 3
n times ï e, if n is even
î 5
Þ tan f = 2 and tan q =
3
3
ìï 2 2 if n is odd 9. A-r; B- s; C-q; D-p
\ ò fff .... f (e) de = íïî4 if n is even (A) f (x) = log (ax3 + (b + a)x2 + (b + c) x + c)
1
= log (x (ax2 + bx + c) + ax2 + bx + c)
7. (a,b,d) = log {(x + 1) (ax2 + bx + c)}
p y p As b2 – 4ac < 0, a > 0 Þ ax2 + bx + c > 0 " x Î R.
Let z = x + iy; arg(z) = Þ = tan and
6 x 6 So, x + 1 > 0 Þ x > – 1
Domain of f (x) (–1, ¥ )
1
x > 0, y > 0 Þ y = x and x > 0, y > 0 which is a Range of f (x) is clearly R.
3
(B) f (x) = ln tan–1 (x3 – 6x2 + 11x – 6) x (ex – 1)
portion of a straight line.
Þ x (ex – 1) (x3 – 6x2 + 11x – 6) > 0
Also, |z – 2 3 i | = r, represents a circle with centre at (0, Þ x (ex – 1) (x – 1) (x – 2) (x – 3) > 0
Domain of f (x) is (1, 2) È (3, ¥ ). Range of f (x) is clearly
2 3 ) and radius r. The straight line will intersect the circle
R.
if the perpendicular distance from the centre on the line < r

0 - 2 3. 3
Þ < r Þ r > 3. Therefore [r] ³ 3. 1 2 3
2 0
EBD_7801
82 Target IIT

sin 2 x - a sin x 2 cos 2 x - a cos x


x2 – 3 x + 2 ( x – 1)( x – 2) x –1 (B) lim = lim
(C) f ( x) = = = , x ¹ 2, –3 x® 0 x 3 x ®0 3x 2
x2 + x – 6 ( x + 3)( x – 2) x + 3
In order that the limit exits, 2 – a = 0 Þ a = 2.
x –1 then limit
Let y = Þ xy + 3 y = x – 1
x+3 -4 sin 2 x + a sin x -8 cos 2 x + a cos x
= lim = lim
x®0 6x x®0 6
3y + 1 which is a finite number (= – 1) if a = 2.
x (y – 1) = – (3y + 1) Þ x = Þ y ¹1
1- y
e1/ x - e - 1/ x 1 - e -2 / x
(C) Q lim+ = lim =1
1 x® 0 e1/ x + e -1/ x x® 0+ 1 + e -2 / x
Also, x ® 2, Þ y ®
5
e1/ x - e -1/ x e2 / x - 1
and lim- = lim = -1
ì1 ü x® 0 e1/ x + e - 1/ x x ® 0- e2 / x + 1
So, Range = R – í ,1ý
î5 þ ìï e1/ x - e -1/ x üï
Domain = R – {–3, 2} Clearly xlim g ( x) í 1/
x -1/ x ý can exist if and only
®0 ïî e + e ïþ
x x
(D) f ( x ) = sin 2 + cos if xlim g ( x) = 0 Þ n ³ 1 .
4 4 ®0
Clearly domain is R. ì x - 2, if x ³ 1 ì1 - x, if x ³ -1 n!
(D) f ( x) = í , g ( x) = í
x x î - x, if x < 1 î x + 3, if x < -1 r !( n - r ) !
f ( x ) = sin 2 + cos
4 4 ì1 - f ( x ), if f ( x ) ³ -1
g ( f ( x )) = í
x x î f ( x ) + 3, if f ( x) < -1
f ( x) = 1 – cos 2 + cos
4 4
ì1 - ( x - 2), if x - 2 ³ -1 and x ³ 1
ï1 - ( - x ), if - x ³ 1 and x < 1
2 ï
æ x x 1ö 1 5 æ x 1ö =í
= 1 – ç cos 2 – cos + ÷ + = – ç cos – ÷
è 4 4 4ø 4 4 è 4 2ø ï x - 2 + 3, if x - 2 < -1 and x ³ 1
ïî - x + 3, if - x < -1 and x < 1
é 5ù
Range of f (x) is ê –1, ú . ì3 - x, if x ³ 1
ë 4û Þ g ( f ( x )) = í
î1 + x, if x < 1
10. A-q; B-s; C-r,s,t; D-p,q,r,s,t
(A) Comparing the graphs of y = x and y = tan–1x, we get Clearly g ( f (x)) is continous everywhere.
11. Ans : 0
tan -1 x The given equation is
that if x > 0, x > tan -1 x > 0 Þ 0 < <1
x æ æ xöö
sin -1 (sin | x |) = sin -1 ç log1 / 2 ç ÷ ÷
y è è p øø
y=x
æ xö
p/2
–1 For this to hold good, 0 £ log1 / 2 ç ÷ £ 1
y = tan x èpø
x
O æ pö x 1
As L.H.S. Î ç 0, ÷ ; 1 ³ ³
è 2ø p 2
–p /2
p
i.e., £ x£p ...(1)
2
tan -1 x
Also, if x < 0 then x < tan -1 x < 0 Þ 0 < <1. æ p pö é pö
x Given x Î çè - , ÷ø \ | x | Î ê0, ÷ø ...(2)
2 2 ë 2
é tan -1 x ù From (1) and (2), x Îf
So, xlim ê ú=0
®0 êë x úû Hence the given equation does not possess any solution.
SOLUTIONS - MOCK TESTS 83
12. Ans : 9 13. Ans : 4
Sectorial area AOB is removed and the remaining part be uuur uuuur
We have AB = diˆ. Let AC = xiˆ + yjˆ. Then we have
folded into a cone of height h and radius r.
ArcAB ArcAB AC = d Þ x 2 + y 2 = d 2 ,
Q q = Angle = = = Arc AB uuur uuur
radius 1
AB . AC = AB ´ AC ´ cos 60°
[ DABC in xy plane is equilateral]
A A

I 1
I
diˆ.( xiˆ + yiˆ) = d .d
O q q O q I 2
h
I I r d2 d 3
B B Þ dx = Þx= and y=± d
2 2 2
circular sheet after removed part conical vessel
uuuur
Now, let AD = piˆ + q ˆj + rkˆ.
2 2
2pr = 2p - q and r + h = 1
Then AD = d Þ p 2 + q 2 + r 2 = d 2 ;
1 1
\ volume of cone V = pr 2 h = pr 2 1 - r 2 uuuur uuuur uuuur uuuur d 2
3 3 AD . AB = AD . AC =
2
p2 r 4 (1 - r 2 )
Þ V2 =
9 d2 d
Þ pd = px + qy = Þ p=
2 2
p2 (r 4 - r 6 )
Let, y = V 2 =
9

dy p2 d 2 y p2
\ = (4r 3 - 6r 5 ) and = (12r 2 - 30r 4 )
dr 9 2 9
dr

dy æ2ö
For max or min of y, =0 \ r= ç ÷
dr è3ø

d2y p2 æ .2 4ö 16 2
Then,
2
= çè 12 - 30. ÷ø = - p
dr r = 2 / 3 9 3 9 27

æ2ö
\ y is maximum and hence V is also maximum at r = ç ÷ 3
è3ø If y = d , then q = pd - px = d
2 q 2 3
1 2
\ Removed sectorial area = .(1) .q 2d 2 2
2 and r 2 = d 2 - p 2 - q 2 = Þr=± d
3 3
1 1 æ 2ö
A1 = .q = (2p - 2pr ) = p(1 - r ) = p çç1 - ÷ [from (1)] 3 d 2
2 2 è 3 ÷ø If y=- d , then q = - and r = ± d.
2 2 3 3
Also, A = p uuur uuur uuur
So, AB, AC and AD can be
2
1+ d d 3 ˆ dˆ d ˆ d 2 ˆ
A 1 3 = 3+ 6 = (i) diˆ, iˆ + j, i + j+ k
\ = = m+ n 2 2 2 2 3 3
A1 2 2
1- 1-
3 3
d d 3 ˆ dˆ d ˆ d 2 ˆ
(ii) diˆ, iˆ + j, i + j- k
\ m+n =3+6=9 2 2 2 2 3 3
EBD_7801
84 Target IIT

d
(iii) dî , î -
d 3ˆ d
j, î -
d ˆ d 2
j+ k̂
(
Thus I = - cot x ln cos x + cos 2 x )
2 2 2 2 3 3
+ cos ec 2 x - 2 - cot x - x + c

(iv) diˆ, d iˆ - d 3 ˆj , d iˆ - d ˆj - d 2 kˆ \ f ( x) = - cot x and g ( x) = cos ec 2 x - 2


2 2 2 2 3 3
Hence, 4 tetrahedra are possible. f 2 ( x ) - g 2 ( x) = cot 2 x - ( cos ec 2 x - 2) 2
14. Ans. : 1
= cot 2 x - cos ec 2 x + 2
(
I = ò cosec 2 x ln cos x + cos 2 x dx ) 15.
= –1 + 2 = 1
(b) Equation of normal in terms of slope is
m3x + (4 – y) m2 + 2 = 0
(
= - cot x.log e cos x + cos 2 x ) point P(h, k) satisfies this equation
m1
1
- ò ( - cot x), \ m3h + (4 – k) m2 + 2 = 0 m2 ..... (1)
cos x + cos 2 x m3

ì 1 -½ ü k-4
í- sin x + ( cos 2 x ) ( - sin 2 x ) .2ý dx Algebraic sum of slopes is m1 + m2 + m3 =
î 2 þ h
16. (b) If two normals are perpendicular then
(
= - cot x ln cos x + cos 2 x ) 2
m1m2 = – 1 and m1m2m3 = -
h
sin x cos 2 x + sin 2 x
- ò cot x dx 2
(
cos 2 x cos x + cos 2 x ) Substituting m3 =
h
= m in (1), we get

4(4 - k)
( )
8
= - cot x ln cos x + cos 2 x 2
+ +2=0
h h2
4 + 2 (4 – k) + h2 = 0
2
cos x cos 2 x - cos x cos 2 x x2 = 2 (y – 12)
-ò dx
cos 2 x sin 2 x \ latus rectum = 2
17. (a) If the slopes are complementary then m1m2 = 1
(
= - cot x ln cos x + cos 2 x ) 2
\ m3 = –
h
cos x
-ò dx + ò cot 2 xdx 2
cos 2 x sin 2 x Using m3 = – in (1) we get
h

cos xdx 8 4
Now, I1 =
ò cos 2 x sin x 2
-
h 2
+ (4 - k)
h2
+2=0

Þ – 4 + 2 (4 – k) + h 2 = 0 Þ h2 + 4 – 2k = 0
cos x dx dt \ x2 = 2 (y – 2)
=ò = ò
2
sin x 1 - 2sin x 2
t 2
1 - 2t 2 x2 = 2Y, where Y = y – 2
Þ a = 1/2
\ directrix is Y = –1/2 or y – 2 = –1/2
1 1
Put t = Þ dt = - 2 du
u u

udu
\ I1 = - ò = - u 2 - 2 = - cos ec 2 x - 2 y= –a
2
u -2
\ 2y – 3 = 0
SOLUTIONS - MOCK TESTS 85
18. (c) The family, which is given by F(x, y, c) = y – cx2, consists 20. (b) Here, F(x, y, c) = x2 + y2 – cx. Implicitly differentiating
of parabola symmetric about the y–axis with vertices the given equation with respect to x, we obtain
at the origin. Differentiating the given equation with
dy
2x + 2y =c
dy dx
respect to x, we obtain = 2cx .
dx Eliminating c between this equation and x2 + y2 – cx =
y
2
y = 2x dy x 2 + y2 dy y 2 - x 2
y=x
2 0, we find 2x + 2y = or =
1 dx x dx 2xy
1 2 y2 + x2 = 2
y= x 2
2 dy 2xy
1 Here f (x, y) = (y2 – x2) / 2xy, becomes =
y2 + x2 = 1 dx x 2 - y 2
2
x
This equation is homogeneous, and its solution gives
1 1 the orthogonal trajectories as x2 + y2 = ky.
y2 + x2 =
1
y = 2x2 2 2 21. (b) Let x1, x2, x3 Î R be the roots of f (x) = 0
y = -x2 \ f (x) = (x – x1) (x – x2) (x – x3)
y = -2x2
f (i) = (i – x1) (i – x2) (i – x3)
| f (i) | = | x1 – i | | x2 – i | | x3 – i | = 1
dy x
Here f (x, y) = =- or x dx + 2y dy = 0
dx 2y \ x12 + 1 x 22 + 1 x 32 + 1 = 1
The solution of this separable equation is This is possible only if x1 = x2 = x3 = 0
1 2 Þ f (x) = x3 Þ a = 0 = b = c Þ a + b + c = 0
x + y2 = k .
2 Þ/ all roots are zero.
These orthogonal trajectories are ellipses. Each ellipse 22. (d) L1 and L2 are obviously non-parallel
intersects each parabola at right angles. Consider the determinant
19. (d) The family F(x, y, c) = x2 + y2 – c2, consists of circles
with centers at the origin and radii c. Implicitly 2 -4 1
differentiating the given equation with respect to x, we D= 2 4 -3
obtain = 2 (8 + 9) + 4 (4 + 3) + 1 (6 – 4)
1 3 2
dy x
2x + 2yy ¢ = 0 or =- = 34 + 28 + 2 = 64 Þ D ¹ 0 Þ skew lines
dx y
Hence, Statement-1 is false.
x dy y
Here f (x, y) = - , so that =
y dx x
23. (a)
y
y = 2x N S N S

y = -x y=x
x2 + y2 = 1 A
x2 + y2 = 2
x2 + y2 = 4
x
S N N S
B

This equation is linear ( and, in differential form,


separable); its solution is y = kx, which represents the 24. (c) Acceleration of mass at distance x
orthogonal trajectories. a = g (sin q – µ0 x cos q)
In figure some members of the family of circles are speed is maximum, when a = 0
shown in solid lines and some members of the family
g (sin q – µ0 x cos q) = 0
(i), which are straight lines through the origin, are shown
in dashed lines. Observe that each straight line tan q
intersects each circle at right angles. x=
m0
EBD_7801
86 Target IIT
25. (a) Rate of water coming from tap T is 0.021×10–3m3/min. 29. (c, d)
PV = nRT
2.1 ´ 10-5 3 TB > TA
F= m /sec
60 WAB = P0 (2V0 – V0) = P0V0
Same amount of water should be ejected in form of WBC = RT ln (3/2) = 2 P0V0 [ln 3 – ln 2]
steam. = 2 P0V0 (2.303) (0.477 – 0.30)
So heat supplied must be = 0.8142 P0V0 < WAB
DQ = DmSDT + DmL 30. (c,d)
or rate of heat transfer The spring pulls both the blocks with same force. Hence
force on both blocks is equal and opposite.
DQ Dm Dm
= SDT + L Since no net external force acts on system and its initial
Dt Dt Dt momentum is zero, therefore net momentum of system is
æ Dm ö always zero. Hence momentum of blocks are equal and
P = rFSDT + rFL çèQ = r ´ F÷
ø opposite.
Dt
31. A-p, s; B-q, s; C-q, s; D-s, t
1000 ´ 2.1 ´ 10 -5 (A) Electrostatic potential energy
P= [4.2 × (100 – 20) + 2300] kJ/sec.
60 1 ( -Q ) 2 Q2
= =
P = 922.6 J/sec or watt 4p Î0 2a 8p Î0 a
26. (b) Upward force by capillary tube on top surface of liquid
is fup = 4sa cos q. Q2
If liquid is raised to a height h then we use (Using U = , C = 4p Î0 a)
2C
4s cos q (B) Electrostatic potential energy
4sa cos q = ha2 rg or h =
arg é (-Q ) ´ (-Q)
1 (-Q )2 ù 3 Q 2
= ê + ú=
27. (a) For whole system 4p Î0 êë 5a / 2 2 (5a / 2) úû 20 p Î0 a
f1 - f 2 = 3 (1)a ......... (1)
(C) Electrostatic potential energy

1 3Q 2 3 Q2
= =
m 4p Î0 5a 20 p Î0 a

(D) Electrostatic potential energy


//////////////////////////////////////
1 é 3Q 2 (-Q )2 (-Q) ´ (-Q) ù 27Q 2
= ê + + ú=
For rear cylinder 4p Î0 ëê 5a 2(2a) 2a ûú 80p Î0 a
100 – f1 = 0.5 (a) ......... (2) 32. A-q; B-r, s; C-s; D-p, q, r
(A) Total thermal energy generated in wire
For front cylinder
= energy stored in capacitor
f2 = 0.5 (a) ......... (3)
(B) e = Bvl
From (1), (2) and (3)
(C) F = qE
100 = 4a
(D) V = iR, Heat = i2Rt
a = 20 m/s2
33. Ans : 1
28. (a, b)
d fs di p
R L/4 es = N s and es = M ;
= dt dt
R1 + R 2 3L / 4 ; 3R = (R1 + R2)
d fs di p
R + R1 2 / 3 \ Ns =M
= dt dt
R2 1/ 3
3R = R2 + R1 d fs 200(2.5 ´ 10-4 - 0)
or M = N s = = 2.5×10–2H
R = 2R2 – R1 di p (2 - 0)
5R 4R But M = 0.025 K Þ 2.5 × 10–2 = 0.025 K
4R = 3R2 ; R1 = , R2 =
\K=1
3 3
SOLUTIONS - MOCK TESTS 87
34. Ans : 9 The final volume
The situation is shown in fig.
Let F be the horizontal force applied on A. T1=300K Atm pressure=105 N/m2
For block A, F = T + f1 = T + m m1g .....(1) Ideal
(Q Block A moves towards left, frictional force f1 acts towards Heater Monoatomic
Gas
right) k=8000N/m
For block B, fB = f2 + f3

= V2 = V1 + xA = 2.4 × 10–3 + 0.1 × 8 × 10–3 = 3.2 × 10–3 m3


PV
1 1 PV P2V2 T1
Applying = 2 2 Þ T2 =
T T1 T2 P1V1
F A
f2 f1 2 ´ 105 ´ 3.2 ´ 10-3 ´ 300
Þ T2 = = 800 K.
B 105 ´ 2.4 ´ 10-3
f3
T \ 100 T = 800 Þ T = 8
37. (c) iLB + kx = ma
(Q Block B moves towards right, frictional forces f2 and f3 dq dq dv
acts towards left). i= ; q = CBLv Þ i = = CBL = BLCa
dt dt dt
T = m m1g + m ( m1 + m2 ) g = m g (2 m1 + m2 ) ...(2) \ B2L2Ca + kx = ma
From eqs. (1) and (2), we get -kx
a=
F = m g (2 m1 + m2 ) + m m1 g or F = m g (3 m1 + m2 ) m + B2 L2 C
\ F = 0.3 ´ 10 ( 3 ´ 10 + 20) = 150 N dv kx
38. (b) v =-
Given : F = 10 (h + 6) dx m + B2 L2C
Þ 150 = 10 (h + 6)
Þ h=9 v2 v20 kx 2
- =-
35. Ans : 4 2 2 2 (m + B2 L2 C)
The displacement of a particle in S.H.M. is given by :
y = a sin (wt + f ) m + B2 L2C
For xmax, v = 0 ; xmax = v0
dy k
velocity = = wa cos (wt + f ) 39. (a) aa – x Þ SHM
dt
The velocity is maximum when the particle passes through
the mean position i.e.
40. (a) Graph 1 :
æ dy ö
çè ÷ø
dt max = w a
hf = KEmax + f Þ 0 = (–E) + f Þ f = E
The kinetic energy at this instant is given by
2
1 æ dy ö 1 – +
mç ÷ = mw2 a2 = 8 × 10–3 joule V
2 è dt ø max 2 41. (a) Graph 2 :

or
1
× (0.1) w2 × (0.1)2 = 8 × 10–3 V +
2 hf + eV = KEmax + f
Solving we get w= ±4 0 + eV = 0 + f
\ Magnitude of w = 4 rad/s. or eV = f = E
36. Ans : 8 E + –
The final pressure on the gas = Atmospheric pressure 42. (b) hf -= KE max + (f) – +
2
+ Pressure due to compression of spring S3 S4
3E V/2
kx Þ hf = 0 +
P2 = Patm + 2
A 3E V
or E1 =
8000 ´ 0.1 2
Þ P2 = 105 + = 2 × 105 N/m2 43. (c) When Dx = 0, for any l the interference will be
8 ´ 10-3
constructive.
EBD_7801
88 Target IIT
44. (d) Peak voltage across the resistance (at resonance) in Xbenzene = 0.7 {Ybenzene > 0.7}
LCR series = source voltage. Hence, statement-1 is false, Þ Ytoluene necessarily < 0.3
statement-2 is true. Statement (A) is correct
O H O O– Na+ Xbenzene = 0.5 & Ytoluene = 0.20
O3 C Conc. NaOH C Statement (B) : Xtoluene = 0.3 Þ Xbenzene = 0.7
45. (d) Zn C CH2OH Þ Ybenzene must be > 0.7
O H
(P) (Q) kw
a1 = kw
49. (b) a =
O k a .C1 ; 2 k a .C2
;

LiAlH4 CH2 OH CH 3COCl CH2 – O – C – CH3 a1 C2 0.01 æ 0.4 ö


\ = Þ = ç =2
CH2 OH (excess) CH – O – C – CH a2 C1 a2 è 0.1 ÷ø
2 3
(R)
O 0.01
(S) Þ a2 = = 0.005
2
Mol. weight (R) = 62 So, [OH–] = 0.4 × 0.005 = 20 × 10–4 = 2 × 10–3 M
Mol. weight (S) = 146
Change in mol. wt. from (R) to (S) = 146 – 62 = 84 10-14
46. (a) If the solution is blue, then absorbed wavelength of and [H+] = = 5 × 10–12 M
2 ´ 10 -3
light will be of orange to transfer electron from t2g to eg.
If the solution is green, then absorbed wavelength of 50. (a, b, c, d)
light will be of red to transfer electron from t2g to eg.
As lorange < lred , hence \ CFSE for blue solution will Step I
no heat only work
be maximum. Step II
OH OH
Br Br no work only heat
P
Br2 SO2
H2 Ar > H2 > SO2
47. (d) (a) H2O
Ar
V
OH OH Q For an ideal gas, the initial & final temperature are same
Br \ DE = 0 for all
Br2 \ C is correct
(b) Also | w | is maximum for SO2 in step I and overall
H2O
DE = 0
\ | q | for SO2 will be maximum.
Br \ option (B) is correct
Since after step I, all have same volume, same moles and
OH OH pressure of Ar is minimum
Br
\ Temperature of Ar will be lowest.
Br2 \ decrease in temperature will be maximum.
(c) \ option (A) is correct
CS2
Q g for Ar and SO2 is different (due to vibrational degrees
of freedom)
OH OH
Br \ graph is different.
Br
\ option (D) is correct.
Br2 51. (a, b, c, d)
(d) H2O (a) Wt. of C in sample = 84gm
84
Br Moles of C = = 7 ; No. of moles of O2 required = 7
12
48. (b) Benzene is more volatile than toluene
Hence from figure n O2 RT 7 ´ 0.0821 ´ 300
VO2 = = = 21
Statement (C) is correct P 8.21
Xtoluene = 0.7 and Ytoluene = 0.4
Þ Xbenzene = 0.3 and Ytoluene = 0.4 100
Vair = 21 ´ = 100 litre
Statement (D) is correct because 21
SOLUTIONS - MOCK TESTS 89
(b) Bond energy order : \ Eq. wt. of metal = 12
s s s s At wt of metal = Eq. wt × valency = 12 × 2 = 24
C - C > Si - Si > Ge - Ge > Sn - Sn [Valency of metal = 2]
2p - 2p 3p - 3p 4p - 4p 5p - 5p Given : 24 = 20 + A \A= 4
Extent of overlapping decreases in the order : 56. Ans : 3
2p – 3p > 3p – 3p > 4p – 4p > 5p – 5p n (n - 1)
Number of waves =
nRT 2 ´ 0.0821 ´ 243.6 2
(c) P1 = = = 2 atm.
V1 20 where n = Principal quantum number or number of orbit
3(3 - 1) 3´ 2
æP ö Number of waves = = =3
DS = nR ln ç 1 ÷ = 2 × 2 ln (2) Þ 4 × 0.693 2 2
è P2 ø
ALTERNATIVELY : In general, the number of waves made
Þ 2.77 cal/K by a Bohr electron in an orbit is equal to its quantum number.
(d) Let volume of solution is V ml According to Bohr’s postulate of angular momentum, in the
80 3rd orbit
V × 1.75 × = 2 × 0.25 × 98 = 35ml.
100 h
52. (a, c) mur = n
2p
Me Me Me
Na CH3 – Cl æ h ö
H OH H ONa H OMe mur = 3 ç ÷ ...(i) [n = 3]
SN2 è 2pø
14 Me 14 Me 14 Me
According to de Broglie relationship
Me Me
h
Red P/I2
I H
MeONa
H OMe l= ...(ii)
SN2 SN2 mu
14 Me 14 Me Substituting (ii) in (i), we get
53. A - p, q ; B - r, s ; C - p, q ; D - p, q æ hö æ h ö é hù
Hydrolysis of an ester is pseudo-first order reaction. çè ÷ø r = 3 ç ÷ or 3l = 2pr êëQ mu = l úû
For first order reaction l è 2pø
0.693 Thus the circumference of the 3rd orbit is equal to 3 times
t1/ 2 = the wavelength of electron i.e. the electron makes three
k
Inversion of cane sugar and decomposition of N2O5 also revolutions around the 3rd orbit.
follow first order kinetics. ALTERNATIVELY : rn for H = r1 × n2
Saponification of an ester is a second order reaction. For a r3 for H = 0.529 × 9 × 10–8 cm
second order reaction units of rate constant are litre mol–1 = .529 × 9 × 10–10 m (Q r1 = 0.529 Å)
sec –1. u1
54. A-r, s; B-s; C-p, q; D-p, q Also un = Z ;
n
Heat
(A) H 4 P2 O 7 ¾¾¾ ® 2HPO 3 + H 2 O. It also contains
2.19 ´ 108 2.19 ´ 10 6
P – O – P bond. \ u3 = cm sec–1 = m sec–1
Heat
3 3
(B) H3 PO 4 ¾¾¾ ¾
® HPO3 + H 2 O
600°C (Q u1 = 2.19 × 108 cm sec–1)
\ No. of waves in one round
H
| 2p r3 2p r3 2 p r3 ´ u3 ´ m
(C) H - O - P - O - H = = =
l h / mv3 h
¯
O 2 ´ 22 ´ 0.529 ´ 9 ´ 10 -10 ´ 2.19 ´ 10 6 ´ 9.108 ´ 10 -31
(D) H3PO2 acts as a reducing agent. = =3
7 ´ 3 ´ 6.62 ´ 10 -34
55. Ans : 4
According to problem, three atoms of M combine with 2 57. Ans : 7
atoms of N 3
\ Formula of compound is M3N2 (Where M is the metal) 4 3 4 22 æ 24 ö
Volume of one balloon = pr = ´ ´ ç ÷ = 7.241 L
14 3 3 7 è 2 ø
Equivalent wt of N =
3 æ diameter ö
(Q valency of N in compound is 3) çQ r = ÷
è 2 ø
Q 28 g N combines with = 72g metal
Let no. of balloons to be filled = n
72 14 \ Total volume occupied by n balloons = 7.241× n
\ 14/3 N combines with = ´ = 12
28 3 Volume of H2 present in cylinder = 2.82 L (given)
EBD_7801
90 Target IIT
\ Total volume of H2 at NTP = (7.241n + 2.82)L after addition of NH3
P1 = 1 atm P2 = 20 atm
V1 = 7.241 × n + 2.82 L V2 = 2.82 L [Ag(NH 3 )2 ]+ 0.05 5
[Ag + ]A = = =
T1 = 273 K T2 = 300 K K f .[NH3 ]2 (K f ) (0.1) 2 Kf
PV
1 1 PV 1 ´ (7.241 ´ n + 2.82) 20 ´ 2.82 Beaker 'B' : Initial concentration [Ag+] = 0.01
= 2 2 or = after addition of NH3
T1 T2 273 300
[Ag(NH3 ) 2 ]+ 0.01 1
48.504 [Ag + ]B = = =
\n= ; 6.69 » 7 K f .[NH 3 ] 2
(K f ) (0.2) 2 4K f
7.241
58. Ans : 2 0.06 [Ag + ]B 1/ 4
E cell = 0 - log = - 0.06 log
1 [Ag + ]A 5
PN = 0.001 atm, T = 300 K, V = 2.46 cm 2
2 = 0.06 log 20 = 0.06 {1 + 0.3} = 0.078 Volt
= 7.8 × 10–2 Volt
\ Number of N 2 molecules 62. (c) N1V1 + N2V2 = N3V3
-3 5.6 33.6 16.8
PV 0.001´ 2.46 ´ 10 ´2+ ´2= ´ V3
= × N AV = × 6.023 × 10 23 5.6 5.6 5.6
RT 0.0821´ 300
14 14 2
= 6.016 × 1016 \ V3 = \ Vw = - 4 = = 0.67L
3 3 3
Now total number of surface sites 63. (d) 2H2O2 ® 2H2O + O2
= Density × Total surface area 5.6 33.6
moles of H2O2 = ´2+ ´2=7
= 6.023 × 10
14 17
× 1000 = 6.023 × 10 11.2 11.2
moles of O2 = 7/2
20 7
Sites occupied by N 2 molecules = × 6.023 × 1017 V= ´ 22.4 = 78.4L
100 2
64. (b) N1V1 + N2V2 = N3V3
= 12.04 × 1016
5.6 33.6 16.8
\ No. of sites occupied by each N 2 molecule ´ 2+ ´ V2 = ´ (2 + V2 )
5.6 5.6 5.6
12.04 ´ 1016 4
= \ V2 =
»2 3
6.016 ´ 1016
4 10
+
\ Total volume prepared = 2 + = = 3.33L
0.0591 [H ] 3 3
59. (d) E cell = log + RHS 65. (c) Carbon reduction of Al2O3 occurs above 2000°C
1 [H ]LHS M.P. of Al : 660°C B.P. of Al : 2467°C
Ecell = 0.0591 ((pH)LHS – (pH)RHS) Hence some Al goes into vapour phase. Also a part of
= 0.0591 (3.95 – 1.45) = 0.14775V liberated Al combines with liberated carbon and form
60. (d) pH of solution (A) : Solution A is a acidic Buffer Al4C3, therefore carbon reduction of Al2O3 is difficult
Al4C3 + 12 H2O ® 4Al(OH)3 + 3CH4
[Salt]
pH = pKa + log Þ pH = 5 66. (b)
[Acid] – – +
+
Þ [H+]A = 10–5 M
pH of solution (B) – + –
+
1 1
pH = [pKw – pKb – log C] Þ [14 – 5 + 1] Except d 2 other d orbitals form d-bond by side ways
2 2 z
pH = 5 Þ [H+]B = 10–5 overlapping between all four lobes.
z
Now, The cell is a concentration cell
Cell reaction : 2HB+ (aq) + 2e– ƒ H2,B(g)
H2,A(g) ƒ 2HA+ (aq) + 2e– +
n =2
2HB+ (aq) + H2,A(g) ƒ 2HA+ (aq) + H2,B(g) x
Nernst Equation for cell
+
0.06 [H + ]2
E cell =0- log + A Þ 0 Volt d z²
2 [H ]2B y

61. (b) Beaker 'A' : Initial concentration [Ag+] = 0.05 No. of nodal planes Þ 0
SOLUTIONS - MOCK TESTS 91
ANSW ER KEY PAP ER - 2
1 (c) 15 (b) 29 (a, c) 43 (d) 57 4
2 (b ) 16 (a) 30 (a, b, c) 44 (d) 58 5
3 (d ) 17 (a) 31 A -p, r; B-p, r; C-q, s ; D-q, s 45 (c) 59 (c)
4 (c) 18 (c) 32 A -q , s ; B-q, s ; C-r, s ; D-s 46 (c) 60 (a)
5 (a) 19 (b) 33 1 47 (a) 61 (d)
6 (b, c, d) 20 (d) 34 2 48 (b) 62 (c)
7 (a, d) 21 (c) 35 3 49 (b) 63 (c)
8 (a, b, c, d) 22 (a) 36 5 50 (a, b, c) 64 (b)
9 A - t; B - r; C - p; D - r, s , t 23 (c) 37 (d) 51 (b, c) 65 (a)
10 A - q, B - p, C - r, D - q 24 (a) 38 (b) 52 (a, c, d) 66 (d)
11 3 25 (d) 39 (c) 53 A -q; B-p, q , r; C-p, q, s ; D-q
12 1 26 (a) 40 (b) 54 A -p; B-q, r, s ; C-p, s ; D-q, s
13 6 27 (a) 41 (d) 55 4
14 1 28 (a, b, d ) 42 (c) 56 1

PAPER - 2 pa 2 pb 2 p 2
Hence, I = + = (a + b2 )
2 2 2
(x + y - 3)2 p 2 p
1. (c) (x – 1)2 + (y – 1)2 = I(a) = [a + (1 - a) 2 ] = [2a 2 - 2a + 1]
2 2 2
x + y-3
i.e., (x - 1) 2 + (y - 1) 2 = , é 1ù éæ 1ö
2

2 = p ê a 2 - a + ú = p êç a - ÷ + ú
ë 2û è 2ø 4 ûú
Which is a parabola whose focus is (1, 1) and directrix ëê
is x + y – 3 = 0 \ I (a) is minimum when a = 1/2 and minimum value is
1 p/4
Distance of (1, 1) from x + y – 3 = 0 is .
2 x a
3. (d) =
\ Length of latus rectum is 2. sin 7° sin150°
p x = 2a sin 7° ........... (1)
(b) I = ò (a sin x + bsin 2x) dx
2
2. ...(1) A
0

p 23° 83°
and I = ò (a sin x - b sin 2x) dx ...(2) 2
a x a
0
150°

Add (1) and (2) M



y
p
30° 37°
Þ 2I = 2ò (a sin x + b sin 2x) dx
2 2 2 2 B C
0 Using cosine rule in D AMC,
p/ 2 p /2 y2 = x2 + a2 – 2ax cos 83°
I=2 ò (a 2 sin 2 x) dx + 2 ò (b 2 sin 2 2x) dx = 4a2 sin2 7° + a2 – 4a2 sin 7°
0 0 y = a2 Þ y = a
2

p /2 Hence, Ð AMC = 83°


p
ò
2
= 2a + 2b2 sin 2 2x dx 4. (c) Obviously D1 = D2 = D3 = 0
4
14
0 4244
3
J 1 -l -1
D= l -1 -1 ¹ 0
p /2
-1
ò 1 1
2
Let J = sin 2x dx , put 2x = t Þ 2dx = dt
0
= (1 - l ) (1 + l ) ¹ 0
p p/ 2
1 p
\ J = 2 ò sin t dt = ò
2
sin 2 t dt = Þ l ¹ 1, –1. Hence, l = R – {–1, 1}
0 0
4
EBD_7801
92 Target IIT
5. (a) Both the lines pass through origin 8. (a, b, c, d)
Line L1 is parallel to the vector y = x1/3 (x – 1)
V1 = (cos q + 3) ˆi + ( 2 sin q) ˆj + (cos q - 3) kˆ dy 4 1/3 1 1 1
= x - . 2/3 = 2/3 [4x - 1]
and L2 is parallel to the vector dx 3 3 x 3x
r
V2 = aiˆ + bjˆ + ckˆ 1 1
Hence, f is ­ for x > and f ¯ for x <
r r 4 4
V1 .V2
\ cos a = r r Also, x 2 / 3 is always positive and at x = 1/4 the curves has
| V1 | | V2 |
a local minima.
y
a(cos q + 3) + (b 2)sin q + c(cos q - 3)
=
a 2 + b 2 + c 2 (cos q + 3) 2 + 2sin 2 q + (cos q - 3)2

(a + c)(cos q) + b 2 sin q + (a - c) 3
=
a 2 + b 2 + c2 2 + 6 1/4
x
In order that cos a is independent of q –1/2 (1,0)
a + c = 0 and b = 0
4 1/3 1 -2/3
2a 3 3 p Now, f ¢(x) = x - .x (non existent at x = 0,
\ cos a = = Þa= 3 3
a 2.2 2 2 6 vertical tangent)
6. (b, c, d) 4 1 1 2 1
f ¢¢ (x) = 2/3
+ . . 5/3
a b c 9x 3 3 x
, , are in H.P..
1 - 2a 1 - 2b 1 - 2c
2 é 1ù 2 é 2x + 1ù
1 - 2a 1 - 2b 1 - 2c
= êë 2 + x úû = 2/3 êë x úû
2/3
Þ , ,
9x 9x
are in A.P..
a b c \ f '' (x) = 0 at x = –1/2 (inflection point)
Area of f (x),
1 1 1 1 1
Þ , , are in A.P. Þ a, b, c are in H.P.. 3 3 ù
a b c A = ò (x 4/3 - x1/3 ) dx = x 3/7 - x 4/3 ú
0
7 4 û0
b b b 3 3 4- 7 9
(b) a - , , c - are in G.P.. = - =3 =
2 2 2 7 4 28 28
9. A - t; B - r; C - p; D - r, s, t
æ bö b æ bö
ln ç a - ÷ , ln , ln ç c - ÷ are in A.P..
è 2ø 2 è 2ø æ 1ö
(A) Equation of tangent to xy = –1 at çè t , - ÷ø is
t
b b b
(c) c - , , a - are in G.P.. 1 2
2 2 2 y= x- and any tangent to y 2 = 8x is
2 t
t
1 1 1
(d) , , are in A.P. Þ e1/a, e1/b, e1/c are in G.P.. 2
a b c y = mx + .
7. (a, d) m

z 4 - z1 1
2 2
= i i.e. z – z = i (z – z ) \m = 2
= and -
Þ t = -1
z 2 - z1 4 1 2 1 t t m
z4 z3 So, the common tangent is y = x + 2.
Also, z1 + z3 = z4 + z2
i.e. z1 + z3 = z1 + i (z2 – z1) + z2 (B) Let f ( x) = [ f ( x ) + f '( x ) + f ''( x )] e - x . Clearly f(x) is
i.e. z3 + i z1 = (1 + i) z2 continuous on [a, b] and differentiable on (a, b).
i.e. 2z2 = (1 – i) z3 + (1 – i) iz1 Also,
= (1 + i) z1 + (1 – i) z3 ....... (1)
z1 z2 f( a ) ì f (a ) + f '( a) + f ''(a ) ü b- a
2 (z1 + z3 – z4) = (1 + i) z1 + (1 – i) z3 =í ýe = ea -b .eb - a = 1
f(b) î f (b) + f '(b) + f ''(b) þ
\ 2z4 = 2z1 + 2z3 – (1 + i) z1 – (1 – i) z3
= (1 – i) z1 + (1 + i) z3 ........ (2) [From the given condition]
\ f(a) = f(b)
SOLUTIONS - MOCK TESTS 93
Hence by Rolle's theorem, these exists c Î (a, b) such
12 æ Bö 1 1
that
\ P(B) = å P(Ei ) P ç ÷ = ´ 12
f'(c) = 0 è Ei ø 13 C4
i =0
Þ –{f (c) + f ¢(c) + f "(c)}e–c + e–c {f ¢(c)
+ f "(c) + f "'(c)} = 0 é 2 C2 ´ 10C2 + 3C2 ´ 9C2 + ....... + 10C2 ´ 2 C2 ù
Þ {f "'(c) – f(c)} e–c = 0 ë û
Þ f "'(c) = f(c)
1 1
(C) f (-2) = f (-1) = f (1) = f (2) = 0 = ´ 12
13 C4
Therefore by Rolle's mean value theorem the equation
f '(x) = 0 has roots in (–2, –1), (–1, 1) and (1, 2).
\[a] = -2, [b] = -1, [ g ] = 1 . ë {
é 2 2 C ´ 10C + 3C ´ 9C + ....... + 5C ´ 7C + 6C ´ 6C ù
2 2 2 2 2 2 2 2û }
dy 1 1 1 1
(D) = , so the normal at the point (t 2 , t) is = ´ (1287) =
dx 2 y 13 495 5
y – t = –2t (x – t2). It passes through (c, 0) if 11. Ans : 3
– t = –2t (c – t2) n
1 1 We have (1 + x)n = å n
Cr x r
t 2 = c - for three distinct values of t 2 > 0 Þ c > . r=0
2 2
10. A - q, B - p, C - r, D - q Integrate under the limit 0 to x.
Let Ei denotes the event that the bag contains i black and x x n
(12 – i) white balls (i = 0, 1, 2,..........,12) and A denotes the
event that the four balls drawn are all black. Then Then ò (1 + x) dx = ò
n å n Cr x r dx
0 0r=0
1
P( Ei ) = (i = 0,1, 2,........, 12) ; (1 + x)n +1 –1
n n
Cr x r +1
13 Þ
n +1
= å r +1
r=0
i
æ Aö æ Aö C
P ç ÷ = 0 for i = 0, 1, 2, 3 ; P ç ÷ = 12 4 for i ³ 4 Integrating again between the limits 0 and x,
è Ei ø è Ei ø C4
x x n
(1 + x)n +1 –1 n
Cr x r +1
12 æ Aö ò dx= ò å dx
(A) P( A) = å P(Ei ) P çè E ÷ø 0 n +1 0 r= 0 r +1
i=0 i
1 é (1 + x)n + 2 – 1 ù n n
Cr x r +1
=
1
´
1 é4
C + 5 C4 + ...... + 12C4 ù
Þ
n +1
ê
êë n+2
– xú =
úû
å (r + 1) (r + 2)
13 12 C4 ë 4 û r=0

Integrating again between the limits 0 and x,


13
C5 1
= = 1 é 1 ìï (1 + x) n + 3 –1 üï x 2 ù
12 5 ê –1ý – ú
13 ´ C4 í
n +1 ëê n + 2 îï n+3 ïþ 2 ûú
æ A ö 10
C4 14
(B) Clearly, P ç = = n n
Cr x r + 3
è E10 ÷ø 12
C4 33 =å
r =0 (r + 1)(r + 2)(r + 3)
(C) By Baye’s theorem,
Put x = – 1 to both the sides, we get
æ A ö 1 14
P ( E10 ) P ç 1 é 1 ì –1 + 1ü – 1 ù
æ E10 ö è E10 ÷ø 13 ´ 33 70 ê í ý ú
Pç = = = n +1 ë n + 2 în + 3 þ 2û
è A ÷ø P( A) 1 429
5 n n
C r (–1) r + 3
(D) Let B denotes the probability of drawing 2 white and 2 = å (r + 1)(r + 2)(r + 3)
r=0
black balls then
æ Bö 1 n n
C (–1) r + 3
P ç ÷ = 0 if i = 0, 1 or 11, 12 Þ
2(n + 3)
= å (r + 1)(rr + 2)(r + 3)
èE øi
r =0

n
3! (–1)r . n C r
æ B ö i C ´12-i C2
P ç ÷ = 2 12 for i = 2, 3,.......,10
Þ =å
2(n + 3) r = 0 (r + 1)(r + 2)(r + 3)
è Ei ø C4
3!
EBD_7801
94 Target IIT

n 13. Ans : 6
(–1)r . n C r
Þ
3!
= å
2(n + 3) r = 0 r + 3 C r
In general consider the r th square. Let the sides of the squares
S1 , S2 , S3 ,..., Sn , Sn+1 be s1 , s2 , s3 ,..., sn , sn+1 respectively..
n n
Cr 3! Thus the side of the square S r = sr .
Þ (n + 3) å (-1)r r+3
Cr
=
2
=3
r =0 Now, given sn = 2sn +1
12. Ans. 1
( ) ( 2)
n -1
Here ABCD is tetrahedron. Let O be the origin and let the \ s1 = 2s2 = 2 2s3 = ... = sn
r r r r
P.V of A, B, C, D be a, b, c, d respectively . We know that
s12 100
four linearly dependent vector can be expressed as: The area of the square S n = ( sn ) =
2
=
r r r r 2n -1 2 n-1
xa + yb + zc + td = 0 [where x, y, z, t are scalars]
r r r r Given the area of the square sn > 2
or yb + zc + td = - xa
r r r r
100
yb + zc + td xa
\ > 2 Þ 100 > 2n Þ n = 1, 2,3, 4,5, 6
Þ =- 2n -1
y+z+t y+z+t
14. Ans. 1
Where L.H.S. is P.V. of a point in the plane BCD and R.H.S is Let P (x1, y1, z1) be a point on ax + by + cz = d. Then
uuur
a point on line AO . ax1 + by1 + cz1= d ...(i)
\ There must be a point common to both the plane and x1 y1 z1
r Let OP = r. Then direction cosines of OP are , ,
- xa r r r
straight line i.e. OP =
y+z+t
x-0 y-0 z-0
r Equation of line OP is = = ,
xa r æ x + y + z + t ör x1 y1 z1
But, AP = OP - OA = - -a = – çç ÷÷a r r r
y+z+t è y+z+t ø
Let Q (a, b, g) be a point on OP such that OQ = l. Then
uuur x æ y + z + t ö uuur
\ OP = .ç ÷ AP æ x l y l z lö
y+z+t è x+ y+z+t ø coordinates of Q ç 1 , 1 , 1 ÷ . Thus
è r r r ø

O ar br gr
x1 = , y1 = , z1 =
l l l
A Now ax1 + by1 + cz1 = d
C
r
Þ (aa + bb + c g ) = d ....(ii)
l
Now OP.OQ = d2
B
D Þ rl = d 2

uuur æ x ö uuur OP x r
OP = . ç ÷ AP Þ =. Þ r l = d . (aa + bb + c g )
è x + y + z + t ø AP x + y +z+t l

OQ y Þ (aa + bb + c g )d = l 2
Similarly =
BQ x + y + z + t
Þ (aa + bb + c g )d = a 2 + b + g 2
OR Z OS t So, locus of (a, b, g) is (ax + by + cz) d = x2 + y2 + z2
=. and =.
CR x+y+z+t DS x + y + z + t
( ax + by + cz ) d
or =1
OP OQ OR OS x2 + y 2 + z 2
Þ + + + =1
AP BQ CR DS
SOLUTIONS - MOCK TESTS 95
15. (b) (1 + y2)dx = xy dy 8
y Tr +1 Cr
>1 Þ >1
Tr < 8
Cr -1 <

8! (r - 1)!(9 - r)!
> Tr ;
Tr +1 < . ³1
r! (8 - r)! 8!

x (9 – r) ³ r Þ 9 ³ 2r
(–Ö 2,0) (Ö 2, 0) For r = 1, 2, 3, 4 this is true
i.e. T5 > T4
But for r = 5, T6 < T5
Þ T5 is the greatest term Þ Correct option is (B)
dx ydy (iii) Again, Tk+1 = 8Ck. 2k.xk ; Tk = 8Ck–1.2k–1.xk–1
ò x ò 1 + y2
=
Tk–1 = 8Ck–2.2k–2.xk–2
2 ln x = ln (1 + y2) + c We want to find the term having the greatest coefficient
Given x = 1, y = 0 Þ c = 0 \ 2k–1. 8Ck–1 > 2k.8Ck .......... (1)
Hence equation of conic C is x2 – y2 = 1,
which is a rectangular hyperbola with eccentricity e = and 2k–1. 8Ck–1 > 2k–2.8Ck–2 .......... (2)

2. 8!2k -1 2k.8! 1 2
Length of the latus rectum of rectangular From (1), > Þ >
(k - 1)! (9 - k)! k! (8 - k)! (9 - k) k
hyperbola = 2a = 2
Þ k > 18 – 2k Þ k > 6
16. (a) Now for ellipse, Again 2k–1. 8Ck–1 > 2k–2.8Ck–2
2
ae = 2 Þ a 2 e2 = 2 Þ a 2 . = 2 Þ a 2 = 3 8!.2 k -1 2 k - 2.8! 2 1
3 > Þ >
(k - 1)!(9 - k)! (k - 2)!(10 - k)! k - 1 10 - k
and b2 = a 2 (1 - e2 ) = 3 æç 1 - ö÷ = 1
2 Þ 20 – 2k > k – 1 Þ 21 > 3k Þ k < 7
è 3ø Þ 6 < k < 7 Þ T6 and T7 term has the greatest coefficient
Þ k = 6 or 7 Þ Sum = 6 + 7 = 13
x 2 y2
Hence equation of ellipse is + =1 18. (c) 19. (b) 20. (d)
3 1
21. (c) If f (x) is odd, then f ' (x) is even but converse is not true
17 (a) Locus of the point of intersection of the perpendicular
tangents is the director circle of the ellipse whose e.g. if f ' (x) = x sin x
equation is then f (x) = sin x – x cos x + C
x2 + y2 = 4 f (–x) = – sin x + x cos x + C
For Qs. 18 to 20 ––––––––––––––––––––––
R = (1 + 2x)n f (x) + f (–x) = constant which need not to be zero.
Put x = 1 to get sum of all coefficients x
\ 3n = 6561 = 38 Þ n = 8 For statement-1 : f (x) = ò 1 + t 2 dt, g(x) = 1 + x 2
0
1
(i) For x = , R = ( 2 + 1)8
2 -x

Consider, f (- x) = ò 1 + t 2 dt, t = - y
0
( 2 + 1)8 + ( 2 - 1)8 = 2 éë 8 C0 ( 2)8 + .....ùû = even integer
144424443
I+ f + f ¢ x
Since I is integer Þ f + f ' must be an integer f (- x) = - ò 1 + y2 dy
but 0 < f + f ' < 2 Þ f + f ' = 1 Þ f ' = 1 – f 0
Now, n + R – Rf \ f (x) + f (–x) = 0 Þ f is odd and g is obviously even.
n n
n + R (1 – f ) = 8 + ( 2 + 1) ( 2 - 1) = 8 + 1 = 9 22. (a) D = 12 5 , using Heroes formula.
1
(ii) Tr+1 in (1 + 2x)8 = 8Cr (2x)r = 8Cr when x = 21 5 æQ R = abc ö
2 R= ç ÷
10 è 4D ø
Now Tr+1 ³ Tr
EBD_7801
96 Target IIT
23. (c) pulse moving For q to be minimum sin a should be max. Þ a = 90°
towards right Þ q = 60°
elementary
section pulse moving æ cö c
towards left \ approach velocity = çè ÷ø cos 60° =
3 6
Velocity
vector of
elementary æ ö
section
ç c ÷ 6f 0
\ f max = f0 ç =
c÷ 5
çc- ÷
Velocity Velocity è 6 ø R/2
profile profile
60° 60°
Figure-1 Figure-2 æ ö R
The velocity profile of each elementary section of the ç c ÷ 6f 0
and f min = f0 ç =
pulse is shown in figure 1 and figure 2. c÷ 7
When both the pulses completely overlap, the velocity çc+ ÷
è 6 ø
profiles of both the pulses in overlap region are
identical. By superposition, velocity of each elementary 28. (a, b, d) y
section doubles. Therefore, KE of each section 1 Qq
F¢ = .
becomes four items. Hence the K.E. in the complete
width of overlap becomes four times. (
4p Î0 x + y2
2
) (0, y) Q
y q q

24. (a) Loss in gravitational P.E. = Gain in spring P.E. Resultant force, x¢ x
x (x, 0)
1 F= (0, –y) Q F¢
mgh = k (h cot a - h) 2 F'2 + F' 2 + 2F' F' cos 2q
2
= 2F' 2 (1 + cos 2q ) y¢
2mg 2mg
or (cot a - 1) = or cot a = 1 +
kh kh 2Qq x
= 2F' cos q =
25. (d) Work done by gas in going isothermally from state A
to B is
4p Î0
( x 2 + y 2 )3 / 2
At a point with coordinates (x, 0) the force is
P
DWAB = nRT ln A = nRT ln 2 ......... (1)
2Qq x dF
PB F= . For F to be maximum, equating
4pe 0 (x 2 + y 2 )3/ 2 dx
Work done by gas in going isothermally from state B
PB P y
to C is DWBC = nRT ln = nRT 0 ......... (2) to zero gives x = ± . The charge is obviously in
PC 2PC 2
It is given that DWBC = 2DWAB equilibrium at the origin. However, the equilibrium is not
stable since the force is repulsive and hence will not be able
P0 P0
ln = ln(2) 2 \ PC = to restore the charge at the origin. The charge therefore
2PC 8 cannot perform oscillatory motion.
26. –lt
(a) Use the relation N = N0e where the decay constant 29. (a, c)
l = (0.693 / Thalf). The half life period Thalf is given to The particle is doing SHM because of the frictional force
be 10min. The amount N of radioactive sample remaining only. The contact force on the block is
at the end of 5 min comes out to be about 0.707g, so
that the amount of sample decayed is (1 – 0.707) F = (mg)2 + f 2 = m g 2 + a 2 = m g 2 + (w 2 x)2
= 0.293g. This is not proportional to x but proportional to m. The
27. (a) For f to be greatest, q should be minimum. In D ABC, contact force on the platform is F only and this is
using the rule independent of M.
30. (a, b, c)
sin (90° - q ) sin a sin a
= Þ cos q =
R/2 R 2 4
Velocity of image of bird in water = 6 ´ = 8 cm / s ¯
3
w.r.t fish = 8 + 4 = 12 cm/s¯
R/2 Velocity of bird in water after reflection from mirror
B C
R = 8 ­cm/s
a w.r.t fish = 8 – 4 = 4 cm/s ­
q
A
3
Velocity of image of fish in air = 4 ´ = 3 cm / s ­
4
SOLUTIONS - MOCK TESTS 97
Velocity of fish w.r.t bird = 3 + 6 = 9 cm/s ­ 34. Ans : 2
Velocity of image of fish after reflection from mirror in air Applying pseudo-force ma and resolving it.
3 Applying Fnet = max for x-direction
= 4´ = 3 cm / s ¯ ma cos q – (f1 + f2) = max
4
w.r.t bird = – 3 + 6 = 3 cm/s ­ ma cos q – µN1 –µN2 = max
31. A - p, r ; B - p, r ; C - q, s ; D - q, s ma cos q – µma sin q – µ mg = max
At positions P and R frictional force is maximum and normal Þ ax = a cos q – µa sin q – µg
reaction is mg. At positions S and Q frictional force is zero
4 2 3 2
and normal reaction points towards centre. = 25 ´ - ´ 25 ´ - ´ 10 = 10 m / s 2 = (5 × 2) m / s2
32. A - q, s; B - q, s; C - r, s; D - s 5 5 5 5
\ a=2
T ­r­ R ­ n­;l­ R ­ n­ ; A­ R ¯ n­
35. Ans : 3
(­ Þ increase and ¯ Þ decrease) As diaphram C is a node, A and B will be antinode (as in an
Volume ­ n ­ (Volume ­ may be due to organ pipe either both ends are antinode or one end node
A ­ l ­ or A ¯ l ­ or A ­ l ¯ , so variation in and the other antinode), i.e., each part will behave as closed
resistance is not certain) end organ pipe so that
33. Ans : 1 vH 1100
We can understand the direction of flow of induced currents fH = = = 550 Hz
4 LH 4 ´ 0.5
as a if a fictitious battery is attached between EF. The
direction of induced current can be found with the help of v0 300
lenz’s law. and f0 = = = 150 Hz
4 L0 4 ´ 0.5
R1 As the two fundamental frequencies are different, the system
A C will vibrate with a common frequency f such that
i1
B(i1+i2)l nH f0 150 3
f = nH fH = n0 f0 i.e., n = f = =
e 0 H 550 11
E L i1+i2 F i.e., the third harmonic of hydrogen and 11th harmonic of
mg oxygen or 6th harmonic of hydrogen and 22nd harmonic of
i2 oxygen will have same frequency. So the minimum common
B D frequency
R2 f = 3 ´ 550 = 1650 Hz Þ 50 (30 + n) = 1650 Þ n = 3
36. Ans : 5
[P.d. across parallel combinations remains the same] The capacity of a parallel plate capacitor in air is given by
Also, P1 = ei1 = 0.76 W
and P2 = ei2 = 1.2 W e0 A
C= ...(1)
i1 1.76 1.76 d
\ = Þ i1 = i2 … (ii) By introducing a slab of thickness t, the new capacitance C´
i2 1.2 1.2
becomes
The horizontal metallic bar L moves with a terminal velocity.
This means that the net force on the bar is zero. e0 A
C'= ...(2)
\ B (i1 + i2 ) = mg d '- t (1 - 1/ K)
mg 0.2 ´ 9.8 49 The charge (Q = CV) remains the same in both the cases.
Þ i1 + i2 = = = amp. Hence
B1 0.6 ´ 1 15
From (ii) and (iii) eo A eoA æ 1ö
= or d = d '- t ç 1 - ÷
d d '- t (1 - 1/ K) è Kø
1.76 49
i2 + i2 = Þ i2 = 2 amp.
1.2 15 d ' = d + 2.4 ´ 10-3 m, t = 3 mm = 3 × 10–3 m.
19 0.76 Substituting these values, we have
Þ i1 = amp. Þ e = = 0.6V
15 19 /15 æ 1ö
The induced emf across L due to the movement of bar L in a d = d + (2.4 ´ 10-3 ) - 3 ´10-3 ç1 - ÷
è Kø
magnetic field
e = BvT L æ 1ö
or ( 2.4 ´ 10 -3 ) = 3 ´ 10 -3 ç1 - ÷
e 0.6 è K ø
Þ vT = = = 1m/s
BL 0.6 ´ 1 Solving it, we get K = 5.
EBD_7801
98 Target IIT
37. (d) Take x-direction along initial velocity vector of spheres R
and y-direction in horizontal plane. cos -1 Þ f2
Just before collision the x-component of each of three X 2L + R2
bodies is same and equal to say ux.
f1 + f2 = 90 Þ cos (f1 + f2 ) = 0
\ From conservation of momentum
Þ tan f1 ´ tan f2 = 1
u
mu + mu = 2mvx + mvx + mvx or v x = XC XL
2 ´ = 1 Þ L = CR2
Let the speed of ball before collision be v. R R
\ From conservation of energy 43. (d) Potential inside shell 2 is V2 alone
and not V 1 + V 2 . Therefore
1 1 1 1 1
mu 2 + mu 2 = mv2x + mv 2 + mv 2 statement-1 is false. Potential is work V1
2 2 2 2 2 done per unit charge hence it is V2
scalar, therefore statement-2 is
3u correct.
Solving we get v =
2
44. (d) The current through solid metallic cylinder also
38. (b) produces magnetic field inside the cylinder. Hence
vx statement-1 is false.
45. (c) Tb – Tf = 105
v (100 + DTb) – (– DTf) = 105
v DTb + DTf = 5 ; (kb + kf) m = 5
2.37m = 5 ; m = 2.1
vx
æ w ö
çè ÷
342 ø
2.1 = ; w = 71.82g
Just before collision, speed of ball be v as shown. 0.1
\ vx = v cos q w » 72
vx u/2 1 46. (c) Edge not covered by atom = a – 2r
or cos q = = =
v 3u / 2 3 a
& in FCC unit cell ; r =
2 2
1
\ tan q =
2 a
Therefore, edge not covered = a – 2 × Þ 0.293a
Hence angle between velocity 2 2
Fraction of edge is not covered = 0.293
1
Vector = 2q = 2 tan -1 OH
2 O
39. (c) The speed of U-tube will be maximum when balls emerge HO OH
P.C.C.
out of U-tube with zero speed. At that instant from (1 mole)
47. (a) (2mole)
conservation of momentum, the speed of U-tube will
be ‘u’.
CH2OH C
V H O
40. (b) i woofer = f ­ X L ­ i woofer ¯ (P)
R 2 + X 2L
O O
V CH3
i tweeter = f ­ XC ¯ i tweeter ­
2
R + X 2C (i) CH3MgBr
Irms1 = Irms2 +
41. (d) (ii) H /H2O
1 1 1 C O C
Þ = Þf =
(wL) + R 2 2 2
(1 / wC) + R 2
2p LC H H O
O (R)
42. (c) Voltage is same for both. In capacitor current leads
(Q)
R
voltage by cos -1 =f,
XC2 + R 2 Lucas reagent
¾¾¾¾¾¾ ® White ppt. immediately
(S)
In inductor current lags voltage by
Since R is a 3° alc, gives Lucas test +ve immediately.
SOLUTIONS - MOCK TESTS 99
48. (b)
2
T2 - 250 = - T2 + 500
Et 3

Keq é 2ù é5ù
T2 ê1 + ú = 750 ; T2 ê ú = 750 ; T = 450 K
ë 3û ë3û 2
Et
Me Me
3
W = DU = 2 ´ R ´ 200 = 600R
(less stable) (more stable) 2
Keq > 1
5
DH = 2 ´ R ´ 200 = 1000R
49. (b) [Ma4 bc]n± : Geometrical Isomers are 2
51. (b, c)
a b (d) There is no formation of H• during photohalogenation
a b a a 52. (a, c, d)
M M Due to deliquescent nature of MgCl2.6H2O, FeCl3.6H2O,
a c a a ZnCl2.6H2O they get hydrolysed by their own water of crys-
a c tallization and hence they are made anhydrous by heating
cis trans in presence of dry HCl gas.
53. A-q; B-p, q, r; C-p, q, s; D-q
On replacing ‘a’ by ‘b’ in cis form :
(A) Here although two products are possible, only one is
b a a formed, the reaction is 100% stereoselective.
a b b b a b (B) The reaction involves the addition of Br 2 on trans-
M M M alkene to form 100% meso product. It is stereospecific
a c a c b c in the sense that it involves the addition on a
a a a stereoisomer (trans) and forms a stereoisomeric product
(meso). Further every stereospecific reaction is
Þ Three isomers as product by replacement. stereoselective. Since here cyclic bromonium ion is
On replacing ‘a’ by ‘b’ in Trans form : formed as an intermediate, the two bromine atoms add
in anti-manner.
b b
b a (C) This reaction is just the reverse of the above one, i.e. it
a a
involves the conversion of an optical isomer to
M M geometrical isomer through elimination reaction, it is
a a a b stereospecific, stereoselective and involves anti
c c elimination.
Þ They are same hence only one isomer product is (D) Here one of the stereoisomeric product (95%) is formed
obtained by replacement. from compound having no stereoisomerism, it is highly
50. (a, b, c) stereoselective.
Let the volume of B is VB as B ® C is an isochoric change, 54. A-p; B-q, r, s; C-p, s; D-q, s
Dipole-Dipole forces depend on temperature. Various forces
so VB = VC involved in solution of HBr in carbon tetrachloride are :
HBr – HBr ; dipole-dipole
nRTC 2 ´ R ´ ( 250 K )
Now VC = = HBr – CCl4; dipole-induced dipole
PC 1 ( bar )
CCl4 – CCl4; dispersion forces
Also TB = A ® B is an isothermal change Dipole-dipole types of forces are found in molecules having
dipole moment. In HBr, this is +d on H atom and – d on Br
2 ´ R ´ ( 500 K ) ´ 1 ( bar ) atom so HBr has dipole moment. There is thus a small dipole-
PB = = 2 bar..
2 ´ R ´ ( 250 K ) dipole force of attraction between adjacent HBr molecules.
Induced Dipole-induced dipole type of forces are observed
é nRT1 nRT1 ù in between non-polar molecule and a molecule having dipole
nCv,m [T2 - T1 ] = - Pext ê - ú moment. e.g. between CCl4 (non-polar) and HBr (molecule
ë P2 P1 û
having dipole moment)
Induced dipole-Induced dipole type of forces are found in
3 é RT R ´ 250 ù
R [T2 - 250] = -3 ê 2 - between molecules having no dipole moment. These are
2 ë 3 1 úû
also known as London dispersion forces.
EBD_7801
100 Target IIT
55. Ans : 4 H
Å
H2SO 4 N N
HCOOH ¾¾¾¾
® H2O + CO 59. (c) (a) H Pentavalent
a moles a moles
H Å H
H SO
H 2 C 2 O 4 ¾¾2 ¾
¾4 ® H 2 O + CO + CO 2 N N
(b) Å H Pentavalent
b moles b moles b moles
Total number of moles of gases formed = a + 2b Å
Moles of gas (CO2) absorbed by KOH = b H–N N–H
(c)

Hence, b = 1 (a + 2b ) [Volume µ moles)


6
a/b = 4 60. (a) (a) ® Anti aromatic
56. Ans : 1
(b) ® Non planar (tube shape)
h
Orbital angular momentum = l (l + 1)
2p
OH
Å

h h
[for s-orbital, l = 0] = 0(0 + 1) =
2p 2 p 61. (d) (a) (Aromatic)
D
57. Ans : 4
Fe2O3 + 2Al ® 2Fe + Al2O3 H
?
2 × 56 + 48 = 160 2 × 27 = 54
H+
Heat of reaction = 399 – 199 = 200 kcal [Al & Fe are in their (b) H2 + (Aromatic)
standard states]
160 54 –
Volume of Al =+ = 20 cc
5.2 2.7
(c) KH
(Aromatic)
160 D
Volume of Fe2O3 = = 30.77 cc
5. 2 62. (c) SO2 vapour may also be evolved from SO32– and
Total volume = 20 + 30.77 = 50.77 cc S2O32– ion by decomposition with dil. H2SO4
63. (c) Aq. suspension of Ag2CO3 Aq. suspension of Ag2SO3
200
\ Fuel value per cc = = 3.94 @ 4 kcal/cc Boil +H2O Boil
50.77
58. Ans : 5 Ag2O ¯ + CO2 ­ 2Ag ¯ + H2SO4
(Brown) (Black)
A+ B ¾¾
®C + D D
64. (b) SO32- + 3Zn + 8H + ¾¾
® H 2S ­ +3Zn 2 + + 3H 2O
Since the reaction is second order in A and second order D
HSO3- + 3Zn + 7H + ¾¾
® H 2S ­ +3Zn 2 + + 3H 2O
- dc
overall, so = k [A]x [B ] y = k [A]2 = k C 2 S2 - + Zn + 2H + ¾¾
D
® H 2S ­ + Zn
dt
65. (a) Statement-1 is true, statement-2 is true; Statement-2 is
a correct explanation for statement-1.
1é 1 1 ù
k= ê - ú Extent of hydration will be greater for Li+ due to greater
t êë C Co úû charge density on Li+ ion therefore Li+ ion will be big-
ger in size and mobility of Li +2 ion will be lower.
(Co = inital concentration; C = concentration after time t)
O O
+ –

1é 1 1 ù 1 1 66. (d) Cl C – C – H NaOH H – C – OH + CCl 3
t = t1/2 = - 3
ê ú= = min
k ë Co / 2 Co û kCo 0.500 × 0.4
O
= 5 min. –
H – C – O + CHCl3
130 + Passage Cum Matching Based Questions A-1

130 + PASSAGE CUM MATCHING BASED QUESTIONS

MATHEMATICS
Passage - 1
By appropriately matching the informations given in the three columns of the following table, give the answer of the question that follows.
Column 1 contains information about equations in two variables x and y where p, q and k are arbitrary constants.
Column 2 contains information about shape of the graphs of given equations in column-1.
Column 3 contains information about the area of the regions bounded by the graphs in column-2.
Column 1 Column 2 Column 3

2k 2
(I) |x – p| + |y – q| = k (i) Rectangle (P)
pq
(II) |x – p| – |y – q| = k (ii) Rhombus (Q) 2k2
(III) p | x | + q | y | = k (iii) Square (R) Undetermined
(IV) | x + y | = p and | x – y | = q (iv) Non-quadrilateral (S) 2pq

1. Which of the following options is the only correct combination?


(a) (II) (i) (S) (b) (III) (ii) (P)
(c) (I) (iii) (S) (d) (IV) (ii) (P)
1 1
2. For x - + y- = k , the area bounded by region is similar to
p q
(a) (I) (iii) (Q) (b) (II) (iv) (R)
(c) (III) (ii) (P) (d) (IV) (i) (S)
3. Which of the following options is the only incorrect combination?
(a) (I) (iii) (Q) (b) (III) (ii) (P)
(c) (II) (iv) (R) (d) (IV) (i) (Q)
Passage – 2
By appropriately matching the information given in the three columns of the following table, give the answer of the questions that
follows.
Column-1 contains information about the numbers to be formed.
Column-2 contains digits with condition to be used to form the numbers mentioned in the column-1.
Column-3 contains number of numbers formed mentioned in the column-1 using the digits mentioned in the column-2.
Column 1 Column 2 Column 3
(I) Four digit odd-numbers (i) 0, 1, 2, 3, 4, 5 (P) 216
(without repetition)
(II) Numbers greater than (ii) 0, 1, 2, 3, 5, 7 (Q) 72
1000 but less than 4000 (with repetition)
(III) Five digit numbers divisible by 3 (iii) 1, 2, 3 (R) 77
(with repetition)
(IV) Seven digit integers with sum of (iv) 0, 1, 2, 3, 4 (S) 720
the digits equal to 10 (without repetition)
4. Which of the following options is the only correct combination?
(a) (I) (ii) (R) (b) (III) (i) (P)
(c) (II) (iv) (S) (d) (IV) (ii) (Q)
5. Which of the following options is the only correct combination?
(a) (IV) (iii) (R) (b) (III) (ii) (Q)
(c) (II) (iv) (P) (d) (I) (i) (S)
6. Which of the following options is the only incorrect combination?
(a) (I) (ii) (S) (b) (III) (i) (P)
(c) (II) (iv) (R) (d) (IV) (iii) (R)
EBD_7801
A-2 130 + Passage Cum Matching Based Questions
Passage – 3
By appropriately matching the informations given in the three columns of the following table, give the answer of the question that follows.
Column 1 Column 2 Column 3
(I) x2 + 2(k + 1)x + (9k – 5) = 0 (i) Only negative roots (P) k>4
(II) x2 – 2(4k – 1)x + 15k2 – 2k – 7 = 0 (ii) No real roots (Q) k>6
(III) 2x2 – 2(2k + 1)x + k(k + 1) = 0 (iii) Absolute value of the (R) k Î R – [– 1, 0]
difference of roots of
the equation exceeds 3k
(IV) x2 + kx + 1 = 0 (iv) One root less than k and (S) 2<k<4
other root greater than k
7. Which of the following options is the only correct combination?
(a) (II) (ii) (S) (b) (I) (i) (Q)
(c) (III) (iv) (P) (d) (IV) (iii) (R)
8. Which of the following options is the only correct combination?
(a) (IV) (ii) (P) (b) (III) (ii) (Q)
(c) (II) (iv) (P) (d) (I) (i) (S)
9. Which of the following options is the only incorrect combination?
(a) (I) (i) (Q) (b) (III) (iv) (R)
(c) (II) (ii) (Q) (d) (IV) (ii) (P)
Passage – 4
Consider the lines L1, L2 and the planes P1, P2. Let ax + by + cz = d be the equation of the plane passing through the point of
intersection of lines L1 and L2, and perpendicular to planes P1 and P2.
By appropriately matching the information given in the three columns of the following table give the answer of the questions that
follows. Column 1, 2 and 3 contains equation of lines, equation of planes and values of a, b, c & d respectively.
Column 1 Column 2 Column 3
x -1 y z+3
(I) L1 : = = , (i) P1 : 7x + y + 2z = 3 (P) a = 5
2 -1 1
x-4 y+3 z +3
L2 : = = P2 : 3x + 5y – 6z = 4 b=4
1 1 2
c=–7
d=1
x - 2 y - 3 z -1
(II) L1 : = = , (ii) P1 : 2x + 5y + 3z = 4 (Q) a = 1
3 4 5
x - 4 y - 2 z -1
L2 : = = P2 : 5x + 3y + 7z = 3 b=–2
1 3 2
c=1
d=0
x -1 y - 0 z - 0
(III) L1 : = = (iii) P1 : x + 2y + 3z = 2 (R) a = 1
1 0 0
x - 0 y -1 z - 0
L2 : = = P2 : 2x + 3y + 4z = 4 b=–3
0 1 0
c=–2
d = 13
x - 2 y - 3 z -1
(IV) L1 : = = (iv) P1 : 5x + 4y + z = 2 (S) a = 2
5 4 3
x-3 y-4 z-2
L2 : = = P2 : 3x + 2y + 5z = 4 b=3
3 2 5
c=3
d=–4
130 + Passage Cum Matching Based Questions A-3

10. Which of the following options is the only correct combination?


(a) (I) (i) (R) (b) (II) (ii) (Q)
(c) (III) (iii) (P) (d) (IV) (iv) (S)
11. Which of the following options is the only correct combination?
(a) (II) (iii) (S) (b) (IV) (i) (Q)
(c) (III) (iv) (P) (d) None of these
12. Which of the following options is the only correct combination?
(a) (I) (iv) (R) (b) (II) (i) (S)
(c) (III) (iii) (Q) (d) (IV) (ii) (P)
Passage – 5
By appropriately matching the informations given in the three columns of the following table, give the answer of the question that follows.
Column 1 Column 2 Column 3
[Differential Equations] [Integrating factors (I.F.)] [Solutions of Differential equation]
dy
(I) (1 + x 2 ) + 2 xy - 4 x 2 = 0 (i) x / 1 - x2 (P) x = y3 + cy
dx e
dy 4 3
(II) ( x + 2 y3 ) =y (ii) e– x (1 + x) (Q) y (1 + x2) = x +c
dx 3
dy x - x / 1 - x2
(III) (1 + x ) - xy = 1 - x (iii) 2y2 (R) y = + ce
dx 1 - x2

dy y x + 1 - x2
(IV) + = (iv) 1 + x2 (S) y (1 + x) = x + cex
dx (1 - x 2 )3/2 (1 - x 2 )2
13. Which of the following options is the only correct combination?
(a) (I) (i) (R) (b) (II) (ii) (S)
(c) (III) (iv) (P) (d) (IV) (i) (R)
14. Which of the following options is the only correct combination?
(a) (IV) (iii) (R) (b) (II) (iv) (P)
(c) (III) (ii) (S) (d) (I) (i) (S)
15. Which of the following options is the only incorrect combination?
(a) (II) (iv) (R) (b) (III) (ii) (S)
(c) (I) (iv) (Q) (d) (IV) (i) (R)
Passage – 6
By appropriately matching the information given in the three columns of the following table, give the answer of the question that
follows. Column 1, 2 and 3 contain function f (x), value of n and information about continuity or differentiability of the function given
in column-1 for the given value(s) of n.
Column 1 Column 2 Column 3

ì 1
ï |x| | x | ³1
(I) f ( x ) = í (i) n = 1, 2 (P) Differentiable everywhere
ï C x2 + C | x | <1
î 1 2
where C1 + C2 = + n
ìï x 2 + 3x + 3 x £ 1
(II) f ( x) = í (ii) n = 5 (Q) Three points of non-differentiability
ïî nx + 2 x >1
(III) f (x) = 2 + 3 | n | sin x (iii) n = + 1 (R) Eleven points of discontinuity
x Î (0, p)
(IV) f (x) = | | | x | – 2 | + n | (iv) n = + 2 (S) One point of discontinuity
16. Which of the following options is the only correct combination?
(a) (II) (ii) (S) (b) (I) (i) (Q)
(c) (IV) (iii) (R) (d) (III) (iv) (P)
EBD_7801
A-4 130 + Passage Cum Matching Based Questions
17. Which of the following options is the only correct combination?
(a) (II) (iv) (P) (b) (IV) (iii) (R)
(c) (III) (ii) (Q) (d) (I) (iii) (P)
18. Which of the following options is the only incorrect combination?
(a) (I) (iii) (P) (b) (II) (iv) (Q)
(c) (IV) (i) (Q) (d) (II) (ii) (S)
Passage – 7
By appropriately matching the information given in the column 1, column 2 and column 3 in the given following table, give the answer
of the question that follows.
Column I Column II Column III

ì p 1
ï x sin x¹0
(I) f ( x) = í x (i) Neither continuous nor (P) {2}
ïî 0 x=0 derivable

ì 2 1
ïï 4 x + [2 x ] x, - 2 £ x < 0
(II) f ( x) = í (ii) Continuous but not differentiable (Q) (– 1, 0)
ï ax 2 - bx, 1
0£x<
ïî 2
(III) f (x) = [x2] + [– x]2 (iii) Continuous and differentiable (R) {0}
æ -1 1 ö
(IV) f (x) = cos p (| x | + [x]) (iv) Differentiable but not continuous (S) ç , ÷
è 2 2ø
Column 1 contains information about the functions.
Column 2 contains information about continuity and differentiability of functions given in column I.
Column 3 contains information about points/intervals where given function is either continuous or differentiable.
19. Which of the following options is the only correct combination?
(a) (II) (i) (S) (b) (III) (i) (P)
(c) (I) (ii) (Q) (d) (IV) (iv) (R)

ì p 1
ï x sin x¹0
20. For 0 < p < 1, check the continuity and differentiability at x = 0 of the function f (x) = í x
ïî 0 x=0

(a) Continuous but not differentiable


(b) Continuous and differentiable
(c) Differentiable but not continuous
(d) Neither continuous nor derivable
21. Which of the following options is the only incorrect combination?
(a) (I) (ii) (R) (b) (II) (iii) (S)
(c) (III) (i) (P) (d) (IV) (ii) (Q)

Passage – 8
By appropriately matching the information given in the three columns of the following table, give the answer of the questions that
follows. Column 1, 2 amd 3 contain function about, continuity and differentiability, intervals in which the given functions are either
differentiable or not.
Column 1 Column 2 Column 3
(I) f : R ® R (i) Twice differentiable (P) R – {0}
f (x) = 15 – | x – 10 |
g (x) = f (f (x))
130 + Passage Cum Matching Based Questions A-5

(II) f (x) = x | x | (ii) Thrice differentiable (Q) R – (– 1, 1)


(III) f (x) = x2 | x | (iii) Differentiable (R) (– 1, 1)
ìï (sin -1 x )2 cos (1/ x) x ¹ 0
(IV) f (x) = í (iv) Not differentiable (S) {5, 10, 15}
ïî 0 x =0
22. Which of the following options is the only correct combination?
(a) (I) (iii) (Q) (b) (II) (i) (R)
(c) (IV) (iv) (S) (d) (III) (ii) (P)
23. Which of the following options is the only correct combination?
(a) (I) (iii) (Q) (b) (II) (i) (P)
(c) (III) (iv) (R) (d) (IV) (i) (S)
24. Which of the following options is the only incorrect combination?
(a) (II) (i) (P) (b) (I) (iv) (S)
(c) (III) (ii) (P) (d) (IV) (iii) (S)
Passage – 9
By appropriately matching the informations given in the three columns of the following table, give the answer of the question that follows.
Column 1 contains information about the functions.
Columns 2 contains information about continuity of functions given in column 1 at some point.
Column 3 contains information about a and b for which function in column 1 is continuous at the point given in column 2.
Column 1 Column 2 Column 3

ì
ï -p
(cos x - sin x )cosec x < x<0
ï 2
ï 1
(I) f (x) = í a x=0 (i) continuity at x = – 2 (P) a= b=4
ï 2
ï e1/ x + e2 x + e3/ x p
0< x<
ï ae 2/ x 3/ x
+ be 2
î

ì ab
ï (1 + | cos x |)|cos x| , p < x < 3p
ï 2
ï a
ïe 3p
(II) f (x) = í b x= (ii) continuity at x = 0 (Q) a = 2 = b
ïe 2
ï cot 2 x
ï cot 8 x 3p 25p
ïe < x<
î 2 16

ì ae1/(| x + 2| - 1
ï -3< x < - 2
ï 2 - e1/(|x + 2|)
ï p 1
(III) f (x) = í b x=-2 (iii) continuity at x = (R) a= b=e
ï 2 e
ï sin çæ x - 16 ÷ö - 2 < x < 0
4
ï ç x5 + 32 ÷
î è ø

ì 1 - sin 3 x p
ï x<
2 2
ï 3 cos x
ï p
ï 3p æ2ö
a x=
(IV) f (x) = íï 2 (iv) continuity at x = (S) a = - sin ç ÷ = b
2 è5ø
ï b (1 - sin x ) p
ï x>
ïî (p - 2 x )2 2
EBD_7801
A-6 130 + Passage Cum Matching Based Questions
25. Which of the following options is the only correct combination?
(a) (I) (ii) (R) (b) (II) (iii) (S)
(c) (III) (i) (P) (d) (IV) (iv) (Q)
26. Which of the following options is the only correct combination?
(a) (I) (ii) (P) (b) (II) (iii) (R)
(c) (III) (i) (S) (d) (IV) (iv) (Q)
27. Which of the following options is the only incorrect combination?
(a) (II) (iv) (Q) (b) (III) (i) (S)
(c) (I) (ii) (R) (d) (IV) (iii) (P)
Passage – 10
By appropriately matching the information given in the three columns of the following table, give the solution of the questions that
follows.
Column 1 contains information about equations having complex variables.
Column 2 contains information about the value of k.
Column 3 contains information about the locus of the complex variable.
Column 1 Column 2 Column 3

æ 2z + 1 ö
(I)Im ç ÷=k (i) k2 > 4 (P) Hyperbola
è iz + 1 ø
(II) | z – i | + | z + i | = k (ii) k=1 (Q) Ellipse
(III) | z + i | – | z – i | = k (iii) –2<k<2 (R) Straight line
æ | z -1| + 4 ö
(IV) log1/2 ç ÷>k (iv) k=–2 (S) Circle
è 3 | z - 1| - 2 ø
2
where | z – 1 | ¹
3
28. Which of the following options is the only correct information?
(a) (I) (iv) (R) (b) (II) (iii) (P)
(c) (III) (ii) (Q) (d) (IV) (i) (S)
29. Which of the following options is the only correct combination?
(a) (I) (iii) (Q) (b) (II) (i) (R)
(c) (III) (i) (S) (d) (II) (i) (Q)
30. Which of the following options is the only incorrect combination?
(a) (I) (iv) (R) (b) (II) (i) (Q)
(c) (III) (ii) (Q) (d) (III) (iii) (P)
Passage – 11
By appropriately matching the information given in the three columns of the following table, give the answer of the questions that
follows.
Column 1, 2 and 3 contain informations about functions, domain of the functions and codomain of the functions respectively.
Column 1 Column 2 Column 3

1
(I) f (x) = , a > 0, a ¹ 1 (i) R– {0} (P) (1, ¥)
log a x
1
(II) f (x) = (ii) R–I (Q) R– {0}
[ x]
1
(III) f (x) = (iii) R+ – {1} (R) R+
{x}
1 ì1 ü
(IV) f (x) = (iv) R – [0, 1) (S) í , n Î I - {0}ý
|x| în þ
130 + Passage Cum Matching Based Questions A-7

31. Which of the following options is the only correct combination?


(a) (II) (i) (P) (b) (I) (iv) (R)
(c) (III) (ii) (P) (d) (IV) (iii) (Q)
32. Which of the following options is the only in incorrect combination?
(a) (II) (i) (Q) (b) (IV) (iii) (P)
(c) (I) (iv) (R) (d) (IV) (ii) (S)
33. Which of the following options is the only incorrect combination?
(a) (III) (iii) (P) (b) (IV) (ii) (S)
(c) (II) (i) (S) (d) (I) (iv) (R)
Passage – 12
By appropriately matching the information given in the three columns of the following table, give the answer of the questions that
follows.
Column 1 contains information about inverse trigonometric equations.
Column 2 contains information about constraints on x.
Column 3 contains information about the value of x.
Column 1 Column 2 Column 3

(I) tan - 1 x ( x + 1) + sin - 1 x 2 + x + 1 (i) 0<|x|< 2 (P) 1

p
=
2
(II) sin [cot– 1 (1 + x)] = cos (tan– 1 x) (ii) –¥<x<¥ (Q) –1
-1
æ x 2 x3 ö 25
(III) sin çç x - + ... ÷ + (iii) –¥<x<0 (R)
è 2 3 ÷ø 23
æ x4 x6 ö p
cos- 1 ç x 2 - + ... ÷ =
ç 2 4 ÷ø 2
è
23é æ n öù -1
(IV) å êcot - 1 ç 1 + å 2k ÷ ú = x
ç k = 1 ÷ú
(iv) 1 < x < 2 (S)
n = 1 êë
2
è øû
34. Which of the following is the only correct option?
(a) (III) (i) (P) (b) (II) (iv) (S)
(c) (IV) (iii) (R) (d) (I) (ii) (Q)
35. Which of the following is the only correct combination?
(a) (I) (iii) (P) (b) (IV) (i) (Q)
(c) (II) (ii) (S) (d) (III) (iv) (R)
36. Which of the following is the only incorrect option?
(a) (I) (iii) (Q) (b) (II) (ii) (S)
(c) (III) (i) (P) (d) (IV) (iv) (S)

Passage – 13
By appropriately matching the informations given in the three columns of the following table, give the answer of the question that follows.
Column 1 Column 2 Column 3

26
(I) P (AC) = 0.3, P (b) = 0.4 (i) P (A Ç B Ç CC) + P (A Ç BC Ç C) (P)
31
P (A Ç BC) = 0.5 + P (AC Ç B Ç C) + P (A Ç B Ç C)
1 4 3
(II) P (a) = , P (b) = , (ii) P [B/ (A Ç BC)] (Q)
5 5 4
7
P (c) =
100
EBD_7801
A-8 130 + Passage Cum Matching Based Questions
1 1 1
(III) P (a) = ; P (b) = = P (c) (iii) P (B/ C) (R)
2 4 4
5
(IV) If A, B, C are pairwise independent (iv) P (A È B È C) (S)
31
events and
1 1 1
P (a) = , P (b) = , P (c) =
2 3 4

37. Which of the following options is the only correct combination?


(a) (I) (ii) (R) (b) (II) (iii) (P)
(c) (III) (iv) (S) (d) (IV) (i) (Q)
38. Which of the following options is the only correct combination?
(a) (I) (iii) (R) (b) (II) (ii) (Q)
(c) (III) (i) (S) (d) (II) (iii) (P)
39. Which of the following options is the only incorrect combination?
(a) (I) (ii) (R) (b) (III) (i) (S)
(c) (II) (iii) (P) (d) (IV) (iv) (Q)

PASSAGE-14
By appropriately matching the information given in the three columns of the following table, give the answer of the questions that
follows.

Column 1 Column 2 Column 3


(I) x = 2y + 1 is
2
(i) Not injective but (P) R ® [0, ¥)
surjective for

(Q) R ® éê - , ¥ ö÷
1
(II) y = | x | + 2 is (ii) Neither injective nor
ë 2 ø
surjective for
1
(III) y = x + is (iii) Injective and surjective for (R) R ® R
x
(IV) y2 = 2x – 4 is (iv) Injective but not (S) R – {0} ® R ~ (– 2, 2)
surjective for
40. Which of the following options is the only correct combination?
(a) (I) (ii) (P) (b) (II) (iii) (S)
(c) (III) (i) (S) (d) (IV) (iv) (Q)
41. Which of the following options is the only correct combination?
(a) (I) (iii) (S) (b) (II) (ii) (R)
(c) (III) (iv) (P) (d) (IV) (i) (Q)
42. Which of the following options is the only incorrect combination?
(a) (I) (i) (Q) (b) (II) (ii) (R)
(c) (III) (i) (S) (d) (IV) (i) (P)

PASSAGE-15
By appropriately matching the information given in the three columns of the following table, give the answer of the questions that
follows.
Consider the hyperbola H : x2 – y2 = 1 and a circle S with center N (x2, 0). Suppose that H and S touch each other at a point P (x1, y1)
with x1 > 0 and y1 > 0. The common tangent to H and S at P intersects the x-axis at point M. If (l, m) is the centroid of the triangle PMN,
then
Column 1 Column 2 Column 3

dl 1
(I) = (i) 1+ (P) Provided x1 > 1, – ¥ < y1 < ¥
dx1 3x12
130 + Passage Cum Matching Based Questions A-9

dm 1
(II) = (ii) 1- (Q) Provided x1 > 1, y1 > 1
dx1 3x12

dm x1
(III) = (iii) (R) Provided x1 > 1, y1 > 0
dy1 3 æç x12 - 1 ö÷
è ø

dl 1
(IV) = (iv) (S) Provided – ¥ < x1 < ¥, y1 > 0
y1 3

43. Which of the following options is the only correct combination ?


(a) (I) (iv) (R) (b) (II) (iii) (P)
(c) (III) (ii) (Q) (d) (IV) (i) (S)
44. Which of the following options is the only correct combination?
(a) (II) (iii) (R) (b) (III) (i) (S)
(c) (IV) (iv) (Q) (d) (I) (ii) (R)
45. Which of the following options is the only correct combination?
(a) (I) (ii) (Q) (b) (II) (iii) (R)
(c) (III) (iv) (S) (d) (IV) (i) (P)

PHYSICS
PASSAGE-1
If dimensions are given, physical quantity may not be unique as two different physical quantities may have same dimensional formula.
Column I and III shows physical quantities while column II gives dimensional formula with respect to M, L and T fundamental
physical quantities mass, length and time respectively.
Column I Column II Column III
I. Torque (i) M0L0T0 (P) Wave number
II. Power of lens (ii) M0L–1T0 (Q) Moment of force
III. Plane angle (iii) M0L0T–1 (R) p
IV. Angular frequency (iv) M1L2T–2 (s) Angular velocity
1. Which of the following shows the correct matching in terms of dimensionally simillar quantity as x in ex.
(a) (I) (i) (R) (b) (II) (iii) (S) (c) (III) (i) (R) (d) (IV) (iii) (S)
2. Which of the follwoing will remain unaffected if mass and length of the body changes?
(a) III (i) R (b) IV (iii) S (c) I (iv) Q (d) Both (a) and (b)
3. The dimensions of physical quantity ‘X’ in the equation
1 'X'
= is same as:
Angular frequency Angular velocity
(a) II (i) R (b) IV (i) R (c) III (i) R (d) I (i) P
PASSAGE-2
1 2
Potential energy of spring is given by kx and for a projectile it is mgh. Column II and III shows increase and decrease of potential
2
and Kinetic energy of the system respectively in different situations of spring/projectile. (Assume air resistance negligible and surface
are smooth)
Column I Column II Column III
I. Projectile during upward journey (i) Potential energy increases (P) Kinetic energy increases by mgh
1 2
bykx
2
II. Projectile during downward journey (ii) Potential energy decreases (Q) Kinetic energy decreases
1
by k x2 by mgh
2
EBD_7801
A-10 130 + Passage Cum Matching Based Questions
III. Spring compressed to length x (iii) Potential energy decreases (R) Kinetic energy increases by
1 2
by mgh kx
2
IV. Spring elongated upto length x (iv) Potential energy increases (S) Kinetic energy decreases
1 2
by mgh by k x
2
4. Which of the following options is correct for the projectile during the upward journey?
(a) I (iv) Q (b) I (iii) Q (c) I (iii) P (d) Both (a) and (c)
5. Which of the following is the correct matching for the spring compressed to length x?
(a) III (i) Q (b) I (ii) P (c) III (i) S (d) III (ii) R
6. Which of the following is matched wrongly?
(a) I (iii) P (b) II (iii) P (c) IV (i) S (d) I (iv) Q
PASSAGE-3
The acceleration of a particle as measured from an inertial frame is given by the vector sum of all the forces acting on the particle
divided by its mass. Column I shows the motion of blocks (m 1, m2, m3) in contact/connected by massless string on a smooth horizontal
surface. If f1 is the contact force between body m1 and m2, f2 between body m2 and m3, T1 be the tension in the string between body
m1 and m2, T2 between body m2 and m3. Column II and III shows different values of f1, f2, T1 and T2 in different situations shown in
column I.
Column I Column II Column III

(m 2 + m3 ) F (m1 + m 2 )F
I. F m1 m2 m3 (i) T1 = (P) f2 =
m1 + m 2 + m3 m1 + m 2 + m3

F
m1F m3 F
II. m1 m2 m3 (ii) T1 = (Q) f2 =
m1 + m 2 + m3 m1 + m 2 + m3

T1 T2 F
m1 m2 m3 m1F (m1 + m 2 ) F
III. (iii) f1 = (R) T2 =
m1 + m 2 + m3 m1 + m 2 + m3

T1 T2 (m2 + m3 ) F m 3F
IV. F m1 m2 m3 (iv) f1 = (S) T2 =
m1 + m 2 + m3 m1 + m 2 + m3

7. If the acceleration of body m1, m2 and m3 respectively shown in situation Column I (I) are (2, 2, 2) then the correct matching is
[Take m1 = 1 kg, m2 = 2 kg, m3 = 3 kg, F = 12 N]
(a) I (iv) Q (b) IV (iv) P (c) III (ii) P (d) I (iii) Q
8. If the value of T1 and T2 shown in the situtation column I (IV) are 10N and 6N respectively then find the correct matching.
[Take m1 = 1 kg, m2 = 2 kg, m3 = 3 kg, F = 12 N].
(a) II (i) R (b) IV (i) S (c) IV (iii) S (d) III (ii) S
9. Which of the following does not show the correct matching?
(a) II (iii) P (b) III (ii) R (c) I (iv) Q (d) IV (i) R
PASSAGE-4
If moment of inertia of a body is given by I = MK2, then radius of gyration is given by K = [I/ M]1/2. Column II and III shows the
moment of inertia about an-axis perpendicular to the plane passing through centre of mass and radius of gyration of different objects
respectively.
Column I Column II Column III

M(L2 +1) L
I. Square plate of perimeter = 4 × L (i) I= (P)
12 6
M L2 L
II. Thin rod of length ‘L’ (ii) I= (Q)
12 2 3
130 + Passage Cum Matching Based Questions A-11

III. Cube of each side length ‘L’ (iii) M L2 (R) L2 + 1


I=
6 2 3
ML2 L
IV. Rectangular plate of length ‘L’ and (iv) I= (S)
3 2
breadth 1 unit
10. Out of the four, two objects gives the same moment of inertia about an axis passing through its centre of mass and perpendicular
to the plane. Which of the following options is correct?
(a) III (iii) P (b) I (iii) P (c) IV (i) S (d) Both (a) and (b)
11. Which of the following shows the wrong matching ?
(a) III (iii) P (b) IV (i) R(c) II (ii) Q (d) I (iii) Q
12. If mass and length of each side of a cube is increased 2 times, then its moment of intertia and radius of gyration increases 8 times
and 2 times respectively. The correct matching for the cube is
(a) III (ii) Q (b) III (iv) S (c) III (iii) P (d) III (ii) R
PASSAGE-5
1/2
éI ù
If moment of inertia of an object about an axis is given by I = MK2, then radius of gyration is given by K= ê ú . Column II and III
ëMû
represents moment of inertia along the tangent (in the plane of the object) and radius of gyration repectively of different objects of
Mass M and radius R.
Column I Column II Column III

5 3
I. Solid sphere (i) MR 2 (P) R
3 2
3 5
II. Disc (ii) MR 2 (Q) R
2 2
7 5
III. Thin spherical shell (iii) MR 2 (R) R
5 3
5 7
IV. Ring (iv) MR 2 (S) R
4 5
13. The correct matching for moment of inertia and radius of gyration of solid sphere of mass M and radius R along the tangent is
(a) I (i) Q (b) (I) (iii) (S) (c) (III) (ii) (R) (d) (I) (iv) (S)
MR 2
14. What is the correct matching for the disc, if the moment of inertia of the disc about diameter in the plane of the disc is
4
(a) II (ii) Q (b) II (iv) Q (c) II (i) R (d) II (iii) S
15. If radius (R) and mass (M) of thin spherical shell are 15m and 2kg respectively, then the moment of inertia and radius of
gyration of the shell along the tangent are 50 kg m2 and 5m respectively. Which of the following is correct for spherical shell?
(a) III (i) R (b) I (iii) Q (c) III (ii) R (d) IV (iii) S
PASSAGE-6
A satellite is transferred to a higher orbit r2 (r2 > r1) as shown in figure. Column I shows different physical quantities that may be
affected while column III gives the proportionality (or relation) with r (radius).
Column I Column II Column III
1
I. Orbital velocity (i) Increases (P) µ
r r1
1 M
II. Time period (ii) Decreases (Q) µ r2 Earth
r
III. Angular momentum (iii) Remains constant (R) µ r3 2
IV. Kinetic energy (s) µ r
16. When the satellite is transferred from r1 = r to r2 = 2r then its time period will increased by 2Ö2 times. The correct matching for
time period will be
(a) II (iii) R (b) II (i) R (c) II (iii) R (d) II (iv) R
EBD_7801
A-12 130 + Passage Cum Matching Based Questions
17. What is the correct matching for the kinetic energy of the satellite?
(a) IV (ii) P (b) IV (ii) R (c) IV (iii) S (d) IV (iii) P
18. Which of the following shows the incorrect matching?
(a) I (ii) Q (b) III (i) S (c) II (i) R (d) IV (iii) P
PASSAGE-7
First Law of Thermodynamics states that, “If certain quantity of heat dQ is added to a system a part of it is used in increasing the
internal energy by dU and dW in performing external work i.e dQ = dU + dW.” The following columns show (Internal energy/Work
done/Heat given or taken out) in the four different thermodynamic processes. Here P = pressure, v = volume. dT = change in tempera-
ture.
Column I Column II Column III
I Isothermal process (i) dU = 0 (P) dQ = 0
V2
II Adiabatic process (ii) dU = nCv dT (Q) dQ = 2.303 nRT V1
log 10
III Isobaric process (iii) dW = 0 (R) dQ = nCp dT
IV Isochoric process (iv) dW = P(V2 – V1) (S) dQ = nCv dT
19. For the process volume remains constant, which of the following options is corrcet?
(a) IV (iii) S (b) I (iii) Q (c) III (i) P (d) II (iv) P
20. Which of the following shows the correct match for the process pressure remains constant?
(a) III (iv) R (b) IV (iii) R (c) I (i) R (d) III (i) S
21. What is the correct match for the process temperature remains constant?
(a) IV (i) R (b) I (i) Q (c) II (ii) P (d) III (ii) S
PASSAGE-8
According to law of equipartition of energy, if we deal with a large no of particles in thermal equilibrium to which we can apply
1
Newtonion mechanics, the energy associated with each degree of freedom has the same average value (i.e. KT ) and this value
2
depends on temperature. Column I, II, & III gives relation between degree of freedom (of monoatomic, di & poly atomic gas), C v and
g.
Column I Column II Column III
7
I. For monoatomic gas (i) R (P) g = 1.67
2
f=3
II. For diatomic gas (ii) 3R (Q) g = 1.33
(molecule translate
& rotate only) f = 5
5
III. For polyatomic gas (iii) R (R) g = 1.4
2
if, f = 6
3
IV. For diatomic gas (molecule translate, (iv) R (S) g = 1.29
2
rotate as well as vibrate also) f = 7
9R
22. If be the value of Cp for diatomic gas when we consider translational, rotational as well as vibrational motion of gas
2
moleclues, the correct matching satisfying the above condition is
(a) IV (ii) R (b) IV (iii) Q (c) IV (i) S (d) IV (iii) P
23. Which of the following shows the correct match for the polyatomic gas?
(a) III (i) P (b) III (ii) Q (c) III (ii) S (d) III (ii) P
24. Which of the following does not show the correct matching?
(a) II (iii) R (b) IV (i) S (c) II (iv) Q (d) I (iv) P
130 + Passage Cum Matching Based Questions A-13

PASSAGE-9
Let us consider an organ pipe (may be open or closed) of length ‘l’. V is the velocity of sound. Column I represents the harmonics,
Column II represents the frequency while column III represents the corresponding wavelength.
Column I Column II Column III
3V
I. First harmonic for open organ pipe (i) u= (P) l = 2l
4l
V 4l
II. Third harmonic for open organ pipe (ii) u= (Q) l=
2l 3
3V 2l
III. First harmonic for closed organ pipe (iii) u= (R) l=
2l 3
V
IV. Third harmonic for closed organ pipe (iv) u= (S) l = 4l
4l
25. If velocity of sound in air is 330 m/s and the length of the open organ pipe is 0.30 m then the wavelength and the frequency for
the first harmonic or fundamental tone is 0.6m and 550 Hz. The correct matching satisfying the above condition is
(a) I (ii) P (b) III (iv) Q (c) III (iv) S (d) I (ii) S
26. Which of following shows the correct match for the first overtone of a closed organ pipe?
(a) III (ii) R (b) IV (i) Q (c) II (iv) S (d) I (iii) R
27. Which of the following does not show the correct relation?
(a) I (ii) P (b) II (iii) R (c) III (iv) Q (d) IV (i) Q
PASSAGE-10
When a source of a sound or observer or both are in motion relative to each-other, there is an apparent change in frequency of sound
heard by the observer. Column I gives the apparent change in frequency, column II gives the source status while column III gives the
observer status [not in synchronized order].
v = velocity of sound in air, vs = velocity of source, v = velocity of observer, n = original frequency
Column I Column II Column III

æ v ö
I. υ1 = υ 0 ç ÷ (i) Source is at rest (P) Observer is at rest
è v - vs ø

æ v + v0 ö
II. υ 2 = υ0 ç ÷ (ii) Source is moving (Q) Observer is moving
è v ø
towards observer towards source

æ v + v0 ö
III. υ3 = υ 0 ç ÷ (iii) Source is moving (R) Observer is moving
è v - vs ø
away from observer away from the source

æ v - v0 ö
IV. υ 4 = υ0 ç ÷ (iv) Source and observer both (S) Source and observer both
è v + vs ø
moving towards each other moving away from each other
in vaccum in vaccum
28. Which of the following is wrongly matched?
(a) I (ii) P (b) II (i) Q (c) III (ii) Q (d) IV (ii) P
29. Two engines cross each other travelling in opposite direction at 72 km/hour. One engine sounds a whistle of frequency 1088 Hz. If the
frequency as heard by an observer on the other engine just before crossing is 1224 Hz. The correct option satisfying the condition is:
(a) IV (ii) Q (b) III (ii) Q (c) IV (iii) P (d) III (iii) R
EBD_7801
A-14 130 + Passage Cum Matching Based Questions
30. For the detail given in the above question, the frequency as heard by an observer on the other engine just after crossing is 967.11
Hz. The correct matching is:
(a) II (iii) Q (b) II (iii) R (c) IV (iii) R (d) I (iii) S
PASSAGE-11
If the charge distribution is continuous, then the electric field strength at any point may be calculated by dividing the charge into
r kdq r
infinitesimal element, and dE = r2
then integrate dE within certain limit. Column II and column III gives electric field at diffrent

points due to various charge.


[x =distance from centre along central axis, R = radius, Q = charge on the body]
Column I Column II Column III

KQ
I. Circular ring (i) Ein = 0 (x < R) (P) E out = for x > R
x2

Qx
II. Circular disc (ii) Ecentre = 0 (Q) E =k for x ³ 0
(R 2
+x )
2 3/2

KQx 2 KQ é x ù
III. Solid non-conducting (iii) Ein = for 0 £ x £ R (R) E= ê1 - ú for x > 0
R3 R 2 êë x 2 + + R 2 úû
sphere
2KQ 2KQx
IV. Spherical shell (iv) Ein = 3 (S) E=
R R4
31. Which of the following shows the correct matching for 0 < x
(a) (IV) (iii) Q (b) (IV) (i) P (c) (III) (iii) P (d) Both (b) and (c)
Q
32. If 6 3πε R 2 is the maximum electric field due to the circular ring having uniformly distributed charge Q, then correct match-
0
ing is:
(a) I (ii) Q (b) I (iii) R (c) IV (iii) Q (d) I (ii) S
33. The correct matching for the disc is:
(a) II (i) Q (b) II (ii) R (c) II (iii) Q (d) II (iv) P
PASSAGE-12
Work done by external agent in rotating the magnet is stored as potential energy of the field-magnet system. Take Uq= 0 [for q = 90º].
uur ur
Column I shows different sitituation of magnet in the field, column II shows the angle between M and B while column III shows the
uur ur
potential energy stored. M = magnetic moment, B = magnetic field.
Column I Column II Column III

B
I. S (i) θ = 60° (P) U = 0

B
II. S N (ii) θ = 0° (Q) U = – MB
130 + Passage Cum Matching Based Questions A-15

B
III. N S (iii) θ =180° (R) U = + MB

N
B +MB
IV. θº (iv) θ = 90° (S) U =
S 2

34. Which of the folllowing gives the correct combination for maximum protential energy stored?
(a) I (i) (P) (b) III (iii) (R) (c) II (ii) (R) (d) IV (i) (S)
35. Which of the following gives the correct combination for minimum potential energy stored?
(a) I (iv) (P) (b) II (ii) (Q) (c) III (iii) (R) (d) IV (i) (S)
36. If MB (cosq0 – cosq) be the work done by the external agent in changing the angle from θ0 to θ in the field – magnet system as
shown in the situation column I (IV), the correct matching for q = 60º is
(a) IV (i) S (b) IV (i) P (c) III (i) Q (d) II (i) Q
PASSAGE-13
Alternating current in a circuit may be controlled by resistance, inductance and capacitance. Column II and Column III represents the
phasor diagram and impedance of an AC-circuit containing different elements respectively. The applied emf (E) and current produced
(I) may be represented as
E = E0 sin wt and I = I0 sin (wt + f) with I0 = E0 /Z (Z = impedance)
Here ‘X’ and ‘Y’ have different meaning with different units for different circuits shown in the column I.
Column I Column II Column III
X Resistance
IX
p/2 1
I. (i) VX
(P) Z =
wX
~
E = E0 sin w t
Resistor only

X Capacitance

II. (ii) (Q) Z = X


~ I E
E = E0 sin w t
Capacitor only

X Resistance Y Inductance
VX
I
f
III. (iii) (R) Z= x 2 + w 2Y 2
VY
~ E
E = E0 sin w t
L-R series circuit

X Resistance Y Capacitance
E
VY
1
IV. (iv) f (S) Z = x2 +
I w Y2
2
~ VX
E = E0 sin w t
C-R series circuit
EBD_7801
A-16 130 + Passage Cum Matching Based Questions

E
37. If the potential difference across the capacitor of capacitance Y unit shown in the C-R series circuit is then the
(X Yw )2 + 1

correct matching for the circuit is


(a) IV (iii) Q (b) IV (iii) S (c) III (iv) S (d) IV (ii) P
38. Which of the following shows the correct matching for L-R series circuit?
(a) III (iii) R (b) III (iv) R (c) III (i) P (d) III (ii) Q
39. Which of the following is wrongly matched?
(a) I (ii) Q (b) II (i) P (c) IV (iv) R (d) IV (iii) S
PASSAGE-14
A lens of focal length f is cut into two equal parts as shown in figure.

or ,

Column I shows different combinations of these two parts. Column II gives equivalent focal length of the combination and column III
the corresponding power.
Column I Column II Column III

I. (i) ¥ (P) 1/2f

II. (ii) f/2 (Q) 0

III. (iii) f (R) 1/f

IV. (iv) 2f (S) 2/f

40. Focal length of each part in case (a) and case (b) are respectively f and 2f the correct

matching is
(a) III (iii) R (b) III (ii) R (c) III (i) R (d) III (ii) Q
41. If focal length of the complete lens is f = 10cm, then focal length of the combination shown in figure Column I (IV) is 10cm.
Which of the following is the correct matching satisfying the above condition?
(a) III (iii) R (b) IV (i) R (c) IV (ii) S (d) IIII (i) Q
42. Which of the following options shows the correct matching?
(a) I (ii) S (b) II (i) Q (c) III (iv) R (d) Both (a) and (b)
130 + Passage Cum Matching Based Questions A-17

PASSAGE-15
Following three columns I, II and III show the different characteristics of various optical instruments, where f0= focal length of
objective fe = focal length of eye piece, m = magnification and D = least distance of distinct vision.
Column I Column II Column III

f0
I. Astronomical telescope (i) length of telescope = f0 – fe (P) M = f
e

when image formed at infinity

f0
II. Galilean telescope (ii) length of telescope = f0 + fe (Q) M = – f
e

When image formed at infinity

æ Dö
III. Compound microscope (iii) used when, object is placed (R) M = ç1 + ÷ When image formed at ‘D’.
è f ø

between optical centre and focus

V0 D
IV. Simple microscope (iv) used when, object is placed (S) M = - u f When image formed at infinity
0 e

just beyond the principal focus


43. The correct matching for the length of the astromonical telescope and its magnification is
(a) I (iii) Q (b) I (i) P (c) I (i) S (d) I (ii) Q
44. Which of the following shows the correct match for the simple microscope?
(a) IV (iv) S (b) IV (iii) R (c) IV (ii) Q (d) IV (iii) S

9f0
45. For a hypothetical case, when f0 = 10fe, length and magnification of Galilean telescope are and +10 respectively. The
10
correct match for Galilean telescope is:
(a) II (i) R (b) II (ii) S (c) II (i) P (d) II (ii) Q

CHEMISTRY

PASSAGE-1
Some reactions are given in Column I and their n-factor and strength is given in Column II & III respectively.

Column I Column II Column III


(I) 9.8% H2SO4 by weight (d = 1.8 g mL–1) (i) n-factor = 3 (P) 3.6 N
(II) 9.8% H3PO4 by weight (density = 1.2 g mL–1) (ii) n-factor = 1 (Q) 1.2 M
(III) 1.8 NA molecules of HCl in 500 mL (iii) n-factor = 2 (R) 1.8 equiv.
(IV) 250 mL of 4 N NaOH + 250 mL of 1.6 M Ca(OH)2 (iv) n-factor = 4 (S) 1.10 m
1. For 9.8% H2SO4 given in column I, the only correct combination is
(a) (I) (iv) (P) (b) (I) (ii) (Q) (c) (I) (iii) (S) (d) (I) (i) (R)
2. For 1.8 NA molecules of HCl given in column I, the only correct combination is
(a) (III) (i) (P) (b) (III) (ii) (R) (c) (III) (iii) (Q) (d) (III) (ii) (S)
3. For 9.8% H3PO4 given in column I, the only correct combination is
(a) (II) (i) (Q) (b) (II) (iii) (S) (c) (II) (ii) (P) (d) (II) (i) (R)
EBD_7801
A-18 130 + Passage Cum Matching Based Questions
PASSAGE-2
Column I, II & III contain different salts, hydrolysis constant and pH of solution respectively.
Column I Column II Column III
Kw 1
(I) KCl, NaNO2, Na2SO4, BaCl2 etc (i) Kh = (P) pH = 7 + (pK a + log C)
Kb 2

1
(II) CH3COONa, Na2CO3, KCN etc (ii) do not undergo hydrolysis (Q) pH = 7 + (pK a - pK b )
2
Kw
(III) NH4Cl, ZnCl2, FeCl3 etc (iii) Kh = (R) pH = 7
Ka

h2 1
(IV) CH3COONH4, NH4CN etc (iv) Kh = (S) pH = 7 - (pK b + log C)
(1 - h) 2
2

4. For the salts of strong acid & strong base the only correct combination is
(a) (III) (ii) (Q) (b) (IV) (ii) (P) (c) (I) (ii) (R) (d) (I) (iv) (S)
5. For the salts of weak base and strong acid the only correct combination is
(a) (III) (i) (S) (b) (I) (i) (Q) (c) (IV) (ii) (P) (d) (III) (iv) (Q)
6. For the salts of weak acid and weak base the only correct combination is
(a) (II) (i) (R) (b) (III) (ii) (S) (c) (IV) (iii) (R) (d) (IV) (iv) (Q)
PASSAGE-3
Column I contains some reactions and Column II & III contains reagents and intermediates involved in corresponding reactions
respectively.
Column I Column II Column III
(Reaction) (Reagent) (Intermediate)
Cl

(I) (i) Cl 2 + hn (P) Carbene


Ph Ph
Ph H
Ph
(II) H Ph (ii) CHBrClI / ONa (Q) Free radical
Ph
Br Cl

Cl
(III) (iii) NBS + hn / HBr (R) Carbanion

Br

(IV) (iv) HCl/Peroxide (S) Carbocation

7. For the given reactions in column I, the only correct combination for carbene addition is
(a) (IV) (iii) (P) (b) (III) (iv) (P) (c) (II) (iii) (P) (d) (I) (ii) (S)
8. For allylic bromination, the correct combination is
(a) (IV) (ii) (R) (b) (IV) (iii) (Q) (c) (III) (iii) (Q) (d) (III) (ii) (R)
9. Which combination proceeds via carbocation intermediate
(a) (II) (iv) (S) (b) (IV) (i) (S) (c) (III) (i) (S) (d) (I) (iv) (S)
130 + Passage Cum Matching Based Questions A-19

PASSAGE-4
Column contains some reaction, Column II contains Reagent involved in teh corresponding reaction & Column III contain mechanism
of reaction.
Column I Column II Column III
(Reaction) (Reagent) (Stereochemistry)
(I) OH (i) (i) Hg (OAc)2 (P) Markovnikov’s addition

(ii) H2O
s
(iii) NaBH4 / OH
H
O
(II) (ii) H2 + [Rh Cl (Ph3P)3] (Q) Syn-addition

(III) (iii) Alc. KOH (R) Anti and Hofmann’s eliminations


D D DH HD

(IV) (iv) (i) B2H6 THF (S) Syn-and anti-Markovnikov’s addition


F
(ii) H2O2, NaOH

10. Hydroboration-oxidation reaction is


(a) (II) (i) (S) (b) (IV) (i) (P) (c) (I) (iv) (S) (d) (I) (iii) (P)
11. For, oxymercuration-demercuration reaction the only correct combination is
(a) (I) (ii) (S) (b) (I) (iv) (P) (c) (IV) (ii) (R) (d) (II) (i) (P)
12. For alkene, the incorrect combination is
(a) (II) (i) (P) (b) (I) (iv) (S) (c) (III) (ii) (Q) (d) (II) (iii) (Q)
PASSAGE - 5
A galvanic cell is obtained when two different electrodes are coupled together. But all galvanic cells cannot be used as commercial
cells or batteries.

Column I Column II Column III


(I) Dry cell (i) 38% H2O (P) Anode:

2H2(g) + 4 OH - (aq) ® 4H2O(l) + 4e–

Cathode :
O2(g) + 2H2O(l) + 4e– ® 4OH– (aq)
(II) Mercury cell (ii) MnO2 + C + Paste (Q) Anode:

4 (aq) ® PbSO4(s) + 2e
of NH4Cl + ZnCl2 Pb(s) + SO 2– –

Cathode :

4 (aq) + 4H (aq) + 2e ® PbSO4(s) + 2H2O(l)


PbO2(s) + SO 2– + –

(III) Lead storage battery (iii) Concentrated KOH (R) Anode :


EBD_7801
A-20 130 + Passage Cum Matching Based Questions
or NaOH Zn(s) ® Zn2+(aq) + 2e–

Cathode :

2MnO2(s) + 2NH +4 (aq) + 2e– ® Mn2O3(s) + 2NH3(aq) +

H2O(l)

(IV) Fuel cell (iv) Paste of KOH + ZnO (S) Anode :

Zn(Hg) + 2 OH - ® ZnO + H2O + 2e–

Cathode :
HgO + H2O + 2e– ® Hg(l) + 2OH–

13. For dry cell, the only correct combination is


(a) (I) (iii) (S) (b) (I) (ii) (R) (c) (I) (iv) (R) (d) (I) (i) (R)
14. For mercury cell, the only correct combination is
(a) (II) (iv) (S) (b) (II) (iii) (S) (c) (II) (i) (S) (d) (II) (ii) (S)
15. For fuel cell give in column I, the only correct combination is
(a) (IV) (i) (Q) (b) (IV) (ii) (R) (c) (IV) (iv) (P) (d) (IV) (iii) (P)\
PASSAGE - 6
Column I, II and III contains order of reaction, integral equation and nature of plot respectively.
Column I Column II Column III

k 3
A ¾¾® Product, order =
log b(a – x)/

1 æ 1 1ö
(I) 2 (i) K = ´ç - ÷ (P)
a(b – x)

t è (a - x) a ø

Time
k(a - b)
Slope =
2.303

k 1 2 æ 1 1 ö
(II) A ¾¾ ® Product, order = (ii) K= ´ç - 1 2 ÷ (Q)
2 t è (a - x)12
a ø
½
1/(a – x)

Time
1
Slope = k 2, intercept =

130 + Passage Cum Matching Based Questions A-21

k 2.303 b(a - x)
(III) A + B ¾¾® Product, order = 2 (ii) K= ´ log (R)
(a) (a) t(a - b) a(b - x)

½
(a – x)
Time
Slope = k 2, intercept = a 2

k 2
(IV) A + B ¾¾ ® Product (iv) K= ´ éa1 2 - (a - x)1 2 ùû (S)

1/(a – x)
(a) (b) t ë

Time
1
Slope = k, intercept =
a
16. For reaction given in column I, the only correct combination is
(a) (I) (iv) (S) (b) (I) (iv) (Q) (c) (I) (ii) (Q) (d) (I) (i) (P)
17. For reaction given in column II, the only correct combination is
(a) (II) (iv) (R) (b) (II) (iv) (Q) (c) (II) (i) (Q) (d) (II) (ii) (S)
18. For reaction given in column III, the only correct combination is
(a) (III) (ii) (P) (b) (III) (i) (Q) (c) (III) (i) (S) (d) (III) (iv) (R)
PASSAGE - 7
Column I contains some reactions and Column II & Column III contains Reagent used and Mechanism respectively.
Column I Column II Column III
(Reaction) (Reagent) (Mechanism)

NH2
(I) (i) Mg / ether / H3OÅ (P) Ring expansion

OH
(II) (ii) PCl5 (Q) Ring contraction

(III) Me (iii) HNO2/D (R) Rearrangement


OH
N
Ph

O O
(IV) (iv) HÅ / D (S) Radical anion
Me Me
19. Beckmann rearrangement is
(a) (III) (i) (R) (b) (IV )(i) (Q) (c) (III) (ii) (R) (d) (I) (ii) (R)
EBD_7801
A-22 130 + Passage Cum Matching Based Questions
20. Which combination gives cyclopentene ?
(a) (I) (iii) (Q) (b) (II) (iii) (R) (c) (I) (iv) (S) (d) (II) (iv) (P)
21. The suitable combination of ring contraction is-
(a) (I) (i) (S) (b) (I) (iii) (Q) (c) (II) (iv) (R) (d) (II) (i) (Q)

PASSAGE - 8
Column I contains some reaction and Column II & Column III contains Reagent used and Products formed respectively.
Column I Column II Column III
(Reaction) (Reagent) (Product)

O
s CHO
(I) (i) (i) BH3 /THF (ii) H 2 O2 /OH (P)
CHO

CHO CHO

(II) (ii) (i) NaBH4/EtOH (Q)

COOH CHO

OH
H
(III) O (iii) (i) Glycol + HCl (R)

(ii) SOCl2 (iii) DIBAL – H

Å
(iv) H3O

O
(IV) (iv) (i) LAH (ii) Conc.H2SO4 / D(S) HO
O

(iii) O3 / Zn or Me2S

22. Find the correct combination


(a) (I) (iii) (P) (b) (II) (iv) (Q) (c) (I) (iv) (P) (d) (IV) (i) (R)
23. Find the combination where acid group is converting into aldehyde group
(a) (I) (iv) (Q) (b) (I) (iii) (S) (c) (II) (iii) (P) (d) (II) (iii) (Q)
24. Find suitable combination which follows hydroboration-oxidation reaction
(a) (III) (i) (R) (b) (IV) (iv) (S) (c) (III) (i) (S) (d) (I) (iii) (S)
130 + Passage Cum Matching Based Questions A-23

PASSAGE - 9
Column I contains reactant and Column II & Column III contains Reagent and product respectively.
Column I Column II Column III
(Reactant) (Reagent) (Product)

OH
(I) (i) CH3NH2 (P)
Br

Br
Cl H3C
(II) (ii) Moist Ag2O (Q)
Cl

Cl
NH2
(III) (iii) Alc. KOH (R)
CH2

Br

H Cl
H3C D
(IV) (iv) (i) One eq. NaCN (S)
H H
(ii) LiAlH4 N
Me

25. Correct combination for ring expansion is


(a) (I) (iii) (S) (b) (IV) (i) (Q) (c) (III) (ii) (P) (d) (IV) (i) (Q)
26. Find appropriate combination for ring formation
(a) (II) (iii) (P) (b) (III) (iii) (P) (c) (I) (iv) (R) (d) (II) (i) (S)
27. Incorrect combination is
(a) (I) (iv) (R) (b) (II) (iv) (P) (c) (IV) (iii) (Q) (d) (III) (ii) (P)
PASSAGE - 10
Column I contains reactant and Column II & Column III contains reagent used for reaction and product respectively.
Column I Column II Column III
(Reactant) (Reagent) (Product)

O O

s
(I) (i) H Å / D(ii)O3 , Zn (iii) OH/ D (P)

OH
EBD_7801
A-24 130 + Passage Cum Matching Based Questions

O
OH
(II) (ii) (i) NH2OH (ii) H2SO4/D (Q)

O
(III) (iii) H2SO4/H2O (R) O

O H

(IV)
s
(iv) OH / D (S) O

O
28. Find appropriate combination in which product is formed through retro aldol followed by intermolecular aldol reaction
(a) (III) (ii) (Q) (b) (IV) (iii) (P) (c) (I) (iv) (Q) (d) (I) (ii) (S)
29. Product ‘R’ is formed by
(a) (III) (ii) (R) (b) (IV) (iv) (R) (c) (I) (ii) (R) (d) (II) (i) (R)
30. Incorrect statement is
(a) (III) (ii) (Q) (b) (IV) (iii) (P) (c) (I) (iv) (Q) (d) (II) (i) (R)

PASSAGE - 11
Column I contains some complex and Column II & III contains no. of unpaired electrons and magnetic moment respectively.
Column I Column II Column III
Complex No. of unpaired electrons m spin (B.M)
(I) [Ni (NH3)6]2+ (i) 1 (P) 3.87
(II) [Ir Cl6]3– (ii) 3 (Q) 1.73
(III) [Mn (CN)6]4– (iii) 0 (R) 2.82
(IV) [Cr (CN)6]3– (iv) 2 (S) 0
31. Find suitable combination which shows lowest dipole moment
(a) (III) (iii) (S) (b) (IV) (i) (Q) (c) (II) (iii) (S) (d) (I) (iii) (Q)
32. The correct combination is
(a) (I) (i) (Q) (b) (I) (iv) (R) (c) (IV) (iv) (R) (d) (III) (i) (P)
33. The highest dipole moment is shown by
(a) (IV) (iv) (R) (b) (I) (iii) (R) (c) (IV) (ii) (P) (d) (III) (ii) (P)

PASSAGE - 12
Column I contains different acids & Column II & III contains their molecular formula and No. of s & p bonds respectively
Column I Column II Column III
Compound Molecular Formula No. of s and p bonds
(I) Thiosulfuric acid (i) H2 S2 O6 (P) 6 s and 2p
(II) Persulfuric acid (ii) H2 S2 O8 (Q) 11 s and 4p
(III) Peroxydisulfuric acid (iii) H2 SO5 (R) 9 s and 4p
(IV) Dithionic acid (iv) H2 S2 O3 (S) 7 s and 2p
34. Find appropriate combination for caro’s acid
(a) (IV) (iii) (R) (b) (II) (iv) (P) (c) (II) (iii) (S) (d) (I) (iii) (Q)
130 + Passage Cum Matching Based Questions A-25

35. Find appropriate combination for marshall’s acid


(a) (III) (i) (R) (b) (IV) (iii) (S) (c) (I) (ii) (P) (d) (III) (ii) (Q)
36. The only correct combination among the following is
(a) (I) (iv) (R) (b) (II) (iii) (Q) (c) (IV) (i) (R) (d) (III) (i) (S)

PASSAGE - 13
Column I contains compound and Column II & III contains their geometry and shape respectively.
Column I Column II Column III
Compound Geometry Shape
(I) XeF4 (i) Trigonal bipyramidal (P) Tetrahedral
(II) XeO2F2 (ii) Octahedral (Q) Octahedral
(III) XeF6 (iii) Tetrahedral (R) See-saw
(IV) XeO4 (iv) Trigonal planar (S) Square planar
37. Find suitable combination for Xenone dioxydifluoride
(a) (II) (iv) (P) (b) (II) (iii) (S) (c) (II) (iii) (Q) (d) (II) (i) (R)
38. Find appropriate combination which has same shape and geometry
(a) (I) (iii) (P) (b) (IV) (ii) (Q) (c) (II) (iv) (S) (d) (IV) (iii) (P)
39. Incorrect combination is
(a) (I) (ii) (S) (b) (IV) (iii) (P) (c) (III) (iv) (S) (d) (II) (i) (R)

PASSAGE - 14
Column I contains compound and Column II & III contains their formula and uses respectively.
Column I Column II Column III
(I) Glauber’s salt (i) FeSO4.(NH4)2 SO4.6H2O (P) Efflorescent
(II) Washing soda (ii) NaHCO3 (Q) Iron plating
(III) Baking soda (iii) Na2CO3.10H2O (R) Deliquescent
(IV) Mohr’s salt (iv) Na2SO4.10H2O (S) Gives CO2 on heating
40. Which combination represents such materials which are used as a laxative in medical field
(a) (III) (ii) (Q) (b) (IV) (ii) (P) (c) (I) (iv) (R) (d) (I) (iii) (R)
41. Find the combination which loses water spontaneously
(a) (I) (iv) (S) (b) (II) (ii) (Q) (c) (IV) (iii) (R) (d) (II) (iii) (P)
42. Correct combination is
(a) (I) (iv) (S) (b) IV (i) (Q) (c) (III) (iii) (S) (d) (II) (ii) (Q)

PASSAGE - 15
For an ideal gas, four different paths A, D (B + C) and D + E, from an initial state P 1, V1, T1 to a final state P2, V2, T1 is shown in the
given figure.
P1, V1, T1 D
P1, V2, T3

E
A
P
P2, V2, T1

B
C
P3, V2, T2

V
EBD_7801
A-26 130 + Passage Cum Matching Based Questions
Column I Column II Column III
Path Path presentation q reversible

(I) A (i) Reversible adiabatic (P) Zero


expansion followed by
reversible heating at
constant volume
(II) B + C (ii) Reversible expansion at (Q) P1 (V2 – V1)
constant pressure
followed by a reversible
cooling at constant volume
V2
(III) D + E (iii) Reversible expansion (R) - nRT1 , ln
V1
at constant pressure
V2
(IV) D (iv) Reversible isothermal (S) - nR, ln
V1
expansion
43. Find combination, which represents the qrev, for path (a)
(a) (I) (iii) (S) (b) (I) (iv) (R) (c) (I) (i) (Q) (d) (I) (ii) (P)
44. qrev, for path (B + C) represented by
(a) (II) (ii) (R) (b) (II) (iv) (S) (c) (IV) (i) (Q) (d) (II) (i) (S)
45. What is qrev, for path (D + E) ?
(a) (IV) (iii) (S) (b) (III) (i) (P) (c) (III) (ii) (Q) (d) (III) (iv) (R)
Telegram @unacademyplusdiscounts

Join Us on Telegram for More Such Books

https://telegram.me/unacademyplusdiscounts

Join us from the above link or search ''unacademyplusdiscounts'' in Telegram


130 + Passage Cum Matching Based Questions A-27

HINTS & SOLUTIONS


· Now the 7 digit numbers using the digits 1, 2 and 3 only
MATHEMATICS
such that the sum of digits in a number is 10
Passage 1 This can be done by taking 2, 2, 2, 1, 1, 1, 1 or by taking 2,
1. (b) 3, 1, 1, 1, 1, 1.
2. (a) 7 7
\ Number of ways = + = 77
3. (d) 3 4 5
· The graph of | x – p | + | y – q | = k is a square. Area of the
region bounded by | x – p | + | y – q | = k is given by 2k2 (area Passage 3
is independent of p and q). 7. (b)
· The resultant figure for a | x – p | + b | y – q | = k is a rhombus. 8. (a)
Area of the region bounded by a | x – p | + b | y – q | = k is
9. (c)
2k 2 (I) If f (x) = x2 + 2(k + 1)x + (9k – 5) has both negative
given by (area is independent of p and q)
ab roots, then condition for this :
· The resultant figure for | x – p | – | y – q | = k is not a closed Þ (a) Discriminant > 0
loop so we cannot find the area bounded by the graph.
Þ 4 (k + 1)2 – 36k + 20 > 0
· The resultant figure which is bounded by | x + y | = p and | x
Þ k2 – 7k + 6 > 0
– y | = q is a rectangle and its area will be ( 2 p) ( 2q) i.e.,
Þ (k – 1) (k – 6) > 0
2pq.
Area of the regions bounded by | x – p | + | y – q | = k and k < 1 or k > 6
(b) a + b < 0 Þ – 2(k + 1) < 0
1 1
x- + y- = k is same. Þ k+1>0Þk>–1
p q
and (c) f (0) > 0 Þ 9k – 5 > 0
Passage 2
4. (b) 5
Þ k>
5. (a) 9
6. (c) Combining the results of (i), (ii) and (iii), we get k > 6
· Number of four digit odd-number which are formed by using (II) f (x) > 0 for all x Þ Discriminant < 0
0, 1, 2, 3, 5, 7 (with repetition)
Þ 4(4k – 1)2 – 4(15k2 – 2k – 7) < 0
= 5 × 6 × 6 × 4 = 36 × 20 = 720
Þ k2 – 6k + 8 < 0 Þ 2 < k < 4
· Number of numbers greater than 1000 but less than 4000
which are formed by using the digits 0, 1, 2, 3, 4, 5 (without (III) If ‘k’ lies between the roots of
repetition) f (x) = 2x2 – 2(2k + 1)x + k(k + 1), then
= 3 × 4 × 3 × 2 = 72 Þ Discriminant > 0 and f (k) < 0
· We know that a number is divisible by 3 only when the sum
Þ 4(2k + 1)2 – 8k(k + 1) > 0
of the digits is divisible by 3.
Now the possible number of combinations of 5 digits out of and 2k2 – 2k(2k + 1) + k(k + 1) < 0
6 different digits 0, 1, 2, 3, 4, 5 (without repetition) are 6C5 Þ – k2 – k < 0
= 6, which are as follows– Þ k Î R – [– 1, 0]
1 + 2 + 3 + 4 + 5 = 15 = 3 × 5 (divisible by 3)
(IV) Consider absolute value of the difference of roots of
0 + 2 + 3 + 4 + 5 = 14 (not divisible by 3)
0 + 1 + 3 + 4 + 5 = 13 (not divisible by 3) the equation x 2 + kx + 1 = 0 exceeds 3k i.e.,
0 + 1 + 2 + 4 + 5 = 12 = 3 × 4 (divisible by 3)
|a–b|> 3k
0 + 1 + 2 + 3 + 5 = 11 (not divisible by 3)
0 + 1 + 2 + 3 + 4 = 10 (not divisible by 3) Þ (a + b)2 – 4ab > 3k
Thus the number should certain the digits 1, 2, 3, 4, 5 or the Þ 2
k – 3k – 4 > 0 and k > 0
digits 0, 1, 2, 4, 5. Þ (k – 4) (k + 1) > 0 and k > 0
Taking 1, 2, 3, 4, 5, the 5 digit numbers are = 5 = 120 Þ k>4
Taking 0, 1, 2, 4, 5, the 5 digit numbers are = 5 - 4 Passage 4
10. (a)
= 96
(I) Any point on L1 is (2l + 1, – l, l – 3)
\ Total number of numbers = 120 + 96 = 216
EBD_7801
A-28 130 + Passage Cum Matching Based Questions
and that on L2 is (µ + 4, µ – 3, 2µ – 3) So, x = y3 + cy
For point of intersection of L1 and L2 dy
2l + 1 = µ + 4, – l = µ – 3, l – 3 = 2µ – 3 For, (1 + x ) - xy = 1 – x
dx
Þ l = 2, µ = 1
I.F. = e– x (1 + x)
\ Intersection point of L1 and L2 is (5, – 2, – 1) So, y (1 + x) = x + cex
Q ax + by + cz = d is perpendicular to P1 and P2
\ 7a + b + 2c = 0 and 3a + 5b – 6c = 0 dy y x + 1 - x2
For, + =
a b c a b c dx (1 - x 2 )3/2 (1 - x 2 )2
Þ = = Þ = =
-16 48 32 1 -3 -2 x

\ Equation of plane is x – 3y – 2z = d 1 - x2
I.F. = e
As it passes through (5, – 2, – 1)
x x / 1 - x2
\ 5 + 6 + 2 = d = 13 y= + ce -
1 - x2
\ a = 1, b = – 3, c = – 2, d = 13
(I) (i) (R) is the correct matching. Passage 6
16. (a)
11. (d) All options (a), (b) and (c) are incorrect combinations.
17. (d)
12. (c) (III) Any point on L1 is (l + 1, 0, 0) 18. (b)
and that on L2 is (0, µ + 1, 0)
For point of intersection of L1 and L2 ì 1
ï |x| | x |³1
l + 1 = 0, 0 = µ + 1, 0 = 0 í
Þ l = – 1, µ = – 1 f (x) =
ï C x2 + C | x | < 1
\ Intersection point of L1 and L2 is (0, 0, 0) î 1 2
Q ax + by + cz = d is perpendicular to x + 2y + 3z = 2 and where C1 + C2 = + n
2x + 3y + 4z = 4 when n = + 1
\ a + 2b + 3c = 0 and 2a + 3b + 4c = 0 then f (x) will be everywhere continuous and everywhere
differentiability.
a b c a b c
Þ = = Þ = = ìï x 2 + 3x + 3 x £ 1
8-9 6-4 3-4 -1 2 -1
For f (x) = í
ïî nx + 3 x >1
a b c
or = = when n = 5
1 -2 1
then there will be one point of discontinuity which will be x
\ Equation of plane is x – 2y + z = d
=1
As it passes through (0, 0, 0)
For f (x) = ||| x | – 2 | + n |
\ 0–0+0=d=0
when n = 1, 2
\ a = 1, b = – 2, c = 1, d = 0
then there will be three points of non-differentiability which
(III) (iii) (Q) is the correct matching.
will be x = 0, ± correct 2
Passage 5
13. (d) For f (x) = [2 + 3 | n | sin x] x Î N, n Î (0, p)
14. (c) when n = + 2
15. (a) then there will be eleven points of discontinuity when [2 +
6 sin x] = – 4, – 3, – 2, 0, 2, 3, 4, 5, 6, 7, 8
dy
(1 + x2) + 2 xy - 4 x 2 = 0 Passage 7
dx
1
2x 19. (b) Here - < x < 0 gives – 1 < 2x < 0
I.F. = ò 1 + x2
dx
= 1 + x2 2
e
4 x2 -1
(1 + x 2 )dx + c So that [2x] = – 1 in <x<0
So, y (1 + x2) = ò 1 + x2 2

4 3 -1
y (1 + x2) = x +c Thus f (x) = 4x2 – x, <x<0
3 2
dy 1
Similarly for, ( x + 2 y3 ) =y f (x) = ax2 – bx, 0 < x <
dx 2
1 -1
I.F. = The function is differentiable in < x < 0 and also in 0 <
y 2
130 + Passage Cum Matching Based Questions A-29

Also f ¢ (0+) = 0. f ¢ (0¯) = 0 Þ f ¢ (0) = 0


1
x< as it is a polynomial of degree 2 in each of the subinterval.
2 ì 2x x>0
ï
Since f (0¯) = f (0) = f (0+) = 0. f (x) is continuous at \ f ¢ (x) = í 0 x =0
x = 0 for all a, b ï - 2x x < 0
î
Now, f ¢ (0¯) = – 1 and f ¢ (0+) = – b.
It follows that f ¢ (0) exists, if b = 1, independent of a. ì 2, x>0
Þ f ¢¢ (x) = í
20. (a) Here f (0) = 0 î - 2, x < 0
Also f ¢¢ (0 ) = 2, f ¢¢ (0¯) = – 2 Þ f ¢¢ (0) does not exist.
+
1
So, f (x) will be continuous, if lim x p sin =0 Hence f (x) is twice differentiable in R – {0}
x ®0 x
If f (x) = x2 | x |
This is possible only when p > 0 ... (i)
ì - x3 , x < 0
f (0 + h) - f (0) ï
f ¢ (0) = lim x=0
h®0 h Then f (x) = í 0,
ï 3
îx , x>0
1
h p sin -0 1
= lim
h
= hlim h p - 1 sin ì - 3x 2 , x < 0
h®0 h ®0 h ï
Þ f ¢ (x) = í 0, x=0
f ¢ (0) will exist only when p > 1 ï 2
\ f (x) will not be differentiable if p < 1 ... (ii) î 3x , x>0
From (i) and (ii), for f (x) to be not differentiable but So, f ¢ (x) exists for all real x.
continuous at x = 0, possible values of p are given by ì - 6 x, x < 0
0<p<1 ï
f ¢¢ (x) = í 0, x=0
21. (d) f (x) = cos p (| x | + [x]) ï 6 x, x>0
î
ì cos p ( - x + ( - 1)), - 1 £ x < 0 However, f ¢¢ (0) does not exist since f ¢¢¢ (0¯) = – 6
= í and f ¢¢¢ (0+) = 6 which are not equal. Thus, the set of points
î cos p ( x + 0), 0 £ x <1
where f (x) is thrice differentiable in R – {0}.
ì - cos px - 1 £ x < 0 ì -1 2 æ1ö
= í ï (sin x ) cos ç ÷ , x ¹ 0
î cos p x 0 £ x < 1 If f (x) = í èxø
Obviously f (x) is discontinuous at x = 0 otherwise f (x) is ï0 x =0
î
continuous and differentiable in (– 1, 0) and (0, 1).
æ1ö
Passage 8 lim f ( x) = lim (sin - 1 x) 2 cos ç ÷
22. (d) x ®0 x ®0 èxø
23. (b) = 0 × (any value between – 1 to 1)
24. (d) =0
f (x) = 15 – | x – 10 | Hence, f (x) is continuous at x = 0
f ( f (x)) = 15 – | (15 – | x – 10| ) – 10 | æ1ö
f ( f (x)) = (sin -1 h ) 2 cos ç ÷ - 0
è hø
ì 15 - | 15 - x | x ³ 10 f ¢ (0+) = lim
í h®0 h
î 15 - | x - 5 | x < 10
æ sin - 1 h ö æ öæ æ 1 öö
ì x + 10, - ¥ < x < 5 = çç hlim ÷ ç lim sin - 1 h ÷ ç lim cos ç ÷ ÷
ï 20 - x, 5 £ x < 10 ® 0 h ÷ èh®0 øè h ® 0 è h øø
ï è ø
f ( f (x)) = í = 1 × (0) × (any value between – 1 to 1) = 0
ïx 10 £ x < 15
ïî 30 - x, 15 £ x < ¥ Similarly, f ¢ (0¯) = 0
Hence, f (x) is continuous and differentiable in [– 1, 1] and
Three points of non-differentiability. (– 1, 1) respectively.
If f (x) = x | x | Passage 9
25. (a)
ïì x ,
2
x³0 (I) We apply check for continuity at x = 0
then f (x) = í 2
ïî - x , x < 0 LHL = lim f ( x ) = lim f (0 - h )
x ® 0¯ h®0
Þ f ¢ (x) = 2x when x > 0
and f ¢ (x) = – 2x when x < 0 ¥
= lim (cos h + sin h) - cosec h (1 form)
h®0
EBD_7801
A-30 130 + Passage Cum Matching Based Questions

ì æ 2 h h h öü ìï 4 (- 2)4 - 1 üï ì 4 ( - 8) ü
ï lim ç - 2 sin 2 + 2 sin 2 cos 2 ÷ï = sin í 5 -1 ý
= sin í ý
ïh ® 0 è øï îï 5 (- 2) þï î 5 (16) þ
= exp í (- 1) ý
ï ´
h hï æ 2ö
ï 2 sin cos ï = sin ç - ÷ ... (2)
î 2 2þ è 5ø

LHL = lim f ( x)
ì æ h höü x ® - 2¯
ïï ç sin 2 - cos 2 ÷ ïï
= exp í lim ç ÷ ý = e– 1 = lim f (- 2 - h)
ï h ® 0 çç h ÷÷ ï h®0
cos
îï è 2 ø ïþ
ae1/(|1 - 2 - h + 2|) - 1
RHL = lim f ( x ) = lim f (0 + h) = lim
x ® 0+ h®0
h®0 2 - e1/(|- 2 - h + 2|)

1 2 3 ae1/ h - 1 a - e-1/ h
= lim = lim
eh + eh + eh h®0 2 - e1/ h h®0 2e-1/ h - 1
lim
= h®0 2 3
ae h + be h a-0
= =–a ... (3)
0 -1
3 æ 2 1 ö From equation (1), (2) and (3) we get
- -
e h ç e h + e h + 1÷
ç ÷ æ2ö æ2ö
è ø=1 – a = sin ç ÷ and b = - sin ç ÷
= lim è5ø è5ø
h®0 3 ì -1 ü b
ï ï
e h í ae h + b ý 27. (d)
ïî ïþ
éæ p ö ù
1 - sin 3 êç ÷ - h ú
é -1 ù ëè 2 ø û
êQ lim eh = 0ú (IV) f [(p/2)¯] = lim
\ For continuity at x = 0 ê h®0 ú h®0 éæ p ö ù
3 cos 2 êç ÷ - h ú
ë û ëè 2 ø û
1
e– 1 = a = b– 1 Þ a = , b=e
e 1 - cos3 h 1
= lim =
26. (c) At x = – 2 h®0 2 2
3 sin h
f (– 2) = b ... (1)
b [1 - sin {(p / 2) + h}]
RHL = lim f ( x ) = lim f ( - 2 + h) f [(p/2)+] = hlim
x ® -2 + h®0 ®0 [ p - 2{( p / 2) + h}]2

æ (- 2 + h)4 - 16 ö b (1 - cos h) b
= lim sin çç ÷ = lim 2 =
5 ÷ h®0 4h 8
h®0
è (- 2 + h) + 32 ø
1 b 1
\ a= = Þa = ,b=4
ïì (h - 2)4 - 24 ïü 2 8 2
= sin í lim 5 5ý
îïh ® 0 2 + (- 2 + h) þï Passage 10
28. (a)
ïì (h - 2)4 - (- 2)4 ïü
sin í lim
= 5 5ý æ 2z + 1 ö
îïh ® 0 (h - 2) - (- 2) þï (I) Im ç ÷=k
è iz + 1 ø
Suppose z = x + iy
ì (h - 2)4 - (- 2) 4 ü
ï ï then,
ï (h - 2) - (- 2) ï
= sin í lim ý é 2 ( x + iy ) + 1 ù
ï h ® 0 (h - 2)5 - (- 2)5 ï Im ê ú=k
ï
î (h - 2) - (- 2) ï
þ ë i ( x + iy ) + 1 û
130 + Passage Cum Matching Based Questions A-31

x2 + (y + 1)2 = k2 + x2 + (y – 1)2
é 2 x + 2iy + 1 ù
Im ê ú =k
ë (1 - y ) + ix û + 2k x 2 + ( y - 1) 2
é (2 x + 1) + 2iy [(1 - y ) - ix ] ù
Im ê ´ ú=k (y + 1)2 – (y – 1)2 = k2 + 2k x 2 + ( y - 1) 2
ë (1 - y ) + ix [(1 - y ) - ix ] û

é[(2 x + 1) (1 - y ) + 2 xy ] + i [(2 x + 1) (- x) ù 2 (2y) – k2 = 2k x 2 + ( y - 1) 2


ê + (1 - y ) 2 y ] úú
Im ê =k (4y – k2) = 2k x 2 + ( y - 1) 2
ê (1 - y ) 2 + x 2 ú
ê ú Again squaring on both sides, we get
ë û
– 2x2 – x + 2y – 2y2
= k [(1 – y)2
+ x2] 16y2 + k4 – 8yk2 = 4k2 [x2 + (y – 1)2]
2 2 2 2
– 2x – x + 2y – 2y = k [x + y – 2y + 1] 16y2 + k4 = 4k2 [x2 + (y + 1)2]
For k = – 2, we will get a linear equation in two variables x For k2 = 4
and y which will be x + 2y = 2. So the locus of the complex 16y2 + 16 = 16 [x2 + (y + 1)2]
variable will be a straight line for k = – 2. x2 + (y + 1)2 – y2 = 1
29. (d)
x2 + 2y = 0
(II) | z – i | + | z + i | = k
So, for – 2 < k < 2, we will get an equation of hyperbola
Suppose z = x + iy
| x + iy – i | + | x + iy + i | = k x2 y2 1
which will be - =-
| x + i (y – 1) | + | x + i(y + 1) | 16 - 4 k 2 4k 2 16

x 2 + ( y - 1) 2 + x 2 + ( y + 1) 2 = k Therefore, if – 2 < k < 2, the locus of z will be a hyperbola.


Passage 11
2 2
x + ( y - 1) = k - x + ( y + 1)2 2 31. (b)
32. (b)
After squaring on both sides, we get 33. (c)
x2 + (y – 1)2 = k2 + x2 + (y + 1)2 – 2k x 2 + ( y + 1) 2 1
Sol. (I) f (x) = , a > 0, a ¹ 1
log a x
k2 + 4y – 2k x 2 + ( y + 1) 2 = 0 Domain = R – [0, 1)
Co-domain = R+
k2 + 4y = 2k x 2 + ( y + 1) 2
1
After squaring on both sides, we get (II) f (x) =
[ x]
k4 + 16y2 + 8k2y = 4k2 [x2 + (y + 1)2] Domain = R – {0}
when k2 = 4 Co-domain = R – {0}
16 + 16y2 + 32y = 16 [x2 + y2 + 1 + 2y] 1
(III) f (x) =
16 [x2 – y2] = 0 {x}
x2 – y2 = 0 Domain = R+ – {1}
So, for k2 > 4, we will get an equation of ellipse, which will Co-domain = (1, ¥)
1
x2 y2 1 (IV) f (x) =
|x|
be 2
+ 2
=
4k - 16 4k 16 Domain = R – I
k2
Therefore, if > 4, the locus of z will be an ellipse. ì 1 ü
Co-domain = í , n Î I - {0} ý
30. (c) î n þ
(III) | z + i | – | z – i | = k Passage 12
34. (a)
Suppose z = x + iy
| x + iy + i | – | x + iy – i | = k æ x 2 x3 ö æ x4 x6 ö
sin - 1 ç x - + ... ÷ + cos - 1 ç x 2 - + ... ÷
ç 2 3 ÷ø ç 2 4 ÷ø
| x + i (y + 1) | – | x + i (y – 1)| = k è è
p
x 2 + ( y + 1)2 - x 2 + ( y - 1)2 = k =
2
x 2 + ( y + 1)2 = k + x 2 + ( y - 1)2 æ x 4 x6 ö
Þ cos- 1 ç x 2 - + ... ÷
ç 2 4 ÷ø
Squaring on both sides, we get è
EBD_7801
A-32 130 + Passage Cum Matching Based Questions
æ Passage 13
p x 2 x3 ö
= - sin - 1 ç x - + ... ÷ 37. (a)
2 ç 2 4 ÷ø P (AC) = 0.3, P (b) = 0.4 and P (A Ç BC) = 0.5
è
æ x4 x6 ö P [B Ç (A È BC )]
Þ cos - 1 ç x 2 - + ... ÷ P [B/ (A Ç BC)] =
ç 2 4 ÷ø P (A È BC )
è
æ x 2 x3 ö P ((B Ç A) È (B Ç BC ))
-1 =
= cos ç x - + ... ÷
ç 2 4 ÷ø P (A È BC )
è
x 4 x6 x2 x3 P (A Ç B)
Þ x2 - + ... = x - + ... =
2 4 2 4 P (A) + P (BC ) - P (A Ç BC )
On both sides we have G.P. of infinite terms
x2 x 2 x2 2x P (A) - P (A Ç BC )
\ = Þ = =
æ - x2 ö æ- xö 2+x 2 2 +x 1 - P (AC ) + 1 - P (B) - P (A Ç BC )
1-ç ÷ 1-ç ÷
ç 2 ÷ è 2 ø
è ø 1 - 0.3 - 0.5 0.2 1
Þ 2x + x = 2x + x Þ x (x – 1) = 0
3 2 3 = = =
1 - 0.3 + 1 - 0.4 - 0.5 0.8 4
Þ x = 0, 1 but 0 < | x | < 2 Þ x = 1
38. (d)
35. (c)
sin [cot– 1 (1 + x)] = cos (tan– 1 x) 1 4 7
P (a) = , P (B) = , P (C) =
é æ öù 5 5 100
1
Þ sin ê sin - 1 ç ÷ú
ê ç 1 + (1 + x ) 2 ÷ú
ë è øû æCö
P ç ÷ P (B)
é æ
1
öù æBö
Pç ÷= èBø
= cos ê cos ç ÷ú
-1
ê ç 1 + x2 ÷ ú èCø æCö æCö
P ç ÷ P (A) + P ç ÷ P (B)
ë è øû A
è ø èBø
1 1
Þ =
2
1 + (1 + x) 1 + x2 80 39
´
Þ 1 + 1 + 2x + x2 = 1 + x2 100 40
=
Þ 2x + 1 = 0 20 30 80 39
´ + ´
-1 100 40 100 40
Þ x=
2
36. (d) é æ C ö 30 æ C ö 39 ù
êQ P ç ÷ = , P ç ÷ = ú
æ n ö ë è A ø 40 è B ø 40 û
cot - 1 ç1 + å 2k ÷ = cot - 1 [1 + n (n + 1)]
ç ÷
è k =1 ø 156 26
= =
é ( n + 1) - n ù 186 31
= tan - 1 ê –1 –1
ú = tan (n + 1) – tan n
ë1 + ( n + 1) n û 1 1
23 39. (b) P (a) = ·P (A) = = P (C)
\ å [tan -1
(n + 1) - tan -1
n] 2 4
n =1 P (A Ç B Ç CC) + P (A Ç BC Ç C)
= tan– 1 24 – tan– 1 1 + P (AC Ç B Ç C) + P (A Ç B Ç C)
23
= tan - 1
25 1 1 3 1 3 1 1 1 1
= ´ ´ + ´ ´ + ´ ´
é 23 2 4 4 2 4 4 2 4 4
æ n öù
\ cot ê å cot - 1 ç1 + å 2k ÷ ú 1 1 1
+ ´ ´
ê n =1 ç k =1 ÷ ú 2 4 4
ë è øû
1
é - 1 23 ù 25 =
= cot ê tan = >1
ë 25 úû 23 4
130 + Passage Cum Matching Based Questions A-33

Passage 14
PHYSICS
1
40. (c) Since y = x +
x
Passage-1
Clearly the given relation is many one and therefore not an 1. (c) x in ex is a dimensionless quantity
injective relation with domain R ~ {0} and range R ~ (– 2, 2) So, [x] = [M0L0T0] = [p] = [Plane angle]
41. (b) Since, y = | x | + 2
2. (d) [Angular Frequency] = [Angular velocity]
Clearly, this relation is many-one and so not injective.
= [M°L°T–1]
Here, domain of this solution is (– ¥, ¥) and range is (2, ¥).
[Plane angle] = [p] = [M0L0T0]
Hence, this solution is not surjective from
R to R whereas it is surjective from R to [2, ¥). which are independent of mass and length so will re-
42. (d) Since y2 = 2x – 4 = 2 (x – 2) main unaffected.
Clearly there are infinitely many pairs of points having same 1
=
[X]
abscissa in (2, ¥) 3. (c)
So, the given relation is one-many relation.
[Angular frequency] [ Angular velocity]
Here, domain of y 2 = 2x – 4 is [2, ¥) and range is Þ [X] = [Angular velocity] / [Angular Frequency]
(– ¥, ¥).
é M°L°T -1 ù
Hence, this relation is surjective from R ® R and also in R ë û
® [0, ¥) = é = [M0L0T0]
M°L°T -1 ù
Passage 15 ë û
43. (b) Passage-2
44. (d) 4. (a) Kinetic energy of the projectile decreases during
45. (c) the upward journey, becomes zero at the
Since H : x2 – y2 = 1 maximum height & then kinetic energy goes on
S : Circle with centre N (x2, 0) increasing during the return journey.
Common tangent to H and S at P (x1, y1) is 5. (c) When spring is compressed or elangated by a length x
x1 from its natural length, the potential energy of the spring
xx1 – yy1 = 1 Þ m1 = 1
y1 increases by k x2 .
Also radius of circle S with centre N (x2, 0) through point of 2
contact (x1, y1) is perpendicular to tangent 6. (a)
Passage-3
x 0 - y1
\ m1m2 = – 1 Þ 1 ´ =–1
y1 x2 - x1 7. (a) F m1 m2 m 3
Þ x1 = x2 – x1 or x2 = 2x1 All the body of mass m1, m2 and m3 will move
M is the point of intersection of tangent at P and x axis. F
with the same acceleration = m + m + m
æ1 ö 1 2 3
\ M ç , 0÷
è x1 ø 12
= = 2, ms –2
Q Centroid of DPMN is (l, m) 1+ 2 + 3
1 8. (b) F m1 T1 m2 T2 m3
\ x1 + + x2 = 3l and y1 = 3m
x1 a
Using x2 = 2x1, F.B.D. of F m1 T1 F – T1 = m1a ...(i)
1æ 1ö y1 a
Þ ç 3x1 + ÷ = l and =m
T1 m T2
3è x1 ø 3 2
T1 – T2 = m2 a .... (ii)

dl 1 dm 1 T2 m
T2 = m3 a ....(iii)
\ =1 - , = 3
dx1 3x12 dy1 3
(i) + (ii) + (iii)
Also (x1, y1) lies on H, a = F/(m1 + m2 + m3)
\ x12 - y12 = 1 or y1 = x12 - 1 T2 = m3 F 3 + 12
m1 + m2 + m3 =
= 6N
1+ 2 + 3
1 dm x1 T1 = m2a +T2 = (m2 + m3) F/(m1 + m2 + m3)
\ m= x12 - 1 \ =
3 dx1 3 x 2 - 1
1 (2 + 3)12
= = 10N
1+ 2 + 3
EBD_7801
A-34 130 + Passage Cum Matching Based Questions
9. (d) For situation (I), I along tangent = Idiameter (c.m.) + MR2
(m 2 + m3 ) F m3 f 5
f1 = , f2 Þ MR 2 = Idiameter + MR2
m1 + m2 + m3 m1 + m2 + m3 4
5 2 MR 2
m1 F Þ Idiameter = MR – MR2 =
Situation (II), f1 = m + m + m , 4 4
1 2 3 15. (a) Moment of inertia of thin spherical shell along the
(m1 + m2 ) F 5 5
f2 = tangent is MR 2 and radius of gyration is R
m1 + m2 + m3 3 3
5
m1 F
Situdation (III), T1 = m + m + m , 3
( )
2
so, I = × 2 × 15 = 50kg m2,
1 2 3
5
(m1 + m2 ) F radius of gyration = × 15 = 5m
T2 = 3
m1 + m2 + m3 Passage-6
16. (b) T µ r 3/2
(m 2 + m3 )F
Situation (IV), T1 = m + m + m , T2 æ 2r ö
3/2
1 2 3 =ç ÷ =2 2
T1 è r ø
m3 F
T2 = Þ T2 = 2Ö2 T1
m1 + m2 + m3 17. (a) When a satellite revolves around a planet in its orbit, it
Passage-4 posses both potential energy (due to its position) and
10. (d) Moment of inertia of square plate and cube both of kinetic energy (due to orbital motion)
mass M and each side length L is given –G M m
U=
M L2 r æ M = mass of earth ö
by =
6 GMm ç m = mass of stellite÷
K .E = ç ÷
M L2 2r è r = radius of orbit ø
11. (d) For square plate, I = , radius of gyration So total energy
6
– G M m G M m –G M m
L = U + K .E. = + =
= r 2r 2r
6 1
M L2 L 18. (d) V(orbital) µ
For cube, I = , radius of gyration = r
6 6
2
Time period µ r 3/ 2
For thin rod, I = M L , radius of gyration Angular momentum µ r
12
L 1
= Kinetic energy µ
2 3 r
Passage-7
M (a 2 + b 2 )
Rectangular plate, I = , radius of gyration 19. (a) For an Isochoric process, volume remains constant and
12 hence, dW = PdV = 0, dU = dQ = nCvdT
a 2 +b2 20. (a) For an Isobaric process, pressure remains constant and
=
2 3 hence, dW = P(V2 – V1), dQ = nCp dT
12. (c) As for cube 21. (b) For an Isothermal process, temperature remians
I a ML2 and R a L constant. As internal energy depends only on
I1 = 8I and R1 = 2R V2

Passage-5 temperature dU = 0 Þ dQ = dW = ò Pdv


13. (b) As we know that moment of inertia of solid sphere V1

2 2 V2 V2
along its diameter is MR . Þ dQ = nRT log e V1 = 2.303 nRT log 10 V1
5
Using parallel axes theorem Itangent = Icm + MR2
2 7
Passage-8
2
= MR + MR2 = MR2 22. (c) For a diatomic gas, average energy of a molecule
5 5
7 5
So the radius of gyration = R is kT if the molecules translate and rotate but
5 2
14. (b) MOI along tangent in the plane is 7
do not vibrate and is kT if they vibrate also i.e.,
5 2
MR 2
4 7 9
f = 7, C v = R , Cp = R + C v = R
2 2
130 + Passage Cum Matching Based Questions A-35
23. (b) As given for polyatomic gas degree of freedom f= 6, Passage-11
6 31. (d) For the spherical shell
then Cv = R = 3R, Ein = 0 for x < R
2
and for solid non conducting sphere
Cp 4
C p = 4 R, g = = = 1.33 KQx
Cv 3 Ein = 3 0 £ x £ R
24. (c) For diatomic gas R
32. (a) For circular ring
Case I. (molecule translate & rotate only)
Qx
5 7 E =k
( )
3/2
Cv = R Cp = R g = 1.40 R 2 +x 2
2 2
Case II. (molecule translate, rotate & vibrate also) R é dE ù
For Emaxm, at x = ± use = 0ú
7 9
Cv = R Cp = R g = 1.29 2 êë dx û
2 2 Q
Passage-9 Emaxm = 6 3πε R 2
0
25. (a) For an open organ pipe wavelength and frequency for
33. (b) Edisc (at x = R) is
the first harmonic or fundamental tone
l = 2l = 2 ´ .3 = .6 m 2kQ æ R ö 2KQ æ 1 ö
ç1 - ÷= 1-
while u = =
v 330
= 550 Hz 2 ç ÷ R 2 çè ÷

R è R + R2
2
ø
2l 2 ´ .3
26. (b) For first overtone or third harmonic of a closed Espherical shell (at x = R) i.e., at the surface
3v 4l
organ pipe u = and l = kQ
4l 3 =
27. (c) For first harmonic of open organ pipe R2
v Esolid non conducting sphere (at x = R) i.e., at the surface =
u = , l = 2l .
2l kQ
Third harmonic of open organ pipe
R2
3v 2 Passage-12
u= , l= l
2l 3
34. (b) U(q) – U(q0) = MB (cosθ0 - cosθ)
First harmonic of closed organ pipe
v given that U(q0) = 0 for q0=90°
u = , l = 4l Þ Uq = –MB cosq
4l
Third harmonic of closed organ pipe So for maximum potential energy
3v 4l q = 180° Þ Umax = MB
u= , l= 35. (b) For minimum potential energy, q = 0°
4l 3
Umin = –MB
Passage-10
36. (a) work done by an external force in changing the angle
28. (d) Apparent change in frequency
from q to q + dq is dW = (MB sinq) dq
(from doppler’s effect) is given by
θ
æ v ± v0 ö
υ = υ0 ç
è v ± vs ø
÷ Wext = ò MBsinθ dθ
θ0
where symbols have their usual meanings
29. (b) The apparent frequency before crossing Wext = MB (cosq0 – cosq)
æ v + v0 ö Wext is stored as potential energy of the field magnet
υ¢ = υ0 ç ÷ system for q = 60º, U = MB/2
è v - vs ø Passage-13
Here v0 = 72 km/h = 20 m/s
vs = 72 km/h = 20 m/s
37. (b) R C
æ 340 + 20 ö 36
v ¢ = 1088 ç ÷ = 1088 ´ = 1224 Hz
è 340 – 20 ø 32
30. (c) The apparent frequency after crossing
æ v – v0 ö æ 340 – 20 ö ~
v ¢ = v0 ç = 1088 ´ ç
è v + v5 ø÷ è 340 + 20 ÷ø E = E0 sin w t
32 For a C-R series circuit
= 1088 ´ = 967.11 Hz
36
1
Z = R2 + 2 2
w C
EBD_7801
A-36 130 + Passage Cum Matching Based Questions
44. (b) Object is placed between the optical centre and focus
E E æ 1 ö
So, I = = , çè here, X C = ÷ D
Z 1 wC ø
R2 + to see the magnified image Magnification M = 1 + ,
f
w 2 C2
when image formed at D.
Voltage across capacitor = I × XC
45. (c) For Galilean telescope
E 1 f 10 f e
= ´ L = f0 – fe and M = 0 = = 10
1 w C
R2 + 2 2 fe fe
w C 10fe –fe = 9fe
E
E VC =
= CHEMISTRY
or (XY w ) 2 + 1
R 2w 2 C 2 + 1
R L
38. (b) For L-R series circuit, Passage - 1
Z = R 2 + w 2 L2 9.8 ´ 10 ´ 1.8
1. (c) Molarity = = 1.8 M
~ 98
wL
Phase difference f = tan -1 æ Mol.wt. 1 ö
R Q d = Mç + ÷
and voltage is leading. è 1000 mø
39. (c)
Passage-14 æ 98 1ö
1.8 = 1.8 ç + ÷
è 1000 m ø
\ m = 1.10
40. (a) focal length of each part = f H2SO4 is a dibasic acid i.e. it has two replaceable
H+. Hence n-factor for this is 2.
2. (b) 1.8 NA molecules = 1.8 mol of HCl in 500 mL = 1.8
Equiv., n-factor for HCl = 1 (only one replaceable H+)
focal length of each part = f 1
9.8 × 10 × 1.2
3. (a) Molarity = = 1.2 M
98
1 1 1 1 2
= 1 + 1 Þ = 1 Þ f 1 = 2f n-factor for H3PO4 = 3
f f f f f Passage - 2
4. (c) Salts of strong acid and strong bases like Na2SO4 do not
undergo hydrolysis. Hence the pH of the solution remains
same.
41. (a) or 5. (a) For salts of weak base and strong acids like NH4Cl
Kw 1
1 1 1 1 Kh = and pH = 7 - (pK b + log C)
= + = Kb 2
f 2f 2f f
6. (d) For the salts of weak acid & weak base like CH3COONH4
Þ F = f = 10
h2 1
Kh = & pH = 7 + (pK a - pK b )
42. (d) For, , (1- h) 2 2
Passage - 3
7. (c) Reaction II proceeds via carbene intermediate in pres-
1 1 1 2 f 2 ence of CHBrClI / ONa
= + = ÞF= , P=
F f f f 2 f s r Br
ONa +
H C I
also for, , Cl
s
1 1 1 OH +Br — C I
= + = 0 Þ F = ¥, P = 0
F + f -f Cl
Passage-15 C-I break
s
43. (d) For an Astronomical telescope, length of the telescope s : C BrCl (Bromochlorocarbene) + I
Ph H
f Ph
= f0 + fe and its magnification = – 0 Ph
fe Ph + :CBrCl H
Br Cl
130 + Passage Cum Matching Based Questions A-37

8. (b) Br O
O H
NBS + h H OBH2
H BH2s Nar
Mechanism: H3C
O O H3C

hn s r
N — Br + HBr Br – Br + N—H BH OHNa
s
H O H O H — OH
O O
The resulting Br2 can be fragmented homolytically:- H3C H3C
hv
Br – Br ¾¾® Br — Br 2Br initiation Anti-Markovnikov's addition
Br H
H OH + HOBH2

11. (d) This is an acid catalysed hydration reaction producing


an alcohol with Markovnikov’s orientation.
+ HBr propagation (1) Oxymercuration addition of Oxygen and mercury.
Oxymercuration is followed by a reductive
Br — Br Br demercuration.
Demercuration in which the mercury acetoxy group is
substituted by hydrogen.
+ Br propagation (2) r s
ˆˆ† Hg (OAc) + OAc
Hg(OAc) 2 ‡ˆˆ
The process repeats it self until the NBS is consumed.
9. (a) HCl with peroxide does not undergo anti-Markovnikov’s d+ s
addition unlike HBr/peroxide, so reaction does not HgOAc O
proceed by free-radical mechanism but by carbocation. r
+ Hg (OAc) + H H
Passage - 4
s
10. (c) The addition of borane to an alkene, followed by O
H H OH
reaction with H 2 O2 /OHs , is called hydroboration r H H
oxidation. O
OH HgOAc
(i) B2H6 , THF
HgOAc Oxymercuration
(ii) H2O2 , NaOH
Mechanism: H
(i) r s O
OH
d

d+ NaBH4
H
H Br H BH2
BH 3 H HgOAc

d +
d
Demercuration
H3C H H3C
H2 + [RhCl (Ph 3P)3]
Syn addition 12. (c)
syn addition
D D DH HD
(ii) HO — O — H + OH HO — O Na + H2O H
better nucleophile O
Na
(d)
O—O—H
This is an example of oxymercuration-demercuration reaction
H BH2 s H BHs2 Nar occurred in presence of Hg(OAc)2/H2O/NaBH4. The option
+O—O—H (d) is not matching, thus answer is (d).
H3C Nar H3C
EBD_7801
A-38 130 + Passage Cum Matching Based Questions
Passage-5
13. (b) The dry cell consists of an outer container made up of 3° carbocation
zinc, which acts as anode. The cathode is a carbon rod
having a brass cap. The space between the cathode and
anode is filled with a mixture of MnO2 and thick paste –H –H
of NH4Cl, ZnCl2 and charcoal. The reactions during the
discharge are
At anode : Zn(s) ¾¾ ® Zn2+ (aq) + 2e–
At cathode : 2MnO2(s) + 2NH +4 (aq) + 2e– ¾¾ ®
Mn2O3(s) + 2NH3 + H2O
Rearrangement
14. (a) A mercury cell consists of an amalgamated zinc plate
anode which is covered by a steel top plate. A paste of Passage - 8
Hg, HgO and carbon powder acts as cathode. The O OH
electrolyte is a paste of KOH and saturated with
Zn(OH)2. The reactions during discharging are 22. (c) LAH H
At anode : Zn + 2OH– ¾¾ ® Zn(OH)2(Hg) + 2e–
At cathode : HgO + H2O + 2e– ¾¾ ® Hg + 2OH–
15. (d) Fuel cells consists of porous carbon electrodes contain-
ing suitable catalysts (Pt or Pd). Concentrated KOH or CHO
NaOH solution is placed between the electrodes to act O3/Red.
as the electrolyte. Hydrogen and oxygen gases are CHO
bubbled through porous electrodes into the KOH/NaOH
solution. Following electrode reactions take place CHO
At anode : 2H2(g) + 4OH - (aq) ¾ ¾ ® 4 H2O(l) + 4e–
-
At cathode : O2(g) + 2H2O(l) + 4e– ¾¾
® 4OH (aq) 23. (d) Protection of CHO group. O O
Passage-6 CH2OH
+ HCl
16. (c) 17. (a) 18. (c)
CH2OH
Passage - 7 COOH
19. (c) Me
OH
PCl5
N
or H COOH
Ph Antimigration
O O
O O
H 3O
Me — C — NH — Ph Me — C — OH SOCl2
DIBAL-H
+
PhNH2

OH OH2
20. (d) COCl
H

Ring expansion O O
CHO
1° carbocation –H 3° carbocation
(less stable) (more stable)
H3 O
NH2 N N
21. (b)
HNO2
CHO CHO
24. (c)

–N2 Ring contraction (i) BH3/THF HO


O O
(ii) H2O2/OH
2° carbocation Preferentially oxidises (C = C) bond.
130 + Passage Cum Matching Based Questions A-39

Passage - 9 Passage - 10
Cl O
25. (c)

OH
Moist Retro aldol
Ag2O 28. (c) O
H
(I)
H
O
2 4
Ring 1 5
Expansion C H
2° Carbocation (less stble) O
H—O—H
OH O
O

H2O
OH /
3° Carbocation Intramolecular
(more stable) C H aldol
O
Cl 29. (d) It undergoes intramolecular aldol condensation
26. (d) CH3 NH2
Cl
OH OH2
H/
NH2 –H
Cl
CH3

–H O3/Zn
NH
Cl
CH3 +
N
HC3 H O CHO s
O
OH
OH
–H

N
O
CH3

Cl –H2O
27. (b) one eq. NaCN
Cl
O N
CN 30. (a) NH2OH
LiAlH4 OH
Cl
(III)
/ H2SO4/
Cl NH2 H SO 4
H2
N O
OH O

One eq. NaCN


(CH2)4Cl2 (Beckmann's
LiAlH4
rearrangement) (Q)
(Not formed)
EBD_7801
A-40 130 + Passage Cum Matching Based Questions
O
O O O F
(b)
H 2SO 4 / H 2 O F — Xe — F Xe
Aldol condensation
O O F
O 3
sp d
Thus option (a) is incorrect.
Passage - 11 Trigonal See-saw shape due to presence of
31. (c) [IrCl6]3–. In 3rd transition element series, D0 is so large bipyramidal one lone pair of electron
that it becomes greater than P (pairing energy) 38. (d) XeO4
( P< D0).Cls acts as strong field ligand and thus pairing
of electrons occur. O
32. (b) [Ni (NH3)6]2+
Ni [Z = 28) = 3d8 4s2 O Xe O
2
Ni2+ = 3d8 ( t 62g eg ) O
2 sp3, Geometry, shape} Tetrahedral
eg
39. (a) XeF4,
8 6 .. ..
3d t 2g F .. F F F F F
..
(VBT), n = 2 CFT, n = 2 Xe or Xe Xe
.. ..
F F F F .. F
unpaired electron = 2 .. Square planar due
Geometry – Octahedral
n(n + 2) B.M =
m= 2(2 + 2) = 8 = 2.82 B.M to presence of two
lone pair of electron.
33. (c) Cr (Z = 24) = 3d5 4s1
(c) XeF6 – sp3d3
3 0
Cr+3 = 3d3 = t 2g eg Geometry – pentagonal bipyramidal
shape – monocapped octahedral
0
eg Thus, option (c) is incorrect.
Passage - 14
3 3
3d t 2g 40. (c) Glauber’s salt - Na2SO4.10H2O - Deliquescent. The
n = 3 CFT, (n = 3) Glauber’s salt was historically used as laxative. It is
effective for the removal of certain drugs such as
(VBT)
paracetamol from the body.
3 (3 + 2) = 15 = 3.87 B.M
m= 41. (d) Washing soda - Na 2 CO 3 .10H 2O – . Efflorescent.
Passage - 12 Efflorescent is the property of material which means
34. (c) Persulfuric acid (H2SO5) is also known as caro’s acid spontaneous loss of water by a hydrated salt.
42. (b) Mohr’s salt (FeSO4.(NH4)2SO4.6H2O is commonly used
O in iron plating
p s
Passage - 15
H s O s O s S s O s H (7s, 2p) 43. (b) Path A - reversible isothermol expansion.
p s According to first low of thermodynamics
O
35. (d) Peroxydisulfuric acid (H2S 2O8) is also knwon as DU = q - w
Marshall’s acid q=w
O O
Isothermal process DU = 0
p s p s
æ V2 ö
HsOsSs OsO s Ss O s H (11s, 4p) = ç nRT1 , ln V ÷
p s p s è 1 ø
O O 44. (d) Path B + C - reversible adiabatic expansion followed
36. (c) Dithionic acid (H2S2O6) by reversible heating at constant volume q = 0
O O q rev
p s p s DS = , q rev = T D S
T
H sOs S s S sOs H (9s, 4p)
p s p s V2
O O \ q rev = nRln
V1
Passage - 13
37. (d) Xenone dioxydifluoride (XeO2F2) 45. (c) D + E - Reversible expansion at constant pressure
followed by a reversible cooling at constant volume.
w = q = P1 (V2 – V1)
Telegram @unacademyplusdiscounts

Join Us on Telegram for More Such Books

https://telegram.me/unacademyplusdiscounts

Join us from the above link or search ''unacademyplusdiscounts'' in Telegram

You might also like